Accsap 9

You might also like

Download as pdf or txt
Download as pdf or txt
You are on page 1of 936

Ar

ryt
hmi
as

1.A67- year-oldwomanwhowasadmi tt
edf orpneumoni awasincidental
lyfoundt o
have par oxysmalat rialfi
bri
ll
ati
on.She has a hi st
ory oft ype 2 di abetes,
hypertension,andhumani mmunodef i
ciencyvirus.Shetakessubcutaneousi nsul
in,
carvedil
ol,darunavi
r,andri
tonavi
r,andiscompl iantwit
hhermedi cati
ons.Whi chof
thefoll
owingwoul dbet hepref
err
edanticoagulant?

a. Dabi
gat r
an.
b. Ri
varoxaban.
c. Warf
ar i
n.
d. Aspi
rin.
e. Cl
opidogrel
.

Answer

C.War
far
in

This pat i
enthas a CHA2DS2- VASc score ofatl east4 and meet s cri
teri
af or
anticoagulation.Nei t
heraspi ri
n norcl opi
dogrelal one arer ecommended f or
prevention ofst roke i
nt he set t
ing ofa high CHA2DS2- VASc scor e without
cont rai
ndications to ant
icoagulati
on.Because ofherpr otease inhi
bit
or -
based
antiretr
oviralt her
apy,sheisnotagoodcandi dat
ef ordabigatranorr i
varoxaban
becauseherhumani mmunodef i
ciencyvi
rusmedicat i
onsarest ronginhibit
orsof
cytochromeP4503A4andP- glycoprot
eindrugeff
lux.Thisinhibit
ionresultsinan
i
ncr ease int hese drugs'effects.Theref
ore,war f
ar i
nist he bestchoi ce for
anticoagulation.

2.A 39- year


-ol
d woman 2 weeks post partum i s admi
tted t
ot he hospi
talwit
h
shortnessofbreat
handpalpi
tat
ions.Shet akesnomedi cati
ons.Herexaminat
ioni
s
notableforaheartrat
eof110bpm,bl oodpr essureof92/60mm Hg,andoxygen
saturati
onof98%.Therear
ebibasil
arralesandanS3  gal
lopand2+pitti
ngedemaof
herlowerextr
emiti
es.Herelectr
ocar
diogram i
sshowni nFi
gur
e1.Whi
choft
he
fol
lowi
ngint
ravenousmedicat
ionsshouldbeadmini
ster
ed?

a. Verapami l.
b. Ivabradine.
c. Adenosi ne.
d. Lidocaine.
e. Magnesi um.

Answer

D.Li
docai
ne

The electr
ocar diogram demonst rat
es vent ri
culart achycardia( VT)wi t
h a wi de
regularQRSr hyt hm,ext r
emer ightaxisdevi at
ion,QRSdur ati
onofanat ypicalri
ght
bundlemor phol ogywi t
ht heRwave>t her 'wave.TheQRSdur ati
oni s>140msec,
alsofavori
ngVTi nar i
ghtbundl emor phologyt achycardia.Ther eisalsoanSwave
>Rwavei nleadV6  f
urtherfavori
ngVT.Oft hechoi cesgiven,lidocaineist hemost
appropriat
echoi cef orthetreatmentofVT.Adenosi neisusef ulforthetreatmentof
supraventr
iculart achycardi
a( SVT) .Verapami lisusef ulfort het r
eatmentofSVT
andsomeVTs, wi t
hamor etypicalri
ghtbundl ebr anchblockmor phology,al ef
taxis
deviati
oninapat i
entwi t
hanor malhear t
.Ivabr adineisusefulf orthetreatmentof
i
nappr opri
atesinust achycardiaandt otreatsinust achycardiai npatient
swi thheart
f
ail
ure.Magnesi
um canbeusef
ulfort
hetr
eatmentoft
orsadesdepoi
ntes,notas
f
ir
st-l
inet
reat
mentf
orpati
ent
swithmonomorphi
cVT.

3.A19- year -oldgi rlwithknownvent ri


cularpre-exci
tati
ononherel ectr
ocardiogram,
butnopr iorhi storyofpal pit
ati
onsorsyncope,i sseeni nt heemer gencyroom i n
acuter espiratorydi stressduet obronchospasm.Shei streat edwi t
hanal buterol
nebulizer,intravenous ( I
V)solumedr ol
,and I V diphenhydr amine.I nt
ubat i
on i s
avoided, andshei smar kedl
yimprovedandr eadyfordischar gewhenshesuddenl y
experiencespal pitati
onsandchestdi scomfort.Herheartr atei s170bpm,bl ood
pressure90/ 64,l ungshaveaf ew scatter
edwheezesandcar diacexami nati
oni s
benignexceptf ort achycardi
a.Anel ect
rocardi
ogram isobt ainedandi sshowni n
Figure1.Car otidsi nusmassageandVal salvamaneuver sarei nef
fecti
ve.
I
Vadmi ni
str
ati
onofwhi
choft
hef
oll
owi
ngi
sthenextmostappr
opr
iat
est
epi
nher
management?

a. Metoprolol.
b. Verapami l
.
c. Pr
ocainami de.
d. Adenosine.
e. Cal
cium gluconat
e.

Answer

B.Ver
apami
l

The pat i
ent has supr avent r
icular t achycardia, most l ikely or t
hodromi c
atri
ovent ri
cul
ar(AV)r eci
procatingt achycar dia.Al
thoughadenosi neisusuallyfi
rst-
l
inetr eatment,t
hispat i
entjustrecover edf rom anacut easthmat icexacerbati
onand
shouldnotr eceiveadenosi neormet opr olol.Therefore,IV ver apamilshouldbe
admi nister
ed.Pr ocainamide is usef ulf ort he treatmentofpr e-
excited at
rial
fi
bril
lationandi snotf i
rst
- l
inetreat mentf orAV r e-ent r
antt achycardia.Calci
um
gluconat eisnotusefultotreatAVr e- entrantt achycardia.

4.A19- year-ol
dst udentlacr
ossepl ayercomplainsoff ourepisodesofpalpit
ati
ons
occurri
ngi nt
hepast3mont hs.Theepisodesoccurabr uptl
yandar eaccompani ed
withmildchestpai n.Theepisodeshavel ast
ed10- 15mi nut
esi ndur
ati
on.Shehad
l
earnedt odoaVal salvamaneuvert obreaktheepisodes.Shetakesnomedi cati
ons,
and has no f amily hi
stor
y ofcar diac di
sease orsudden deat h.Herphysi cal
examinationi
snor mal ,
andherelectrocardi
ogram i
sshowni nFigure1.
Whi
choft
hef
oll
owi
ngi
sthemostappr
opr
iat
est
epi
nhermanagement
?

a.Rest r
ictionfrom compet i
ti
vesports.
b.Electrophysiol ogystudyandcat heterablat
ion.
c.Implant ablecar diovert
er-def
ibr
il
lator(I
CD)implant
ati
on.
d.Ami odar onet herapy.
e.Met oprol olt
wi cedaily.

Answer

B.Electr
ophysiologystudyandcat heterablation.
Thepat ienthassympt omaticWol f
f-Par ki
nson-Whi t
e( WPW)syndr ome.Cat heter
ablati
oni saCl assIi ndicati
onduet osympt omsandi ncreasedr i
skofsudden
cardiacdeath.Restri
cti
onf r
om compet i
ti
vespor tsi
snotr ecommendedf orpatients
withWPW,ast heepisodescanoccuratanyt ime,andsuddendeat hcanoccur
regardl
essofexer ci
sebehavi or.I
CDi mpl antati
onisaCl assI I
Irecommendationt o
preventsuddendeat hi napat i
entwhohasanar rhythmiacur abl
ewi t
hablat i
on.
Met opr
ololandami odar onearenotr ecommendedf orpatientswithWPW.I tmay
decreasethefrequencyofsupr aventr
iculartachycardi
a,butitdoesnotdecreaset he
r
iskofsuddencar
diacdeat
h.

5.A 23- year-ol


d graduate studentcomes t ot he emergency depar
tmentwi t
h
l
ightheadedness,palpit
ati
ons,andshor t
nessofbreath.Shehasnopr i
ormedi cal
hist
oryand t akes no medicat i
ons.On physicalexaminati
on,herhear trat
ei s
i
rregular,appr
oaching220bpm.Herbl oodpressureis82/56mm Hg.Shei salert
butanxious.Hercardiacexami nationi
ssigni
fi
cantforanirr
egul
artachycar
dicrate,
butotherwiseisnormal.Herelectrocar
diogr
am isshowninFigure1.

Basedonherel
ect
rocar
diogr
am,
whi
choft
hef
oll
owi
ngi
str
ue?

a.Sheisati ncreasedriskforsuddendeat h.
b.Metoprololwi l
lter
mi nat
ethet achycar
dia.
c.Impl
antat i
on of an i mplantable cardi
over
ter
-def
ibr
il
lat
or (
ICD) i
s
appropri
at e.
d.Hyperkalemiai spresent.
e.Adenosinewi llter
minatethetachycardia.
Answer

A.Shei
satr
iskedf
orsuddendeat
h.

Thepat i
enthaspr e- exci
tedatri
alfi
bril
lation(AF)( AFconduct ingt othevent ricles
downanaccessor ypat hway).Theel ectrocardiogram demonst ratesani r
regul ar
widecompl extachycar di
aataver yfastr ate.Thef astratesoft het achycar dia,at
ti
meswi thR- Ri ntervalsof<250msec,ar ear iskf actorforsuddencar diacdeat h.
Becauset hemor phol ogyoftheQRScompl exesi ssi mil
ar,shel ikelyhasasi ngle
accessor ypat hway.Herpot assium l
eveli sli
kelyt obenor mal.Theacut et reatment ,
becauseshei shemodynami callyst
abl e,isint
r avenouspr ocai
nami de.I funst abl e,
electr
icalcar dioversioni sappropri
ate.I CD impl antati
oni saCl assIII-indicat ed
procedur ei n a pat i
entwho hasan ar rhythmi a amenabl eto cat heterabl ation.
Met oprolol,adenosine,andot heratr
iovent ri
cul
arnodalbl ocki
ngagent smayr esul t
i
nsel ectiveuseoft heaccessorypathwayandi ncreaseconduct iont othevent ricles
throught hepat hway.

6.Unli
kemostot heror
alant
iar
rhyt
hmicdrugs,amiodaroneisoftenini
ti
atedwi t
ha
l
oad( ahigherdoseforsever
aldaysorweeks)beforedecreasingtomaintenance
dosi
ng.Whichoft hef
oll
owi
ngpharmacol
ogicproper
tiesbestexplai
nst
her easonto
l
oadami odarone?

a.Multi
pleantiarrhyt
hmi cclassef
fect
.
b.Longhal f
-li
fe.
c.Largevolumeofdi st
ri
but i
on.
d.Fi
rst-passeffectintheliver.
e.Highiodineconcent r
ation.

Answer

C.Lar
gevol
umeofdi
str
ibut
ion

Drugssuchasami odarone,whi
chhaveal ar
gevolumeofdist
ri
but
ion,areoft
en
l
oadedwi thagr eaterdosageadmini
ster
edearl
yon.Alt
houghamiodaronedoes
haveal onghalf
-li
fe,incr
easi
ngthedoseofthedrugear
lyondoesnotal l
ow t
he
achievementofasteadystat
eanyfast
er.
 

7.A72- year- ol
dwomanpr esentswit
hcompl aintsofweaknessandf at
igue.Shehasa
pasthi storysignif
icantforatri
alfibri
ll
ati
onmanagedwi t
hr atecontrolstr
ategy,
peri
pher alvasculardisease,andhyper t
ension.Sher ecent
lywasdi schargedfrom
thehospi talf
oll
owingar ecurr
entepisodeofatrialf
ibr
il
lat
ionrequi
ri
ngcar di
oversi
on.
Shei
sunclearwhatherhomemedi
cat
ionsar
e,butknowsshewast
aki
ngsever
al
newmedi
cationsf
orherhear
t.
 

Uponphysi calexaminati
on,sheisconfusedandlethar
gic.Herbloodpressurei
s
91/54mm Hgandpul seis103bpm.Physi calexaminati
onisremarkablefora
tachycardi
cr hyt
hm and no mur murs.She compl ai
ns ofblurred vi
sion.An
elect
rocardi
ogram (
ECG)obt ai
nedint
heemer gencydepartmenti
sshowni nFigur
e
1.

Therecentaddi
tionofwhichofthefoll
owingmedi cati
onstothi
spat
ient
'smedi
cat
ion
regi
meismostlikel
yresponsi
bleforhercl
ini
calf
indingsandECGchanges?

a.Dronedarone.
b.Ami l
odipi
ne.
c.Nadolol.
d.Apixaban.
e.Labetal
ol.
Answer

A.Dr
onedar
one

Dronedarone i s a pot ent i nhi


bit
or of P- glycoprotei
n. When admi ni
ster
ed
concomitantlywithdigoxin,itmayleadt osubstantiali
ncreasesindi goxinsteady-
statelevels.Co- administrati
on ofdigoxin and dr onedarone r equires a dose
reduct
ionindi goxin(asisal sothecasewi thami odaroneco- admi nistrati
on).The
ECG demonst rat
es bidi
rect i
onalventr
icul
art achycardia,a classi calf indi
ng in
digoxi
nt oxici
ty.Thepat ientisdigoxintoxi
candshoul dbet reatedwi t
hdi goxi
n
i
mmuneFab( ovine).Theot heragentsar enoti nhibit
orsofP- glycopr otei
nand
wouldnotbeexpect edtoresultint
heclini
calconditi
ondescr i
bed. 

8.A 67- year


-ol
d man pr esent
st o youroffi
ce wit
h complai
nts ofdizzi
ness and
l
ightheadedness.About3 mont hs ago,he underwenti
mplantat
ion ofa dual-
chamberpacemakerf orsympt omaticcomplet
ehear tbl
ock.Hisl ef
tventr
icul
ar
eject
ion fr
acti
on atthe time was normal.His otherhi
stor
yi sremarkablefor
rheumat oi
dart
hri
ti
sandhyper tensi
on. 

About10daysago,hewasadmi t
tedtothehospi
talfol
lowinganepisodeofatr
ial
fi
bri
ll
ati
onandj oi
ntpai
n.Hewascar di
over
tedatt hatti
me.Hewasdi schar
ged
homewi thsever
alnew medi
cati
onsthathedoesnotr ecallthenamesof,buthe
stat
esthathehasbeencompl
iantwit
hthem. 

Atthet i
meofyourexami nati
on,thepati
ent'
sbloodpressur
emeasur ed92/47mm
Hgint heri
ghtarm and97/ 52mm Hgi ntheleftarm.Hispul
semeasur ed39bpm.
Uponauscul t
ati
on,nomur murswereappr eci
ated.CannonA waveswer enot
ed
uponneckexami nation.Labor
atorystudi
esincludenormalrenalf
unctionandan
i
nternati
onalnor
malizedrati
oof1.1.

Therecentaddi
tionofwhi
chofthefol
lowi
ngmedi
cat
ionsi
smostl
ikel
yresponsi
ble
fort
hecli
nicalf
indi
ngsint
hispat
ient
?

a.Dof
eti
li
de.
b.Ver
apamil
.
c.Amiodarone.
d.Pr
ednisone.
e.Fl
ecaini
de.

Answer

E.Fl
ecai
nide

This pat ientpr esentsf oll


owi ng a r ecentboutofat ri
alf ibril
lati
on wi th cli
ni cal
evidenceofcompl etehear tbl ock.Hi spul seisbr adycar di
c,andhedemonst rat es
cannonA waves.Hei spr esumabl ydevi ce-dependentwi thacl inicalhi storyof
compl etehear tbl ock.Fl ecai nidei sknownt oi ncreasepaci ngt hreshol ds.Att he
ti
me of i niti
at i
on of cl ass I c agent s, pacer -dependent pat i
ent s r equi re
reprogr ammi ngoft heirdevi ceswi t
hi ncreasedleadout putst oensur emyocar dial
capt ure.Thi spat ient'
scl i
ni calpr esentationi sclassi calf oradevi ce- dependent
patientwhoi si niti
atedonacl assI cagentwi t
hnor epr ogr ammi ngofdevi ceout put s.
Followi nganadequat eamountofdr ugl oad,pacingt hr esholdr iseandmyocar dial
capt uremaybel ost,r
esul t
ingi nl ossofcapt ureandar esultantescaper hythm.The
otheragent sl i
steddonotr aisepaci ngt hresholds(steroidsact uallylowert hem and
arei ncorpor atedi ntol eadt ipst oreducet hresholdsatt het imeofl eadi mpl ant).I n
thi
si nstance,f lecainideshoul dbedi scont i
nuedandpaci ngout putsi ncreasedt o
ensur emyocar dialcapt ure.

9.A 24- year-


old compet i
ti
ve runner present
s f or evaluati
on of t achycardic
palpit
ati
ons.Shedescri
best hem assuddeninonsetandt erminati
on.Theybegan
when shewasabout11 year sold.Overt heyears,shehasnot ed thatt hese
episodesareprogressivel
ywor seni
ng infrequencyand sympt oms.Herf ami l
y
histor
yi sr
emarkabl
ef oranuncl ewhodi edofamassi vehear tatt
ackatage51
years. 

Baseli
neelectrocar diogr
am ( ECG)isnor malasisherphysi calexaminati
on.Event
monitori
ng discloses evi dence ofsust ai
ned narr
ow compl ex t
achycardi
a.The
pat
ientisr ef
erredf orelectrophysiol
ogictest
ingandpossi blecathet
erabl at
ion.
Duri
ngt hecourseofel ect
rophysiologytest
ing,
thetachycardiashowni nFigure1i s
i
nduced.Dur ing t achycardia,a dr ug i
s administ
ered,resulti
ng inthe findi
ngs
di
splayedinFigur e2.
Fi
gur
e1.

Fi
gur
e2.
Whi
choft
hef
oll
owi
ngi
sthemostl
ikel
ymechani
sm oft
achycar
dia?

a.Orthodromi creciprocati
ng t
achycardi
a uti
li
zing a l
eftl
ater
alaccessor
y
pathway.
b.AtypicalAVNRT.
c.Ventricul
artachycardia(VT).
d.Leftatri
altachycardia.
e.Typicalatr
ioventri
cularnodalre-
entr
anttachycar
dia(AVNRT).

Answer

E.Typi
calat
ri
ovent
ri
cul
arnodalr
e-ent
rantt
achycar
dia(
AVNRT)
.

Figure1demonst ratesi nit


iationoft achycar dia.Theel ectrogramsar earranged
from topt obot tom asf ol
lows:sur f
aceECG,hi ghr ightatrialrecording,Hisbundl e
recording,cor onary si nus r ecor ding ( proximalt o distal)
,and r i
ghtvent ri
cular
recording.Dur ing t achycar dia,t her ei sat ypicalcol umnarappear ance ofal l
electr
icalact i
vity,wi thaver yshor tvent ri
culoatr i
al(VA)t i
me.Ear l
iestatri
alacti
vi t
y
i
snot edatt heHi sbundl er ecor ding.Thi sshor tVA t imeexcl udest achycardia
util
izi
nganaccessor ypat hwayaspar toft het achycar diccircuit,soor t
hodromi c
reciprocatingt achycar diai snotcor rect.Leftat ri
altachycar diawoul dbeexpect ed
todemonst r
ateear liestat r
ialact ivit
yatt hedist alcoronar ysinusel ectr
ograms( CS
1,2).Thisiscl ear l
ynott hecaseher e.VTi snott ypical
lyanar r
owcompl exinnat ure
and of ten exhi bit
sVA di ssoci ation.I nt hisi nstance,t heV and t heA ar e1: 1
associ atedwi t
haver yshor tVAi nterval.Thisessent i
all
yexcl udesVT.  

Thedi ff
erencebet weent ypicalandat ypicalAVNRTi sr elatedt ot heVAi nter
val
dur i
ngt achycar dia, asthi
sr efl
ectst hedi recti
onalityoft here- ent
rantci rcuit. Typi
cal
AVNRTi schar act erizedbyant erogr adeconduct iondownsl ow f i
ber soft heAVN
compl ex,wi tht he r etr
ograde l imb occur ri
ng overf asterconduct i
ng fiber s.In
atypicalAVNRT,t heopposi tei sessent i
allythecase.Hence,i nat ypicalAVNRT,t he
VAi nt ervaltypicallyi sprolonged( >70msec) ,andt hi
smani f
estsasal ongerRP
intervalon sur f
ace ECG.I nt his case,dur i
ng t achycar dia,t he sur face ECG
lead V1 doesshowsomeevi denceofshor tRPi nterval(pseudo- R'),reflecti
veoft he
veryshor tVAi nter valseenont hei nt r
acar di
acel ectrograms.Consequent ly,inthis
case,t hecor r
ectansweri st ypicalAVNRT.Thedr ugadmi nisteredi nFi gur e2was
adenosi ne, result
ingi nt
ermi nati
onoft achycar diawi t
hbl ocki nthesl owpat hwayof
theAVnodeandsubsequentcompl eteAVbl ock( notet heat ri
alel ectrogramswi th
noassoci atedHi sel ectr
ogramsorvent r
icularactivit
yt other i
ghtoft het r
aci ng).

10.
An84- year-
oldfemalewit
hapastmedi calhist
orynot
abl
ef orat
ri
alfi
bri
ll
ationand
hyper
tension,present
stothehospi
talaft
erasyncopalepisodewithoutprodrome
whil
eeat i
ngdinner.Sher
egai
nedconsciousnessaft
er20seconds,andotherthana
contusiononhercheekf rom wher eherf acehi thersoupbowl ,shewasunhar med.
Therewasno i ncont i
nenceorgener ali
zed t onicclonicact i
vit
y.Herexam was
notablef orasyst olicejectionmur mur ,butther ewer enoot hersi
gni f
icantf
indings.
Anel ectrocardi
ogr am ( ECG)wasobt ainedthatdemonst r
atedar i
ghtbundl ebranch
block( RBBB)andl eftanter i
orfasci
cul arbl
ock.Anechocar diogram demonst rated
normalbi vent
ri
cularf unctionandaor t
icscler osis.Myocar di
alperfusi
onst r
esst est
revealednoevi denceofi schemi a.Att heel ectrophysiol
ogy( EP)study,therewas
not evi dence of i nducible vent r
icular tachycar dia (VT),and t he foll
owing
elect
rogr am wasr ecor ded( Figur
e1).

Whi
choft
hef
oll
owi
ngi
sthemostappr
opr
iat
einhercar
e?

a.Single-
chamberpacemaker .
b.Dual-chamberpacemaker .
c.Eventmoni t
or.
d.Implantabl
elooprecorder
.
e.Cardiacresynchr
onizat
ionther
apypacemaker
.
Answer

B.Dual
-chamberpacemaker
.

Thepat ientdevel oped syncopei nt heset ti


ng ofbi fasci
cularblock.Diagnost i
c
testi
ngr evealsapr ol
ongedHV i nterval(nor mali s35- 55msec).Int hissetti
ng,
permanentpaci ngisindicated.I
nt hispatientwi thnor malejecti
onfract
ion,cardiac
resynchronizati
ont herapyisnotindicated.Ani mplant ablecardi
overt
er-defi
bri
ll
ator
i
snoti ndicated, asherEPst udyshowednoi nducibleVTandherej ecti
onf r
acti
oni s
>35%.Cur rentgui del
inesgi veaCl assIi ndi cati
ont opermanentpaci ngi nt he
setti
ngofpr olongedHV i nter
val,andaCl assI Iai ndicati
ont oimplantati
onf or
unexplai
nedsyncopei nt hesetti
ngofbi fascicularblock.

11.
A59-year-
oldmanpr esent
swithr
ecur
rentunexpl
ainedsyncope,
wit
houtpr
odr
ome.
Hispastmedicalhi
storyisnot
abl
eforacoronaryarter
ybypass10yearsagoand
hyper
tensi
on. 

Physicalexami nati
onr evealsahear trateof80bpm,bl oodpr essureof120/ 74mm
Hg,andj ugularvenouspr essureof8cm.Car di
ovascul arexami nationisr egul
ar
withoutmur mur .Anel ectr
ocardiogram demonst ratednor malsi nusr hythm with
QRSdur ati
onof119msec.Echocar diogram showsi nferol
ateralakinesi
sandl ef
t
ventri
cularejecti
onf r
act i
onof45%.Si ngle-photonemi ssioncomput edtomogr aphy
myocar dialper f
usion i maging shows i nferol
ateralscar wi thoutevi dence of
ischemia.

Whi
choft
hef
oll
owi
ngi
smostappr
opr
iat
eint
hecar
eoft
hispat
ient
?

a.Si
ngl e-
chamberimpl antablecar
dioverter
-def
ibr
il
lat
or(
ICD)i
mpl
ant
ati
on.
b.Diagnosti
celectr
ophysiology(EP)study.
c.Cardiacmagneticresonancei maging.
d.Coronaryangiography.
e.I
mpl antat
ionofani mplantabl
eloopr ecor
der.

Answer

B.Di
agnost
icel
ect
rophysi
ology(
EP)st
udy.

The pati
enthas syncope,str
uct
uralhear
tdisease,and evidence ofscaron
echocar
diogr
am andmyocar di
alperf
usi
onimaging.Ther
efore,revascul
ari
zat
ion
andi magingwithanaddi t
ionalmodali
ty,suchasmagnet i
cr esonanceimagi ng,ar
e
notindicated.Thenextmostappr opriatestepist oriskstrati
fyhimf orinducibl
e
ventri
culararr
hythmiaswithanEPst udy.Onlyifinduci
blevent r
icul
art
achycar diai
s
foundshoul danI CDbeimpl ant
ed.Apost i
nfar
ctionpat i
entwi t
hsyncopewar rant
s
EPt esti
ngt oexcludeinducibl
eventri
culararrhythmiaspriort oimplant
ationofa
looprecorder.

12.
A19- year-oldfemalepresentstot heemer gencydepartmentwit
h6hour shistor
yof
recurrentpalpit
ati
ons.Anel ect
rocardiogram isobtained(Fi
gure1),andadenosine
is given,t erminat
ing the rhythm.Att he time ofel ect
rophysi
ology test
ing,
atri
oventri
cular(AV)nodalre-entranttachycardi
aisinduced.

Lowerr i
skofper
manentcompl
etehear
tbl
ock;hi
gherl
ong-
ter
m ar
rhyt
hmi
a-f
ree
sur
vival
.

a.Samer i
skofpermanentcomplet
eheartblock;samel ong-ter
m arrhyt
hmia-f
ree
survi
val
.
b.Higherriskofpermanentcompl
eteheartblock;higherlong-ter
m arrhyt
hmia-
fr
eesurvival
.
c.Lowerriskofpermanentcompl
etehear
tblock;lowerlong-ter
m arrhyt
hmia-f
ree
survi
val
.
d.Higherr
iskofper
manentcompl
etehear
tbl
ock;l
owerl
ong-
ter
m ar
rhyt
hmi
a-f
ree
survi
val
.

Answer

B.Higherr
iskofper
manentcompl
etehear
tbl
ock:hi
gherl
ong-
ter
m ar
rhyt
hmi
a-f
ree
sur
vival
.

Radiofrequencyablat
ionprovidesamor edurabl
elesiont
hancryoablat
ion.However,
the risk of inadvertent permanent i
njuryt ot he AV node is hi gher wi
th
radiof
requencyablati
on.Successr atesforsl
owpat hwaymodifi
cat
ionf orAVnodal
re-ent
ranttachycardi
aar e90-95%wi thradi
ofr
equencyener
gy,andar eroughl
y80%
withcryoablati
on.

13.
A71- year-ol
dmanwi thahi st
oryofatri
alfibr
il
lat
ionpresentswi t
hconf usi
on1
monthaf t
erfal
li
ngandstr
iki
nghisheadwhileonapixaban.Duetot heconfusi
on,he
hasnottakenanyofhismedicati
onsin4days.Physicalexaminati
ondemonst rat
es
abloodpr essur
eof124/72mm Hg,cl earlungsounds,andar egularr
hythm.An
elect
rocardi
ogram isobtainedandshowslackofadisti
nctPwaveandahear trate
of64 bpm ( Fi
gure1) .A comput ed t
omographyscan demonstr
atesa chronic
subduralhemat oma.Bef oretaki
nghimt otheoperat
ingroom,theneur
osurgeon
requestsacar diol
ogyconsul ttoclari
fytherhyt
hm andwhetheranyaddi t
ional
i
nt er
venti
onisrequired.

Whi
choft
hef
oll
owi
ngdoyout
ellt
hecar
diol
ogi
st?

a.Thepati
entisinatri
alfibri
ll
ati
onandshoul dbest ar
tedondi goxinanda
bet
a-bl
ocker.
b.Thepati
entisin4:
1r at
e- contr
oll
edatri
alf
lutt
er.Noadditi
onalint
erventi
on
i
srequi
red.
c.Thepat
ienti
sinnormalsi nusrhythm.Nointer
venti
onisrequi
red.
d.Thepati
entisi
n4:1at r
ialtachycardi
aandshouldbest ar
tedonadi lt
iazem
dri
ptocontr
olther
ate.
e.Thepat
ienti
sinatr
ialfl
utterwithvari
abl
eblock.Abeta-bl
ockerisindi
cated.

Answer

B.Thepat i
enti
sin4:
1rat
e-cont
rol
ledat
ri
alf
lut
ter
.Noaddi
ti
onali
nter
vent
ioni
s
requi
red.

The pat i
enti si nr ate-cont r
oll
ed at r
ialf l
ut t
er.Typi cal( type I)at rialfl
utteri s
char act eri
zedbya" sawt oot h"patt
er ninthei nferior(I
I,I
II
, aVF)leads.Usual ly,at
rial
fl
ut terpr esentsin2: 1conduct i
onbl ockwher et herearet wof lutt
erwavesf orever y
conduct edQRScompl ex.Int heabsenceofat r
ioventri
cularnodalbl ockingagent s,
ratios>2: 1( e.
g.,3:1,4:1)suggestunder lyingconduct i
onsyst em di sease.Inthi s
situation, thepatientwasr atecontroll
edwi t
houtt heneedf orratecont roll
ingagent s
suchasbet a-bl
ocker sorcal cium channelbl ockers.Noi nterventi
oni sneededpr i
or
to sur ger y.Af t
err ecover i
ng f r
om sur gery,he was subsequent l
yt aken tot he
elect r
ophysi ologylabor atorywher ethedi agnosi soft ypi
cal ,cavo-tr
icuspi dist
hmus
atrialflutterwasconf irmed.Heunder wentsuccessf ulradiofrequencyabl ati
on.

14.
A 32-year-ol
dwomanwi t
hal onghistoryofr arepal pi
tat
ionspresentst oyour
cardi
ologycli
nicwi t
hincreasingl
yfr
equentepi sodes.Sheis10weekspr egnantand
isotherwiseheal t
hy.Sher eportsthatdur i
ngherpr egnancy,herepisodeshave
become increasinglyfrequent,and int he pastweek,t he episodes have been
occurr
ing multipletimes a day lasting severalmi nutes ata t i
me.She can
sometimesstopt heepisodeswi t
hcoughi ngorsquatti
ng.Herpul seisusuall
yinthe
140
-
150
bpm

rangeduringtheepisodes.Shei soccasi onal


lydizzywi t
htheepi sodes,buthasnot
passedout .Herbaselineel ect
rocardi
ogr am (
ECG)i snormal,butwhi l
einyourof fi
ce
shehasanepi sode,andherECGdur i
ngt heepisodei sshowni nFigure1.Herbl ood
pressurewas95/ 60mm Hgdur i
ngt achycardia.Vagalmaneuver sint heof fi
ce
fai
ledtoterminatetheepi sode,butitstoppedspont aneousl
yaf t
er5mi nutes.Aft
er
theepisodeterminated, hercardiacexami nat
ionwasnor mal
, wit
hahear tr
ateof84
bpm andbl oodpressur eof105/ 60mm Hg.

Whi
choft
hef
oll
owi
ngi
stheappr
opr
iat
enextt
her
apy?

a.At
enol
ol25mgdai
ly.
b.Di
goxi
n0.125mgdai
ly.
c.Adenosine6mgonce.
d.Cathet
erablat
ion.
e.Amiodarone200mgdail
y.

Answer

1.B.Di
goxi
n0.
125mgdai
ly

TheECGi sconsi stentwi t


hat r
ioventricularnodalr e-ent ranttachycar di a.Themaj or
concerndur ingt reat mentofsupr avent r
iculartachycar dia(SVT)dur ingpr egnancyi s
thepot entialf oradver seef fectsont hef etus,asal lcommonl yusedant i
arrhyt
hmi c
drugscr osst hepl acent albar riertosomeext ent.Al t
hought hef ir
st8weeksaf ter
concept i
oni st heper iodassoci at edwitht hegr eatestt er at
ogeni crisk,ot heradver se
eff
ectsmayoccurwi t
hdr ugexposur el ateri npr egnancy.Themaj orconcer nwi th
antiarr
hythmi cdr ugst akendur ingthesecondandt hirdt ri
mest ersi st headver se
eff
ectonf etalgr owt handdevel opmentaswel last her iskofpr oarrhyt hmi a.Aswi th
manyot herdr ugsusedi npr egnancy, useofcer tai
nant iarr
hyt hmicagent shascr ept
i
nt ocommonpr acticebecauseofanabsenceofr epor tedilleffects,rathert hanasa
resultofcont roll
ed st udi es.Al lant i
ar r
hythmi c drugs shoul d be r egarded as
potentiall
yt oxi ctot hef etusandshoul dbeavoi dedi fpossi ble,especi allydur i
ngt he
fi
rsttri
mest er.Al lcurrent l
yavai lableant iarrhythmi cdr ugst hatar eusedf orSVTar e
categor i
zedasCl assCdr ugs( usingt heUSFoodandDr ugAdmi nistration(FDA)
drugcl assificat i
onsyst em) ,exceptf orsot al
ol( aCl assBagent )andat enololand
ami odarone( ClassDagent s). 

Inpat ientswi thmi l


dsympt omsandst ructurallynor malhear ts,not r
eatmentot her
thanr eassur anceshoul dbepr ovi ded.Ant i
arrhythmi cdr ugt herapyshoul dbeused
onlyi fsympt oms ar ei ntol erable ori ft he t achycar dia causes hemodynami c
compr omi se.Ifprophyl acticdr ugt her apyi sneeded, thendi goxinorabet a-blocki ng
agent( i.
e.,propranol olormet opr olol)ist hef irst-li
neagent .Theexper iencewi th
digoxini sext ensive,andi tisconsi der edoneoft hesaf estant i
arr
hythmi cdrugst o
takedur i
ngpr egnancy;however ,
itsef fi
cacyf orar rhythmi at r
eatmentorpr ophyl axis
hasneverbeendemonst r
ated.Pr opranol olandmet opr ololar egenerallyconsider ed
tobesaf e,butar ebestavoi dedi nt hef irsttri
mest er.Thepot enti
alforintrauterine
growt hr etardationhasbeenr epor tedwi thpr opr anololandhasr aisedconcer ns,
especi all
ywheni ti st akeni nt hef irstt ri
mest er.Lat erst udiesr eport
edgr owt h
retardat i
oni nbabi esr eceivingat enol olint hefirsttrimest erandahi gherpr evalence
ofpr eterm del i
very.At enololi s,ther efore,classifiedasacl assDagentbyt heFDA.
Inviewoft heser esults, beta- bl
ocker sshoul dbeavoi deddur ingthef i
rstt
rimest er,i
f
possi ble.Ifthepr eviouslyment ioneddr ugsf ail
,t hensot alolmaybeconsi dered.
Althoughsot alolhasbeenusedsuccessf ullydur i
ngpr egnancyf orotherindications,
theexper i
encei slimit
ed,socaut i
onisst il
ladvised.Catheterablati
onr equi
res
fl
uoroscopyandshoul dthereforebeavoi dedexceptinr ef
ract
orycases.Itmaybe
appropri
atef ordr ug-r
efract
or y,poorlyt ol
erated SVT.Ifneeded,i tshoul d be
perf
ormedi nt hesecondt ri
mest er.Adenosinecanbeusedi npr egnancyforacute
ter
minationofSVT.However , i
nthispati
entwi thfr
equentparoxysmalepisodesand
onethathasal readyter
minat ed,therei
snoi ndicat
ionforuseacutely.
 

15.
A22- year-oldmanwi t
hahi st
oryofsur gical
lyr epairedtetralogyofFal l
otpr esents
toyourcl inicf oreval uationofepisodesofpal pit
ations.Hehasgener all
ybeendoi ng
wel l
,butaf ew mont hsagobeganhavi ngepi sodesofpal pi
tat
ionswi thel evated
hear tratesof120- 130bpm,whi chhavebeenl asti
ngaf ew hoursatat ime.He
thinkst hatover allhi sexer ci
secapaci tyhasbeenal i
ttl
ewor sef orthepast6
mont hs as wel l
.He wor eat r
anstelephonic moni torpr iortot his vi
sit,whi ch
demonst rateda2- hourepi sodeofat rialfl
utterwi t
hvar i
abl eatri
oventri
cul ar(AV)
conduct ion, correlatingwi t
hhissympt oms.Hi scar diacexami nat
iont odayr evealsa
regularr ateandr hythm,nor malS1/ S2 wit
houtgal l
op,andagr ade2/ 6syst oli
c
ejectionmur mur( unchangedf rom pr i
or).Electrocar di
ogram demonst ratessi nus
rhythm wi t
haf irst-degr eeAVdel ayandr i
ghtbundl ebr anchbl ockwi thQRS140
msec.

Whi
choft
hef
oll
owi
ngi
sthemostappr
opr
iat
enextst
epi
nmanagement
?

a.Transthoracicechocardiogram.
b.Metopr ol
ol25mgt wicedaily.
c.Radiofrequencycat het
erablati
on.
d.Sotalol120mgt wicedaily.
e.Diagnosticcardiaccatheteri
zati
on.

Answer

A.Tr
anst
hor
aci
cEchocar
diogr
am

Whi l
et reatmentwithmedicati
onsorabl ationmayevent ual
lybeappr opriateint his
patient,t he mostimpor t
antf i
rstst ep herei st or eassess hi s structuraland
hemodynami cstat
us.Thedevel opmentofat ri
altachycardia,atrialfl
utter,orat ri
al
fi
br i
ll
ati
on i n adul
tcongenitalhear tdi sease patientsis often associ ated wi t
h
progressive hemodynami c det eri
orati
on of t he under l
ying di sease.Sur gical
treatmentoft hehemodynami cpr oblemsdoesnotel i
minateatrialarrhythmi as,and
ablationofat ri
alarr
hythmiasalonecoul dal l
owsi gnifi
canthemodynami cissuest o
progressandpot enti
all
ydeteri
orate.Successfultreatmenti nvolvesassessi ngbot h
thear rhythmiaandt hecont r
ibuti
nghemodynami cchangesandaddr essingbot h
whenindicatedandfeasibl
e.Cardiaccathet
eri
zat
ionmaybeappr opri
atet
operform
mor edetai
ledhemodynami cassessment,butinl i
ghtofhisunchangedphysical
examinati
on,echocardi
ographywi l
lprovi
deimportanti
nfor
mati
onwi t
houttheneed
foraninvasivepr
ocedure. 

Early exper i
ence in adul ts wi th unoper ated at rialsept aldef ects and at r
ial
arrhythmiasdemonst ratedt hei mpor tanceofani ntegratedappr oachf orarr
hyt hmi a
andhemodynami cpr oblems;si mi larpr i
ncipl esappl yt opat i
ent swi thtetr
alogyof
Fallot
,Ebst ein’
sanomal y,andsi ngle-ventriclephysi ology;t hesepat i
entsar eat
highest r i
sk of ar rhythmi a devel opment , wi th concur rent hemodynami c
abnor malit
ies.Beta-blocker s maydecr ease cat echolami ne- r
elated tri
ggers and
provideAVnodalbl ockadedur ingr ecur r
entat ri
alar r
hythmi as.Onest udyofadul ts
withtransposi t
ionofthegr eatar teri
esandpr ioratrialswitchr epairswi t
himpl anted
defibri
ll
ators demonst rated supr aventricular t achycardia pr eceding vent ricular
tachycardiain50%ofpat ients;useofbet a-bl ockermedi cat i
onsi nt hispopulat i
on
wasassoci atedwi t
hdecr easedi ncidenceofappr opriatedefibril
latorshocks. 

Observationalstudiesont heuseofsot alolinACHDpat ientsreportfreedom from


recur
rentat ri
altachycardi
ai n41- 46%ofpat ient sduri
ngshor t
-ter
mf ollow-up.Use
ofeithermedi cati
on int he set t
ing ofsi gnifi
cantsi nus node dysf unction may
exacerbatebr adycardi
aandr equirescarefulmoni tor
ing.I ni
ti
ati
onofsot al
olinthis
populati
oni srecommendeddur i
ngi npatientmoni t
ori
ngf orproarrhyt
hmi afor48- 72
hours.Mul tipl
eobser vati
onalandmul ticenterst udieshavedemonst ratedacut e
successr atesof65- 100%f ortreatmentofsupr avent
riculartachycar
diaassoci ated
with ACHD.Acut esuccessr atesvar ybyt achycardia mechani sm and t ypeof
congenitalheartdiseaseandr epair.

16.
A30- year-oldmanwhoi sapr ofessi onalsoccerpl ayerisadmi t
tedt hroughthe
emer gencydepar t
mentt ot hecar diologyt elemet ryunitaf t
errecur rentepisodesof
syncope.Hehasahi st
oryofpr emat urevent ri
cularcont r
actionandunder went
premat ureventricularcont r
actionabl ati
onf rom t her i
ghtvent ricular(RV)out f
low
tract3year sago.Hi scardiacexami nat ionr evealsar egularrat eandr hythm with
nor malS1/ S2 
and no mur murs orgal l
ops,and t he remainderofhi s physical
exami nationisnormal .His12- l
eadel ectrocar diogram (
ECG)i sshowni nFigure1.A
car diacmagnet icr esonancei magi ngi sper formed,whi chr eveal samoder ately
dilated RV wi t
h moder atelyreduced f unction ( RV ejecti
on f racti
on 38%)and
hypoki nesisofthebasalRVf r
eewal lwher edel ayedenhancementi salsoseen.An
electrophysi ol
ogyst udyisper formed,whi chi nducesvent ri
culart achycar di
a(VT)
(Figur e2) .Heunder goesi mplantationofani mpl ant
ablecar diover t
er-defi
bri
ll
ator
(ICD) .
Fi
gur
e1.

Fi
gur
e2.
Basedonrecentguidel
ines,whichofthefol
lowi
ngwoul
dyour
ecommendt
othi
s
pat
ientt
ominimizehisprogressi
onofdi
sease?

a.Ami odaronetherapy.
b.Calcium channelblockerther
apy.
c.Cardiacresynchronizati
ontherapy.
d.Beta-blockertherapy.
e.Cessationofcompet i
ti
veathl
etics.

Answer

E.Cessat
ionofcompet
it
iveat
hlet
ics.

Thispati
enthasarrhythmogenicRVcar diomyopathy(ARVC).Hehasseveralmajor
ARVCt askfor
cecriter
ia,i
ncludi
ngepsi l
onwaves,leftbundlesuperi
oraxi
sVT,and
magneticresonance imaging wi t
hf ocalRV dysfuncti
on and reduced ej
ect
ion
fr
acti
on.T-wavei nversi
onst hroughV4inthepr esenceofar i
ghtbundlebranch
blockarealsoami norelectr
ocardiogr
am cri
teri
on.Whi l
er andomizedcont
roll
ed
dataf ort hemanagementofARVCpat ientsar esomewhatl imited,obser vational
studiesandwor kext rapol atingthemanagementofvent ricularar r
hythmi ashave
beenusedt oder ivet hecur rentgui delinesf orthemanagementofpat i
ent swi th
ARVC.Fewsyst emat i
cdat aar eavail
abl eont heefficacyofant iar
rhyt hmicdr ugsi n
ARVC, andt hei mpactofmedi caltherapyonmor t
alityi sunknown.Basedl argelyon
serialel ect r
ophysi ologyt est i
ng,beta- blockersar econvent i
onal l
yr ecommendedas
thef irst-li
neappr oachi npat ient
swi thf requentvent ricularect opyornonsust ai
ned
VT.I nvasi veel ectrophysi ologicaltest i
ngwi t
hvol t
agemappi ngcanbeusedt o
identifyr egi ons off ibro- fattyreplacementand t o gui de cat heterabl ati
on of
ventr i
cul arar rhythmi as.As nei t
herant i
arrhythmi c dr ugs norcat heterabl ation
providessuf fi
cientpr otectionagai nstsuddencar di acdeat h,abl ationshoul dbe
used t or educe t he f r
equencyofar r
hythmi a epi sodes r athert han t oi mpr ove
prognosi s.Endur ancet raining ata compet i
ti
vel evelpr obabl yexacer batest he
phenot ypeofARVC,andmostgui deli
nest hereforer ecommendavoi danceofhi gh-
levelexer cise.Ar ecentpr ospect i
veobser vationst udyofARVCpat i
entsfoundt hat
high- levelcompet i
ti
veat hleteswhor educet heiract ivitylevelt oeithernoexer cise
oronl yr ecr eationalact i
vityhadar educt ioni nvent ricularar rhythmi asorsudden
deat h.

17.
A67- year-ol
dmani sadmi t
tedtothecar diacintensi
vecareuni tafterpresenting
withpr esyncopeatwor k.Emergencymedi caltechnici
ansfoundhim wi thasyst oli
c
bloodpr essureof70andat hreadypulse.Anext er
naldefi
bri
ll
atordemonst rateda
widecompl ext achycardiaat185bpm,andhewassuccessf ull
ycar diovert
edwi th
returnofcirculation.Hewashemodynami callystabl
eintheemer gencydepar tment ,
andeval uat i
onf oracutecoronarysyndromewasnegat i
ve.Hehasapastmedi cal
historyofischemi ccar
diomyopathywithpr i
orinferi
ormyocardiali
nfarction6year s
pr
ior

His electrocar
diogram showed i nf
er iorQ waves and no i schemi c changes.
Transthoracicechocar diogram r
eveal ed i
nfer
iorakinesi
sand thinning,withleft
ventri
cular(LV)ejecti
onf racti
on(
EF)of35%, sl
ightl
yreducedf
rom hisEFof40%on
hislastechocardiogram 1yearpr ior.Hi smedicati
onsathomei ncludeaspi r
in81
mgdai l
y,met oprolol50mgdai ly,andsi mvastat
in20mgdai ly.Heunder goes
implantati
onofasi ngle-chamberimpl antabl
ecardiover
ter-
def
ibr
il
lator(I
CD).

Whi
choft
hef
oll
owi
ngt
reat
ment
swi
llr
educehi
sri
skofI
CDshocks?
a.Digoxi
n0. 125mgoncedail
y.
b.Li
sinopri
l20mgoncedaily.
c.Sotal
ol120mgt wi
cedail
y.
d.Fl
ecaini
de100mgt wicedail
y.
e.Dronedarone400mgtwicedail
y.

Answer

C.Sot
alol120mg.t
wicedai
ly

Inpat ientswi thst ructuralhear tdi sease,r educedEF,andsecondar yprevention


ICDs( f
orhi storyofvent ri
culararr hythmiasorsyncopewi thinducibleventri
cular
arr
hyt hmi as),bothsot alolandami odar onehavebeendemonst ratedtobothreduce
ther iskofI CD shocksand r educemor tali
ty( OPTIC[ OptimalPhar macological
Ther apy i n CardioverterDef i
bril
latorPat ients]st udy,Sot al
olI CD [Sotal
olas
Adjunct ive Therapy t oI mplantabl e Car diovert
er-Def i
bril
lator
si n Hear
tFai lure
Patients]st udy).Empi rictreatmentwi ththesemedi cati
onsmaybeappr opri
atei n
pati
ent s wi th a historyofvent riculart achycar di
a( VT)and I CD.However ,t he
potent i
alf orsi de ef fects,effects on def i
bril
lat
ion t hreshold,and al t
ernative
str
at egiessuchasVTabl ationshoul dbegi vencar efulconsi derati
oninthiscli
nical
scenar io. 

TheclassI cant i
ar r
hyt hmicmedi cati
ons(flecaini
de,propafenone)havebeenshown
toincreaset heriskofar rhythmi cdeat hinpost myocardiali
nfarcti
onpati
ent
s( CAST
[Cardi
acAr rhythmi aSuppr essionTr i
al]t
rial)
,andt herefore,shouldbeavoidedi n
thi
spat ient.Lisinoprilislikelyindicatedfort hispati
entwi threducedLVfunction,
buthas notbeen shown t o have an ef f
ecton t he incidence ofventri
cular
arrhyt
hmi as.Di goxinhasal sonotbeenshownt or educet her i
skofVTorI CD
shocks.Dr onedar onei scont rai
ndi cat
edi npat i
entswithcongest i
veheartf
ail
ureor
reducedLVsyst olicfunction.

18.
A52- year
-oldmanwi thahi storyofparoxysmalat ri
alfibril
lati
onpr esentst othe
emer gencydepar t
mentaf teranepisodeofsyncopewhi l
ehewascooki ngdi nnerin
hiskitchen.Her eportshef el
tli
ghtheadedf orasecondbef orecollapsi ng;hiswife
witnessedt heepi sode,andhewokeupandf eltfi
ne10secondsl ater.Int he
emer gencydepar t
ment ,hi
shear trat
eis80bpm,hi sbloodpr essurei s140/ 76mm
Hg,andheappear swel l
.Homemedi cati
onsi ncl
udeflecainide100mgt wicedai l
y
andmet oprolol50mgt wicedaily.Hi
sel ectr
ocardi
ogram ( ECG)i nt heemer gency
depar tment(Figure1)iscompar edwi t
hhispr i
orECGf r
om l astyear( Figure2) .
Fi
gur
e1.

Fi
gur
e2.
Whi
choft
hef
oll
owi
ngmostl
ikel
yexpl
ainshi
scur
rentECGf
indi
ngsandsyncope?

a.Increasedvagaltone.
b.Excessivebeta-bl
ockade.
c.Brugadasyndr ome.
d.Ischemia.
e.Torsadesdepoi ntes.

Answer

C.Br
ugadaSyndr
ome

Thispat ient'sECGdemonst ratest ype1Br ugadapat t


er n.Sodium channelbl ocker
s
(fl
ecaini de,pr opafenone)canexacer batethetransientECGabnor mal iti
esthatoccur
inpat ient swi thBr ugadasyndr omewhocommonl yhavenor malECGs.Ser ialECG
testi
ngmaydemonst ratethei ntermi t
tentchanges,anduseofcl ass1cagent s
duringel ectrophysi ol
ogict esti
ngcanbeusedt ounmaskBr ugadapat t
ern.This
patient'sECGandhi storyaremostconcer ni
ngf orBr ugada- i
nducedpol ymor phi
c
ventricul artachycar dia/ventr
icul arf ibr
il
lati
on.Excessi vebeta- blockadeandvagal
tonemayr esultinsyncope,par ticularlyintheset ti
ngofaconver sionpausef rom
atri
alf ibrill
ation,butt hesewoul dnotexpl ai
nt heECGchanges.Ther earenoECG
fi
ndingsofi schemi a,andt heQTi ntervalisnormal.Tor sadesdepoi nteswoul dbea
concer ni ft
hepat i
entwer eonacl assI I
Iant i
arr
hythmi cdr ug(sotalol
, dofet
il
ide)and
hisQTwaspr olonged.

19.
A68- year
-ol
dwomanisadmitt
edwit
hdyspneaandwei ghtgai
nforthepastmonth.
Shehasnochestpainorsyncope.Shehasahi stor
yofhyper t
ensionandbreast
cancer,buthadbeenprevi
ousl
ywell.Onexam,shei scomfortable,wi
thbl
ood
pressureof138/
80mm Hgandani rregul
arhear
trateof138bpm.Hervenous
pressur eis16cm H2O,car oti
dsar ebr i
sk,andl ungexami nati
onisnot ablefor
dullnessoverr i
ghtlungbase.Hear tsoundsar ei r
regular,withaf ai
ntrespi
rophasi c
systolicmur murovertheleftlowersternalborder;ani ntermitt
entgall
opispr esent.
Theabdomeni sobese,wi t
hapal pableliveredge.Theext remit
iesarewar m with
traceedema.Herel ectr
ocardiogram showsat ri
alfibril
lati
on( AF)wit
hal eftbundl e
branchbl ock.Anechocar di
ogr am showsanej ecti
onf racti
onof35%,wi t
hanend-
diastoli
c di mension of 6 cm,moder atet ri
cuspid r egurgit
ati
on,and mi tral
regurgitat
ionwi t
hl ef
tatri
alenl ar
gement.Thyr oidst i
mul at
inghor monei snormal .
Hergl omer ul
arfi
lt
rati
onrateis30cc/ min.Acar dioversionisplanned.

Whi
choft
hef
oll
owi
ngi
sthemostappr
opr
iat
edr
ugt
omai
ntai
nsi
nusr
hyt
hm?

a.Amiodar one.
b.Dofet
il
ide.
c.Sotal
ol.
d.Dronedar one.
e.Fl
ecainide.

Answer

A.Ami
odar
one

Thi spat i
enti spresent ingwi thacut edecompensat edhear tfailure(HF) ,compl i
cated
byr api dAFandr enalf ail
ure.AftertreatmentofherHF, i
twoul dber easonabl etore-
establ i
sh si nus r hythm because i tmay be pl ayi ng a r olei n hernew- onset
car diomyopat hy( e.g.,tachycar di
a- i
nduced) .Herr enalf ail
ur emakessot al
oland
dof etil
idepoorchoi cesbecauset heyar ebot hr enallyexcr eted.Fl ecainideandot her
class I c agent s ar e cont raindicated i n HF. Dr onedar one has si mil
ar
electrophysi ologicpr oper t
iestoami odar one,butl ackst heiodi nemoi ety.Basedon
theANDROMEDA( IncreasedMor tali
tyaf terDr onedar oneTher apyf orSever eHeart
Failure)andPALLASPALLAS( Per manentAt r
ialfi
br iLLAti
onOut comeSt udyUsi ng
Dronedar oneonTopofSt andar dTher apy)t ri
als,iti scont raindicatedi npat i
ents
withNew Yor kHear tAssoci ationcl assI VHForcl assI IorI I
IHFwi thar ecent
decompensat ion r equi r
ing hospi t
alization.Ami odar one isar easonabl e choi
ce,
althought herear enol ar
ge-scal er andomi zedt r
ialsi nacut eHF.I nt heAF- CHF
(AtrialFibr i
llationandCongest i
veHear tFailure)tri
al ,therewasnomor talit
ybenef i
t
tot hest r ategyofr hythm ver susr atecont rol
,al t
hought hiswasnotat r
ialofAF
versussi nusr hythm.  

20.
A45-
year
-ol
dmanwi
thpar
oxysmalat
ri
alf
ibr
il
lat
ionpr
esent
stot
heemer
gency
depart
mentwi t
h palpitat
ions and pr
esyncope.Hi
s home medi cati
ons include
fl
ecai
nide100mgt wi
cedai l
yandmet opr
ololXL25mgdai l
y.Hisheartrat
ei s190
bpm,hisbloodpr essureis82/40mm Hg,andheappear spaleanduncomf ortable.
Hisiniti
alelectr
ocardiogram (ECG)is shown in Fi
gure 1.Before he can be
cardi
overt
ed,hishear trat
eabr upt
lysl
owst o105bpm.Ar epeatECGi sshowni n
Figur
e2.

Fi
gur
e1.
Fi
gur
e2

Whichoft hef
oll
owi
ngmechani
smsexpl
ainswhyhi
sQRSwaswi
derathi
gher
heartr
ates?

a.Funny-cur
renti
nhi
bit
ion.
b.Anti
arr
hythmiccl
assIcinducedventri
culart
achycar
dia(
VT)
.
c.Anti
arr
hythmiccl
assIIadrenergi
cblockade.
d.Anti
arr
hythmiccl
assIIIr
everse-usedependency.
e.Anti
arr
hythmiccl
assIcusedependency.

Answer

E.Ant
iar
rhyt
hmi
ccl
assI
cusedependency.

ClassIcantiarr
hyt
hmi cdrugs( fl
ecai
nideandpr opafenone)haveuse- dependent
sodium channelblocking ef
fectsbecauset hechanneli spreferent
ial
lyblocked
wheni ntheopenst ate.Athigherhear tr
ates,thesodi um channelisopenf ora
l
argerproporti
onofthecar di
accycle,r
esulti
ngini ncreasedblockade.Thisresult
s
i
nQRSwi dening.Thiscasei l
lust
rat
esaf requentpr esentat
ionofpatient
streated
withclassIcagentsforatr
ialf
ibri
ll
ati
on.Themedi cati
oncansl ow/organi
zeatri
al
fi
bri
ll
ationintofl
utt
erandall
owf or1:1atr
iovent
ri
cularconducti
on,whichathigher
heartr
at esresul
tsi
nwidecomplextachycardi
athatmaybemi st
akenforVT. 

Useofbet a-blockersi srecommendedwhenusi ngclassI cagent stor educet her isk


ofthi shappeni ng.Thi spat ientwasappr opr i
atel
yonbet a-blockers,althought his
does notal ways pr event1: 1 conduct i
on off l
utterf rom occur ri
ng.Cl ass I II
antiarrhythmi cdr ugs( sotalol,dofeti
li
de)blockpot assium channel spreferentiall
yi n
thei nactivest ateand,t herefore,haver everseuse- dependentef fects,resultingi n
greateref fectatsl owerhear trates.Thisexplainswhyr iskoft orsadesdepoi ntesi s
higheratsl owerhear trates.ClassI Ianti
arrhythmicdr ugs( beta-blockers)decr ease
hear trate,butdonotsl ow intramyocardialconduct i
onsi gnif
icantlyand,t herefor e,
donotwi dent heQRS.Funny- currenti
nhibitors(i
vabradi ne)actsol el
yont hesi nus
node,r esul ti
ngi nsl owerhear tratesviadecr easedaut omat icit
y,anddonotaf fect
QRSdur at ion.Whi leclassI cagent scanhavevent ri
cul arproar r
hythmi ceffects,t he
i
niti
alECGi snotconsi stentwi t
hVT.

21.
A60- year-oldwomanhasahi st
oryofparoxysmalatri
alfi
bril
lat
ion(AF)t hathas
beensympt omati
cenought owar r
anttr
eatment.Shewasstartedonsot alol80mg
bid,whichwast it
ratedupt o120mgbi dbecauseofongoingrecurrences.Shewas
start
edonhydr ochl
or ot
hiazi
de25mgqd1weekagof orhypertensi
on.Shewas
brought by ambul ance to t he emergency room for mul ti
ple episodes of
li
ghtheadednessanddi zziness.Herbloodpr essur
eis120/ 76mm Hg,andher
serum potassium i
s3.2mEq/ L.Herel
ectr
ocardi
ogram i
sshowni nFigure1.
Fi
gur
e1.

Whi
choft
hef
oll
owi
ngshoul
dbet
henextst
epi
nhermanagement
?

a.Elect
ri
calcar diover
sion.
b.Tempor arytransvenouspacing.
c.IVmet oprolol
.
d.Intr
avenous( IV)potassi
um andmagnesi
um.
e.IVami odarone.

Answer

D.I
ntr
avenousPot
assi
um andMagnesi
um.

Sot
alolisaclassI
IIanti
arr
hyt
hmi cagentusedpri
maril
yfort
het
reat
mentofAF.It
bl
ockstheinwardrecti
fyi
ngpotassi
um channel
,resul
ti
nginapr
olongat
ionoft
he
QTinter
val

Figure1showsnonsust ainedepisodesoft or
sadesdepointes(TdP),
account i
ngf or
thepatient
’scomplaint
sofpalpitati
onsanddi zzi
ness.HerQTcis570msec( QT400
msec/ squarerootoftheRRi ntervalof600msec) .Ther i
skofTdPwi thsotalolis
<2%,buti s hi
gheri nthe setti
ng ofbr adycardi
a,female sex,pre-exi
sti
ng QT
prolongati
on,hist
oryofheartfailure,hi
storyofventri
cul
artachycar
dia/ventr
icular
fi
bril
lat
ion,orhypokal
emia. 

Thetr
eatmentofsust ainedTdPinahemodynami call
yunstablepatientisprompt
el
ect
ricaldef
ibr
il
lat
ion.I nastabl
epati
ent,IVmagnesium iseffect
iveinbot hthe
tr
eat
mentandpr eventionofTdP.Temporarytr
ansvenousoverdri
vepaci ngtotr
eat
TdPusuall
yisreser
vedf orpat
ient
snotr
espondingtoIVmagnesium. 

ClassI I
Iantiarrhyt
hmi cdrugssuchassot alolhaver everse-usedependence,such
thattheQTl engthensast heheartrat
e( HR)sl ows,andt heQTshor t
ensast heHR
i
ncr eases.Thi shelpsexplai
nwhyt empor arypaci ngandI Visoproter
enolhelpt r
eat
TdPbyi ncreasingt heHRanddecr easi
ngt heQTi nterval
,wher easslowingt heHR
withbet a-blockerswouldbecont rai
ndicat ed.I Vami odar one,whichalsopr olongs
theQT i nt
erval,woul dbecont r
aindi
cat ed.Ther efore,themostappr opriat
ef ir
st
steps int reating thi
s pati
entwoul d be t reat
ing herhypokal emia,infusing IV
magnesi um andpot assi
um, anddi
scontinui nghersot al
ol.

22.
An 81- year-old woman wi th hypertension,noni schemi c car diomyopat hy and
congest ivehear tfai
lure, andpermanentat rialfi
br i
ll
ati
onpr esent st ot heemer gency
depar tmentwi th alt
ered ment alst atus and f evers.Home medi cat ions include
met oprolol,li
sinopril
,di goxi
n,andf urosemi de.Onexami nation,shei sdi soriented,
hert emper atureis38° C, herhear
tratei s140bpm, herbloodpr essur ei s95/ 60mm
Hg, andshei ssat ur
ati
ng93%onr oom ai r
.Shei stachycardic, wit
hnor malS1  andS2,
+S3,and no mur murs.Lung exam r eveal s bibasil
arr ales,herj ugularvenous
pressur ei s12cm H2O,andshehas2+bi l
ateralper i
pher aledema.Labor at
or y
evaluationi snot abl
ef orawhi tebloodcel lcountof15, 000,cr eatininei s2.5mg/ dl
,
potassium i s5. 6mEq/ L,andur i
nalysisisconsi st
entwi t
haur i
nar yt ractinfecti
on.
Herel ectrocardiogram isshowni nFi gure1.
Fi
gur
e1.

Whi
choft
hef
oll
owi
ngi
sthemostappr
opr
iat
etr
eat
mentf
orhert
achycar
dia?

a.I
ntr
avenous( I
V)beta-
blockers.
b.I
Vflui
ds.
c.I
Vcalcium channelbl
ockers.
d.Di
goxinimmuneFab.
e.Amiodarone.

Answer

D.Di
goxi
nimmunef
ab.

Thispat ient'
sel
ectrocardi
ogram demonst r
atesbidi
recti
onalvent r
iculart
achycardia
(VT),whi chismostcommonl yseenint hesett
ingofdigoxintoxicit
y.Digoxi
nblocks
thesodi um- pot
assi
um adenosinet r
iphosphatase,l
eadingt oincreasedintr
acell
ular
calcif
icationandincreasedvagaltone.Toxi ci
tycanresultinpr oarrhyt
hmiceffects
(at
rialt achycardi
a wi t
h block,bidirecti
onalVT) .Di goxini s excreted by P-
glycopr oteintranspor t,and t hus,t oxici
tyr isk i
si ncreased int he pr esence of
ami odarone,dronedar one,andver apami l.Di goxinisr enall
ycl ear ed,andi nt hi
s
pat ient,herdecreasedr enalfunct
ioni sthel ikelyexpl
anat i
onf orherdi goxintoxici
ty.
Tr eatmentofdi goxint oxicit
yi ncludes cor rect
ion ofpot assium/ magnesi um,
atr opineorpaci ngf orbr adycardi
a,lidocai nef orVT/ventricul
arf i
br i
llati
on( VF),and
Fab ant ibody admi nistrati
on forr efractory VT/ VF and hyper kalemi a.However ,
treat mentwi thl i
docai neorami odar oneal onemayt erminatet heVT,butt hese
pat ientsmayhaveunder lyi
ngcompl etehear tblockaswel lbecauseoft hedigoxin
toxi ci
ty,andt herefore,i tismor eappr opriatet oreverset het oxicitywi thdigoxin
i
mmuneFab.

23.
A72- year-ol
dmanwi t
hat r
ialfibr
il
lat
ion,forwhichheisonwar f
ari
n,cal
lsyour
off
ice because hi
si nt
ernat
ionalnor mali
zed rati
o(INR)was 10.5.He has no
complaintsanddeniesanyevidenceofbleeding.

I
n addi
ti
on to hol
ding warf
arin,which oft
hef
oll
owi
ng do you r
ecommend f
or
managementofhisanti
coagulati
on?

a.Monit
or i
ngonly.
b.Pr
othrombincompl exconcent
rat
e.
c.Or
alvitaminK.
d.Fr
eshfrozenplasma.
e.Pl
atel
ettransf
usion.

Answer

C.Or
alVi
tami
nK

War f
arincanber eversedwitheit
hervi
taminKorf reshf r
ozenpl asmat onor malize
theI NR.Whent heINRis4-10wi t
houtevi
denceofbl eedi
ng, holdingwar far
infor1- 2
dosesandf requentINRmoni t
ori
ngisgener al
lyal lthatisr ecommended;asmal l
or aldoseofvi taminKmaybeconsi der
edifpatientsar eatel evatedriskofbleeding.
Forpat i
ents wi t
hI NR >10 even withoutbleedi ng,or alvi tamin K shoul d be
admi ni
steredandwar f
ari
nheldfor1-2doses.Pr othrombi ncompl exconcent r
at e,
freshf rozenplasma,andpl at
elet
sareallintr
avenoust herapiest hatwouldr equire
hospi tali
zati
onandar eonlyappropri
ateforemer gencybl eedingduet other i
skof
thr ombot i
ccompl i
cat
ionsandtransf
usionreacti
ons. 

24.
A45- year
-oldwomanwit
hnonvalvul
arat
ri
alf
ibr
il
lat
ionpr
esent
saf
teramechani
cal
fal
l.Shesli
ppedandhi
therheadonthecurbandhaschangesi
nherment
alstat
us.
Comput
edt omogr
aphyoft heheadr eveal
sasmal lsubdur
albl
eed.Hercur
rent
medi
cat
ionsaremetopr
ololsucci
nat
eanddabigat
ran.

Whichofthefol
lowi
ngbestdescr
ibest
hemechani
sm bywhi
chdabi
gat
rancauses
ant
icoagul
ati
on?

a.Vit
ami nKantagoni sm.
b.Anti
thrombinIIIi
nhi bi
ti
on.
c.Adenosinediphosphat e(ADP)r
ecept
orant
agoni
sm.
d.Dir
ectthrombininhi bi
ti
on.
e.FactorXainhibit
ion.

Answer

D.Di
rectt
hrombi
ninhi
bit
ion.

Dabigatr
anisadi rectthrombininhibi
tor.Warfar
inisavi t
aminKant agoni
st.Low
molecularweightheparin,aswel lasri
voroxaban,edoxaban,andapixabaninhi
bit
f
actorXa.Cl opidogrelinhibi
ts ADP receptor
s on the plat
eletsur
face.Hepari
n
i
nhibit
santit
hrombinI I
I.
 

25.
Whichofthefol
lowi
ngbestdescr
ibest
hemechani
sm bywhi
chedoxabanexer
tsi
ts
ant
icoagul
antef
fect
?

a. I
nhibi
tionoffactorXa.
b. I
ncreasedfi
brinolysi
s.
c. I
nibi
ti
onoft hrombin.
d. I
nhibi
tionofti
ssuef act
or.
e. I
nhibi
tionoffactorVII
.

Answer

A.I
nhi
bit
ionoff
act
orXa.

War far
ini san anticoagul
antthatwor ksbyi nhibit
ing thevit
amin K-dependent
clotti
ng factor
s( II
,VI I
,IX,and X).Ri var
oxaban,api xaban,and edoxaban ar e
anticoagulantsthatwor kt hr
ough factorXa inhibiti
on.Dabigatr
an is a direct
thrombini nhi
bit
or.Alloft
heseagent
smaybeusedf ornonvalvul
aratr
ialf
ibr
il
lat
ion. 

26.
Concerns r
egarding dr
ug int
eract
ions have i
ncr
eased wi
tht
he use ofnewer
medicati
onsforcardi
ovascul
arther
apy.
Whichoft
hefoll
owingmedicat
ionsmayi
ncr
easet
heser
um l
evel
sofdabi
gat
rani
n
vi
voviaP-
glycopr
otei
nint
eract
ions?

a.Car bamazepine.
b.Ami odar one.
c.Ri
f ampi n.
d.Phenyt oin.
e.St.John' sWor t
.

Answer

B.Ami
odar
one

Thecorrectansweri sami
odaronebecauseiti
san inhi
bit
orofP- gl
ycopr
otein.
Hence,
co- admini
str
ati
onof
 ami
odaronewit
hdabi
gat
ranmayi ncr
easeser
um levels
ofdabi
gatran.
 

Theot hermedicat
ionsareP-glycoprot
eini
nducers,therebyexer
tingt
heopposi
te
ef
fect.Prescr
iber
sshouldbecogni zantofthepot enti
alforincr
easedef
fi
cacyof
dabigatr
anwhencoadmi ni
ster
ingP- gl
ycopr
otei
ninhibi
tors.

27.
A 68- year-ol
dmani sr ef
erredt oyourcl i
nicf orevaluati
onofnew- onsetatr
ial
fi
brill
ation(AF).Physi calexami nati
oni sremarkableonlyforani rr
egularpul
se.An
electrocardi
ogram demonst ratesAFwi thavent ri
cularresponseofappr oxi
mately
90bpm.I naddi t
iont oor deringanechocar diogram anddi scussingrate-ver
sus
rhythm- contr
olst rategies,you r ecommend ant i
coagulat
ion withthe noveloral
anticoagulantrivaroxaban.Heexpr essesconcer naboutant i
coagulati
onbecause
hisspouseuseswar f ari
nandhashadt omakedi etaryandmedi cati
onchangesto
avoi
dint
eract
ionst
hatl
edt
oexcessbl
eedi
ngr
isk.

Whi
chofthefoll
owi
ngdr
ugswoul
dbemostl
ikel
ytocausebl
eedi
ngi
fcombi
ned
wi
thri
var
oxaban?

a. At
or vastat
in.
b. Sotalol
.
c. Tr
imet hoprim-sul
famet
hoxazol
e.
d. Cl
ar i
thromycin.
e. Phenytoin.
Answer

D.Cl
ari
thr
omyci
n

Rivaroxabanisanovelor alanticoagul
antwithasi mil
areffi
cacyt owar f
ari
nf or
preventi
ngstrokeinnonval vul
arAFandi sassoci
atedwi thasimi l
arbleedi
ngr i
sk,
alt
houghwi thfewerdrug-druganddi et
aryint
eract
ionsthanwar f
arin.However,asa
substrateoft
hecytochromeP450( CYP3A4)andP- glycoprot
einsystemsi nt
heliver
,
therearecli
nical
lyr
elevantdrug-drugint
eract
ionsthatcanaffectthesafeuseoft he
drug. 

Strongi nhi
bit
orsofCYP3A4cani ncreaser ivaroxabanl evels,
leadingt obl eeding:1)
azole-antimycoti
cs,i ncluding ketoconazol e,itraconazole,and vor iconazol e;2)
human i mmunodef ici
ency vi r
us pr otease i nhibit
ors,such as r itonavir;and 3)
clari
thromycin.Mostant ibioti
csincreasewar f
arinact i
vit
ybychangi nggutf l
oraand
product i
onofvitami nK.Sul f
amet hoxazol eisalsoaCYP2C9i nhibitort hatincreases
war f
ari
nl evel
s,butdoesnoti ncreaser ivaroxabanl evels.Phenytoini sani nducerof
CYP3A4andsowoul dbeexpect edt odecr easer ivaroxabanlevel s,andal t
hough
thismaybecl inicall
ysignifi
cant( decreasedant icoagulati
onef f
icacy) ,therewoul d
notbei ncreasedbl eedi
ngr isk.Sotalolandat orvastati
ndonothaveanysi gnificant
i
nt er
actionwithrivaroxaban.

28.
A 76- year-old man wi t
h a hi storyofnoni schemic cardiomyopat hyand mi tral
regurgitati
ont reat edwi thami tralvalver epai rwithamechani calvalveisadmi t
ted
wi t
hr ecurrentat rialfibri
ll
ation.Hehaspr eviouslybeent reat edwi thdofetil
ide,but
hashadt wobr eakt hroughsont histher apy.Onadmi ssi
on,hishear tr
ateis125bpm,
buthei sot herwi sehemodynami call
yst able.Hehasbeenonwar f
arinat7. 5mg
dailyforhismechani calvalveandhi si nternat i
onalnormalizedr at
io(INR)hasbeen
therapeutic f or t he past 3 mont hs. Whi le in hospi tal, he under goes a
transesophagealechocar di
ogr am andcar dioversion,andisl oadedonami odarone.
Ondi schar ge,hecont inuesonmet opr olol12. 5mgor all
yt wi cedaily,l
isinopri
l10
mgdai l
y,aldact one25mgdai l
y,andami odarone200mgdai ly.Whi choft he
foll
owi ngistheappr opriat
eant icoagul ati
onr egimenondi schar ge?

a.Api
xaban5mgor all
ytwicedail
y.
b.Warf
arin5mgor all
yeverynightatbedti
me.
c.Warf
arin7.5mgor al
lyeveryni
ghtatbedtime.
d.Ri
varaoxaban15mgor al
lydai
ly.
e.Warf
arin10mgor all
yeverynightatbedti
me.
Answer

B.War
far
in5mgor
all
yever
yni
ghtatbedt
ime.

Ami odar onei nhibitsCYP2C9,whi chisi mpor t


anti nt hemet abol ism ofwar f
arin.
Ther efore, ami odar one can i ncr ease t he l evel of war farin and l ead t o
supr atherapeut i
cI NRs.Thedoseofwar f
arinshoul dbeadj ustedt oaccountf orthe
inter acti
onwi thami odar one,andt hesuggest edr educt i
oni s25- 33%.I NRsmustbe
closel ymoni t
oredunt ilt hepat i
enti sonast ablewar f
arindosei ntheset ti
ngof
ami odar one.Leavi ngt hedoseofwar f
ari
nt hesame, orincreasingt hewar far
indose,
woul d notbeappr opr iatebecausebot h mi ghtl ead to asupr atherapeuti
cI NR.
Because t he pat i
ent has a mechani calmi t
ralval ve,nei ther api xaban nor
rivar oxabanar eanappr ovedant icoagulationr egimen.TheRE- ALI GN( Dabigatran
ver susWar fari
ni nPat ientswi thMechani calHear tVal ves)st udy,whi chtreated
pat ients wi th mechani calval ves wi th dabi gatran,showed i ncr eased rates of
thromboembol ic as wel las bl eeding compl icat i
ons compar ed wi th war f
arin.
Ther efore,thenovelor alant i
coagul antshavenotbeenappr ovedbyt heFoodand
Dr ugAdmi nistr
ationf orusei nt heset ti
ngofmechani calvalves.

29.
A56- year-ol
dmanwi thahi stor
yofhypertension,diabet
es,andparoxysmalatri
al
fibr
il
lation(AF)wasiniti
all
yseeni nyourcli
nic1yearagof ormanagementofhi sAF.
Hehadj ustbeendi schargedf r
om thehospi talforhisAF.Wor kupatt hatti
me
incl
uded a nor malD- di
mer ,a nor malt hryoi
d-sti
mulati
ng hormone,and an
echocar diogr
am demonst rati
nganej ect
ionfracti
onof55%,al ef
tatri
aldiameterof
4.5cm, andventr
icul
arwal lthi
cknessof1.1cm.  

Hewasst art
edonapi xabanandmet oprol
olandwasdi scharged.Att hatti
meyou
discussedopt i
ons,i
ncludingarat
e-contr
olstrategy,anantiar
rhythmicdrug-based
rhythm-controlst
rat
egy, andacat
heterablat
ion-basedrhythm- cont
rolstrat
egy.He
electedtotr
yanant i
arrhythmi
cdrugandwasst art
edondr onedarone. 

Her et
urnstoyourof
fi
ce6mont hslat
erforf
oll
ow-upandi
sfoundtobebacki
nAF.
Inthei nt
eri
m,her eport
st hathewasadmi ttedt
oal ocalhospi
talwi
thl
ower
extr
emityedemaandwast reatedforcongeti
vehear
tfai
lur
e.Hewast ol
datthat
ti
met hathi
sheartwas"outofrhythm."
Whi
choft
hef
oll
owi
ngi
sthebestnextst
ep?

a.Disconti
nuemet oprol
ol.
b.Disconti
nuedr onedar
one.
c.Addf l
ecaini
de.
d.Addr anol
azine.
e.Adddi l
ti
azem.

Answer

B.Di
scont
inuedr
onedar
one.

Based on t he PALLAS ( PermanentAt r


ialfi
briLLAti
on Outcome St udy Using
Dronedar oneonTopofSt andar dTher
apy)andATHENA( TheEff
ectofDronedarone
on Car diovascular Events i n Atri
al Fibri
ll
ati
on) st udi
es, dronedarone is
contraindicatedinpat i
entswi t
hNew Yor kHear tAssociati
on( NYHA)classIII-
IV
sympt oms,wi t
hcl assIIsympt omsifther
ei sar ecentdecompensation,orthose
whohaveper sist
entAF.Ther efor
e,t
hebestopt i
oni nthi
spat i
entistodiscont
inue
dronedar one. 

Stoppingmet opr
ololmightleadt or ecur
r entrapidventr i
cul
arresponseandwor sen
bloodpr essurecontrol
.Whilehemayneedf urtherrat
e- cont
rolagentsl
ikedil
ti
azem,
thebestopt ionhereistodiscontinuet hedr onedaroneandavoi dadverseoutcomes.
Whi l
et hecombi nat
ionofr anolazineanddr onedaronehavel ookedpromisingin
cli
nicaltri
als,thecombinationremai nsinvestigati
onal.Theaddi t
ionoffl
ecai
nideis
l
ikelytobepr oar
rhyt
hmic,andt herefore,isnotr ecommended.

30.
A 62- year-oldwomanwi t
hhyper t
ensionandmi ldobesitypresent
st ocl i
nicfor
evaluationofsever almont hshist
oryofi nt
ermit
tentepisodesofirregul
arr apid
heartbeat.Epi sodesaretri
ggeredbyal coholandexer ci
se,butareshort-l
ivedand
notassoci atedwi t
hnear-syncopeorsyncope.
Whichoft hef oll
owingfactorsincr
easest heriskthatthesesymptomsar eduet o
atr
ialfibri
ll
ation(AF)?

a.Lowbi rthweight.
b.Blackr ace.
c.Femal esex.
d.Increasedhei ght
.

Answer
D.I
ncr
easedHei
ght

Ar ecentlydevelopedr i
skscoref orAFi nt heFr amingham Hear tStudyi dent i
fi
edt he
clinicalf
actorsofage, malesex, elevatedbodymassi ndex,systoli
cbl oodpr essur e,
treatmentf orhyper tensi
on,pr olonged PR i nter
val,cli
nicall
ysi gnifi
cantcar diac
mur mur,and hear tfail
ureto beassoci ated withincidentAF.I nterestingl
y,t he
epi demiologyofAFi snotal waysconsi stentwi thothercar diovasculardi seases;
despi teani ncr
easedbur denofr iskf actors,blacksareatl essr i
skt hanwhi tesfor
devel opi
ngAF.Par adoxicall
y,lowbi rthwei ghtandshor tstature,knownr iskfactors
forcar di
ovasculardiseaseandmor t
alit
y,areinverselyassociatedwi t
hl aterlif
eAF.

31.
Inpati
ent
swithreducedl
eftvent
ri
cul
arej
ect
ionfr
acti
ons(LVEF)<40%,
whichoft
he
fol
lowi
ngbestdescri
bescl
ini
calout
comesassoci
atedwi
ththeuseofsotal
ol?

a.Adecreasedriskofmor t
ali
ty.
b.Anincreasedriskofmor tal
ity.
c.Nochangei nmor t al
it
y.
d.Superi
ort o ami odar one forsuppr ession ofventr
icul
artachycar
diai
n
pat
ient
swi thimpl antablecar di
over
ter-defi
bri
ll
ator
s(I
CDs).

Answer

A.Adecr
easedr
iskofmor
tal
it
y

Sotalolwasassoci atedwi thincreasedmor talit


yintheSWORD( Survivalwi t
hOr alD
-Sotalol)trial
.The t ri
alwas st opped owi ng to excess mor tal
ityi nt he group
randomi zedt osotal oltherapy.Thi strialonl yincl
udedpat ientswi thr educedEFs.
TheOPTI C( Opti
malPhar macol ogicalTher apyi nCardi
overt
er -Def i
bril
latorPatient s)
tri
aldemonst r
atedt hesuper ior
ityofami odaroneplusbet a-
bl ockersoversot alolf or
thesuppr essionofI CD shocksi npat ientswi thICDs.Tobeenr oll
edi nOPTI C,
patientshadt odemonst rat
eanLVEF<40% aswel l
.Consequent l
y,t hecor r ect
answeri st hatsotalolisassoci atedwi thani ncreasedriskofmor tali
tyinpat ient s
withdi minishedLVEFwi thoutdef ibr
il
lators,andi slesseffectivet hanami odar one
plusbet a-blockersf orthesuppr essi
onofappr opri
ateICDshocksi npat i
entswi th
reducedLVEF.

32.
A 71-
year
-ol
d woman wi
th an i
schemi
ccar
diomyopat
hyand per
manentat
ri
al
fi
bri
llati
on( AF)underwentbivent
ri
culari
mpl
antablecardiover
ter
-defi
bri
ll
ator(I
CD)
i
mpl antati
on 6 mont hs pri
or.She has New York Hear tAssociat
ion class I
II
sympt omsandcont i
nuest ocomplai
nofveryli
mi t
edf uncti
onalcapaci
ty,withno
changesi nceresynchroni
zati
onther
apywasini
ti
ated. 

Shei scur r
entl
ytakingcarvedi
lol25mgbi d,li
sinopr
il10mgqd,f ur
osemide40mg
bid,atorvastat
in40mgqd,aspi r
in81mgqd,andwar f
ari
n5mgqd.Hervi talsi
gns
i
nt heof f
icetodayarebloodpressure90/60mm Hg, heartrat
e111bpm, respir
ati
on
rate20,andoxygensat urati
on94% r oom air
.Shei safebri
le.Herexami nati
onis
signi
ficantforbibasil
arral
es,irr
egularheart
beat,and1+bi l
aterallowerextremit
y
edema.  

Int
err
ogati
onofherbi vent
ri
cul
arICDrevealsgoodpacingandsensi
ngthreshol
ds.
The bat
terystatus and l
ead i
mpedances are adequat
e.Ther
e have been t
wo
epi
sodesofnonsust ai
nedventr
icul
artachycar
dia,upto15beats,butnoshocks
havebeenrequi
red.Sheis25%pacedoverthepast3mont hs.

I
nadditiont
ot r
eat
mentofherhear
tfai
lur
e,whi
choft
hef
oll
owi
ngshoul
dbet
he
nextst
epinhermanagement
?

a.I
ncreasecar vedi
loldose.
b.Addsotalol.
c.Atr
iovent
ricularnodalablat
ion.
d.Refer
ralf
orhear ttranspl
anteval
uat
ion.

Answer

C.At
ri
ovent
ri
cul
arnodalabl
ati
on

The benef i
t of cardiac resynchr onizat
ion t herapy is hi ghest wi t
h maximal
biventricul
arpacing.Thispat i
entispacedonl y25%oft het i
meduet oAFwi thrapi
d
vent r
icularresponse.Restorati
onofsi nusr hythm isunlikel
yt obesuccessf ulina
patientwi t
hper manentAF.Sot al
oliscont raindicat
edi npat i
entswi t
hhear tf
ail
ure.
Herbl oodpr essureist
enuous, soincr easi
ngherbet a-blockerisnotr ecommended.
Transpl antevaluati
onpriort outil
izingavai lableguideli
ne-directedtherapyisnot
indicated.

33.
A25-year-
oldwomanpr esent
stot
heemergencydepart
mentaft
erafai
nti
ngspel
l
wit
nessedbyaf r
iend.Shewasseentosuddenl
ycollapseandappear
edtostop
breat
hingforabout30seconds, af
terwhi chshestart
edt
oregainconsciousness.A
si
mi l
areventoccurred2mont hsagoaf tersheracedupthreefli
ght
sofst air
s.She
hasnosi gnif
icantpastmedicalhi stor
yandheronl ymedicati
onisazithromycin,
whichwasst arted3daysagof oracutebr onchi
ti
s.Atpr
esent,shefeel
st i
red,wit
h
someshor t
nessofbr eat
h.Intheemer gencydepar t
ment
,shelosesconsciousness
andar hyt
hm stripi
srecor
ded( Figure1).

Fi
gur
e1.

Whichofthef
oll
owi
ngbestexpl
ainst
hemechani
sm f
ort
hei
nit
iat
ionoft
his
heartr
hyt
hm?

a.Tri
ggeredact
ivi
tyduetoearlyafter
depol
arizat
ions.
b.Re-ent
ryduet oenhancedpot assi
um eff
luxdur i
ngphaseII
Ioftheact
ion
potent
ial
.
c.Re-ent
ryduetolossoftheepicardi
alact
ion-potent
ialdomei
nphaseI
I.
d.Re-ent
ryduetomyocardialscarandunidir
ecti
onalblock.

Answer

A.Tr
igger
edact
ivi
tyduet
oaf
ter
depol
ari
zat
ions.
Thispat i
enthasapr olongedQTi nter valwi t
hpol ymorphicventriculartachycar dia
(torsades de poi ntes) qui ckl
y degener ati
ng i nt
o ventri
cular f i
bril
lati
on.The
prolongedQTi ntervalt riggersear l
yaf terdepolar
izati
ons.Azit
hromyci nisknownt o
i
ncr easetheQTi ntervalandmaypr edi sposeapat ientwithlongQTsyndr omet o
thisevent.Ayoungwomanwi t
houtevi denceofapr i
ormyocar dialinfarcti
onisnot
l
ikelyt o have scar -
r elated ventricul ar tachycardia. Catecholami ne-sensitive
rhythmsar emedi atedt hroughdel ayedaf t
erdepolari
zati
onsi not herwi seheal thy
ti
ssueandt ypical
lyar er elatedt oexer cisewi t
houtabnor mali
ti
esoft heQTi nterval.

34.
Whichofthefoll
owingbestdescri
besoutcomesassociatedwit
hami odar
oneuse
comparedwithli
docai
neusedur i
ngout-
of-hospi
talar
restexhi
bit
ingcardi
over
sion
ref
ract
oryvent
ri
cularf
ibr
il
lat
ion(
VF)?

a.Sur
vivalt
ohospi
taldi
schargewashi
gherinthel
idocaine-tr
eatedgr oup.
b.Sur
vivalt
ohospi
taladmissi
onwashigheri
ntheami odarone-tr
eatedgr oup.
c.Sur
vivalt
ohospi
taladmissi
onwashigheri
ntheli
docaine-treatedgroup.
d.Sur
vivalt
ohospi
taldi
schargewashi
gherintheamiodarone- t
reatedgroup.

Answer

B.Sur
vivalt
ohospi
taladmi
ssi
onwashi
gheri
ntheami
odar
one-
treat
edgr
oup.

TheALI VE (Ami odar oneversusLi docai


nei n PrehospitalVent ri
cularFi bril
lati
on
Evaluation)t
rialcompar edtheuseofami odaronever suslidocaineforthet reatment
ofcar di
oversionr efr
act or
yVFi nthefiel
d.Ast at
ist
icallysignifi
cantimpr ovementi n
survivaltohospi taladmi ssi
onwasobser vedi nt heami odarone-treatedgr oup.
Therewasnodi ffer
ence,however ,insurvivaltohospi taldischargebet weent he
amiodar one-and l idocaine-
treat
ed groups int hose pat ients who sur vived t o
hospi t
aladmission.

35.
A 50- year-oldmanwi t
hahi storyofpar oxysmalat ri
alfibril
lat
ionandcor onar y
arterydiseaseisini
ti
atedondof eti
li
de.Hismedi cat
ionsi ncludeaspiri
n81mgdai ly,
li
sinopril10mgdai l
y,simvastatin40mgdai ly,andmet opr ol
ol50mgt wicedaily.
Hishear trateis55bpm, bloodpr essureis123/ 75mm Hg, andt heremainderofthe
exami nation is unremar kable. Hi s electrocardiogram ( ECG) shows si nus
bradycar diaat54bpm wi t
hanonspeci f
icint r
aventricularconductiondelay,QRS
dur at
ionof105msec, andQTi nt er
valof430msec.Labor at
or ypanelshowsnor mal
renalfunct i
on,andserum potassium levelof5.6mEq/ L.
Whichofthefol
lowingef
fect
sofdof
eti
li
demayi
ncr
easet
her
iskf
ort
orsadesde
poi
ntesi
nthi
spat i
ent?

a.Decreaseddrugmet aboli
sm inthepresenceofsimvast atin.
b.Decreasedpotassium ef
fluxexacerbatedbyhyper kalemi a.
c.I
ncreasedpotassium eff
luxexacerbatedbyhyper kalemi a.
d.Usedependencet hatisexacerbatedbybr adycar
di a.
e.Reverse-usedependencet hati
sexacer batedbybr adycar di
a.

Answer

E.Rever
se-
usedependencet
hati
sexacer
bat
edbybr
adycar
dia.

Dofetil
ideexhi bi
tsreverse-
usedependence,wit
hani ncreasedef fectatsl
ow heart
rat
es.Br adycardi
ai nt hi
spatientmayexacerbatet heQT- prol
ongingeff
ectsof
dofeti
li
de.Dof etil
idedoesnothaveadr ugint
eract
ionwi thsi mvastati
n.Dof
eti
li
de
has many dr ug inter
acti
ons,especi
allythose thatpr olong t he QTc,cause
hypokal emia, or cause hypomagnesemi a. Verapami l also al t
ers dof
eti
li
de
met abolism andincreasestoxi
cit
y. 

36.
A 64- year-ol
d man i sreferred tot he cl
ini
cf orevaluation ofan abnormal
elect
rocardiogram (ECG)( Figure1) .Hispastmedi calhistoryissignif
icantf
or
hypertension f or which he i s tr
eated with li
sinopri
l. He denies cardi
ac
sympt omat ol
ogy.Hisexami nationi
sunr emarkabl
e.Hei sacti
ve,worksful
l-t
imeas
acarsal esman, andplaysgolfregularl
y.
Whi
choft
hef
oll
owi
ngi
sthenextbestst
epf
ort
hispat
ient
?

a.Cardi
acmagnet i
cresonancei
magi
ng.
b.Echocar
diogram.
c.Carot
idsi
nusmassage.
d.Bloodcul
tures.
e.Pacemaker.

Answer

C.Car
oti
dsi
nusmassage.

TheECGi sconsi stentwi thsi nusr hythm wi th2: 1atriovent ri


cular( AV)block.The
diff
erent
iali sMobi t
zt ype1( Wenckebach)second- degr eeAVbl ockver susMobi t
z
type2second- degreeAVbl ock.Noni nvasi vevagalandsympat het i
cmaneuver scan
helpt odistinguishonef rom t heot herbecauseMobi t
zt ype1bl ocki susual lyAV
nodal,andMobi tztype2bl ocki susual l
yi nt heHi s-Pur kinjesyst em.I napat i
ent
withAVnodalbl ock,car otidsi nusmassagewi llslowthesi nusr ateandwor senAV
conduction,whi ch wi l
lwor sen t he AV bl ock.The sl owi ng ofsi nus rate and
depressi
onofAVconduct ionwi l
lhaveapr otectiveeffectont hedi stalconduct ion
system,soconduct ioni nt heHi s- Purkinjemayi mpr ove.Exer ci
se( sympat het i
c
stimulat
ion)wi l
limpr oveAVnodalconduct i
onandwor senHi s- Purkinjeconduct i
on.
HisECG( Figure1)i ssuggest iveofMobi tzt ype1second- degreebl ockbasedont he
l
ongPRi nter valandnar rowQRSi nterval.Al thought hedet ectionofst ruct
ur alhear t
diseasemayhel pgui depat ientt herapy,iti snott henextbestdi agnost icmaneuver
to assesst hedegr eeand l evelofAV bl ock.Ther ei sno cl inicalevi denceof
endocardi t
is.Ifthesi teofbl ocki swi thi
nt heAVnodei nanasympt omat i
cpat ient,a
pacemakeri snoti ndicated.Car di
acmagnet i
cr esonancei magi ngi snoti ndicated.

37.
A56- year-oldmal ecomest oseeyouf orevaluat i
onofpal pi
tati
ons,whi chhave
beenbot hersomenow f oratl east2mont hs.Pr evi
ousl y,hewasdi agnosedwi t
h
severemi t
ralvalver egurgi
tat
ion.Atsur gery,hismi tralvalvewasnott houghtt
obe
repai
rabl e,andt herefor
e,heunder wentmi t
ralvalver eplacementwi t
hamechani cal
prosthesisanddi dwel lunti
lr ecent
ly.Ot hersympt omsi ncludedecr easedeffort
tol
erance, shortnessofbr eathwi t
hexertion,andoccasi onallightheadedness. 
Exami nationreveal
sawel l
-nouri
shedmaleinnoacutedistress.Hisbloodpressur e
i
s110/ 68mm Hg,hear trateis140bpm,andj ugularvenouspr essureisvisible.
Hear tsoundsar enormal,wit
hsoftsyst
oli
cmur murheardbestatt heapexwi thout
radiation and no diast
oli
c murmur.Lungs have bibasi
lari nspirat
ory cr
ackl es.
Abdomen i s softand nontender.Thereis no organomegal y.You obt ain an
electrocardi
ogram (Fi
gure1).

Fi
gur
e1.

Whi
choft
hef
oll
owi
ngi
sthemostl
ikel
ydi
agnosi
s?
a.At
ri
ovent
ri
cul
ar(
AV)nodalr
e-ent
rantt
achycar
dia.

b.Si
nust
achycar
dia.

c.At
ypi
calf
lut
ter
.

d.AVr
eci
procat
ingt
achycar
dia.

Answer

C.At
ypi
calf
lut
ter

Thet r
acingdemonst ratesatri
alflutt
erwi t
h2: 1conduct i
on.Thecircuitshowni s
mostlikel
yt hel
eftatr
ium, aresul
tofthemi tralval
vesurgerywit
hleftatri
alscar
ring
thatprovi
desasubst rat
ef orr
e-entrywit
hinthel ef
tat
rium.Themor phologyoftheP
wave i s nottypicalofsi nus tachycardi
a.The 2: 1 conducti
on r ul
es outAV
reci
procating t
achycardia,and makes AV nodalr e-entr
anttachycardia hi
ghly
unli
kely.

38.
A35- year -oldother
wisehealt
hymanpr esent
st otheemer gencydepartmentat5: 00
a.m.af t
erawakeni ng 2 hoursago wi t
h a sensation ofa rapid heartrate.An
electrocar diogram (
ECG)hasbeenper formedshowi ngat r
ialf
ibr
il
lat
ion(AF)atar ate
of94bpm.Hehasnopastmedi calhistoryanddoesnotconsumeal cohol.Thereis
nof amilyhi st
oryofatrialf
ibri
ll
ati
onexcepti nr elati
vesovert heageof75.He
typicall
yr uns5- 10mi l
esaday.Onexami nati
on,hi sheartrat
ei s96bpm wi tha
blood pr essure of110/ 60 mm Hg,and t he remai nderoft he examination is
unremar kabl e.

Whichofthefol
lowi
ngel
ect
rophysi
ologi
cmechani
smsbestexpl
ainst
hispat
ient
's
i
nit
iat
ionofAF?

a.Sympatheti
cactivati
onoft heI
KATPchannel.
b.Phosphoryl
ati
onoft heICachannelatslowhear
trat
es.
c.Parasympathet
icactivat
ionoftheIKACHchannel
.
d.Parasympathet
icactivat
ionoftheIfchannel
.
e.EnhancedITOact i
vit
ydur i
ngsleep.

Answer

C.Par
asympat
het
icact
ivat
ionoft
heI
KACHchannel
.
AFi n young highlyconditi
oned indivi
dualswi t
h ventr
icularr ates<100 bpm i s
consistent wit
h par asympatheti
call
y medi ated AF. Par asympathetic (vagal)
sti
mul ati
onacti
vatesmuscar i
nicreceptor
s, whicharecoupl edt otheIKACHchannel
byaGpr otei
n.Activat
ionoftheIKACHchannell eadstosl owi ngofthesinusr at
e, a
decreasei natr
ioventr
icul
arnodalconduct i
on,andi mpor tant l
y,adecreasei nt he
durati
onoft heactionpotenti
alintheat ri
um.Thi slastef f
ecti sthoughttoexpl ain
theincreasedincidenceofAFi nt heset t
ingofahei ghtenedvagalt one,suchas
duri
ngsl eep.

39.
An18- year
-ol
dmanpr esentsf
oraspor
tspar
ti
cipat
ionphysicalpr
iort
ostarti
ng
col
legeasami ddl
e-di
stancer
unner
.Heisascholarshi
pathlet
eataDi vi
sionI
schoolandhasnevernot i
cedanyexerci
sel i
mit
ations.Her eport
spalpi
tati
onsover
thepastsever alyearsthatoccurevery1- 2mont hs,lastfrom 10secondst o5
minutes,andspont aneousl
yremit
.Thepal pi
tat
ionsar eassociatedwit
hdi zzi
ness,
buthehasnothadsyncope.Onexami nation,agrade2/ 6systoli
cmur murisheard
atthel ef
tlowersternalborder
.At r
ansthoraci
cechocar diogram isobtai
nedand
showni nVideos

2.(ht
tp:
//medi
a.mycr
owdwisdom.com.s3.
amazonaws.
com/acc/
quest
ions/
asset
_
arr
/cd8293f
8-2a8f
-47f
8-bc05-
96d8ec7b3584.
mp4)and

3.(ht
tp:
//media.
mycrowdwisdom.com.s3.
amazonaws.com/
acc/
quest
ions/
asset
_
arr
/2edbd3b1-0908-
4429-b825-8d95e9ceee66.
mp4).

Basedontheechocar
diogram f
indi
ngs,whi
choft
hef
oll
owi
ngar
rhyt
hmi
asi
sthemost
l
ikel
ycauseofhi
ssympt oms?

a.At
ri
alf
ibr
il
lat
ion.

b.Si
nust
achycar
dia.

c.Vent
ri
cul
art
achycar
dia.

d.Mul
ti
focalat
ri
alt
achycar
dia.

e.At
ri
ovent
ri
cul
arr
e-ent
ryt
achycar
dia(
AVRT)
.

Answer

E.At
ri
ovent
ri
cul
arr
e-ent
ryt
achycar
dia(
AVRT)
.
Inapat i
entwi thEbst ein'sanomal yandpal pitati
onsconsistentwi that achycar dia,
themostl ikelycausei sanAVRTr elatedt oabypasst r
actthatof tenmani f
est sas
pre-excitationonar estingelectr
ocar diogram.Al thoughat r
ialarrhythmiassuchas
atri
alflut
terorat r i
alfi
bril
lat
ionoccur ,thoser hythmst endtobeseeni npat i
ent swi t
h
mor esever edi seaseandhear tfail
ure.Sever ediseasewoul dnotbeexpect edi na
pati
entwi t
hsuf fi
cientcar di
opulmonar yf it
nesst obeaschol arshiptrackat hlete,and
the echocar diogram shows r elati
vel y mi l
d di sease.The epi sodi
c nat urei s
i
nconsi stentwi thsinust achycar
dia.Mul ti
focalatri
altachycardiaismor eof tenseen
i
npat ientswi t
hunder l
yinglungdi sease.Thought heotherf i
ndingsar ecommonl y
foundi npat i
entswi thEbstein'
sanomal y,nonewoul dbeexpect edt ober elatedto
sympt omat icpal pit
ati
ons.

40.
A70- year-
oldmanpr esent
swi thchronicweakness,exer ci
seintolerance,fat
igue,
andnearsyncope.Her eportsthatasayoungmanhef aintedeasilyduringblood
drawsandmedi calprocedures,butthatheseemedt ogrowoutofi t.Overt hepast5
or6year s,however,hehasi ncreasingl
yfel
tli
ght headedandmi ldlydiaphoretic
occasional
lydur
ingor di
naryactivi
ti
esandof t
enfeelsast houghhei sgoingt opass
outunlesshesitsdown.Hi ssympt omsseem wor sedur inghotweat herandaf ter
eati
ng. 

Hisphysi
calexaminati
onisunremarkableexceptthathi
sbloodpressureis150/85
mm Hgwhi l
esupine,130/80mm Hgwhi lesi
t t
ing,
and70/40mm Hgwhi l
estandi
ng,
atwhichti
mehebecomesl ight
headedandnearsyncopal.Hisheartrat
eis80bpm
supi
ne,90bpm sitti
ng,and100bpm st anding.Hehasar esti
ngtremorand" cog-
wheel
ing.

Elect
rocardi
ogram,echocar
diogram,and st
resst
estresul
tsarenor
mal
,asar
e
resul
tsfrom t
hyroi
dandadrenalstudi
esandacompl
etebl
oodcount
.

I
nadditi
ont oavoidanceandeducationaboutcount
erpressuremaneuver
s,whi
ch
ofthefol
lowingist hemostappropriat
etherapytoreducethefr
equencyofthi
s
pat
ient
’sl
ightheadednessandnearsyncope?

a.Droxidopa.
b.Cl
oni di
ne.
c.Met oprolol.
d.Disopyrami de.
e.Salttablets.
Answer

A.Dr
oxi
dopa

Thi
spati
enthaswhatsoundslikeahi st
oryofneur
ocar
diogeni
csyncope,whi
ch
maybeassoci
atedwi
thor
thost
aticst
ress.

However,now t hesymptomshavechangedandhehasaphysi calexaminati


on
consist
entwithneurogeni
corthost
ati
chypotension,andthi
sisalsoconsi
stentwit
h
thefactthathehasevi dencepotenti
all
yforPar ki
nson'sdi
seasewhichcan,and
oft
endoes, occurwi
thneurogenicor
thostat
ichypotensi
on.

Thepr oblem canbecompl extomanage.Ther easoni smanysuchpat ientshave


supine hyper tensi on and or t
hostatic sympt omat i
c hypot ension.Tr eati
ng the
ort
hostat i
chypot ensionmayexacer batet hehyper tension.Onedr ugthatmaybe
usefulisdr oxidopa.Thi sdr ugisi
ndi catedf orneurogenicor t
hostatichypotensi
on.
Otheropt i
onsi ncludepyridosti
gmine,mi dodri
ne,andf ludrocort
isone.Thesedr ugs
mayexacer batehyper t
ension.I
tisal sowor thwhil
et ohavehi m puttheheadofhi s
bedupabout30°t oof f
setthesupineandeveni nghyper t
ensionwhent her ei
soften
fl
uidr etention.The pat i
entshoul d al so specifi
cally avoid eating very sweet
carbohydrat e-l
adenf oodsi nthemor ning.

When symptoms int


erfere wit
h qual
it
y oflif
e despi
te adequat
e hydrat
ion and
i
sometri
cexerci
sesatthet imeofimpendingsyncopetodel ayormiti
gatelossof
consci
ousness,
medicaltherapymaybeconsidered.
 

Mi dodri
nehasbeenshownt oreducesympt omsi nsmallstudi
es,par
ti
cularl
yin
pat i
entswit
hpredomi nant
lyhypotensivesymptoms,al t
houghconfir
matorystudi
es
arel acki
ng.Therei s some evidence thatclonidine can hel
p,butitcan also
exacerbatetheproblem ofhypotension.Andanel der
lymanshoul dnotbet aki
ng
disopyramidebecauseitmayexacer bateuri
naryretent
ion.

41.
A 56-year-old woman i sr ef
err
ed f oreval
uation ofi nt
ermit
tentdi
zziness wit
h
occasi
onalnear -syncope.Episodesoccur1- 2t i
mesamont h,donothaveany
speci
fi
ct ri
ggers,andl ast<1mi nute.Shehasnothadasyncopalevent ,buthas
comecl ose.Shehasnoot hercardiacsympt omatologyandi sother
wisehealthy.
Herfami l
yhistoryispositi
vef orhypertensi
onandat r
ialfi
bri
ll
ati
oninhermot her,
andhergr andmotherhadapacemaker .

Whi
choft
hef
oll
owi
ngi
sthenextbestdi
agnost
ict
estf
ort
hispat
ient
?
a.Tiltt
ablet est.
b.Pat i
ent-activatedeventmoni t
or.
c.Electrophysiologi cst
udy(EPS).
d.A24- hourcont inuousel
ectrocar
diogr
am (
ECG)(
Hol
ter
).
e.Implant edloopr ecor
der
.

Answer

B.Pat
ientact
ivat
edmoni
tor
.

Apat i
ent -
activatedeventmoni torcanr ecordECGt r
aci ngsimmedi atelypr iortoand
duri
ng sympt oms,al l
owi ng ident i
fi
cat i
on ofpossi ble arrhyt
hmi as.A 24- hour
conti
nuousECG( Holt
er)isnotcor r
ect:Epi sodesarelikel ytobet ooinfrequentt obe
capturedona24- hourHol t
er.Ani mplant edloopr ecor dermaybeconsi deredwhen
noninvasivedi agnostict esti
ngi sunr eveal i
ng,ifsympt om frequencyi st ool ow to
captureoneventmoni tor,ori fthepat i
ent’ssympt om i ssyncope.Al thoughEPS
all
owsdi rectassessmentoft hecar diacconduct i
on syst em,i tdoesnotal l
ow
corr
elationofsympt omswi t
hanar r hythmi a.Til
tt ablet esti
ngcaneval uatefor
ort
host ati
csympt omsorpost uralor thostatictachycar diasyndr ome,buti snot
appropr i
ateforidentif
yingot herarrhythmi as.

42.
A 45-year-ol
d man sees you i
n consul
tati
on int
he of
fi
ce r
egarding f
requent
pal
pit
ations.Hedescr
ibespoundi
ngheartbeats,
1or2perhourt
hroughouttheday,
whi chincreaseduringtimesofst r
ess.Anechocardiogram wasnormal.Anexercise
stresstesthadt obest oppedduet oincr
easingventri
cul
arectopy,i
ncludi
ngaf ive
beatr unofvent ri
culart achycardiathatterminat
edassoonasexer cisestopped;
therewer enoot herelectrocardi
ographic(ECG)changesdur i
ngthest
resstest.A12
-leadECGi sshowni nFigur e1.

Whi
choft
hef
oll
owi
ngi
sthemostl
ikel
yor
igi
noft
hepr
emat
urevent
ri
cul
arcont
ract
ions
(
PVCs)
.

a.RVapi calmorphol
ogy.
b.Bodyoft heLV.
c.Rightventri
cul
aroutfl
owtr
act(
RVOT)or
igi
n.
d.Leftventr
icul
arout
flowtr
act(
LVOT)
.

Answer

D.Lef
tvent
ri
cul
arout
fl
owt
ract(
LVOT)
.

Thispat i
ent'spal pitati
onsar eduet oi di
opathicvent ri
cularout flow t ractPVCs.
These ar e commonl y associated with nonsust ai
ned and sust ained vent r
icular
arrhythmi asdur ing st ressordur ing 24-hourECG ( Holter
)moni toring.Possi ble
originsoft hisarrhythmi aar ei
nt heRVOT( morecommon)orLVOT( l
esscommon) .
ThePVCsi ntheECG exampl ehaveal eftbundle,inferi
or-axismor phol ogy.The
precor di
alR- wavet ransi ti
on(total
lyposi t
iveQRScompl exesi nleadV3)i sear l
ier
than i stypicall
yseen i n RVOT vent r
iculartachycardia.The LVOT ( sinuses of
Valsal va specif
ically)i spost eri
ort ot heRVOT,whi ch resultsi n mor eposi tive
voltageandear li
ert ransi ti
onint heprecordialleads.Wheni diopathicout f
lowt ract
PVCsar emor efrequentt han10%oft otalheartbeats,theymaybeassoci atedwi th
devel opmentofcar diomyopat hy.Bet a-blockers ornondi hydropyridine cal ci
um
channelbl ockersar eof teneffecti
ve,wi t
habl ati
onr eservedf orpat ientswhoar e
refractoryt o medi calt her apyorwi t
h car di
omyopat hyduet ot hehi gh- density
arrhythmi a. 

43.
A55- year-oldmanwi th3- yearhi
storyofparoxysmalatrialf
ibril
lat
ion( AF)presents
to cli
nicf or evaluati
on.He has weekl y episodes ofAF t hat pr oduce an
uncomf ort
ablesensat ionofhear traci
ngandshor t
nessofbr eath.Hispastmedi cal
histor
yi ncludes cor onary arter
y disease,hyper t
ension,t ype 2 di abetes,and
hyperli
pidemia.Hi scur r
entmedi cati
onsar ewar f
ari
n5mg/ day,met oprolol50mg
twicedai l
y,hydr ochlorothiazi
de25 mg/ day,r amipri
l5 mg/ day,si mvastati
n 20
mg/day,andglybur
ide5mg/ day.Hisel ectr
ocardiogr
am showssi nusrhythm ata
rat
eof55bpm wi t
hacor rect
edQTi nter
valof440msec.Hi slabor
atorypanel
showsnor malel
ect
rolyt
esandr enalfunct i
onandani nt
ernat
ionalnor
malizedrati
o
of2.5.Youdeci
detoinit
iat
edofeti
li
det ocont r
olhisAF.

Whi
choft
hef
oll
owi
ngmedi
cat
ionchangesi
srequi
redwhenst
art
ingdof
eti
li
de?

a.Di
sconti
nuehydrochlor
othi
azi
de.
b.Decr
easewarfari
ndose.
c.Di
sconti
nuemet opr
olol
.
d.Di
sconti
nueglyburi
de.
e.Decr
easesimvastat
indose.

Answer

A.Di
scont
inuehydr
ochl
orot
hiazi
de.

Several dr ugs ar e contrai


ndicated wi t
h t he use of dof eti
li
de, includi
ng
hydrochlorothi
azide,tr
imethopri
m,ver apamil,ketoconazol
e,pr ochlorper
azine,and
megestrol.Dof et
il
ideisnotcontraindicat
edint hepresenceofcor onarydiseaseor
i
schemi a,nori nheartfai
lure.Itdoesnothaveani nt
eracti
onwi t
hwar f
arinor
si
mvast ati
n.Dof eti
li
dedoesnoti nteractwit
hgl yburi
de.Dofetil
idedoesnoti nteract
wit
hmet opr ol
ol,andindeed,beta- bl
ockersshoul dbecont inuedi npat i
entswi th
coronaryar t
erydisease.

44.
A50-
year
-ol
dmanunder
goesr
out
ineappendect
omy.Shor
tl
yfol
lowi
ngext
ubat
ion
i
nt her ecoveryroom,heisbrief
lyunresponsi ve,andt hetr
acingshowni nFi
gure1i s
retr
ievedf rom themonitor.Hepr esent
lyissl eepybutr esponsi
ve,acknowledging
abdomi nalpain.Review ofpr eoperati
ve eval uat
ion shows a nor malst ress
electr
ocar diogr
am 6mont hsagoandnopr iorhi stor
yofhear tdi
sease.Hereceived
adoseoff entanyl30minutesago.Thel astser um potassium was3.6mEq/dl.
Whi
choft
hef
oll
owi
ngshoul
dbedonepr
iorhospi
taldi
schar
ge?

a.I
mplantablecardioverter
- defi
bri
ll
atorpl
acement
.
b.Noadditi
onaleval uat
ioni srequi
red.
c.Chr
onicami odaronether apy.
d.Cat
heterablati
on.
e.Cor
onaryangi ography.

Answer

E.Cor
onar
yangi
ogr
aphy.

Thet
raci
ngshowssi
nusr
hyt
hm f
orf
ivebeat
s,f
oll
owedbyapr
emat
urevent
ri
cul
ar
contr acti
ont hatinitiatespol ymor phi cvent ri
culartachycar dia(VT)degener atingto
ventricularf i
bril
lation( VF)( Figure1) .Mar chingt heQRScompl exesf orwar di ntothe
arrhyt hmiadoesnotsuggestar tifact ,andt hepat ientwasbr ieflyunr esponsi ve,
consi stentwi t
hanepi sodeofhypot ension,wi t
hspont aneoust erminationoft he
arrhyt hmia, whichcanoccurwi thpol ymor phicVTandevenVF.Pol ymorphi cVTdue
to QT pr olongat ion i s a consi der ation,butt he QT i ntervali s notpr olonged.
Further mor e,theepi sodei snotpr ecededbyapauseort ransientsl owingofhear t
rate,asi susual lyseenwi thacqui redl ongQTsyndr ome.Themostl i
kelycausei s
myocar dialischemi a,whi chmaybet ransientandneedst obeeval uat ed.Thus,
optionAi scor rectandshoul dbei mmedi atelyconsidered, taki
ngi ntoconsi der at
ion
ri
sksi mposedbyt her ecentsur ger yandanyaddi tionalevi dencesuggest ingan
acutecor onar ysyndr ome.VFduet oanacut eischemi csyndr omeusual lydoesnot
war rantpl acementofani mpl antabl ecar dioverter
-defibril
lator
.Cat heterabl ationi s
rarelyper formedf orpol ymor phicVT,andt henonl yf orr ecurrentepi sodeswi t
h
premat urevent ricul arcont ract
ion t ri
gger sthatar enotduet o acut ei schemi a.
Althoughacut eint ravenousami odar onet herapycoul dbeconsi der ed,chr onicor al
therapyshoul dnotbei ni t
iatedatt hist i
me.

45.
A 52- year-ol
d woman wi th hypertension and paroxysmalat r
ialfibr
ill
ati
on (AF)
presents t o the emergency depar t
ment wi t
h acut e onset of pal pit
ati
ons,
li
ghtheadedness,and dyspnea t hatst ar
ted approximatel
y 16 hour s ago.An
echocar di
ogram 6 mont hs priorshowed nor mall eftventr
icularstructur
e and
functi
on.Hercur r
entmedicati
onsar eli
sinopri
l10mg/ dayandaspi ri
n81mg/ day.
Uponpr esent
ation,sheappearsdyspnei c.Herbloodpr essureis155/ 90mm Hg,
heartr ateis125bpm andi rregular,andwei ghtis70kg.Ther emainderoft he
physicalexami nati
onisnormal .Herel ectr
ocardi
ogram showsAFatar ateof118
bpm withouti
schemicchanges.Labor
atorypanelshowsnor malel
ect
rol
ytesand
renalandhepati
cfunct
ion.Dil
ti
azem 10mgi ntravenoushasbeenadminist
ered,
resul
ti
nginaheartr
ateof80bpm withabloodpressureof135/
85mm Hg.

Whichofthefol
lowingist
henextbestst
epi
nmanagementt
ophar
macol
ogi
cal
lyr
e
-est
abl
ishsinusrhythm?

a.Pr
opaf enone150mgbymout honce.
b.Amiodar one400mgbymout hnowandr epeatin8hours.
c.Di
goxin0. 25mgint
ravenousandrepeataf
ter6hours.
d.Di
lt
iazem 90mgbymout handrepeatin6hours.
e.Fl
ecainide300mgbymout honce.
Answer

E.Fl
ecai
nide300mgbymout
honce.

Cardi oversionofr ecent-onsetAFwi thoralmedi cationissomet imesreferredt oas


the" pill-
in-the-pocket "appr oach.Initi
alrat
e- contr
olmedi cationshoul dbepr ovided
(suchasabet a-blockerorcal ci
um bl ocker)t opr eventrapidconduct ionofat r
ial
fl
utter/ AF,andshoul dbef oll
owedbyadmi nistr
ationofasi ngl ebolusdoseofa
classIant iarrhythmi cdrug( f
lecainide200- 300mgorpr opafenone450- 600mg) .
Not et hatclassIant i
arrhyt
hmi cdrugsar econt rai
ndi cat
edinpat ientswithstr uctural
hear tdi sease orconduct i
on syst em di sease ( bundle branch bl ock).Pat i
ent s
receivingt hist herapyshoul dbemoni t
oredthef ir
stt imeitisused,assi deef f
ect s
i
ncl udi ng hypot ension can occur .Ami odaronecan beconsi dered,butt ypically
takesmuchl ongert oachi eveconver si
oncompar edwi t
hacl assIcagent .Di goxin
anddi lti
azem ar erate-controlagent s,butwillnotcausecar di
over si
on.

46.
A46- year-oldwomani sr ef
erredt oyourcli
nicforrecurrentsyncopalepisodes.The
episodesar espor adic,buthavei ncreasedinfrequencyi nthepast3mont hs.Most
occurwhi leshei sst anding,butcanoccurwi t
hl i
ttl
ewar ning.Shehassecondsof
dizzi
nesspr i
ort othef all,buthasnoot hersympt oms.Shei sotherwisei ngood
healthandi stakingnomedi cati
ons.Herbl oodpr essureint hecli
nicis125/ 85mm
Hg,and hear tr atei s 80 bpm i n si
nus rhythm.The car di
ac and r espir
ator
y
exami nat
ion isunr emar kable.Herechocar di
ogram showed no st ructuralheart
disease.A24- hourHol termoni torshowednosi gnif
icantar r
hythmias,butshewas
asympt omat i
c dur ing t he moni tori
ng. A tilttablet est was ar r
anged and
demonst r
at edther esult
sshowni nFigure1.
Accordi
ngtother
esul
tsoft
hist
est
,whi
choft
hef
oll
owi
ngmanagementst
epswoul
d
yourecommend.

a.Diagnost
icelect
rophysiol
ogyst udy.
b.Implant
ablelooprecorderi
nsertion.
c.Permanentpacemakeri mplantation.
d.Ini
ti
atebeta-
blocker.
e.Emphasizesaltandfluidi
ntake.

Answer

E.Emphasi
zesal
tandf
lui
dint
ake.

Thet il
tt abl
est udyshowsa mi xed vasodepressorand cardio-i
nhibit
oryvagal
response.Thedi agnosi
si sclear
,so adi agnosticelect
rophysiol
ogyst udyand
i
mpl antable loop recorder are not indi
cated. Pacemaker i nser
tion is not
recommended.Bet a-
channelantagoni
stshavenotbeenshownt obeef
ficaci
ousf
or
vasovagalsyncope.Pat
ientreassur
anceandeducation,emphasizi
ngf
luidandsal
t
intake,
arerecommended.

47.
A57-year
- oldmanpr esent
stoyourcli
nicwit
hcompl ai
ntsofshor t
nessofbr eath
whil
ewalkingl esst hanabl ock.Hehasapastmedi calhistorysignif
icantfor
cor
onaryarterydisease,wit
hananteri
orSTelevati
onmyocar dialinf
arcti
ont r
eated
wit
hdrug-eluti
ngstent6mont hsago,
hypert
ension,andhypercholester
olemia. 

Onphysi calexaminati
on,bloodpressureis118/72mm Hgandhear trat
eis38bpm.
Recentst udies inthe past6 mont hs incl
ude an echocardiogr
am wi t
hal eft
ventri
culareject
ionfracti
onof50%,andHol t
ermonit
or i
ngwi thsinusbradycardi
a
withanaver ageheartrateof40bpm andmi ni
mum heartr ateof30bpm whi l
e
sleeping.Hismedi cati
onsi ncl
udeaspi r
in81mgdai ly,clopidogr
el75mgdai l
y,
met oprol
ol25mgt wi
cedai l
y,at
orvastat
in40mgdai l
y,andlisinopri
l10mgdai l
y.

Whichofthef
oll
owi
ngi
sthemostappr
opr
iat
erecommendat
ionf
ort
hepat
ient
’s
symptoms?

a.Perform anexer ci
setreadmi l
ltest
.
b.I
mpl antadual -chamberpacemaker .
c.Perform el
ect
r ophysiologicstudy.
d.Disconti
nuemet oprolol.
e.Continuecurrentmedi cat
ions.

Answer

B.I
mpl
antadual
-chamberpacemaker
.

Basedoncur rentgui del


ines,thispat ientmeet saCl assIi ndi
cationforper manent
pacemaker .Per manentpacemakeri mpl antati
oni sindicatedf orsympt omat icsi
nus
bradycar di
at hatr esultsfrom r equireddr ugt herapyf ormedi calcondi t
ions.This
patientispresent i
ngwi thsympt omat i
csi nusbr adycardi athatisli
mi t
inghisact i
vit
y.
Althoughmet opr ol
olcoul dbet hepot entialcausef orhi ssympt omsduet othe
historyofanant eri
ormyocar dialinfarctionandhyper tension,thismedi cationisan
essentialcomponentofhi st r
eat ment .Becauset hepat i
entisexper iencingsinus
bradycar di
a,treatmenti nterventionisr equired,andnotdoi nganyt hi
ngisi ncorr
ect.
Nof urtherstudiesar enecessar yt oeval uat
et hepat i
ent’scondi t
ion. 
48.
A 45-year-
oldmanpr esentsf oreval uati
onofpal pi
tati
ons.Hehasf el
tar apid
heart
beatfort hepastsever alweeks.Hi sexerci
set ol
erancehasdecl inedaswel l
duri
ngt hi
st ime.Hispasthi stor
yi ncludeshypertension,whichwasdi agnosed5
yearsagoandt r
eatedwithmet oprolol50mgt wicedaily.Onexami nat
ion,hisheart
rat
eis85bpm andbl oodpr essur eis145/ 90mm Hg.Hi slungsoundsar eclear.On
auscult
ati
on,t he hear
tr atei si r
regular
.The r emainderoft he exami nati
on is
unremarkable.Anelect
rocardi ogram (ECG)isobtained(Figure1)
.

Whi
choft
hef
oll
owi
ngi
sthenextst
epi
nthemanagement
?

a.OrderaHol t
ermonitor.
b.I
ncreasemet oprol
olto100mgt wi
cedai
ly.
c.Referforcat
heterablat
ion.
d.I
niti
atedigoxi
nat0.25mgdai l
y.
e.I
mpl antadual-chamberpacemaker.

Answer

C.Ref
ert
ocat
het
erabl
ati
on.

ThisECG demonstratesanect
opi
catrialr
hythm atabout100bpm,withsecond
degreeat
riovent
ri
cularblock(
Wenckebach)
.Ther hyt
hm isnotsinus,astheP
wavesar enegativeinl eadI I.Thi
srhythm persist
sdespitetheadmi
nist
rati
onof
beta-
blockade,andher emai nssymptomat i
c.Thenextst epistoproceedwi t
h
el
ectrophysi
ologi
cst udyt omapandabl atetheatrialf
ocus.Apacemakerisnot
needed ast helevelofbl ocki sunli
kelyt o besubnodal.Holt
ermonitori
ng i
s
unnecessaryatthispointast hediagnosisisclearfr
om theECG,andt
hehi st
oryi
s
consi
stentwithsympt omsr elatedt
othisrhythm.

49.
A75- year-ol
dmancomest ocl inicf orf oll
ow- up.Hehasf eltwellunt i
labout3
mont hsagoandnowi sshor tofbr eathwhenhewal ks>100yar dsandi sbot hered
bypul sati
onsi nhi sneck.Hispasthi storyincludesdyslipidemi a,hyper t
ension,and
type2di abetes.Fiveyear sago,heunder wenti mplantationofasi ngle-chamber
pacemakerf orsyncope,wi tha5- secondpauseseenonaneventmoni tor.He
previouslyunder wentanucl earst ressi magi ngstudy6mont hsagof orevaluat i
on
ofchestdi scomf ort
,showi ngnor mall eftvent ri
cul
arf unct ionandnoi schemi a.
Currentmedi cati
onsar emet opr olol25mgt wi cedail
y,li
sinopr il10mgdai l
y, aspiri
n
325mgdai l
y,andsi mvast atin20mgdai ly.Onphysi calexami nati
on,hi sbl ood
pressurei s120/ 70mm Hgandhear trateis60bpm.Thepacemakersi tei swel l
healedi nt heleftchest.Uponauscul t
ati
on,t hehear trhyt hm i sregularand  S2 is
paradoxicallyspli
t.Uponi nt
er rogationofhi spacemaker ,ther eisnoi ntri
nsicactivity
atsingle-chambervent ricul
arpaci ng( VVI)40bpm.Hi sel ectrocar di
ogram isshown
inFigure1.
Whi
choft
hef
oll
owi
ngwoul
dbet
henextst
epi
nmanagement
?

a.Enablerate-
responsivepacing.
b.Upgradetodual -chamberpacemaker.
c.Disconti
nuemet oprolol.
d.Raisethepacingr at
et o80bpm.
e.Upgradetoabi ventri
cularpacemaker
.

Answer

B.Upgr
adet
odual
-chamberpacemaker
.

Thisel ect rocardiogr am showsr egularvent r


icularpacingatar ateof60bpm, wi thP
wavesseenj ustf ollowi ngeachpacedQRScompl exduet oretrogradeconduct i
on
throught heat riovent r
icular(AV)node.Wi thnoi ntri
nsicact ivit
yatVVI40bpm, there
i
sei thercompl etehear tblockpresentor ,lesslikel
y,atrialstandst i
ll(i
.e.,nointri
nsic
sinusact ivit
y).Hi ssympt omsofexer ciseintoleranceandneckpul sationsar elikel
y
tor efl
ectpacemakersyndr ome.Upgr ade oft he devi ce t o a dual -chamber
pacemakerwi llr estore AV synchr ony and i mprove sympt oms.Di scontinuing
met opr ololwi l
lnotr eversecompl et
ehear tblockorat r
ialstandst i
llifpresentandso
willnotchanget hepat i
ent'ssympt oms.I ncreasingt hepaci ngratewi llnotaddr ess
thel ackofAVsynchr ony.Si milarl
y,rate-responsivepaci ngmodes( i
.e.,VVI),will
notr estoreAV synchr ony.Bi ventri
cularpaci ng,at reatmentf orhear tfail
urei n
patientswi thawi deQRS, alsodoesnotaddr essAVsynchr ony.
50.
A35- year-oldwomanpr esentsforeval
uat i
onofepi sodesofr api
dheartbeating,
whereshef eelsshei saboutt ofai
nt.Overall
,shehasspel l
saboutonceamont h,
whichusual lyr esolveontheirowni nabout5- 10mi nutes,butont wooccasions
haver equi
redt herapyintheemer gencydepar t
ment .Witht hi
spastepisode,she
wastol dherdi agnosiswassupr avent
ri
culartachycardi
a( SVT)andthatsheshould
consultwithacar diol
ogi
st.Shebr i
ngswi t
hhert odayt heelectr
ocar
diogr
am (ECG)
thatwasobt ained on t
hel astepisode( Fi
gure1) .Shedoesnotwantt otake
medicationandwant st
oknow mor eaboutcat het
erabl ati
on,butisconcerned

abouti
tspot
ent
ialr
isks.

Whichofthefol
lowi
ngmajorcompl
icat
ionsshoul
dshebeadvi
sedofbasedont
he
l
ikel
yet
iol
ogyofSVTrepr
esent
edi
ntheECGshowni nFi
gur
e1?

a.3%r
iskofstr
oke.
b.3%r
iskoftamponade.
c.3%r
iskofpulmonaryvei
nst
enosi
s.
d.1%r
iskofphreni
cnervei
njur
y.
e.1%r
iskofheartbl
ock.

Answer

E.1%r
iskofhear
tbl
ock.

ThisECGshowsSVTwi thanar rowPwavei nscr i


bedatt heendpor tionoft heQRS
compl ex( bestseenasanegat i
vedef l
ectioni nthei nfer
iorl i
mbl eadsandaposi tive
deflect
ioni nl eadV1)f orwhi cht hemostl ikelydiagnosi sist ypi calat ri
oventri
cul ar
(AV)noder e- entry.Cat heterabl at
ioni sper f
ormedi nt hesl ow pat hwayr egi on,
whichi st reatedbyabl atingj ustant er
iortot heosoft hecor onar ysinus.Ther iskof
compl etehear tblocki sl ow,about1%.ForSVTduet oAVr e- entrantt achycar dia
usinganaccessor ybypasst ract,ifablationi sr equir
edf rom t hel efthear t
,t hen
anticoagul ationi sadmi nisteredt opr eventst r
oke,andt her iskofst rokeint his
sit
uat i
oni sest i
mat edat1%.Phr enicner vei njuryisapot ent i
alr iskf orablation
performedneart hehi ghcr istat erminali
s,whi chi sapot entialconcer ni nablati
onof
atri
alt achycar diamappedt ot hisregion.Pul monar yvei nst enosi si sapot ent i
al
concer nf orabl ationint hel eftat r
ium fortreatmentofat rialfibril
lation( orfocalatr i
al
tachycar diar elatedt oapul monar yvein).

51.
An18- year-oldmanpr esentstotheclinicrefer
redbyhispri
mar ycarephysi
cian.He
denies any past car diac issues and has no hi stor
y of syncope. Hi s
electr
ocardiogram (ECG)showsapat t
er nofpseudor i
ghtbundlebranchblockwith
STel evat
ionandT- wavei nver
sioninl eads V1andV2.Youinterpr
etthisECGasa
typeIpatternofBr ugadasyndr ome.Hi soldestbrot
herdiedsuddenlywhenhewas
25year sold,andnoECGsofhi sbrotherareavai l
abl
e.
Whi chofthefollowingist hemostappr opri
atenextstep?

a.Advisetoavoidst r
enuousact i
vit
y.
b.I
mpl antimplantabl
ecar di
overt
er-def
ibr
il
lat
or(I
CD).
c.Continuedcli
nicalfol
low-up.
d.Provocativet
estingwithsodium channelbl
ockade.
e.I
mpl antlooprecorder.

Answer

C.Cont
inuedf
oll
ow-
up.

Thepati
enthasat
ype1Br ugadasyndromeECGpat t
ern.Anolderbr
otherwhodied
suddenl
ydoesnotincr
easehisri
skf orsuddendeath,andbecausethepati
entis
asympt omat i
c,he does notmeetcr i
ter
iaforI CD implantati
on.The ut il
it
y of
electrophysi
ologicstudyfortr
eat mentdecisi
onr emai
nscont roversi
al.Provocati
ve
testingwithsodium channelblockersisnotindi
catedinat ype1ECGpat t
ern.There
i
snoneedf orthispati
enttorefrainfr
om physicalact
ivi
ty.Atthispoint,t
hepat i
ent
needst obef ol
lowedandadvi sedt oavoiddrugswi thsodium channelbl ocking
activit
y(seewww. br
ugadadrugs.org)
.

52.
A45- year-oldwomanpr esentst oyourcl ini
cforeval uati
on.Sher eportshaving
episodesofr apidheartbeatingfort hepast10year s.Theepi sodesoccur2t o3
ti
mesayearwi t
hsympt omsofchestpai nandl i
ghtheadedness.Mostepi sodes
stopspont aneouslywithi
naf ewmi nutes,butlastmont hsher equiredtreatmentin
theemer gencydepar t
mentwi thi ntravenousadenosi ne.Shehasnoot herpast
medi calhistoryandhast akendi l
tiazem 180mgdai l
yaspr escr ibedbyherpr imary
physician.Revi ew ofherel ectr
ocar diogram duringanepi sodeshowsanar row
compl ex tachycar di
a,shows a hear trate of165 bpm,and i s suggesti
ve of
atri
oventricular(AV)nodalre-entrantt achycardi
a.Anechocar diogram donedur ing
heremer gencydepar t
mentvisitshowednor malfindings.

Whi
choft
hef
oll
owi
ngi
sthenextmostappr
opr
iat
est
epi
nmanagement
?

a. I
nit
iateami odarone.
b. Catheterablati
on.
c. I
nit
iateflecaini
de.
d. Continuedclini
calfol
low-
up.
e. I
nit
iatemet oprolol
.

Answer

B.Cat
het
erAbl
ati
on

Thi s pati
enti s a young woman wi t
h sympt omati
c and r ecur
rentepi sodes of
supr avent
riculartachycardiawhocont inuest ohavesympt omsdespi t
eAVnodal
blockingagent s.Catheterablat
ionisaCl assIopt i
onf ormanagement .Fur ther
adjustmentofAV nodalbl ocki
ng agents( eit
herbyi ncreasing thedi l
tiazem or
addi ngmet oprolol)canbeconsideredifdesiredbyt hepat ient.Si
mi l
arl
y,flecainide
also maysuppr essevent sofsupr aventri
culartachycardi a.However ,themost
effecti
veopt ionf ormanagementi nt hispat i
entwhohasal r
eadyf ai
ledmedi cal
therapyi scat heterablati
on,wit
ha>95% successr at
e.Inayoungsympt omat i
c
pat i
ent,amiodar onewoul dnotbeagoodl ong- t
erm choice.
53.
A70-year-
oldmanpr esent
stot
heemergencyroom wi
thseverel
ight
headedness,
shor
tnessof 
breat
h,andchestti
ght
nessthatstar
tedabr
uptly1hourago.Past
hist
oryi ncl
udes  t
ype 2 di abetes for t he past 10 year s,hypertension,and
hyperli
pidemi a.Heal sohasahi storyofsmoki ng,which hestopped5year sago.
Currentmedi cat i
onsar easpi
rin81mg,met oprolol25mgbi d,li
sinopril20mg,
metformin500mgbi d,andatorvastati
n80mg.Onphysi calexamination,heispale
anddi aphoret i
c.Vitalsignsarebl oodpr essure70/ 40mm Hgandhear tr at
e170
bpm.Hehascl earl ung f
iel
ds,S1- S2 
distant
,t achycar
dic,and no mur mur .His
el
ectrocar di
ogram (ECG)i sshowni nFigure1.

Whi
choft
hef
oll
owi
ngi
sthenextst
epi
nmanagement
?

a.Over-
drivepaci
ng.
b.Li
docaine.
c.Adenosine.
d.Amiodarone.
e.Cardi
oversi
on.
Answer

E.Car
diover
sion

TheECGshowsvent riculart
achycardi
a,andthepasthi
storyi
shighl
ysuggest
iveof
cor
onaryarter
ydisease.Hei shypotensi
veanduncomfortabl
e,andt
hebestcourse
ofact
ionistorestor
esi nusrhythm prompt
ly.Whil
eamiodaroneandli
docai
necan
beconsi
dered,gi
venthehypot ensi
on,iti
sbestnott
owait.

54.
A 75-year-oldmanpr esentst otheemer gencyr oom ( ER)wi t
hcompl aintsofa
raci
nghear tbeat,severelightheadedness,andmoder atechestt i
ghtness.Hebr ief
ly
passedoutwheni tfi
rststarted.IntheER,hehasawi decompl extachycardiaata
rateof170bpm,abl oodpr essureof75/ 40mm Hg,andanel ectr
ocardiogram
confi
rmingvent ricul
art achycar di
a( VT).Cardiover
sioni sper f
ormedi ntheERt o
rest
oresinusr hythm.Subsequenteval uati
oni nthehospi talshowsapeakt roponi
n
of1.0ng/ dl.Cardiaccat heterizat
ionreveal
st hree-vesselcoronarydiseasewi tha
lef
tventri
cular(LV)ej ecti
onf ractionof35-40%,conf irmedbyechocar di
ogram.He
undergoescor onaryar t
erybypassgr af
tsurgeryduringhishospi t
ali
zati
on.

Whi
choft
hef
oll
owi
ngi
sthenextbestst
ep?

a.I
nit
iateami odaroneandr eassessLVf uncti
onin90days.
b.Wear ablecardi
overter-
defi
bri
llatorfor45days.
c.I
mpl antICDi fi
sVTi nducedinel ectr
ophysiologicst
udy.
d.Per
for m cathet
erablati
on.
e.I
mpl ant i mplantable cardioverter-
defibr
il
lator (I
CD) bef
ore hospi
tal
di
schar ge.

Answer

E.I
mpl
anti
mpl
ant
abl
ecar
diover
ter
-def
ibr
il
lat
or(
ICD)bef
orehospi
taldi
schar
ge.

This patientmeets secondary prevention cri


ter
iaf orimplantofan I CD,and
implantati
onshouldnotbedel ayedf or90days.TheAVI D(Anti
arrhythmicsVersus
Implantable Def
ibr
il
lat
ors)studyenr ol
led pati
ents who werer esuscitated f
rom
vent
r i
cularfi
bri
ll
ati
on,wit
hsustainedVTwi t
hsyncope,orwithsustainedVTwi than
LVej ect
ionfract
ion<40%associ at
edwi thseveresympt oms( heartfail
ure,angi
na),
showi ngthesuperiori
tyofanI CDf orsur vi
valoveranti
arrhyt
hmict her
apy.About
10%oft heAVIDpopulati
onunder wentr evascularization,andthebenefi
tofanI CD
was i ndependent of revascular
izati
on. Ther efore,t he Ameri
can College of
Cardiol
ogyFoundation/Amer i
canHear tAssociation/ EuropeanSoci
etyofCardiology
gui
del i
nesregardtheimpl antofanI CDasaCl assIr ecommendationforpatients
wit
hsust ai
nedVTwi t
hst ruct
uralheartdi sease,r egardlessofwhetherstableor
unstable.

55.
A 30- year-ol
dwomanhasasuddenonsetofpal pi
tati
onsandl ightheadedness
whi leplayi
ngwi t
hherchildreninthepark.Af
teranhour,
withnorel
ief,shepresents
tot heemer gencyroom.Hervi talsi
gnsarebloodpr
essure120/70mm Hgandheat
rate180bpm.Shei sthengi venadenosine6and12mgbutwi thoutef f
ect.Her
electrocardi
ogram (ECG)i sshowni nFigur
e1.Shei sof f
eredcardioversi
onbut
refuses.

Whi
choft
hef
oll
owi
ngmedi
cat
ionsi
smostl
ikel
ytochemi
cal
lycar
diover
tthi
srhyt
hm?
a.Ibutil
ide.
b.Adenosi ne( hi
ghdose)
.
c.Met oprolol.
d.Verapami l
.
e.Lidocaine.

Answer

D.Ver
apami
l

Int his young patientwi t


houtst ructur
alhear tdi sease,one mustf ir
stassess
hemodynami cstabili
ty.Becauseshehasnochestpai n,dyspnea,orhypot ension,
emer genttherapyisnotnecessar y.ThisECGshowsat achycardiawithjustami l
dly
wideQRScompl ex.Thereisar i
ghtbundl ebranchbl ock( RBBB) -l
ikemorphol ogy.In
a young per son who i s otherwise healthy,this ECG i s mostconsi stentwi th
i
diopat hi
cl ef
tventricl
e( f
ascicul
ar)vent r
iculartachycar dia,whichi sar e-entrant
tachycardiainvolvi
ngmostcommonl ythel eftposteriorfascicl
e,givinganRBBB-
l
ikeappear anceandasuper ioraxis.Thisrhythm ishi ghlysensiti
vet overapami l
.It
i
sal ow- r
isktachycardia,andcat heterablati
onoft hef ascicl
einvolvedisusual ly
curative.

56.
Thepat i
enti sa67- year-oldmanwi thhistoryofcor onaryarter
ydiseaseforwhich
he under went cor onar y ar t
ery bypass gr aft surgery 5 years ago.Recent
echocardi
ogr am showed a mi l
dlyenlar
ged l ef
tvent ri
clewi t
hal ef
tventr
icul
ar
eject
ionfractionof25%.Hei sablet
oexer cise4t i
mesaweekf or1hourandal so
hikesforacoupl eofhour severyweekend.Hei sonopt imalmedicalther
apywith
carvedi
lol
,lisinopri
l,aspirin,andatorvast
atin.Hiselectrocar
diogram showsnormal
sinusrhythm wi thar ightbundl ebranchbl ockpat t
ernandQRSdur ati
onof138
msec.

Whi
choft
hef
oll
owi
ngi
sthemostappr
opr
iat
erecommendat
ionf
ort
hispat
ient
?

a.I
mpl antabivent
ricul
arICD.
b.I
nit
iatesotal
oltherapy.
c.I
mpl antabivent
ricul
arpacemaker .
d.Continuecurr
entmedi calt
herapy.
e.I
mpl antanimplantablecar
diovert
er-def
ibr
il
lat
or(
ICD)
.

Answer

E.I
mpl
antani
mpl
ant
abl
ecar
diover
ter
-def
ibr
il
lat
or(
ICD)
.
The pat ientmeet s cr i
teri
af ora pr imar y pr evention ICD based on MADI T-II
(Multi
cent erAut omatic Def i
bril
latorI mpl antation Tr i
alI I
)cl inicaltrialr esults.
Continuingcur rentmedi caltherapyandi niti
atingsot aloltherapyar eincor r
ect ,as
thepatientdoesmeetcr it
eri
af ordevi cei mpl antandant iarr
hythmi cther apyisnot
demonst ratedt oimprovesur vival.Impl antingabi ventr
icularICDandi mpl antinga
biventr
icularpacemakerar eal soi ncor rect,ast henew devi cet herapygui delines
from 2012 st ateitis a Cl ass I I
Ii ndication forbi vent r
icularpaci ng car diac
resynchronizationtherapyinpat ientswi t hNewYor kHear tAssoci ati
oncl assIorI I
andanon- l
eftbundlebranchbl ockpat ter nwithQRSdur ati
on<150msec.

57.
A63- year
-ol
dmancomestoseeyouinyourcl
ini
c.Thepati
enthasapastmedical
hist
oryofcoronar
yar
ter
ydi
seaseforwhi
chheunder wentper
cutaneouscor
onary
interventi
ont ot hemi dleftant eri
ordescendingandmi dr i
ghtcoronaryar tery3
year sago.Hi sr ecentechocar diogram showedadi l
atedlef
tventr
iclewi thal eft
vent r
icleej
ectionfracti
onof26%.Hei sabletoperform moderatephysicalactivi
ty3
timesaweekandf eelsmildlimi t
ati
onwhenheexer cisesf
oroveranhour .Hehas
nosympt omsorcompl ai
ntsatt hepr esentti
me.Hei sonopt i
malmedi caltherapy,
including carvedil
oland l i
sinopr i
l.Hi s el
ect
rocardiogram shows nor malsi nus
rhythm andl eftbundlebranchbl ockwi t
haQRSwi dthof160msec.

Att
hispoi
nt,
whi
choft
hef
oll
owi
ngi
showt
hispat
ientshoul
dbeadvi
sed?

a.I
mplantadual-chamberI CD.
b.I
mplantabiventr
icul
arimpl antabl
ecar
diover
ter
-def
ibr
il
lat
or(
ICD)
.
c.Cont
inuecurr
entmedi caltherapy.
d.I
mplantasingle-
chamberI CD.

Answer

B.I
mpl
antabi
vent
ri
cul
ari
mpl
ant
abl
ecar
diover
ter
-def
ibr
il
lat
or(
ICD)
.

Therightansweri st oimplantabi ventri


cul
arI CDt odecreaset heriskofdeat hor
heartfailurehospi tali
zationbasedont her esult
sf rom MADI T-CRT ( Mul t
icenter
Automat i
cDef i
bril
latorImplantati
onTr i
alWi t
hCar diacResynchr oni
zat i
onTher apy)
and RAFT ( Resynchr onizati
on/Defi
bril
lat
ion forAmbul at
ory HeartFai lure Tr i
al)
,
whichdemonst rat
edt hatabi ventri
cularICD i napat i
entwithNew Yor kHear t
Associati
oncl assI Ireducedt hecomposi teofdeat handhear tfail
ureevent s.Int he
RAFT t ri
al,t he secondar y endpointofmor tal
it
y also was achi eved wi tha
bi
ventricularICDcompar edwi thICDal one.Cont i
nuingcurrentmedi calt herapyi s
i
ncorrect,ast hepat ientmeet scri
teri
aatt hi
st i
mef oradevi ceimplant .I
nt he2012
updatet ot he2008Amer i
canCol l
egeofCar diol
ogyFoundat ion/Amer icanHear t
Associat
ion/HeartRhyt
hm Soci
etydevicegui
deli
nes,i
tisnowaCl assIindicati
onto
i
mpl antabiventr
icul
arICDinapatientwit
hal ef
tventr
icleeject
ionfracti
onof35%,
i
nsi nusrhythm,inNew YorkHeartAssociat
ionclassII,II
I,orambul at
oryI V,wi
th
l
eftbundlebranchblockandQRSdur ati
on>150msec.

58.A64- year-
oldmanwi thahi st
oryofsinusnodedysfuncti
onandcompl et
eheart
blockwhounder wentdual-chamberpermanentpacemaker2year sagocomest o
yourof fi
cecompl ai
ning ofintermi
tt
entpalpi
tati
ons.Dur i
ng perf
ormanceofhi s
elect
rocardiogr
am bythenur se,hef
eelstheonsetofpal
pitati
ons,whichconti
nued
forami nuteaft
ertheelect
rocardi
ogr
am wascompl et
ed(Figure1).

Which oft hef


oll
owi
ng i
nter
vent
ionsi
smostl
ikel
yto decr
easehi
ssympt
omsof
pal
pit
ations?

a.Vent
ri
cul
arl
eadr
evi
sion.
b.Shorteni
nghispost vent
ricul
aratri
alr
efr
act
oryper
iod.
c.I
ncreasinghislowerr at
et o80bpm.
d.Metoprolol25mgbi d.
e.Decreasinghisatr
ioventri
culardel
ay.

Answer

D.Met
ropol
ol25mg.bi
d.

Thebeat sareventricul
arectopicsand ar enotr el
ated t
o pacing.Thereisno
evidenceofpacemakerl ead malf
unction,asthereisappr opri
atesensing and
ventri
cularcaptur
e.At ri
alofanant i
arrhythmi
c,suchasabet ablocker,shoul
dbe
attempted.Ifunsuccessfulandher emai nshi
ghl
ysympt omatic,t
henavent ri
cul
ar
tachycardi
aablati
oncoul dbeconsidered.

59.
A24- year-oldmanwi t
hapastmedi calhistoryofbi
cuspidaorti
cvalvestat
uspost
aor t
ic valve repl
acementi s admitt
ed over ni
ghtaft
eror thopedi
c surger
yf ora
fracturesust ai
nedwhi l
eski i
ng.Hisor i
ginalcardi
acsurgeryseveralyear
sagowas
compl i
catedbysi nusnodedysf unct
ion,andheunder wentpacemakerpl acement.
Whi l
eont elemetr
y, t
hefindi
ngshowni nFigure1wasnot ed.Thepatientwasasleep
dur i
ngt hetimeoft heepisode.
Thef
indi
ngsont
elemet
ryr
epr
esentwhi
choft
hef
oll
owi
ngexpl
anat
ions?

a.Undersensing.
b.Fai
luretocapt ur
e.
c.Normalf unct
ion.
d.Rate-dropresponse.
e.Oversensing.

Answer

C.Nor
malf
unct
ion.

Specif
icall
y,whatisdemonstrat
edismanagedventri
cul
arpaci
ng.Al
thoughthi
s
part
icularal
gori
thm i
sspeci
fi
ctoonecar
diacdevi
cecompany,
mostmanufact
urer
s
now havespeci ali
zedalgorit
hmsavai labl etomi nimi zet heamountofvent ri
cular
pacing.Theseal gori
thmsar ei ncr easingl ypopularpr ogr ammi ngchoicesduet o
findings ofwor sened cardiac funct ion wi t
h ventr
icul arpaci ng in heartf ail
ure
pat i
entsandar educedriskofat r
ialf i
brillati
oni npatientswi thsi nusnodedi sease.
Thepat i
entisasleep,young,andat hletic.Hehasnohi storyofat ri
ovent
ricul
ar( AV)
conduct iondiseaseandi sexhi bit
ingWenckebachphenomenon( Mobit
zt ypeIAV
block),whi chisnor mal.Despi teat ri
alpaci ng,thevent ricleisnotpacedast he
algor i
thm isdesignedtoal l
ow foral ossofAVconduct ionf oronecycl e.Thenext
ventr i
cularimpulseisapacedbeat ,andwi t
hashor tenedpacedAVdel ay,whi chis
also acl uetot hispacing algorithm.Thi si snotover sensing ori nappropr i
ate
inhibiti
on of paci ng. Undersensi ng i s usual l
yr ecogni zed by i nappropr i
ate
pacemakerar ti
factdespiteelectricalact ivi
ty,whichi snotseenher e.Rat e-drop
responsei sanalgori
thm thatforcesdual -chamberpaci ngi nr esponset oadr opi n
nat i
veat r
ialr
ateand i snotdemonst r ated on thi
st r
aci ng.Lackofcapt ur eis
diagnosedwhenapacemakerar tif
acti snotf oll
owedbyaPwaveorQRScompl ex.

60.
An18-year-ol
dwomanwi t
hnosignif
icantpastmedi calhist
oryisbei
ngevaluat
ed
atacoll
egesport
sphysical
.Herphysi
cialexaminati
oni snormal.Al
thoughshehas
swum befor
e,att
hesecondmeetoftheyear ,sheexperiencesanepi
sodeofsudden
car
diacdeathwhi
leswimmi ng.

Which ofthefol
lowi
ng mechani
smsexpl
ainst
hel
ikel
ypat
hophysi
ologyoft
he
suddendeath?

a.Gainoff
unct
ioni
nthesodium channel
.
b.Lossoff
unct
ioni
nthesodium channel
.
c.Gainoff
unct
ioni
nthepotassi
um channel
.
d.Lossoff
unct
ioni
nthepotassi
um channel
.
e.Gainoff
unct
ioni
nthecal
cium channel
.

Answer

D.Lossoff
unct
ioni
nthepot
assi
um channel
.

A young adul
twi t
h sudden cardiac deat
h shouldr ai
se the suspici
on ofan
under
lyi
ngprimaryelect
ri
caldisorderorhypertr
ophiccardi
omyopat hy.Thereare
cer
tai
nsetti
ngsthatareregar
dedascl assicforcer
tainsyndr
omes.Oft hese,l
ong
QTsyndrometype1,withamut at
ionintheIKspotassi
um channelleadingtoaloss
offuncti
onandthusadel
ayi
nmembr
aner
epol
ari
zat
ion,hasacl
assi
cpr
edi
lect
ion
forevent
swithswimmi
ng.

61.
A 30- year -oldwomanpr esent sf oreval uationofepi sodesof  
recur rentsyncope.
Overt hepast20year sshehasf aint ed10t i
mes,buti nt hepast6mont hsshehas
hadt hreeepi sodes.Typi calspel l
soccurwhenshesi ngsi nthechur chchoi randar e
precededbyabout30secondsofnauseaandsweat ing.Shehasnoti njuredher self
,
butisembar rassed,ast hel asttimeshef ai
nt edt hepar amedicswer ecal led.She
has no ot herpastmedi calhi story and t akes no medi cations.On physi cal
exami nat i
on,herbl oodpr essur e( BP)i s110/ 60mm Hgandhear tratei s90bpm.
Herj ugularvenous pul se i s 
5 cm.Ther ei s a si ngle S1 and physi ologi cally
split
 S2 withoutmur mur.Ther emai nderofherexami nat i
oni sunr emar kabl e.An
electrocardiogr am i ntheof ficeshowsnor malsi nusr hythm atar at eof93bpm.The
foll
owi ngday,sheunder goesat il
tt ablestudyshowi ngasupi neBPof120/ 70mm
Hgandahear trat
eof80bpm.Wi thpassi vet i
lt,theBPovert hef irst5mi nut esi s
110- 120/ 55- 65mm Hgandhear trat esare80- 100bpm.At8mi nut esint ot het i
lt
,
shecompl ainsofnauseaj ustl i
keshef eelsbef oreherusualf ai nts,andt hen30
secondsaf tert hatshel osesconsci ousness.HerBPr eadingobt ainedabout20
secondsbef oref ainti
ngi s65/ 40mm Hgandhear tratei s40bpm.Att hemoment
whenshef aint s,herhear tr atei s30bpm.Shei simmedi at
elyl aidsupi ne,wi t
h
promptr eturnofherBPandhear trat etobasel inel evel
s.

Whi
choft
hef
oll
owi
ngi
sthenextbestst
epi
nhermanagement
?

a.I
niti
atemetoprolol
.
b.Avoidtri
gger
sforsyncope.
c.I
niti
atemidodri
ne.
d.I
mpl antadual-chamberpacemaker
.

Answer

B.Avoidtriggersf
orsyncope
Thispatienthasvasovagalsyncopewi thawel l
-def
inedt rigger:st
andingi nthe
choiratchur ch.Hert il
tt abl
e study suggests a mi xed vasodepr essorand
cardi
oinhibit
oryresponse,and i treproduced hersympt oms.Whi l
e a cl ini
cal
syncopalspel lmay show a di f
fer
entphysi ol
ogicr esponse,t he fi
rstst ep in
managementshoul dbeconser vati
ve,str
essinghydration,sal t,andavoi danceof
tr
igger
s.Whi l
eotherdiagnosti
ctestssuchasechocar diogr am andst resstesti
ng
arecommonl yused,thedi agnosisinthispat i
entiscl ear,wi thouttheneedf or
furt
herdiagnosti
ct esting.Li kewise,electr
ophysiol
ogicstudycan beusef ulfor
pati
entswithsyncopesuspect edtobeduet oavent r
icul
ararrhyt
hmi a,butdoesnot
playarol
ef ortheworkupofvasovagalsyncope.I fconser
vati
vemeasur esfai
l,then
ani mpl
antableloopr ecor dercanbeconsi der
edt odetermineifclinicalepisodes
showamar kedcardio-inhi bi
toryresponse(despitetheti
ltt
ablefindings),asthere
canbear oleforpaci
ngf orpatient
swi thprol
ongedper i
odsofasystole. 

62.
A 45- year -
old man i s sentt o you f orpr eoperati
ve clear ance because ofan
abnor malel ect
rocardiogram ( ECG) .Hehasnopr iormedi calhi storyasidef r
om a
ri
ghti nguinalherni
af orwhi chhei sscheduledt ounder gosur gicalr epai
rnextweek.
Hewasseenbyt heanest hesiologistyesterdayforapr eoperativeassessmentand
bringst heECGt hatwasper for medatt hattime.Hef eelswel lasidef r
om discomf ort
relatedtohi sherni
aanddeni esanyshor t
nessofbr eat
h, chestpai n, pal
pitat
ions, or
dizziness.Heusual l
ypl aysr acquet bal
l3t i
mesaweek, whi chhehasdonesi ncehi s
collegedays.Het akesnomedi cati
ons.Hi sphysicalexami nati
oni sunremar kable,
withabl oodpressureof115/ 70mm Hgandhear trateof60bpm.Hebr ingsi nthe
ECGf rom yester
day’sappoi ntmentwi t
ht heanest hesiologist(panelA) ,whi chyou
compar ewiththeECGobt ainedt oday(panelB) ,
asshowni nFi gur e1.

Whi
ch of t
he
foll
owi
ngis the next
step in

management
?

a.Proceedwi
thherni
arepai
r.
b.Obtainast
ressECG.
c.Ordera24-hourHol
termoni
tor
.
d.Referf
orel
ect
rophysi
ologi
cst
udy.
e.Orderatr
anst
horaci
cechocar
diogr
am.

Answer
A.Pr oceedwithherniar epai
r.
TheECGi npanelAshowspr e-excit
ation,indi
catingabypasst racti
nt hi
spat i
ent,
butwhi chisnotpresenti ntheECGper f
ormedt oday(panelB).Hei sasymptomat i
c
withregardtothisfinding,andf urt
hermor e,hehasexcellentexerci
setoler
anceby
histor
y.Hei sthereforeatver yl ow r
iskf ornoncardiacsurgeryandcanpr oceed
withoutf ur
thertesting.El ect
rophysiologic study isindicat
ed ifhe devel ops
sympt omssuggestiveofsupr aventri
culartachycardi
a.

63.
A35- year -oldmancomest oyourcl inicforhissympt omsofpal pi
tati
ons.Fort he
past5year shehasf eltani r
regul arbeatwi t
hfrequent” skips.”Hesayst hatinthe
past6mont hshisr unningcapaci tyhasdecl i
nedaswel l,andi ttakeslongerf orhim
tocompl etehi susual5- miledailyr un.Hedeni esanysyncopeorchestpai n.Hehas
nopastmedi calhistory.Fami l
yhi storyi snotableforanacut emyocar dialinfar
cti
on
in hi
sf at heratage 50 year s.He t akes no regularmedi cati
ons.On physi cal
examination,hi sbl oodpr essurei s108/ 60mm Hgwi thapul seof70bpm.He
weighs170pounds( bodymassi ndexi s25.8)
.Thej ugul arvenouspul seisseenat
6cm,andcar oti
dupst rokesar enor malandwi t
houtbr uit.Lungsoundsar eclear,
andcar diacauscul tationshowsasi ngleS1,physiologicallyspl i
tS2,andagr ade1/6
shortsyst olic mur mur .An ect opic beataboutever yt hree beat si s heard on
auscult
ation.Anel ectrocardiogram ( ECG)i sobtained.
Whi
choft
hef
oll
owi
ngbestexpl
ainst
hef
indi
ngsont
heECG(
Figur
e1)
?

a.Automat i
city.
b.Micro–re-entryaroundaf i
xedscar.
c.Earl
yaf t
erdepolari
zations.
d.Delayedafterdepolari
zati
ons.
e.Macr o–re-entr
yar oundthepulmonaryval
ve.

Answer

D.Del
ayedaf
ter
depol
ari
zat
ions

This ECG shows pr ematur e ventricul


arcontracti
ons (PVCs)wi thl eftbundl e
mor phologyandinfer
ioraxis,suggest i
veofafocusatt her i
ghtvent r
icularoutfl
ow
tract
.In a young adultwi thoutst ruct
uralheartdisease,t he PVCs ar ise f
rom
tri
ggeredacti
vit
yduet ocatecholami ne-i
nduceddelayedafterdepolari
zations.Beta-
blockersandcalci
um channelbl ockersmaybeef fect
ive,
butcat heterablati
onoft he
focusisusuall
ycurati
ve.
64.
A70- year -
oldmanpr esentstot heemer gencydepar t
mentaf teranearsyncopal
epi
sodet oday.Her eportsthathewasathomeandhadbeenwal kingtothekit
chen
whenhesuddenl yfeltveryf ai
ntandsliddownt hewalltot hefloor,wherehever y
nearl
yl ostcompl eteconsci ousness.Hedi dnotinjur
ehimsel f
.Hehashadper iods
ofbri
efl ightheadedness,suddeni nonset,lasti
ngaf ew secondsovert hepast2
months,whi chpr ompt edhispr i
mar ycarephysiciantoorderaneventmoni tor.His
pasthistoryincludeshyper t
ension,whichistreat
edwi t
hl i
sinopr
il10mgdai l
y. 

Anechocar
diogr
am perf
ormed6mont hsagoshowednormalventri
cul
arfunct
ion
andval
ves,
withamil
dlydil
atedl
eftat
ri
um andaneject
ionf
ract
ionof60%.
 

Hisvi t
alsignsareabl oodpressureof120/ 70mm Hg,hear trat
eof70bpm,and
respir
ationsof10/minute.Thecardiacexami nat
ionshowsar egul
arr hyt
hm anda
paradoxicall
yspl
itS2 
withoutmurmur .Aneventmonitorstr
ipi
savail
abletoyouthat
correspondstothetimeoft hepat
ient’snearsyncopaleventt
oday,showni nFi
gure
1.

Whi
choft
hef
oll
owi
ngi
sthenextbestst
epi
nmanagement
?
a.Referforti
lttablestudy.
b.Referforel
ect rophysi
ologicstudy.
c.Referforpacemakeri mpl ant
.
d.I
niti
atewar farin.
e.Disconti
nuel isinopr
il
.

Answer

C.Ref
erf
orpacemakeri
mpl
ant
.

Thiseventmonitorstr
ipshowsaper i
odofasystolewithcompl eteheartblock.
Therear ebri
efperi
odsshownwher econductionresumeswi thapr olongedPR
i
ntervalandawideQRScompl ex(which,gi
venthefi
ndingsonphysicalexaminati
on,
would suggestaleftbundlebranch block)
.Thisi sasi tuat
ion wheretherei s
advancedconducti
ondiseaseandi nter
mitt
entcompleteheartblockast hecause
forsyncope,
andapacemakerneedst obeimplantedurgent
ly.

65.
A50- year
-oldwomanwi thapasthistor
yofanoni schemi ccardi
omyopathy,witha
lef
tventri
cularej
ecti
onf r
acti
onof35%pr esent
stot hepacemakercli
nicforrouti
ne
fol
low-upf orherimplantabl
ecardiover
ter-def
ibr
il
lator(I
CD).Adual-chamberI CD
wasi mplanted6mont hsagoforpr i
marypr event
ionf orsuddendeath.Shef eel
s
well,
buthasnot i
cedbriefpal
pit
ati
onsper cei
vedasar api
dheartr
atef
oryears. 

Hercurr
entmedi
cati
onsincl
udecarvedi
lol12.
5mgbid,l
isi
nopr
il10mgdail
y,and
aspi
ri
n81mgdaily.Herbl
oodpressur
eis110/60mm Hgandheartr
atei
s65bpm. 

TheICDiswel lhealedinthelef
tchest
.Inter
rogationofherICDt odayshows15
epi
sodesoftachycardi
ainthepast3mont hs,allofwhicharesuccessful
lyt
reat
ed
wit
hantit
achycardi
apacing(noshocksneeded).Arepresent
ati
veepisodeisshown
i
nFigure1.
Whi
choft
hef
oll
owi
ngi
sthenextbestst
epi
nmanagement
?

a.Ref
erforablation.
b.I
nit
iatedigoxin0.125mgdail
y.
c.Conti
nuecur rentmanagement.
d.I
ncreasecarvedilolt
o25mgt wicedai
ly.
e.I
nit
iatesotalol80mgt wi
cedail
y.

Answer

A.Ref
erf
orabl
ati
on

TheICDinter
rogat i
onshowst achycardi
aatar at
eofabout200bpm,wi t
htheatri
al
si
gnal consistentl
yf oll
owing the vent r
icular si
gnal about 30 msec l at
er.
Ant
it
achycardiapaci ngt henter
minatesthet achycardi
a.Thispat
ter
ni ssuggest
ive
ofsupravent
riculart achycardi
a,specif
icall
yat ri
oventr
icul
arnoder e-ent
ry.Ina
patientwit
hal ong-standinghi
stor
yofpalpit
ati
ons,andwi t
htheconcernthatt
his
tachycardi
amayi ncreaseherri
skforaninappropr
iateshock,i
tisadvi
sabl
eforher
toundergoelectr
ophysiologi
cstudyandcathet
erablat
ion.

66.
An85- year-
oldmani sseenint hepacemakercl i
nicforusualdevi
cefollow- up.He
feel
swel loveral
lwith no specificcomplai
nts.Hi spasthi st
oryincludesat r
ial
fi
bril
lation (
AF),si
ck sinus syndrome,hypertension,and moder at
e Al zheimer’s
dement i
a.His AF had been par oxysmaluntil6 mont hs ago when itbecame
persi
st entandnow isregardedasper manent.Hei ssedentaryandliveswi t
hhi s
caregiverswhoaccompanyhi mt ohisappoi
ntmentint hepacemakercli
nic. 

Curr
entmedi
cationsarer
ivar
oxaban20mgdai
ly,
met
opr
olol25mgt
wicedai
ly,
and
hydr
ochl
orot
hiazide25mgdaily.
 

Hi
sbl oodpressuretodayis135/
80mm Hgwi
thahear
trat
eof70bpm.Hi
s
el
ect
rocar
diogram showsAF.
 

Hehasadual -chamberpacemakert hatwasi mplanted7year sagoandsett oa


modeofdual -chamber,r
ate-responsi
vewi t
hal owr at
eof60bpm.Theat r
ialout
put
issetat5V at1msec,andvent ri
cularoutputissetat2V at0. 5msec.The
ventri
cul
arleadhasexcel lentsensingandpaci ngthresholds.Theatrialleadis
sensingpoorl
y;inappr
opriateatri
alpacingisnotedondevi ceinter
rogat
ion.Batt
ery
longevi
tyi
sest i
matedat18mont hsbasedoncur rentset
tings.

Whi
choft
hef
oll
owi
ngi
sthenextst
epi
nmanagement
?

a.Disconti
nuet hemet oprolol
.
b.Perform acardioversi
on.
c.I
ncr easethesensiti
vit
yoft heatri
all
ead.
d.I
mpl antanewat ri
alpacinglead.
e.Repr ogr
am thepacemakert osingl
e-chambervent
ri
cul
arpaci
ng(
VVI
).

Answer

E.Repr
ogr
am t
hepacemakert
osi
ngl
e-chambervent
ri
cul
arpaci
ng(
VVI
).

Thispat
ientisi nper manentAFand,t heref
ore,doesnotr equi
redual
-chamber
paci
ng,whi ch is contr
aindi
cat
ed int he set
ting ofpermanentAF.The atri
al
under
sensing and high paci
ng thr
esholdsaredr aini
ng t
hebatter
yaswel l
.By
reprogr ammi ngthedevi cet oVVI ,t
hepat i
entmaywel lavoi
daf utur
egener ator
change by ext ending the batter
yl ongevity signi
ficantl
y.Whilei ncreasing the
sensitivityoftheat ri
alleadmayi mprovetheunder sensingproblem,itwoul dnot
eli
mi nat ethepossibil
ityofinappropri
ateatrialsensi
ngandf ai
lur
et omodeswi tch;
therefore,programmi ngt oVVIist hebestopt ion.Ast hepacingleadsar echronic,
dislodgementi shi ghlyunli
kely.Cardiover
siont osinusr hythm i
sal soinadvisabl e
givent hatAFi sr egardedasper manent .Givent hepat i
ent’
scomor bidi
ties,arat e-
cont r
olst rat
egyismostappr opr
iate.

67.
A 62-year
-ol
dmancomest otheemer gencydepar t
mentwithpalpi
tat
ionst
hat
star
ted3hourspri
or.Hehashadfivepri
orepisodesoverthepast2weeks,each
last
ingupto1minut
e.Hehashyper
tensi
on,butisotherwi
seheal
thy.
 

He takes aspir
in 81 mg and ramipri
l5 mg dai ly.Hi
s blood pr
essureint he
emergencyr oom is130/70mm Hgandhear trat
ei s95bpm.Thel ungsareclear
wit
hnoper ipheraledema.Hear
tsoundsar enormal,andthereisnoj
ugularvenous
di
stenti
on.Thepr esent
ingel
ect
rocar
diogram i
sshowni nFigure1.
Whi
choft
hef
oll
owi
ngi
sthebestmanagement
?

a.Atr
opine0. 5mgIV.
b.Consult
at i
onforpermanentpacemakeri
nser
ti
on.
c.I
nserti
onoft emporarypacemakerwir
e.
d.Metoprolol5mgi nt
ravenous(I
V).
e.Adenosine6mgI Vpush.

Answer

D.Met
ropol
ol5mgi
ntr
avenous(
IV)
.

Theelectr
ocardiogram showsat ri
altachycardi
aat160bpm,wi t
h3:2Wenckebach
conducti
onandr ightbundl ebranchbl ock.TheWenckebachi sphysi
ologicdueto
thehighatri
alrate,andt heref
ore,thereisnoindicat
ionforper
manentort empor
ary
paci
ngorat ropine.Atrialtachycardiadoesnott ypi
call
yrespondtoadenosine,so
metoprol
olisthebestr esponse.

68.
Theper
sonwi
tht
heel
ect
rocar
diogr
am showni
nFi
gur
e1haswhi
choft
hef
oll
owi
ng?

a.

si
ngl
e-chamberI
CDt
hati
sunder
sensi
ngvent
ri
cul
art
achycar
dia.
b.Adual-chamberICDoperati
ngintheVATmode.
c.An i mplant
able cardi
overt
er-
defi
bri
llat
or (I
CD) with a pacing rate
progr
ammedsi ngle-
chamberventri
cularpaci
ng40bpm.
d.A dual-chamberpacemakerprogrammeddual -
chamber
,rat
e-r
esponsive
60-120bpm.
e.AnAAI Rpacemakerprogrammedwi t
hal owerrat
eof40bpm.

Answer

C.
Animplant
ablecardi
overter
-defi
bri
ll
ator(
ICD)wi
thapaci
ngr
atepr
ogr
ammed
si
ngl
e-chambervent
ri
cularpacing40bpm.

Thet r
acingshowsasel f-terminati
ngr unofventr
icul
artachycardi
a,pr
obablynot
treatedbyt heICD duet oi tsrel
ativelyslow r
ateofself-termi
nati
ngnatur
e.Itis
unlikelytobeunder sensingt hisrhyt hm.Thet hi
rdt olastbeatispacedint he
vent r
icl
eduringthePwave, consist
entwi thaventr
icul
ar-onlypaci
ngdevi
ce,wit
ha
lowerr ateofapproxi
mat ely40bpm.I tisnotanat ri
alpacemaker,asthepacing
spikecapt ur
estheventr
iclewhenpaci ng.

69.
A60- year-ol
dmanpr esentstot heemergencydepar tmentf orincr
easedswel l
ing
aroundhi spacemakersi te.Hehasahi storyofsi
cksi nussyndr ome,hypertensi
on,
andpar oxysmalat rialfi
bri
ll
ation.Adual-chamberpacemakerwasi mplanted10
yearsago,andt hepacemakergener atorwasr eplaced4mont hsago.Hi slast
pacemakercheckwas3mont hsagowhenhewast oldeverythi
ngwaswor king
normally.Hismedi cati
onsincludewarfar
in,metoprolol,andli
sinopri
l.Lastweekhe
noti
cedsomet endernessar oundhispacemaker ,wi t
hr ednessdevelopingatt he
loweredgeoft hepocket2daysago.Hedeni esanyf eversorchill
s. 

Onexami nati
onofthepacemakersi t
e,thereisasmal lt
omoder at
eamountof
swell
ingandanar eaofer
ythemawi t
hindurati
ont hati
sabout2cm insizeatt he
i
nferi
or/l
ater
alaspectoft he pacemakersi t
e.The r emai
nderofhi s physi
cal
examinati
onisnormal,wi
thnor malvi
talsigns.Anechocardiogr
am isperfor
med
withnormalfi
ndi
ngs.Bl
oodcult
ureshavebeenor dered.

Whi
choft
hef
oll
owi
ngi
sthenextst
epi
nmanagement
?

a.Ini
ti
atecephal
exin,500mgqi d,andr eassessin7days.
b.Ini
ti
atevancomycin,1gintravenousq12hr ,andreassessi
n3days.
c.Perf
orm needl
easpi r
ati
onoft hepacemakerpocket .
d.Revisepacemakerpocketandr eplacegenerator
.
e.Extr
actpacemakergeneratorandl eads.
Answer

E.Ext
ractpacemakergener
atorandl
eads.

Thispat i
enthasapacemakerpocketinf
ecti
onwi thevi
denceofabscessf ormation.
Wi t
hi nf
ecti
oninthepocket
,boththegenerat
orandpaci ngleadsareinvolved,and
extracti
onoftheenti
resyst
em i
sneededt oeradicat
etheinf
ection(
ClassI )
.Ast his
patientisotherwi
seingoodheal t
h,thi
sist hebestcourseofact i
on.Ant ibi
otics
alonewi llnoteradi
catetheinfect
ion.Aspirat
ion ofthepocketshoul d notbe
performed(ClassI
II
).

70.
A 60- year -
old woman i s admi tted to t he hospi tal f or palpitati
ons and
li
ght headednessandi sr eferr
edt oyouf orconsul t
ati
on.Fort hepast3mont hs,she
hashadspel l
saboutonceaweekwher eshesuddenl yfeelsver yl ightheaded,
foll
owedbyasensat ionofbr eathlessnessandanxi ety.Twicei nthepastmont hshe
hasf ai
ntedcompl et
ely,oneofwhi chwaswi t
nessedbyherson.Herpasthi story
includesobesi tyandf ibr
omyal gia.Hermedi cati
onsar ecitalopram 20mgdai l
yand
pregabal in50mg3t imesdai ly.Onphysi calexami nation,herbl oodpr essur ei s
130/ 80mm Hg,hear tratei s70bpm,andwei ghtis180pounds( bodymassi ndex
32).Car diovascularexami nation r eveal
s an i rr
egularr hythm wi t
houtmur mur.
Abdomeni sobese,butt her emai nderoft heexami nati
oni sunr emar kable.An
electrocardiogram showssi nusr hythm at80bpm,wi thabor derl
inePRi ntervalof
200msec.An echocar diogram wasper f
or med yesterdaywi th normalf i
ndi ngs.
Labor atorystudiesarenor malexceptf orpotassium of3.5mEq/ dl.Ont elemet ry,the
stri
psshowni nFigure1wer erecor deddur ingyourphysi calexami nat i
on.
Whi
choft
hef
oll
owi
ngi
sthenextst
epi
nmanagement
?

a. Pacemakeri
mpl ant
.
b. Corr
ectpot
assium.
c. Conti
nuedobservat
ion.
d. Di
sconti
nuecit
alopram.
e. Di
sconti
nuepregabali
n.

Answer

A.PacemakerI
mpl
ant

Thesestri
psshow second-
degreeat r
iovent
ricular(
AV)blockwithperi
odsofbot h
2:1AVblockandMobi t
ztype2bl ock.ThefindingofMobi t
ztype2blockindi
cates
thatthesit
eofblockisbelow theAVnode( attheleveloftheHisbundle)
,and
ther
efor
e,i
smorelikel
ytoprogr
esst ocompl eteheartbl
ock.Shether
efor
equalif
ies
forapacemaker .Premat ur
eect opi
c(narr
ow)beat sal soar eseen(
thesedonot
aff
ectthesinusrate,whichi sabout75bpm) .Herepi sodesofsyncopear el
ikel
y
rel
atedtoherconduct i
ondi sease.Di
sconti
nuingmedi cat
ionsorsupplementi
ng
potassi
um wil
lnottreathersubnodalconduct
iondisease.

71.
An18- year- ol
dwomani sref
err
edf orevaluati
onf oraf amilyhistoryofl ongQT
syndrome( LQTS) .Shef eel
swellanddeni esanyhi stor
yoff aint
ing,al t
houghshe
doesnot ethatherhear tpoundswithexerci
se.Shehasnopastmedi calhistoryand
doesnotsmokeordr i
nkalcohol
.Heruncl ewasr esusci
tatedfrom acar diacar rest
and subsequent ly diagnosed wi th LQTS based on hi s postcardiac ar rest
elect
rocardiogram ( ECG).Onphysi calexaminati
on,herbl oodpr essurei s110/ 60
mm Hgwi thahear trat
eof70bpm, andtheremainderofherexami nati
oni snor mal.
Shebr i
ngsi nacopyofanECGdonebyherf amilyphysician2weeksago,whi ch
showssi nusr hythm at75bpm wi thaQTcof460msec.AnECGi nyourof ficetoday
hassimilarfindings, wi
thaQTcof450msec.

Whi
choft
hef
oll
owi
ngi
sthenextst
epi
nmanagement
?

a.Implantablecar
diovert
er-
def
ibr
il
lat
or(
ICD)i
mpl
ant
.
b.Electr
ophysiol
ogicstudy.
c.Stresstest.
d.RepeatECGst anding.
e.Genetictesti
ng.

Answer

E. Gener
ict
est
ing

Thispat i
enthasaf amil
yhistoryofpr obableLQTSi nheruncl
e.HerECGshows
bor derl
ineQTpr olongat
ion.Genet i
ct esti
ng,starti
ngwi t
hheruncl e,shoul
dbe
performed.I ni
ti
ationofbeta-blockadewoul dber easonabl
eifadiagnosi
sofLQTS
canbemadei nt he18-year-oldpatient.Therei snor ol
eforelect
rophysi
ologi
c
testing,andI CD implantcar r
iesaCl assI Ibindicati
ononlyint hepresenceof
strong r i
sk factorsf orsudden car diac death,whi ch thi
s patienthas not
demonst rat
ed.

72.
A69- year-
oldmalepr esent
st oyourcli
nic.Hehasbeenbotheredbyshor t
nessof
breaththathasdevelopedovert hepast10mont hswhenwal ki
ng.Hehasapast
hist
oryofcompl etehear tblockforwhi chheunderwentpermanentpacemaker
implant3 years ago.He al so has a histor
y ofcor
onary art
ery di
sease and
underwentpercut
aneouscor onaryint
erventi
ontotheri
ghtcor
onaryar t
ery7years
ago.Hei spaced100%oft hetime, andhi spacemakeri ssett odual -chamber ,r at
e-
responsi vewi thanat ri
oventri
culardel ayof180msec.Hi slastpacemakercheck
was1 mont h ago,and hewast old ever yt
hing wasf uncti
oning nor mal l
y.Hi s
under l
yingr hythm issinusr hythm wi thcompl et
ehear tblockandavent ri
cular
escape of45 bpm.Hi s 12- l
ead el ect
rocardiogram shows si nus r hythm wi th
ventri
cularpaci ngwi t
hal ef
tbundl ebr anchbl ock( LBBB)mor phology.TheQRS
widthis210msec.Anechocar diogr am per for
medear l
ierthismont hshowedal ef
t
ventri
cularej ectionfr
acti
onof30%.Hi scur rentmedi cat
ionsincludel i
sinopr i
l10mg
dail
y,f ur osemi de 20 mg dai l
y,car vedilol25 mg bi d,spironolactone 25 mg,
atorvastat i
n80mg,andaspi ri
n81mgdai ly.Onphysi calexaminat i
on,hi sbl ood
pressurei s108/ 67mm Hgandhear trateis65bpm.Thej ugularvenouspul seis
seenat6cm H2O.Lungsoundsar ecl ear,andoncar di
acauscul tat
ion,ther hythm is
regularwi th a par adoxi
call
y spl itS2.The r emai nderoft he exami nati
on i s
unremar kabl e.

Whi
choft
hef
oll
owi
ngi
sthemostappr
opr
iat
erecommendat
ion?

a.Upgradetoabiventr
icul
ari
mpl antablecardi
over
ter
-def
ibr
il
lat
or(I
CD).
b.Conti
nuecurrentmanagementandr outi
nepacemakerf
oll
ow- up.
c.Reprogr
am thepacemakertosi ngle-chambervent
ri
cul
arpacingwit
halow
rat
eof40bpm.
d.Upgradetoadual-chamberICD.
e.Upgradetoabiventr
icul
arpacemaker .

Answer

A.Upgr
adet
oabi
vent
ri
cul
ari
mpl
ant
abl
ecar
diover
ter
-def
ibr
il
lat
or(
ICD)
.

Thispat ient '


sNew Yor kHear tAssoci ati
on( NYHA)st atusisdet eriorati
ngdespi t
e
guideli
ne- directed medi cal t herapy of bet a-blocker s, angiotensin- convert
ing
enzyme i nhi bi
tors,and spi r
onolactone.He has a wi de QRS wi th an LBBB
mor phologyandi spaced100%oft het i
me.Het her eforemeet scriteri
af orupgr ade
to a cardi
ac resynchr onization therapy device. The RAFT
(Resynchr oni zati
on–Def i
bril
lati
onf orAmbul atoryHear tFail
ureTr ial)studyi n2010
showedt hebenef itofabi ventri
cularI CDf orpatient sinaNYHAcl assI IorI I
Iwi t
h
reducedr at esofdeat horhospi tali
zationf r
om hear tf ail
ure.I
nclusioncr i
teri
afort hi
s
studywer eal eftventr
icularejectionf ract
ionof30%,QRS>120msec,orapaced
QRSdur at ionof>200msec.

73.
A52- year-
oldmancomestotheemergencydepart
mentformil
dpal
pit
ati
onsthat
star
ted thepri
oreveni
ng.Pal
pit
ati
onspersi
sted when heawokethefoll
owing
mor ning,andt herefore,hedr ovehimsel ft othehospi t
al.Hehashadt hreepr i
or
episodesovert hepast2mont hs,eachl asti
ngonl y5-10mi nutes.Hehashada
priormyocar diali
nfarctionwithasingleangi oplastyandst enttot heleftci
rcumf l
ex
arteryandanover allleftventri
cul
arejectionfracti
onof45%.Het akesaspirin81mg,
clopidogrel75mg, rami pri
l5mgdai ly,bisoprolol5mg, andat orvastati
n40mgdai l
y.
Hei ssomewhatanxi ousappear i
ng,althoughnoti ndistress.Hi sbloodpr essurein
theemer gencydepar tmenti s90/65mm Hgandhear tratei s135bpm.Thel ungs
arecl earwi thnoper ipheraledema.Thepr esenti
ngel ectrocardiogram (ECG)i s
showni nFigure1.

Whi
choft
hef
oll
owi
ngi
sthenextbestst
epi
nmanagement
?

a.Per
form car
oti
dsinusmassage.
b.Admini
steri
ntr
avenousadenosi
ne.
c.Per
form synchroni
zedcar
dioversion.
d.Admini
sterint
ravenousi
buti
li
de.
e.Admini
sterint
ravenousmetoprolol.

Answer

C.Per
for
m synchr
oni
zedcar
diover
sion.

The ECG shows wi de compl ex tachycardia wi th atrioventri


culardi ssoci
ation
diagnost i
cofvent r
iculartachycardia(VT) .Thepat i
entishemodynami callystable,
althoughsomewhathypot ensiveandanxi ous.Thepr iorinfarctandVTmor phology
i
ssuggest iveofascar -
basedVT.Car dioversioni sthebestopt i
onf orthispat i
ent
(althoughami odaroneorpr ocai
nami demaybeconsi deredaswel l)
.I buti
li
dei s
appr opri
atef orat r
ialfibri
ll
ationorat rialfl
utter,whi l
ecar oti
dsi nusmassageor
adenosi newoul dbeappr opriat
eforsupr avent r
iculartachycar di
a.Met oprololalso
woul dbei neffect
iveandpot enti
all
yhar mfulgivent hepat i
ent’shypot ensivestate.

A 44- year-old woman i sreferr


ed to yourcli
nicforr ecurrentpal pi
tat
ions.The
episodesar esporadic,buthaveincreasedinfr
equencyint hepast12mont hs.Most
aresel f-l
imit
ing,butt helastepisodelastedhoursandr esolvedonl yafterbeing
given 6 mg ofi ntr
avenous adenosine inthe emer gency depar tment.She i s
otherwisei ngoodheal thandi staki
ngnomedi cat
ions.Herbl oodpr essur
ei nclini
c
i
s110/ 70mm Hgandhear tratei
s60bpm.Thecar di
acandr espirat
oryexami nat
ion
i
sunr emar kable.Herechocar di
ogram showednost r
ucturalhear tdi
sease.Thel ead
IIrhythm st ri
psf rom theemer gencydepart
mentpr e-and post -adenosinear e
showni nFigure1.
Whi
choft
hef
oll
owi
ngi
sthemostl
ikel
ydi
agnosi
s?

a.At
rialtachycar di
a.
b.At
rioventricularnodalre-ent
ranttachycar
dia(
AVNRT)
.
c.Ort
hodr omi cAVr e-entr
anttachycardi
a.
d.At
rialfl
utter.
e.I
nappr opri
at esinustachycardi
a.

Answer

B.At
ri
ovent
ri
cul
arnodalr
e-ent
rantt
achycar
dia(
AVNRT)
.

The rhythm str


ip shows a supraventri
cul
art achycardi
a.Compar ing t
he QRS
morphologyduri
ngt achycar
diawiththatinsinusr hyt
hm,t henegati
vePwaveat
theterminalpor
tionoftheQRScanbeeasi lyseen.Asuper i
orl
ydirect
edP-wave
vect
orrulesoutinappr
opriat
esinustachycar
dia.Thepr oxi
mityofthePwavetothe
QRS( shor
tRPtachycar
dia)excl
udesort
hodromicAVr e-ent
ranttachycar
dia.The
t
erminati
onofthetachycardi
awi t
hadenosinedecreasesthelikel
ihoodofatrial
t
achycardi
aorat
ri
alfl
utt
er.Themostli
kel
ydiagnosi
sisAVNRT.

75.A 32- year-oldwomani sat18weeksgest ationofhersecondpr egnancyand


present stot heemer gencydepar tmentwi t
h an epi sodeofpal pit
ati
ons.Shehad
mul tipl
eepisodesdur i
ngherf i
rstpregnancy,buttheywer eshor t
-li
vedandshedi dnot
seekmedi calattentionfort hem att hetime.Sheisot herwiseingoodheal thandisonl
y
takingpr enatalvitami nsuppl ement s.Herbloodpr essurei s100/60mm Hgandhear t
ratei s140bpm.Ot herthant het achycardia,
thecardi acandrespiratoryexaminat
ioni
s
unremar kable.Herel ectr
ocar diogram isshowni nFigur e1.Shei sgivenadenosine12
mgi ntr
avenous, andt her hythm termi nat
es.

Which oft
hef
oll
owi
ng i
sthebestwayt
o pr
eventf
utur
erecur
rencesdur
ing her
pr
egnancy?
a.Atenol
ol.
b.Dil
ti
azem.
c.Amiodarone.
d.Metoprol
ol.
e.Verapamil
.

Answer

C.Met
ropol
ol

The electrocardiogram shows a supr aventriculartachycardia.Forpr ophylacti


c
therapy,fi
rst-
li
net herapiesincl udemet opr ol
olandpr opranolol
.Digoxinisregarded
assaf etotakei npregnancy, althoughi tsef f
icacyhasnotbeendemonst rat
edi nthe
prophylaxisofsupr aventr
iculart achycardia.Ami odaronei sacat egoryD dr ugin
pregnancyandshoul dbeavoi ded.At enololisal soregardedasacat egoryDdr ug.
Verapami lcarri
esacl assIIbi ndicati
onf orpr ophylaxis.Flecai
nideandsot alolcan
beusedsaf el
yinpr egnancy( classI I
a).

76.Thepat ientisa65- year-ol


dmal ewhopr esent st otheemer gencydepar tmentfor
evaluation ofsyncope.Thepat i
entwaswal king i
n hisnei ghbor hood when he
suddenl y became di zzy and di aphor
eti
c and passed out . He r egai
ned
consciousnessspont aneouslywithin30seconds.Uponpr esent ati
on,hehadno
compl aints.Fouryear sago,hesuf feredaninferiormyocar di
alinfarct i
on.Anuclear
myocar dialperfusionscan1mont hagoshowedal eftventr
icularej ecti
onfr
action
(EF)of40%,anonr eversi
bledefectoftheinferiorwall,andnoi schemi a.Hisother
pasthi storyincludeshyper tensi
on.Pr i
ortot hesyncopalevent ,hehasnotnot ed
anychangei nhisexer cisetoler
anceandwasf reeofangi na.

Onexami nat i
on, hi
shear tr
ateis65bpm, hisbloodpr essureis112/ 74mm Hg, and
heisaf ebr i
le.Hisj ugularvenouspr essur eis7cm,l ungsar ecl
eart oauscul tation,
and car diac exami nation revealsar egularr hythm and nor malS1- S2 
without
mur mur s.Hi selectrocar di
ogram showssi nusr hythm at70bpm,wi t
hnor malPR,
QRS,andQTdur ation,andQwavesseeni nleadsI ,I
I,andaVF.Hi smedi cations
consistofaspi r
in81mgperday, atorvastatin20mgperday, carvedilol12.5mgbi d,
andl i
sinopr il10mgperday.Hi sbl oodwor kshowsnor malval uesf orcompl et
e
blood count ,electrol
ytes,and r enalf unct i
on.Car diacbi omarkerswer enor mal .
Echocar diogram i nt heemer gencydepar tmentshowednochangei nEFandno
si
gnificantval vulardisease.Over ni
ghtt elemet r
yr evealsnormalsinusr hythm.
Which oft
hef
oll
owi
ng i
sthemostappr
opr
iat
est
ep t
o eval
uat
ethi
spat
ient
's
syncope?

a.A48- hourHoltermoni tor.


b.Tiltt
abletesti
ng.
c.Elect
rophysiologicstudy(EPS)
.
d.Exercisestr
esst esti
ng.
e.Implantedloopr ecorder.

Answer

C.El
ect
rophysi
ologyst
udy(
EPS)
.

Thispat ienthassi gnifi


cantst r
ucturalheartdiseasewi t
hpr i
ormyocar dialinfarction
andr educedl eftventricularfuncti
on.Afteranegat i
veischemi aeval uation,apat ient
withcor onar
yar t
erydi sease( CAD)andsyncopeshoul dunder goEPS.EPSt est
sinusnodef unctionandat ri
oventri
cularconduct i
onar eal soexcel lentt estst o
i
dent i
fyvent ri
culart achyar r
hythmiasandr i
skofdeat hinpat ientswi thCAD and
syncope.Pat ientswi thCAD,syncope,andi nduciblevent r
iculart achycardiaonEPS
should be t reated wi thi mplantable cardioverter
-defibr
ill
ator.A Hol terorl oop
moni torcanbeusedi fanEPSi snegat i
veandadef i
nit
ivedi agnosisi sr equired.
Thispat ienthadaver yr ecentassessmentofi schemiaandEFbynucl earst udy.Ti l
t
tablet esti
ngi snoti ndicatedi nthissettingast hispat i
enthasst ructuralhear t
disease, andtherei snoevi denceforvasovagalsympt oms. 

77.A 28- year -oldmanwi t


hhyper tr
ophiccar diomyopat hy (HCM)pr esentstocl i
nic
aft
ermovi ngt othearea.Hewasdi agnosedatage19duet odyspneaonexer t
ion.
Hedeni eschestpai n,palpit
ations,near-syncope,orsyncope.Hi sf amil
yhi storyis
negativef orsuddendeat horHCM.Hei sanact ivef at
heroft wo,butdoesnot
exerci
se r out i
nely.He has been managed wi t
h met oprololXL 25 mg/ day.
Examinat ionshowsawel l
-appear i
ngman,bl oodpr essur e(BP)124/ 70mm Hg,
heartr ate72bpm,andnor malj ugularvenouspr essure.Hear trhythm isr egular
,
withanor malS1  andS2,anS4,andasof tgrade2/ 6hol osystolicmur muratt he
lowerl eftst ernalborder;extremiti
esar ewi t
houtedema.Hi sechocar diogr
am l ast
yearwasst ablefrom 3year spr i
orandshowedanej ectionf r
act i
onof82%, maximal
septalwal lt hicknessof26mm,l eftvent
ricular(LV)out fl
owgr adientof20mm Hg,
andmoder at emi t
ralregurgit
ation.AHol termoni t
orwasper formed3mont hsago
withnoar rhythmi asseen.

Whi
choft
hef
oll
owi
ngi
sthenextbestmanagementst
epf
orhi
scar
e?
a.Exercisestr
esst esti
ng.
b.Endomyocar dialbiopsy.
c.Incr
easemet opr olol
.
d.Cardiaccomput edt omographyscan.
e.Genetictest
ing.

Answer

A.Exer
cisest
resst
est
ing

Riskst ratif
icat
ionforsuddencar diacdeat h( SCD)i sessent i
alint hemanagementof
HCM,andi mplantabledefibril
latort herapyhasbeenshownt obeef fect
ivei nt his
popul ation.Inpat ient
swhohavenotexper iencedapr iorvent ri
cularar rhythmi c
event,t herear efi
vemaj orr iskf act orsforHCM- associatedSCD:f ami l
yhi storyof
SCD ( parti
cularl
yi fmul t
ipl
e event s)
,massi ve LV hyper tr
ophy ( >30 mm wal l
thickness) ,syncope,nonsust ainedvent riculart achycar dia,andanabnor malBP
responset oexercisewi t
hf ai
lur et oaugmentBP.Thi spat i
enthasnof amilyhi story
ofSCD,LVhyper t
rophywi th<30mm wal lt hickness,ar ecentnegat i
veHol ter,and
nohi stor yofsyncope.Ther eisaCl assI I
ai ndi cationforexer cisestresst esting to
assess f or an abnor mal BP r esponse. Wi t
h t he except ion of vent r
icular
tachycar dia/fi
bri
ll
ationandcar diacar r
est,eachi ndivi
dualmaj orr i
skfactorf orSCD
hasal ow positi
vepr edict
iveval ue.Ther i
sksofmul ti
plemar kersar enotcl early
addi t
ive. 

Int hei nter


nationalHCM- i
mpl ant
abl ecar diovert
er -def
ibri
ll
at or(ICD)r egi st
ry,the
numberofr iskfactorsdidnotcor r
elatewitht herat eofappr opr i
ateICDshocks.The
dat asuggestt hatthepr esenceofasi ngler i
skfact ormaybesuf f
icienttoconsi der
i
mpl ant
ationofanI CD, butt hedecisionshoul dbei ndivi
dualizedwi threspectt oage
oft hepatientandst r
engt hoft heriskfactor.Sever alotherlesswel l-establishedr i
sk
factorsar eofincreasingi nterestandhavebeenassoci atedwi t
hani ncreasedr i
sk
ofSCDi nHCM.Thi sincludesl ategadolinium enhancementonmagnet icresonance
i
magi ng and sever albet a-myosi n heavy chai n and t r oponin T mut ati
ons.
Endomyocar dialbiopsydoesnotpl ayar oleint hemanagementordi agnosi sof
pat i
ents with HCM.Car diac comput ed t omogr aphy wi llnotpr ovide addi ti
onal
i
nf ormation.Thi s pati
enti s asympt omat ic,so t herei s no need f orincr eased
met oprolol.

78.A75-year-
oldmani sseenint heemergencyr oom complai
ningofpal
pitat
ions.He
hasahistor
yofhypertensi
onandcompl et
ehear tblockforwhi
chheunder wentdual-
chamberpacemakerimplantat
ion2year sago.Hi spacemakerhasbeenpr ogrammed
totheDDDmode50- 120bpm.Fi gure1istheelectrocar
diogr
am (ECG)obt
ained. 

Anint
ervent
ionisper
formed,wit
hresol
uti
onoft hepati
ent
’ssympt
oms.Thesecond
ECGshowninFigur
e2i sobt
ainedaf
tert
heint
ervent
ion.
Based on t
he ECG shown in Fi
gur
e 2,whi ch oft
he f
oll
owi
ng i
nter
vent
ions was
per
for
medi nor
dertoall
evi
atethepat
ient
’ssymptoms?

Fi
gur
e1.

Fi
gur
e2.

a.Admi
nist
eri
ngadenosi
ne.
b.I
ncreasi
ngt
heatr
ialsensitivi
ty.
c.I
ncreasi
ngt
heventr
icularpaci ngout
put
.
d.Pr
ogrammingt
othesi ngle-chambervent
ri
cul
arpaci
ngVVImode.
e.Pr
ogrammingt
ohyst eresisoff.

Answer

D.Pr
ogr
ammi
ngt
othesi
ngl
e-chambervent
ri
cul
arpaci
ngVVImode.

Thef irstt r
aci ng demonst ratesvent ri
cularpacing at110 bpm,consi stentwi th
tracking of ei ther a supr aventri
cular tachycardia or pacemaker -medi ated
tachycar dia.I nt hesecondt racing,theratehasbeendecr easedt o50bpm wi than
under lyingat rialtachycar di
aat110bpm.Ther eisnor elati
onbet weent hePwaves
andpacedQRSmor phol
ogy, consistentwithreprogrammi ngt hedevi cet oasi ngle-
chambervent ri
cul arpacingmode.Adenosi necausest ransientat r
ioventri
cul ar( AV)
block,whi chwi l
lnotaf f
ectpacemakerf unct
ion,andt herefore,wi l
lnotr esol vet he
rhythm.I ncr easi ng the vent ri
cularpacerout putis noti ndicated because t he
pacemakerhasadequat event ri
cularcapt ur
e.Increasingt heat rialsensi t
ivitywi l
l
notr esol vet her hythm becauset herei sno evi denceoff ailuret ot rackat rial
rhythms.Hyst eresis promot es intri
nsic acti
vit
yl owert han t he nor malr ate by
allowing sl ow but appr opriate intr
insic rhythms. Changi ng t he hyst eresi s
par amet er swi llnothel presolvet hetachycardia.

79.An81- year-oldmani sseenintheemer gencyr oom forinter


mi t
tentpal pi
tat
ions.He
al
sonot esoccasi onalperi
odsofdizzinessandpr esyncopeimmedi atelyaf t
eraper i
od
ofpalpit
ati
ons.Hehasnochr onicmedi calil
lnessesandt akesonlyamul t
ivit
amindaily.
Onphysi calexami nati
on,hi
sbloodpr essurei s160/ 80mm Hgandpul sei s130bpm.
Hisrhythm isirregular
,wit
hapar adoxicall
yspl i
tS2.Ther emainderoft heexami nati
on
i
sunr emarkable.Hi sadmissi
onel ectrocardiogram dur ingpalpit
ationsi sshowni n
Fi
gure1.  

Af
ter10mi nut
es,thepalpi
tat
ionssubside,buthebecomesl ightheadedandnear
ly
l
osesconsci
ousness.Fi
gure2isthesecondelect
rocar
diogr
am obtained.

Fi
gur
e1.
Fi
gur
e2.
Whi
choft
hef
oll
owi
ngi
sthenextst
epi
nmanagementt
orel
ievehi
ssympt
oms?

a.I
nit
iati
onofmet opr
olol.
b.I
nit
iati
onofsotalol
.
c.Permanentpacemakerimpl ant
ati
on.
d.I
nit
iati
onofmet opr
ololaft
erpermanentpacemakeri
mplant
ati
on
e.Atr
iovent
ri
cul
arnodeabl ati
onafterper
manentpacemakeri
mplant
ati
on.

Answer

D.I
nit
iat
ionofmet
ropol
olaf
terper
manentpacemakeri
mpl
ant
ati
on.

Thepat ienthasparoxysmalatr
ialfi
bril
lat
ion(AF)witharapidventr
icularresponse
andsi nusnodedysf unct
ion(t
achybr adysyndrome).Thebesttreat
menti sabet a-
bl
ockert ocontroltherat
eduringAFandapacemakert opreventsignifi
cantsinus
bradycardia, which may wor sen af t
er admi ni
str
ati
on of a bet a-blocker.
Anticoagulat
ionalsoshouldbeiniti
atedtopreventstr
oke.Administ
rati
onofsot alol
andmet oprol
olmaydecr easehisepisodesofAFandpal pit
ati
ons,butwil
lli
kel
y
worsenhissinusnodedysf uncti
on.Paci
ngwi l
lunli
kel
ysigni
fi
cantl
yaff
ecthisAF
burden.Atri
alofpharmacologi
ctherapytocont
rolhear
trateshoul
dbeconsider
ed
pri
ortoatri
ovent
ri
cularnodeablati
on.

80.A 65- year-oldmanwi thasi ngle-chamberi mplantablecar dioverter-def


ibril
lat
or
(I
CD)i mplantedf orpri
mar ypr event i
onofsuddendeat h6mont hsagocomest othe
emer gencydepar t
menttodayaf terhavingashockf r
om hi sdevice.Hehadbeenf eeli
ng
palpitati
onsal lduri
ngtheday, andt henwhenhebr i
sklywal kedupt wof l
ightsofst airs
hef elttheshock.Hedeni esanyshor tnessofbr eathorchestdi scomf or t
,norsyncope.
Hehasahi storyofsystolichear tfail
ur eresulti
ngf r
om anant er
iorwal lmyocar dial
i
nfar cti
on5year sago,withal eftvent r
icular(LV)ejecti
onf racti
onof25%, andisabl eto
domostact i
viti
esofdailyli
vingconsi stentwi t
haNewYor kHear tAssoci ati
onclassI I.
Hisot hermedi calpr
oblemsi ncl udepar oxysmalat ri
alfibri
ll
ati
on( AF) ,hyperli
pidemi a,
andhyper tension.Hismedi cationsar easpi r
in81mgdai ly,dabigatran150mgt wice
dail
y, carvedil
ol25mgt wi cedai l
y, l
isi
nopr il20mgdai ly,andat orvastatin40mgdai l
y. 

Uponpr esent
ati
on,hi
sweightis70kg,hi
sbloodpr essureis105/60mm Hgwi t
ha
heartrat
eof70bpm, andhi
sICDsit
eiswel
lhealedinthelef
tchest
.Hi sj
ugul
arvenous
pressureisat6cm H2O,l ungsoundsareclear,andheartexami nat
ioni
snor mal
withoutmurmur.
 

His electrocardi
ogram shows si nus rhyt
hm at 70 bpm, wi th a nonspeci fi
c
int
raventri
cularconducti
ondelayandQRSdur at
ionof135msec.Inter
rogationofhis
ICDisper formed.HisICDi sprogrammedwi t
htwozones:aVTzonest ar
tingat170
bpm treatedwi t
hful
lenergyshocks, andaVFzonestart
ingat200bpm t
reatedwithfull
energy shocks.The I CD shock del i
ver
ed t
oday was fora rhyt
hm ofAF,wi tha
ventri
cularrateof173bpm.

Whi
choft
hef
oll
owi
ngi
sthenextst
epi
nmanagement
?

a.Upgradet oadual -
chamberICD.
b.Addantitachycar
diapacingtotheVTzone.
c.Adddilt
iazem 120mgdai ly.
d.Incr
easecar vedi
lolto50mgt wicedai
ly.
e.Eli
minatetheVTzoneonhi sICD.
Answer

E.El
imi
nat
etheVTzoneonhi
sICD.

Thispat ienthadani nappr opr i


ateshockf orAF,occur r
ingdur ingexer ci
se.The
MADI T-RI T (Multicenter Aut omat i
c Def ibril
lat
or Impl
antat i
on Tr i
al- Reduce
Inappr opriateTher apy)t rialinvest i
gatedast rategyofasi nglezoneat200bpm
compar edwi t
hconvent ionalpr ogrammi ng( VTzoneof170bpm andVFzoneat200
bpm) .(Thet ri
alalsoincl udedadel ayed- t
her apyarmthatincludedaVTzonewi tha
60- seconddel ayeddet ection).MADI T-RITshowedt hatthesi ngle-zoneprogram
strategyr esul
tedi nar educt i
oni ninappropr i
atether
api
esandi mpr ovedsurvi
val.
Based on hi swei ght,hei satt hemaxi maldosef orcar vedi
lol.Given hi
sLV
dysf unction,hei sapoorcandi dat
ef ordi lti
azem.Anti
tachycardiapaci ngwould
havenoi mpactonAFwi thar apidr at
e,andupgr adeofhisdevi ceisnotneeded.
Upgr adet oadual -chamberdevi cehasnotbeenshownt odecr easet henumberof
i
nappr opriateshocks.

81.A76- year-ol
dmani sseeninclinictodayt odi
scusshisupcomi ngneedt oundergo
replacementofhi spacemakergener ator.Hehasadual -
chamberpacemakert hat
wasi mpl anted8year sagoforcompl eteheartblock,andt hegener at
orhasj ust
reachedi tsalertforrepl
acementl astweek.Hef eel
swel landhasnocompl ai
nts.
Hispastmedi calhi
storyalsoincludeshyper t
ension,type2di abetes,par
oxysmal
atri
alfibri
ll
ati
on, andapriorst
roke2mont hsagowi t
hfullrecovery.Hismedicati
ons
i
ncludel isi
nopr i
l,metf
ormin,met opr
ol ol,andwarfari
n.Hi sinter
nat i
onalnormali
zed
rati
ousual l
yr unsintherangeof2. 2-2.8.

Whichoft
hefol
lowi
ngishowanticoagul
ati
oni
nthi
spat
ientshoul
dbemanagedf
or
hi
supcomingpacemakerpr
ocedure?

a.Continuewar f
ari
nthroughouttheper i
procedureper i
od.
b.Holdwar far
in5dayspr i
orandr esumeaf tertheprocedur e.
c.Replace warfari
n with aspir
in( 325 mg dai ly)starti
ng 5 days pri
orto
procedure.
d.Holdwar far
in5dayspr iortoprocedureandbr idgewi thenoxapari
nunti
l
i
nternati
onalnormali
zedr ati
ois>2postpr ocedure.
e.Replacewar f
arinwit
hdabi gatr
an( 150mgt wicedaily)start
ing5dayspri
or
toprocedure.
Answer

A.Cont
inuewar
far
int
hroughoutt
heper
ipr
ocedur
eper
iod.

Inpat i
entsundergoingdevi cepr ocedureswi thant i
coagul ation,thereist heconcer n
fori ncreased r
isk ofbl eedi ng.However ,cont i
nuing war f
ari
nt hroughoutt he
periprocedureperiodcarri
esal owerr iskforbl eedingcompar edwi t
hast r ategyof
withholding ant
icoagulati
on and br i
dging wi th enoxapar in.BRUI SE CONTROL
(Bri
dgeorCont inueCoumadi nf orDevi ceSur geryRandomi zedCont rol
ledTr ial
)
randomi zedpati
ent stobot hstrategies,butwasst oppedear l
yfort hefindingoft he
super i
orit
yofmai ntai
ningwar farin.Mai nt
ainingant icoagulat i
onovert hepr ocedure
periodi spref
err
edgi vent hispat ient
’shighr iskforst roke( CHA2DS2- VASc) .Whi l
e
ther ecentBRIDGE( Eff
ectivenessofBr i
dgingAnt icoagul ationf orSur gery)st udy
showed no benef i
tt o br i
dging over wi thholding ant icoagul ants dur ing the
perioperati
veperiod,pati
ent swithahi storyofst r
okewi t
hint hepr ior12weekswer e
excluded.

82.A65- year-ol
dwomanpr esent sforf ol
low- upintheheartfai
lur
ecl inic.Shei sabl e
todoherusualact ivit
ies, butislimitedwhent r
yingtowalkupstairs.Shedeni esany
chestpai n.Herpastmedi calhistor
yi ncl
udessyst ol
icheartfai
lure( l
ef tvent
ricular
ejecti
onf ract
ion30%)f ort hepast3year s,coronaryarter
ydiseasewi thanant erior
wallmyocar dialinfarction 5 year s ago,t ype 2 diabet
es,hyper l
ipidemia,and
hypertension.Hermedi cat ionsincludel i
sinopri
l20mgdai l
y,carvedilol25mgbi d,
atorvastatin40mgdai ly,aspi ri
n81mgdai l
y,andmet for
mi n850mgbi d.On
exami nati
on,herbl ood pr essureis100/ 70mm Hg and pul sei s70bpm.Her
exami nati
on is otherwi se unr emar kable and euvolemic.Herel ectrocardi
ogr am
showssi nusr hyt
hm at70bpm,wi thal eftbundlebranchblock( LBBB)andQRS
durationof145msec.

I
naddi ti
ontor
efer
ri
nghertocardi
acr
ehabi
li
tat
ion,whi
choft
hef
oll
owi
ngi
sthe
nextbestst
epi
nthi
spat
ient
'scar
e?

a.Destinati
onl eftventri
cul
arassi
stdevi
ce.
b.Referforcardiact ranspl
ant
ati
on.
c.Bi
vent ri
cularpaci ng.
d.Continuecur renttherapy
Answer

C.Bi
vent
ri
cul
arpaci
ng.

Theupdatedgui deli
nesf orbivent
ri
cularpaci
ngspecifythetypeofQRScompl ex,
QRSdur at
ion,andNew Yor kHeartAssociat
ion(NYHA)f unct
ionalcl
ass.ClassI
i
ndicat
ionforbiventri
cularpacingisforaQRSofatl east150msecandanLBBB
andaNYHAcl assofI I
,III,orambulat
oryIV.Thispat
ienthasacl assI
Iaindi
cati
on
wit
hclass2hear tfai
lur
e, LBBB,andQRSdur ati
onof120-149.

83.A74- year-
oldwomanwi thhyper tensionpresentstoclini
cf orevaluati
onofnew
sympt omsofpal pi
tat
ions.Shedescr ibesthissensati
onasaf eel
ingofherhear t
skipping a beatand occasi onall
yr aci
ng.She does nothave chestpai n or
discomfort,
butdoeshavedyspneawi thmorepr ol
ongedepisodes.Episodesar enot
tri
ggeredbyact ivi
tyorexerciseandmostof tenoccurafterdinner.Shehasnothad
syncope.Epi sodesoccursever alt i
mesamont handl astf oraf ew mi nut
est o
severalhours.Herbl oodpr essurei smanagedwi t
hlow- doseat enol
ol.Shehasa
familyhist
oryofst r
oke.Herelectrocardiogr
am isshowni nFigure1.
Whi
choft
hef
oll
owi
ngi
sthenextbestst
epi
nhermanagement
?

a.Stresstest
.
b.Eventmoni t
or.
c.Increaseat
enolol
.
d.Reassurance.
e.Holtermonitor
.

Answer

B.Eventmoni
tor

When eval uati


ng palpit
ati
ons,the historyi sfundamentalbecause i tenables
assessmentofbot hthediff
erenti
aldiagnosisofsympt omsandt hepossi bil
it
yof
struct
uralheartdiseasethatputthepatientatarrhythmicr
isk.Palpit
at i
onscanbe
ofcardiacorigi
n(supraventr
icul
arorventri
cularectopyorar
rhythmia, valvul
arheart
diseaseorcar diomyopathy),psychosomat i
c( anxiet
yordepr ession),neurologic
(autonomi cner voussyst em dysf unction/neur ocardi
ogenic),dr ug-i
nduced,ordue
toot herpat hology( thyroiddi sorderoranemi a).Evaluati
onofsympt omatology
requiresassessmentoft heimpactofsympt omson t hepat i
ent
’slif
eand t he
potentialofriskduet ost ructur
alhear tdiseaseorar r
hythmi a.Inthiscase,because
thepat ienthasr iskf actorsforhear tdi sease,correl
ation ofsympt omswi tha
potentialarrhythmi aiscr iti
cal
,andr eassur ancewi t
houtf urtherevaluati
onwoul d
notbeopt i
mal .Increasingat enololwoul dbedi ff
icul
tduet obasel i
nebradycardia.
Whendeci dingbet weenusi nga24- hourHol terandaneventmoni tortocorrelate
sympt omswi thapot entialarrhythmi a,thef requencyofsympt omsi simportant.
Over al
l,ambul atoryeventmoni t
oringof2- 4weeksdur ati
onhasbeenshownt o
provideahi gheryieldofdi agnosist hanHol t
er.
 

84.An82- year -
oldwomanwi t
hahi st
orynoni schemiccar diomyopat hyandNew Yor k
HeartAssoci ati
oncl assI Ihear tfai
lur
eisseeni nyourof fi
cef orinter
mi tt
entepisodesof
malaiseandaf ull
nessi nherneck.Shehadapr imarypr eventi
onsi ngle-chamber
i
mplantabl ecadi overter-defi
br i
ll
ator(
ICD)impl anted2year sago, whi chisprogrammed
atVVI40wi t
hasi nglet achyar r
hythmiazoneat200bpm t r
eat edwi thshocks.Device
i
nterr
ogat i
ondemonst rat
esnor maldevicefunct i
onwi t
hnoar r
hythmi asrecorded.Her
medicationsi ncludel i
sinopril40mgdai l
y, f
urosemide40mgdai l
y,andspi ronolact
one
25mgdai ly.Shehaddi scontinuedmet oprolol1yearagoduet oincreasingfati
gue.Her
examinationi sunr emar kableexceptf ortr
acepi tt
ingedema.Aneventr ecor
dingdur i
ng
anepisodewhi lewal kingtot hebat hr
oom isshowni nFigure1.
Whi
choft
hef
oll
owi
ngi
sthenextmostappr
opr
iat
est
epi
nhermanagement
?

a.Upgradetodual-chamberI CD.
b.Addcarvedi
lol6.
25mgdai ly.
c.Upgradetocardi
acr esynchroni
zationtherapydefi
bri
ll
ator(
CRT-
D)syst
em.
d.I
ncreasefur
osemi deto80mgdai ly.
e.Cat
heterabl
ationofpr ematureventri
cularcont
racti
ons.

Answer

A.Upgr
adet
odual
-chamberI
CD.

Thispatientcomplai
nsofmal aiseandneckf ull
nessdueinpar ttothelossofat r
ial
synchronywhenvent r
icularpacingisat40bpm.Themostappr opri
ateint
erventi
on
i
supgr adingherdevi
cet oadual -chambermodet oall
owat r
ioventr
icul
arsynchrony.
Beta-bl
ockademayal sobei nit
iatedaf teranatr
iall
eadisinsertedfori
tsbenefit
sin
pati
entswi thhear
tfail
ure,buti finit
iatedbeforeatri
all
eadi nsert
ion,wouldlikely
worsensympt omsduet ogreaterpaci ng.Shedoesnothavesi gnsofheartfail
ure,
soincreasi
ngdiuret
icsatt
histimewoul
dnotbeappr
opr
iat
e.Shedoesnotmeet
cr
it
eriaforCRTgivenQRSdurat
ion.

85.A22- year-oldmanwi tharrhythmogeni crightvent r


icularcardi
omyopat hypresentsto
cli
nictoestabli
shcare.Hisdiagnosiswasmadebasedont hefindi
ngsofsympt omatic
nonsustainedventr
icul
artachycardia,anel ectrocardiogram withT-wavei nver
sionsin
leads V1-V3,and a cardiac magnet icr esonance i maging with an el
evated ri
ght
ventri
cularend-di
astol
icvolumeandr egionalhypoki nesis.Hesubsequentlyunderwent
implantati
on of an i mplantabl
e car di
overter-defibri
ll
ator
. Dur i
ng discussion of
managementofhi sconditi
on, heasksf oradvicer egar di
nglif
est
yleinter
venti
ons.

Whi
choft
hef
oll
owi
ngl
if
est
ylemodi
fi
cat
ionsdoyousuggest
?

a.Li
mitcaffei
ne.
b.Li
mitcompet i
ti
vesport
spar
ti
cipat
ion.
c.Avoi
dai rtr
avel
.
d.Li
mitalcohol.
e.Rest
ri
ctf l
uidi
ntake.

Answer

B.Li
mitcompet
it
ivespor
tspar
ti
cipat
ion

Arr
hyt hmogeni c car diomyopat hyi s a pr ogressive,herit
able disease ofcar diac
muscl er esultingi nahi ghriskofvent riculararrhythmiasandsuddendeat h,even
when car diacst ructuralchangesar emi nimal.Based on t hei ncr eased riskof
cardi
acar restandsuddendeat hinat hl
eteswi t
har rhyt
hmogeni ccar diomyopat hy,
pati
ent sar eadvi sedt oavoidcompet iti
vespor tsandendur ancet raining.Physi cal
acti
vitymayaccel eratestructuralpr ogressi onofar rhyt
hmogeni ccar diomyopat hy;
i
n a st udy ofhet erozyogous pl akoglobi n- defici
entmi ce,endur ance training
accel
er ated t he devel opmentofar rhythmi as and r i
ghtvent r
icul ardysf unct i
on.
Alt
houghcaf feine,alcohol,dehydr ation,andmi ldhypoxiamayt r
iggerar rhyt
hmi ain
i
ndividual s,therear enoknownspeci fi
clinkst oarrhythmogeniccar diomyopat hy.

86.A40- year-ol
dwomani sreferredforeval uat
ionofpal pit
ations.Shehasf eltski
pped
andirr
egularbeat sforthepast20year s,butint hepast6mont hs,sheisfindingthat
herexerci
setolerancei sdecr eased.Shedeni esanychestpai norsyncope.Shehasa
pastmedicalhistoryofobesi ty.Shesawacar diologi
st1yearagof orthi
sproblem and
remembersget ti
ngast r
essechocar diogram,butshewast ol dthepicturesont hetest
wereunclear;acar di
accat heteri
zationwasr ecommended,butshedecl i
ned.Fami ly
hist
oryisnotablef oranuncl ewhodi edofar hythm problem thatoccur r
eddur inga
hospi
tal
izat
ion20year
sagof
orast
aphyl
ococcusi
nfect
ion.
 

Onphysicalexaminati
on,herbl oodpr essur
eis123/
70mm Hg,pul seis60bpm,and
weighti
s170pounds( bodymassi ndex30).Thej
ugularvenouspul
seisat6cm,wit
h
occasi
onalcannonAwavespr esent.Lungsoundsar eclear
.Auscul
tat
ionshowsa
si
ngleS1,si
ngleS2,andfrequentectopicbeats.
 

Ther emai
nderoftheexaminat
ioni
snor
mal
.Her12-
leadel
ect
rocar
diogr
am (
ECG)
rhyt
hm str
ipi
sshowninFigur
e1.
Whi
choft
hef
oll
owi
ngi
sthenextbestst
epi
nmanagement
?

a.Str
essnucl earperfusi
onstudy.
b.Loopr ecor
deri mplant.
c.Cardiacmagnet icresonancei
magi
ng(
MRI
).
d.Cardiaccatheteri
zati
on.
e.Thir
ty-dayeventmoni t
ori
ng.

Answer

C.Car
diacmagnet
icr
esonancei
magi
ng(
MRI
).

This ECG shows pr emature vent ricular cont racti


ons ( PVCs) of mul ti
ple
mor phologies.PVCsshow ar i
ghtbundl emor phol ogy(namel yposit
iveinV1) ,but
wi t
hdi ff
erentampl i
tudesint hel atepr ecordi
all eads.ThePVCsar el i
kelyar i
sing
from theleftvent ri
cle,butfr
om di ff
erentlocations.I nawomanwi thnor iskf actors
forcor onarydi seaseandal ong- standinghi storyofsympt omsl i
kelyr elatedt o
thesePVCs,oneshoul dfi
rstconsi deracar diomyopat hy.Asanechocar diogram
waspr evi
ousl yr egardedassubopt imal,thebestnexti magingst udyisacar diac
MRI .Thispat ient’scardiacMRIr evealedl ef
tvent ri
cularnoncompact i
on.Anevent
moni t
orori mpl antableloopr ecorderi snothel pfulinthi
scase,ast hef r
equent
PVCsar ealreadyevi dentonherof fi
ceeval uati
on.

87.A 55- year -old woman pr esent st ot he emer gency r oom wi th 3 mont hs of
progr essiveshor t
nessofbr eathandbi l
aterallowerext r
emi tyswel l
ing.Shehasa
pastmedi calhi storyofhyper tensi onandat ri
alfi
brill
ation.Hermedi cationsi nclude
carvedi l
ol6. 25mgt wi cedai ly,lisinopri
l10mgdai ly,andhydr ochlorothiazi de12. 5
mgdai ly.Anucl earmyocar dialper f
usionscanper for med6mont hsagoshoweda
normall eftvent ricul
arej ectionf ractionwi thoutper fusiondef ects.Onexami nation,
herbl oodpr essur eis105/ 70mm Hg, herhear trateis135bpm, andshei saf ebril
e.
Herj ugul arvenouspr essurei s11cm H20;cr acklesar eaudi bleoverbot hl owerl ung
fi
elds,andcar diacauscul t
at i
onr evealsat achycardi candi rregularhear tr hythm.
Herl owerext r
emi ti
eshave2+ pi t
ti
ng edemabi lateral
ly.Herel ectrocar diogr am
showsat ri
alf i
br i
ll
ationat120bpm andl eftbundl ebr anchbl ock.Shei sadmi ttedto
thehospi tal,andanechocar diogr am revealsagl obal lydilat
edl eftventriclewi than
estimat edej ecti
onf ractionof15%.Ther ei snoevi denceofl eftatr
ialthr ombuson
transesophagealechocar diogr aphy.Shei sst art
edonahepar indrip,lasix40mg
dai
ly,andami odar
one200mgthr
eeti
mesdail
y,whi
lecar
vedi
loli
sti
tr
atedt
o12.
5
mgt wicedail
y.Sheconver
tst
osinusrhyt
hm wi
tharateof65bpm onday2of
hospit
ali
zat
ion.

Whichoft
hef
oll
owi
ngi
sthemostappr
opr
iat
enextst
epi
nthemanagementoft
his
pat
ient
?

a.Echocardi
ogram in3mont hs.
b.Multi
gatedacquisi
tionscan.
c.El
ectrophysi
ologystudy.
d.24-hourHolt
ermoni tor
ing.
e.Cardi
acmagnet icresonancei
magi
ng.

Answer

A.Echocar
diogr
am i
n3mont
hs.

Thispati
enthasat achycardi
a-inducedcardiomyopat hyduetoatri
alfi
br i
ll
ationwith
fastventr
icul
arrat es.Controlofat ri
alfibri
ll
ati
on ( orfastventr
icularr ates)is
expect
edt oresul
ti nnormalizati
onofl eftventr
icularfuncti
on.Theot herchoi ces
wouldnotchangemanagement .Alt
houghaneval uati
onisnotunreasonabl e,itcan
bedeferr
eduntilt
heef f
ectsofrestori
ngsinusrhythm havebeenr eassessed.

88.A55- year-ol
dmanisrefer
redforcar
diacmagnet
icresonancei
magi
ng(MRI)because
ofrecurrentri
ghtbundl
ebr anchmorphol
ogicnonsustai
nedvent
ri
cul
artachycar
dia
(Fi
gure1).

Fi
gur
e 1.
I
magesseenoncar
diacMRIar
emostconsi
stentwi
thwhi
choft
hef
oll
owi
ng?

a.Myocarditi
s.
b.Pri
ormyocar di
alinfar
cti
on.
c.Hemochr omatosi
s.
d.Nonischemiccardiomyopat
hy.

Answer

B.Pr
iormyocar
diali
nfar
cti
on

Car diacMRIi sausef ulmodal ityt o assessf orpossi blecausesofvent ri


cular
arrhyt hmia,gi veni t
sabi li
tytodemonst ratevariousmyocar dialdi
seasest hatform
thesubst ratef orarrhythmia.Thecl assicinfarctscarinvol vesthesubendocar dium
andmaybenont ransmur al,asshowni nthispat ientwit
hevi denceofpr i
ori nf
erior
wal lmyocar dialinfarctionbyl atepost -gadolinium enhancementi maging.Unl ike
i
nf arctscar ,thef i
brosisofnoni schemi ccardiomyopat hyismor eli
kelytoappearas
mi dmyocar dialhyper enhancement ,andmyocar di
ti
smayappearasmi dwal lor
mor ecommonl yepicardialhyperenhancement .Hemochr omat osi
sisdet ectednot
by l at e gadol ini
um enhancement ,butwi t
h a di f
fer
entnoncont rasti magi ng
techni quet hatt akesadvant ageoft heef f
ectofi rononT2* ,amagnet icrelaxation
par amet erthatshor tenswi thincreasedt i
ssueirondeposi tion.

89.A70- year-ol
dwhi temanpr esentstot heemer gencydepar tmentwi tha1- week
historyofprogressiveshor tnessofbr eath.Hedeni esanyf ever,sputum product i
on,
chestpai n,hemopt ysis,legedema,orpal pit
ations.Hehasahi st
oryofi schemi c
cardiomyopathyf ollowingt wopr iormyocar dialinfar
cti
ons5year sago.Hi sleft
ventri
cularejection f r
action is 20%,and he has an i mpl antable cardiovert
er-
defibri
ll
ator(I
CD).Duet or ecurrentventriculartachycardiaandI CDshocks,hewas
placedonami odarone2year sago, af
terwhi chhehadnof urtherICDshocks.Thr ee
mont hsago,hewasadmi ttedtot hehospi talfordecompensat edheartf ai
lure(HF).
Hiscur rentmedicalr egimeni ncludesmet oprololsuccinate100mgdai ly,enalapri
l
10mgbi d,f
urosemi de40mgbi d,ami odarone200mgdai ly,andmet f
ormi n1000
mgdai l
y. 

Onphysi
calexami
nat
ion,
heisi
nmoder
ater
espi
ratorydist
ress.Hi
sbl
oodpressur
e
i
s110/80mm Hg,hi spulsei
s110bpm andr egular
,andhei safebr
il
e.Lung
examinationreveal
sdi f
fusebil
ateralr
ales.Hisjugul
arvenouspr essur
eis8cm and
thereisno hepat ojugularref
lux.Car di
acexami nati
on r eveal
sa nor malS1 and
S2 wit
hnoS3  ormur mur.Hisabdomeni sbenign,andhi slowerextremiti
esar e
war m andfreeofedema.Labor atorytestsshowacr eatinineof1mg/ dl,N-t
erminal
pro–B- t
ypenat r
iur
eticpepti
de(NT- pr
oBNP)of300pg/ ml ,andhemogl obin13g/dl.
ChestX- rayreveal
sdi f
fusebil
aterali
nfil
tr
ates.Theelectrocardi
ogram showssinus
tachycardiaandisotherwiseunchangedf rom hi
sbaseline.

Whi
choft
hef
oll
owi
ngi
shi
smostl
ikel
ydi
agnosi
s?

a.Pulmonaryemboli
sm.
b.AcutedecompensatedHF.
c.Pneumonia.
d.Pulmonaryhemorrhage.
e.Pulmonaryfi
brosi
s.

Answer

E.Pul
monar
yfi
brosi
s.

Thepat ientpr esentswi t


hacl inicalpi
ctur econsi stentwithami odaronepul monar y
toxici
ty.Physi ciansneedt omai nt
ainahei ghtenedawar enessoft hispot ential
ly
fatalcompl i
cat i
on.Ot hertoxicitiesofami odarone,i ncl
udinghepat icinjur
y,t hyroi
d
distur
bances,phot osensit
ivit
y,ski n di scolorati
on,per i
pheralneur opat hy,and
cornealdeposi t
s,shoul dber emember edaswel l.Inthecur rentclini
calscenar i
o,
decompensat edHFi slesslikel ygivent heabsenceofj ugularvenousdi stentionor
hepatojugularr efl
ux and t he NT- proBNP l evel.Ther e wer e no const it
utional
sympt omsorsput um productiont osuggestpneumoni a,norwast herehemopt ysis
oranemi atosuggestpul monar yhemor rhage.Apul monar yembol uswoul dnotbe
expectedt oresul ti
nbi l
ater
alpul monar yi nfi
lt
ratesonchestX- ray.

90.A30- year-
oldmanpr esentstoyouf orcar
diol
ogyconsul
tati
on.Hewasor i
ginal
ly
eval
uatedbyhi si
nter
nistforchestpainandshort
nessofbreathwit
hexerci
se.His
i
nterni
stchecked an el ectr
ocardi
ogram,which demonstrated l
eftventr
icul
ar
hypert
rophy(LVH)wit
hst r
ainpatt
ern.
 

Thepati
entreport
sexert
ionalchestdiscomfortanddyspneaf orthepast3mont hs.
Hehasnopastmedi calhistoryandt akesnomedi cat
ions.Hisfatherdiedi nacar
acci
dentattheageof45,andhi smotherisali
veandwel l.Onexami nat
ion,hisheart
rat
eis76bpm andbl oodpr essur
ei s110/70mm Hg.Ther ei sal ate-peaki
ng
syst
oli
cejecti
onmurmurhear dbestatthelef
tuppersternalborder.
  
Echocardi
ogram r
evealsLVHwi t
hanLVsept um andpost
eri
orwal
lthi
cknessof17
cm and18mm,r especti
vely.TheLVeject
ionfr
actioni
s65%,andt
hecavitysi
zeis
normal.ThepeakgradientacrosstheLVoutf
lowtracti
s28mm Hg.

Whi
choft
hef
oll
owi
ngi
sthenextbestst
epi
nhi
scar
e?

a.Stressechocardiogr am.
b.Leftandrightheartcat het
eri
zat
ion.
c.Cardiacmagnet i
cr esonanceimaging.
d.Exerciset
olerancet est.
e.Transesophagealechocar diogr
am (TEE)
.

Answer

A.St
ressechocar
diogr
am

Thecl i
nicaldiagnosi
sofhyper tr
ophi
ccardiomyopat hy(HCM)i sconvent i
onal l
y
made mostcommonl y witht wo-di
mensionalechocar di
ography.Mor phologic
diagnosisisbasedont hepresenceofahyper trophiedandnondi l
atedLVi nt he
absence ofanot hercar di
ac orsyst emic disease capable ofpr oducing t he
magni t
udeofhyper tr
ophyevi dentinapatient( usuall
y≥15mm i nadul ts)
.Thi s
patient
,whohasnopasthi st
oryandnohyper tension,afamilyhist
oryconsi stent
withsuddendeat h,andsi gnifi
cantLVH onechocar diogr
am,hasapr esentation
consist
entwithHCM.
 
Identi
ficationoft hepr esenceandmagni tudeofout f
low obst r
uct i
oni sessent i
alin
guidi
ngpr ognosi sandt her apy.Thepr esenceandmagni t
udeofout f
lowobst ruction
are usual ly assessed wi tht wo-di mensi onalechocar diography and cont i
nuous
waveDoppl er.Itisal ate-peakingsyst ol
icvelocitythatr efl
ect st heoccur renceof
subaor ti
cobst r
uct i
onlatei nsyst ol e,andt hepeaki nstantaneousgr adientder i
ved
from t he peak vel oci
ty shoul d be r eported.A peak gr adient>30 mm Hg i s
consider edsi gnif
icant.Themeangr adientdoesnothavephysi ologicsi gnifi
cance
anddoesnotaf f
ectt reatment ,andt husi snotassessed.I ft her est ingout f
low
gradienti s≤50mm Hg,pr ovocativemeasur esmaybeusedt oascer taini fhi gher
gradients can be el icit
ed, pr eferably wi t
h physi ologic exer cise ( stress
echocar diography) ,butal t
ernati
vel ywi ththeVal salvamaneuverorsel ect i
velywi t
h
amylni trit
e.Int hi
spat i
ent ,provocat ivetestingisimpor tantbecauset hepat ienthas
sympt omst hatappearoutofpr opor t
iontot herestinggr adient ,andt her ef
or e,the
presence of a physi ol
ogical ly pr ovoked obst r
uct ion shoul d be
assessed.  Provocat i
onwi thdobut ami neinfusiondur ingDoppl erechocar diogr aphy
i
snol ongerr ecommendedasast rat egytoinduceout fl
owgr adi entsinHCM.  
TEEi susef ulint
raoper ati
velytoguideal coholsept alablati
onormyomect omyori n
caseswher ethereisconcer nformi tr
alval vepathol ogyorf ixedoutflowobst r uction
notwel lcharacteri
zedont ransthoracicechocar diogram.I nthi
scase,adequat e
i
nformat ionhasnotyetbeenobt ainedf rom at ranst horacicechocar diogram,and
thus,TEE i s notwar r
anted.I n equivocalcases,car diac catheterizati
on wi th
i
sopr oterenolinfusionmayf ur
therai dinel ici
ti
ngapr ovocabl egr adient.Invasi ve
cardiaccat heter
izationtodocumentout f
lowgr adient sisnecessar yonl ywhent here
aredi scordantdat af r
om Doppl erechocar di
ographyandt hephysi calexami nat i
on.
Furthermor e,coronar yangiographyi sindicatedonl yincaseswher et hereisahi gh
l
ikeli
hoodofcor onar yarter
ydi sease,whi chisnott hecasei nthisyoungpat i
ent
withnor i
skfactors.

91.A30- year-ol
dmanwi thahi storyofhyper trophiccar diomyopat hy( HCM)pr esent sto
youf oranopi niononani mpl antablecar diovert
er-def i
br i
ll
ator( I
CD)pl acementf or
suddencar di
acdeat h(SCD)r i
sk.Hewasr ecent l
ydi agnosedbyanot hercardiologist
.
Thepat ientreportsnosympt oms.Thedi agnosi swasmadeaf terhisinter
nisthear da
systol
ic mur muron exami nation,and an el ectrocar diogram ( ECG)showed l ef
t
ventri
cular(LV)hypertr
ophywi thst r
ainpattern.Heexer cisesbywal kingatabr iskpace
for30 mi nut
esdai l
ywi thoutl imit
ati
ons,and hewor ksf ullt i
measan i nsur ance
executive.Het akesnomedi cations.Hehasnopasthi story.Hi spar entsandt wool der
sibl
ingsar eali
veandwel l
.Ther eisnof amil
yhi st
oryofSCD.  

Onexami nation,hishear tr
atei s68bpm, andhisbloodpr
essur eis106/64mm Hg.He
hasahar sh,late-peakingsyst ol
icej
ectionmur murheardbestatther i
ghtupperst er
nal
border.Hisechocar diogram demonst ratesnormalLVcavit
ysi zewithejectionfract
ion
of68%.LVsept um andpost eriorwallare20mm and18mm,r espectively.TheLV
outfl
ow tract( LVOT)gr adientatr estis40mm Hg.Onst ressechocar diogram,he
exerci
sesf or13:00ont heBr uceprotocol,st
oppingduetofatigue.Hisbloodpr essure
atrestis110/ 70mm Hgandr i
sesto180/ 60mm Hg.TheLVOTgr adientroset o60mm
Hgatpeakexer cise.

Whi
choft
hef
oll
owi
ngdoyour
ecommendnext
?

a.Foll
ow- upeval uat
ioni n1year
.
b.El
ectrophysiologyst udy.
c.Ambul ator
yECGmoni tor
.
d.Proceedwi t
hI CDpl acement.
e.Echocardiogram wi t
ht i
ssueDoppl
eri
magi
ng.

Answer
C.Ambul
ator
yECGmoni
tor

Thedeci siont oplaceanI CDi npatient


swi thHCM shoul dincl
udeappl icati
onof
indivi
dualclinicaljudgment ,aswel lasat horoughdi scussionoft hestrengthof
evidence,benef i
ts,andr i
skst oal
low theinformedpat ient’
sacti
vepar ti
cipati
oni n
decisionmaki ng.ICDpl acementisr ecommendedf orpatientswit
hHCM wi thpr i
or
document edcar di
acarrest,ventr
icularfi
bri
ll
ati
on,orhemodynami callysignifi
cant
vent r
icul
artachycardia(VT).Itisr
easonablet orecommendanI CDf orpati
ent swith
HCM wi t
hsuddendeat hpr esumabl ycausedbyHCM i noneormor efi
rst-degree
relati
ves,a maxi mum LV wal lthickness of30 mm,orone ormor er ecent
unexpl ai
nedsyncopalepi sodes.

Ast hispat i
enthasnoper sonalorf amilyhi st
oryofSCD, LVwal lthickness<30mm,
andnosyncope,I CDi snotwar rantedwi t
houtf urtheri nformat i
on.AnI CDcanbe
usefulinsel ectpatientswi thnonsust ainedVT( NSVT) ,parti
cul arlythosepat ients
withar estingLVOTgr adi ent>30mm Hg,LV api calaneur ysm,l ategadolinium
enhancementoncar diacmagnet icresonancei magi ng,ordoubl eorcompound
genet i
cmut ations.Thispat ienthasar estingLVOTgr adient>30mm Hg,sof urther
i
nvest i
gationi swar rantedt odet ermineifanI CDshoul dbeconsi dered.Thefactt hat
hei sasympt omat i
cdoesnoti mpactt hedeci si
on t or i
sk- stratif
yhi m forI CD
placementf orpr i
mar ypr event ionofSCD,sosi mpl yrecommendi ngannualf ollow-
upi snotsuf fi
cientwi t
houtf i
rstobt ai
ningmor ei nformat i
on.Thi spat i
entdoesnot
haveanabnor malbloodpr essur eresponset oexer cise, defi
nedasei therafai
lur eto
i
ncr easethesyst oli
cbl oodpr essureby20mm Hgoradr opof20mm Hgdur ing
effort
.However ,thet est ing donepr ovidesno i nfor mat i
on on t hepr esenceor
absenceofNSVT,whi chwoul dr equireambul atoryECGmoni t oring.Othertest i
ng,
suchasel ect r
ophysiologyst udiest oinduceVTort issueDoppl erimagi ngtoassess
fordyssynchr ony,doesnotof ferhelpfulinformationf orr iskstrat i
fi
cat i
onofSCD.

92.A59- year-oldwomani sadmit


tedtot hehospitalforatr
ialfi
bri
ll
ati
on(AF)after
presenting wit
h palpit
ati
onsand shortnessofbr eath.Shehasapastmedi cal
historyofhypertension,t
ype2diabetes,chroni
cki dneydi
sease(stageIII
),anda
prior episode of AF. Thr ee months ago she under went tr
ansesophageal
echocar di
ogram (TEE),dir
ectcurr
entcar di
over
sion( DCCV),andwasst artedon
war far
in. 

Curr
enthomemedi cati
onsi ncludewar farin 5 mg dail
y,li
sinopril40 mg daily,
hydr
ochlor
othi
azi
de25mgdai ly,aml odi
pine5mgdai ly,omeprazole20mgdai ly,
si
mvastati
n40mgdai l
yandgl yburi
de5mgt wicedai l
y.Laboratorydatareveal:
sodi
um 138mmol /L,potassi
um 4. 2mmol /L,magnesium 2.2mg/ dl,cr
eati
nine1.6
mg/dl(esti
mated creati
nine clearance [
Cr Cl
]of45 ml /min),and inter
nati
onal
normal i
zedrati
o2.2.Thepati
ent
’screati
ninehasbeenstableovert
hepast2years.
Electr
ocardi
ogram onadmissi
onrevealsAFwi t
haventri
cularr
ateof112bpm anda
calculat
ed QTc of420 msec.TEE and DCCV wi t
h subsequentinit
iat
ion of
antiar
rhythmicdrugther
apyarepl
anned.

Whichofthef
oll
owingismostl
ikel
ytocont
ri
but
etoQTpr
olongat
ioni
ntheset
ti
ng
ofdof
eti
li
deadmini
str
ati
oni
nthi
spat i
ent
?

a.Li
sini
pr i
l.
b.Amlodipine.
c.Si
mvast ati
n.
d.Omepr azole.
e.Hydrochlorot
hiazi
de.

Answer

E.Hydr
ochl
orot
hiazi
de

Currentmanuf acturerrecommendat i
onsf orini ti
ationofdof eti
li
deincludeabasel ine
QTi ntervalorQTcof<440msec( oraQTi nter valorQTcof<500mseci npat i
ents
with an i ntraventr
icul arconduct ion del ay).Recommendat i
ons fort he use of
dofetil
idei npat i
ent swi t
hchr onickidneydi seasei ncl
udeacont r
aindi cationf or
usingt hedr uginpat ientswithaCr Cl<20ml /mi n.Dat aonf il
ewiththemanuf acturer
showasi gnif
icantphar macokineti
caswel lasphar macodynami cinteract i
onwhen
dofetil
idei sused concomi tantl
ywi t
h hydr ochl orothi
azide.Co- administ ration of
dofetil
ide500mcgt wicedai l
ywi thhydr ochl orothiazide50mgdai lyresul t
edi nan
i
ncr ease i n dofeti
lide ar ea undert he pl asma dr ug concentrati
on- ti
me cur ve
(appropriateusecr iteri
a)by27%andpeakconcent rati
on(Cmax)by21%.I naddi ti
on,
thephar macodynami cef f
ectofdof eti
li
dei ncr eased197%( asmeasur edbyt heQTc
i
ncr ease over t i
me) and a 95% i ncrease i nt he maxi mum QTc i ncr ease.
Phar macoki neti
c st udies in healthy volunt eer s did notsuggestaml odipine or
omepr azol ehadasi gnifi
canteffectont hephar macoki net i
csofdof et
ili
de,andno
currentr ecommendat i
onsexi stforavoi dingi nit
iatingdof eti
li
deinpat ientst aking
thesemedi cati
ons. 

93.A65- year-ol
dwomanwi thahi storyofdi abetesmel l
it
ust ype2,hyper t
ensi
on,and
chronicki dneydi seasest age4( glomerularf i
lt
rat
ionrate22ml /min)pr esent
stot he
emer gencydepar tmentwi thacut eleft
-sidedhemi paresis.Shei sdiagnosedwi than
acuter ight-si
dedmi ddlecerebralart
eryst rokeandr eceivest hr
ombolytics.Daysl at
er,
whilebei ngmoni toredi nthestrokeuni t
,shei snotedont el
emetrytohaveanepi sode
ofatrialfibr
ill
ati
on( AF)wi thventri
cularratesof130bpm.Shei sasympt omat i
cdur i
ng
thi
s epi sode. Tr ansthor
acic echocar diogr am r eveal
s moder at
e l eft ventr
icular
hyper
trophyandmoderatel
eftatr
ialdil
ati
on.Ther
ear
enoval
vul
arabnor
mal
it
iesnot
ed.
Thepatienthasnopr
iorhi
storyofbleedi
ng.

Whichofthef
oll
owi
ngwoul
dbet
hemostappr
opr
iat
ether
apyf
orsecondar
yst
roke
pr
event
ion?

a.Dabi
gatran150mgbymout ht
wicedail
y.
b.Warf
arintotar
geti
nter
nat
ionalnormali
zedr
ati
o(INR)of2-
3.
c.Ri
varoxaban20mgbymout hdai
ly.
d.Aspi
rin325mgdaily.
e.Aspi
rin325mgdailyandclopi
dogrel75mgdail
y.

Answer

B.War
far
int
otar
geti
nter
nat
ionalnor
mal
izedr
ati
o(NR)of2-
3.

Thispat i
enthasaCHA2DS2  VAScscor eof5( hyper t
ension,di abetesmel l
it
ust ype2,
diastoli
chear tfail
ure,cer ebr ovascul aracci dent )andhashi ghr i
skoff uturest roke
duet oAF.Gi venherCHA2DS2  VAScscor eof5  andl ackofpr iormaj orbl eeding,a
vi
tami nKant agoni st,adi rectt hrombi ni nhi bitor,oraf actorXai nhi bitorwoul dbe
appr opri
ateandwoul dpr ovi debet terpr otect ionagai nstst roket hanaspi ri
nal one.
Addi ti
onally,anor alant icoagul antwoul dpr ovidebet terstrokepr otect i
ont hant he
combi nationofaspi ri
nandcl opidogr el(asseeni ntheACTI VE  W[ AtrialFibril
lation
ClopidogrelTr ialWi t
hI rbesar tanf orPr event i
onofVascul arEvent s]t ri
al),andt he
patienthasnocont r
aindi cationst oor alant icoagul ati
on.However ,bot hdabi gat ran
andr ivar
oxabanar eprimar i
lyexcr etedi nt heki dneys, andt hispat ient’smoder atet o
sever echronicki dneydi seaseneedst obeconsi dered.Dabi gatranusehasnotbeen
studiedinpat i
ent swithcr eat i
ninecl earance<30ml /
mi nandshoul dbeavoi ded.I n
patientswi thcr eati
ninecl ear anceof15- 50ml /mi n,r
ivaroxabancanbeconsi der ed,
butatar educeddoseof15mgdai ly.Ther efore, t
hemostappr opr iate optionint his
scenar i
o iswar fari
nwi thgoalI NRof2- 3.

94.A 59 year -
old woman i s admi t
ted t
ot he hospitalwith a chi efcomplaintof
palpitat
ions,shortnessofbr eat
h, andfastheartrat
e.Thepat ienthasapastmedi cal
historythatissigni f
icantforhyper tensi
on,type2di abetes,chr oni
ckidneydisease
(stage3) ,andapr eviousepi sodeofat ri
alfi
bril
lat
ion3mont hspr iorf
orwhi chthe
patientunder wentsuccessf ultransesophagealechocar di
ographyanddi rectcurr
ent
cardioversion.Thepat ientreceivedwar f
ari
nf or4weeksf oll
owi ngthecardi
oversi
on.
Currenthomemedi cati
onsincludeaspiri
n 81 mg dail
y,li
sinopr
il40 mg dai
ly,
hydrochl
orot
hiazi
de(HCTZ)25mgdai l
y,amlodi
pine5mgdaily,omeprazol
e20mg
dail
y,si
mvastati
n40mgdai l
y,andglyburi
de5mgt wi
cedai
ly.

Laboratorydat afrom todayr evealsodium 138mmol /L,pot


assium 4.2mmol /L,
magnesi um 2.2mg/ dl
,andcr eati
nine1.6mg/ dl( givesanest i
mat edcr eat
ini
ne
clear
ance[ CrCl]of45ml /
min) .Thepatient
’screatininehasbeenst abl
eovert he
past2year s.Anel ectr
ocardiogram onadmi ssi
onr evealedat
ri
alfibri
ll
ati
onwi tha
ventr
icularresponserateof112bpm andacal culatedQTcof420msec.Thepl anis
for t
ransesophagealechocar diography and directcur r
entcardioversi
on wi t
h
subsequenti ni
ti
ati
onofant i
arrhythmicdrugther
apy.

Whichoft
hef ol
lowingwouldbet
hebestchoi
ceast
owhydof
eti
li
deshoul
dnotbeused
i
nthispat
ientatthecurr
entti
me?

a. Thedrugi
nter
acti
onwi t
haml odi
pine.
b. Thedrugi
nter
acti
onwi t
hHCTZ.
c. Thedrugi
nter
acti
onwi t
homepr azole.
d. TheCrCli
stoolowtoinit
iat
edofeti
lidetherapy.
e. Thebasel
ineQTcistoolongtoinit
iatedofeti
li
dether
apy.

Answer

B.Thedr
ugi
nter
act
ionwi
thHCTZ.

Currentmanuf act ur
err
ecommendat ionsf orini
ti
ationofdof eti
li
deincludeabasel ine
QTi ntervalorQTc<440msec( oraQTi nt er
valorQTc<500mseci npat ientswithan
intr
avent r
icularconduct iondel ay).Recommendat ionsf ortheuseofdof et
il
idei n
patientswi t
hchr oni
ckidneydi seasei ncludeacont raindicati
onforusi ngt hedrugi n
patients with a Cr Cl<20 ml /min.Dat a on f i
le wi t
ht he manuf act urershow a
signif
icant phar macokineti
c as wel las phar macodynami ci nteract i
on when
dofetil
ideisusedconcomi t
ant l
ywi t
hHCTZ.Co- admi nist
rati
onofdof etili
de500mcg
twicedai l
ywi t
hHCTZ50mgdai lyr esultedinani ncreasei ndofeti
lideappr opr i
ate
usecr iter
iaby27%andCmaxby21%.I naddi t
ion,t hephar macodynami ceff
ectof
dofetil
ideincreased197%( asmeasur edbyt heQTci ncr
easeovert ime)anda95%
increase int he maximum QTc i ncrease.Phar macoki net
ic studies i n healthy
volunteersdidnotsuggestaml odipineoromepr azolehadasi gnif
icantef f
ectont he
phar macokinet i
csofdof eti
lide,andnocur rentrecommendat i
onsexi stf oravoiding
init
iati
ngdof etil
ideinpatientstakingt hesemedi cations.

95.A69- year
-ol
dwomanpr esentedtotheemer
gencydepar
tmentwi
thnew onsetof
shor
tnessofbreat
hthathaslast
edfor2daysandaninabi
li
tyt
opart
ici
patei
nher
usualday-to-dayacti
viti
es.Sher epor
tsthatshefeel
slikeherhear ti
sraci
ngand
shecannotcat chherbreat
h.Thepat i
enthasahistor
yofhypertensionanddi
abet
es.
Shehasahi storyofgastr
ointest
inal(
GI)bleedi
ngwhil
et aki
ngababyaspi ri
nand
anover-the-counternonsteroi
dalanti
-inf
lammatorydrug,sot hesearenotbei
ng
used.
 

An echocardi
ogram is obtained that reveal
s car di
ac normal functi
on. An
el
ect
rocardi
ogram demonstr
at esnew- onsetatr
ialfibri
ll
ati
on (
AF).Thepat i
ent’
s
l
abor
atorydataar eobtai
nedandar ewi thi
nnor malr ange,exceptforaserum
cr
eat
inineof2.
6mg/ dlandcalculat
edcreati
nineclear
anceof20ml /min. 

Thepat ientwei ghs58kg.Shei shemodynami cal


lystabl
ewi t
habl oodpr essureof
110/ 70mm Hgandhear tr at
eof110- 130bpm andi sstart
edonmet oprolol
.On
met oprolol,herheartrat esdropt othe90sbpm,butshecont inuest or emain
sympt omat i
cal
lyuncomf ort
ablewithshortnessofbr eat
h.Thepat i
entunder goes
directcur rentcardi
oversionandi sinit
iat
edondr onedaronetomai nt
ainnor mal
sinusrhyt hm.Shehasr emai nedinnormalsinusrhyt
hm ondr onedaronefor3days.

Whi
choft
hef
oll
owi
ngi
sthebestant
icoagul
antchoi
cef
ort
hispat
ient
?

a.Apixaban5mgt wi
cedaily.
b.Dabigatran75mgt wicedail
y.
c.Ri
var oxaban20mgoncedai l
y.
d.Dose- adjust
edwarf
arin.
e.Aspirin81mgaday.

Answer

D.Dose-
adj
ust
edwar
far
in.

HerCHA2DS2- VAScscor eis4.Thepat i


enthasacal cul
atedcreatinineclearanceof
20ml /
min.Itisrecommendedt hatthedoseofr ivaroxabanber educedt o15mg
dail
yf oracr eati
nineclearanceof15- 50ml /min,so20mgdai lywoul dnotbe
recommended.Al ternati
vel
y,reduceddoses( 2.5mgt wicedaily)wer epr ovidedif
patientshadtwoormor eofthef ol
lowingcrit
eri
a:age80year s,bodywei ght60kg,
orser um creati
nineof1. 5 mg/ dl
.Theuseofdose- adjust
ed war fari
n maybe
preferr
ed int hi
s pat i
entwi t
h a hi stor
y ofgast roi
ntesti
nal( GI)bl eeding on
combi nati
onaspiri
nandnonst er
oidalanti-
infl
ammat orydrugther apybecauset he
i
ncidenceofGIbl eedingwasgr eaterwi t
hdabi gatranandr i
varoxabancompar ed
withwar f
arinintheclini
caltri
alsoftheseagent sf orAF.Undert heci rcumst ances
presentedinthiscaseandt hechoi cespresented,closelymanagedwar far
ini sthe
bestchoi
cef
orant
icoagul
ati
onf
ort
hispat
ient

96.Inpatient
swithatr
ialf
ibr
il
lati
on( AF)
,ther
ear etwomnenomi cst
hathavebeen
devel
opedtohel
prememberr i
skfactor
sforst
roke/
transi
enti
schemi
cat
tack(
TIA)

Forwhichofthefol
lowi
ngpatient
swouldanti
coagul
ati
onber
ecommendedbyt
he
CHA2DS2-VAScscor
e,butnotbytheCHADS2 
scor
e?

a.A 79- year


-old man wi
th hypert
ension and di abetes and 6 years of
persi
stentAF.
b.A66-year -
oldwomanwithoutpri
ormedicalhistoryandnew- onsetAF.
c.A69- year-
oldwomanwi t
hahi st
oryofrheumat i
cmi tralstenosi
sandnew-
onsetAF.
d.A25-year -
oldmanwithahist
oryofdrugabuseandcocai ne-i
nducedAF.
e.A42-year -
oldmanwithnew-onsethypert
hyroidism andAF.
Answer

B.A66-
year
-ol
dwomanwi
thoutpr
iormedi
calhi
stor
yandnew-
onsetAF.

The66- year -oldwomanhasaCHADS2  scor eof0andaCHA2DS2- VAScscor eof2.


Shepi cksup2poi nt sintheCHA2DS2- VAScsyst em forbeingf emaleandherage.
The CHADS2  scori
ng syst em i ncludes 1 poi nt for congest i
ve hear tf ai l
ure,
hypertension,age>75,anddi abetesand2poi ntsf orstrokeorTI A.TheCHA2DS2-
VAScscor ei s1poi ntf orcongesti
vehear tfail
ure,hypertension,age>65,di abet es,
vasculardi sease( historyofmyocar dialinfarcti
on,cor onaryarterydisease,pl aque
oni maging,orper ipheralvasculardisease) ,andf emal esex,and2poi ntsf orage
>75andst roke.Basedont hecur rentguidel i
nes,therapyf oraCHA2DS2  scor eof0
requir
esnot herapy.Ascor eof1r equiresnoneoraspi r
in,orantithr
ombi nther apy
may be used.A scor e of≥2 r equires ant i
thrombi nt herapy.Thi s applies to
nonvalvularAF.

97.A26- year-
oldwomanwi thnopastmedi calhist
orycomestoseeyoui nclini
cforfoll
ow
-upaf t
eranemer gencydepartmentvi si
tforanepisodeofsyncope.Shesayst hatshe
wasexer ci
singwhenshebeganf eeli
ngdizzyandnauseated.Shet r
iedtowal ktothe
bathroom andt henlostconsciousness.Herhusbandr ant ohersideandf oundher
awakeandor i
ented,butwit
har apidpulse.Bytheti
meemer gencymedi calt
echnici
ans
arr
ived,hervitalsi
gnswer enormal ,andherevaluati
onintheemer gencydepar t
ment
wasunr emarkable. 

Onphysicalexaminati
ontoday,herheartrat
ewas66bpm,andbl oodpressur
ewas
108/
72mm Hg.Sheappear swel l.Jugul
arvenouspressurei
snormal,andlungsare
cl
ear. There are no mur mur s. She has no l ower ext
remity edema. Her
el
ect
rocardi
ogram (ECG)demonstratesnormalsinusrhyt
hm andrightbundl
ebranch
block(RBBB)withQRSdur ationof128msec.El ectrolytesar ewi t
hinnormallimit
s.
Echocardi
ogram showsnormalleftventr
icular(
LV)f unction,mi l
drightventr
icul
ar(RV)
enlar
gement,mi l
d RV dysfuncti
on,and mi ldtricuspidr egurgi
tati
on.An exercise
tr
eadmillt
estinwhichthepati
enthasmul t
ipler
unsoft her hythm i
sshowni nFigure1.
Thereisnofamil
yhistor
yofsyncope, car
diacarrhythmia, orsuddendeat h. 

Whi
choft
hef
oll
owi
ngi
sthemostappr
opr
iat
enextst
ep?

a.Cardi
acmagnet icresonancei
magi
ng(
MRI
).
b.Cardi
accatheteri
zati
on.
c.Endomyocardialbi
opsy.
d.Geneti
ctest
ing.

Answer

A.Car
diacmagnet
icr
esonancei
magi
ng(
MRI
).
The concer nist hatt his young woman has ar rhythmogenicrightvent ricular
cardi
omyopat hy(ARVC)becauseshehasexer ci
se-inducedventri
culartachycar dia
andRVenl ar
gementonechocar di ogram,aswel lasRBBBonECG.I nor dertomeet
cri
ter
iaforARVC,t hereshouldbet womaj orcri
teri
a,oronemaj orandt womi nor
cri
ter
iai nt he cat egori
es ofvent ri
cul
ar dysfunction,ti
ssue char acteri
zat ion,
arr
hythmias,andECGde/ repolarizationcri
teri
a.TheMRIwi llal
lowbot heval uat i
on
ofventr
iculardysfunctionandsi ze,aswel last hetissueandf i
brofattyinfil
tration.
Endomyocar di
albi opsywoul dal sobehel pfulfort i
ssuecharacterizati
on,buti s
i
nvasiveandsubj ecttosampl i
nger ror.MRIprovidesmor edatawithlessr isktot he
pati
ent.Once t he di agnosisi s made,you can r eferforgenet i
ct esti
ng f or
confi
rmationandf ami l
yplanning.
Coronary Artery Disease ( ACCSAP-9 2016 )

1. A 19-year-old African-American male college soccer player is referred to see you in clinic for
screening prior to participation in collegiate athletics. He has a history of childhood asthma, but is
otherwise healthy and takes no medications. There is no family history of cardiomyopathy,
coronary artery disease, or sudden cardiac death. His physical examination is normal. He hands you
an electrocardiogram (ECG) that his primary care physician performed.
Which of the following findings, if present, would be of most concern?

a. Left ventricular hypertrophy (LVH) by the limb-lead voltage criteria (R in lead aVL of 13 mm).
b. 0.5 mm ST depressions in leads I and aVL.
c. Wenckebach pattern (Mobitz I second-degree atrioventricular [AV] block).
d. Domed/convex upsloping ST elevation with T-wave inversions in V2-V3.
e. Right-axis deviation of QRS complex to 105 degrees.
1. b. 0.5 mm ST depressions in leads I and aVL

Elite athletes may demonstrate ECG changes that overlap with findings seen with cardiomyopathy.
Several criteria, including the European Society of Cardiology criteria, Seattle criteria, and recently
published “refined criteria” aim to distinguish between normal ECG changes seen with athletic
adaptation versus those that may be associated with increased risk for sudden death. Importantly,
ECG changes may be more common in athletes of African/Caribbean ancestry. Right-axis deviations
of the QRS complex up to 115 degrees, type 1 second-degree AV block, and LVH by voltage criteria
alone (without evidence of axis changes, repolarization changes, atrial enlargement, or increased
QRS width) can be seen in healthy athletes. In addition, healthy black athletes may have convex ST-
segment elevation associated with T-wave inversions in leads V1-V4. However, ST-segment
depression ≥0.5 mm in two or more leads is considered an abnormal finding.
2. A 74-year-old man with hypertension and hyperlipidemia presents to your office with chest
discomfort on exertion. Exercise nuclear stress test demonstrates minimal inferior ischemia and
transient ischemic dilatation, with a left ventricular ejection fraction (LVEF) of 38%. Coronary
angiography reveals a 60% proximal left anterior descending artery (LAD) stenosis, 90% left
circumflex stenosis, and 70% mid right coronary artery stenosis. Fractional flow reserve (FFR)
evaluation of the LAD lesion is 0.72.

Which of the following is the most appropriate next step in his treatment?

a. Coronary artery bypass surgery.


b. Percutaneous coronary intervention (PCI) of the angiographically significant lesions.
c. LAD PCI and medical management of the remaining coronary artery disease.
d. Metoprolol and sublingual nitroglycerin.
e. Increase atorvastatin to 80 mg daily.
2. a. Coronary artery bypass surgery

This patient presents with multivessel coronary artery disease on coronary angiography including
the proximal LAD, based on an FFR <0.8. Regardless of stable anginal symptoms, surgical
revascularization of severe left main disease or proximal LAD with multivessel disease, in the
setting of reduced LVEF, has a proven mortality benefit and is recommended. The preferred
method would be surgical revascularization in the absence of contraindications. Optimal medical
therapy should be initiated in conjunction with revascularization, not as stand-alone therapy.
3. A 56-year-old man with a history of hypertension and hyperlipidemia is evaluated for
symptoms of progressive chest discomfort. He has increasing substernal chest tightness with
associated shortness of breath when he walks five blocks or climbs two flights of stairs.

On physical exam, his heart rate is 58 bpm, blood pressure is 132/82 mm Hg, and cardiac
examination is normal. His current medications include aspirin 81 mg daily, metoprolol 25 mg twice
daily, and simvastatin 20 mg daily. An exercise nuclear stress test reveals mild inferior ischemia and
a normal left ventricular ejection fraction. Coronary angiography reveals a 70% stenosis of the mid
right coronary artery (RCA) and no other obstructive lesions.

Which of the following is the most appropriate next step in his management?

a. Fractional flow reserve (FFR) of the RCA.


b. Isosorbide mononitrate 30 mg daily.
c. Intravascular ultrasound (IVUS) of the RCA.
d. Percutaneous coronary intervention (PCI) to the RCA.
e. Clopidogrel 75 mg daily.
3. b. Isosorbide mononitrate 30 mg daily.

This patient has angina with a low-risk stress test, low-risk angiographic findings, and is not on
optimal medical therapy. Based on the Appropriate Use Criteria guidelines for revascularization,
this patient should have a trial of optimal medical therapy, which is defined as treatment with at
least two maximally-tolerated antianginal medications (beta-blockers, calcium channel blockers,
long-acting nitrates, and ranolazine).

In this patient, the addition of a second antianginal medication is the most appropriate next step,
before PCI would be performed. FFR or IVUS is not recommended in the setting of a stress test that
has already demonstrated ischemia in the territory of the angiographically abnormal lesion.

Clopidogrel does not have antianginal properties, and there are no data for its use in this scenario.
4. A 48-year-old man is seen for cardiovascular (CV) risk assessment. He has a history of tobacco
use, but quit 15 years ago. His father underwent coronary artery bypass grafting at age 47. He is
active, but does not exercise regularly. He has no other significant health issues. His blood pressure
is 135/80 mm Hg in both arms, and his exam is unremarkable.

His 10-year risk of atherosclerotic CV disease (ASCVD) based on the Pooled Cohort Equation is
7.8%. He states an interest in alternative medicine and prefers supplements over conventional
medicines. He is concerned about his CV health and would like advice. He tells you that he does not
like traditional medications and prefers a more natural approach.

In addition to encouraging lifestyle changes, which of the following should be initiated?

a. Vitamin D supplementation 2000 international units.


b. Atorvastatin 40 mg.
c. Daily multivitamin.
d. Red yeast rice 1200 mg.
e. Vitamin E 400 international units and beta-carotene 15 mg.
4. b. Atorvastatin 40 mg.

Based on his risk factors, this patient’s 10-year risk of ASCVD based on the Pooled Cohort Equation
is 7.8%. According to the most recent guidelines, the initiation of a moderate- or high-intensity
statin is appropriate. Supplements such as red yeast rice contain varying amounts of monacolin K,
the active ingredient in lovastatin. As with many supplements, they are not well regulated, and
therefore the amount of active ingredients may vary considerably.

Although vitamin D deficiency is a significant concern for many Americans, to date, there are no
data to support its use for prevention of CVD. The Physicians’ Health Study randomized 14,641
male physicians, the large majority without coronary artery disease, to either multivitamin daily or
placebo. No benefit was seen with taking multivitamins. The U.S. Preventive Services Task Force
has recommended against use of vitamin E and beta-carotene for CV prevention.
5. A 50-year-old man presents to the emergency department with an acute inferior myocardial
infarction. A drug-eluting stent is placed in the mid right coronary artery, and he recovers well. The
patient remarks, "I just had a stress test last week and passed with flying colors. How can I have
had a heart attack?" You explain that atherosclerotic plaques may be too small to induce ischemia
on stress testing, but may be vulnerable to rupture and result in acute thrombotic arterial
occlusion.

Which of the following features is such vulnerable plaques most likely to have?

a. Increased smooth muscle proliferation.


b. Reduced inflammatory markers.
c. Large lipid-filled cores.
d. Thick overlying fibrous caps.
e. Low macrophage content.
5. c. Large lipid-filled cores.

Vulnerable plaques are most often plaques with lipid-rich cores, evidence of inflammation, and thin
fibrous caps. Therefore, an exercise stress test may not predict the presence of such a plaque,
because they are not necessarily flow limiting. These plaques typically have a high macrophage
content relative to smooth muscle cells.
6. A 55-year-old man who is a smoker but otherwise healthy presents to the emergency
department with several hours of stuttering chest and left arm pain. He is on no medications at
home but took an aspirin before he came to the emergency department, and his pain was
somewhat reduced by the time of arrival. His blood pressure is 135/87 mm Hg and heart rate is 90
bpm. Physical examination is otherwise normal. His electrocardiogram shows T-wave inversions
and 1 mm ST depressions in the inferior and lateral precordial leads. His point of care troponin is
elevated. He is initially treated with oxygen, metoprolol, and intravenous nitroglycerin and started
on systemic anticoagulation.

Which of the following physiologic components of this patient's acute coronary syndrome explains
why the aspirin has helped improve his pain?

a. Inhibition of thrombin.
b. Inhibition of antithrombin III.
c. Inhibition of the cyclooxygenase-1 enzyme.
d. Inhibition of the P2Y12 receptor.
e. Inhibition of glycoprotein IIb/IIIa interaction with fibrinogen.
6. c. Inhibition of the cyclooxygenase-1 enzyme.

Aspirin inhibits platelet activation by irreversibly targeting the cyclooxygenase-1 enzyme.


Clopidogrel, prasugrel, and ticagrelor target the P2Y12 adenosine diphosphate receptor.
Glycoprotein IIb/IIIa inhibitors include eptifibatide, abciximab, and tirofiban. Bivalirudin is the only
intravenous direct thrombin inhibitor used in the treatment of acute coronary syndrome. Heparins
inhibit the activity of antithrombin III.
7. The critical step in platelet aggregation involves which of the following receptors?

a. Glycoprotein Ib/IX.
b. Thrombin.
c. Glycoprotein IIb/IIIa.
d. Inositol trisphosphate 3.
e. Actin.
7. c. Glycoprotein IIb/IIIa.

Platelets have four major functions: adherence, activation, aggregation, and interaction with
coagulation factors. Platelet activation occurs when platelets are exposed to soluble factors such as
adenosine diphosphate or thrombin at the site of vascular injury. This initiates a shape change in
the platelet, and multiple filopodial projections are formed by rearrangements in the cytoskeleton.
These cytoskeletal changes open the extracellular side of the glycoprotein (GP) IIb/IIIa receptor and
allow it to bind to fibrinogen. The binding of fibrinogen allows connections between platelets and is
critical for platelet aggregation. Therefore, the receptor critical to platelet aggregation is the GP
IIb/IIIa receptor.

GP Ib/IX is cleaved during the process of platelet activation, but does not have a role in
aggregation. Inositol trisphosphate 3 receptors are involved in the calcium signaling required for
activation, but do not have a role in aggregation. Thrombin is responsible for initiation of platelet
activation, and its receptor on the platelet surface is protease-activated receptor 1. Actin is a
cytoskeletal protein.
8. A 59-year-old woman presents to the emergency department with a 3-month history of
exertional chest pain. Her physical exam is unremarkable. Her resting electrocardiogram (ECG) and
cardiac biomarkers are normal. She experiences chest pain during a stress test with associated ST-
segment depression. However, perfusion imaging is normal. Subsequent coronary angiography
demonstrates mild luminal irregularities.

Which of the following is the most likely explanation for her chest pain?

a. Hypertrophic cardiomyopathy.
b. Noncardiac chest pain.
c. Coronary microvascular dysfunction.
d. Coronary vasospasm.
e. Atherosclerotic coronary artery disease (CAD).
8. c. Coronary microvascular dysfunction.

Coronary microvascular dysfunction is diagnosed based on the findings of typical anginal symptoms
and noncritical epicardial CAD usually associated with abnormal exercise treadmill stress testing.
Women with normal or near normal coronary arteries with evidence of ischemia and angina have
an overall higher risk of adverse cardiovascular events compared with patients without angina and
without CAD. The prognosis of this patient would not be expected to be better than patients
without cardiovascular disease. In general, patients with angina and fixed obstructive CAD have a
worse outcome than patients with angina and without fixed obstructive CAD.

Vasospasm is typically diagnosed with ST-segment elevation at the time of symptoms, but stress
testing is not sensitive for the diagnosis. Hypertrophic cardiomyopathy can cause chest pain, but
would be expected to be associated with ECG changes, as well as physical exam findings suggestive
of outflow obstruction. Atherosclerotic CAD is unlikely to be an adequate explanation, based on the
findings described on her angiogram.
9. A 53-year-old woman with hypertension, hyperlipidemia, diabetes, and smoking history has
progressively worsening exertional jaw discomfort for the past 6 months. She is generally
sedentary, has not seen a physician in many years, and is presently taking aspirin and atorvastatin
80 mg daily for medications. Blood pressure in the office is 145/78 mm Hg and pulse is 82 bpm. Her
low-density lipoprotein (LDL) returned at 137 mg/dl and her glycated hemoglobin is 7.5%.

She was referred for a pharmacologic myocardial perfusion imaging scan (MPS), which showed a
reversible defect in the mid to distal anterior wall. Coronary angiography shows diffuse plaque in
the mid right coronary artery with only 20% narrowing and serial 20-30% lesions in the mid left
anterior descending artery (LAD).

Which of the following is the next most appropriate step in the patient's management, in addition
to aggressive risk factor modification and medical therapy with a beta-blocker, angiotensin-
converting enzyme inhibitor, and statin?

a. Percutaneous coronary intervention (PCI) of the LAD.


b. Isosorbide mononitrate.
c. Ranolazine.
d. Extracorporeal counterpulsation.
e. Fractional flow reserve (FFR).
9. b. Isosorbide mononitrate.

The prognosis for patients with coronary microvascular dysfunction is not benign and may be
similar to selected age- and sex-matched patients with epicardial coronary artery obstructive
disease. This finding of no obstructive disease should not lead to complacency relative to
management. The first line of management should be risk factor modification beginning with
lifestyle modification. The patient should be encouraged to increase her daily activities, and if
possible, begin a regular exercise program like walking. She should also understand that if angina is
induced with light exercise, such as walking, it is not dangerous and titration of isosorbide
mononitrate may permit her to walk comfortably without angina. The goal is to work toward the
American Heart Association recommended 10,000 steps per day, achieve excellent blood pressure
control (<120/90 mm Hg), LDL-lowering, and optimal management of glycemia.

PCI of a nonobstructive lesion will not result in symptom relief. However, her MPS defect and serial
stenoses are colocalized to the same coronary distribution, and serial stenoses in the 30% diameter
range could contribute to important obstruction. Thus, if her symptoms persist, FFR would be
helpful to assess the functional significance of the serial stenoses. If these lesions are confirmed to
be truly nonobstructive, coronary flow reserve measurements would be useful to assess for
coronary microvascular dysfunction. Ranolazine and extracorporeal counterpulsation should not be
the first line of management.
10. A 49-year-old woman with a history of diabetes and hypertension, and a family history of
coronary artery disease (CAD) presents to her physician with the chief complaint of chest pain and
heaviness that occurs with exertion and infrequently at rest. Her symptoms have been persistent
for nearly a year. She initially attributed her symptoms to acid reflux.

She is referred for an exercise treadmill test. Her body mass index is 30 kg/m2. She is able to
exercise for 8 metabolic equivalents (METs) on a standard Bruce protocol, but exercise is limited by
her typical angina. Her electrocardiogram shows 3-4 mm ST-downsloping segment depressions in
the lateral leads. She is referred for coronary angiography that shows no obstructive CAD and no
luminal irregularities.

Which of the following is the most appropriate next best step in the patient's management?

a. Esophagogastroduodenoscopy (EGD).
b. Intravascular ultrasound (IVUS).
c. Ergonovine challenge.
d. Coronary flow reserve measurement.
e. Assess response to empiric antispasm medication.
10. d. Coronary flow reserve measurement.

This patient has multiple cardiovascular risk factors and evidence of ischemia on stress testing. She
has no obstructive CAD on coronary angiography. The most likely diagnosis is coronary
microvascular dysfunction (CMD). The exact mechanism of CMD is not entirely clear. From a
pathophysiological view, CMD as reduced coronary flow reserve (CFR) most often results from a
combination of different alterations, such as impaired vasodilation, enhanced vasoconstrictor
responsiveness, and/or structural remodeling of the coronary microvasculature. CMD can cause
abnormalities in stress testing indicative of ischemia without obstructive epicardial CAD.

CFR measurement either invasively (intracoronary Doppler catheter) or noninvasively (positon


emission tomography, cardiac magnetic resonance imaging, or transthoracic Doppler), can be very
helpful in determining if CMD is present. There is no role for IVUS in this setting, nor does
"prophylactic" stenting of borderline plaques improve outcomes and may be harmful. Coronary
spasm is not the likely etiology in this case, particularly given the appearance of smooth coronaries
on angiography and lack of prominent component of her symptoms occurring at rest. Nevertheless,
testing with intracoronary acetylcholine would exclude spasm with a high degree of certainty and
perhaps avoid need to expose her to multiple calcium antagonists and nitrates.
11. A 90-year-old man with hypertension and hyperlipidemia had previously enjoyed a good
quality of life without symptoms secondary to cardiovascular disease. However, over the last 3
months, he has noted the progressive onset of symptoms consistent with exertional angina. He
generally enjoys taking walks around a pond near his home. However, he now notes that after 100
yards, he develops heaviness in the center of his chest that resolves with rest. He was started on a
beta-blocker, aspirin, and a statin, but continues to have lifestyle-limiting angina.

He is referred for an exercise tolerance test, where he is able to perform 5 METs of exercise prior to
developing limiting angina and 3 mm ST depressions in the lateral leads. He is referred for cardiac
catheterization, where he is found to have a 90% focal lesion in the mid left anterior descending
artery (LAD), a 50% lesion in the first obtuse marginal artery (OM1), and a focal 99% stenosis of the
right coronary artery (RCA) with collaterals to the distal RCA from the LAD.

Which of the following is the most appropriate next step for this patient?

a. Medical therapy.
b. Hybrid CABG/percutaneous coronary intervention (PCI).
c. Referral for coronary artery bypass grafting (CABG).
d. PCI of the RCA and LAD.
e. PCI of the RCA, OM, and LAD.
11. d. PCI of the RCA and LAD.

The patient's age is not a limitation to revascularization. With advancements in both surgical and
interventional techniques, the safety of revascularization procedures has improved over time and
the elderly have as much to gain by revascularization as the nonelderly. This patient has a low
SYNTAX score, which would suggest that the outcomes of multivessel PCI are comparable with
CABG in terms of death and myocardial infarction out to 5 years.

PCI likely offers a reduced risk of stroke. Therefore, PCI is the preferred revascularization strategy
for this patient given that it is less invasive. Fractional flow reserve of the OM lesion may be
reasonable to show that the lesion is not hemodynamically significant. At this time, there are no
definitive outcome data to support routine use of hybrid revascularization procedures.
12. A 57-year-old man with hypertension, tobacco use, and diabetes presents with 3 weeks of
exertional chest tightness. An exercise treadmill stress test is notable for reproducible chest
discomfort associated with 2 mm inferolateral ST-segment depressions and ST-segment elevation
in lead aVR.

He is started on aspirin, simvastatin, and metoprolol and referred for cardiac catheterization, which
demonstrates a right-dominant system with focal 90% proximal left anterior descending artery
(LAD), 80% mid left circumflex, and 80% mid right coronary artery stenoses. You discuss with him
the options of surgical versus percutaneous revascularization.

Which of the following outcomes is associated with surgical revascularization in this patient?

a. Decreased bleeding risk.


b. Increased need for repeat revascularization.
c. Decreased stroke risk.
d. Improved ejection fraction.
e. Improved survival.
12. e. Improved survival.

In patients with diabetes and significant left main coronary artery disease (=50%) or severe
coronary artery disease (=70%) in three major coronary arteries or involving the proximal LAD plus
one other coronary artery, coronary artery bypass grafting is the recommended revascularization
strategy and has been shown to improve survival. In contrast, while percutaneous coronary
intervention (PCI) may improve anginal symptoms in patients with stable ischemic heart disease, no
studies to date have demonstrated that PCI confers a survival advantage without recent acute
coronary syndrome. Stroke risk is increased in CABG versus PCI.
13. A 52-year-old man with hypertension and hyperlipidemia is referred for cardiac stress testing
due to atypical chest pain and recently declining exercise tolerance. He completes 7 minutes of a
standard Bruce protocol and stops due to fatigue and chest burning. Perfusion imaging reveals a
moderately severe, reversible apical defect. He is started on aspirin 81 mg daily, simvastatin 20 mg
daily, metoprolol 25 mg daily, and amlodipine 5 mg daily, but his symptoms persist. He is referred
for cardiac catheterization, which reveals a 70% mid left anterior descending artery stenosis
treated successfully with percutaneous coronary intervention (PCI).

Which of the following best describes the long-term outcomes of PCI versus coronary artery bypass
grafting (CABG) in this patient?

a. No difference.
b. Decreased mortality with PCI.
c. Improved left ventricular ejection fraction with CABG.
d. Decreased target vessel revascularization with PCI.
e. Increased recurrent myocardial infarction (MI) with CABG.
13. a. No difference.

More than 20 randomized controlled trials have compared CABG with balloon angioplasty or
stenting in patients with single-vessel coronary artery disease, demonstrating similar outcomes
regarding survival and recurrent MI. CABG has a significantly higher rate of stroke, but lower repeat
revascularization rate compared with PCI. In this patient with one significant stenosis amenable to
revascularization and unacceptable angina despite guideline-directed medical therapy, PCI or CABG
are both beneficial to improve anginal symptoms.
14. A 62-year-old man with a history of smoking, hypercholesterolemia, and obesity is being
evaluated for progressive dyspnea on exertion. He denies chest pain, orthopnea, or peripheral
edema. An echocardiogram demonstrates left ventricular ejection fraction (EF) of 35% with global
hypokinesis and no significant valvular dysfunction. Coronary angiography reveals significant three-
vessel coronary artery disease (CAD) with an occluded but well-collateralized dominant right
coronary artery, and calculated SYNTAX score of 35.

Which of the following is the most appropriate next step?

a. Hybrid CABG/PCI.
b. Implantable cardioverter-defibrillator (ICD).
c. Coronary artery bypass grafting (CABG).
d. Medical therapy alone.
e. Multivessel percutaneous coronary intervention (PCI).
14. c. Coronary artery bypass grafting (CABG).

The SYNTAX trial randomized 1,800 patients with left main or three-vessel CAD to revascularization
with drug-eluting stents or CABG. Overall, at 3 years, major adverse cardiac events were lower for
CABG over PCI (20.2% vs. 28.0%), driven by decreased recurrent myocardial infarction and repeat
revascularization in the CABG arm.

In a post hoc analysis, an angiographic SYNTAX score was derived for each patient as a surrogate
measure of the CAD complexity, based on the location, severity, and extent of CAD (low SYNTAX
score, 22; intermediate, 23-32; and high, 33). While patients with a low SYNTAX score had similar
outcomes with PCI or CABG, those with intermediate and high SYNTAX scores treated with CABG
exhibited lower mortality at 3 years compared with PCI, with significantly decreased target vessel
revascularization.

In summary, the SYNTAX results suggest that PCI or CABG is comparable in less complex disease,
while in patients with more complex disease, CABG is the preferred revascularization strategy.
There are no data as of yet to support use of hybrid CABG/PCI. Medical therapy alone is not
sufficient in the setting of reduced EF. ICD is warranted only if EF remains <35% at 3 months after
revascularization.
15. An 82-year-old man with a history of coronary artery disease (CAD) status/post percutaneous
coronary intervention (PCI) to the left anterior descending artery (LAD) 10 years ago, chronic
obstructive pulmonary disease, obesity, and chronic kidney disease develops progressive angina on
exertion. He has been managed successfully for many years on a combination of aspirin,
atorvastatin, and atenolol. Echocardiography reveals preserved left ventricular function.

He is referred for coronary angiography, which demonstrates 60% ostial left main coronary
stenosis, a widely patent LAD stent with diffuse luminal irregularities, no significant left circumflex
disease, and focal 70% mid-right coronary artery stenosis.

Which of the following is the next best step in this patient's management?

a. Heart Team evaluation.


b. Multivessel PCI.
c. Medical therapy alone.
d. Coronary artery bypass grafting (CABG).
e. PCI to the left main coronary artery.
15. a. Heart Team evaluation.

The current stable ischemic heart disease guideline recommends that patients with significant left
main or complex multivessel CAD be evaluated by a multidisciplinary Heart Team to determine the
most appropriate method of revascularization. This strategy mimics typical protocols used in
clinical trials, where a Heart Team, composed of an interventional cardiologist and cardiac surgeon,
review all available patient data and discuss potential treatment options with the patient.
Calculation of the Society of Thoracic Surgeons (STS) and SYNTAX scores assist in determining the
surgical risk category and complexity of CAD.

While PCI may be a reasonable strategy in patients with amenable coronary anatomy, high surgical
risk, or other considerations, in general, ad hoc PCI immediately following diagnostic coronary
angiography is discouraged in patients with complex multivessel CAD.

Although escalation of medical therapy may improve angina, if the patient is a candidate for
revascularization, a Heart Team evaluation would be beneficial.
16. A 65-year-old man with diabetes controlled with medications comes to you for evaluation of
exertional chest pain. He has been appropriately managed for chronic stable angina with beta-
blockers, statin, nitrates, and aspirin.

The patient is able to exercise 7 minutes on a Bruce protocol, his classic chest pain recurs at stage
3, and there are 2 mm ST depressions in the anterolateral leads with reversible perfusion defects
anteriorly. His left ventricular ejection fraction (LVEF) is 50%. Cardiac catheterization reveals 90%
promimal left anterior descending artery (LAD), 70% first obtuse marginal, 80% mid left circumflex,
and 90% mid right coronary artery stenoses.
Which of the following is the next best step in the management of this patient?
a. Ranolazine extended-release 500 mg PO daily.
b. Coronary artery bypass grafting (CABG) revascularization.
c. LAD PCI revascularization only.
d. Increase metoprolol.
e. Multivessel percutaneous coronary intervention (PCI) revascularization.
16. b. Coronary artery bypass grafting (CABG) revascularization.

This patient has diabetes, severe multivessel coronary artery disease (CAD), and a reduced LVEF. He
should undergo complete revascularization with CABG.

Recently, results from the FREEDOM trial in patients with diabetes and multivessel CAD treated
with CABG had a significantly lower composite rate of death from any cause, nonfatal myocardial
infarction, or nonfatal stroke when compared with diabetic patients treated with PCI.

An analysis performed in 2009 of data in 7,812 patients (1,233 with diabetes) in 10 randomized
clinical trials demonstrated a worse long-term survival rate in patients with diabetes after balloon
angioplasty or bare-metal stent implantation than after CABG.
The BARI 2D trial randomly assigned 2,368 patients with type 2 diabetes and CAD to undergo
intensive medical therapy or prompt revascularization with PCI or CABG, according to whichever
was thought to be more appropriate. By study design, those with less extensive CAD more often
received PCI, whereas those with more extensive CAD were more likely to be treated with CABG.
The study was not designed to compare PCI with CABG. At 5-year follow-up, no difference in rates
of survival or major adverse cardiac events (MACE) between the medical therapy group and those
treated with revascularization was noted. In the PCI stratum, no significant difference in MACE
between medical therapy and revascularization was demonstrated (drug-eluting stent in 35%;
bare-metal stent in 56%); in the CABG stratum, MACE occurred less often in the revascularization
group.
17. A 73-year-old morbidly obese woman with insulin-dependent diabetes, dyslipidemia, and
severe ambulatory dysfunction is sent to you for preoperative assessment before total knee
replacement. She has chronic kidney disease with a creatinine of 2.1 mg/dl. Her heart rate is 80
bpm, blood pressure is 150/80 mm Hg, and respiratory rate is 20 breaths/minute. She has no
evidence of heart failure.

Her electrocardiogram (ECG) reveals sinus rhythm with inferior Q waves noted. The patient's
echocardiogram is normal. She is on insulin, losartan 100 mg PO daily, and atorvastatin 40 mg PO
daily.
Which of the following is the next best step in the preoperative cardiac risk assessment for this
patient?
a. Pharmacologic nuclear stress test.
b. Coronary computed tomographic angiography.
c. Cardiac catheterization.
d. No further cardiac evaluation is necessary.
e. Exercise ECG stress test.
17. a. Pharmacologic nuclear stress test.

This patient has several risk factors for coronary artery disease. While knee replacement is not a
high-risk surgery compared with vascular surgery and head and neck surgery, this patient has
several concerning clinical features, including an abnormal resting ECG, increased creatinine, insulin
therapy, uncontrolled blood pressure, and unclear functional capacity.

Multivariate risk indices such as the revised cardiac risk index (RCRI) and 2014 American College of
Cardiology (ACC)/American Heart Association (AHA) perioperative cardiovascular evaluation
guideline can be applied to determinate the major adverse cardiac event (MACE) risks. Her RCRI
risk is 11% for MACE (includes possible myocardial infarction, insulin use, and creatinine >2.0
mg/dl).

According to the ACC/AHA 2014 guideline for perioperative cardiovascular evaluation for
noncardiac surgery, this is a Class IIa indication:

"It is reasonable for patients who are at an elevated risk for noncardiac surgery and have poor
functional capacity (<4 METs) to undergo noninvasive pharmacological stress testing (either
dobutamine stress echocardiogram or pharmacological stress perfusion imaging) if it will change
management. (Level of Evidence: B)"

Figure 1 outlines the ACC/AHA perioperative guideline. A pharmacologic stress test is appropriate
in someone who cannot exercise.
(Figure 1)
18. A 55-year-old man comes to you for preoperative cardiovascular risk stratification for inguinal
hernia repair. The patient has hypertension and takes atenolol 25 mg PO daily. He has a strong
history of smoking 30 pack-years and has not had his lipid profile evaluated for 2 years. He has a
family history of coronary artery disease; his father had a myocardial infarction at 53 years of age.
The patient notices pain and discomfort in the groin area while walking. However, he is able to
walk up a hill and mow the lawn.

On examination, heart rate is 53 bpm, blood pressure is 120/83 mm Hg, and respiratory rate is 15
breaths/minute. His electrocardiogram shows sinus rhythm at 54 bpm with an incomplete right
bundle branch block.

Based on these findings, which of the following is your next best step in evaluating this patient?

a. Transthoracic echocardiogram.
b. Exercise treadmill stress testing.
c. Proceed with surgery.
d. Dobutamine stress echocardiography.
e. Coronary computed tomography angiography.
18. c. Proceed with surgery.

Functional status is a reliable predictor of perioperative and long-term cardiac events. Patients with
reduced functional status preoperatively are at increased risk of complications. Conversely, those
with good functional status preoperatively are at lower risk. Moreover, in highly functional
asymptomatic patients, it is often appropriate to proceed with planned surgery without further
cardiovascular testing, as outlined in the 2014 perioperative cardiovascular evaluation guideline
(Figure 1).
19. A 63-year-old woman comes to your office for preoperative clearance for a hysterectomy. The
patient has been complaining of dysfunctional uterine bleeding and was found to have multiple
uterine fibriods that have not improved with embolization. She has history of insulin-dependent
diabetes, hypertension, and chronic kidney disease (stage 3). Her medications include lisinopril 40
mg daily, ferrous sulfate, and insulin. She can do her activities of daily living, but becomes short of
breath climbing a flight of stairs.

On exam, her heart rate is 88 bpm and blood pressure is 133/83 mm Hg, respiration rate is 15
breaths/minute, and conjunctiva are pale. Cardiovascular examination reveals soft systolic flow
murmur in the left sternal border. There are no rales or rhonchi and no pedal edema.

An electrocardiogram in your office shows sinus rhythm with left bundle branch block (LBBB). Her
most recent labs include glycated hemoglobin of 8.9%, hemoglobin of 9.0 mg/dl, and creatinine of
2 mg/dl.
Which of the following is the next best step in her management?
a. Pharmacologic nuclear stress test.
b. Exercise treadmill stress test.
c. Proceed with surgery.
d. Dobutamine stress echo.
e. Start beta-blockers.
19. c. Proceed with surgery.

This patient has several cardiovascular risk factors and evidence of an LBBB. She can, however,
climb a flight of stairs, and thus has a functional capacity >4 METs. She does not require stress
testing prior to undergoing medically necessary surgery.

Functional status is a reliable predictor of perioperative and long-term cardiac events. Patients with
reduced functional status preoperatively are at increased risk of complications. Conversely, those
with good functional status preoperatively are at lower risk. Moreover, in highly functional
asymptomatic patients, it is often appropriate to proceed with planned surgery without further
cardiovascular testing.

If a patient has not had a recent exercise test before noncardiac surgery, functional status can
usually be estimated from activities of daily living. Functional capacity is often expressed in terms
of METs, where 1 MET is the resting or basal oxygen consumption of a 40-year-old man who weighs
70 kg. In the perioperative literature, functional capacity is classified as excellent (>10 METs), good
(7-10 METs), moderate (4-6 METs), poor (<4 METs), or unknown.
Perioperative cardiac and long-term risks are increased in patients unable to perform 4 METs of
work during daily activities. Examples of activities associated with <4 METs are slow ballroom
dancing, golfing with a cart, playing a musical instrument, and walking at approximately 2-3 mph.
Examples of activities associated with >4 METs are climbing a flight of stairs or walking up a hill,
walking on level ground at 4 mph, and performing heavy work around the house.
20. A 65-year-old man with intermittent bilateral lower-extremity claudication, hypertension, and
diabetes presents to your office for assessment prior to planned femoral-popliteal bypass surgery.
He denies chest pain, but is significantly limited by his claudication and cannot walk more than a
block without stopping. A pharmacologic stress test reveals a large area of ischemia in the anterior
wall; left ventricular ejection fraction is 40%. Coronary angiography reveals three-vessel coronary
artery disease.
Which of the following is the most appropriate next step in his management?
a. Maximize medical therapy and proceed with vascular surgery.
b. Multivessel percutaneous coronary intervention (PCI) with drug-eluting stents.
c. Maximize medical therapy and cancel surgery.
d. Multivessel PCI with bare-metal stents.
e. Coronary artery bypass grafting.
20. e. Coronary artery bypass grafting.

This patient has significant ischemic disease, reduced left ventricular function, and diabetes.
Assessment of his functional status is limited by his significant claudication, but he has a high-risk
nuclear perfusion scan and multivessel coronary artery disease. Regardless of the plans for elective
surgery, revascularization of his coronary artery disease is indicated.

Coronary artery bypass grafting is preferable to PCI in light of his diabetes, as shown in the
FREEDOM trial.
21. A 60-year-old woman with hypertension presents to the emergency room with chest pain.
She describes acute onset of chest pain radiating to her back associated with leg weakness and
presyncope. Blood pressure is 162/100 mm Hg, pulse is 100 bpm, and respiratory rate is 20
breaths/minute. Cardiac exam reveals an S4 gallop with no murmur. She has no known drug
allergies. Her electrocardiogram (ECG) is shown in Figure 1.
Which of the following is the most appropriate next step?
a. Coronary angiography.
b. Chest computed tomography angiography (CTA).
c. Transthoracic echocardiogram.
d. Thrombolytics.
e. Chest X-ray.
21. b. Chest computed tomography angiography (CTA).

The patient's symptoms of chest pain radiating to the back and weakness of the lower extremities
are classic for aortic dissection, in this case complicated by extension of the dissection into the
ostium right coronary artery, as evidenced by the inferior injury current on the ECG, which may be
present in 7% of dissections. This combination may lead to misdiagnosis and delayed therapy.
Because the symptoms are so suggestive of dissection, emergent CTA of the chest or
transesophageal echocardiography imaging of the aorta and cardiac surgical consultation for the
life-threatening dissection is more appropriate than CTA (see management algorithm in Figure 2).
Chest X-ray may show a widened mediastinum, which is not very sensitive or specific and would
not provide enough information to proceed with surgery. Thrombolytic therapy would be
contraindicated, with the patient's symptoms the central factor in making the correct diagnosis.
22. Which of the following patients most likely has a myocardial infarction (MI), according to the
Third Universal Definition of MI?
a. A 48-year-old woman with 2 weeks of chest pain, palpitations, progressive dyspnea, and volume
overload. Echocardiogram reveals diffuse hypokinesis with left ventricular ejection fraction of 25%.
Endomyocardial biopsy shows diffuse necrosis, multinucleated cells, and lymphocytic and
eosinophilic infiltrates.

b. A 38-year-old obese man with 10-year tobacco history, recent upper respiratory symptoms, 2 days of
pleuritic chest pain, and diffuse ST elevations; troponin values are 0.10, 0.11, and 0.09.

c. A 56-year-old male unrestrained driver with hypertension after a motor vehicle accident and steering
wheel trauma to the chest. His echocardiogram shows anterior wall hypokinesis.

d. A 61-year-old woman with noninsulin-dependent diabetes, obesity, and sleep apnea presents with
new-onset atrial fibrillation, heart rate 130 bpm, and ST depressions in leads II, III, and aVF.

e. A 68-year-old woman with hypertension, diabetes, and end-stage renal disease presents with
shortness of breath after missing two hemodialysis sessions. Serial troponin values are 0.05, 0.04,
0.06, and 0.03. Her electrocardiogram shows left ventricular hypertrophy, but no ST/T wave changes.
22. d.A 61-year-old woman with noninsulin-dependent diabetes, obesity, and sleep apnea
presents with new-onset atrial fibrillation, heart rate 130 bpm, and ST depressions in
leads II, III, and aVF.

The Third Universal Definition of MI requires elevation in cardiac troponins, with a rise and/or fall
in levels over serial measurement, together with symptoms or signs of cardiac ischemia. These may
include new significant ST-segment T-wave changes, new left bundle branch block, new pathologic
Q waves, imaging evidence of new regional wall motion abnormalities or new loss of viable
myocardium, or intracoronary thrombus demonstrated by angiography or autopsy. Many
conditions other than cardiac ischemia may lead to elevation in troponins. Moreover, MI may be
due either to plaque rupture (type 1 MI) or due to supply-demand mismatch, with or without
underlying coronary artery disease (type 2 MI).

The patient presenting with atrial fibrillation and rapid ventricular response with
electrocardiographic evidence of ischemia most likely has a type 2 MI. The other clinical scenarios
most likely represent giant cell myocarditis, myopericarditis, troponin elevation without clear rise
and fall in the setting of renal disease, and myocardial injury due to contusion. If no active ischemia
is suspected, then these would not be considered instances of MI, according to the Third Universal
Definition of MI. Troponin elevations due to defibrillation shocks, receipt of cardiotoxic
chemotherapeutic agents, and rhabdomyolysis with cardiac involvement are other examples where
myocardial injury occurs apart from ischemia and which would not be considered MI.
(Figure 2)
23. A 53-year-old man presents to the emergency room with 1 hour of chest pain. Current vital
signs are pulse 122 bpm and irregular, blood pressure 178/102 mm Hg, respiratory rate 18
breaths/minute, Saturation of peripheral oxygen is 98% on room air. The neck veins are flat. The
lungs are clear and there is no murmur or gallop.

His electrocardiogram reveals atrial fibrillation with rapid ventricular response and 1 mm ST-
elevations in leads I and aVL.
Which of the following is most appropriate to manage his atrial fibrillation?
a. Oral or intravenous metoprolol.
b. Intravenous diltiazem.
c. Intravenous ibutilide.
d. Sublingual nifedipine.
e. Oral amiodarone.
23. a. Oral or intravenous metoprolol.

In the setting of ST-segment elevation myocardial infarction (STEMI), beta-blockers are the
preferred rate control agent for atrial fibrillation. Routine use of intravenous beta-blockers for
STEMI is not recommended, but it may be used in a hypertensive urgency or tachyarrhythmia in
the absence of other contraindications. Oral beta-blockade within 24 hours of presentation
remains a Class I indication in STEMI. Calcium channel blockers have no demonstrated benefit in
STEMI, although they could be considered for rate control in atrial fibrillation, but not if systolic
dysfunction is present.

Short-acting nifedipine for hypertension is harmful because of the potential for hypotension and
reflex tachycardia. Oral amiodarone would not be useful acutely in this setting. Ibutilide would be
inappropriate, as this patient has no indication for emergent cardioversion and the duration of
atrial fibrillation is unknown.
24. A 58-year-old woman with rheumatoid arthritis presents with 1 week of intermittent nausea
and exertional chest pressure and 30 minutes of constant intense chest pressure. Her
electrocardiogram is notable for up to 2.5 mm ST elevations in leads I, aVL, and V5-V6. She is found
to have occlusion of a large second obtuse marginal coronary artery branch and undergoes drug-
eluting stent placement. Follow-up testing shows normal cardiac function and hemoglobin A1c of
5.3%.
Which of the following is most likely to be beneficial for chronic outpatient therapy for this patient?
a. Isosorbide mononitrate 30 mg daily.
b. Verapamil sustained release 180 mg daily.
c. Metoprolol succinate 50 mg daily.
d. Enalapril 10 mg bid.
e. Spironolactone 25 mg daily.
24. c. Metoprolol succinate 50 mg daily.

Beta-blockers have a Class I indication for continuation during and after hospitalization for all
patients with ST-segment elevation myocardial infarction (STEMI) in the absence of
contraindications. Certain conditions warrant caution or avoidance of beta-blocker therapy:
decompensated heart failure, low-output states, high risk of cardiogenic shock, reactive airways
disease or active bronchospasm, and prolonged first-degree or high-grade atrioventricular block.

Angiotensin-converting enzyme (ACE) inhibitors have a Class I indication for use in patients with
STEMI with anterior involvement, heart failure, and left ventricular ejection fraction (LVEF) ≤40% in
the absence of contraindications. Angiotensin-receptor blockers may be used for patients who are
intolerant of ACE inhibitors.

For patients such as this one, who do not have anterior involvement, heart failure, or low EF, ACE
inhibitors have a Class IIa indication. Aldosterone antagonists should be used (Class I indication) in
patients with STEMI and no contraindications who are already receiving an ACE inhibitor and beta-
blocker and who have LVEF ≤40% AND either symptomatic heart failure or diabetes. This patient
would not meet these criteria.

Nitrates are not indicated for routine use after STEMI. Calcium channel blockers have not been
shown to reduce infarct size or rate of reinfarction, but may be helpful in the treatment of
ischemia, hypertension, or rapid ventricular rate in patients with atrial fibrillation who are
intolerant of beta-blockers.
25. A 53-year-old man with diabetes is admitted with an acute inferoposterior ST-segment
elevation myocardial infarction (STEMI). He undergoes primary percutaneous coronary
intervention with placement of a drug-eluting stent to the proximal left circumflex artery. He has
single-vessel disease, left ventricular end-diastolic pressure of 14 mm Hg, and an ejection fraction
(EF) of 35%.
According to the 2013 STEMI guideline, which of the following is a Class I indication for initiating an
angiotensin-converting enzyme (ACE) inhibitor within 24 hours in this patient?
a. EF of 35%.STEMI.
b. Culprit lesion location.
c. Chronic kidney disease.
d. Diabetes.
25. a. EF of 35%.

An ACE inhibitor should be administered within the first 24 hours to all patients with STEMI with an
anterior location, heart failure, or EF of ≤40%, unless contraindicated (Class I, Level of Evidence A).
Oral ACE inhibitors reduce fatal and nonfatal major cardiovascular events in patients with STEMI.
The magnitude of clinical benefit is greatest in high-risk patient subgroups. Demonstration of an
early benefit (within the first 24 hours) supports the prompt use of these agents in patients without
existing contraindications (e.g., hypotension, shock, bilateral renal artery stenosis, or history of
worsening of renal function with ACE inhibitor/angiotensin-receptor blocker exposure, renal
failure, or drug allergy). The role of routine long-term ACE inhibitor therapy in low-risk patients
after STEMI who have been revascularized and treated with aggressive lipid-lowering therapies is
less certain.
26. A 60-year-old woman presents to the office for follow-up after cardiac catheterization. She
had single-vessel disease with a moderate 50% stenosis and noncritical fractional flow reserve
(0.84). Her medical history is significant for treated hyperlipidemia. Her blood pressure is 120/60
mm Hg with a heart rate of 70 bpm, and her exam is normal. She is asymptomatic on aspirin, statin,
and beta-blocker. She asks about her risk of myocardial infarction.

Which of the following best correlated with her risk of plaque rupture?
a. High cholesterol.
b. Plaque macrophage content.
c. Plaque calcium content.
d. It is unpredictable.
e. Stenosis severity.
26. b. Plaque macrophage content.

The conversion of a stable plaque-to-plaque rupture or erosion does not have much to do with the
stenosis severity. Plaque rupture is predicted by several anatomic and systemic factors, including
inflammation, integrity of the fibrous cap, plaque lipid content, and plaque location.
27. A 40-year-old woman presents to the emergency department with chest discomfort. She
describes sharp chest pain that lasts for less than a minute and occurs at various times during the
day. Her exam is unremarkable. Her electrocardiogram is normal.
Which of the following risk scores should be used to best guide management in the emergency
department?
a. Duke treadmill score.
b. Framingham Coronary Heart Disease Risk Score.
c. HEART (History, Electrocardiogram, Age, Risk factors, and Troponin) score.
d. GRACE (Global Registry of Acute Coronary Events) score.
e. TIMI (Thrombolysis in Myocardial Infarction) score.
27. c. HEART (History, Electrocardiogram, Age, Risk factors, and Troponin) score.

Risk scores can be used to determine the short-term risk of adverse events in patients with chest
pain or definite acute coronary syndrome (ACS). The HEART score is a prospectively studied scoring
system to help emergency departments risk-stratifiy chest pain patients according to risk of major
adverse cardiac events within 6 weeks. The HEART score includes history, electrocardiogram, age,
risk factors, and troponin.

TIMI and GRACE scores are used to estimate risk of death or MI for patients with ACS, and are not
as useful for patients with chest pain of unclear etiology. The Framingham Coronary Heart Disease
Risk Score estimates risk of heart attack in 10 years in a stable patient. The Duke treadmill score is
specific to predicting coronary artery disease in a patient undergoing stress testing.
28. A 47-year-old woman presents to the emergency department with 2 days of substernal chest
pressure radiating to the jaw with minimal exertion relieved with rest. The last episode was 1 hour
prior to presentation and lasted 15 minutes. She has no medical history and takes no medication.
Her resting electrocardiogram is normal.
Which of the following is the most accurate biomarker strategy for diagnosing myocardial
infarction (MI) in this patient?
a. High-sensitivity troponin at time 0 hours.
b. Creatine kinase-myocardial isoenzyme (CK-MB) at time 0 and 6 hours.
c. Cardiac-specific troponin at time 0 and 6 hours.
d. B-type natriuretic peptide (BNP) after 6 hours of observation.
e. Myoglobin at the time of presentation (time 0).
28. c. Cardiac-specific troponin at time 0 and 6 hours.

Cardiac biomarkers are part of the universal definition of MI. Although the specificity of cardiac-
specific and high-sensitivity troponin assays are of concern, they are the biomarker of choice in
patients with suspected acute coronary syndrome (ACS). The current American Heart
Association/American College of Cardiology guideline recommends that cardiac-specific troponin
(troponin I or T when a contemporary assay is used) levels should be measured at presentation and
3-6 hours after symptom onset in all patients who present with symptoms consistent with ACS to
identify a rising and/or falling pattern (Class I).

Although high-sensitivity troponin may be positive earlier, a single measure at presentation is not
sufficient to exclude MI. With contemporary troponin assays, creatine kinase myocardial isoenzyme
(CK-MB) and myoglobin are not useful for diagnosis of ACS (Class III). Use of selected newer
biomarkers, especially BNP, may be reasonable to provide additional prognostic information (Class
IIb), but are not used for the initial diagnosis.
29. A 68-year-old man presents to the emergency department with two episodes of resting chest
pressure radiating to the jaw and arm and lasting 15 minutes in the 6 hours prior to presentation.
His pain resolved without treatment and he is now pain free. His medical history is significant for
hyperlipidemia, hypertension, and tobacco abuse. His medications include aspirin, diltiazem, and
atorvastatin. His initial high-sensitivity troponin is negative and his electrocardiogram is normal.
Which of the following is the most appropriate management for this patient?
a. Triage to observation unit for serial biomarkers.
b. Discharge with outpatient stress testing within 48 hours.
c. Admission for stress testing.
d. Dual antiplatelet therapy and catheterization within 24 hours.
e. Initiation of a glycoprotein IIb/IIIa inhibitor.
29. d. Dual antiplatelet therapy and catheterization within 24 hours.

Patients with acute coronary syndromes (ACS) can be risk stratified with clinical risk scores
including the TIMI (Thrombolysis in Myocardial Infarction) and GRACE (Global Registry of Acute
Coronary Events) scores. Although this patient has no objective ischemia, his TIMI risk score is 4
(age over 64, severe angina, three cardiac risk factors, and aspirin use), which predicts a 20% risk of
all-cause mortality, new or recurrent MI, or severe recurrent ischemia requiring urgent
revascularization at 14 days. Several randomized trials have compared an early invasive strategy
with a delayed, selective, or conservative strategy in patients with ACS.

In the TIMACS trial, patients with a GRACE score >140, but not those with a GRACE score <140, had
a reduction in the ischemic endpoint of death, MI, or stroke. Also in the TIMACS trial, death, MI,
and rehospitalization from ACS was lower with an invasive strategy in patients with positive
troponin, ST deviation, or a TIMI risk score >3.

Upstream glycoprotein inhibitor use does not improve outcomes compared to selective use at the
time of percutaneous coronary intervention. This patient should not be discharged from the
emergency room without further cardiovascular evaluation.
30. A 45-year-old male active smoker with diabetes and hypertension presents to the emergency
room with chest pain. An urgent electrocardiogram demonstrates 4 ml of ST-segment elevation in
leads V1-V4. Emergent coronary angiography identifies acute plaque rupture in the ostial left
anterior descending coronary artery with hazy thrombus and Thrombolysis In Myocardial Infarction
(TIMI) grade 1 coronary flow. After export thrombectomy, the ruptured plaque is treated with
direct stenting with a 3.5 ml x 18 ml drug-eluting stent, which is post-dilated at 18 atm.

Which of the following statements is true about the pathobiology of acute plaque rupture?
a. The majority of coronary artery atherosclerotic plaque rupture occurs at the middle portion
of the plaque away from the plaque shoulder because the middle portion of the plaque
experiences greater tensile force from flowing blood.
b. Platelets are the major source of the myeloperoxidase enzyme, which generates free radicals
and increases inflammation in rupture-prone coronary artery atherosclerotic plaques.
c. Coronary calcium does not participate in the pathobiology of coronary artery atherosclerotic
plaque rupture, and calcification is rarely present in ruptured plaques.
d. Branch points at coronary bifurcations experience high shear stress conditions that promote
coronary artery atherosclerotic plaque inflammation and infiltration of monocytes that
promote plaque rupture.
e. Macrophage-derived matrix metalloproteinases degrade the fibrous cap and promote
coronary artery atherosclerotic plaque rupture.
30. d. Branch points at coronary bifurcations experience high shear stress conditions
that promote coronary artery atherosclerotic plaque inflammation and infiltration of
monocytes that promote plaque rupture.

The majority of coronary artery atherosclerotic plaque rupture occurs at the plaque shoulder,
which experiences greater tensile force from flowing blood.

Branch points at coronary bifurcations experience low shear stress conditions that promote
coronary artery atherosclerotic plaque inflammation and infiltration of monocytes that promote
plaque rupture.

Neutrophils and macrophages are the major sources of the myeloperoxidase enzyme, which
generates free radicals and increases inflammation in rupture-prone coronary artery
atherosclerotic plaques.

Coronary calcium participates prominently in the pathobiology of coronary artery


atherosclerotic plaque rupture, and calcification is present in 80% of ruptured plaques.
31. A 65-year-old woman with hypertension and hyperlipidemia developed chest pain and
collapsed while walking her dog. Emergency medical responders used an automated external
defibrillator to administer one shock, which converted the patient from ventricular fibrillation to
sinus rhythm.
The patient is treated emergently for an ST-segment elevation myocardial infarction (STEMI) with
primary percutaneous coronary intervention, where a stent is used to treat a thrombotic occlusion
of the left anterior descending artery.Which of the following statements is correct regarding
mechanisms of coronary artery disease that led to arterial thrombosis and MI in this patient?
a. Coronary artery atherosclerotic plaque rupture is frequently clinically silent; however, silent
plaque rupture events speed plaque growth and promote luminal narrowing.
b. Coronary artery atherosclerotic plaque neovascularization increases plaque stability and
reduces plaque inflammation.
c. Superficial plaque erosion-induced coronary artery thrombosis occurs most often at sites of
macrophage-laden coronary artery atherosclerotic plaques that have a low content of smooth
muscle cells.
d. Ischemic preconditioning increases the size of the infarct zone in patients presenting with
STEMIs.
e. Distal embolization of platelet aggregates rarely occurs in non-STEMI patients.
31. a. Coronary artery atherosclerotic plaque rupture is frequently clinically silent;
however, silent plaque rupture events speed plaque growth and promote luminal narrowing.

Ischemic preconditioning reduces the size of the infarct zone in patients presenting with STEMI.
Distal embolization of platelet aggregates frequently occurs in non-STEMI patients, and
embolization is a major cause of myocardial necrosis. Superficial plaque erosion-induced
coronary artery thrombosis occurs most often within atherosclerotic plaques that have a high
content of smooth muscle cells and a low lipid content.

Coronary artery atherosclerotic plaque neovascularization decreases plaque stability and


increases plaque inflammation. Neovascularization likely increases risk of plaque rupture.
32. A 69-year-old man with a history of hypertension and stroke was admitted with a non–ST-
segment elevation myocardial infarction (NSTEMI). After medical stabilization, he was managed
with an early invasive strategy and a drug-eluting stent was placed into a thrombotic 80% stenosis
in the right coronary artery.
Which of the following regimens would be part of the optimal medical management for this patient
following drug-eluting stent implantation to treat an NSTEMI?
a. Aspirin 325 mg daily and ticagrelor 90 mg twice daily.
b. Aspirin 325 mg daily and prasugrel 10 mg daily.
c. Aspirin 81 mg daily, ticagrelor 90 mg twice daily, and rivaroxaban 5 mg twice daily.
d. Aspirin 81 mg daily and prasugrel 10 mg daily.
e. Aspirin 81 mg daily and ticagrelor 90 mg twice daily.
32. e. Aspirin 81 mg daily and ticagrelor 90 mg twice daily.

Prasugrel should not be administered to patients with a prior history of stroke or transient ischemic
attack. Aspirin 70-100 mg is the optimal dose when aspirin is combined with ticagrelor. Triple
therapy with aspirin, ticagrelor, and rivaroxaban has not been studied in acute coronary syndrome
management. Rivaroxaban reduces adverse cardiovascular events in acute coronary syndrome
patients when added to aspirin and clopidogrel.
33. A 63-year-old woman with a 2-day history of intermittent chest pain, sometimes responsive
to belching, presents to the emergency department for evaluation. Her medical history is
significant for hypertension. Her medications include amlodipine, hydrochlorothiazide, and aspirin.

Her physical examination reveals blood pressure 128/82 mm Hg, heart rate 82 bpm, and oxygen
saturation 99% on room air. Her jugular venous pressure is 6 cm water, lungs are clear, and
cardiovascular exam is normal. Her electrocardiogram shows normal sinus rhythm with nonspecific
T-wave changes. Her chest x-ray is normal. Laboratory values include two serial troponin I values of
<0.01 ng/L, normal electrolytes, creatinine 0.8 mg/dl, D-dimer 130 ng/ml, and normal blood cell
count.
Which of the following is most appropriate in her care?
a. No further testing is needed.
b. Transthoracic echocardiogram.
c. Invasive coronary angiography.
d. Ventilation/perfusion (V/Q) scan.
e. Exercise myocardial perfusion.
33. e. Exercise myocardial perfusion.

In this patient with suspected non-ST-segment elevation acute coronary syndrome, further risk
stratification is warranted. The history, physical exam, and findings are important to consider in the
initial risk stratification. This patient's Thrombolysis in Myocardial Infarction (TIMI) risk score is 1,
which gives her a risk of major adverse cardiac events of 5% in the next 14 days. Noninvasive stress
testing can be used to further risk stratify this patient. Resting transthoracic echocardiogram will
not provide information about inducible ischemia. With a normal D-dimer and low pretest
probability of pulmonary embolism, a V/Q scan is not indicated.
34. A 61-year-old woman presents to the emergency department with 2 hours of chest pain
associated with shortness of breath. Her medical history is significant for hypertension and
hyperlipidemia. Her medications include lisinopril 20 mg and hydrochlorothiazide 25 mg daily.

On exam, her blood pressure is 122/84 mm Hg, heart rate is 94 bpm, oxygen saturation is 98% on
room air, and body mass index is 24 kg/m2. Jugular venous pressure is 7 cm H2O, and cardiovascular
exam reveals normal S1 and S2 without murmurs. Lungs are clear and there is no peripheral edema.

Her electrocardiogram shows sinus rhythm with 0.5 mm ST depression in leads V4-V6. Laboratory
values include serum creatinine 1.4 mg/dl, troponin I 0.8 ng/ml, hemoglobin 12 g/dl, and N-
terminal pro-B-type natriuretic peptide 202 ng/L. She is chest-pain free on intravenous
nitroglycerin and heparin. Repeat troponin I 6 hours later is 1.1 ng/ml.
Which of the following is the most appropriate management strategy in this patient's care?
a. Pharmacologic nuclear stress test.
b. Coronary angiography.
c. Exercise stress test.
d. Coronary computed tomography (CT) angiography.
e. Transthoracic echocardiogram.
34. b. Coronary angiography.
Optimal risk stratification of patients with suspected non-ST elevation acute coronary syndrome
should include risk stratification models such as the Global Registry of Acute Coronary Events
(GRACE) or Thrombolysis in Myocardial Infarction (TIMI) scores, which employ multiple
prognostic factors to help determine management strategies. This patient's GRACE score is 159,
which places her in the high risk category for in-hospital mortality. Therefore, she should be
managed aggressively with an early invasive strategy.

Stress testing in this patient with rising troponin is not appropriate. CT angiography may be
performed in patients with intermediate risk. A normal echocardiogram will not obviate the
need for cardiac catheterization.
35. A 57-year-old woman presents to the emergency room with new-onset chest pressure that
has been escalating for the last 2 hours.
Her medical history is only remarkable for hypertension and obesity. Initial
electrocardiogram demonstrates ST-segment elevation in leads V1-V4 with associated changes
inferiorly.
She is urgently taken to cardiac catheterization where a mid left anterior descending subtotal
occlusion is successfully revascularized using a drug-eluting stent, restoring Thrombolysis in
Myocardial Infarction (TIMI) III flow.
Following percutaneous coronary intervention, a transthoracic echocardiogram demonstrates left
ventricular ejection fraction (LVEF) of 25-30% with severe hypokinesis of the anterior wall LV.
Medical therapy is initiated with atorvastatin, metoprolol, lisinopril, aspirin, clopidogrel, and
eplerenone. Telemetry on hospital day 2 reveals several brief runs of asymptomatic accelerated
idioventricular rhythm.Which of the following do you recommend for subsequent risk stratification
of sudden cardiac death?
a. Reassessment of LVEF at least 40 days post MI.
b. Invasive electrophysiology (EP) testing prior to hospital discharge.
c. Cardiac magnetic resonance imaging (MRI) prior to hospital discharge.
d. Invasive EP testing at least 40 days post MI.
e. Reassessment of LVEF at least 90 days post MI.
35. a. Reassessment of LVEF at least 40 days post MI.

The correct answer is reassessment of LVEF at least 40 days post MI. Invasive EP testing has no role
in the immediate post MI period in the absence of significant ventricular arrhythmias. The slow
idioventricular rhythm is likely related to myocardial reperfusion and does not require EP testing.
Invasive EP testing >40 days post MI may be considered for moderately decreased LVEF (35-40%) in
patients with evidence of nonsustained ventricular tachycardia detected on noninvasive
monitoring. Cardiac MRI to define scar burden has been found to be useful to predict ventricular
arrhythmias in cardiomyopathic patients. However, cardiac MRI has not yet been integrated into
the guidelines for risk stratification post MI.

The 2013 guideline for the management of ST-elevation myocardial infarction indicates that repeat
assessment of LVEF at least 40 days post MI carries a Class I indication for risk stratification of
sudden cardiac death. Primary prevention implantable cardioverter-defibrillator (ICD) implantation
during the infarct-related hospitalization has not been demonstrated to be beneficial (DEFINITE
trial). Allowing for a period of myocardial healing post MI may avert the need for subsequent ICD
implantation.

Greater than 40 days post MI, LVEF should be reassessed. If the LVEF does not improve remains </=
35% with class II or III symptoms, or remains < 30% independent of symptoms, despite guidelines
directed medical therapy, then this patient would become a candidate for pr primary prevention
ICD implant.
36. A 52-year-old man presents to the emergency room after 3 hours of intermittent sharp
stabbing chest pain noted when he was raking leaves. His medical history is significant for
hypertension. He denies tobacco, alcohol, or drug use. His current medications include
hydrochlorothiazide.

His physical exam reveals blood pressure of 148/94 mm Hg and heart rate of 72 bpm. His
cardiovascular exam is unremarkable. His electrocardiogram shows normal sinus rhythm without
ST- or T-wave changes. His chest x-ray shows normal cardiac silhouette and clear lung fields.
Which of the following is the most appropriate next step?
a. Creatine kinase-myocardial band (CK-MB).
b. N-terminal pro-B-type natriuretic peptide (NT-proBNP).
c. Urine toxicology.
d. High-sensitivity C-reactive protein (hs-CRP).
e. Troponin I.
36. e. Troponin I.

Cardiac-specific troponin should be performed in patients with clinical findings or symptoms that
may be related to potential acute coronary syndrome (ACS) (Class Ia). It should be measured at
presentation and 3-6 hours after symptom onset in all patients with suspected ACS. If it is negative
within the first 6 hours of onset of symptoms, it should be repeated in patients with
electrocardiographic changes and/or intermediate/high risk clinical features. In order to diagnose
myocardial necrosis, it is important to determine both the peak troponin level and the pattern of
change.

CK-MB and myoglobin are not useful for diagnosis of ACS (Class III). NT-proBNP may be reasonable
for additional prognostic information (Class IIb). Urine toxicology should be performed in patients
with suspected drug use, as this may be a causal contributor to ACS.
37. A 62-year-old Caucasian woman with type 2 diabetes presents to the emergency department
with complaints of constant substernal chest pressure at rest for the last hour, briefly relieved by
nitroglycerin. She is not on antiplatelet therapy. An electrocardiogram shows lateral precordial ST-
segment depressions and T-wave inversions. She has no known drug allergies. The troponin I level
is 2.12 ng/ml.
Which of the following pharmacotherapeutic strategies is the most appropriate next step for this
patient?
a. Ticagrelor 180 mg.
b. Aspirin 325 mg and clopidogrel 600 mg.
c. Immediate eptifibatide bolus followed by infusion as monotherapy.
d. Aspirin 162 mg and prasugrel 60 mg.
e. Clopidogrel 600 mg.
37. b. Aspirin 325 mg and clopidogrel 600 mg.

This woman is experiencing a non-ST-segment elevation myocardial infarction (NSTEMI).


Clopidogrel and ticagrelor at the described loading and maintenance doses are both reasonable
therapies for NSTEMI. However, they have been studied in conjunction with and should be
administered along with aspirin. Prasugrel may be given to patients undergoing percutaneous
coronary intervention (PCI) in the setting of NSTEMI, but is not guideline-recommended to be
administered until the anatomy is known and it is certain that the patient will be underoing PCI. In
the ACCOAST trial, treatment with prasugrel in NSTEMI before PCI did not reduce ischemic events
and was associated with more bleeding. The best answer is aspirin in conjunction with clopidogrel.
Glycoprotein IIb/IIIa antagonists are only administered on top of oral antiplatelet therapy.
38. A 40-year-old man with a history of type 2 diabetes is seen in the office. His blood pressure is
123/75 mm Hg. Labs include a total cholesterol of 130 mg/dl and high-density lipoprotein
cholesterol of 40 mg/dl. He is a lifelong nonsmoker. He is active, but overweight with a body mass
index of 29 kg/m2. He is interested in reducing his risk of developing complications of
atherosclerotic cardiovascular disease (ASCVD), but wants to be sure any new medications are
worth the potential side effects.
Which of the following do you recommend to reduce the risk of ASCVD in this patient?
a. High-intensity statin therapy.
b. Dietary modifications alone.
c. Fenofibrate.
d. Moderate-intensity statin therapy.
e. Ezetimibe.
38. d. Moderate-intensity statin therapy.

Using the ASCVD risk calculator (estimator), this patient has a 10-year calculated risk of 1.2% and a
50% lifetime risk. Although primary prevention is reserved for nondiabetic patients with a 10-year
risk ≥7.5%, the fact that this patient has diabetes means that moderate-intensity statin therapy is
recommended (Class I, Level of Evidence A). High-intensity statin therapy would be indicated in this
diabetic patient if the 10-year risk was ≥7.5%.y.
39. A 66-year-old man is seen in clinic for follow-up after hospitalization for an acute coronary
syndrome. Current medications include atorvastatin 80 mg, ticagrelor 90 mg bid, aspirin 81 mg,
metoprolol 25 mg bid, and lisinopril 10 mg daily. He smoked one-half pack of cigarettes daily for
the past 40 years.

His current lipid panel includes total cholesterol 150 mg/dl, triglycerides 199 mg/dl, low-density
lipoprotein 61 mg/dl, and high-density lipoprotein 49 mg/dl. Hemoglobin A1c is 6.0%.
Which of the following steps will best reduce his risk of future myocardial infarction (MI)?
a. Ezetimibe.
b. Omega-3 fatty acids.
c. Varenicline.
d. Coenzyme Q10.
e. Metformin.
39. c. Varenicline.

Lifestyle modification including smoking cessation can result in significant improvement in


atherosclerotic cardiovascular risk. A meta-analysis of smoking cessation in patients with MI found
it was associated with a 50% reduction in cardiovascular risk. Therefore, varenicline is the best
answer, because it has been proven to be efficacious for smoking cessation. Ezetimibe in addition
to statin was associated with an approximately 2% absolute risk reduction (number needed to treat
= 50) in atherosclerotic cardiovascular disease events in the IMPROVE-IT trial; given his lipid panel,
this would not likely be as large a benefit as smoking cessation. Adding metformin is not indicated
with a hemoglobin A1c of 6% and would not be expected to reduce his MI risk. Coenzyme Q10 and
omega-3 fatty acids have not been shown to reduce global cardiovascular risk.
40. A 63-year-old woman with a history of hypertension presents with an acute coronary
syndrome. Coronary angiography demonstrates a 90% lesion in her posterior descending artery.
She subsequently undergoes stent placement.

Labs include peak troponin of 1.2 ng/ml and hemoglobin A1c of 5.6%. Fasting lipid profile includes
total cholesterol of 160 mg/dl, high-density lipoprotein (HDL) of 38 mg/dl, low-density lipoprotein
(LDL) of 67 mg/dl, and triglyceride (TG) of 240 mg/dl.
Which of the following is the most appropriate lipid management strategy?
a. Fenofibrate 145 mg qd.
b. Fish oil 2 g qd.
c. Ezetimibe 10 mg qd.
d. Niacin 500 mg qd.
e. Atorvastatin 80 mg qd.
40. d. Niacin 500 mg qd.

It is important to obtain a fasting lipid profile on all patients presenting with acute coronary
syndrome. High-intensity statin therapy is indicated for all patients ages <75 years with
atherosclerotic cardiovascular disease (ASCVD), even with a "normal" LDL. Despite low HDL and
elevated TG, initial therapy with statin is preferred over the other options.

When added to statin monotherapy, the addition of niacin or fenofibrate has not been shown to
reduce CV events despite improvement in lipid profile. While ezetimibe as add-on therapy to statin
reduced CV events in the IMPROVE-IT trial, it should not be used as monotherapy in a patient who
is statin naive. To date, fish oil has not been definitively shown to reduce CV events in patients with
ASCVD and should not be used over statin therapy.
41. You are counseling a 58-year-old man prior to discharge from the hospital following an acute
myocardial infarction. He received primary percutaneous coronary intervention with a drug-eluting
stent to the left anterior descending artery. Left ventricular ejection fraction is 55%. He had no
known medical history prior to admission. His current medications include atorvastatin 80 mg,
lisinopril 5 mg, metoprolol succinate 25 mg, aspirin 81 mg, and ticagrelor 90 mg bid.
In addition to reviewing the importance of taking all his medications, which of the following
recommendations is the most appropriate?

a. Limit total calories to <2000 per day.


b. Reduce saturated fats, trans fatty acid, and cholesterol intake.
c. Recommend antioxidant vitamin supplements.
d. Increase monounsaturated fat intake and reduce cholesterol intake.
e. Limit sodium intake to <2000 mg per day.
41. b. Niacin 500 mg qd.

Dietary modification is recommended for all patients with acute coronary syndrome (ACS).
Reducing saturated fats to <7% of total calories, eliminating or reducing trans fatty acids, and
limiting dietary cholesterol to <200 mg daily are all likely to improve cardiovascular health.

Limiting total calories alone is not specific enough to reduce saturated fat intake. Sodium
restriction to <2000 mg/day is not indicated, as he does not have left ventricular systolic
dysfunction or heart failure, although lowering sodium intake by 1000 mg/day from baseline or to
approximately 2400 mg/day with the DASH diet may reduce hypertension.

Antioxidants, including vitamin E, vitamin C, and beta carotene, have not been shown to confer any
benefit in secondary prevention of ACS. Increased monounsaturated fat alone will not ensure a
reduction in dietary saturated fat and trans fatty acid intake.
42. A 72-year-old man with persistent atrial fibrillation presents to the emergency room with a
non–ST-segment elevation myocardial infarction. He has a history of hypertension and diabetes.

The patient undergoes angiography, and a drug-eluting stent is placed in the left circumflex to treat
a hazy area of acute plaque rupture.
Which of the following is the best maintenance therapy for this patient?
a. Aspirin 81 mg, clopidogrel 75 mg, and oral anticoagulation.
b. Aspirin 325 mg.
c. Aspirin 325 mg and clopidogrel 75 mg therapy.
d. Clopidogrel 75 mg.
e. Aspirin 325 mg and oral anticoagulation.
42. a. Aspirin 81 mg, clopidogrel 75 mg, and oral anticoagulation.

In the presence of atrial fibrillation and an acute myocardial infarction, the addition of an oral
anticoagulant remains controversial following percutaneous coronary intervention. The addition of
an oral anticoagulation should be guided by use of the CHA2DS2-VASc score. Individualization can
also be determined by the HAS-BLED scoring system for risk of bleeding.
43. A 70-year-old man on the trauma service is being evaluated because of a positive troponin.
He was in a motor vehicle accident yesterday and fractured his femur. He underwent surgical open
reduction and internal fixation this morning. He has diabetes and hypertension, but no known
coronary artery disease. There was no steering wheel contact; he was wearing his seat belt. He has
no symptoms.

His examination is normal except for pallor and his now casted left leg. His electrocardiogram
shows sinus rhythm and nonspecific T-wave flattening over the precordial leads. His laboratory
tests are notable for a hemoglobin of 7.5 mg/dl. The troponin is 1.2 ng/dl.
Which of the following is the most appropriate next step?
a. Enoxaparin 40 mg daily.
b. Aspirin 325 mg daily.
c. Clopidogrel 300 mg loading dose.
d. Heparin 60 U/kg bolus.
e. Ticagrelor 180 mg loading dose.
43. a. Enoxaparin 40 mg daily.

This patient has an asymptomatic rise in his troponin in the absence of symptoms and nonspecific
changes on his electrocardiogram. Moreover, he has undergone significant trauma and is very
anemic. The low-grade troponin is most likely due to demand ischemia. The clinical scenario is not
consistent with an acute coronary syndrome and therefore does not require treatment for an acute
coronary syndrome (e.g., antiplatelet agents, intravenous antithrombins). His greatest risk is deep
venous thrombosis, and he should receive deep vein thrombosis prophylaxis (e.g., enoxaparin).
44. A 50-year-old woman presents with chest pain. It occurs with exertion and is relieved by rest.
Her history is notable for hypertension; her father died suddenly at 55 years of age of unknown
causes. She completes 6 minutes of a Bruce protocol stress test with myocardial perfusion (MIBI)
that demonstrates 2 mm ST-depressions in the lateral leads and a reversible defect in the
inferolateral wall. Her blood pressure and heart rate responses are normal. She undergoes
coronary angiography that is notable for a left dominant coronary circulation with a 70% mid left
anterior descending (LAD) lesion and luminal irregularities of the first obtuse marginal branch.
Fractional flow reserve of the LAD lesion is 0.85.
Which of the following is the most likely cause of her chest pain?
a. Coronary vasospasm.
b. 70% mid LAD lesion.
c. Pericarditis.
d. Coronary dissection.
e. Microvascular angina.
44. e. Microvascular angina.

Coronary microvascular dysfunction is an important prognostic factor in a wide range of diseases.


In this case, microvascular angina is the manifestation of her atherosclerotic coronary artery
disease. Evidence for ischemia includes her electrocardiographic ST depressions and stress MIBI
changes. Pericarditis would not produce the ST changes or the stress MIBI changes. Coronary
vasospasm generally is not provokable with exercise stress testing and typically occurs in the early
morning hours; it classically involves the epicardial coronary arteries, although microvascular
spasm has been speculated. Coronary dissection usually presents as a myocardial infarction, and
the diagnosis is generally apparent on coronary angiography, although intravascular ultrasound and
ocular coherence tomography may be required for definitive diagnosis.
45. A 45-year-old woman presents to your office with a 6-month history of chest pain. She
reports episodes of chest pain lasting 10-15 minutes that occur with exertion. The pain radiates to
her left shoulder and she becomes short of breath. She has no family history of coronary artery
disease and she has smoked for 10 years. She does not have a history of diabetes or hypertension.

Her physical examination is unremarkable, except for a body mass index of 32. A 12-lead
electrocardiogram (ECG) is normal. A previous cardiologist had performed a treadmill stress test,
during which she developed chest pain and a 2 mm ST-depression in leads V5 and V6. She undergoes
diagnostic cardiac catheterization, and her coronary arteries are free of stenosis except for a 20%
lesion in the circumflex. Echocardiogram shows normal left ventricular and right ventricular (RV)
function and normal RV systolic pressure.
What is the most likely cause of her symptoms?
a. Gastroesophageal reflux disease (GERD).
b. Stress-induced cardiomyopathy.
c. Chronic thromboembolic pulmonary hypertension (CTEPH).
d. Pericarditis.
e. Coronary microvascular dysfunction.
45. e. Gastroesophageal reflux disease (GERD).

Her history is consistent with angina. Although she does not have any significant coronary artery
stenosis expected to lead to flow limitation, coronary microvascular dysfunction is the most likely
cause of her symptoms. This syndrome occurs in women more than men, can present with classical
angina, and can be associated with ST depression provoked by exercise, and it occurs in the
absence of significant epicardial coronary stenoses.

CTEPH is not typically associated with anginal type symptoms or with exercise-induced ischemia.
Stress-induced cardiomyopathy (Takotsubo) results in wall motion abnormalities on
echocardiography, usually apical ballooning. There is no evidence of pericarditis, because her ECG
is normal and chest pain is only exertional. While GERD is a common cause of chest pain, it would
not result in exercise-induced symptoms or ECG changes.
46. A 50-year-old woman presents to your office for a second opinion. She reports a 3-year
history of exertional chest discomfort. Her previous cardiologist had performed a treadmill stress
test during which she developed a 1.5 mm ST-depression in 3 leads, after which he recommended
the patient undergo a cardiac catheterization. Her coronary arteries had no epicardial stenoses and
the cardiologist attributed the chest pain to anxiety. The patient continues to have exertional chest
pain, and she is willing to undergo additional noninvasive diagnostic testing, because she remains
concerned about this symptom. Her physical examination is unremarkable.
Which of the following noninvasive diagnostic tests do you recommend?
a. Computed tomography (CT) angiography of the coronary arteries.
b. Exercise thallium.
c. Resting transthoracic echocardiogram.
d. Assessment of coronary artery calcium by CT.
e. Repeat stress treadmill with electrocardiogram (ECG) monitoring.
46. b. Exercise thallium.

The patient may be presenting with coronary microvascular dysfunction. Patients with this
condition can develop ischemia in the absence of significant epicardial coronary artery stenosis.
Such patients may have a reversible perfusion defect as assessed by stress thallium perfusion
study, though absence of this finding does not exclude that diagnosis. There would be little utility in
repeating the treadmill test with ECG monitoring, because this was already positive. Neither CT
angiography or assessment of coronary artery calcium would be helpful given the recent
demonstration of no significant stenosis at left heart catheterization. A resting transthoracic
echocardiogram would not establish the diagnosis of coronary microvascular dysfunction.
47. A 48-year-old woman presents with exertional chest pain with associated dyspnea. Following
a stress perfusion study showing a reversible defect consistent with ischemia, she undergoes
cardiac catheterization, including assessment of hemodynamics. Her pulmonary artery pressure
(PAP) is 30/15/22 mm Hg and cardiac index is 2.5 L/min/m2. Coronary angiography reveals no
significant epicardial stenoses. The ratio of the coronary flow following administration of
adenosine, to that at baseline, is 2. A catheter is placed into her left ventricle, and the left
ventricular end-diastolic pressure (LVEDP) is 12 mm Hg.
Which of the following diagnoses would be most consistent with these findings?
a. Coronary microvascular dysfunction.
b. Noncardiac chest pain.
c. Pulmonary embolism.
d. Heart failure.
e. Pulmonary hypertension.
47. a. Coronary microvascular dysfunction.

A coronary flow reserve (CFR) of 2.5 is abnormal and consistent with coronary microvascular
dysfunction. Her LVEDP is low, which is not consistent with heart failure. Her mean PAP is 25 mm
Hg, so she does not have pulmonary hypertension. While we cannot exclude pulmonary embolism
to be the cause of her symptoms, she does not have associated pulmonary hypertension. The
finding of a low CFR makes coronary microvascular dysfunction a more likely explanation. The
patient has abnormal CFR and a positive stress test, making noncardiac chest pain less likely.
48. You are asked to consult on a 55-year-old man in the emergency department with chest pain.
He describes a dull chest discomfort that started while he was climbing the attic stairs to put away
some boxes about 2 hours ago. He was sweating and breathing heavily from the work when the
pain started. The discomfort became so intense that he thought he was going to pass out and
gradually resolved over 10 minutes when he sat down to catch his breath. Medical history includes
hypertension managed with amlodipine and gastroesophageal reflux disease managed with
omeprazole.

His electrocardiogram is shown in Figure 1. Blood work drawn on arrival in the emergency
department included chemistries, which were normal, and cardiac troponin I, which was
undetectable.

Which of the following laboratory tests should be obtained within the next 6 hours?
a. Creatine kinase-myocardial band.
b. High-sensitivity cardiac troponin I.
c. Heart-type fatty acid binding protein.
d. Myoglobin.
e. Lactate dehydrogenase.
48. b. High-sensitivity cardiac troponin I.

Injury to cardiac myocytes results in loss of cell membrane integrity and subsequent diffusion of
intracellular proteins into the extracellular space, where they are detectable by laboratory
assays on plasma. These biomarkers of myocyte injury include myoglobin (present in all muscle
cells), lactate dehydrogenase (present in all muscle cells), creatine kinase (present in all muscle
cells but with subtypes that are more prevalent in cardiac muscle cells), heart-type fatty acid
binding protein (a subtype of a low molecular weight protein specific to cardiac muscle), and
cardiac troponin I (present only in cardiac muscle cells). Troponin T may be expressed by some
skeletal muscle cells, but in most clinical settings will have similar specificity to troponin I.

The high specificity of cardiac troponin for cardiac muscle injury and predictable kinetics make it
the preferred biomarker to confirm or exclude the diagnosis of myocardial infarction. Cardiac
troponins generally will be detectable first in the blood 2-3 hours after the event in 80% of
patients and within 6 hours of the event in essentially all patients. Serial measurements, as in
this patient with undetectable troponin immediately after the event, are therefore critical for
ruling out the diagnosis of myocardial infarction and for ruling in the diagnosis by demonstrating
the characteristic rise and fall of troponin in the blood.
49. A 60-year-old man with a 60 pack-year history of smoking presents to the emergency
department with 2 hours of crushing substernal chest pain. His electrocardiogram shows 3 mm ST
elevation in the anterior leads. He is taken urgently to the cardiac catheterization laboratory.
Which of the following strategies would require additional antithrombotic therapy during primary
percutaenous coronary intervention (PCI)?
a. Unfractionated heparin.
b. Bivalirudin.
c. Fondaparinux.
d. Unfractionated heparin with eptifibatide.
e. Unfractionated heparin with abciximab.
49. c. Fondaparinux.

Fondaparinux is not recommended as the sole anticoagulant during primary PCI due to an
increased risk of catheter thrombosis (Class III recommendation). Unfractionated heparin with or
without concomitant intravenous glycoprotein IIb/IIIa therapy (including abciximab and
eptifibatide), and bivalirudin, are acceptable anticoagulant therapies during primary PCI.
50. A 31-year-old man with no significant medical history presents to the emergency department
with sudden-onset severe substernal chest pain that began 15 minutes prior to presentation. His
friend admits that they had been smoking marijuana that was laced with cocaine. The patient
appears somewhat disheveled and irritable. His pupils are dilated, heart rate is 105 bpm, and blood
pressure is 163/106. He was started on nitroglycerin, but has persistent chest pain. An
electrocardiogram shows ST elevations in leads V3-V6.
Which of the following medications would be appropriate to administer?
a. Flumazenil.
b. Lorazepam.
c. Naloxone.
d. Metoprolol.
e. Dabigatran.
50. b. Lorazepam.

Benzodiazepines with or without nitroglycerin may be used to manage hypertension and


tachycardia in patients who present with non-ST elevation acute coronary syndromes (NSTE-ACS)
and signs of acute cocaine or methamphetamine intoxication. For STE-ACS, lorazepam would be
the most reasonable choice of the options listed, although the primary focus would be on
emergent coronary angiography and percutaneous coronary intervention, if indicated.

Per the NSTE-ACS guidelines, "Beta-blockers should not be administered to patients with ACS with
a recent history of cocaine or methamphetamine use who demonstrate signs of acute intoxication
due to the risk of potentiating coronary spasm." Naloxone is used to treat opioid overdose and
flumazenil is used to treat benzodiazepine overdose, so neither would be appropriate here.
Dabigatran is not indicated for the management of ACS.
51. A 67-year-old woman with history of diabetes and asthma is admitted to the cardiac intensive
care unit after percutaneous intervention with drug-eluting stent placement for an ST-elevation
myocardial infarction (STEMI) involving the right coronary artery. She has not had any recurrent
chest discomfort since her procedure. Her vital signs are temperature 37.5°C, heart rate 116 bpm,
blood pressure 153/73 mm Hg, respiratory rate 24 breaths/minute, and oxygen saturation 96%.
Physical examination is notable for an irregularly irregular heart rhythm, no jugular venous
pressure, and moderate expiratory wheezing. Electrocardiogram demonstrates atrial fibrillation
with a rapid ventricular response, but isoelectric ST segments and no Q waves. Her medications
include aspirin, clopidogrel, and atorvastatin.
Which of the following would be the next best medication to consider?
a. Spironolactone.
b. Metoprolol.
c. Amiodarone.
d. Diltiazem.
e. Nitroglycerin.
51. d. Diltiazem.

Certain calcium channel blockers may be useful for the treatment of angina, hypertension, and
atrial tachyarrhythmias in select patients after STEMI when beta-blockers (or nitrates) are
ineffective, not tolerated, or contraindicated and in patients who have no signs of congestive heart
failure, left ventricular dysfunction, or atrioventricular block. In this patient with recent STEMI,
atrial fibrillation with rapid ventricular response, and elevated blood pressure, diltiazem would be a
good choice to slow her ventricular rate and help control blood pressure. There is no evidence of
active ischemia, and thus immediate electrical cardioversion is not necessary.

Metoprolol is contraindicated given the history of asthma and evidence of active bronchospasm.
The duration of atrial fibrillation is unknown, thus amiodarone would not be a good choice at this
point, as it also can cause cardioversion and other attempts at rate control have not been made.
Nitroglycerin is not ideal in the setting of tachycardia and would not help to control heart rate.
Aldosterone antagonists are recommended for patients with STEMI who are already receiving an
angiotensin-converting enzyme inhibitor and beta-blocker and who have a left ventricular ejection
fraction ≤40% and either symptomatic heart failure or diabetes. This patient would not qualify for
an aldosterone antagonist at this time.
52. A 51-year-old woman is admitted to the cardiac intensive care unit (ICU) immediately
following primary percutaneous coronary intervention for an anterior wall myocardial infarction
(MI). She was intubated for respiratory distress en route to the hospital and no history is available.
Current vital signs are pulse 108 bpm, respiratory rate 12 breaths/minute with 100% fraction of
inspired oxygen from the ventilator, blood pressure 98/70 mm Hg, and oxygen saturation 99%.
Weight is 240 lbs. Physical exam reveals acanthosis of the neck and bilateral rales; the peripheral
pulses are weak.

Lab exam results were remarkable for random glucose 264 mg/dl; there was no anion gap.
Creatinine and other chemistries were within normal limits. An infusion with insulin is started for
control of hyperglycemia.
Which of the following is the most appropriate target of glucose in this patient?
a. 181-220 mg/dl.
b. <80 mg/dl.
c. 221-280 mg/dl.
d. 81-110 mg/dl.
e. 111-180 mg/dl.
52. e. 111-180 mg/dl.

Nearly one-quarter of patients with ST-elevation MI (STEMI) have diabetes, and up to one-third of
those may be diagnosed at the time of hospitalization for STEMI. Diabetic patients have higher
mortality from STEMI, and hyperglycemia is a predictor of early mortality. However, there is a U- or
J-shaped relationship between serum glucose and death, with the risk of mortality similar for either
hypoglycemia (random glucose <70 mg/dl) or hyperglycemia (variously defined as random glucose
>140, >180, or <200 mg/dl).

The NICE-SUGAR trial randomized 6,104 ICU patients to either intensive blood glucose control
(target blood glucose 81-108 mg/dl) or conventional blood glucose control (target blood glucose
<180 mg/dl) and found lower mortality (24.9% vs. 27.5%, p = 0.02) for the conventional glucose
control strategy, with fewer episodes of hypoglycemia (0.5% vs. 6.8%, p < 0.001).

Therefore, "modest" rather than "intensive" glucose control, that is, target <180 mg/dl with
avoidance of hypoglycemia, is the current recommendation for critically ill patients with STEMI.
53. A 68-year-old Caucasian man is seen in the clinic for cardiovascular risk assessment. He has a
history of hypertension and type 2 diabetes . He has never smoked. His medications include
metformin 500 mg bid and lisinopril 10 mg qd. His blood pressure is 130/80 mm Hg. His
examination is otherwise unremarkable. His lipid profile shows total cholesterol 161 mg/dl, high-
density lipoprotein 45 mg/dl, low-density lipoprotein 80 mg/dl, and triglyceride 180 mg/dl. His
hemoglobin A1c is 6.4%.
Which of the following is the most appropriate next step?
a. Start glipizide 5 mg qd.
b. Start fenofibrate 145 mg qd.
c. No additional therapy.
d. Start atorvastatin 40 mg qd.
e. Increase lisinopril to 20 mg qd.
53. d. Start atorvastatin 40 mg qd.

Initiating statin therapy is appropriate for diabetics 40-75 years of age with low-density lipoprotein
cholesterol >70 mg/dl. In this patient whose 10-year risk of atherosclerotic cardiovascular disease is
11.2% based on pooled cohort equation, high-intensity statin therapy is recommended. In the
ACCORD trial, intensifying glycemic control to target A1c <6% was associated with higher rates of
hypoglycemia and increased mortality and did not significantly reduce major cardiovascular events.
Similarly, the addition of fenofibrate was not associated with significant reduction in rate of
cardiovascular events. This patient's blood pressure is at goal; up-titration of lisinopril is not
indicated.
54. A 74-year-old man with a history of hypertension is brought to the emergency department
with chest pain and an acute inferior wall myocardial infarction. His only medication on admission
is lisinopril 40 mg daily. He undergoes drug-eluting stent placement without complications and is
placed on aspirin 81 mg daily, clopidogrel 75 mg daily, atorvastatin 80 mg daily, and metoprolol 50
mg twice daily. Prior to discharge, he experiences similar chest pain and diaphoresis. His
electrocardiogram shows ST elevation in the inferior leads. Stent thrombosis is diagnosed at
catheterization and successfully remedied.
Which of the following factors may have contributed to development of subacute stent
thrombosis?
a. Inadequate dosing of aspirin.
b. Heparin-induced thrombocytopenia.
c. Nickel allergy.
d. Resistance to clopidogrel.
e. Resistance to aspirin.
54. d. Resistance to clopidogrel.

On March 12, 2010, the Food and Drug Administration approved a new label for clopidogrel with a “boxed
warning” about the diminished effectiveness of clopidogrel in patients with impaired ability to convert the drug
into its active metabolite. Patients with decreased cytochrome P450 2C19 (CYP2C19) function because of genetic
polymorphisms metabolize clopidogrel poorly and have higher rates of cardiovascular events after acute coronary
syndrome and percutaneous coronary intervention (PCI) than patients with normal CYP2C19 function. The
warning also notes that tests are available to identify patients with genetic polymorphisms and that alternative
treatment strategies should be considered for patients who are poor metabolizers. The clopidogrel boxed
warning leaves the issue of whether to perform CYP2C19 testing up to the individual physician. It does not
specifically require genetic testing or other changes in evaluation or treatment and does not imply that there are
solid evidence-based reasons for such actions. Rather, it serves to inform clinicians of genetic variations in
response to clopidogrel and to emphasize that clinicians should use this knowledge to make decisions about how
to treat individual patients. At the present time, the evidence base is insufficient to recommend routine genetic
testing in patients undergoing PCI. There may be a potential role for genetic testing for patients undergoing
elective high-risk PCI procedures (e.g., unprotected left main, bifurcating left main, or last patent coronary
artery).

The recommended maintenance dose of aspirin is 70-100 mg following stent implantation. Higher doses have
been shown to increase bleeding risk without reducing major adverse cardiac events. The prevalence of aspirin
resistance is controversial, but has not been linked to stent thrombosis. Heparin-induced thrombocytopenia
increases thrombosis risk in general, but has not been linked to subacute stent thrombosis. Most modern stents
are constructed of cobalt-chromium alloys rather than with nickel titanium; furthermore, there is no proven
association between nickel allergy and stent thrombosis.
55. A 69-year-old man is hospitalized for primary percutaneous coronary intervention for an ST-
segment elevation myocardial infarction. His medical history includes hypertension and a previous
ischemic stroke. His current medications include aspirin 81 mg, carvedilol 12.5 mg bid, lisinopril 20
mg daily, and atorvastatin 80 mg daily.
Which of the following best explains the benefits of ticagrelor over clopidogrel?
a. Ticagrelor is associated with lower rates of side effects.
b. Ticagrelor is associated with lower stroke risk.
c. Ticagrelor is associated with lower rates of stent thrombosis.
d. Ticagrelor is associated with fewer drug-drug interactions.
e. Ticagrelor is associated with lower bleeding risk.
55. c. Ticagrelor is associated with lower rates of stent thrombosis.

In the PLATO trial, ticagrelor was associated with lower rates of death, stroke, and stent thrombosis
compared with clopidogrel in acute coronary syndrome patients. However, non-coronary artery
bypass grafting major bleeding rates were higher with ticagrelor compared with clopidogrel,
although overall bleeding rates were similar. Ticagrelor and clopidogrel were associated with a
similar risk of stroke, although there were more hemorrhagic strokes with ticagrelor (0.2% vs.
0.1%). Ticagrelor is metabolized through cytochrome P450 3A and cannot be combined with strong
inducers or inhibitors of that enzyme. Ticagrelor is associated with more side effects, including
dyspnea and bradycardia. Because of increased bleeding when combined with higher doses of
aspirin, ticagrelor should only be combined with aspirin 81 mg.
56. A 65-year-old retired biology professor presents for cardiac catheterization in the setting of a
recently positive stress test. You discuss the procedure in detail and obtain informed consent for
coronary angiography plus percutaneous intervention if needed. He has been reading about
antiplatelet therapy and asks if you will screen him for genetic mutations related to clopidogrel
metabolism.
Which of the following is appropriate routine use of genetic testing to screen patients treated with
clopidogrel after percutaneous coronary intervention?
a. Recommended in those receiving drug-eluting stents only.
b. Recommended, as the evidence suggests that this reduces adverse outcomes.
c. Recommended in those with concomitant atrial fibrillation.
d. Recommended in patients who develop a rash after taking clopidogrel.
e. Not recommended, as the evidence to support this is insufficient.
56. e. Not recommended, as the evidence to support this is insufficient.

Clopidogrel is a prodrug that requires conversion into an active metabolite. The major enzyme
involved in the generation of the active metabolite is cytochrome P450 2C19 (CYP2C19). Among
several common polymorphisms of CYP2C19 are loss of function mutations that result in a
premature stop codon and a truncated protein. In theory, patients with these polymorphisms who
are taking clopidogrel would have a reduced amount of the active metabolite present for platelet
inhibition.

Because of concerns over reduced activity in these patients, the Food and Drug Administration
issued a boxed warning for clopidogrel in 2010. Multiple studies since then have evaluated the
relationship between the presence of these polymorphisms and adverse outcomes in patients, and
the data are inconclusive. Therefore, the routine use of genetic screening in patients undergoing
percutaneous coronary intervention is not recommended.

Genetic testing for CYP2C19 mutations has nothing to do with allergy or rash to clopidogrel.
Screening is not recommended in patients receiving bare metal or drug-eluting stents. The
presence of atrial fibrillation does not affect the recommendation against screening.
57. A 45-year-old man presents to the emergency department with crushing substernal chest
pain and ST elevation in the inferior leads on the electrocardiogram. His blood pressure is 185/100
mm Hg and his heart rate is 100 bpm. The cardiac catheterization laboratory is activated, aspirin
325 mg is administered orally, and a heparin bolus and drip are started. The patient is anxious and
reports no medical history, no medication allergies, and he takes a multivitamin daily and sildenafil
100 mg as needed for erectile dysfunction (most recently the evening prior to presentation). A
urinary toxicology screen returns that is positive for cocaine.

Which of the following medications would be appropriate to treat the patient's hypertension, while
waiting for coronary angiography?
a. Sodium nitroprusside.
b. Nitroglycerin.
c. Lorazepam.
d. Nifedipine.
e. Metoprolol.
57. c. Lorazepam.

Cocaine-induced chest pain may be related to myocardial infarction (MI), aortic dissection, or
anxiety. Among those with MI, this may be due to coronary vasospasm and/or thrombotic
occlusion of one or more coronary vessels. Patients with acute coronary syndromes and severe
hypertension upon presentation should be treated appropriately with aspirin, heparin (assuming
aortic dissection has been ruled out or is considered unlikely), and benzodiazepenes to control
central sympathetic outflow and anxiety.

Beta-blockers are contraindicated in cocaine-related MI, as they may result in increased coronary
vasospasm and decreased coronary blood flow due to unopposed alpha activity. Combined alpha-
and beta-blocker agents, such as labetalol or carvedilol, may be acceptable alternatives. Short-
acting nifedipine is contraindicated in acute MI. While nitrates to control blood pressure and chest
pain would be appropriate in cocaine-associated MI, they are contraindicated in this patient due to
recent use of sildenafil.
58. A 55-year-old man presents to the emergency department with progressive fatigue and
shortness of breath for the past 3 weeks. In the past 2 days he has also developed some lower
extremity edema and feels short of breath in bed at night.

His medical history is notable for deep vein thrombosis (DVT), for which he is currently maintained
on apixaban, and he has not missed any doses. He had a recent echocardiography, demonstrating a
left ventricular ejection fraction of 35%. On examination, his heart rate is 165 bpm and irregularly
irregular. His blood pressure is 105/75 mm Hg, jugular venous pressure is 7 cm, and there are
bibasilar crackles on lung exam.
Cardiac exam is notable for the irregularly irregular rhythm, without appreciable murmur, rub, or
gallop. Extremities are warm with 1+ bilateral edema to his calves. There are no palpable cords or
focal tenderness. Electrocardiogram reveals atrial fibrillation with a rapid ventricular response, and
nonspecific T-wave changes. Laboratory values show a normal troponin, normal basic metabolic
panel, and an elevated brain natriuretic peptide.
Which of the following treatments is contraindicated?
a. Amiodarone.
b. Esmolol.
c. Diltiazem.
d. Dofetilide.
e. Digoxin.
58. c. Diltiazem.

This patient is presenting with signs of acute heart failure in the setting of atrial fibrillation with
a rapid ventricular response. Rate control is an important step in management, in addition to
diuresis in light of his signs of volume overload. All of the listed agents may provide rate control
and are indicated for the acute treatment of supraventricular tachycardia. However,
nondihydropyridine calcium channel antagonists are contraindicated in the setting of left
ventricular systolic dysfunction or congestive heart failure, and should be avoided. Cautious use
of beta-blockers is more appropriate, although monitoring for exacerbation of cardiac function is
also warranted when they are used in this setting.

Amiodarone use may be appropriate acutely in this patient, because despite his unknown
duration of atrial fibrillation, he is already anticoagulated for a history of DVT. Therefore, the risk
of stroke from cardioversion is low. Dofetilide is not contraindicated, because he has been
anticoagulated, but will not help with rate control.
59. A 76-year-old man is brought to the emergency room by ambulance with an acute anterior
wall myocardial infarction. He has a history of hypertension and permanent atrial fibrillation for
which he takes metoprolol succinate 100 mg daily and warfarin to maintain an international
normalized ratio (INR) between 2.0 and 3.0. His creatinine clearance is 65 ml/min. A sirolimus-
eluting stent is placed in his proximal left anterior descending coronary artery. He was given aspirin
and clopidogrel at loading doses, and the interventional cardiologist recommends continuing
aspirin at 81 mg daily and clopidogrel 75 mg daily.

Which of the following medical therapy adjustments should also be made?


a. Warfarin should be discontinued.
b. Change warfarin to rivaroxaban 15 mg daily.
c. Change warfarin to dabigatran 75 mg twice daily.
d. Target INR should be 1.5-2.0.
e. Target INR should be 2.0-2.5.
59. e. Target INR should be 2.0-2.5.

Due to his elevated CHA2DS2-VASc score (4, due to age, hypertension, and vascular disease) and
CHADS2 score (2, due to age and hypertension), anticoagulation with warfarin or a novel oral
anticoagulant is indicated. Setting a target INR below the therapeutic window is not advised.
Setting the target INR at 2.0-2.5 in the lower therapeutic range is recommended to decrease the
risk of bleeding while keeping the risk of stroke and systemic emboli low. There is no current
recommendation to use non-vitamin K oral anticoagulants as part of a triple therapy strategy.
60. A 60-year-old woman with a history of nicotine dependence, hypertension, diabetes, and
right common iliac artery stenting presents to the emergency department with a 3-day history of
mild exertional substernal chest pain relieved by rest. Her brother underwent coronary artery
bypass graft surgery at age 55 years. She takes aspirin 81 mg daily, clopidogrel 75 mg daily,
metoprolol 50 mg twice daily, lisinopril 20 mg daily, and metformin 500 mg twice daily.

In the emergency department, her blood pressure is 120/70 mm Hg and her heart rate is 65 bpm.
Electrocardiogram shows normal sinus rhythm with evidence of left ventricular hypertrophy.
Laboratory values reveal of creatinine 1.2 mg/dl, and the initial troponin is elevated. Sublingual
nitroglycerin completely resolves her chest pain. She is admitted to the inpatient ward overnight,
with a plan for coronary angiography in the morning.
Which of the following medications is most appropriate, in addition to aspirin 324 mg?
a. IV nitroglycerin.
b. IV abciximab.
c. No additional therapy.
d. Intravenous (IV) unfractionated heparin.
e. IV fibrinolytics.
60. d. Intravenous (IV) unfractionated heparin.

This patient presents with a non-ST-elevation myocardial infarction (NSTEMI). Her TIMI risk score is
>3. The correct answer is IV unfractionated heparin (Class I, Level of Evidence B).

In patients with NSTEMI, anticoagulation in addition to antiplatelet therapy is recommended for all
patients, if there are no contraindications. Anticoagulant options include subcutaneous enoxaparin
1 mg/kg every 12 hours (reduce dose to 1 mg/kg subcutaneously daily if creatinine clearance is <30
ml/min). IV bivalirudin 0.1 mg/kg loading dose, followed by 0.25 mg/kg/hr can be administered in
patients being managed with an early invasive strategy, that is, within 24 hours (Class IB).
Fondaparinux 2.5 mg subcutaneously daily can be administered either for the duration of the
hospitalization or until percutaneous coronary intervention (PCI) is performed (Class IB). IV
fibrinolytics are contraindicated in NSTEMI (Class III). Glycoprotein IIb/IIIa inhibitors can be
considered, in addition to aspirin and P2Y12 inhibitors in high-risk NSTEMI patients. IV nitroglycerin
is recommended for persistent ischemia, heart failure, or hypertension (Class IB).
61. A 60-year-old man with a history of hypertension and diabetes mellitus was admitted to your
hospital for management of a non–ST-segment elevation myocardial infarction (NSTEMI). His
coronary angiogram revealed three-vessel coronary artery disease. Coronary artery bypass graft
(CABG) surgery was recommended. His medications include aspirin 81 mg daily, clopidogrel 75 mg
daily, enoxaparin subcutaneous 70 mg twice daily (weight is 75 kg and creatinine is normal), and
metoprolol 25 mg orally twice daily. His clopidogrel is to be discontinued 5 days prior to surgery.
Which of the following is the appropriate timing to discontinue his enoxaparin prior to surgery?
a. 36 hours.
b. 9 hours.
c. 6 hours.
d. 1 hour.
e. 12 hours.
61. e. 12 hours.

The correct answer is 12 hours. The half-life for enoxaparin is 4-6 hours. Enoxaparin should be
discontinued 12- 24 hours prior to CABG, and substituted with intravenous unfractionated heparin.
Compared with unfractionated heparin, the use of enoxaparin <12 hours prior to CABG is
associated with lower postoperative hemoglobin and a higher risk of blood transfusion.
62. A 56-year-old man with a history of nicotine dependence, diabetes, and hypertension
presents to the emergency department (ED) with a 10-hour history of severe exertional shoulder
pain. His father underwent three-vessel coronary artery bypass graft (CABG) surgery at age 60
years.

The patient takes aspirin 81 mg daily and metformin 500 mg twice daily. In the ED, his blood
pressure is 100/70 mm Hg, and heart rate 90 bpm. His electrocardiogram (ECG) is shown (Figure
1).

His initial troponin is negative. His pain does not improve with sublingual nitroglycerin. Aspirin 324
mg orally, intravenous (IV) unfractionated heparin, and clopidogrel 600 mg are initiated. Your
facility is a percutaneous coronary intervention (PCI)-capable hospital.
Which of the following is the most appropriate next step?
a. Intra-aortic balloon pump (IABP).
b. IV metoprolol.
c. IV Reteplase.
d. IV tenecteplase.
e. Emergent PCI.
62. e. Emergent PCI.

The correct answer is emergent PCI. His ECG shows an ST-elevation myocardial infarction (STEMI),
which is defined by the ESC/ACCF/AHA/World Heart Federation Task Force as a new ST elevation at
the J-point in at least two contiguous leads of ≥2 mm in men or ≥1.5 mm in women; in leads V2 -
V3 and/or 1 mm in other contiguous chest or limb leads. New left bundle branch block is considered
an STEMI equivalent, and may interfere with ST-elevation analysis.

Reperfusion therapy should be administered to all eligible patients with STEMI with symptom onset
within the prior 12 hours (Class IA). Primary PCI is the recommended method of reperfusion when
it can be performed in a timely fashion by experienced operators (Class IA). Fibrinolytics, such as
tenecteplase and reteplase, are not correct because the question states that the hospital is PCI-
capable. If the hospital is not PCI-capable, and the patient is greater than 120 minutes away from a
PCI-capable center, then fibrinolytic therapy would be indicated. IABP is not indicated, because the
patient is hemodynamically stable and needs urgent PCI. IV metoprolol is not the best answer,
because the patient has ST elevation and needs urgent reperfusion.
63. A 59-year-old farmer with a history of hypertension and hyperlipidemia presents with
crushing substernal chest discomfort that started approximately 1 hour prior to arrival in the local
emergency department. An electrocardiogram demonstrates 2 mm ST-elevations in V1-V3 with
reciprocal ST depressions in the inferior leads. He continues to have 7/10 chest pain. The closest
percutaneous coronary intervention-(PCI) capable facility is 3 hours away. He has no
contraindications to thrombolysis, so intravenous alteplase is administered. His chest discomfort
and ST elevations resolve, and he remains hemodynamically stable.
Which of the following is the best next step in this patient's management?
a. Emergent coronary angiography.
b. Transfer to facility with cardiac surgery capability.
c. Transfer to facility with PCI capability.
d. Echocardiogram.
e. Admit to the intensive care unit (ICU) for close monitoring.
63. c. Transfer to facility with PCI capability.

Patients who present to a non–PCI-capable hospital with ST-elevation myocardial infarction


(STEMI) within 12 hours of symptom onset should be immediately transferred to a PCI-capable
hospital for primary PCI, if the time from first medical contact to intervention will be less than 120
minutes. If the anticipated time exceeds this threshold, then thrombolysis within 30 minutes of
arrival should be considered if there are no contraindications.

For patients receiving fibrinolysis, transfer to a PCI-capable facility is reasonable even when they
are hemodynamically stable and have clinical evidence of successful reperfusion therapy, as in this
case (Class IIa). In this patient population, angiography should be performed when logistically
feasible at the receiving hospital (ideally within the first 24 hours), but not within the first 2-3 hours
following fibrinolysis because of increased bleeding risk. This “pharmacoinvasive” approach has
been demonstrated in clinical trials to decrease the risk of recurrent MI and death compared with a
conservative strategy. Although cardiac surgery backup may be necessary, the patient primarily
needs PCI capability. Admission to the ICU and echocardiography are warranted, but should not
delay transfer to a PCI facility.
64. An 82-year-old active, thin woman with a history of remote tobacco use and hypertension
presents with new-onset chest pain. The symptoms began 2 weeks prior, initially occurring with
exertion climbing a flight of stairs, but gradually have became more progressive. On the day of
presentation, she developed an episode of chest discomfort at rest and her husband brought her to
the emergency department for evaluation.

On arrival, she was pain free and her electrocardiogram demonstrated inferolateral T-wave
inversions. She was admitted to the hospital where serial cardiac biomarkers revealed moderately
elevated troponin levels. Her other blood work was normal, aside from a mild chronic anemia.
Based on these findings, an early invasive strategy was planned with cardiac catheterization the
next day.
Which of the following management strategies would offer the best risk profile in this patient?
a. Aspirin, clopidogrel, and enoxaparin.
b. Aspirin, clopidogrel, and unfractionated heparin.
c. Aspirin, prasugrel, and unfractionated heparin.
d. Aspirin, clopidogrel, unfractionated heparin, and glycoprotein (GP) IIb/IIIa receptor inhibitor.
e. Aspirin, prasugrel, unfractionated heparin, and GP IIb/IIIa receptor inhibitor.
64. b. Aspirin, clopidogrel, and unfractionated heparin.

In patients with non-ST-elevation acute coronary syndrome (NSTE-ACS), bleeding is associated with
worse clinical outcomes. Therefore, the choice of antiplatelet and antithrombotic agents must be
considered carefully. In fact, the ACUITY trial suggested that the mortality risk associated with
significant bleeding may be at least as great as that related to the myocardial infarction.

This patient has multiple bleeding risk factors, including female sex, age, low body weight, and
anemia. Of the available choices, the combintation of aspirin, clopidogrel, and unfractionated
heparin would likely incur the lowest bleeding risk.

All NSTE-ACS patients should receive aspirin. Compared with clopidogrel, prasugrel is associated
with higher rates of bleeding and should also be avoided in patients >75 years old or low body
weight, as well as those with a history of prior cerebrovascular accident. The addition of GP IIb/IIIa
receptor inhibitors upstream of percutaneous intervention would significantly increase the
bleeding risk and has not been shown to have significant clinical benefit. Enoxaparin was associated
with increased major bleeding compared with unfractionated heparin in NSTE-ACS patients treated
with an early invasive strategy in the SYNERGY trial.
65. A 66-year-old male smoker with a history of diabetes mellitus, hyperlipidemia, and
gastroesophageal reflux disease (GERD) presents with central chest discomfort occurring at rest.
The pain began nearly 10 hours before, but he believed it was related to a severe flare of his GERD
and initially avoided seeking medical attention until the following morning. On arrival in the
emergency department, his electrocardiogram (ECG) demonstrated Q waves with ST elevations in
V2-V3. However, he is now free of chest pain. Urgent coronary angiography demonstrated occlusion
of the mid left anterior descending (LAD) without collaterals.

Which of the following strategies is the best next step to reduce the risk of cardiac rupture related
to myocardial infarction?
a. Intra-aortic balloon pump.
b. Glycoprotein (GP) IIb/IIIa receptor inhibitor.
c. Angiotensin-converting enzyme inhibitor (ACEI).
d. Percutaneous coronary intervention.
e. Beta-blocker.
65. d. Percutaneous coronary intervention.

All patients who present with ST-elevation myocardial infarction (STEMI) within 12 hours of
symptom onset should receive urgent reperfusion therapy. The risk factors for free wall rupture are
female sex, older age, presence of Q waves on ECG indicating completed infarct, LAD involvement,
absence of collaterals, no prior history of MI, hypertension during the acute phase of STEMI, use of
steroids or nonsteroidal anti-inflammatory drugs, and fibrinolytic therapy greater than 14 hours
after symptoms onset.

Reperfusion therapy within 12 hours can decrease this risk. An intra-aortic balloon pump and GP
IIb/IIIa receptor inhibitors do not decrease the risk of free wall rupture. Medical management with
ACEIs and beta-blockers are likely warranted in this patient, but would be instituted after
reperfusion therapy when the patient is hemodynamically stable.
66. A 69-year-old woman with hypertension and hyperlipidemia presents to the emergency room
with acute shortness of breath. Two nights ago she had the onset of flu-like symptoms. She has no
other significant past medical history. On exam she is sitting upright. Her blood pressure is 80/50
mm Hg, her heart rate is 110 bpm, and her oxygen saturation is 88% on 6-liter nasal cannula. Her
jugular venous pressure is 15 cm. Lung exam reveals rales to the mid lung fields bilaterally.
Cardiovascular exam reveals tachycardia without murmur or rub appreciated. Extremities are cool.
Electrocardiogram shows a 1 mm ST elevation in leads II, III, and a ventricular fibrillation.
Which of the following is the most likely explanation of her clinical presentation?
a. Contained free wall rupture.
b. Right ventricular (RV) infarct.
c. Ventricular septal rupture.
d. Papillary muscle rupture.
e. Giant cell myocarditis.
66. d. Papillary muscle rupture.

Mechanical complications of acute myocardial infarction (MI) that can result in shock include
papillary muscle rupture, ventricular septal rupture, free wall rupture, and RV infarct. In this patient
presenting with an inferior MI and acute pulmonary edema, the most likely etiology would be
papillary muscle rupture and severe mitral regurgitation. In the acute setting, a murmur may not be
heard. This typically occurs from 2 to 7 days post MI. Ventricular septal rupture may result in a
similar presentation, but usually occurs at 3-5 days and occurs more commonly with anterior MIs.
RV infarct typically presents with hypotension but clear lung fields. Giant cell myocarditis would not
be expected to be associated with inferior ST elevations.
67. A 75-year-old woman presents to the emergency department (ED) 4 days after the onset of chest pain. She
lives in a remote area and did not initially seek medical attention. Today her family visited her and found her
barely responsive in her home. Upon arrival to the ED, she is cool, clammy, and confused.

Her blood pressure is 80/50 mm Hg, heart rate is 110 bpm in sinus rhythm, and oxygen saturation is 90%. She has
bibasilar fine rales, and elevated jugular venous pressure and a thrill felt along the left sternal border. On
auscultation, there is an audible grade 4/6 holosystolic murmur and an S3. She has no edema. Her initial 12-lead
electrocardiogram reveals evidence of an acute anterior myocardial infarction (MI). Her initial cardiac enzymes
are abnormal.

An echocardiogram confirms your suspicion of an acute ventricular septal defect (VSD) with anterior akinesia and
left ventricular ejection fraction of 30%. She is taken quickly to the cardiac catheterization laboratory, where a
total occlusion of the proximal left anterior descending is found, along with a 70% lesion in a large circumflex
obtuse marginal. Urgent surgical consultation is obtained.

Which of the following is the best timing to consider operative repair of the VSD?
a. In more than 28 days.
b. The risk is unrelated to the time period after the infarction.
c. In 15-28 days.
d. In 7-14 days.
e. In less than 7 days.
67. e. In less than 7 days.

The correct answer is less than 7 days. In post MI VSD, overall surgical mortality is 42.9%,
according to the Society of Thoracic Surgeons (STS) database. The mortality rate is highly
associated with the timing of surgery post MI. In the STS database, if repair was performed in
less than 7 days post MI, operative mortality was 54.1%, while it was 18.4% if surgery was more
than 7 days post MI.

Additional independent risk factors for mortality include female sex, presence of shock, older
age, preoperative balloon pump, hemodialysis, moderate to severe mitral regurgitation, and
whether the surgery is a re-do cardiac surgery.

The remaining answers are incorrect because the highest mortality is seen early in the post MI
course. The progressively lower odds of mortality is due to subselection of a healthier cohort
and healing of the myocardium with time, which improves surgical outcomes. Percutaneous
closure of the VSD may be considered in high-risk patients who do not require additional surgical
procedures, such as coronary bypass or valve replacement, within the acute time frame.
However, the issue of post-MI friable myocardium still applies to this intervention.
68. A 55-year-old man is shoveling snow when he is seen grabbing his chest and falls to the
ground. Bystander cardiopulmonary resuscitation (CPR) is performed until emergency medicine
services (EMS) arrives. An automated external defibrillator (AED) is attached and shock is advised
and delivered. CPR is continued and there is no return of spontaneous circulation. Awaiting
advanced cardiac life support (ACLS), the AED advises another shock, which is administered and
followed by return of a palpable carotid pulse. He is intubated on scene, after the administration of
5 mg of midazolam. Prior to intubation, there is no witnessed spontaneous limb movement.

In the hospital emergency department, an electrocardiogram (ECG) is performed that


demonstrates 2 mm inferolateral ST-segment depressions. Norepinephrine is initiated for
hypotension and cardiogneic shock. His vital signs are stable and he is cooled to a temperature of
35 degrees Centigrade. On neurologic examination, the pupils are fixed and nonreactive. He has no
spontaneous movements, nor does he withdraw or grimace to noxious stimulus. He is
overbreathing the ventilator. Aspirin is administered through an orogastric tube. A noncontrast
head computed tomography is unremarkable.
Which of the following is the best next step in this patient's management?
a. Immediate coronary angiography.
b. Coronary angiography only if patient can follow commands.
c. Coronary angiography only if cardiac-specific troponin is elevated.
d. Coronary angiography only if ST elevations develop on ECG.
e. Coronary angiography only if pupils become reactive to light.
68. a. Immediate coronary angiography.

Out-of-hospital cardiac arrest is often caused by acute coronary syndromes. Patients with ST-
elevation myocardial infarction (STEMI) by ECG after return of spontaneous circulation should go
emergently for coronary angiography. Similarly, patients with non-STEMI or other evidence of
acute coronary syndromes with electrical or hemodynamic instability should go emergently for
coronary angiography.

Recommendations for emergent coronary angiography are independent of neurologic status, as


the neurologic exam is unreliable early post arrest. The administration of sedatives or paralytics
further confounds neurologic prognostication. The absence of spontaneous movement post
arrest should not be interpreted as poor neurologic prognosis. Similarly, pupillary reactivity early
post arrest lacks sensitivity or specificity for a poor neurologic outcome. Neurologic
prognostication is improved at 72 hours post return of spontaneous circulation. If the patient
was cooled, neurologic examination and prognostication should be delayed until 72 hours post
rewarming.
69. During acute coronary syndromes, there is thrombus formation at the site of plaque rupture.
Which of the following patient factors contributes to the formation of thrombus on a ruptured
coronary plaque?
a. A thick fibrous cap.
b. Laminar flow in the culprit vessel.
c. Cigarette smoking.
d. Branch point coronary anatomy.
e. Increased high-density lipoprotein cholesterol (HDL-C).
69. c. Cigarette smoking.

Cigarette smoking is the correct answer, because it has been shown to correlate with increased
systemic thrombotic propensity in patients. Increased thrombotic propensity is one of three factors
in Virchow’s triad that contributes to thrombus formation in acute plaque rupture. Laminar flow
would not increase the risk of thrombosis. Branch point areas may have increased plaque
formation, but not necessarily plaque rupture. Thickness of the fibrous cap correlates with risk of
plaque rupture, but not directly with thrombus formation. Increased HDL-C does not increase foam
cell formation as opposed to increased low-density lipoprotein cholesterol.
70. A 66-year-old man presents with substernal chest pain suspicious for myocardial ischemia
that began approximately 45 minutes ago, but has now subsided. He has a past history of heavy
smoking and carries the diagnosis of chronic obstructive pulmonary disease, for which he is being
treated with inhalers that he intermittently uses. He has been noncompliant with medications in
the past and has been homeless at various times in his life. He has hypertension and borderline
hyperlipidemia.

His examination reveals normal blood pressure with a heart rate of 102 bpm. His oxygen saturation
is 89%. He has reduced breath sounds throughout, but a normal cardiac exam. His initial
electrocardiogram (ECG) reveals sinus tachycardia with left ventricular hypertrophy and secondary
repolarization changes.
Which of the following is the most sensitive and specific laboratory test (or tests) for the diagnosis
of myocardial infarction (MI) in this situation?
a. Cardiac-specific troponin plus creatine kinase-myocardial band (CK-MB).
b. Cardiac-specific troponin plus D-dimer.
c. Cardiac-specific troponin plus N-terminal pro-brain natriuretic peptide (NT-proBNP).
d. Cardiac-specific troponin plus CK total.
e. Cardiac specific troponin only.
70. e. Cardiac specific troponin only.

In the appropriate clinical context, an MI is diagnosed when there is an elevation of cardiac-specific


biomarkers >99% above the upper reference limit and at least one of the following: 1) symptoms of
ischemia, 2) new or presumed significant ST-segment–T wave (ST–T) changes or new left bundle
branch block (LBBB), 3) development of pathological Q waves in the ECG, 4) imaging evidence of
new loss of viable myocardium, 5) a new regional wall motion abnormality, or 6) identification of
an intracoronary thrombus by angiography or autopsy.

The definition revolves around an elevation in cardiac-specific troponin (the preferred biomarker),
as it is the most sensitive and specific marker of myocardial necrosis. If troponin is unavailable, CK-
MB would be the next most sensitive biomarker and may be used. Creatine phosphokinase (CPK) is
nonspecific and released by skeletal and cardiac muscle breakdown; therefore it is unreliable as a
biomarker of myocardial necrosis. NT-proBNP is a biomarker that reflects myocardial wall stress
and has been shown to be related to prognosis in acute MI, but is not a specific marker of
myocardial necrosis. D-dimer is nonspecific, although very sensitive for thrombus, and does not
reflect myocardial necrosis.
71. A 28-year-old woman who is 32 weeks pregnant presents to the emergency department with
1 hour of chest pressure and dyspnea. She has no prior history of cardiac disease. She is tachycardic
with a heart rate of 120 bpm and a blood pressure of 150/60 mm Hg. Her oxygen saturation is 95%
on 2 liters of nasal cannula supplemental oxygen. She has a 1/6 early peaking systolic murmur
across the precordium. Her jugular venous pressure is elevated. There are faint rales at the bases of
her lungs bilaterally. Her electrocardiogram (ECG) demonstrates sinus tachycardia at 120 bpm with
3 mm of ST-elevation in leads V1-V4.
Which of the following is the most likely etiology for her presentation?
a. Coronary artery dissection.
b. Pneumothorax.
c. Amniotic fluid embolism.
d. Pulmonary embolism.
e. Eclampsia.
71. a. Coronary artery dissection.

The correct answer is coronary artery dissection. There is a risk for coronary artery dissection
during pregnancy and shortly after delivery in the postpartum period. It is a rare, but well
recognized complication of pregnancy. Her presentation is most consistent with this diagnosis
given the clinical features of chest pain, mild pulmonary edema on examination, and ST-segment
elevation in the left anterior descending territory.

The ECG findings and clinical features are not characteristic for pericarditis. There is a risk of
pulmonary embolism during pregnancy and in the postpartum period due to the hypercoagulability
of pregnancy. However, the features of her presentation are not consistent with this diagnosis.
Likewise, pneumothorax is also on the differential diagnosis for chest pain and dyspnea, but her
physical exam is not consistent with this, because there is no history of trauma and there is no
increased risk of pneumothorax during pregnancy. Amniotic fluid embolus would occur postpartum
and presents with shock. Eclampsia presents with significant hypertension and seizures and would
not cause ST elevation.
72. A 78-year-old man is seen in the office 2 weeks after an anterior ST-segment elevation MI
(STEMI). He underwent primary percutaneous coronary intervention with a drug-eluting stent to
the left anterior descending. His ejection fraction is 40%. His medical history is significant for a
transient ischemic attack (TIA) 5 years ago, chronic kidney disease stage 3b, and peripheral arterial
disease with mild intermittent calf claudication. He was discharged on dual antiplatelet therapy
with ticagrelor 90 mg twice daily and aspirin 81 mg daily.
Which of the following is a contraindication to vorapaxar in this patient?
a. History of cerebrovascular event.
b. Concomitant use of ticagrelor.
c. Chronic kidney disease.
d. Age greater than 75 years.
e. Recent STEMI.
History of cerebrovascular event.

Vorapaxar is an antiplatelet drug that inhibits PAR-1, the thrombin receptor on platelets. The TRA
2°P-TIMI 50 (Thrombin Receptor Antagonist inSecondary Prevention of Atherothrombotic Ischemic
Events Thrombolysis in Myocardial Infarction) trial randomized 26,449 patients who had a history
of MI, ischemic stroke, or peripheral arterial disease to receive vorapaxar (2.5 mg daily) or
matching placebo. Cardiovascular death, MI, or stroke was significantly lower in the vorapaxar
group compared with the placebo group. Moderate or severe bleeding occurred in 4.2% of patients
who received vorapaxar and 2.5% of those who received placebo.

There was an increase in the rate of intracranial hemorrhage in the vorapaxar group. Patients with
a history of stroke had an excessive risk of intracranial hemorrhage. Therefore, the drug is
contraindicated in patients with a history of a TIA or stroke. In the TRA 2°P-TIMI 50 trial, the use
andtype of P2Y12 inhibitor was left to the discretion of the physician. There are no specific data on
the risk of vorapaxar in patients treated with clopidogrel compared with more potent agents. A
small number of patients (0.7%) received prasugrel during the study. Ticagrelor is not
contraindicated, but may alter the efficacy or safety. Because of liver metabolism, vorapaxar can be
used in patients with chronic kidney disease.
73. A 58-year-old man presents to your office to establish cardiovascular (CV) care. He has a
history of myocardial infarction (MI) treated with a drug-eluting stent 2 years ago. At that time, he
had single-vessel disease and a preserved ejection fraction. He was treated with dual antiplatelet
therapy for 12 months and he has been on low-dose aspirin since that time. He has diabetes
requiring oral medication and hypercholesterolemia. He has no history of stroke or bleeding. He is
asymptomatic and has not had recurrent ischemia.
Which of the following do you recommend to decrease the risk of major adverse CV events?
a. Clopidogrel 75 mg daily.
b. Ticagrelor 90 mg twice daily.
c. Prasugrel 10 mg daily.
d. Ticagrelor 60 mg twice daily.
e. Prasugrel 5 mg daily.
73. d. Ticagrelor 60 mg twice daily.

The PEGASUS-TIMI 54 (Prevention of Cardiovascular Events in Patients with Prior Heart Attack
Using Ticagrelor Compared to Placebo on a Background of Aspirin Thrombolysis in Myocardial
Infarction 54) trial randomly assigned in a 1:1:1 double-blind fashion, 21,162 patients who had an
MI in the past 1 to 3 years to ticagrelor 90 mg twice daily, ticagrelor 60 mg twice daily, or placebo.
Ticagrelor reduced the rate of the primary efficacy endpoint (death, MI, or stroke), with Kaplan-
Meier rates at 3 years of 7.85% for ticagrelor 90 mg twice daily, 7.77% for ticagrelor 60 mg twice
daily, and 9.04% for placebo group. For ticagrelor 90 mg versus placebo, the hazard ratio (HR) was
0.85 and the confidence interval (CI) was 0.75 to 0.96 (p = 0.008), and for ticagrelor 60 mg versus
placebo the HR was 0.84 and the CI was 0.74 to 0.95 (p = 0.004). The rates of TIMI major bleeding
were higher with ticagrelor (2.60% with 90 mg and 2.30% with 60 mg) than with placebo (1.06%).
74. A 74-year-old woman is referred to you with indigestion, belching, and mild exertional
“discomfort,” but no chest pain. She is overweight and has hypertension, diabetes (non-insulin
dependent), and sleep apnea. There is a family history of coronary artery disease (CAD) in an older
brother who had successful percutaneous coronary intervention (PCI).

Her current medications include chlorthalidone 25 mg, lisinopril 40 mg, metformin 500 mg twice
daily, and a multivitamin. Her baseline labs are normal, including her lipid profile and HbA1c. Her
electrocardiogram (ECG) is normal. Physical examination reveals a body mass index of 34.5, heart
rate of 78 bpm and regular, blood pressure of 135/85 mm Hg, and oxygen saturation of 94%. She
has an auscultatory S4 and a soft grade 2/6 aortic flow murmur, but no bruits, and a negative
cardiac exam otherwise.

Which of the following do you recommend as the next step in her work-up?
a. Nuclear exercise stress test.
b. No further cardiac workup necessary.
c. Echocardiogram.
d. Gastrointestinal (GI) referral and possible endoscopy.
e. Coronary computed tomographic angiography (CTA).
74. a. Nuclear exercise stress test.

While she does not have classic angina, she has multiple features that make her presentation
suggestive of atypical angina. Atypical angina may present as indigestion, belching, and exertional
dyspnea, and it is more common in women, older patients, and those with diabetes.

Her baseline ECG is normal and the first test to be considered should be a stress ECG with imaging.
In women, the use of imaging improves diagnostic accuracy when compared with standardized ECG
stress testing. An echocardiogram to assess left ventricular function might be useful, but her exam
and ECG is normal. GI endoscopy might be a consideration as well, but the exertional component to
her chest “discomfort” warrants further evaluation. Coronary CTA and a calcium score might also
be helpful, because a negative study at her age would suggest there is little chance that CAD is
present. Because angina can be difficult to diagnose in this population, some effort to exclude a
coronary source should be undertaken.
75. A 57-year-old man is referred to you with shortness of breath. He has not been followed
regularly and he says he was told he had a murmur as a teenager, but that he would likely
“outgrow it.” He works in heating and air conditioning and has a fairly physical work environment.
He does not smoke and is on no medication.
Which of the following could be considered an etiology for cardiac-related dyspnea?
a. A left-to-right intracardiac shunt Qp/Qs of 2.1:1.
b. Pulmonary capillary wedge pressure (PCWP) of 11 mm Hg.
c. A distal left circumflex coronary lesion of 60%.
d. Cardiac index of 3.0 L/m2.
e. Mean pulmonary artery pressure of 28 mm Hg during exercise.
75. a. A left-to-right intracardiac shunt Qp/Qs of 2.1:1.

The correct answer is a left-to-right cardiac shunt with its associated volume overload.

Cardiac-related dyspnea can occur when there is an elevated PCWP with resultant stimulation of
the interstitial J receptors and/or pulmonary edema. Usually there is associated orthopnea or
paroxysmal nocturnal dyspnea (PND), which occurs 2-4 hours after the onset of sleep and subsides
minutes after sitting upright or standing. The PCWP of 11 mm Hg is normal in this situation.

Patients with low cardiac output (cardiac index of 3.0 L/m2) experience dyspnea on exertion and
fatigue in the absence of orthopnea or PND. Dyspnea may also be due to ischemia during exercise,
but it is very unlikely to be associated with the degree of coronary disease that jeopardizes only a
small amount of myocardium as noted.
76. The jugular venous pressure (JVP) reflects the right atrial (RA) pressure. It is a critical
component of the cardiac examination, as an elevation suggests an elevated RA pressure and has
prognostic importance.
Which of the following is correct regarding the assessment of an elevated JVP?
a. It is best measured as the vertical distance above the clavicle in the upright position.
b. It is best measured as the vertical distance between the top of the pulsation and the angle of
c. Louis.
d. It can best be assessed with the patient lying at a 20 degree elevation.
e. The degree of elevation cannot be reliably assessed during atrial fibrillation.
f. Either the external or internal jugular veins can be assessed reliably.
76. a. It is best measured as the vertical distance above the clavicle in the upright position.

The height of the JVP is reflective of RA pressure. The best answer is to measure the height of the
JVP in the upright position above the clavicle. Using that as a reference point is effective, because
the RA pressure must be at least 10 cm high for the waveforms to be visible in that position. Thus,
any evidence of waveforms in the upright position means the JVP is elevated.

While either the external or the internal jugular can be used, the jugular vein valves may interfere
with accuracy and the internal jugular vein is preferred. Using the angle of Louis (the sternal
inflection point where the manubrium meets the sternum), a distance >3 cm has been considered
abnormal. However, the distance from the sternal angle and the RA is quite variable among
patients, and this leads to an underestimation of the venous pressure. Atrial fibrillation will affect
the waveforms observed, but the total height of the JVP column will still reflect mean RA pressures.
The height of the JVP cannot be reliably assessed with the patient in a semisupine position.
76. A number of early diastolic heart sounds may be audible in certain patients. These include an
S3 in young patients and in older patients suffering from heart failure, a pericardial knock in those
with constrictive pericarditis, and a mitral opening snap in patients with rheumatic mitral stenosis.
The timing from S2 of each of the early diastolic heart sounds varies, depending on the origin of the
sound.
Which of the following is the correct sequence one could expect these heart sounds to occur if they
were present, from a timing perspective in early diastole?
a. Opening snap, followed by pericardial knock, followed by an S3.
b. Opening snap, followed by an S3, followed by pericardial knock.
c. Pericardial knock, followed by an S3, followed by opening snap.
d. Pericardial knock, followed by opening snap, followed by an S3.
e. S3, followed by an opening snap, followed by a pericardial knock.
77. a. Opening snap, followed by pericardial knock, followed by an S3.
These early diastolic sounds reflect the hemodynamic changes in early diastole. The opening snap is
a high-pitched sound occurring shortly after S2 and reflects the abrupt opening of a mobile but
stenotic mitral valve. The interval between the S2 and opening snap decreases as the left atrial (LA)
pressure rises (worsening mitral stenosis). It is the first sound heard in early diastole, as it reflects
the mitral valve opening prior to the initiation of left ventricular (LV) filling. The second possible
sound is the pericardial knock. It occurs once the mitral valve is open and is a high-pitched sound
reflecting the impact of the cessation of rapid filling on the LV wall. In constriction, the LV fills
rapidly from the elevated LA pressure, to the point it cannot expand further due to the restriction
placed on it from the constrictive pericarditis.

An S3 is a low-pitched sound that reflects the rapid filling phase of diastolic LV filling and occurs
later than the impact sound from a pericardial knock. An S3 may be normal in children, adolescents,
and young adults, but implies heart failure in adults. It can also be heard in severe mitral
regurgitation. Thus, the correct answer is opening snap, followed by pericardial knock, followed by
an S3.
78. The Valsalva maneuver can help distinguish the origin of some cardiac murmurs and is known
to be abnormal in heart failure. For patients with an intact autonomic nervous system, when the
Valsalva maneuver is performed there are four phases evident from the hemodynamic tracings.

Which of the following occurs during phase 3 of the Valsalva maneuver?


a. The systemic pressure declines and the heart rate (HR) increases.
b. The systemic pressure increases due to increased blood pressure return.
c. The systemic pressure increases with slowing of the HR.
d. The systemic pressure declines in response to a decrease in intrathoracic pressure.
e. The systemic pressure increases transiently due to an increase in intrathoracic pressure.
78. d. The systemic pressure declines in response to a decrease in intrathoracic pressure.

The four phases of the Valsalva maneuver are shown in Figure 1, where the relative HR and aortic
pressure are outlined. In phase 1, with the initiation of the Valsalva, the intrathoracic pressure
increases and blood is forced out of the pulmonary circuit. There is an increase in left ventricular
(LV) filling and stroke volume and the systemic blood pressure rises. In phase 2, with the
maintenance of the Valsalva, the LV filling declines as the increased intrathoracic pressure impedes
venous return and stroke volume drops, resulting in vasoconstriction with a compensatory
tachycardia.

In phase 3, with release of the Valsalva, the aortic pressure declines further in response to the
decreased intrathoracic pressure and decreased left atrial return associated with the increased
aortic volume and the delay before venous return catches up to the LV. In phase 4, the LV preload
increases with the return of the venous flow; stroke volume increases rapidly into the
vasoconstricted aortic system and the blood pressure overshoots and the baroreceptors are
stimulated with a subsequent reduction in the HR. Some have noted that when the ratio of the the
fastest HR (during Valsalva) divided by the slowest HR (upon release) is <1.4, there is evidence for
autonomic impairment.
(Figure 1)
79. The use of the Valsalva maneuver can help differentiate the origin of certain cardiac
murmurs. The four phases of the Valsalva maneuver also may be abnormal in patients with
autonomic dysfunction and in heart failure.
Which of the following summarizes the abnormality in the phases of the Valsalva maneuver that
occurs in heart failure?
a. There is an increase in BP with little change in heart rate (HR) during phase 2.
b. There is a decrease in blood pressure with no change in HR during phase 3.
c. There is an increase in BP and bradycardia during phase 4.
d. There is an increase in blood pressure (BP) and bradycardia during phase 2.
e. There is a decrease in BP with little change in HR during phase 2.
79. d. There is an increase in blood pressure (BP) and bradycardia during phase 2.

There is an increase in BP with little change in HR during phase 2 of the Valsalva maneuver in
patients with significant heart failure. This is shown graphically in Figure 1. As opposed to the
normal phase 4 of the Valsalva, in patients with severe heart failure there is a “square wave”
response with an initial increase in BP (phase 1) that remains elevated during the entire strain
period (phase 2) and returns to resting levels in phase 3. This is thought to be due to maintenance
of the left ventricular (LV) filling during the strain phase, despite the decrease in venous return. This
is presumably due to the increased central blood volume that serves as a reservoir to maintain LV
preload. This can be detected with BP monitoring during Valsalva or by an abnormal response as
assessed by the Korotkoff sounds as shown in Figure 2. In the latter figure, the loss of the
overshoot in BP has been correlated with loss of systolic function and the square root response has
been correlated with an elevated pulmonary capillary wedge.
(Figure 1)
(Figure 2)
80. A 65-year-old woman with hypertension is admitted to a rural hospital with chest pain for 1
hour and an ST-segment elevation myocardial infarction (STEMI). She immediately undergoes
therapy with tenecteplase.

On arrival to the tertiary referral center, she is comfortable without chest pain, her
electrocardiogram is now normal, and she is hemodynamically stable. Her exam is only remarkable
for S4.
Which of the following is the next most appropriate step?
a. Emergent coronary angiography.
b. Coronary computed tomography angiography.
c. Elective coronary angiography.
d. Cardiac magnetic resonance imaging.
80. c. Elective coronary angiography.

Assessment of clinical reperfusion following lytic therapy for STEMI is essential to guiding the
urgency of subsequent coronary angiography. Resolution of symptoms, improvement of ST-
segment elevation by at least 50%, and reperfusion arrhythmias (e.g., accelerated idioventricular
rhythm) are indicative of epicardial coronary patency. If there is clinical uncertainty about the
status of reperfusion, emergent coronary angiography is warranted (e.g., rescue percutaneous
coronary intervention). Most patients following STEMI treated with lytic therapy will undergo
elective coronary angiography to define the coronary anatomy within a day of presentation (Class
IIa). In high-risk scenarios like shock, heart failure, or recurrent chest pain, urgent coronary
angiography is recommended (Class I).
81. A 49-year-old man with no significant medical history presents with 1 week of intermittent
chest pain that worsened over the 2 hours prior to presentation.

Blood pressure measures 84/43 mm Hg with a pulse of 113 bpm. Oxygen saturation on 2 liters of
nasal cannula is 91%. He is tachycardic with an S3 and rales on lung exam. Initial electrocardiogram
demonstrates ST-segment elevation (STE) in leads V1-V5.

The patient undergoes urgent cardiac catheterization with placement of a drug-eluting stent in the
proximal left anterior descending artery. His blood pressure improves post revascularization and he
is admitted to the coronary care unit. A post-percutaneous coronary intervention echocardiogram
shows severe hypokinesis of the anterior and anterolateral wall. Left ventricular ejection fraction is
25%.

On day 2 of hospitalization, the patient develops dyspnea and the rhythm seen in Figure 1.

Which of the following best describes the incidence and degree of myocardial injury associated
with the rhythm seen in Figure 1 in an anterior as compared with inferior myocardial infarction
(MI)?
a. The incidence during MI is lower, the degree of myocardial injury is lower, and the location of
block is at the AV node.
b. The incidence during MI is greater, the degree of myocardial injury is lower, and the location
of block is at the atrioventricular (AV) node.
c. The incidence during MI is the same, the degree of myocardial injury is greater, and the
location of block is below the AV node.
d. The incidence during MI is lower, the degree of myocardial injury is greater, and the location
of block is below the AV node.
81. d. The incidence during MI is lower, the degree of myocardial injury is greater, and the
location of block is below the AV node.

The rhythm strip reveals the development of complete heart block (CHB) occurring in the setting of
recent anterior wall MI. This is a serious complication of anterior MI, which occurs in roughly 1% of
cases. CHB in anterior MI is associated with poorer short-term outcomes, including survival, as
compared with anterior MI patients who do not develop CHB.

The incidence of CHB in inferior distribution ST-elevation (STEMI) is higher, approximately 4%, and
may be transient. CHB occurring in inferior STEMI patients is associated with a higher location of AV
block (typically the AV node), whereas in anterior wall MI, the location of block is usually below the
AV node, resulting in higher long-term block.

The degree of myocardial injury in anterior wall STEMI exhibiting CHB is greater than the amount of
myocardial injury observed in inferior STEMI complicated by CHB.
82. A 75-year-old woman with diabetes is admitted to the coronary care unit after a drug-eluting
stent (DES) placement in her mid right coronary artery following an acute inferior wall myocardial
infarction (MI). Earlier, she had received 600 mg of clopidogrel in the emergency department and
had been administered a bolus of bivalirudin followed by an intravenous infusion. She has a history
of a prior ischemic stroke and has been a lifelong asthmatic.

Her electrocardiogram shows sinus rhythm at 50 bpm, and inferior Q waves with almost complete
resolution of her inferior ST elevation. Her body mass index is 20 kg/m2, blood pressure is 148/70
mm Hg, and lungs are clear to auscultation. Cardiac examination reveals a fourth heart sound and a
loud aortic component of the second heart sound.
At discharge, which of the following medications is the most appropriate choice?
a. Ticagrelor.
b. Ticlopidine.
c. Prasugrel.
d. Cangrelor.
e. Clopidogrel.
82. d. Cangrelor.

The choice of antiplatelet agents following DES placement for acute MI should be guided by specific
patient characteristics outlined by clinical trials. Prasugrel is contraindicated in this patient due to
her size, age, and previous stroke. Ticagrelor is not preferred due to the history of asthma and
bradycardia on exam. Ticlopidine is no longer a first-line choice due to its numerous adverse
effects, including rash, diarrhea, and neutropenia. Cangrelor is only available intravenously.
83. Mr. K is a 76-year-old man who presents to the emergency department. He complains of
epigastric pressure, shortness of breath, nausea, and fatigue, all of which suddenly started about 2
hours earlier. He has been in good health in the past and his history is remarkable only for diabetes
treated with oral hypoglycemic medication. He is clinically stable with a blood pressure of 112/78
mm Hg, heart rate of 75 bpm, S02 of 96%, and temperature of 36.4°C. His electrocardiogram
(ECG) is normal. An initial set of cardiac biomarkers turns out to be negative. After a tablet of
sublingual nitroglycerin, he says that his symptoms are slightly improved.
Which of the following is the most appropriate next step for management?
a. Transesophageal echocardiography.
b. Outpatient stress test.
c. ECG with posterior leads.
d. Ventilation/perfusion scan.
e. Coronary computed tomography angiography (CTA).
83. c. ECG with posterior leads.

This patient presents with strong pretest probability for an acute ischemic syndrome, although
the ECG is normal. Posterolateral infarctions are often electrocardiographically silent and should be
considered in these situations. Presence of ST-segment elevation in V7-V9 will confirm the diagnosis
of true posterior ST-segment elevation myocardial infarction and the need to perform urgent
reperfusion.

Outpatient stress testing would not be appropriate in a patient with high pretest probability of an
acute ischemic syndrome. Coronary CTA performs best in intermediate-risk patients. The pretest
probability for pulmonary embolism and aortic disease is sufficiently low such that they would not
be appropriate when the most likely diagnosis is still an acute coronary syndrome.
84. Which of the following phrases best describes tenecteplase?
a. Nonantigenic, administered as a single bolus, fibrin specific.
b. Nonantigenic, administered as two boluses 10 minutes apart, fibrin specific.
c. Antigenic, administered as a single bolus, fibrin specific.
d. Antigenic, administered as two boluses 10 minutes apart, not fibrin specific.
e. Nonantigenic, administered as single bolus, not fibrin specific.
84. a. Nonantigenic, administered as a single bolus, fibrin specific.

Tenecteplase, or TNK, is a fibrin-specific, nonantigenic fibrinolytic. Although its administration is


weight based, its main advantage is that it can be administered as a single bolus. Reteplase, or
Retavase, has limited fibrin specificity. It is given as two boluses 10 minutes apart, but does not
have to be adjusted for weight. Tissue plasminogen activator has moderate fibrin specificity, but is
administered as a drip. Streptokinase has no fibrin specificity and is highly antigenic.
85. A 63-year-old man presents with chief complaints of chest heaviness. He tells you about an
episode in the morning that subsided spontaneously after 10 minutes, and a new episode 2 hours
before presentation that lasted about 30 minutes. Now he is asymptomatic, but his wife insisted on
bringing him to the emergency department. He has been diabetic for the last 15 years and treated
with metformin and glimepiride. He has also been prescribed amlodipine to control his blood
pressure.

His blood pressure is now 145/90 mm Hg, his heart rate is 62 bpm, and creatinine is 1.12 mg/dl.
The physical exam is unremarkable. He smokes about 10 cigarettes per day. His electrocardiogram
is normal and on the first set, his troponin T is 0.06 ng/ml (99th percentile, 0.01 ng/ml). You
calculate a GRACE score of 106 and TIMI score of 3.
Which of the following is the optimal management for this patient?
a. Emergent coronary angiography.
b. Coronary angiography within 24 hours.
c. No further diagnostic testing.
d. Coronary computed tomography angiography.
e. Stress test.
85. b. Coronary angiography within 24 hours.

The risk profile of this patient does not warrant an emergent catheterization. The 2011 focused
update of the Guidelines for the Management of Patients With Unstable Angina/Non-ST-Elevation
Myocardial Infarction (NSTEMI) considers an early invasive management strategy within 24 hours
reasonable in the case of high-risk criteria or clinical instability. This patient is diabetic and has
positive troponin, both of which favor an invasive approach within 24 hours.

This patient has a confirmed NSTEMI, and a stress test to provoke ischemia in this setting is not
diagnostically useful and is unsafe. Coronary computed tomography angiography is predominantly
useful when there is diagnostic uncertainty and the pretest probability is either low or
intermediate.
86. A 50-year-old man with diabetes mellitus and a history of cigarette smoking underwent
percutaneous coronary intervention of his mid left anterior descending artery 2 months ago. At the
time of his procedure, he was noted to have a total cholesterol of 350 mg/dl, low-density
lipoprotein cholesterol (LDL-C) of 180 mg/dl, high-density lipoprotein cholesterol (HDL-C) of 20
mg/dl, and triglycerides of 280 mg/dl. He was placed on atorvastatin 40 mg daily, and repeat
laboratory tests obtained today show a total cholesterol of 200 mg/dl, LDL-C of 120 mg/dl, HDL-C
of 23 mg/dl, and triglycerides of 250 mg/dl. The patient is compliant with his medical regimen.

Which of the following is the most appropriate next step for this patient?
a. Add niacin.
b. Increase atorvastatin dose to 80 mg daily.
c. Add omega-3 fatty acids.
d. Add ezetimibe.
e. Add gemfibrozil.
86. b. Increase atorvastatin dose to 80 mg daily.

For individuals with established coronary artery disease and diabetes, an LDL-C reduction of at least
50% is the recommended therapeutic strategy. Although administration of a statin reduced the
patient's LDL-C, since it is not at goal, the statin dose should be increased.

Gemfibrozil is used primarily to treat hypertriglyceridemia, and an increased incidence of myositis


is observed when it is used concomitantly with a statin. The addition of niacin to a statin is
attractive because it may further increase HDL and reduce triglycerides, but it has not been shown
to reduce cardiovascular events. Ezetimibe and omega-3 fatty acids are not indicated without
maximizing statin therapy.
87. Which of the following findings is most useful for establishing or excluding the diagnosis of ST-
segment elevation myocardial infarction (STEMI)?
a. Normal cardiac troponin 1 hour after symptom onset.
b. ECG with 100% ventricular pacing and 3 mm of ST-segment elevation in precordial leads.
c. ECG with 1 mm of ST-segment elevation in leads V2-V3 in an asymptomatic 44-year-old man.
d. Electrocardiogram (ECG) with left bundle branch block (LBBB) and 2 mm of concordant ST-
segment elevation in two leads with a positive QRS deflection.
87. d. Electrocardiogram (ECG) with left bundle branch block (LBBB) and 2 mm of concordant
ST-segment elevation in two leads with a positive QRS deflection.

Neither cardiac troponin nor creatine kinase-myocardial band (CK-MB) are sufficiently sensitive
within the first 1-2 hours after symptom onset to exclude a diagnosis of MI.

The multisociety Task Force for the Universal Definition of MI has defined the diagnostic criteria for
STEMI (at the J point) as follows: ≥2 mm in men or ≥1.5 mm in women in leads V2-V3 and/or ≥1 mm
in at least two other contiguous chest leads or two limb leads. Moreover, particularly in young
men, a multisociety scientific statement recommends a higher threshold of ≥2.5 mm in leads V2-V3.

The Sgarbossa criteria for diagnosis of MI emphasize concordant ST-segment elevation in two leads
with a positive QRS as most indicative of STEMI in the setting of an LBBB. It is challenging to
diagnose STEMI in the setting of ventricular pacing.
88. A 60-year-old businessman has a history of obesity, with a body mass index of 32 kg/m2. His
history includes prior cigarette smoking, hypertension, and hyperlipidemia. Six months ago, he
decided to start climbing three flights of stairs to his office each morning. When performing this
activity, he notes a pressure sensation localized to his substernal region that resolves within 2-3
minutes once he sits down at his desk. He describes this symptom to his physician, who orders an
exercise treadmill test. The results are as follows: exercise duration 8 minutes of the Bruce
protocol; the patient requests to stop because of his chest pressure; exercise electrocardiogram
(ECG) is negative for ischemia.
Based on the ECG stress results, his post-test probability of coronary disease is which of the
following?
a. >90%.
b. 60-90%.
c. 0-30%.
d. 30-60%.
88. a. >90%.

A 60-year-old man with typical angina has a pretest probability of coronary artery disease (CAD)
that is >90%. According to Bayes Theorem, a negative exercise ECG shifts this probability very little,
and the post-test probability of CAD remains >90%. This patient’s Duke treadmill score can be
calculated as 0: 8 minutes Bruce protocol – 0 (for no ST-segment depression) – 8 (4 x 2, for test-
limiting angina). A Duke treadmill score of zero places a patient into the intermediate-risk category
for cardiac death, where annual cardiac mortality ranges between 1-3%.
89. Ionizing radiation can result in cellular DNA injury. A stochastic injury from medical radiation
exposure results in which of the following consequences?
a. Carcinogenesis.
b. Nausea and vomiting.
c. Skin injury.
d. Hair loss.
e. Cellular death.
89. a. Carcinogenesis.

Stochastic injuries can occur after a single exposure to the cell. It is therefore not threshold
dependent. The DNA injury that results does not kill the cell, but rather the cell lives on mutated.
This can eventually lead to cancers later in life. A deterministic injury results when the cell dies
from the DNA damage. If enough cells of an organ die, then the organ becomes dysfunctional. This
is seen most often in skin damage, but can result in hair loss, or if extensive (such as acute radiation
injury), can result in sloughing of the gastrointestinal tract. Deterministic injury only occurs after a
threshold level of radiation exposure has occurred.
90. A 65-year-old active woman has a history of type 2 diabetes and uncomplicated inferior wall
myocardial infarction treated with lytic therapy therapy 10 years ago. She has experienced a dozen
episodes of interscapular back and chest discomfort over the past month. This discomfort usually
occurs while gardening. She has tried nitroglycerin twice and thinks that it has helped. Her
electrocardiogram (ECG) shows old Q waves in the inferior leads and is otherwise normal.
Which of the following is the procedure of choice for this woman?
a. Standard treadmill test.
b. Coronary angiogram.
c. Regadenoson single-photon emission computed tomography (SPECT).
d. Dobutamine ECG.
90. a. Standard treadmill test.

This patient is describing atypical angina and is therefore a candidate for noninvasive assessment,
rather than proceeding directly to coronary angiography. The multisociety stable ischemic heart
disease guideline recommends consideration of three criteria for deciding between the standard
treadmill test and stress imaging: a history of prior coronary revascularization, ability to adequately
exercise, and findings on a resting ECG.

ECG abnormalities that warrant proceeding with stress imaging instead of the standard treadmill
test include pre-excitation, left bundle branch block, paced ventricular rhythm, or >1 mm of ST-
segment depression. The presence of Q waves is not an indication to proceed with stress imaging
instead of the standard exercise treadmill test. Although this patient has risk factors that place her
at a higher risk (i.e., history of established coronary artery disease and the presence of diabetes),
the standard treadmill test would still be the procedure of choice in this setting, according to the
guideline.
91. Which of the following procedures results in the highest potential effective radiation dose
being delivered to the patient?
a. Technetium-99m sestamibi rest-stress scan.
b. Thallium-201 rest-stress scan.
c. Diagnostic coronary angiogram.
d. CT coronary angiogram.
e. Coronary calcium computed tomography (CT).
91. b. Thallium-201 rest-stress scan.

Diagnostic coronary angiography, technectium-99m scans, and CT angiography (if the newest
techniques are applied) result in similar effective dose exposure to patients. Increased dose occurs
with percutaneous coronary intervention because of the time, angulated views, the use of
magnified views, etc. Thallium-201 has a long half-life and slow excretion, resulting in the greatest
exposure compared to the other agents.
92. A 65-year-old man presents for cardiac computed tomography (CT) due to a myocardial
perfusion study in which the imaging and electrocardiogram findings are discordant.
Which of the following presenting conditions should lead to cancellation of the cardiac CT order?
a. Use of sildenafil 24 hours ago.
b. Body weight 42 kg/m2.
c. Sinus bradycardia at 40 bpm.
d. Atrial fibrillation, rate 80-110 bpm.
92. d. Atrial fibrillation, rate 80-110 bpm.

Atrial fibrillation due to the extreme R-R variability causes motion artifacts on cardiac CT that lead
to an unacceptable rate of nondiagnostic examinations. Although some scan modes can enable
moderately accurate cardiac CT angiography in the setting of atrial fibrillation (e.g., helical CT
without tube current modulation or single-beat wide-area detector CT), atrial fibrillation represents
a strong contraindication to the test. Bradycardia is not a contraindication. Increased body mass
index can lead to noisy cardiac CT images and will necessitate higher radiation exposure. Sildenafil
use contraindicates use of nitroglycerin, but scanning can proceed without its use.
93. You are called to see a patient in the emergency room 2 days after percutaneous coronary
intervention, who has low-grade fever, nausea, diffuse pruritus, and a maculopapular rash on the
trunk and upper extremities. The patient was initiated on clopidogrel after the procedure.
Which of the following is your next step in management of this patient?
a. Intravenous (IV) immunoglobulin (IG).
b. IV steroids.
c. Diphenhydramine therapy.
d. Stop the clopidogrel.
93. c. Diphenhydramine therapy.

Late contrast reactions are T-cell mediated and are more common with nonionic dimers. If a
patient has a prior history of such a reaction, choosing a contrast other than a nonionic dimer, such
as a nonionic monomer (iopamidol), is advised. This is a self-limiting condition and symptomatic
treatment is all that is required. This is not a reaction to the clopidogrel and thus should not be
discontinued.
94. In a patient who receives enoxaparin with the electrocardiogram shown in Figure 1, which of
the following conditions would indicate antifactor Xa (anti-Xa) level monitoring?

a. Liver disease.
b. Concomitant warfarin
therapy.
c. Renal insufficiency.
d. Diabetes.
e. Stroke.
94. c. Renal insufficiency.

The patient’s degree of renal dysfunction warrants anti-Xa monitoring. According to the
current American Heart Association/American College of Cardiology guideline, low molecular
weight heparin agents should be avoided in patients with severe renal dysfunction (creatinine >2.5
in men, >2.0 in women). In patients with moderate renal dysfunction, dose adjustment should be
considered and anti-Xa monitoring may be necessary.

Enoxaparin is not hepatically metabolized; liver function testing abnormalities do not necessitate
dose adjustment or anti-Xa monitoring. Dose adjustment and anti-Xa monitoring are indicated in
patients with a weight exceeding 100 kg, but at weights below that standard dosing is appropriate
and anti-Xa monitoring is not necessary, independent of body mass index.

While the use of chronic warfarin causes concern for the possibility of increased bleeding risk in the
setting of enoxaparin, it does not affect dosing or the role of anti-Xa monitoring. Management of
enoxaparin in the cardiac catheterization laboratory is based on the time from the last dose
administered, not anti-Xa level.
95. A 79-year-old man presents to his local community hospital with substernal chest pain lasting
3 hours. His resting electrocardiogram (ECG) shows 3 mm ST elevation across the anterior
precordium with presence of QS complexes in V1-V4. His resting heart rate is 96 bpm, and his blood
pressure in the right upper limb in the supine position is 100/70 mm Hg.

Physical examination reveals an apical third heart sound, a soft systolic murmur in the aortic area
that increases with expiration, and crepitations in bilateral lung bases. His respiratory rate is 20
breaths/minute, and a pulse oximetry on room air is 92%. The emergency department physician
activates the catheterization laboratory planning for primary percutaneous coronary intervention
(PCI) and places a nasal cannula delivering oxygen at 4 L/min.
Which of the following medical interventions is the most appropriate?
a. Reteplase.
b. Diltiazem.
c. Enalaprilat.
d. Isosorbide.
e. Furosemide.
95. e. Furosemide.

The use of intravenous (IV) enalaprilat in the CONSENSUS 2 trials was not associated with a
mortality benefit. This lack of benefit was attributed to hypotension in the early postreperfusion
setting. The borderline blood pressure, high resting heart rate, and evidence of pulmonary
congestion in this elderly patient with an extensive anterior myocardial infarction (MI) on ECG are
likely due to a significant acute decrease in stroke volume.

Treatment with IV beta-blockade as noted in the COMMIT 2 trial increases the risk of precipitating
cardiogenic shock in this setting and should be avoided. The dosage of unfractionated IV heparin in
the setting of ST elevation MI should be weight-adjusted with a bolus dose of 60 U/kg not to
exceed 4000 U, and an infusion rate of 12 U/kg not to exceed 1000 U/h.
96. A 78-year-old woman presents with resting chest pain and anterior T-wave inversions on
electrocardiogram (ECG). Her cardiac enzymes are normal. She has no cardiac history, no history of
diabetes, and no known allergies. She is treated with aspirin and subcutaneous enoxaparin and
remains chest pain free. However, 6 hours after admission, she develops massive hematemesis.
Which of the following is the next best step in her management?
a. Fresh frozen plasma (FFP).
b. Cryoprecipitate.
c. Protamine.
d. Lepirudin.
96. c. Protamine.

Although protamine incompletely reverses the effect of enoxaparin on factor Xa, it does reverse its
effect on thrombin (factor IIa), and as such, is indicated for patients with bleeding complications
due to enoxaparin. Adverse reactions to protamine can be seen among patients who are allergic to
fish or who have received pork insulin. Cryoprecipitate and FFP do not reverse the effects of
indirect thrombin inhibitors such as heparins. A direct thrombin inhibitor, such as lepirudin, is
contraindicated in the presence of active bleeding.
97. A 52-year-old man develops intermittent chest pain while on a business trip and presents to
the emergency department for evaluation. He does not have diabetes and is a nonsmoker. His
outpatient medical regimen includes atorvastatin 40 mg, and lisinopril 20 mg daily. His exam is
normal. His electrocardiogram (ECG) shows nonspecific flattening of the T waves in the lateral
leads, and serial cardiac enzymes are normal. He is treated with aspirin and unfractionated heparin
for 24 hours and his chest pain has not recurred. He is anxious to return home and asks if he can
catch his flight later that afternoon.

Which of the following is most appropriate for the care of this patient?
a. Exercise treadmill test.
b. Coronary angiography.
c. No further testing prior to travel.
d. Heparin for additional 24 hours.
97. a. Exercise treadmill test.

The patient presents with a symptom complex consistent with possible unstable angina. However,
there are no objective signs of ischemia, the clinical presentation is low risk given his young age, his
cardiac enzymes are normal, and there is an absence of diagnostic ECG changes. In low-risk
patients, an initial conservative (ischemia-guided) approach is reasonable in which the patient is
treated medically and undergoes risk stratification prior to discharge. Although 48-72 hours of
medical therapy prior to stress testing is recommended when definite acute coronary syndrome
(ACS) is present, stress testing can be performed safely earlier in low-risk patients with possible
ACS, and exercise stress testing provides a prudent solution that meets the wishes of the patient
and avoids excessive testing. Exercise stress testing prior to discharge would be recommended
because the patient has planned air travel.
98. A 60-year-old woman with atypical chest pain undergoes a treadmill stress test. The stress
test findings are:
 Heart rate response: 64-127 bpm
 Blood pressure (BP) response: 132/66 to 174/80 mm Hg
 Symptoms: Mild fatigue at peak
 Electrocardiogram (ECG): 2 mm of horizontal ST-segment depression in leads V5-V6 at peak
exercise
 Exercise duration: 3.0 minutes on Bruce protocol
 Perfusion images: Reversible apical ischemia

Otherwise, it was a normal study.

Which of the following findings best correlate with a highest risk for cardiac death?
a. Perfusion ischemia.
b. Poor functional capacity.
c. Hypertensive BP response.
d. Heart rate response.
e. ECG ischemia with 2 mm ST-segment depression.
98. b. Poor functional capacity.

Poor functional capacity is the best predictor of cardiac death. Perfusion images or ECG ischemia
predict ischemic events, but are less predictive of death than exercise duration. Neither her heart
rate response nor BP response is particularly concerning.
99. A 65-year-old woman with a history of coronary artery disease is admitted with a 3-day
history of chest pain, nausea, and vomiting. Her electrocardiogram shows ST elevation in inferior
leads. She is taken to the cardiac catheterization laboratory and undergoes percutaneous coronary
intervention of the right coronary artery. In the cardiac care unit, she is somnolent, and her blood
pressure is 88/60 mm Hg, heart rate is 92 bpm, and oxygen saturation is 96% on room air. Jugular
venous pressure is 10 cm H2O. Lung fields are clear, and cardiovascular examination has normal
S1and S2 without murmurs appreciated. Extremities are cool.
Which of the following is the most likely diagnosis?
a. Papillary muscle rupture.
b. Ventricular septal defect (VSD).
c. Free wall rupture.
d. Right ventricular (RV) infarct.
e. Pulmonary embolism.
99. d. Right ventricular (RV) infarct.

The patient has signs of RV involvement in her inferior myocardial infarction (MI) with hypotension
and evidence of poor perfusion. With hypotension and inferior MI, clinical suspicion of RV
involvement should be high. While other etiologies of her hypotension could include over-sedation,
bleeding, and cardiogenic shock, management is quite different for an RV infarct and needs to be
identified early.

Papillary muscle rupture and VSD are later mechanical complications of MIs. A post-MI VSD may be
accompanied by a thrill along the left sternal edge in 90% of cases, and the murmur would be
holosystolic. Free wall rupture is more common in anterior MIs and is more commonly seen in first
presentation of coronary disease and in elderly women. Pulmonary embolism is less likely given her
normal oxygen saturation.
100. A 58-year-old man presents to a community hospital 90 minutes from the nearest cardiac
catheterization laboratory with 2 hours of substernal chest pain. He is noted on electrocardiogram
to have marked ST elevation in leads V1-V5, with reciprocal ST depression in the inferior leads,
consistent with an evolving anterior ST-segment elevation myocardial infarction (STEMI).

Two years previously, he underwent an aortic valve replacement with a St. Jude mechanical
prosthesis, due to chronic aortic insufficiency. Prior to the aortic valve replacement, his coronary
angiogram revealed a 30% stenosis in the proximal left anterior descending coronary artery, which
was not bypassed. He is taking warfarin, and 3 days ago his international normalized ratio (INR) was
3.2.
Which of the following is the most appropriate treatment in this patient's care?
a. Aspirin, clopidogrel, intravenous unfractionated heparin, and tenecteplase.
b. Aspirin, clopidogrel, half-dose tenecteplase, followed by transfer for PCI.
c. Aspirin, clopidogrel, and transfer for primary percutaneous coronary intervention (PCI).
d. Aspirin plus tirofiban.
100. c. Aspirin, clopidogrel, and transfer for primary percutaneous coronary intervention
(PCI).

Transfer to a primary PCI-capable facility is indicated if first medical contact to device time is <120
minutes, as in this case. Transfer should also be considered for those at high risk of bleed or signs
of shock. His risk for bleeding is higher with thrombolytics given his chronic anticoagulation. Even if
there was a time delay for primary PCI, one might consider transfer since bleeding risk increases
with a higher INR and thrombolytics. Aspirin plus tirofiban is not definitive reperfusion. Facilitated
PCI has not been demonstrated to improve outcomes.
101. An 82-year-old man presents with a non-ST elevation myocardial infarction, complicated by
congestive heart failure. His initial laboratory values are notable for a serum creatinine of 3.0
mg/dl, a creatine kinase-myocardial band of 147 mg/dl, and a cardiac troponin I of 23 mg/dl. He is
treated with aspirin, heparin, and eptifibatide and undergoes cardiac catheterization on hospital
Day 2. He has successful percutaneous transluminal coronary angioplasty and stenting of a 90%
stenosis in the proximal left anterior descending artery, and initially does well.

After the procedure, unfractionated heparin was discontinued, and the patient was maintained on
aspirin and a tirofiban infusion, which finished 1 hour ago. Sheaths were successfully removed soon
after the procedure. Later that evening, he is noted to be confused and agitated. On examination,
his heart rate is 116 bpm, blood pressure 88/58 mm Hg, and respirations are 20 breaths/minute.
His peripheral pulses are intact, and there is no groin hematoma. His electrocardiogram (ECG) is
unchanged.
In addition to intravenous saline, which of the following is the next best step in his care?
a. Urgent repeat coronary angiography.
b. Abdominal/pelvic computed tomography.
c. Platelet transfusion.
d. Intravenous Solu-Medrol.
101. b. Abdominal/pelvic computed tomography.

The most likely diagnosis is retroperitoneal hemorrhage, as a complication of cardiac


catheterization and antiplatelet therapy. Importantly, the patient has a significant renal
dysfunction, which will potentiate the antiplatelet effects of tirofiban. Since tirofiban and
eptifibatide are reversible inhibitors of the glycoprotein IIb/IIIa receptor and have a short half-life,
platelet infusions are rarely necessary or helpful in patients receiving these agents.

In contrast, abciximab binds avidly to the receptor and thus has a very prolonged effect on platelet
aggregation (despite low free abciximab levels). Thus, platelet transfusions may be considered in
patients with bleeding complications following abciximab administration. The presentation is not
consistent with allergic reaction, therefore, steroids are not indicated. Repeat ECG is unchanged,
therefore a repeat catheterization is not indicated.
102. A 54-year-old man is brought to the emergency department (ED) by his coworker with acute-
onset dyspnea and syncope. He lost consciousness at work for approximately 15 seconds, but was
spontaneously revived after falling to the floor. His medical history includes hypertension, active
smoking, diabetes, and chronic kidney disease. He took an aspirin while on route to the ED.

His heart rate is 92 bpm and regular, blood pressure is 156/88 mm Hg, and respiration rate is 24
breaths/minute. Jugular venous pressure is normal, lung fields have scant bibasilar crackles, and he
has no cardiac murmur. Initial cardiac enzymes are negative, blood urea nitrogen is 42, and
creatinine is 2.6 mg/dl. Electrocardiography shows normal sinus rhythm with 2-4 mm ST elevation
in V2-V4. Symptoms began 45 minutes before arrival. In traffic, the nearest percutaneous coronary
intervention (PCI)-capable facility is 2.5 hours away at this time of day. Helicopter transport is not
available due to ocean fog.
Which of the following treatment strategies would you choose prior to transfer?
a. Tirofiban, half-dose tenecteplase, IV UFH, aspirin.
b. Full-dose tenecteplase, intravenous (IV) unfractionated heparin (UFH), aspirin, clopidogrel.
c. IV UFH, aspirin, clopidogrel.
d. Abciximab, half-dose tenecteplase, IV UFH, aspirin.
e. Full-dose tenecteplase, IV low molecular weight heparin (LMWH), aspirin, clopidogrel.
b. Full-dose tenecteplase, intravenous (IV) unfractionated heparin (UFH), aspirin,
clopidogrel.

Given the time constraints associated with transfer for primary PCI (i.e., first medical contact to
device time >120 minutes), full-dose lytic therapy with transfer is the single best choice (Class I). IV
UFH is preferred over LMWH in the setting of renal insufficiency of this magnitude. Combination
reperfusion therapy with abciximab and half-dose tenecteplase is not recommended by the current
ST-segment elevation myocardial infarction (STEMI) guideline. The use of heparin, aspirin, and
clopidogrel alone is not appropriate for treatment of a STEMI.
103. A 74-year-old man presents with pain in the toes of his right foot. The pain is most severe at
night when he is in bed, such that he often sits up and dangles his foot over the side of the bed for
relief.
On examination, there is a fissure at the tip of the right great toe that is tender to touch. There is
mild bilateral ankle edema. The dorsalis pedis and posterior tibial pulses are not palpable in either
foot.
Which the following diagnostic tests should be performed to determine appropriate treatment for
this patient?
a. Magnetic resonance (MR) angiogram.
b. Venous duplex ultrasound.
c. Foot X-ray.
d. Computed tomography (CT) angiogram.
103. a. Magnetic resonance (MR) angiogram.

This patient presents with symptoms and findings of peripheral arterial disease (PAD). Ischemic
pain in the right foot is exacerbated when he is in bed, so he dangles the foot to improve perfusion
pressure (via gravitational effects) and reduce his pain. The finding of a skin lesion is consistent
with severe ischemia. The ankle swelling reflects the prolonged periods of putting his feet in a
dependent position; it is not a consequence of venous thrombosis. Therefore, a venous ultrasound
examination is not required.

A foot X-ray might useful to detect osteomyelitis, but these are not diagnostic for PAD. An
arteriogram is indicated to define his vascular anatomy and plan a revascularization procedure. MR
imaging is preferred (Class I, Level of Evidence A) versus CT angiogram (Class IIb, Level of Evidence
B) to diagnose the anatomic location of the lesion.
104. A 24-year-old woman with mixed connective tissue disease is admitted with weakness and
dizziness. Her blood pressure is 105/70 mm Hg, neck veins are elevated, chest is clear, cardiac exam
is normal, and there is no edema. She is referred to the cardiac catheterization laboratory. An intra-
arterial catheter yields the pressure tracing shown in Figure 1.
Which of the following hemodynamic abnormalities is demonstrated?

a. Pulsus parvus et tardus.


b. Pulsus paradoxus.
c. Wide pulse pressure.
d. Pulsus alternans.
104. b. Pulsus paradoxus.

There is marked (>10 mm Hg) variation in the systolic arterial pressure. This finding indicates
exaggerated variation in systolic arterial pressure with the respiratory cycle, and is known as pulsus
paradoxus.

Pulsus parvus et tardus, a low pulse pressure with a slow upstroke, is observed in severe aortic
stenosis. Pulsus alternans, in which there are alternating strong and weak beats with alternating
higher and lower systolic arterial pressure, is seen in severe left ventricular systolic dysfunction. A
wide pulse pressure is observed in aortic regurgitation.
105. A 78-year-old woman with an anterior ST-segment elevation myocardial infarction (STEMI) is
treated with fibrinolysis, low molecular weight heparin (LMWH), aspirin, and clopidogrel, beginning
42 minutes after onset of pain. She is transported to the nearest percutaneous coronary
intervention center. Two hours later, she develops severe headache and is less responsive. A head
computed tomography scan shows a right frontal lobe hemorrhage.
In addition to neurosurgical consultation, which of the following treatments would you
recommend?
a. Epsilon aminocaproic acid.
b. Platelet transfusion.
c. Aprotinin.
d. Vitamin K.
e. Desmopressin acetate (DDAVP).
105. b. Platelet transfusion.

This woman has suffered a fibrinolytic-related intracranial hemorrhage. Risk factors for bleeding
after thrombolytics include age, sex, small body size, and wide pulse pressure, in addition to the
particular agent used.

Recommended treatments include:

 Cessation of fibrinolytic, antithrombin, and antiplatelet therapies


 Protamine to reverse the effects of heparin or LMWH
 Fresh frozen plasma to provide clotting factors V and VIII
 Prothrombin complex concentrate to provide additional clotting factors
 Platelets
 Elevation of the head of the bed, hyperventilation, mannitol, and neurosurgical evacuation if
indicated

The other choices listed are not recommended. DDAVP is not indicated in the treatment of
bleeding after thrombolytics. Aprotinin, an antifibrinolytic agent, was withdrawn from the US
market in May 2008 due to adverse findings in a study of patients undergoing cardiac surgery.
Epsilon aminocaproic acid is not approved for this indication. Vitamin K is used for reversal of
bleeding related to vitamin K antagonists such as warfarin.
106. A 62-year-old woman is admitted with progressive angina, ST depression in leads V4-V6, and a
cardiac troponin value that peaks at twice the upper reference limit. Invasive therapy is
recommended, but declined by the patient. She stabilizes after treatment with intravenous
heparin, aspirin 81 mg, atorvastatin 80 mg, and metoprolol 25 mg bid. At the time of discharge, she
is chest-pain free while walking around the unit. Her echocardiogram demonstrates normal left
ventricular function.
Which of the following do you recommend for medical therapy, in addition to the medications
listed above?
a. Cilostazol 100 mg twice daily for 30 days.
b. Enoxaparin 1.0 mg/kg twice daily for 14 days.
c. Ticagrelor 90 mg BID for 14 days.
d. Clopidogrel 75 mg daily for 1 year.
e. Prasugrel 10 mg daily for 14 days.
106. c. Ticagrelor 90 mg BID for 14 days.

For patients with unstable angina/non-ST-segment elevation myocardial infarction (NSTEMI) for
whom a conservative strategy is chosen, aspirin (75-162 mg) is continued indefinitely (Class I).
Antithrombin therapy with either unfractionated heparin, enoxaparin, or fondaparinux can be
given for the duration of hospitalization or up to 8 days (Class I). Based on the CURE trial, the
unstable angina/NSTEMI clinical practice guideline recommends clopidogrel 75 mg daily for at least
30 days and ideally for up to 1 year after presentation (Class I).

Cilostazol has been used as an alternative to thienopyridine therapy in allergic patients after
percutaneous coronary intervention, but has not been studied in this setting. Prasugrel has not
been studied in a conservative management strategy. Ticagrelor is an option for conservative
management therapy, but the duration should be for 1 year. Enoxaparin might be used, but not for
14 days.
107. A 62-year-old man with diabetes and dyslipidemia is hospitalized with recurrent angina 4
months after percutaneous coronary intervention (PCI) with deployment of a bare-metal stent for
treatment of a 90% mid left anterior descending stenosis. During his previous admission, he
developed a deep vein thrombosis and was treated with intravenous unfractionated heparin. His
platelet count fell from 223,000 to 62,000, and a platelet factor (PF4) assay at time of discharge
was positive. He was discharged on warfarin, aspirin, clopidogrel, atorvastatin, and metoprolol.

He was readmitted with chest pain and ST-segment depression 3 months later. His hepatic function
and cardiac enzymes are normal, but he has stage III kidney disease. His pain resolves and his
electrocardiogram normalizes after 2 hours.

A repeat cardiac catheterization reveals a 70% in-stent stenosis.


Which of the following medications would you consider for adjunctive therapy at the time of
repeat PCI?
a. Unfractionated heparin.
b. Bivalirudin.
c. Fondaparinux.
d. Enoxaparin plus a glycoprotein IIb/IIIa receptor antagonist.
107. b. Bivalirudin.

This man developed heparin-immune thrombocytopenia during his first admission. Unfractionated
heparin and low molecular weight heparin are contraindicated in this situation. Upstream use of
fondaparinux has been associated with higher rates of catheter thrombus at time of PCI,
necessitating the use of heparin.
108. Which of the following is the correct order of events in the ischemic cascade?
a. ECG changes, perfusion defects, systolic dysfunction, diastolic dysfunction, chest pain.
b. Diastolic dysfunction, perfusion defects, ECG changes, chest pain, systolic dysfunction.
c. Perfusion defects, diastolic dysfunction, systolic dysfunction, electrocardiogram
(ECG) changes, chest pain.
d. Perfusion defects, diastolic dysfunction, systolic dysfunction, chest pain, ECG changes.
e. Diastolic dysfunction, systolic dysfunction, perfusion defects, chest pain, ECG changes.
108. c. Perfusion defects, diastolic dysfunction, systolic dysfunction, electrocardiogram (ECG)
changes, chest pain.

Understanding the order of events that occur in the ischemic cascade is important in understanding
how stress testing can detect subclinical disease. Single-photon emission computed tomography
(SPECT) myocardial perfusion imaging (MPI) unmasks exercise- or pharmacologic-induced
differences in perfusion, revealing subtle patterns of perfusion defects and early atherosclerotic
disease. This explains why nuclear perfusion-based techniques, which typically detect abnormalities
in coronary flow reserve even in the absence of ischemia, are more sensitive than contractile
dysfunction-dependent techniques, like stress or dobutamine echocardiography, which require
subendocardial ischemia to be abnormal. ECG changes and chest pain are relatively late
presentations of ischemia, which explains why the sensitivities of coronary artery disease detection
are lower than with imaging techniques.
109. A 60-year-old man underwent bare-metal stenting of a discrete left anterior descending
artery lesion after non-ST-segment elevation myocardial infarction and was placed on aspirin and
clopidogrel. One week after the procedure, he presents with a lower gastrointestinal bleed and is
found to have a previously undetected colon cancer that requires resection. Because of the
extensive surgical exploration that is anticipated, the surgeon requests that the clopidogrel be
stopped prior to surgery.
Which of the following is the most appropriate length of clopidogrel therapy in this setting?
a. 9 months.
b. 12 months.
c. 1 week.
d. 6 months.
e. 4 weeks.
109. e. 4 weeks.

The issue in this clinical situation is a tension between stent thrombosis, which is reduced by dual
antiplatelet therapy, versus periprocedural bleeding, which is accentuated with antiplatelet
therapy. For truly elective yet major surgery, studies suggest that waiting for 4-6 weeks after bare-
metal stenting allows the coronary stent to endothelialize so that perioperative stent thrombosis
off dual antiplatelet drugs is rare. Operating on dual antiplatelet therapy would expose the patient
to unacceptable bleeding risk. To stop clopidogrel now would increase the risk of stent thrombosis.
Heparin has not been shown to reduce the risk of early stent thrombosis. The risk of waiting 6
months to resect the colon cancer outweighs the benefits of 6 months of dual antiplatelet therapy.
110. In a patient with hypertrophic obstructive cardiomyopathy, which of the following
combinations is true of the post-extrasystolic beat?
a. The gradient increases and the pulse pressure decreases.
b. The gradient and pulse pressure both decrease.
c. The gradient decreases and the pulse pressure increases.
d. The gradient and pulse pressure both increase.
110. a. The gradient increases and the pulse pressure decreases.

This is the Brockenbrough sign. The decrease in pulse pressure following a premature ventricular
contraction is caused by a reduced stroke volume that results from increased dynamic obstruction,
as evidenced by an increase in the gradient.
111. A 56-year-old woman presents for evaluation of 8 months of shortness of breath. Her exam
shows blood pressure 150/80 mm Hg, heart rate 78 bpm, and her body mass index is 40 kg/m2.
Jugular venous pressure is mildly elevated with clear lungs. No murmur is heard. She has an
S4 gallop and trace peripheral edema.

A transthoracic echocardiogram is performed, but is of limited quality due to body habitus. Her left
ventricular ejection fraction appears grossly normal. She undergoes right and left heart
catheterization for further evaluation of her symptoms. There is no significant coronary artery
disease on angiography. Right heart catheterization shows the following: right atrium 10 mm Hg,
pulmonary artery 55/25 mm Hg (mean 35), and pulmonary capillary wedge pressure 23 mm Hg
with prominent V waves.
Which of the following is the most likely cause of her symptoms?
a. Precapillary pulmonary hypertension.
b. Mitral regurgitation.
c. Obesity.
d. Heart failure with preserved ejection fraction (HFpEF).
111. d. Heart failure with preserved ejection fraction (HFpEF).

The hemodynamics shown are consistent elevated filling pressures in a noncompliant left atrium.
This constellation of symptoms and hemodynamic findings are most compatible with HFpEF.

Mitral regurgitation would be expected to be associated with a systolic murmur in most cases.
While obesity may be a cause of her symptoms, it would not necessarily explain the
hemodynamics. Precapillary pulmonary hypertension would result in a significant transpulmonary
gradient that is not present here.
112. A 44-year-old woman is in the coronary care unit following presentation with an inferior
myocardial infarction. She was treated with primary angioplasty and stenting to a lone proximal
right coronary artery (RCA) stenosis.

Upon returning to the coronary care unit, she develops mild palpitations. She has no further chest
discomfort. Her blood pressure is 110/70 mm Hg and heart rate is 90 bpm. The 12-lead
electrocardiogram is shown in Figure 1.
Which of the following is the best management for this rhythm?

a. Atropine.
b. Synchronized direct current cardioversion.
c. No therapy required.
d. Repeat percutaneous coronary intervention (PCI).
e. Lidocaine.
112. c. No therapy required.

The rhythm strip shows a sinus beat with inferior Q waves followed by an accelerated
idioventricular rhythm. In the context of PCI to the RCA, this represents a reperfusion rhythm; this
is self-limiting and does not require treatment.
113. A 17-year-old is referred by his pediatric cardiologist for advanced cardiovascular imaging
after a classic tetralogy of Fallot repair at the age of 2 months. His exercise tolerance is normal, and
he is not cyanotic. An echocardiogram reveals right ventricular enlargement and pulmonic
regurgitation.
Which of the following is most appropriate at this time to further assess the status of his congenital
heart disease?
a. Radionuclide angiography.
b. Computed tomographic angiography.
c. Magnetic resonance imaging (MRI).
d. Transesophageal echocardiography.
e. Right-heart catheterization.
113. c. Magnetic resonance imaging (MRI).

Cardiac MRI is the only test that can provide all the components needed to quantify all the
parameters necessary, including right ventricular and right atrial volumes, valvular regurgitation,
and residual shunting. The right ventricular structures are difficult to quantify by echocardiography,
and computed tomography does not provide dynamic flow information. Right-heart catheterization
will not provide volumes or accurate quantification of valvular regurgitation.
114. A 25-year-old college basketball player is referred to see you due to an abnormal screening
electrocardiogram (ECG) obtained during basketball practice (Figure 1). He has no complaints, has a
normal physical examination, and has no family history of arrhythmias or sudden death.
Which of the following is the next most appropriate step in his management?

a. Repeat electrocardiogram (ECG).


b. Cardiac catheterization.
c. Echocardiogram.
d. Stress test.
e. Holter monitor.
114. a. Repeat electrocardiogram (ECG).

This ECG is normal except for limb lead reversal. This can be seen by the negative P wave QRS
complex and T wave in lead I, as well as a total lack of voltage in lead II. It can be repeated with the
limb leads correctly placed to document the lack of underlying pathology. Other imaging or
diagnostic studies are not indicated in an asymptomatic patient with an otherwise normal ECG
without a family history of arrhythmias or premature death.
115. Which of the following patients has the highest probability of coronary artery disease (CAD)?
a. A 65-year-old woman with atypical angina and a positive exercise ECG.
b. A 65-year-old man with typical angina and a negative exercise electrocardiogram (ECG).
c. A 45-year-old man with atypical angina and a positive exercise ECG.
d. A 45-year-old woman with typical angina and a positive exercise ECG.
115. b. A 65-year-old man with typical angina and a negative exercise electrocardiogram
(ECG).

According to the Bayes Theorem, the post-test probability that a patient has CAD depends on both
the pretest probability of the patient having CAD, and the results of the exercise test (Figure 1).
Pretest probability of CAD depends on age, sex, and character of chest pain, and can be determined
for an individual patient from published tables.

For patients at the extreme (low <10%, high >90%), the results of the exercise test shift post-test
probability of CAD very little (<10%). The 45-year-old man with atypical angina and a positive
exercise ECG, still has a post-test probability of CAD that exceeds 85-90%, because pretest
probability is so high. This patient has a pretest probability of CAD that exceeds 90%.

The other patients described all have a pretest probability of CAD of approximately 50%. A positive
exercise test shifts their post-test probability to approximately 75-80%.
(Figure 1)
116. A 48-year-old African American man presents to your office with palpitations. He is otherwise
healthy and has no other complaints. His physical examination is unremarkable. His
electrocardiogram shows sinus rhythm with a PR interval of 230 msec. On Holter evaluation, he has
6,000 multifocal premature ventricular contractions (PVCs) over a 24-hour period. Transthoracic
echocardiography reveals normal biventricular function without valvular abnormalities.
Which of the following is the next best step in his care?
a. Left-heart catheterization.
b. Cardiac computed tomographic angiography.
c. Cardiac magnetic resonance imaging (MRI).
d. Single-photon emission computed tomography (SPECT) myocardial perfusion imaging (MPI).
e. Transesophageal echocardiography.
116. c. Cardiac magnetic resonance imaging (MRI).

This patient's clinical presentation is most consistent with sarcoidosis. Given his race, high PVC
burden, and conduction system disease, excluding this diagnosis is essential. Cardiac MRI and
fluorodeoxyglucose (FDG) PET are two modalities that can help establish the diagnosis of sarcoid.

He has no cardiovascular symptoms of ischemia. Thus, cardiac catheterization and SPECT MPI are
not indicated. Transesophageal echocardiography will not add additional information.
117. A 75-year-old woman was admitted 4 days after the onset of substernal chest pain. The patient is an active
smoker and has a history of high cholesterol, hypertension, and diabetes. On admission, she was free of chest and
epigastric pain and had a blood pressure of 142/70 mm Hg with a heart rate of 89 bpm.

Laboratory testing revealed that the troponin I was elevated at 20 ng/ml and the creatine kinase-myocardial band
was within normal limits. The patient's electrocardiogram had Q waves in leads II, III, and aVF and there were no
other ischemic changes. Diagnostic angiography demonstrated an occluded right coronary artery (RCA),
nonobstructive left anterior descending artery disease, and a 70% proximal left circumflex artery.

The patient was managed conservatively and medical therapy was implemented. Two days later in the cardiac
telemetry unit, the patient became acutely hypoxic, hypotensive, and required intubation. On examination, she
appears acutely ill with bilateral rales and a loud systolic murmur is present, which was not heard on admission.
Her blood pressure is 85/60 mm Hg and her heart rate is 110 bpm. An echocardiogram reveals an acute
ventricular septal defect (VSD). An intra-aortic balloon pump is placed and her hemodynamics stabilize.
Which of the following is the next appropriate management step?

a. PCI of the RCA.


b. Implant a left ventricular assist device (LVAD).
c. Stabilize for 4 weeks, then cardiovascular surgery.
d. Urgent surgical VSD repair.
e. Percutaneous coronary intervention (PCI) of the left circumflex artery.
117. d. Urgent surgical VSD repair.

Post-infarct VSDs typically occur 2-8 days after a myocardial infarction (MI). This is often after a
first MI. Medical management of post-MI VSDs have very high mortality rates (>90%) and even
with cardiac surgical or percutaneous repair, the mortality rates are >50%. Patients who survive
do better with early surgical VSD repair, as the ruptured site can expand abruptly. Percutaneous
closure is an emerging potential option, but early results suggest that residual shunting persists.
As device development improves, this may become a viable option. PCI alone would not correct
the hemodynamic situation, and an LVAD will not address the shunting problem.
118. A 45-year-old woman presents to the emergency department with 2 weeks of intermittent
rest substernal chest pain, which she describes as sharp. It is not provoked by exercise and has
lasted up to 30 minutes. There are no cardiac risk factors, her electrocardiogram (ECG) and cardiac
enzymes are normal, and she is able to exercise.

Which of the following is most appropriate in this patient's care?


a. Coronary angiogram.
b. Exercise stress echocardiogram.
c. Cardiac computed tomography (CT) angiography.
d. Exercise myocardial perfusion imaging.
e. Exercise stress test.
118. e. Exercise stress test.

In this low-risk patient with a normal ECG and ability to exercise, exercise stress testing is the most
appropriate first step. Cardiac CT angiography may be helpful in patients who are at intermediate
risk. This patient with atypical nonexertional chest pain is low risk. With a baseline normal ECG,
stress testing with additional imaging should be reserved for equivocal exercise stress testing
results, inability to exercise, or high-risk exercise features.
119. An 89-year-old woman residing at a nursing home comes to the emergency department of your hospital with a 24-hour
history of dyspnea and confusion. The patient has significant dementia, as well as hypertension, heart failure with preserved ejection
fraction, and insulin-dependent diabetes. She is agitated and disoriented, and unable to answer questions.

Her electrocardiogram demonstrates a heart rate of 72 bpm, with a normal axis, PR interval of 155 msec, a narrow QRS complex,
and inferior ST elevation. Her troponin T is 1.6 ng/ml and her creatinine is 2.8 mg/dl; her creatinine clearance is calculated as 25
ml/min. She weighs 60 kg.

Initial treatment and response: Emergency Medical Service has already given the patient 325 mg of aspirin. As you are preparing to
wheel the patient to the cardiac catheterization laboratory, her daughter arrives, who holds medical power of attorney. She tells you
that her mother has a Do Not Resuscitate, Do Not Intubate (DNR/DNI) document and would not want to undergo a procedure.
However, she agrees to the administration of fibrinolytic therapy.

In addition to giving fibrinolytics and aspirin 81 mg qd, which of the following do you recommend as additional antiplatelet and
anticoagulant therapy?

a. Give a clopidogrel dose of 75 mg orally, followed by 75 mg daily. Also initiate unfractionated heparin with weight-based
dosing.
b. Give a clopidogrel load of 300 mg orally, followed by 75 mg daily. Also initiate enoxaparin 60 mg subcutaneously twice daily.
c. Give a clopidogrel dose of 75 mg orally, followed by 75 mg daily. Also initiate enoxaparin 60 mg subcutaneously twice daily.
d. Give a clopidogrel load of 300 mg orally, followed by 75 mg daily. Also initiate unfractionated heparin with weight-based
dosing.
e. Give a prasugrel load of 60 mg orally, followed by 10 mg daily. Also initiate unfractionated heparin with weight-based dosing.
119. a. Give a clopidogrel dose of 75 mg orally, followed by 75 mg daily. Also initiate
unfractionated heparin with weight-based dosing.

The correct answer is to give 75 mg of clopidogrel, followed by 75 mg a day, followed by the use of
unfractionated heparin. In this elderly patient with central nervous system symptoms and renal
dysfunction, adjunctive antithrombotic therapy is limited following fibrinolytic therapy.

Prasugrel is not recommended in patients over 75 years of age. Enoxaparin dosing is complex in
patients with chronic kidney disease and every 24-hour dosing is recommended if creatinine
clearance is <30 cc/min. In patients over the age of 75 years, no loading dose of clopidogrel should
be used. Fondaparinux is also contraindicated if the creatinine clearance is <30 cc/min.
120. A 62-year-old woman presented to the emergency department with 2 hours of crushing
substernal chest pain. An electrocardiogram obtained demonstrates sinus rhythm at 95 bpm with
1-2 mm of ST elevation in leads II, III, and aVF. Her medical history is significant for dyslipidemia
and a possible transient ischemic attack. Her current medications include simvastatin. The patient
receives aspirin and is taken to the catheterization laboratory.
Which of the following agents confers the greatest survival advantage?
a. Prasugrel 60 mg orally.
b. Enoxaparin 1 mg/kg.
c. Clopidogrel 600 mg orally.
d. Intravenous heparin 70 U/kg.
e. Ticagrelor 180 mg orally.
120. e. Ticagrelor 180 mg orally.

Either clopidogrel or ticagrelor are Class I recommendations. Prasugrel should be administered


after the anatomy is known and it is certain that the patient will be undergoing percutaneous
coronary intervention (PCI). In patients with non-ST-elevation acute coronary syndrome (NSTE-ACS)
and high-risk features (e.g., elevated troponin) and not adequately pretreated with clopidogrel or
ticagrelor, it is useful to administer a glycoprotein IIb/IIIa inhibitor (abciximab, double-bolus
eptifibatide, or high-dose bolus tirofiban) at the time of PCI (Class I).

Ticagrelor therapy was associated with less cardiovascular mortality than clopidogrel in the PLATO
trial. Prasugrel therapy was associated with fewer ischemic events than clopidogrel in the TRITON
trial, but mortality did not differ between treatment groups. Pretreatment with heparin or
bivalirudin has never been shown to reduce death.
121. There are multiple centers in Gotham City that perform >500 percutaneous coronary
interventions (PCIs) per year. Two of these centers spend significant resources advertising their
expertise in radial access.
PCI performed via radial access has a demonstrated mortality benefit compared with femoral
access when performed for which of the following indications?
a. Non–STEMI.
b. Assessment of valvular heart disease.
c. Chronic stable angina.
d. Unstable angina with negative biomarkers.
e. ST-segment elevation myocardial infarction (STEMI).
121. e. ST-segment elevation myocardial infarction (STEMI).

The mortality benefit of radial access PCI has been demonstrated specifically in STEMI. While there
are multiple other benefits of radial access over femoral access, including greater patient
satisfaction and lower access site complications such as bleeding, mortality benefit has not been
demonstrated in the other clinical situations listed.
122. A thin 62-year-old man is referred by the patient’s family practitioner for preoperative
assessment of a large (5.8 cm) abdominal aortic aneurysm. He has a medical history of chronic
obstructive pulmonary disease (COPD) treated with inhalers, a left hemispheric stroke with little
residual, diabetes requiring insulin, and chronic renal insufficiency. The patient has one-block
claudication.

On exam, his vital signs are normal, and he has no evidence of cardiac murmurs, rubs, or gallops.
Chest examination reveals bilateral wheezes. There is no lower extremity edema. The
electrocardiogram (ECG) is normal.
Which of the following would be your next step in this man’s preoperative evaluation?
a. Dobutamine echocardiography.
b. Exercise ECG stress testing.
c. Resting echocardiogram.
d. No additional studies are indicated.
e. Pharmacologic stress testing using regadenoson.
122. a. Dobutamine echocardiography.

A vasodilator study would not be wise in this patient, because there is evidence of wheezes on the lung exam.
Adenosine may induce bronchoconstriction and respiratory compromise by activating the A2B and/or
A3 receptors, resulting in mast cell degranulation, immunoglobulin E, and histamine release. Regadenoson is an
A2A-receptor agonist, which theoretically does not appreciably agonize A2B and/or A3 receptors, but as per the
American Society of Nuclear Cardiology imaging guidelines for nuclear cardiology procedures, patients with
bronchospasm are considered a contraindication to the use of regadenoson, because there are inadequate data
regarding its safety with active wheezing. In a recent publication of 356 patients with asthma or COPD undergoing
stress testing with regadenoson, however, there was no evidence of severe bronchoconstriction; despite this, the
authors still advised caution in these patients.

Exercise ECG stress testing is unlikely to have diagnostic value because the likelihood of achieving an adequate
heart rate and blood pressure response is low due to his disabling claudication.

His abdominal aneurysm is considered large. Aneurysm diameter is the most important factor predisposing to
rupture. Based on this patient’s aneurysm diameter, his annual rate of rupture is 3-15%, as per the Joint Council
of the American Association for Vascular Surgery and Society for Vascular Surgery. This patient should undergo
repair of his aneurysm.
According to the guideline on perioperative cardiovascular evaluation and management of noncardiac surgery
patients, those with high cardiovascular risks with functional status <4 metabolic equivalents should have stress
testing if it would alter preoperative management by the identification of severe coronary artery disease. A
dobutamine echocardiogram is therefore the best available test in this situation.

The patient does not require a resting echocardiogram, as there are no signs of fluid overload and the cardiac
physical exam is normal. In the absence of history of heart failure or ischemic heart disease, there would be no
indication for the resting echocardiogram.
123. A 51-year-old woman is diagnosed with breast cancer. She will undergo a mastectomy with
adjuvant chemotherapy with doxorubicin hydrochloride. The hospital protocol requires the patient
to have periodic evaluations of cardiac ejection fraction (EF).

An echocardiogram is performed and her EF cannot be evaluated despite the use of echo contrast
due to body habitus. The patient is extremely claustrophobic. The electrocardiogram (ECG) reveals
criteria for left ventricular (LV) hypertrophy.
Which of the following is the best imaging alternative in the care of this patient?
a. A gated single-photon emission CT (SPECT) study.
b. A radionuclide ventriculography (RVG).
c. Computed tomography (CT) angiogram.
d. A 3D echocardiogram.
e. Cardiac magnetic resonance imaging (MRI).
123. b. A radionuclide ventriculography (RVG).

As cardiologists, our role is to provide the oncologist with the necessary information to make therapeutic decisions. It is up to the
oncologist to decide if an alternative to the doxorubicin hydrochloride is as effective in this type of malignancy.

Gated-equilibrium RVG is a procedure in which the patient’s red blood cells (RBCs) are radiolabeled and ECG-gated cardiac
scintigraphy is obtained. Single or multiple measurements of LV and/or right ventricular function are determined. Data are
collected from several hundred cardiac cycles to generate an image set of the beating heart that is presented as a single,
composite cardiac cycle. The method provides the clinician with regional and global wall motion, cardiac chamber size and
morphology, and ventricular systolic and diastolic function, including LV and right ventricular EFs. The most common clinical
settings in which RVG may be useful include: known or suspected coronary artery disease, distinguishing systolic from diastolic
causes of congestive heart failure, evaluating cardiac function in patients undergoing chemotherapy, and assessing ventricular
function in patients with valvular disease. There are certain potential sources of error for this technique, which include red blood
cell labeling (certain medications alter the labeling efficiency of the isotope), patient positioning, gating errors, and image
statistics. Since this technique involves radiation, its use has decreased (but it is still a viable alternative to other modalities).

Cardiac MRI would have been a good alternative, because it is considered a gold standard for evaluation of the EF, but the patient
is claustrophobic, which is a major limitation of this technique.

LV chamber and mass quantification have been studied extensively using 3D echocardiography. This method has been shown to
be more accurate and reproducible than 2D, but the accuracy of the volume calculations is highly dependent on image quality.
The patient had a 2D echocardiogram performed, but image quality was poor despite the use of echo contrast. A 3D echo would
have the same issues.

The accuracy of EF measurement from gated-SPECT has been validated with respect to a variety of radiopharmaceuticals and
against a large number of gold standards (MRI, 3D multigated acqusition scan, electron beam CT, etc.). EF can be underestimated
in patients with LV hypertrophy, though, because algorithms are either calibrated for the range of thickness most typically
encountered in clinical practice or assume a fixed myocardial thickness. The reported discrepancy in EF may be up to 4 points.
This suggests the RVG would provide a more accurate EF determination in this setting.
124. For a patient with aortic stenosis and 3+ aortic regurgitation (AR), you are asked to provide an
aortic valve area (AVA) to help decide about surgical intervention.

The following data are obtained at cardiac catheterization:

 Peak aortic gradient 50 mm Hg


 Mean aortic gradient 36 mm Hg
 Fick cardiac output 3 L/min
 Thermodilution cardiac output 3.2 L/min

Based just on these data, which of the following is correct regarding the determination of the
estimated AVA?
a. It cannot be determined in the catheterization laboratory because of significant AR.
b. The AVA will be falsely smaller using the Fick cardiac output.
c. The AVA can be determined using the thermodilution cardiac output.
d. The AVA can be determined using the Fick cardiac output.
124. a. It cannot be determined in the catheterization laboratory because of significant AR.

One can estimate the AVA by use of the Hakki equation. It is defined by:
flow past the valve (L/min) / square root of the mean gradient (mm Hg). The Fick cardiac output
and the thermodilution cardiac output describe the forward flow through the heart to the
periphery (actually pulmonary flow, but it should equal systemic flow if no shunt). The flow past
the aortic valve, though, includes both the forward flow and the regurgitant flow due to the AR,
and this is what is needed to do the calculations.

In this case, the estimated AVA would be: 6 L/min / 6 mm Hg or an AVA of 1.0 cm2. Using the Fick
output, the flow would erroneously be less than actual, resulting in a falsely low AVA.
125. A 75-year-old man with a medical history of hypertension, hyperlipidemia, diabetes, and a
remote history of neck injury after a fall, presents to the emergency department with vertigo and
blurred vision that lasted 20 minutes and resolved spontaneously. His symptoms started
immediately after turning his head to the right. Physical examination is normal except for a bruit on
the left side of his neck. Noncontrast computed tomography (CT) of the brain is normal.
Which of the following is the most appropriate test for this patient?
a. Dix-Hallpike test.
b. Carotid Doppler ultrasonography.
c. Cerebral angiogram.
d. Head and neck CT angiography (CTA).
125. d. Head and neck CT angiography (CTA).

The patient’s symptoms are consistent with posterior circulation cerebrovascular syndrome (or
vertebrobasilar insufficiency). Typical symptoms of posterior circulation ischemia can include
vertigo, presyncope, syncope, diplopia, blurred vision, perioral numbness, tinnitus, ataxia, or
bilateral sensory deficits. His symptoms began with turning his head. Posterior circulation ischemia
can be caused by vessel impingement from the transverse processes in the cervical spine that
occurs with head movement. Given the patient’s history of neck injury, it is possible that an
unrecognized neck fracture of the transverse process could have occurred. Vertigo that occurs with
head movement is typically due to benign paroxysmal positional vertigo and can be assessed using
the Dix-Hallpike maneuver. However, blurred vision is unlikely to occur with benign paroxysmal
positional vertigo, thus the response Dix-Hallpike test is incorrect.

Despite having a left carotid bruit, the patient's symptoms are not consistent with anterior
circulation ischemia. For vertebral artery disease, the sensitivity and specificity of CTA or magnetic
resonance angiography is 94% and 95%, respectively, and significantly better than carotid Doppler
ultrasonography (sensitivity = 70%). Therefore, carotid Doppler ultrasonography is incorrect.
Cerebral angiography should be reserved for patients that require revascularization based on
noninvasive imaging. Thus, cerebral angiogram is incorrect. CTA is the correct answer.
126. You are the preceptor of a cardiology clinic in a university hospital setting. One of the
residents is evaluating a middle-aged woman with intermediate risk for having coronary artery
disease and an abnormal resting electrocardiogram (ECG). She has poor echocardiographic
windows and requires a nuclear stress test.
Which of the following protocols results in the LEAST radiation exposure to the patient?
a. One-day rest/stress with technetium-99m.
b. Stress only with technetium-99m.
c. Dual isotope (thallium and technetium).
d. PET scan using rubidium-82.
e. A positron emission tomography (PET) scan with fluorodeoxyglucose (FDG) and ammonia.
126. e. A positron emission tomography (PET) scan with fluorodeoxyglucose (FDG) and
ammonia.

Ammonia-13 (N-13) is a blood flow tracer and has a half-life of 10 minutes. It is generated in a cyclotron and the
average effective dose resulting from one of these studies is 2 mSv, making this stress protocol the correct
answer. The dosimetry is relatively low due to the short half-life of N-13 and the low energy of the emitted
positrons: 1.48 mSv in 20 mCi (usually cardiac studies utilize 10-20 mCi/study). The 1-day rest/stress protocol with
technetium is linked to an average effective dose of about 12 mSv. This protocol is one of the most frequently
used protocols in single-positron emission computed tomography (SPECT).

Rubidium-82 is a perfusion isotope used in PET. The half-life is 75 seconds and is produced in a generator. The
average effective dose generated by this protocol (rest/stress) is about 15 mSv. In an average study, 1.75-7.5 mSv
in 60 mCi (usually cardiac studies utilize 20-40 mCi/study) is used.
The purpose of a rest image on a nuclear stress test is to define reversibility of stress-induced perfusion
abnormalities. In more than 50% of the patients, rest imaging is redundant, and does not provide additional
diagnostic or prognostic information. The average effective dose obtained from a stress-only study using
technetium-99m is 9 mSv. The ideal candidate for this study has a low pretest probability for coronary artery
disease, weighs between 120-250 lbs, and has had a normal stress test in the past 3 years. Potential drawbacks
include the inability to assess ventricular size and function, both at rest and after stress. Transient ischemic
dilation will not be seen, but the likelihood is very low in patients with normal stress perfusion. There are
prognostic and long-term data that suggest that this approach is effective. This would be a reasonable alternative
in the absence of a PET camera.
The average effective dose obtained from a dual isotope study is 35 mSv, which is one of the reasons (if not the
main reason) why this test is rarely used anymore. The half-life of thallium is 73.1 hours and the patient is given
between 2.5-4 mCi per study in addition to the technetium. Thallium thus results in the highest radiation
exposure.
127. During a routine cardiac catheterization procedure, which of the following will DECREASE the
total X-ray dose produced by the X-ray system during a procedure?
a. Using magnified views.
b. Panning over the bony structures compared with the lung fields.
c. Using angulated views.
d. Using lower framing rates.
e. Increasing the source-to-image distance.
127. d. Using lower framing rates.

Only lowering the acquisition framing rate will help lower the X-ray dose among these scenarios.
The X-ray exposure equation must be satisfied for each frame imaged, and while the dose per
frame may be similar, reducing the number of frames per second reduces the overall dose
delivered during a case.

X-rays are divergent, and increasing the source-to-image distance results in magnification of the
image onto the face of the image detector and also loss of X-rays that diverge outside the face of
the image receptor. Since there are fewer X-rays that strike the image detector due to this loss
outside, more X-rays are needed to produce a satisfactory image on the detector, and the dose
necessarily increases.

Similarly, magnified views are created by using less of the face of the image receptor and, again to
produce a satisfactory image, more X-rays are needed. Panning over bony structures compared
with the lung fields also requires more X-rays because of the differences in density of the materials
and the resultant effect on attenuation of the X-ray beam. Angulated views result in an increase in
the source-to-image distance and require a greater X-ray dose.
128. A 68-year-old man (weight 87 kg) with a history of type 2 diabetes, hypertension,
gastroesophageal reflux disease, and seizure disorder presented to the emergency department
(ED) with complaints of chest pain and diaphoresis. The patient first noticed the onset of the chest
pressure the night prior to admission while he was walking on his treadmill. The pain resolved with
rest and recurred while he was getting dressed on the morning of admission.

In the ED, laboratory data were notable for a serum creatinine of 1.9 mg/dl and cardiac biomarkers,
including a troponin T of 0.6 ng/ml and a creatine kinase-myocardial band of 24 U/L. The
electrocardiogram revealed 1 mm of new ST depression in the inferior leads. Current home
medications include simvastatin 40 mg daily, lisinopril 10 mg daily, lansoprazole 30 mg daily, and
phenytoin 300 mg at bedtime. The patient was scheduled for a cardiac catheterization in the
morning.
Which of the following medications would preclude the use of ticagrelor therapy in this patient?
a. Phenytoin.
b. Lisinopril.
c. Simvastatin.
d. Lansoprazole.
e. Metoprolol.
128. a. Phenytoin.

Ticagrelor is metabolized by the hepatic cytochrome P450 (CYP) 3A4 enzyme. Based on this (and
the observation in pharmacokinetic studies of reduced antiplatelet effect when ticagrelor was
given concomitantly with drugs that induce the CYP 3A4 isoenzyme such as rifampin), the
manufacturer currently recommends avoiding the use of ticagrelor with strong inducers of the CYP
3A4 enzyme, including rifampin, phenytoin, carbamazepine, phenobarbital, and dexamethasone.
129. A 62-year-old financial planner with a history of hypertension, diabetes, and peptic ulcer
disease comes to your office for evaluation. She describes nausea and abdominal discomfort that
has been bothering her for several months. She reports that this has been increasingly bothersome
to her despite frequent use of antacids. She notes some relation to consumption of large meals or
at the beginning of exercise. It lasts for several minutes then abates spontaneously. She denies
dyspnea, orthopnea, palpitations, or lightheadedness. She has no other medical conditions.

On physical exam, there is point tenderness located over the left sternum, but otherwise is normal.
Her resting electrocardiogram is normal.

Which of the following options is the most appropriate next step?


a. Cardiac catheterization.
b. Reassurance.
c. Referral for gastrointestinal source of symptoms.
d. Exercise stress testing.
e. Echocardiography.
129. d. Exercise stress testing.

This woman has atypical symptoms, but has several features consistent with ischemic heart disease
and has risk factors for coronary artery disease. The relation of symptoms to exercise and after
meals is consistent with angina. While typical angina pain is described as heaviness, squeezing, or
tightness, atypical symptoms are common in women and the elderly. Nausea and difficult to
characterize discomfort is common in these groups.

In the WISE (Women's Ischemic Syndrome Evaluation) study, 65% of women with ischemia
presented with atypical symptoms. Pain reproduced by palpation is suggestive of a musculoskeletal
etiology, but does not exclude the possibility of her other symptoms being related to ischemic
heart disease. She has diabetes, and gastroparesis may be contributing.

The next appropriate study would be stress testing to assess for coronary artery disease.
130. A 77-year-old man comes to the office to discuss an upcoming cardiac catheterization
procedure that will be performed for angina that is refractory to medical therapy. He has
hypertension, hyperlipidemia, and no other chronic medical conditions. His laboratory tests include
hemoglobin 12 g/dl, creatinine 1.1 mg/dl, and international normalized ratio (INR) 1.0.
You tell him the most common complication of cardiac catheterization with coronary angiography
is which of the following?
a. Vascular access complication.
b. Anaphylactoid reaction to contrast agent.
c. Stroke.
d. Myocardial infarction.
e. Acute renal failure.
130. a. Vascular access complication.

Myocardial infarction, stroke, and death are infrequent complications with a frequency of <0.1%.
More common complications are tachy- or bradyarrhythmias, acute renal failure, and vascular
complications. Vascular access site bleeding is the most common complication of cardiac
catheterization, occurring in 1.5-2.0% of patients. The Registry of the Society for Cardiac
Angiography and Interventions reported 519 major contrast media complications (0.23%) with four
deaths (1 in 55,000) in 222,553 patients undergoing selective coronary cineangiography.

While the radial approach has gained wider acceptance and has fewer bleeding complications
compared with the femoral approach, the incidence of radial artery occlusion is approximately 1-
4%, but this is generally asymptomatic. Although major access site complications are rare,
atrioventricular fistula, nerve injury, radial artery dissection and perforation, hematomas, and
pseudoaneurysm still occur with the radial approach at a rate that exceeds the other listed
complications.
131. An 82-year-old man with diabetes, hypertension, hyperlipidemia, and stable angina comes to
your office for routine follow-up. He has no change in his angina pattern. He plays tennis 3 times
per week with no symptoms. His blood pressure is 140/95 mm Hg and heart rate is 67 bpm. His
glycated hemoglobin is 7%. Urinalysis shows 1+ proteinuria. He is currently only on metoprolol 25
mg bid, atorvastatin 20 mg, and aspirin 81 mg.
Which of the following should be initiated?
a. Sliding scale insulin.
b. Metformin 500 mg daily.
c. Lisinopril 5 mg daily.
d. Hydrochlorothiazide 12.5 mg daily.
e. Clopidogrel 75 mg daily.
131. c. Lisinopril 5 mg daily.

Angiotensin-converting enzyme (ACE) inhibitors are indicated in many patients with stable ischemic
heart disease, especially those with diabetes or left ventricular dysfunction. Weight loss, aerobic
exercise, an American Heart Association Step II diet, and ACE inhibitors decrease the risk of
microvascular complications and potentially cardiac events in patients with diabetes. Tight glycemic
control does not reduce the risk of macrovascular complications in patients with type 2 diabetes; it
may actually increase the risk of cardiovascular death and complications. According to the
guidelines, the goal blood pressure is <140/90 mm Hg in patients with diabetes or chronic kidney
disease. An ACE inhibitor or angiotensin-receptor blocker is first-line therapy in diabetes to prevent
progression of renal disease.
132. A 78-year-old man with peripheral arterial disease is referred to your clinic for preoperative
evaluation prior to femoral-popliteal bypass. He currently can only walk 1-2 blocks due to
claudication. He underwent percutaneous coronary intervention with a bare-metal stent for single-
vessel coronary disease affecting the right coronary artery 2 years ago. He denies angina or
shortness of breath.

A persantine sestamibi stress test demonstrates a small area of ischemia at the apex without
angina. Medications include aspirin 81 mg daily and metoprolol tartrate 50 mg twice daily. Blood
pressure is 112/72 mm Hg, and heart rate is 62 bpm. Creatinine is 1.5 mg/dl. His low-density
lipoprotein is 65 mg/dl, high-density lipoprotein is 45 mg/dl, and triglycerides are 100 mg/dl.
Which of the following do you recommend?
a. Add nitrates.
b. Increase beta-blocker.
c. Proceed with surgery.
d. Coronary revascularization.
e. Full-dose aspirin.
132. c. Proceed with surgery.

This patient is at elevated risk for perioperative cardiac events for a moderate-risk surgery, but in
the face of appropriate medical therapy including aspirin and beta-blockade, there is no evidence
that revascularization will mitigate this risk. Results from the randomized, prospective CARP
(Coronary Artery Revascularization Prophylaxis) trial demonstrated that coronary artery
revascularization before elective major vascular surgery did not improve long-term survival or alter
early postoperative outcomes, including death, myocardial infarction, and length of the hospital
stay among patients with stable coronary artery disease. This patient’s heart rate suggests that he
is well beta-blocked, and you would not change the dose just prior to surgery. The patient should
proceed with surgery on current medical therapy. There is no evidence that an increased dose of
aspirin or nitrates will reduce perioperative events. The small amount of ischemia would not justify
revascularization.
133. A 47-year-old male smoker with hyperlipidemia calls for emergency medical services after
developing substernal chest pressure while watching television. Emergency medical technicians
arrive and administer sublingual nitroglycerin, which completely relieves his symptoms.

Upon arrival to the emergency department, the patient notes recurrent chest pressure with pain,
and his electrocardiogram shows 2.5 mm ST-segment elevation in V2-5. Both resolve with
administration of sublingual nitroglycerin. He is given aspirin, heparin, morphine, and intravenous
nitroglycerin. Coronary angiography reveals minimal nonobstructive plaque in the left anterior
descending artery and right coronary artery. Toxicology screening is negative.
In addition to smoking cessation, which of the following is the next most appropriate course of
action?
a. Metoprolol.
b. Niacin.
c. Omega-3 fatty acids.
d. Prazosin.
e. Diltiazem.
133. e. Diltiazem.

While the clinical findings suggest coronary spasm, the diagnosis is assumed, but not proven, as
spasm was not actually observed. Coronary vascular smooth muscle hyper-reactivity is a key factor
in the pathogenesis of coronary artery spasm. Cigarette smoking is a major risk factor for variant
angina and endothelial dysfunction; patients should be counseled to avoid tobacco use.

Diltiazem, a nondihydropyridine agent, is often first-line therapy for coronary spasm, but if
symptoms recur, spasm testing with intracoronary acetylcholine is indicated.

Treatment with either niacin or omega-3 fatty acids is not indicated here, as they have not been
shown to improve coronary spasm. Patients should not be treated with beta-1 and beta-2 selective
agents, because of the possibility of unopposed alpha-adrenergic activity. If beta-blockade is
needed for management, the new agents with vasodilator activity (nebivolol, carvedilol, etc.) would
be better choices.
134. A 62-year-old man comes to your office with 3 months of dyspnea on exertion. He has a
medical history of hypertension treated with a calcium channel blocker, hyperlipidemia treated
with atorvastatin, and long-standing asthma well controlled with theophylline. He also has been
complaining of severe knee pain that he mitigates with nonsteroidal anti-inflammatory drugs.

On exam, his blood pressure is 98/65 mm Hg. His cardiovascular exam is unremarkable and lungs
are clear to auscultation bilaterally. His electrocardiogram (ECG) shows sinus rhythm with first-
degree atrioventricular (AV) block.

You decide he needs a pharmacologic nuclear stress test.


Which of the following factors would contraindicate using adenosine as your stress of choice?
a. Use of calcium channel blocker.
b. Low normal blood pressure.
c. First-degree AV block on ECG.
d. History of asthma.
e. Theophylline use.
134. e. Theophylline use.

Adenosine is a vasodilator agent that works by producing stimulation of A2A receptors.


Methylxanthines (caffeine, theophylline, and theobromine) are competitive inhibitors of this effect,
which require withholding prior to testing. Second- or third-degree AV block is a contraindication;
first-degree is not. Systolic blood pressure <90 mm Hg is another contraindication.

Asthmatic patients with ongoing wheezing should not undergo adenosine stress testing. However,
it has been reported that patients with adequately controlled asthma can undergo an adenosine
stress test and can have pretreatment with two puffs of an inhaled bronchodilator. This patient had
clear lungs on auscultation. Another contraindication to adenosine is unstable acute myocardial
infarction or acute coronary syndrome. The patient was admitted and had negative cardiac
enzymes.
135. A 45-year-old man discharged from the hospital 6 weeks ago following an admission for
unstable angina during which he undergoes stenting of the right coronary artery presents to the
clinic. He reports no history of diabetes, smoking, hypertension, or a family history of premature
heart disease. Discharge daily medications include metoprolol succinate 25 mg, aspirin 81 mg,
atorvastatin 20 mg, and clopidogrel 75 mg. His body mass index is 27 kg/m2, waist circumference is
42 inches, and blood pressure is 135/85 mm Hg.

His laboratory results (on admission) were:


 Total cholesterol 230 mg/dl
 Triglycerides 350 mg/dl
 High-density lipoprotein 35 mg/dl
 Low-density lipoprotein 125 mg/dl
 Fasting glucose 99 mg/dl

Which of the following is the most appropriate next step for this patient?
a. Add fenofibrate.
b. Increase atorvastatin to 80 mg.
c. Continue current therapy.
d. Add ezetimibe.
135. b. Increase atorvastatin to 80 mg.

Current guidelines recommend that this patient with atherosclerotic cardiovascular disease should
be on a high-dose statin for maximal effect. He had elevated triglycerides on admission, but these
should be reassessed on a higher-dose statin and with lifestyle changes prior to addition of any
other pharmacologic therapy such as fenofibrate. This patient is not on a maximal dose of statin.
Therefore, addition of ezetimibe at this time is not appropriate.
136. A 43-year-old man is referred for dyslipidemia. He has no history of hypertension or diabetes.
His father had a myocardial infarction at the age of 38. The patient is a nonsmoker who exercises
regularly and eats a vegetarian diet.

Body mass index is 20 kg/m2, waist circumference is 30 inches, blood pressure is 110/80 mm Hg,
and heart rate is 48 bpm.

The patient’s internist provides the following lab results:


 Total cholesterol 395 mg/dl
 Low-density lipoprotein (LDL) 321 mg/dl
 High-density lipoprotein 54 mg/dl
 Triglycerides 100 mg/dl

Which of the following is the appropriate next step?


a. Exercise treadmill stress test.
b. Coronary artery calcium score.
c. Statin therapy.
d. Genetic testing.
136. c. Statin therapy.

For an initial evaluation of hyperlipidemia, testing for secondary causes of elevated cholesterol is
usually indicated, as disorders such as hypothyroidism, nephrotic syndrome, and medications can
be secondary causes of dyslipidemia and should be screened for too. In this patient who is likely to
have the heterozygous form of familial hypercholesterolemia, the most appropriate next step is
treatment with a statin.

Familial hypercholesterolemia is an autosomal-dominant disorder, caused by defective LDL-


receptors. Total cholesterols are usually in the 300-500 mg/dl range in many patients. Tendon
xanthomas on the Achilles tendon and extensor tendons of the hands are often present. These
patients carry a risk of premature coronary heart disease, as evidenced by the family history for
this patient. The prevalence is approximately 1:500 in the United States. This patient should be
started on lipid-lowering therapy along with continuation of therapeutic lifestyle changes, diet, and
physical activity. Furthermore, lipid testing is recommended for offspring.

Stress testing in this asymptomatic patient is not indicated. Calcium scoring does not change the
management in this patient, as he is already in one of the four statin treatment groups.
137. A 69-year-old woman sees you in the office for evaluation of fatigue with exertion for the past
6 months. She says that when she is climbing stairs, especially if she is carrying something, she is
more tired than usual. Symptoms resolve within 5 minutes of rest. She has hypertension, treated
with losartan and a diuretic. She reports numerous food allergies and she once had an episode of
wheezing and nasal congestion after taking aspirin.

You order a nuclear stress test, which reveals a small, mild reversible inferior perfusion abnormality
with a normal left ventricular ejection fraction.
Which of the following should be added to her medical regimen?
a. Prasugrel.
b. Cilostazol.
c. Dipyridamole.
d. Clopidogrel.
e. Ticlopidine.
137. d. Ticlopidine.

Ordinarily, aspirin (75-162 mg/day) would be recommended for antiplatelet therapy to prevent
myocardial infarction in patients with established atherosclerotic disease. However, this patient
has an indication of an aspirin allergy. Aspirin therapy is therefore contraindicated. Clopidogrel has
proven efficacy for prevention of ischemic events based on the CAPRIE trial, and therefore can be
used in place of aspirin.

Cilostazol is a phosphodiesterase-3 inhibitor that acts as both an arterial dilating agent and an
antiplatelet agent; it can increase symptom-free walking distance in patients with intermittent
claudication, but has no proven benefit in preventing ischemic coronary events. Dipyridamole in
combination with aspirin has been shown to reduce subsequent strokes and transient ischemic
attacks in patients with cerebrovascular disease, but not ischemic coronary events. Ticlopidine is an
adenosine diphosphate (ADP)-receptor inhibitor similar in mechanism of action to clopidogrel, but
has more serious side effects, including neutropenia and thrombotic thrombocytopenic purpura,
thus has been essentially replaced by clopidogrel. Prasugrel is an ADP-receptor inhibitor that is
approved for patients with acute coronary syndrome. This patient has chronic angina.
138. A 67-year-old man is seen in the cardiology clinic for a routine follow-up visit. He currently has
no symptoms of angina, dyspnea, orthopnea, or ankle swelling. He plays golf for exercise, saying
that he walks all 18 holes four times a week. He suffered an inferior wall myocardial infarction 4
years ago, which was treated with primary percutaneous coronary intervention. Nonobstructive
stenoses were noted in the left anterior descending artery at the time, but a total occlusion of the
right coronary artery was deemed to be the culprit lesion and received treatment with a drug-
eluting stent. He received medical therapy for lipids and hypertension as well as dual antiplatelet
therapy for 1 year following his myocardial infarction. An echocardiogram 1 year ago demonstrated
normal left ventricular systolic function. He is currently taking aspirin 81 mg daily,
hydrochlorothiazide 25 mg daily, atorvastatin 40 mg daily, and lisinopril 20 mg daily.

Which of the following interventions should you recommend at this time to reduce morbidity and
mortality for this patient?
a. Depression screening.
b. Cardiac rehabilitation.
c. Aldosterone antagonist therapy.
d. Influenza vaccination.
e. Beta-blocker therapy.
138. d. Influenza vaccination.

Vaccination against influenza has been shown to reduce morbidity and mortality in both patients
with acute coronary syndrome and in patients with prior cardiovascular events. Beta-blocker
therapy has not been studied in patients with a normal ejection fraction more than 3 years after a
coronary event. Depression screening may be important for quality of life, assuming supports to
treat depression are available, but has not been proven to reduce morbidity and mortality among
patients with coronary artery disease. Aldosterone antagonists are only proven to be beneficial in
patients with reduced ejection fraction following coronary events. Cardiac rehabilitation is
important following acute events and for the management of symptoms including angina, but
neither apply to this patient.
139. A 61-year-old woman presents for follow-up after a recent hospitalization for a non-ST-
segment elevation myocardial infarction (NSTEMI). She has type 2 diabetes, hypertension, and
stage 2 chronic kidney disease. Six weeks ago, she went to the emergency department with chest
pressure and underwent coronary angiography, which revealed a focal stenosis in the left anterior
descending coronary artery, for which she received a single drug-eluting stent. Her medications
include aspirin 81 mg daily, clopidogrel 75 mg daily, and simvastatin 20 mg daily, glargine insulin,
metoprolol succinate 25 mg qd, amlodipine 5 mg qd, and lisinopril 20 mg qd.

Today, she reports feeling well, has had no recurrent chest pressure or dyspnea, and is
participating in cardiac rehabilitation. Her blood pressure is 122/76 mm Hg and heart rate is 68
bpm. Her lipid profile shows total cholesterol of 210 mg/dl, low-density lipoprotein of 120 mg/dl,
high-density lipoprotein of 40 mg/dl, and triglycerides of 200 mg/dl.
Which of the following is the most appropriate next step, in addition to lifestyle modification?
a. Change simvastatin to atorvastatin 40 mg.
b. Add ezetimibe 10 mg daily.
c. Add fenofibrate 145 mg daily.
d. Add fish oil 3 g daily.
e. Increase simvastatin to 40 mg.
139. a. Change simvastatin to atorvastatin 40 mg.

This patient has established coronary artery disease and had a recent acute coronary syndrome.
High-intensity statin therapy should be continued in all patients with NSTEMI and with no
contraindications to its use (Class I, Level of Evidence A). Replacement of simvastatin with
atorvastatin 80 mg daily is supported by several large randomized trials. High-intensity statin
therapy used in randomized controlled trials includes atorvastatin 40-80 mg and rosuvastatin 20-40
mg. Adding ezetimibe, fenofibrate, fish oil, and niacin is not indicated, as the primary goal is high-
intensity statin therapy.
140. A 43-year-old man without any significant medical history presents for an office visit to
discuss prevention of cardiovascular disease (CVD). He is a nonsmoker. He takes no medications
and exercises 2-3 times per week at the local gym.

On physical examination, his weight is 220 lbs, blood pressure is 136/84 mm Hg, and pulse is 76
bpm. His body mass index is 33 kg/m2 and waist circumference is 40 inches. His physical
examination is otherwise unrevealing. Laboratory studies reveal a total cholesterol of 190 mg/dl,
low-density lipoprotein cholesterol of 130 mg/dl, high-density lipoprotein cholesterol of 39 mg/dl,
triglycerides of 209 mg/dl, and a fasting glucose of 110 mg/dl.

Which of the following is the most appropriate next step?


a. Start aspirin.
b. Recommend lifestyle modification.
c. Start statin.
d. Measure carotid intima-media thickness (CIMT).
e. Obtain exercise stress test.
140. b. Recommend lifestyle modification.

This patient's risk of atherosclerotic CVD (ASCVD) is low (<5%). Current CVD risk guidelines stress
the importance of lifestyle modifications to lower CVD risk. This includes eating a heart-healthy
diet, regular aerobic exercises, maintenance of desirable body weight, and avoidance of tobacco
products.

Due to his low risk, he would not benefit from statin or aspirin therapy. Further risk assessment
may be considered if a treatment decision is uncertain. Assessment of one or more of the
following–family history, high-sensitivity C-reactive protein, coronary artery calcium score, or ankle
brachial index—may be considered to inform treatment decision -aking (Class IIb, Level of Evidence
B). However, measurement of CIMT is not recommended for risk assessment for ASCVD (Class III,
Level of Evidence B). In a low risk, asymptomatic patient, stress testing is inappropriate.
141. A 67-year-old Spanish-speaking man from Mexico comes to the hospital for evaluation of
chest pain that occurred previously with exertion and now at rest. He has a history of hypertension,
hyperlipidemia, and diabetes. His admission electrocardiogram was notable for 1 mm ST
depression in the inferior leads. His initial troponin level was 0.2 ng/dl (normal <0.04 ng/dl). His
chest pain was relieved in the emergency room with sublingual nitroglycerin and morphine.

You believe that the next appropriate step is coronary angiography and would like to discuss the
risks/benefits of this invasive procedure with him in order to obtain consent. He speaks broken
English, but appears to understand much of what you say.
Which of the following is the most appropriate method to communicate this information to the
patient?
a. Having his bilingual daughter translate, who is with him now.
b. Giving him a copy of the consent form in Spanish to read on his own.
c. Referring him to your colleague who speaks Spanish and who will be on service tomorrow.
d. Explaining in English and having him nod to ensure understanding.
e. Requesting and waiting for a hospital interpreter in person or by phone
141. e. Requesting and waiting for a hospital interpreter in person or by phone

In cases of patients whose English skills are limited, an official medical interpreter is the best choice
for conveying accurate medical information, especially when it concerns an invasive procedure with
risks of serious complications. A family member is generally only acceptable in emergencies or if
the patient specifically requests him or her to interpret for a small matter. Minors (<18 years old)
should be avoided, and the age of the patient's daughter is unknown. Relying on nodding to convey
understanding of the nature of a complex procedure is not sufficient in this patient with limited
English skills. Reading and signing the consent form alone is not adequate for obtaining informed
consent and provides no opportunity for the patient to ask questions. Delaying care of a non-ST-
elevation myocardial infarction by a day or more to involve your colleague is providing suboptimal
care and not an acceptable option.
142. A 68-year-old man with a history of ongoing tobacco use, hypertension, and a 4.2 cm
abdominal aortic aneurysm (AAA) is seen for scheduled follow-up. His AAA has been followed with
yearly ultrasounds and is stable in size. He is retired and walks for 30 minutes each morning with
his dog. Medications include lisinopril, aspirin, and a multivitamin.

On examination, his blood pressure is 128/82 mm Hg and pulse is 88 bpm. There is a soft
abdominal bruit and an accentuated abdominal aortic pulsation. A fasting lipid profile reveals total
cholesterol of 220 mg/dl, high-density lipoprotein (HDL) of 31 mg/dl, and triglycerides of 316
mg/dl.
Which of the following is the most appropriate recommendation?
a. Losartan.
b. Fenofibrate.
c. Atorvastatin.
d. Fish oil.
e. Niacin.
142. c. Atorvastatin.

This patient presents with dyslipidemia with elevated total cholesterol and triglycerides and low
HDL. The first step in management of dyslipidemia includes assessing patient risk. This patient’s
AAA places him in the highest risk category for future cardiac events. Other coronary risk
equivalents include the presence of peripheral arterial disease, diabetes, or cerebrovascular
disease.

Given his high risk, moderate- to high-intensity statin therapy is recommended. Fenofibrate is
effective at lowering triglycerides. However, in this patient, the non-HDL cholesterol should be
treated first. If triglycerides remain elevated despite non-HDL cholesterol at goal, therapy directed
toward hypertriglyceridemia is reasonable, although concurrent fibrate and statin therapy increase
the risk of statin-induced myopathy. Fish oil can also be useful in management of
hypertriglyceridemia, but should not be used as the initial therapy in this case. Niacin is effective at
raising HDL. However, the incremental value of raising low HDL with pharmacotherapy has not
been proven in several large randomized trials, and is an area of ongoing study and debate. Finally,
addition of a second antihypertensive agent is not indicated, as his blood pressure is well controlled
to <130/80 mm Hg.
143. A 48-year-old man presents for follow-up. He feels well and recently completed his first 5k
road race. His medical history includes dyslipidemia managed with atorvastatin. Physical
examination is normal. He obtained laboratory work at a recent “employee health fair,” which he
brings for review. The total cholesterol is 160 mg/dl, high-density lipoprotein cholesterol is 48
mg/dl, and triglycerides are 102 mg/dl. The high-sensitivity C-reactive protein level is within the
normal range. Serum chemistries including creatinine are normal. Dipstick urinalysis is negative for
blood, protein, and glucose. Serum creatine kinase (CK) level is 301 U/L (reference range, 26-192).
Liver function tests are normal.
Which of the following is the best next step in management of this patient?
a. Decrease frequency of atorvastatin to every-other-day dosing.
b. Decrease the dose of atorvastatin and add ezetimibe.
c. Stop atorvastatin and prescribe cholestyramine.
d. Stop atorvastatin and prescribe rosuvastatin.
e. Continue atorvastatin and check CK in 3 months.
143. e. Continue atorvastatin and check CK in 3 months.

This patient is asymptomatic and presents with a slightly increased CK level while on statin therapy.
Guidelines recommend checking a single CK level upon initiation of statin therapy to obtain a
baseline value for comparison, should myopathic symptoms develop. CK should be rechecked with
any new muscle symptoms, but routine monitoring of CK in asymptomatic patients is not
recommended. Statin therapy should be stopped in patients with severe myopathic symptoms
regardless of CK level and if the CK is greater than 10-times the upper limit of normal. For this
asymptomatic patient with a slightly elevated CK, no change in therapy is needed and he can be
followed serially. The elevations in CK are exacerbated by his training/running.

Cholestyramine can be used in lieu of statin therapy in statin-intolerant patients, as there are data
for an improvement in mortality in the primary prevention setting. Rosuvastatin and pravastatin
are both more hydrophilic than atorvastatin, and in some patients with statin myalgia (not
myopathy), transition to a more hydrophilic statin can result in symptom improvement. Similarly,
for statin-intolerant patients, an every other day dosing regimen can allow for some statin therapy.
A combination of simvastatin and ezetimibe was not superior to simvastatin alone in a randomized
controlled trial, and no change in therapy is indicated for this patient.
144. A 55-year-old woman with stable anginal chest pain on no cardiac medication underwent a
computed tomography (CT) coronary angiogram to assess for possible coronary artery disease
(Figure 1). Her electrocardiogram (ECG) showed no acute ischemic changes and is normal. Her
blood pressure is normal and she has no diabetes.
Based on the CT image shown in Figure 2, which of the following is the next best step?

Figure 2
a. An exercise ECG stress test.
b. Cardiac catheterization.
c. Begin medical therapy for coronary disease.
d. An exercise nuclear stress test.
144. c. Begin medical therapy for coronary disease.

The CT suggests a mid left anterior descending artery lesion and single-vessel disease. She needs
initial treatment with medical therapy for stable angina before further testing is indicated.
145. A 58-year-old man presents to the emergency department with a 5-hour history of crushing, substernal
chest pain. The patient history is significant for type 2 diabetes, hypertension, dyslipidemia, and coronary artery
disease with stenting of his proximal left anterior descending artery 3 years ago.

Admission electrocardiogram shows anterior ST-segment elevation, including ST elevation in lead aVR. The
patient is referred for emergent cardiac catheterization. Initial coronary angiography shows a 95% ostial left main
stenosis with thrombus. An intra-aortic balloon pump (IABP) is inserted and successful stenting of the left main is
performed with 0% residual stenosis and TIMI 3 flow throughout the left coronary system. The right coronary
artery is widely patent. The IABP is continued at a 1:1 counterpulsation ratio.

One hour after the procedure, the patient develops atrial fibrillation with a rapid ventricular response. The
patient remains asymptomatic. Hemodynamic variables after percutaneous coronary intervention (PCI) are
shown in the table in Figure 1.
Which of the following choices would be the best next step in the management of this patient?
a. Add flecainide.
b. Repeat coronary angiography.
c. Add amiodarone.
d. Insert an Impella 5.0 left percutaneous device.
e. Add dobutamine.
145. c. Add amiodarone.

The IABP is a catheter-mounted balloon that augments pulsatile blood flow by inflating during
diastole, which displaces blood volume in the descending aorta and increases mean aortic
pressure, thereby augmenting coronary perfusion. Upon deflation, during systole, the IABP
generates a pressure sink, which is filled by ejecting blood from the heart. The net effect of IABP
therapy is an increase in mean arterial pressure and augmented ventricular stroke volume. For
this reason, proper IABP function depends on native contractile function of the heart. During
periods of tachycardia, the IABP is unable to adequately inflate, and therefore cannot augment
mean arterial pressure.

Thus, pharmacological management of reducing the heart rate is a reasonable next step to
optimize IABP function and stabilize the patient’s hemodynamics. Escalating mechanical support
to an Impella, which requires surgical vascular access, is not required. There is no indication that
the patient has developed recurrent thrombosis of the left main artery. Initiation of inotropes
and vasopressors may worsen the tachycardia and lead to further decompensation. Direct
current cardioversion may be appropriate but is not listed as an option. Flecainide is
contraindicated in patients with coronary artery disease.
146. A 65-year-old woman was admitted with an anterior ST-segment elevation myocardial
infarction (STEMI). She was taken urgently to the catheterization laboratory, where she underwent
placement of two bare-metal stents to the proximal and mid-left anterior descending coronary
artery. Forty-eight hours later, she complains of shortness of breath and chest pain of a different
quality than her presenting symptoms. On examination, her blood pressure is 80/57 mm Hg, heart
rate is 102 bpm, and oxygen saturation is 94% on room air. Her jugular venous pressure (JVP) is
elevated to the angle of the jaw. Lung exam is clear.
What is the most likely finding on cardiovascular examination?
a. Loud pansystolic murmur.
b. Muffled heart sounds.
c. Displaced point of maximal impulse.
d. Loud S4.
146. a. Loud pansystolic murmur.

The patient is expected to have muffled heart sounds secondary to free wall rupture and increased
pericardial fluid. This diagnosis is consistent with elevated JVP and clear lungs, and occurs more
frequently in the setting of an anterior wall MI. It is considered a surgical emergency with a high
mortality.

A loud pansystolic murmur at the lower left sternal border, accompanied by a thrill, is consistent
with ventricular septal defect and is usually accompanied by pulmonary congestion. The treatment
is urgent surgery. There is a higher mortality in patients with inferior MI compared with those with
anterior MI. A fourth heart sound may be heard in the setting of an acute MI, but given signs
consistent with pericardial effusion, this is likely not to be audible. A displaced apex is most
consistent with chronic systolic heart failure.
147. A 57-year-old man has had 2 years of progressively worsening angina with exertion. He was
initially managed with medical therapy, but has had progressively worsening angina despite
maximization of two antianginal therapies. A stress test showed moderate exercise tolerance, and
nuclear imaging suggested a large area of inferior ischemia involving >10% of the left ventricular
muscle mass. A diagnostic cardiac catheterization was performed and showed a chronic total
occlusion of the right coronary artery, as shown in Figure 1.

Given his lifestyle-limiting symptoms despite maximal medical therapy, you have referred him for
percutaneous coronary intervention (PCI). You have obtained careful consent and have advised him
of the risk of radiation-induced skin injury. You have assured him that you will take every
precaution to minimize the radiation exposure to the patient during what may be a lengthy
procedure.

Which of the following techniques will increase radiation exposure to the patient?
Figure 1
a. Use of less magnified views.
b. Use of fluoroscopy storage instead of cine storage.
c. Use of collimation.
d. Steep imaging intensifier angles.
e. Use of fluoroscopy frame rate of 7.5 frames per second (FPS).
147. d. Steep imaging intensifier angles.

Patients with cardiovascular disease are exposed to a wide array of diagnostic and therapeutic procedures that
result in a significant cumulative exposure of ionizing radiation. There is a growing awareness of this cumulative
exposure, and every effort should be made in the catheterization laboratory to minimize the radiation exposure
to the patient (and the operator). The majority of PCI procedures can be completed, as most modern imaging
equipment in the catheterization laboratory has a fluoroscopy storage function where a maneuver performed
under fluoroscopy can be permanently stored and recorded. In many cases, the fluoro image will provide
diagnostic grade imaging so that cine-angiography will not be necessary and further radiation exposure can be
reduced. On average, cine radiation doses can be as much as 20-times higher than fluoroscopy doses.

Reducing the frame rate for image acquisition will also significantly reduce radiation exposure during fluoroscopy.
While many operators prefer using 15 FPS, a frame rate of 7.5 FPS is sufficient in most patients for effective
imaging and will reduce radiation use by 50% compared to 15 FPS. Collimation and use of lower magnified
imaging fields are also critical for reduction of radiation exposure.

Use of steep imaging angles will increase the radiation exposure to the patient. With steep angulation, fewer X-
rays reach the image intensifier, resulting in feedback to the radiation generator to generate greater energy in
order to provide a sufficient image as the source-to-image distance is increased. Every effort should be made to
use angles as shallow as possible during PCI, and to use multiple angles so as not to concentrate the radiation
beam to one area.
148. A 76-year-old man with atrial fibrillation presents to clinic with a 2-year history of chest pain
at rest located in the mid-epigastrium radiating to the mid-substernal region. Computed
tomography (CT) angiography of the chest reveals the finding shown in Figure 1.

Which of the following is the abnormality shown in Figure 1?


Figure 1

a. Primary cardiac tumor.


b. Coronary artery aneurysm.
c. Pericardial cyst.
d. Atrial appendage thrombus.
148. b. Coronary artery aneurysm.

The abnormality shown on the CT image has peripheral calcifications with central contrast and
surrounding soft tissue density. It is continuous with the atherosclerotic coronary artery and
consistent with coronary artery aneurysm, rather than a pericardial cyst or primary cardiac tumor.
Neither of these would be expected to have peripheral calcification. Pericardial cysts are generally
benign and usually right-sided, often in the atrioventricular groove.

The abnormality is anterior to the left atrium and not consistent with an atrial appendage
thrombus.
149. A 75-year-old man is found by his family, unresponsive and crumpled over in the bathroom.
Emergency medical service is called, but no bystander cardiopulmonary resuscitation (CPR) is
attempted. Upon arrival 7 minutes later, emergency medical service personnel begin CPR, and after
2 minutes the initial rhythm is ventricular fibrillation. After a 20-minute period of CPR effort,
including epinephrine and repeated shocks, spontaneous circulation is established. He is intubated.
Upon arrival at the emergency department, the patient has a weak pulse and blood pressure of
95/65 mm Hg. He remains unresponsive, without spontaneous movement. The monitor reveals a
wide QRS complex tachycardia at 100 bpm.
Which of the following is the next most appropriate postresuscitation therapy?
a. Discuss with the family withdrawal of therapy.
b. Therapeutic hypothermia.
c. Electroencephalogram.
d. Cardioversion for wide QRS tachycardia.
149. b. Therapeutic hypothermia.

This patient’s prognosis is poor but not futile. Since he remains comatose, hypothermia is the best
chance to limit long-term neurological damage. Long-term survival can be improved with
hypothermia, generally from previous improvement rates of 1-2% to an improvement rate of 5-
20%. Recent data suggest this may not be the case in children. It is important to realize that in
order to achieve these improved survival rates, adequate time after rewarming must be allowed
before reliable prognostication can be done and further efforts terminated. There is no need for
cardioversion of his postresuscitation wide QRS rhythm since it is providing a current acceptable
blood pressure and most certainly represents a transient epinephrine/norepinephrine response. An
electroencephalogram may be important later to help assess his neurologic status.
150. A 42-year-old man with a long-standing human immunodeficiency virus (HIV) infection comes
to your office out of concern of his cardiovascular risk. He is on antiretroviral drugs. He smokes half
a pack per day and drinks 1-2 drinks per week. Pertinent routine tests show a blood pressure of
127/88 mm Hg, low-density lipoprotein 177 mg/dl, high-density lipoprotein 27 mg/dl, and
triglycerides 334 mg/dl.
Which of the following is an appropriate therapy for this patient, in addition to smoking cessation?
a. Ezetimibe 10 mg a day.
b. Begin 2 g fish oil daily.
c. Rosuvastatin 20 mg a day.
d. Atorvastatin 10 mg a day.
e. Fenofibrate 150 mg a day.
150. d. Atorvastatin 10 mg a day.

HIV-infected individuals have high rates of smoking, and smoking cessation is perhaps the most
modifiable risk factor to minimize cardiovascular disease risk in these persons. While this patient
requires antiretroviral drugs and needs to improve his lipid profile, smoking cessation would have
the most immediate effect on his overall health and cardiovascular disease risk. He warrants
therapy for his hyperlipidemia. Because of the interaction with antiretroviral drugs, a low-dose
statin is recommended.
151. A 57-year-old man with a history of hypertension, hyperlipidemia, and a prior stent to the
right coronary artery is seen for routine follow-up. Although he does not exercise regularly, he
denies anginal symptoms during daily activities. He drinks one glass of wine daily and does not
smoke. His medications are aspirin 81 mg qd, metoprolol succinate 25 mg qd, lisinopril 10 mg qd,
and atorvastatin 40 mg qd. His blood pressure is 112/75 mm Hg. His most recent lipid panel shows
a low-density lipoprotein (LDL) of 68 mg/dl and high-density lipoprotein (HDL) of 25 mg/dl. He
wants to know how to best improve his cardiovascular risk profile.

Which of the following is the most appropriate recommendation?


a. Add colesevelam 3750 mg qd.
b. Start a regular aerobic exercise program.
c. Add niacin 500 mg bid.
d. Increase daily alcohol intake to 3-4 drinks.
e. Change to simvastatin 80 mg daily.
151. b. Start a regular aerobic exercise program.

Abundant epidemiologic evidence supports the relationship between increased physical activity
and reduced cardiovascular risk. Currently, the best advice for patients is to perform regular
aerobic exercise, which has beneficial effects on cardiovascular health. While niacin is effective at
raising HDL levels, recent trials have shown that the addition of niacin fails to reduces
cardiovascular events in patients with well-controlled LDL levels on background statin therapy.

The AIM-HIGH trial tested the effects of adding niacin for patients with low HDL levels already on a
statin. Despite increasing HDL levels, there was no benefit with niacin for reducing cardiovascular
events and there was a question of a signal of increased stroke.

The HPS2-THRIVE trial also examined the effects of niacin/laropiprant in patients with well-
controlled LDL levels on a statin. There was no difference in rates of cardiovascular events, and
niacin treatment was associated with more side effects. Given these results, it is not clear that
niacin provides any additional benefit in a patient such as this who already is on a statin and at
target LDL.

Initiation of simvastatin at 80 mg or uptitration to 80 mg dose is contraindicated in patients due to


risks of muscle injury. Colesevelam is a bile acid sequestrant that has primarily LDL-lowering effects.
While moderate alcohol intake (especially red wine) may have beneficial cardiovascular effects,
consuming more than 1-2 drinks daily has been shown to have detrimental effects on health.
152. A 45-year-old man presents with substernal chest discomfort that comes on predictably with
walking two blocks. The discomfort is alleviated with rest. He has a long-standing history of
hypertension and was diagnosed with diabetes and dyslipidemia 6 months earlier. Current
medications include metoprolol, lisinopril, simvastatin, and baby aspirin.

A stress myocardial perfusion imaging study is ordered. He is unable to complete stage 2 of the
standard Bruce protocol due to recurrence of chest discomfort. At peak stress and 2 minutes into
recovery, 1.5-2 mm ST-segment depression is noted in the anterolateral leads. His imaging study is
inconclusive due to submaximal workload.

The most appropriate next test that also minimizes radiation exposure would be which of the
following?
a. Pharmacologic myocardial perfusion imaging.
b. Diagnostic cardiac catheterization.
c. Coronary computed tomography (CT) angiography.
d. Dobutamine stress echocardiography.
e. Stress echocardiography.
152. b. Diagnostic cardiac catheterization.

This patient has a high pretest likelihood of coronary artery disease based on his risk factors and
characteristics of typical angina. Electrocardiographic changes during peak stress are suggestive of
inducible ischemia, even though imaging was “inconclusive.” Risk-benefit ratio must be taken into
account with diagnostic imaging studies. Although repeating the stress test would probably confirm
the clinical suspicion, he will still need further evaluation of the coronary arteries.

Coronary CT angiography is an excellent tool in patients with equivocal or nondiagnostic stress


tests. However, in patients with a high pretest likelihood of coronary artery disease, coronary CT
angiography is considered an inappropriate test, without added value of layered testing. Among
the choices provided, triage to diagnostic cardiac catheterization with the option for
revascularization based on anatomy would be the most appropriate decision in order to avoid
additional radiation exposure.
153. A 48-year-old man with a history of hypertension comes to your clinic to establish care. He
states that he was told a few years ago that he had high cholesterol but he did not take further
action. He does not smoke and drinks alcohol rarely. He states he eats a balanced diet with limited
carbohydrates. His only medication is amlodipine. His blood pressure is 129/81 mm Hg, heart rate
is 76 bpm, and the remainder of his exam is unremarkable. His body mass index is 24.8 and his
glycated hemoglobin is normal.

A fasting lipid panel from this morning reveals a total cholesterol of 210 mg/dl, triglycerides 630
mg/dl, high-density lipoprotein (HDL) 42 mg/dl. His low-density lipoprotein (LDL) cannot be
calculated. His thyroid-stimulating hormone is normal.
In addition to recommending therapeutic lifestyle changes, which of the following is the next most
appropriate step in his care?

a. Recheck lipids in 3 months.


b. Start low-dose aspirin.
c. Switch amlodipine to hydrochlorothiazide.
d. Start atorvastatin.
e. Start fenofibrate.
153. e. Start fenofibrate.

The patient has very high triglyceride level, as defined by a level >500 mg/dl in the Adult Treatment
Panel III guidelines. Such a severe elevation in triglycerides can lead to pancreatitis, so priority
should be made to reduce levels as quickly as possible with effective triglyceride-lowering
medications. Appropriate first-line choices include fibrates and niacin. Statins are less efficacious on
this front, and would only be considered for LDL-lowering purposes once triglyceride levels are
below 500 mg/dl.

Therapeutic lifestyle changes should be recommended, but are not adequate to prevent
pancreatitis in such a setting. Certain cardiovascular medications, such as hydrochlorothiazide, can
increase triglyceride levels, thus would be detrimental in this case. Amlodipine has no such effects.
The calculated LDL value in a lipid panel will be falsely low in the setting of elevated triglycerides,
thus cannot be properly interpreted. Alternative approaches are repeating the lipid panel when
triglycerides are better-controlled, measuring direct LDL, or following a non-HDL cholesterol
treatment goal.
154. A 50-year-old man with hypertension comes to your office for consultation on management
of his cardiovascular risk. He has no symptoms suggestive of cardiovascular disease. His
hypertension has been treated with hydrochlorothiazide and lisinopril, and his treated resting
blood pressure is 130/80 mm Hg. His fasting lipid profile reveals a total cholesterol of 170 mg/dl,
low-density lipoprotein (LDL) of 110 mg/dl, high-density lipoprotein of 45 mg/dl, and triglycerides
of 75 mg/dl. His fasting glucose is 90 mg/dl. He does not smoke cigarettes. He has no family history
of early-onset atherosclerotic cardiovascular disease (ASCVD).

Which of the following therapeutic options is supported by the risk profile?


a. Omega-3 fatty acids.
b. Statin therapy.
c. No lipid-lowering medications.
d. Nicotinic acid.
e. Fibrate therapy.
154. c. No lipid-lowering medications.

The 2013 American College of Cardiology (ACC)/American Heart Association (AHA) cholesterol
guideline identified four major statin benefit groups. Using the 2013 ACC/AHA algorithm, this
patient does not have clinical ASCVD, LDL ≥190 mg/dl, or diabetes. Using the Pooled Cohort
Equation, his 10-year risk of coronary death, nonfatal myocardial infarction, fatal stroke, and
nonfatal stroke is 3.7%. This risk level falls below the 7.5% threshold at which the benefit of statin
therapy significantly exceeds the risk of incident diabetes. While further risk assessment may be
considered, such as coronary artery calcium scoring, based on the available risk profile, no lipid-
lowering medications are recommended. There is no randomized clinical trial evidence that
initiation of a fibrate, niacin, or fatty acids would be beneficial in primary prevention in this patient.
155. A 65-year-old woman with hypertension comes to your office to seek consultation on
management of her cardiovascular risk factors. She has no history or symptoms suggestive of
cardiovascular disease. Her hypertension has been treated with hydrochlorothiazide and lisinopril
and her resting blood pressure is 130/80 mm Hg. On fasting laboratories, she has a total cholesterol
of 190 mg/dl, low-density lipoprotein (LDL) of 120 mg/dl, high-density lipoprotein (HDL) of 40
mg/dl, triglycerides of 150 mg/dl, and glucose of 90 mg/dl. She currently smokes half a pack per
day of cigarettes.
After counseling her on smoking cessation and discussing the risks and benefits of testing, you
recommend that she consider which of the following?
a. Obtain an exercise stress test.
b. Measure high-sensitivity C-reactive protein (hs-CRP) level.
c. Start moderate- to high-intensity statin therapy.
d. Measure glycated hemoglobin (HbA1c) level.
155. c. Start moderate- to high-intensity statin therapy.

The 2013 American College of Cardiology (ACC)/American Heart Association (AHA) cholesterol
guideline identified four major statin benefit groups. Using the 2013 ACC/AHA algorithm, this
patient does not have clinical atherosclerotic cardiovascular disease, does not have familial
hyperlipidemia with an LDL above 190, and does not have diabetes. She is between the age of 40
and 75 years, and therefore, the Pooled Cohort Equation should be used to estimate her 10-year
atherosclerotic risk. This calculated to a 10-year atherosclerotic risk of 15.4%, well above the
recommended 7.5% risk threshold for initiating statin therapy, and therefore, additional testing
with hs-CRP is not needed.

She has already had appropriate screening for diabetes with a fasting glucose level, therefore,
HbA1c is not necessary. The patient has no symptoms suggestive of ischemic heart disease, and
therefore, stress testing in this asymptomatic patient is not appropriate.
156. A 55-year-old woman is seen for a second opinion on her lipid management. She has a recent
diagnosis of non–insulin-dependent diabetes and no known history of cardiovascular disease. At
the time of the diagnosis of her diabetes 6 months ago, her fasting lipid profile revealed a total
cholesterol of 200 mg/dl, low-density lipoprotein (LDL) of 110 mg/dl, high-density lipoprotein of 45
mg/dl, and triglycerides of 225 mg/dl. Although pharmacotherapy for her lipids was recommended
by her primary care physician, she opted for lifestyle changes first.

After 6 months of therapy for her diabetes and diet modification, her lipid profile is unchanged. Her
body mass index is 28 kg/m2 and her glycated hemoglobin is 6.9%. She has a blood pressure of
135/80 mm Hg and does not smoke. No proteinuria noted on urinalysis.
In addition to continued lifestyle modification with regular exercise, diet, and modest weight loss
for a body mass index goal <25, which of the following do you recommend as the next step for this
patient?
a. Amlodipine 5 mg.
b. Atorvastatin 20 mg.
c. Niacian ER 500 mg bid.
d. Fenofibrate 145 mg.
e. No change in medications at this time.
156. b. Atorvastatin 20 mg.

The 2013 American College of Cardiology (ACC)/American Heart Association (AHA) cholesterol
guideline identified four major statin benefit groups. Using the 2013 ACC/AHA algorithm, this
patient has diabetes with a Pooled Cohort Equation estimated 10-year atherosclerotic risk of 4.4%.
As per the 2013 ACC/AHA algorithm, in diabetics (40-75 years, LDL 70-189 mg/dl), with a 10-year
atherosclerotic cardiovascular disease risk <7.5%, a moderate-intensity statin should be initiated.

This patient's blood pressure is <140 mm Hg, and antihypertensive medication is not indicated. The
2013 guideline did not find data supporting the routine use of nonstatin drugs combined with
statin therapy to reduce further atherosclerotic cardiovascular events. Because her risk is <7.5%, a
high-intensity statin is not indicated.
157. A 56-year-old African American woman with a history of hypertension treated with
hydrochlorothiazide, and a family history of premature coronary disease presents to your clinic.
She is a current smoker. Her blood pressure is 140/70 mm Hg, heart rate is 55 bpm, and body mass
index is 28.9.

Her lipid profile after a 10-week very low fat diet with increased physical activity is as follows:
 Total cholesterol 240 mg/dl
 Triglycerides 450 mg/dl
 High-density lipoprotein (HDL) 50 mg/dl
 Low-density lipoprotein (LDL) 145 mg/dl

Which of the following is the first therapy to initiate?


a. Omega-3 fatty acids.
b. Fibrate.
c. Statin.
d. Nicotinic acid.
157. c. Statin.

The 2013 American College of Cardiology (ACC)/American Heart Association (AHA) cholesterol
guideline identified four major statin benefit groups. Using the 2013 ACC/AHA algorithm, this
patient does not have clinical atherosclerotic cardiovascular disease, does not have familial
hyperlipidemia with an LDL above 190 mg/dl, and does not have diabetes. She is between the age
of 40 and 75 years, and therefore, the Pooled Cohort Equation should be used to estimate her 10-
year atherosclerotic risk. This calculated to a 10-year atherosclerotic risk of 10.6%, and is probably
even higher given her family history of premature coronary artery disease, thus warrants treatment
with a statin to reduce this risk.
158. Effective dose-reduction strategies include appropriate patient selection.
For which of the following patients is coronary computed tomography angiography (CCTA)
appropriate?
a. A 68-year-old man with high risk for coronary disease with acute chest pain.
b. A 50-year-old man unable to undergo risk assessment.
c. An asymptomatic 59-year-old man with a history of coronary artery bypass grafting surgery 2
years ago.
d. A 51-year-old woman with low risk for coronary disease and typical angina.
e. A 45-year-old male executive with intermediate risk for coronary disease and worsening chest
pain with activity.
158. d. A 51-year-old woman with low risk for coronary disease and typical angina.

Appropriate patient selection is an important first step in dose reduction. In general, due to its high
sensitivity and negative predictive value, CCTA performs well in ruling out coronary artery disease
in low-to-intermediate risk individuals. In patients with a high likelihood of coronary disease
needing triage to invasive assessment, CCTA results in unnecessary radiation exposure with little
added value. In such patients, CCTA is considered inappropriate.
159. A 53-year-old woman presents to the heart failure clinic for evaluation and management. She experiences
angina on minimal exertion despite medical therapy. She had an implantable cardioverter-defibrillator (ICD)
inserted 3 months after her most recent myocardial infarction (MI). She had suffered two prior MIs and had a
percutaneous coronary intervention (PCI) of the left anterior descending (LAD) and right coronary artery. Her
most recent ejection fraction is 25% with a dilated left ventricle and no valvular abnormalities. Her
electrocardiogram shows normal sinus rhythm with a QRS of 110 msec and Q waves in the inferior leads and
precordial leads.

A coronary angiogram shows patent right and left circumflex coronary arteries with a subtotal highly calcified
occlusion of the proximal LAD. The distal LAD is collateralized from the right and left circumflex coronary arteries.

PCI was not technically feasible. She undergoes rest/4-hour redistribution thallium. Results show 60%
redistribution in the distal half of the anterior wall and septum.
Which of the following is the next best step in this patient's care?

a. Surgical revascularization.
b. Biventricular ICD.
c. Intensify medical treatment.
d. Transplant evaluation.
e. Cardiac magnetic resonance imaging (MRI).
159. a. Surgical revascularization.

This patient is presenting with ischemic cardiomyopathy and New York Heart Association class III-IV
heart failure. She has advanced left ventricular dysfunction. On the single-photon emission
computed tomography images, there is evidence of thallium uptake at 4 hours in her LAD
distribution. Therefore, this area may improve with revascularization.

In a meta-analysis comparing all imaging modalities that can identify hibernating myocardium and
predicting recovery of regional function after revascularization, positron emission tomography
(PET) using fluorodeoxyglucose (FDG) was the most sensitive, while dobutamine was the most
specific, but when global recovery after revascularization was analyzed, thallium and PET were very
similar.

She does not meet criteria for upgrading to a biventricular ICD based on her QRS. An MRI would
not provide further information and is contraindicated in patients with an ICD. If she does not
improve with revascularization, then cardiac transplant should be considered.

Treatment of triglycerides and non-HDL is secondary to initiating a statin, and there has been no
evidence to support treatment with a fibrate or nicotinic acid to optimize non-HDL levels.
160. A 68-year-old woman with hypertension and diabetes is being referred to the nuclear
laboratory for a myocardial perfusion study. The nurse asks you to choose the imaging protocol for
the patient. The patient’s body mass index (BMI) is 38 kg/m2. The patient walks her dog routinely.
The patient lives close to the clinic where you would perform the test and is retired.

Which of the following protocols would optimize patient images?


a. One-day low-dose rest/high-dose stress with technetium.
b. Two-day high-dose rest/stress imaging with technetium.
c. One-day rest/stress dual-isotope imaging.
d. One-day low-dose stress/high-dose rest with technetium.
e. One-day stress/redistribution/reinjection with thallium.
160. b. Two-day high-dose rest/stress imaging with technetium.

Ideally, stress and rest imaging with technetium-99 agents should be performed on two separate
days to avoid having residual activity from the first study contaminating the second study in
overweight patients (i.e., >250 lbs. or body mass index >30 kg/m2) or in women where significant
breast attenuation is anticipated. A low dose of technetium-99 radiotracer may result in
suboptimal images and a 2-day imaging protocol is preferred in this patient population.

The effective dose of radiation is higher with dual-isotope and should be avoided. The rest-stress
protocol is a good option if the patient could not have a 2-day protocol. The patient does not
require an assessment for myocardial viability. The low-dose stress should not be performed in this
population because of the potential poor quality images on the stress study.
Coronary Artery Disease ( ACCSAP-9 2016 )

1.

A 19-year-old African-American male college soccer player is referred to see you in clinic for

screening prior to participation in collegiate athletics. He has a history of childhood asthma, but is
otherwise healthy and takes no medications. There is no family history of cardiomyopathy,

coronary artery disease, or sudden cardiac death. His physical examination is normal. He hands you
an electrocardiogram (ECG) that his primary care physician performed.

Which of the following findings, if present, would be of most concern?

a. Left ventricular hypertrophy (LVH) by the limb-lead voltage criteria (R in lead aVL of 13 mm).

b. 0.5 mm ST depressions in leads I and aVL.

c. Wenckebach pattern (Mobitz I second-degree atrioventricular [AV] block).

d. Domed/convex upsloping ST elevation with T-wave inversions in V2-V3.

e. Right-axis deviation of QRS complex to 105 degrees.

1.
b.
0.5 mm ST depressions in leads I and aVL

Elite athletes may demonstrate ECG changes that overlap with findings seen with cardiomyopathy.

Several criteria, including the European Society of Cardiology criteria, Seattle criteria, and recently
published “refined criteria” aim to distinguish between normal ECG changes seen with athletic

adaptation versus those that may be associated with increased risk for sudden death. Importantly, ECG
changes may be more common in athletes of African/Caribbean ancestry. Right-axis deviations

of the QRS complex up to 115 degrees, type 1 second-degree AV block, and LVH by voltage criteria

alone (without evidence of axis changes, repolarization changes, atrial enlargement, or increased
QRS width) can be seen in healthy athletes. In addition, healthy black athletes may have convex ST-
segment elevation associated with T-wave inversions in leads V1-V4. However, ST-segment

depression ≥0.5 mm in two or more leads is considered an abnormal finding.

2.

A 74-year-old man with hypertension and hyperlipidemia presents to your office with chest

discomfort on exertion. Exercise nuclear stress test demonstrates minimal inferior ischemia and

transient ischemic dilatation, with a left ventricular ejection fraction (LVEF) of 38%. Coronary

angiography reveals a 60% proximal left anterior descending artery (LAD) stenosis, 90% left

circumflex stenosis, and 70% mid right coronary artery stenosis. Fractional flow reserve (FFR)

evaluation of the LAD lesion is 0.72.

Which of the following is the most appropriate next step in his treatment?

a. Coronary artery bypass surgery.

b. Percutaneous coronary intervention (PCI) of the angiographically significant lesions.

c. LAD PCI and medical management of the remaining coronary artery disease.

d. Metoprolol and sublingual nitroglycerin.

e. Increase atorvastatin to 80 mg daily.

2.

a.
Coronary artery bypass surgery
This patient presents with multivessel coronary artery disease on coronary angiography including

the proximal LAD, based on an FFR <0.8. Regardless of stable anginal symptoms, surgical

revascularization of severe left main disease or proximal LAD with multivessel disease, in the

setting of reduced LVEF, has a proven mortality benefit and is recommended. The preferred

method would be surgical revascularization in the absence of contraindications. Optimal medical

therapy should be initiated in conjunction with revascularization, not as stand-alone therapy.

3.

A 56-year-old man with a history of hypertension and hyperlipidemia is evaluated for

symptoms of progressive chest discomfort. He has increasing substernal chest tightness with

associated shortness of breath when he walks five blocks or climbs two flights of stairs.

On physical exam, his heart rate is 58 bpm, blood pressure is 132/82 mm Hg, and cardiac

examination is normal. His current medications include aspirin 81 mg daily, metoprolol 25 mg twice
daily, and simvastatin 20 mg daily. An exercise nuclear stress test reveals mild inferior ischemia and a
normal left ventricular ejection fraction. Coronary angiography reveals a 70% stenosis of the mid
right coronary artery (RCA) and no other obstructive lesions.

Which of the following is the most appropriate next step in his management?

a. Fractional flow reserve (FFR) of the RCA.

b. Isosorbide mononitrate 30 mg daily.

c. Intravascular ultrasound (IVUS) of the RCA.

d. Percutaneous coronary intervention (PCI) to the RCA.

e. Clopidogrel 75 mg daily.

3.

b.
Isosorbide mononitrate 30 mg daily.
This patient has angina with a low-risk stress test, low-risk angiographic findings, and is not on
optimal medical therapy. Based on the Appropriate Use Criteria guidelines for revascularization,

this patient should have a trial of optimal medical therapy, which is defined as treatment with at least
two maximally-tolerated antianginal medications (beta-blockers, calcium channel blockers,

long-acting nitrates, and ranolazine).

In this patient, the addition of a second antianginal medication is the most appropriate next step,
before PCI would be performed. FFR or IVUS is not recommended in the setting of a stress test that
has already demonstrated ischemia in the territory of the angiographically abnormal lesion.

Clopidogrel does not have antianginal properties, and there are no data for its use in this scenario.

4.

A 48-year-old man is seen for cardiovascular (CV) risk assessment. He has a history of tobacco

use, but quit 15 years ago. His father underwent coronary artery bypass grafting at age 47. He is
active, but does not exercise regularly. He has no other significant health issues. His blood pressure is
135/80 mm Hg in both arms, and his exam is unremarkable.

His 10-year risk of atherosclerotic CV disease (ASCVD) based on the Pooled Cohort Equation is

7.8%. He states an interest in alternative medicine and prefers supplements over conventional

medicines. He is concerned about his CV health and would like advice. He tells you that he does not
like traditional medications and prefers a more natural approach.

In addition to encouraging lifestyle changes, which of the following should be initiated?

a. Vitamin D supplementation 2000 international units.

b. Atorvastatin 40 mg.

c. Daily multivitamin.

d. Red yeast rice 1200 mg.

e. Vitamin E 400 international units and beta-carotene 15 mg.

4.

b.
Atorvastatin 40 mg.
Based on his risk factors, this patient’s 10-year risk of ASCVD based on the Pooled Cohort Equation
is 7.8%. According to the most recent guidelines, the initiation of a moderate- or high-intensity statin
is appropriate. Supplements such as red yeast rice contain varying amounts of monacolin K, the active
ingredient in lovastatin. As with many supplements, they are not well regulated, and

therefore the amount of active ingredients may vary considerably.

Although vitamin D deficiency is a significant concern for many Americans, to date, there are no

data to support its use for prevention of CVD. The Physicians’ Health Study randomized 14,641

male physicians, the large majority without coronary artery disease, to either multivitamin daily or
placebo. No benefit was seen with taking multivitamins. The U.S. Preventive Services Task Force

has recommended against use of vitamin E and beta-carotene for CV prevention.

5.

A 50-year-old man presents to the emergency department with an acute inferior myocardial

infarction. A drug-eluting stent is placed in the mid right coronary artery, and he recovers well. The
patient remarks, "I just had a stress test last week and passed with flying colors. How can I have had a
heart attack?" You explain that atherosclerotic plaques may be too small to induce ischemia on stress
testing, but may be vulnerable to rupture and result in acute thrombotic arterial

occlusion.

Which of the following features is such vulnerable plaques most likely to have?

a. Increased smooth muscle proliferation.

b. Reduced inflammatory markers.

c. Large lipid-filled cores.

d. Thick overlying fibrous caps.

e. Low macrophage content.

5.
c.
Large lipid-filled cores.

Vulnerable plaques are most often plaques with lipid-rich cores, evidence of inflammation, and thin
fibrous caps. Therefore, an exercise stress test may not predict the presence of such a plaque,

because they are not necessarily flow limiting. These plaques typically have a high macrophage

content relative to smooth muscle cells.

6.

A 55-year-old man who is a smoker but otherwise healthy presents to the emergency

department with several hours of stuttering chest and left arm pain. He is on no medications at

home but took an aspirin before he came to the emergency department, and his pain was

somewhat reduced by the time of arrival. His blood pressure is 135/87 mm Hg and heart rate is 90

bpm. Physical examination is otherwise normal. His electrocardiogram shows T-wave inversions

and 1 mm ST depressions in the inferior and lateral precordial leads. His point of care troponin is
elevated. He is initially treated with oxygen, metoprolol, and intravenous nitroglycerin and started on
systemic anticoagulation.

Which of the following physiologic components of this patient's acute coronary syndrome explains

why the aspirin has helped improve his pain?

a. Inhibition of thrombin.

b. Inhibition of antithrombin III.

c. Inhibition of the cyclooxygenase-1 enzyme.

d. Inhibition of the P2Y12 receptor.

e. Inhibition of glycoprotein IIb/IIIa interaction with fibrinogen.

6.
c.
Inhibition of the cyclooxygenase-1 enzyme.

Aspirin inhibits platelet activation by irreversibly targeting the cyclooxygenase-1 enzyme.

Clopidogrel, prasugrel, and ticagrelor target the P2Y12 adenosine diphosphate receptor.

Glycoprotein IIb/IIIa inhibitors include eptifibatide, abciximab, and tirofiban. Bivalirudin is the only
intravenous direct thrombin inhibitor used in the treatment of acute coronary syndrome. Heparins

inhibit the activity of antithrombin III.

7.

The critical step in platelet aggregation involves which of the following receptors?

a. Glycoprotein Ib/IX.

b. Thrombin.

c. Glycoprotein IIb/IIIa.

d. Inositol trisphosphate 3.

e. Actin.

7.
c.
Glycoprotein IIb/IIIa.

Platelets have four major functions: adherence, activation, aggregation, and interaction with

coagulation factors. Platelet activation occurs when platelets are exposed to soluble factors such as
adenosine diphosphate or thrombin at the site of vascular injury. This initiates a shape change in the
platelet, and multiple filopodial projections are formed by rearrangements in the cytoskeleton.

These cytoskeletal changes open the extracellular side of the glycoprotein (GP) IIb/IIIa receptor and
allow it to bind to fibrinogen. The binding of fibrinogen allows connections between platelets and is
critical for platelet aggregation. Therefore, the receptor critical to platelet aggregation is the GP

IIb/IIIa receptor.

GP Ib/IX is cleaved during the process of platelet activation, but does not have a role in

aggregation. Inositol trisphosphate 3 receptors are involved in the calcium signaling required for
activation, but do not have a role in aggregation. Thrombin is responsible for initiation of platelet
activation, and its receptor on the platelet surface is protease-activated receptor 1. Actin is a
cytoskeletal protein.

8.

A 59-year-old woman presents to the emergency department with a 3-month history of

exertional chest pain. Her physical exam is unremarkable. Her resting electrocardiogram (ECG) and
cardiac biomarkers are normal. She experiences chest pain during a stress test with associated ST-
segment depression. However, perfusion imaging is normal. Subsequent coronary angiography

demonstrates mild luminal irregularities.

Which of the following is the most likely explanation for her chest pain?

a. Hypertrophic cardiomyopathy.

b. Noncardiac chest pain.

c. Coronary microvascular dysfunction.

d. Coronary vasospasm.

e. Atherosclerotic coronary artery disease (CAD).

8.

c.
Coronary microvascular dysfunction.
Coronary microvascular dysfunction is diagnosed based on the findings of typical anginal symptoms
and noncritical epicardial CAD usually associated with abnormal exercise treadmill stress testing.

Women with normal or near normal coronary arteries with evidence of ischemia and angina have

an overall higher risk of adverse cardiovascular events compared with patients without angina and
without CAD. The prognosis of this patient would not be expected to be better than patients

without cardiovascular disease. In general, patients with angina and fixed obstructive CAD have a
worse outcome than patients with angina and without fixed obstructive CAD.

Vasospasm is typically diagnosed with ST-segment elevation at the time of symptoms, but stress

testing is not sensitive for the diagnosis. Hypertrophic cardiomyopathy can cause chest pain, but
would be expected to be associated with ECG changes, as well as physical exam findings suggestive
of outflow obstruction. Atherosclerotic CAD is unlikely to be an adequate explanation, based on the
findings described on her angiogram.

9.

A 53-year-old woman with hypertension, hyperlipidemia, diabetes, and smoking history has

progressively worsening exertional jaw discomfort for the past 6 months. She is generally

sedentary, has not seen a physician in many years, and is presently taking aspirin and atorvastatin 80
mg daily for medications. Blood pressure in the office is 145/78 mm Hg and pulse is 82 bpm. Her
low-density lipoprotein (LDL) returned at 137 mg/dl and her glycated hemoglobin is 7.5%.

She was referred for a pharmacologic myocardial perfusion imaging scan (MPS), which showed a

reversible defect in the mid to distal anterior wall. Coronary angiography shows diffuse plaque in the
mid right coronary artery with only 20% narrowing and serial 20-30% lesions in the mid left

anterior descending artery (LAD).

Which of the following is the next most appropriate step in the patient's management, in addition to
aggressive risk factor modification and medical therapy with a beta-blocker, angiotensin-converting
enzyme inhibitor, and statin?

a. Percutaneous coronary intervention (PCI) of the LAD.

b. Isosorbide mononitrate.

c. Ranolazine.

d. Extracorporeal counterpulsation.
e. Fractional flow reserve (FFR).

9.

b.
Isosorbide mononitrate.
The prognosis for patients with coronary microvascular dysfunction is not benign and may be

similar to selected age- and sex-matched patients with epicardial coronary artery obstructive

disease. This finding of no obstructive disease should not lead to complacency relative to

management. The first line of management should be risk factor modification beginning with

lifestyle modification. The patient should be encouraged to increase her daily activities, and if
possible, begin a regular exercise program like walking. She should also understand that if angina is
induced with light exercise, such as walking, it is not dangerous and titration of isosorbide

mononitrate may permit her to walk comfortably without angina. The goal is to work toward the

American Heart Association recommended 10,000 steps per day, achieve excellent blood pressure

control (<120/90 mm Hg), LDL-lowering, and optimal management of glycemia.

PCI of a nonobstructive lesion will not result in symptom relief. However, her MPS defect and serial
stenoses are colocalized to the same coronary distribution, and serial stenoses in the 30% diameter
range could contribute to important obstruction. Thus, if her symptoms persist, FFR would be

helpful to assess the functional significance of the serial stenoses. If these lesions are confirmed to be
truly nonobstructive, coronary flow reserve measurements would be useful to assess for

coronary microvascular dysfunction. Ranolazine and extracorporeal counterpulsation should not be

the first line of management.

10. A 49-year-old woman with a history of diabetes and hypertension, and a family history of

coronary artery disease (CAD) presents to her physician with the chief complaint of chest pain and
heaviness that occurs with exertion and infrequently at rest. Her symptoms have been persistent

for nearly a year. She initially attributed her symptoms to acid reflux.

She is referred for an exercise treadmill test. Her body mass index is 30 kg/m2. She is able to

exercise for 8 metabolic equivalents (METs) on a standard Bruce protocol, but exercise is limited by
her typical angina. Her electrocardiogram shows 3-4 mm ST-downsloping segment depressions in

the lateral leads. She is referred for coronary angiography that shows no obstructive CAD and no

luminal irregularities.

Which of the following is the most appropriate next best step in the patient's management?
a. Esophagogastroduodenoscopy (EGD).

b. Intravascular ultrasound (IVUS).

c. Ergonovine challenge.

d. Coronary flow reserve measurement.

e. Assess response to empiric antispasm medication.

10. d.
Coronary flow reserve measurement.
This patient has multiple cardiovascular risk factors and evidence of ischemia on stress testing. She
has no obstructive CAD on coronary angiography. The most likely diagnosis is coronary

microvascular dysfunction (CMD). The exact mechanism of CMD is not entirely clear. From a

pathophysiological view, CMD as reduced coronary flow reserve (CFR) most often results from a

combination of different alterations, such as impaired vasodilation, enhanced vasoconstrictor

responsiveness, and/or structural remodeling of the coronary microvasculature. CMD can cause

abnormalities in stress testing indicative of ischemia without obstructive epicardial CAD.

CFR measurement either invasively (intracoronary Doppler catheter) or noninvasively (positon

emission tomography, cardiac magnetic resonance imaging, or transthoracic Doppler), can be very

helpful in determining if CMD is present. There is no role for IVUS in this setting, nor does

"prophylactic" stenting of borderline plaques improve outcomes and may be harmful. Coronary
spasm is not the likely etiology in this case, particularly given the appearance of smooth coronaries
on angiography and lack of prominent component of her symptoms occurring at rest. Nevertheless,

testing with intracoronary acetylcholine would exclude spasm with a high degree of certainty and

perhaps avoid need to expose her to multiple calcium antagonists and nitrates.

11. A 90-year-old man with hypertension and hyperlipidemia had previously enjoyed a good

quality of life without symptoms secondary to cardiovascular disease. However, over the last 3

months, he has noted the progressive onset of symptoms consistent with exertional angina. He

generally enjoys taking walks around a pond near his home. However, he now notes that after 100

yards, he develops heaviness in the center of his chest that resolves with rest. He was started on a beta-
blocker, aspirin, and a statin, but continues to have lifestyle-limiting angina.

He is referred for an exercise tolerance test, where he is able to perform 5 METs of exercise prior to
developing limiting angina and 3 mm ST depressions in the lateral leads. He is referred for cardiac
catheterization, where he is found to have a 90% focal lesion in the mid left anterior descending
artery (LAD), a 50% lesion in the first obtuse marginal artery (OM1), and a focal 99% stenosis of the
right coronary artery (RCA) with collaterals to the distal RCA from the LAD.

Which of the following is the most appropriate next step for this patient?

a. Medical therapy.
b. Hybrid CABG/percutaneous coronary intervention (PCI).

c. Referral for coronary artery bypass grafting (CABG).

d. PCI of the RCA and LAD.

e. PCI of the RCA, OM, and LAD.

11. d.

PCI of the RCA and LAD.

The patient's age is not a limitation to revascularization. With advancements in both surgical and
interventional techniques, the safety of revascularization procedures has improved over time and

the elderly have as much to gain by revascularization as the nonelderly. This patient has a low

SYNTAX score, which would suggest that the outcomes of multivessel PCI are comparable with

CABG in terms of death and myocardial infarction out to 5 years.

PCI likely offers a reduced risk of stroke. Therefore, PCI is the preferred revascularization strategy
for this patient given that it is less invasive. Fractional flow reserve of the OM lesion may be

reasonable to show that the lesion is not hemodynamically significant. At this time, there are no
definitive outcome data to support routine use of hybrid revascularization procedures.

12. A 57-year-old man with hypertension, tobacco use, and diabetes presents with 3 weeks of
exertional chest tightness. An exercise treadmill stress test is notable for reproducible chest

discomfort associated with 2 mm inferolateral ST-segment depressions and ST-segment elevation

in lead aVR.

He is started on aspirin, simvastatin, and metoprolol and referred for cardiac catheterization, which
demonstrates a right-dominant system with focal 90% proximal left anterior descending artery

(LAD), 80% mid left circumflex, and 80% mid right coronary artery stenoses. You discuss with him

the options of surgical versus percutaneous revascularization.

Which of the following outcomes is associated with surgical revascularization in this patient?

a. Decreased bleeding risk.

b. Increased need for repeat revascularization.

c. Decreased stroke risk.

d. Improved ejection fraction.


e. Improved survival.

12. e.
Improved survival.
In patients with diabetes and significant left main coronary artery disease (=50%) or severe

coronary artery disease (=70%) in three major coronary arteries or involving the proximal LAD plus
one other coronary artery, coronary artery bypass grafting is the recommended revascularization

strategy and has been shown to improve survival. In contrast, while percutaneous coronary

intervention (PCI) may improve anginal symptoms in patients with stable ischemic heart disease, no
studies to date have demonstrated that PCI confers a survival advantage without recent acute

coronary syndrome. Stroke risk is increased in CABG versus PCI.

13. A 52-year-old man with hypertension and hyperlipidemia is referred for cardiac stress testing due
to atypical chest pain and recently declining exercise tolerance. He completes 7 minutes of a standard
Bruce protocol and stops due to fatigue and chest burning. Perfusion imaging reveals a

moderately severe, reversible apical defect. He is started on aspirin 81 mg daily, simvastatin 20 mg


daily, metoprolol 25 mg daily, and amlodipine 5 mg daily, but his symptoms persist. He is referred
for cardiac catheterization, which reveals a 70% mid left anterior descending artery stenosis

treated successfully with percutaneous coronary intervention (PCI).

Which of the following best describes the long-term outcomes of PCI versus coronary artery bypass
grafting (CABG) in this patient?

a. No difference.

b. Decreased mortality with PCI.

c. Improved left ventricular ejection fraction with CABG.

d. Decreased target vessel revascularization with PCI.

e. Increased recurrent myocardial infarction (MI) with CABG.

13. a.
No difference.
More than 20 randomized controlled trials have compared CABG with balloon angioplasty or

stenting in patients with single-vessel coronary artery disease, demonstrating similar outcomes

regarding survival and recurrent MI. CABG has a significantly higher rate of stroke, but lower repeat
revascularization rate compared with PCI. In this patient with one significant stenosis amenable to
revascularization and unacceptable angina despite guideline-directed medical therapy, PCI or CABG

are both beneficial to improve anginal symptoms.

14. A 62-year-old man with a history of smoking, hypercholesterolemia, and obesity is being

evaluated for progressive dyspnea on exertion. He denies chest pain, orthopnea, or peripheral

edema. An echocardiogram demonstrates left ventricular ejection fraction (EF) of 35% with global

hypokinesis and no significant valvular dysfunction. Coronary angiography reveals significant three-
vessel coronary artery disease (CAD) with an occluded but well-collateralized dominant right

coronary artery, and calculated SYNTAX score of 35.

Which of the following is the most appropriate next step?

a. Hybrid CABG/PCI.

b. Implantable cardioverter-defibrillator (ICD).

c. Coronary artery bypass grafting (CABG).

d. Medical therapy alone.

e. Multivessel percutaneous coronary intervention (PCI).

14. c.

Coronary artery bypass grafting (CABG).

The SYNTAX trial randomized 1,800 patients with left main or three-vessel CAD to revascularization
with drug-eluting stents or CABG. Overall, at 3 years, major adverse cardiac events were lower for
CABG over PCI (20.2% vs. 28.0%), driven by decreased recurrent myocardial infarction and repeat

revascularization in the CABG arm.

In a post hoc analysis, an angiographic SYNTAX score was derived for each patient as a surrogate

measure of the CAD complexity, based on the location, severity, and extent of CAD (low SYNTAX
score, 22; intermediate, 23-32; and high, 33). While patients with a low SYNTAX score had similar
outcomes with PCI or CABG, those with intermediate and high SYNTAX scores treated with CABG

exhibited lower mortality at 3 years compared with PCI, with significantly decreased target vessel
revascularization.

In summary, the SYNTAX results suggest that PCI or CABG is comparable in less complex disease,

while in patients with more complex disease, CABG is the preferred revascularization strategy.

There are no data as of yet to support use of hybrid CABG/PCI. Medical therapy alone is not

sufficient in the setting of reduced EF. ICD is warranted only if EF remains <35% at 3 months after
revascularization.

15. An 82-year-old man with a history of coronary artery disease (CAD) status/post percutaneous

coronary intervention (PCI) to the left anterior descending artery (LAD) 10 years ago, chronic

obstructive pulmonary disease, obesity, and chronic kidney disease develops progressive angina on
exertion. He has been managed successfully for many years on a combination of aspirin,

atorvastatin, and atenolol. Echocardiography reveals preserved left ventricular function.

He is referred for coronary angiography, which demonstrates 60% ostial left main coronary

stenosis, a widely patent LAD stent with diffuse luminal irregularities, no significant left circumflex
disease, and focal 70% mid-right coronary artery stenosis.

Which of the following is the next best step in this patient's management?

a. Heart Team evaluation.

b. Multivessel PCI.

c. Medical therapy alone.

d. Coronary artery bypass grafting (CABG).

e. PCI to the left main coronary artery.

15. a.
Heart Team evaluation.
The current stable ischemic heart disease guideline recommends that patients with significant left
main or complex multivessel CAD be evaluated by a multidisciplinary Heart Team to determine the

most appropriate method of revascularization. This strategy mimics typical protocols used in

clinical trials, where a Heart Team, composed of an interventional cardiologist and cardiac surgeon,
review all available patient data and discuss potential treatment options with the patient.

Calculation of the Society of Thoracic Surgeons (STS) and SYNTAX scores assist in determining the
surgical risk category and complexity of CAD.

While PCI may be a reasonable strategy in patients with amenable coronary anatomy, high surgical

risk, or other considerations, in general, ad hoc PCI immediately following diagnostic coronary

angiography is discouraged in patients with complex multivessel CAD.

Although escalation of medical therapy may improve angina, if the patient is a candidate for

revascularization, a Heart Team evaluation would be beneficial.

16. A 65-year-old man with diabetes controlled with medications comes to you for evaluation of

exertional chest pain. He has been appropriately managed for chronic stable angina with beta-

blockers, statin, nitrates, and aspirin.

The patient is able to exercise 7 minutes on a Bruce protocol, his classic chest pain recurs at stage 3,
and there are 2 mm ST depressions in the anterolateral leads with reversible perfusion defects
anteriorly. His left ventricular ejection fraction (LVEF) is 50%. Cardiac catheterization reveals 90%

promimal left anterior descending artery (LAD), 70% first obtuse marginal, 80% mid left circumflex,
and 90% mid right coronary artery stenoses.

Which of the following is the next best step in the management of this patient?

a. Ranolazine extended-release 500 mg PO daily.

b. Coronary artery bypass grafting (CABG) revascularization.

c. LAD PCI revascularization only.

d. Increase metoprolol.

e. Multivessel percutaneous coronary intervention (PCI) revascularization.

16. b.
Coronary artery bypass grafting (CABG) revascularization.

This patient has diabetes, severe multivessel coronary artery disease (CAD), and a reduced LVEF. He
should undergo complete revascularization with CABG.

Recently, results from the FREEDOM trial in patients with diabetes and multivessel CAD treated

with CABG had a significantly lower composite rate of death from any cause, nonfatal myocardial

infarction, or nonfatal stroke when compared with diabetic patients treated with PCI.

An analysis performed in 2009 of data in 7,812 patients (1,233 with diabetes) in 10 randomized

clinical trials demonstrated a worse long-term survival rate in patients with diabetes after balloon
angioplasty or bare-metal stent implantation than after CABG.

The BARI 2D trial randomly assigned 2,368 patients with type 2 diabetes and CAD to undergo

intensive medical therapy or prompt revascularization with PCI or CABG, according to whichever

was thought to be more appropriate. By study design, those with less extensive CAD more often

received PCI, whereas those with more extensive CAD were more likely to be treated with CABG.

The study was not designed to compare PCI with CABG. At 5-year follow-up, no difference in rates

of survival or major adverse cardiac events (MACE) between the medical therapy group and those

treated with revascularization was noted. In the PCI stratum, no significant difference in MACE

between medical therapy and revascularization was demonstrated (drug-eluting stent in 35%;

bare-metal stent in 56%); in the CABG stratum, MACE occurred less often in the revascularization

group.

17. A 73-year-old morbidly obese woman with insulin-dependent diabetes, dyslipidemia, and

severe ambulatory dysfunction is sent to you for preoperative assessment before total knee

replacement. She has chronic kidney disease with a creatinine of 2.1 mg/dl. Her heart rate is 80

bpm, blood pressure is 150/80 mm Hg, and respiratory rate is 20 breaths/minute. She has no

evidence of heart failure.

Her electrocardiogram (ECG) reveals sinus rhythm with inferior Q waves noted. The patient's

echocardiogram is normal. She is on insulin, losartan 100 mg PO daily, and atorvastatin 40 mg PO


daily.

Which of the following is the next best step in the preoperative cardiac risk assessment for this
patient?

a. Pharmacologic nuclear stress test.

b. Coronary computed tomographic angiography.

c. Cardiac catheterization.

d. No further cardiac evaluation is necessary.

e. Exercise ECG stress test.

17. a.
Pharmacologic nuclear stress test.
This patient has several risk factors for coronary artery disease. While knee replacement is not a
high-risk surgery compared with vascular surgery and head and neck surgery, this patient has

several concerning clinical features, including an abnormal resting ECG, increased creatinine, insulin
therapy, uncontrolled blood pressure, and unclear functional capacity.

Multivariate risk indices such as the revised cardiac risk index (RCRI) and 2014 American College of
Cardiology (ACC)/American Heart Association (AHA) perioperative cardiovascular evaluation

guideline can be applied to determinate the major adverse cardiac event (MACE) risks. Her RCRI

risk is 11% for MACE (includes possible myocardial infarction, insulin use, and creatinine >2.0

mg/dl).

According to the ACC/AHA 2014 guideline for perioperative cardiovascular evaluation for

noncardiac surgery, this is a Class IIa indication:

"It is reasonable for patients who are at an elevated risk for noncardiac surgery and have poor
functional capacity (<4 METs) to undergo noninvasive pharmacological stress testing (either

dobutamine stress echocardiogram or pharmacological stress perfusion imaging) if it will change

management. (Level of Evidence: B)"

Figure 1 outlines the ACC/AHA perioperative guideline. A pharmacologic stress test is appropriate in
someone who cannot exercise.
(Figure 1)

18. A 55-year-old man comes to you for preoperative cardiovascular risk stratification for inguinal
hernia repair. The patient has hypertension and takes atenolol 25 mg PO daily. He has a strong

history of smoking 30 pack-years and has not had his lipid profile evaluated for 2 years. He has a
family history of coronary artery disease; his father had a myocardial infarction at 53 years of age.

The patient notices pain and discomfort in the groin area while walking. However, he is able to

walk up a hill and mow the lawn.

On examination, heart rate is 53 bpm, blood pressure is 120/83 mm Hg, and respiratory rate is 15

breaths/minute. His electrocardiogram shows sinus rhythm at 54 bpm with an incomplete right

bundle branch block.

Based on these findings, which of the following is your next best step in evaluating this patient?

a.
Transthoracic echocardiogram.

b.

Exercise treadmill stress testing.

c.

Proceed with surgery.

d.

Dobutamine stress echocardiography.

e.

Coronary computed tomography angiography.

18. c.
Proceed with surgery.
Functional status is a reliable predictor of perioperative and long-term cardiac events. Patients with
reduced functional status preoperatively are at increased risk of complications. Conversely, those
with good functional status preoperatively are at lower risk. Moreover, in highly functional

asymptomatic patients, it is often appropriate to proceed with planned surgery without further

cardiovascular testing, as outlined in the 2014 perioperative cardiovascular evaluation guideline


(Figure 1).

19. A 63-year-old woman comes to your office for preoperative clearance for a hysterectomy. The
patient has been complaining of dysfunctional uterine bleeding and was found to have multiple

uterine fibriods that have not improved with embolization. She has history of insulin-dependent

diabetes, hypertension, and chronic kidney disease (stage 3). Her medications include lisinopril 40

mg daily, ferrous sulfate, and insulin. She can do her activities of daily living, but becomes short of
breath climbing a flight of stairs.
On exam, her heart rate is 88 bpm and blood pressure is 133/83 mm Hg, respiration rate is 15

breaths/minute, and conjunctiva are pale. Cardiovascular examination reveals soft systolic flow

murmur in the left sternal border. There are no rales or rhonchi and no pedal edema.

An electrocardiogram in your office shows sinus rhythm with left bundle branch block (LBBB). Her

most recent labs include glycated hemoglobin of 8.9%, hemoglobin of 9.0 mg/dl, and creatinine of

2 mg/dl.

Which of the following is the next best step in her management?

a. Pharmacologic nuclear stress test.

b. Exercise treadmill stress test.

c. Proceed with surgery.

d. Dobutamine stress echo.

e. Start beta-blockers.

19. c.
Proceed with surgery.
This patient has several cardiovascular risk factors and evidence of an LBBB. She can, however,

climb a flight of stairs, and thus has a functional capacity >4 METs. She does not require stress testing
prior to undergoing medically necessary surgery.

Functional status is a reliable predictor of perioperative and long-term cardiac events. Patients with
reduced functional status preoperatively are at increased risk of complications. Conversely, those
with good functional status preoperatively are at lower risk. Moreover, in highly functional

asymptomatic patients, it is often appropriate to proceed with planned surgery without further

cardiovascular testing.

If a patient has not had a recent exercise test before noncardiac surgery, functional status can

usually be estimated from activities of daily living. Functional capacity is often expressed in terms of
METs, where 1 MET is the resting or basal oxygen consumption of a 40-year-old man who weighs

70 kg. In the perioperative literature, functional capacity is classified as excellent (>10 METs), good
(7-10 METs), moderate (4-6 METs), poor (<4 METs), or unknown.

Perioperative cardiac and long-term risks are increased in patients unable to perform 4 METs of

work during daily activities. Examples of activities associated with <4 METs are slow ballroom
dancing, golfing with a cart, playing a musical instrument, and walking at approximately 2-3 mph.

Examples of activities associated with >4 METs are climbing a flight of stairs or walking up a hill,
walking on level ground at 4 mph, and performing heavy work around the house.

20. A 65-year-old man with intermittent bilateral lower-extremity claudication, hypertension, and
diabetes presents to your office for assessment prior to planned femoral-popliteal bypass surgery.

He denies chest pain, but is significantly limited by his claudication and cannot walk more than a
block without stopping. A pharmacologic stress test reveals a large area of ischemia in the anterior
wall; left ventricular ejection fraction is 40%. Coronary angiography reveals three-vessel coronary
artery disease.

Which of the following is the most appropriate next step in his management?

a. Maximize medical therapy and proceed with vascular surgery.

b. Multivessel percutaneous coronary intervention (PCI) with drug-eluting stents.

c. Maximize medical therapy and cancel surgery.

d. Multivessel PCI with bare-metal stents.


e. Coronary artery bypass grafting.

20. e.
Coronary artery bypass grafting.
This patient has significant ischemic disease, reduced left ventricular function, and diabetes.

Assessment of his functional status is limited by his significant claudication, but he has a high-risk
nuclear perfusion scan and multivessel coronary artery disease. Regardless of the plans for elective
surgery, revascularization of his coronary artery disease is indicated.

Coronary artery bypass grafting is preferable to PCI in light of his diabetes, as shown in the

FREEDOM trial.

21. A 60-year-old woman with hypertension presents to the emergency room with chest pain.

She describes acute onset of chest pain radiating to her back associated with leg weakness and

presyncope. Blood pressure is 162/100 mm Hg, pulse is 100 bpm, and respiratory rate is 20

breaths/minute. Cardiac exam reveals an S4 gallop with no murmur. She has no known drug

allergies. Her electrocardiogram (ECG) is shown in Figure 1.

Which of the following is the most appropriate next step?


a. Coronary angiography.

b. Chest computed tomography angiography (CTA).

c. Transthoracic echocardiogram.

d. Thrombolytics.

e. Chest X-ray.

21. b.

Chest computed tomography angiography (CTA).

The patient's symptoms of chest pain radiating to the back and weakness of the lower extremities

are classic for aortic dissection, in this case complicated by extension of the dissection into the ostium
right coronary artery, as evidenced by the inferior injury current on the ECG, which may be present
in 7% of dissections. This combination may lead to misdiagnosis and delayed therapy.

Because the symptoms are so suggestive of dissection, emergent CTA of the chest or
transesophageal echocardiography imaging of the aorta and cardiac surgical consultation for the

life-threatening dissection is more appropriate than CTA (see management algorithm in Figure 2).

Chest X-ray may show a widened mediastinum, which is not very sensitive or specific and would

not provide enough information to proceed with surgery. Thrombolytic therapy would be

contraindicated, with the patient's symptoms the central factor in making the correct diagnosis.

22. Which of the following patients most likely has a myocardial infarction (MI), according to the
Third Universal Definition of MI?

a.

A 48-year-old woman with 2 weeks of chest pain, palpitations, progressive dyspnea, and volume

overload. Echocardiogram reveals diffuse hypokinesis with left ventricular ejection fraction of 25%.

Endomyocardial biopsy shows diffuse necrosis, multinucleated cells, and lymphocytic and

eosinophilic infiltrates.

b.

A 38-year-old obese man with 10-year tobacco history, recent upper respiratory symptoms, 2 days of
pleuritic chest pain, and diffuse ST elevations; troponin values are 0.10, 0.11, and 0.09.

c.

A 56-year-old male unrestrained driver with hypertension after a motor vehicle accident and steering
wheel trauma to the chest. His echocardiogram shows anterior wall hypokinesis.

d.

A 61-year-old woman with noninsulin-dependent diabetes, obesity, and sleep apnea presents with

new-onset atrial fibrillation, heart rate 130 bpm, and ST depressions in leads II, III, and aVF.

e.

A 68-year-old woman with hypertension, diabetes, and end-stage renal disease presents with

shortness of breath after missing two hemodialysis sessions. Serial troponin values are 0.05, 0.04,
0.06, and 0.03. Her electrocardiogram shows left ventricular hypertrophy, but no ST/T wave changes.

22. d. A 61-year-old woman with noninsulin-dependent diabetes, obesity, and sleep apnea
presents with new-onset atrial fibrillation, heart rate 130 bpm, and ST depressions in

leads II, III, and aVF.


The Third Universal Definition of MI requires elevation in cardiac troponins, with a rise and/or fall
in levels over serial measurement, together with symptoms or signs of cardiac ischemia. These may
include new significant ST-segment T-wave changes, new left bundle branch block, new pathologic

Q waves, imaging evidence of new regional wall motion abnormalities or new loss of viable

myocardium, or intracoronary thrombus demonstrated by angiography or autopsy. Many

conditions other than cardiac ischemia may lead to elevation in troponins. Moreover, MI may be

due either to plaque rupture (type 1 MI) or due to supply-demand mismatch, with or without

underlying coronary artery disease (type 2 MI).

The patient presenting with atrial fibrillation and rapid ventricular response with

electrocardiographic evidence of ischemia most likely has a type 2 MI. The other clinical scenarios
most likely represent giant cell myocarditis, myopericarditis, troponin elevation without clear rise
and fall in the setting of renal disease, and myocardial injury due to contusion. If no active ischemia is
suspected, then these would not be considered instances of MI, according to the Third Universal
Definition of MI. Troponin elevations due to defibrillation shocks, receipt of cardiotoxic

chemotherapeutic agents, and rhabdomyolysis with cardiac involvement are other examples where

myocardial injury occurs apart from ischemia and which would not be considered MI.
(Figure 2)

23. A 53-year-old man presents to the emergency room with 1 hour of chest pain. Current vital signs
are pulse 122 bpm and irregular, blood pressure 178/102 mm Hg, respiratory rate 18

breaths/minute, Saturation of peripheral oxygen is 98% on room air. The neck veins are flat. The

lungs are clear and there is no murmur or gallop.

His electrocardiogram reveals atrial fibrillation with rapid ventricular response and 1 mm ST-

elevations in leads I and aVL.

Which of the following is most appropriate to manage his atrial fibrillation?

a. Oral or intravenous metoprolol.

b. Intravenous diltiazem.

c. Intravenous ibutilide.
d. Sublingual nifedipine.

e. Oral amiodarone.

23. a.
Oral or intravenous metoprolol.
In the setting of ST-segment elevation myocardial infarction (STEMI), beta-blockers are the

preferred rate control agent for atrial fibrillation. Routine use of intravenous beta-blockers for
STEMI is not recommended, but it may be used in a hypertensive urgency or tachyarrhythmia in

the absence of other contraindications. Oral beta-blockade within 24 hours of presentation

remains a Class I indication in STEMI. Calcium channel blockers have no demonstrated benefit in

STEMI, although they could be considered for rate control in atrial fibrillation, but not if systolic
dysfunction is present.

Short-acting nifedipine for hypertension is harmful because of the potential for hypotension and

reflex tachycardia. Oral amiodarone would not be useful acutely in this setting. Ibutilide would be
inappropriate, as this patient has no indication for emergent cardioversion and the duration of

atrial fibrillation is unknown.

24. A 58-year-old woman with rheumatoid arthritis presents with 1 week of intermittent nausea

and exertional chest pressure and 30 minutes of constant intense chest pressure. Her

electrocardiogram is notable for up to 2.5 mm ST elevations in leads I, aVL, and V5-V6. She is found
to have occlusion of a large second obtuse marginal coronary artery branch and undergoes drug-
eluting stent placement. Follow-up testing shows normal cardiac function and hemoglobin A1c of

5.3%.

Which of the following is most likely to be beneficial for chronic outpatient therapy for this patient?

a.

Isosorbide mononitrate 30 mg daily.

b.

Verapamil sustained release 180 mg daily.

c.

Metoprolol succinate 50 mg daily.

d.

Enalapril 10 mg bid.
e.

Spironolactone 25 mg daily.

24. c.
Metoprolol succinate 50 mg daily.
Beta-blockers have a Class I indication for continuation during and after hospitalization for all
patients with ST-segment elevation myocardial infarction (STEMI) in the absence of

contraindications. Certain conditions warrant caution or avoidance of beta-blocker therapy:

decompensated heart failure, low-output states, high risk of cardiogenic shock, reactive airways

disease or active bronchospasm, and prolonged first-degree or high-grade atrioventricular block.

Angiotensin-converting enzyme (ACE) inhibitors have a Class I indication for use in patients with
STEMI with anterior involvement, heart failure, and left ventricular ejection fraction (LVEF) ≤40% in
the absence of contraindications. Angiotensin-receptor blockers may be used for patients who are

intolerant of ACE inhibitors.

For patients such as this one, who do not have anterior involvement, heart failure, or low EF, ACE

inhibitors have a Class IIa indication. Aldosterone antagonists should be used (Class I indication) in
patients with STEMI and no contraindications who are already receiving an ACE inhibitor and beta-
blocker and who have LVEF ≤40% AND either symptomatic heart failure or diabetes. This patient

would not meet these criteria.

Nitrates are not indicated for routine use after STEMI. Calcium channel blockers have not been

shown to reduce infarct size or rate of reinfarction, but may be helpful in the treatment of

ischemia, hypertension, or rapid ventricular rate in patients with atrial fibrillation who are

intolerant of beta-blockers.

25. A 53-year-old man with diabetes is admitted with an acute inferoposterior ST-segment elevation
myocardial infarction (STEMI). He undergoes primary percutaneous coronary

intervention with placement of a drug-eluting stent to the proximal left circumflex artery. He has
single-vessel disease, left ventricular end-diastolic pressure of 14 mm Hg, and an ejection fraction
(EF) of 35%.

According to the 2013 STEMI guideline, which of the following is a Class I indication for initiating
an angiotensin-converting enzyme (ACE) inhibitor within 24 hours in this patient?

a.

EF of 35%.STEMI.

b.
Culprit lesion location.

c.

Chronic kidney disease.

d.

Diabetes.

25. a.

EF of 35%.

An ACE inhibitor should be administered within the first 24 hours to all patients with STEMI with an
anterior location, heart failure, or EF of ≤40%, unless contraindicated (Class I, Level of Evidence A).

Oral ACE inhibitors reduce fatal and nonfatal major cardiovascular events in patients with STEMI.

The magnitude of clinical benefit is greatest in high-risk patient subgroups. Demonstration of an


early benefit (within the first 24 hours) supports the prompt use of these agents in patients without
existing contraindications (e.g., hypotension, shock, bilateral renal artery stenosis, or history of
worsening of renal function with ACE inhibitor/angiotensin-receptor blocker exposure, renal

failure, or drug allergy). The role of routine long-term ACE inhibitor therapy in low-risk patients
after STEMI who have been revascularized and treated with aggressive lipid-lowering therapies is

less certain.

26. A 60-year-old woman presents to the office for follow-up after cardiac catheterization. She

had single-vessel disease with a moderate 50% stenosis and noncritical fractional flow reserve

(0.84). Her medical history is significant for treated hyperlipidemia. Her blood pressure is 120/60

mm Hg with a heart rate of 70 bpm, and her exam is normal. She is asymptomatic on aspirin, statin,
and beta-blocker. She asks about her risk of myocardial infarction.

Which of the following best correlated with her risk of plaque rupture?

a. High cholesterol.

b. Plaque macrophage content.

c. Plaque calcium content.

d. It is unpredictable.

e. Stenosis severity.
26. b.
Plaque macrophage content.
The conversion of a stable plaque-to-plaque rupture or erosion does not have much to do with the

stenosis severity. Plaque rupture is predicted by several anatomic and systemic factors, including
inflammation, integrity of the fibrous cap, plaque lipid content, and plaque location.

27. A 40-year-old woman presents to the emergency department with chest discomfort. She

describes sharp chest pain that lasts for less than a minute and occurs at various times during the day.
Her exam is unremarkable. Her electrocardiogram is normal.

Which of the following risk scores should be used to best guide management in the emergency

department?

a. Duke treadmill score.

b. Framingham Coronary Heart Disease Risk Score.

c. HEART (History, Electrocardiogram, Age, Risk factors, and Troponin) score.

d. GRACE (Global Registry of Acute Coronary Events) score.

e. TIMI (Thrombolysis in Myocardial Infarction) score.

27. c.

HEART (History, Electrocardiogram, Age, Risk factors, and Troponin) score.

Risk scores can be used to determine the short-term risk of adverse events in patients with chest pain
or definite acute coronary syndrome (ACS). The HEART score is a prospectively studied scoring
system to help emergency departments risk-stratifiy chest pain patients according to risk of major
adverse cardiac events within 6 weeks. The HEART score includes history, electrocardiogram, age,

risk factors, and troponin.

TIMI and GRACE scores are used to estimate risk of death or MI for patients with ACS, and are not as
useful for patients with chest pain of unclear etiology. The Framingham Coronary Heart Disease Risk
Score estimates risk of heart attack in 10 years in a stable patient. The Duke treadmill score is specific
to predicting coronary artery disease in a patient undergoing stress testing.

28. A 47-year-old woman presents to the emergency department with 2 days of substernal chest

pressure radiating to the jaw with minimal exertion relieved with rest. The last episode was 1 hour
prior to presentation and lasted 15 minutes. She has no medical history and takes no medication.

Her resting electrocardiogram is normal.


Which of the following is the most accurate biomarker strategy for diagnosing myocardial

infarction (MI) in this patient?

a. High-sensitivity troponin at time 0 hours.

b. Creatine kinase-myocardial isoenzyme (CK-MB) at time 0 and 6 hours.

c. Cardiac-specific troponin at time 0 and 6 hours.

d. B-type natriuretic peptide (BNP) after 6 hours of observation.

e. Myoglobin at the time of presentation (time 0).

28. c.

Cardiac-specific troponin at time 0 and 6 hours.

Cardiac biomarkers are part of the universal definition of MI. Although the specificity of cardiac-
specific and high-sensitivity troponin assays are of concern, they are the biomarker of choice in
patients with suspected acute coronary syndrome (ACS). The current American Heart

Association/American College of Cardiology guideline recommends that cardiac-specific troponin

(troponin I or T when a contemporary assay is used) levels should be measured at presentation and 3-
6 hours after symptom onset in all patients who present with symptoms consistent with ACS to

identify a rising and/or falling pattern (Class I).

Although high-sensitivity troponin may be positive earlier, a single measure at presentation is not
sufficient to exclude MI. With contemporary troponin assays, creatine kinase myocardial isoenzyme
(CK-MB) and myoglobin are not useful for diagnosis of ACS (Class III). Use of selected newer

biomarkers, especially BNP, may be reasonable to provide additional prognostic information (Class
IIb), but are not used for the initial diagnosis.

29. A 68-year-old man presents to the emergency department with two episodes of resting chest

pressure radiating to the jaw and arm and lasting 15 minutes in the 6 hours prior to presentation.

His pain resolved without treatment and he is now pain free. His medical history is significant for
hyperlipidemia, hypertension, and tobacco abuse. His medications include aspirin, diltiazem, and

atorvastatin. His initial high-sensitivity troponin is negative and his electrocardiogram is normal.

Which of the following is the most appropriate management for this patient?

a.

Triage to observation unit for serial biomarkers.


b.

Discharge with outpatient stress testing within 48 hours.

c.

Admission for stress testing.

d.

Dual antiplatelet therapy and catheterization within 24 hours.

e.

Initiation of a glycoprotein IIb/IIIa inhibitor.

29. d.

Dual antiplatelet therapy and catheterization within 24 hours.

Patients with acute coronary syndromes (ACS) can be risk stratified with clinical risk scores

including the TIMI (Thrombolysis in Myocardial Infarction) and GRACE (Global Registry of Acute

Coronary Events) scores. Although this patient has no objective ischemia, his TIMI risk score is 4

(age over 64, severe angina, three cardiac risk factors, and aspirin use), which predicts a 20% risk of
all-cause mortality, new or recurrent MI, or severe recurrent ischemia requiring urgent

revascularization at 14 days. Several randomized trials have compared an early invasive strategy

with a delayed, selective, or conservative strategy in patients with ACS.

In the TIMACS trial, patients with a GRACE score >140, but not those with a GRACE score <140, had
a reduction in the ischemic endpoint of death, MI, or stroke. Also in the TIMACS trial, death, MI, and
rehospitalization from ACS was lower with an invasive strategy in patients with positive

troponin, ST deviation, or a TIMI risk score >3.

Upstream glycoprotein inhibitor use does not improve outcomes compared to selective use at the

time of percutaneous coronary intervention. This patient should not be discharged from the

emergency room without further cardiovascular evaluation.

30. A 45-year-old male active smoker with diabetes and hypertension presents to the emergency

room with chest pain. An urgent electrocardiogram demonstrates 4 ml of ST-segment elevation in

leads V1-V4. Emergent coronary angiography identifies acute plaque rupture in the ostial left
anterior descending coronary artery with hazy thrombus and Thrombolysis In Myocardial Infarction

(TIMI) grade 1 coronary flow. After export thrombectomy, the ruptured plaque is treated with

direct stenting with a 3.5 ml x 18 ml drug-eluting stent, which is post-dilated at 18 atm.

Which of the following statements is true about the pathobiology of acute plaque rupture?

a. The majority of coronary artery atherosclerotic plaque rupture occurs at the middle portion

of the plaque away from the plaque shoulder because the middle portion of the plaque

experiences greater tensile force from flowing blood.

b. Platelets are the major source of the myeloperoxidase enzyme, which generates free radicals

and increases inflammation in rupture-prone coronary artery atherosclerotic plaques.

c. Coronary calcium does not participate in the pathobiology of coronary artery atherosclerotic

plaque rupture, and calcification is rarely present in ruptured plaques.

d. Branch points at coronary bifurcations experience high shear stress conditions that promote

coronary artery atherosclerotic plaque inflammation and infiltration of monocytes that

promote plaque rupture.

e. Macrophage-derived matrix metalloproteinases degrade the fibrous cap and promote

coronary artery atherosclerotic plaque rupture.

30. d.

Branch points at coronary bifurcations experience high shear stress conditions

that promote coronary artery atherosclerotic plaque inflammation and infiltration of


monocytes that promote plaque rupture.
The majority of coronary artery atherosclerotic plaque rupture occurs at the plaque shoulder,

which experiences greater tensile force from flowing blood.

Branch points at coronary bifurcations experience low shear stress conditions that promote

coronary artery atherosclerotic plaque inflammation and infiltration of monocytes that promote

plaque rupture.

Neutrophils and macrophages are the major sources of the myeloperoxidase enzyme, which

generates free radicals and increases inflammation in rupture-prone coronary artery

atherosclerotic plaques.

Coronary calcium participates prominently in the pathobiology of coronary artery

atherosclerotic plaque rupture, and calcification is present in 80% of ruptured plaques.

31. A 65-year-old woman with hypertension and hyperlipidemia developed chest pain and

collapsed while walking her dog. Emergency medical responders used an automated external

defibrillator to administer one shock, which converted the patient from ventricular fibrillation to
sinus rhythm.

The patient is treated emergently for an ST-segment elevation myocardial infarction (STEMI) with

primary percutaneous coronary intervention, where a stent is used to treat a thrombotic occlusion of
the left anterior descending artery.Which of the following statements is correct regarding

mechanisms of coronary artery disease that led to arterial thrombosis and MI in this patient?

a. Coronary artery atherosclerotic plaque rupture is frequently clinically silent; however, silent
plaque rupture events speed plaque growth and promote luminal narrowing.

b. Coronary artery atherosclerotic plaque neovascularization increases plaque stability and

reduces plaque inflammation.

c. Superficial plaque erosion-induced coronary artery thrombosis occurs most often at sites of

macrophage-laden coronary artery atherosclerotic plaques that have a low content of smooth

muscle cells.
d. Ischemic preconditioning increases the size of the infarct zone in patients presenting with

STEMIs.

e. Distal embolization of platelet aggregates rarely occurs in non-STEMI patients.

31. a.

Coronary artery atherosclerotic plaque rupture is frequently clinically silent;

however, silent plaque rupture events speed plaque growth and promote luminal narrowing.

Ischemic preconditioning reduces the size of the infarct zone in patients presenting with STEMI.

Distal embolization of platelet aggregates frequently occurs in non-STEMI patients, and

embolization is a major cause of myocardial necrosis. Superficial plaque erosion-induced

coronary artery thrombosis occurs most often within atherosclerotic plaques that have a high

content of smooth muscle cells and a low lipid content.

Coronary artery atherosclerotic plaque neovascularization decreases plaque stability and

increases plaque inflammation. Neovascularization likely increases risk of plaque rupture.

32. A 69-year-old man with a history of hypertension and stroke was admitted with a non–ST-segment
elevation myocardial infarction (NSTEMI). After medical stabilization, he was managed

with an early invasive strategy and a drug-eluting stent was placed into a thrombotic 80% stenosis in
the right coronary artery.

Which of the following regimens would be part of the optimal medical management for this patient

following drug-eluting stent implantation to treat an NSTEMI?

a. Aspirin 325 mg daily and ticagrelor 90 mg twice daily.

b. Aspirin 325 mg daily and prasugrel 10 mg daily.

c. Aspirin 81 mg daily, ticagrelor 90 mg twice daily, and rivaroxaban 5 mg twice daily.

d. Aspirin 81 mg daily and prasugrel 10 mg daily.

e. Aspirin 81 mg daily and ticagrelor 90 mg twice daily.

32. e.

Aspirin 81 mg daily and ticagrelor 90 mg twice daily.


Prasugrel should not be administered to patients with a prior history of stroke or transient ischemic
attack. Aspirin 70-100 mg is the optimal dose when aspirin is combined with ticagrelor. Triple

therapy with aspirin, ticagrelor, and rivaroxaban has not been studied in acute coronary syndrome
management. Rivaroxaban reduces adverse cardiovascular events in acute coronary syndrome

patients when added to aspirin and clopidogrel.

33. A 63-year-old woman with a 2-day history of intermittent chest pain, sometimes responsive

to belching, presents to the emergency department for evaluation. Her medical history is

significant for hypertension. Her medications include amlodipine, hydrochlorothiazide, and aspirin.

Her physical examination reveals blood pressure 128/82 mm Hg, heart rate 82 bpm, and oxygen

saturation 99% on room air. Her jugular venous pressure is 6 cm water, lungs are clear, and

cardiovascular exam is normal. Her electrocardiogram shows normal sinus rhythm with nonspecific

T-wave changes. Her chest x-ray is normal. Laboratory values include two serial troponin I values of

<0.01 ng/L, normal electrolytes, creatinine 0.8 mg/dl, D-dimer 130 ng/ml, and normal blood cell
count.

Which of the following is most appropriate in her care?

a. No further testing is needed.

b. Transthoracic echocardiogram.

c. Invasive coronary angiography.

d. Ventilation/perfusion (V/Q) scan.

e. Exercise myocardial perfusion.

33. e.
Exercise myocardial perfusion.
In this patient with suspected non-ST-segment elevation acute coronary syndrome, further risk

stratification is warranted. The history, physical exam, and findings are important to consider in the
initial risk stratification. This patient's Thrombolysis in Myocardial Infarction (TIMI) risk score is 1,
which gives her a risk of major adverse cardiac events of 5% in the next 14 days. Noninvasive stress
testing can be used to further risk stratify this patient. Resting transthoracic echocardiogram will not
provide information about inducible ischemia. With a normal D-dimer and low pretest

probability of pulmonary embolism, a V/Q scan is not indicated.

34. A 61-year-old woman presents to the emergency department with 2 hours of chest pain

associated with shortness of breath. Her medical history is significant for hypertension and

hyperlipidemia. Her medications include lisinopril 20 mg and hydrochlorothiazide 25 mg daily.

On exam, her blood pressure is 122/84 mm Hg, heart rate is 94 bpm, oxygen saturation is 98% on

room air, and body mass index is 24 kg/m2. Jugular venous pressure is 7 cm H2O, and
cardiovascular exam reveals normal S1 and S2 without murmurs. Lungs are clear and there is no
peripheral edema.

Her electrocardiogram shows sinus rhythm with 0.5 mm ST depression in leads V4-V6. Laboratory

values include serum creatinine 1.4 mg/dl, troponin I 0.8 ng/ml, hemoglobin 12 g/dl, and N-

terminal pro-B-type natriuretic peptide 202 ng/L. She is chest-pain free on intravenous

nitroglycerin and heparin. Repeat troponin I 6 hours later is 1.1 ng/ml.

Which of the following is the most appropriate management strategy in this patient's care?

a. Pharmacologic nuclear stress test.

b. Coronary angiography.

c. Exercise stress test.

d. Coronary computed tomography (CT) angiography.

e. Transthoracic echocardiogram.

34.

b.
Coronary angiography.
Optimal risk stratification of patients with suspected non-ST elevation acute coronary syndrome

should include risk stratification models such as the Global Registry of Acute Coronary Events

(GRACE) or Thrombolysis in Myocardial Infarction (TIMI) scores, which employ multiple

prognostic factors to help determine management strategies. This patient's GRACE score is 159,

which places her in the high risk category for in-hospital mortality. Therefore, she should be

managed aggressively with an early invasive strategy.

Stress testing in this patient with rising troponin is not appropriate. CT angiography may be

performed in patients with intermediate risk. A normal echocardiogram will not obviate the

need for cardiac catheterization.

35. A 57-year-old woman presents to the emergency room with new-onset chest pressure that

has been escalating for the last 2 hours.

Her medical history is only remarkable for hypertension and obesity. Initial

electrocardiogram demonstrates ST-segment elevation in leads V1-V4 with associated changes

inferiorly.

She is urgently taken to cardiac catheterization where a mid left anterior descending subtotal

occlusion is successfully revascularized using a drug-eluting stent, restoring Thrombolysis in

Myocardial Infarction (TIMI) III flow.

Following percutaneous coronary intervention, a transthoracic echocardiogram demonstrates left

ventricular ejection fraction (LVEF) of 25-30% with severe hypokinesis of the anterior wall LV.

Medical therapy is initiated with atorvastatin, metoprolol, lisinopril, aspirin, clopidogrel, and
eplerenone. Telemetry on hospital day 2 reveals several brief runs of asymptomatic accelerated

idioventricular rhythm.Which of the following do you recommend for subsequent risk stratification
of sudden cardiac death?

a. Reassessment of LVEF at least 40 days post MI.

b. Invasive electrophysiology (EP) testing prior to hospital discharge.


c. Cardiac magnetic resonance imaging (MRI) prior to hospital discharge.

d. Invasive EP testing at least 40 days post MI.

e. Reassessment of LVEF at least 90 days post MI.

35. a.

Reassessment of LVEF at least 40 days post MI.

The correct answer is reassessment of LVEF at least 40 days post MI. Invasive EP testing has no role
in the immediate post MI period in the absence of significant ventricular arrhythmias. The slow

idioventricular rhythm is likely related to myocardial reperfusion and does not require EP testing.

Invasive EP testing >40 days post MI may be considered for moderately decreased LVEF (35-40%) in
patients with evidence of nonsustained ventricular tachycardia detected on noninvasive

monitoring. Cardiac MRI to define scar burden has been found to be useful to predict ventricular

arrhythmias in cardiomyopathic patients. However, cardiac MRI has not yet been integrated into

the guidelines for risk stratification post MI.

The 2013 guideline for the management of ST-elevation myocardial infarction indicates that repeat
assessment of LVEF at least 40 days post MI carries a Class I indication for risk stratification of
sudden cardiac death. Primary prevention implantable cardioverter-defibrillator (ICD) implantation
during the infarct-related hospitalization has not been demonstrated to be beneficial (DEFINITE

trial). Allowing for a period of myocardial healing post MI may avert the need for subsequent ICD

implantation.

Greater than 40 days post MI, LVEF should be reassessed. If the LVEF does not improve remains </=

35% with class II or III symptoms, or remains < 30% independent of symptoms, despite guidelines
directed medical therapy, then this patient would become a candidate for pr primary prevention

ICD implant.

36. A 52-year-old man presents to the emergency room after 3 hours of intermittent sharp

stabbing chest pain noted when he was raking leaves. His medical history is significant for

hypertension. He denies tobacco, alcohol, or drug use. His current medications include

hydrochlorothiazide.

His physical exam reveals blood pressure of 148/94 mm Hg and heart rate of 72 bpm. His
cardiovascular exam is unremarkable. His electrocardiogram shows normal sinus rhythm without

ST- or T-wave changes. His chest x-ray shows normal cardiac silhouette and clear lung fields.

Which of the following is the most appropriate next step?

a. Creatine kinase-myocardial band (CK-MB).

b. N-terminal pro-B-type natriuretic peptide (NT-proBNP).

c. Urine toxicology.

d. High-sensitivity C-reactive protein (hs-CRP).

e. Troponin I.

36. e.
Troponin I.
Cardiac-specific troponin should be performed in patients with clinical findings or symptoms that
may be related to potential acute coronary syndrome (ACS) (Class Ia). It should be measured at

presentation and 3-6 hours after symptom onset in all patients with suspected ACS. If it is negative
within the first 6 hours of onset of symptoms, it should be repeated in patients with

electrocardiographic changes and/or intermediate/high risk clinical features. In order to diagnose


myocardial necrosis, it is important to determine both the peak troponin level and the pattern of
change.

CK-MB and myoglobin are not useful for diagnosis of ACS (Class III). NT-proBNP may be
reasonable

for additional prognostic information (Class IIb). Urine toxicology should be performed in patients
with suspected drug use, as this may be a causal contributor to ACS.

37. A 62-year-old Caucasian woman with type 2 diabetes presents to the emergency department

with complaints of constant substernal chest pressure at rest for the last hour, briefly relieved by
nitroglycerin. She is not on antiplatelet therapy. An electrocardiogram shows lateral precordial ST-
segment depressions and T-wave inversions. She has no known drug allergies. The troponin I level

is 2.12 ng/ml.

Which of the following pharmacotherapeutic strategies is the most appropriate next step for this

patient?

a. Ticagrelor 180 mg.

b. Aspirin 325 mg and clopidogrel 600 mg.

c. Immediate eptifibatide bolus followed by infusion as monotherapy.

d. Aspirin 162 mg and prasugrel 60 mg.

e. Clopidogrel 600 mg.

37. b.

Aspirin 325 mg and clopidogrel 600 mg.

This woman is experiencing a non-ST-segment elevation myocardial infarction (NSTEMI).

Clopidogrel and ticagrelor at the described loading and maintenance doses are both reasonable

therapies for NSTEMI. However, they have been studied in conjunction with and should be
administered along with aspirin. Prasugrel may be given to patients undergoing percutaneous

coronary intervention (PCI) in the setting of NSTEMI, but is not guideline-recommended to be

administered until the anatomy is known and it is certain that the patient will be underoing PCI. In the
ACCOAST trial, treatment with prasugrel in NSTEMI before PCI did not reduce ischemic events

and was associated with more bleeding. The best answer is aspirin in conjunction with clopidogrel.

Glycoprotein IIb/IIIa antagonists are only administered on top of oral antiplatelet therapy.

38. A 40-year-old man with a history of type 2 diabetes is seen in the office. His blood pressure is
123/75 mm Hg. Labs include a total cholesterol of 130 mg/dl and high-density lipoprotein

cholesterol of 40 mg/dl. He is a lifelong nonsmoker. He is active, but overweight with a body mass
index of 29 kg/m2. He is interested in reducing his risk of developing complications of

atherosclerotic cardiovascular disease (ASCVD), but wants to be sure any new medications are

worth the potential side effects.

Which of the following do you recommend to reduce the risk of ASCVD in this patient?

a. High-intensity statin therapy.

b. Dietary modifications alone.

c. Fenofibrate.

d. Moderate-intensity statin therapy.

e. Ezetimibe.

38. d.

Moderate-intensity statin therapy.

Using the ASCVD risk calculator (estimator), this patient has a 10-year calculated risk of 1.2% and a
50% lifetime risk. Although primary prevention is reserved for nondiabetic patients with a 10-year
risk ≥7.5%, the fact that this patient has diabetes means that moderate-intensity statin therapy is
recommended (Class I, Level of Evidence A). High-intensity statin therapy would be indicated in this
diabetic patient if the 10-year risk was ≥7.5%.y.

39. A 66-year-old man is seen in clinic for follow-up after hospitalization for an acute coronary
syndrome. Current medications include atorvastatin 80 mg, ticagrelor 90 mg bid, aspirin 81 mg,

metoprolol 25 mg bid, and lisinopril 10 mg daily. He smoked one-half pack of cigarettes daily for the
past 40 years.
His current lipid panel includes total cholesterol 150 mg/dl, triglycerides 199 mg/dl, low-density
lipoprotein 61 mg/dl, and high-density lipoprotein 49 mg/dl. Hemoglobin A1c is 6.0%.

Which of the following steps will best reduce his risk of future myocardial infarction (MI)?

a. Ezetimibe.

b. Omega-3 fatty acids.

c. Varenicline.

d. Coenzyme Q10.

e. Metformin.

39. c.
Varenicline.
Lifestyle modification including smoking cessation can result in significant improvement in

atherosclerotic cardiovascular risk. A meta-analysis of smoking cessation in patients with MI found it


was associated with a 50% reduction in cardiovascular risk. Therefore, varenicline is the best answer,
because it has been proven to be efficacious for smoking cessation. Ezetimibe in addition to statin was
associated with an approximately 2% absolute risk reduction (number needed to treat

= 50) in atherosclerotic cardiovascular disease events in the IMPROVE-IT trial; given his lipid panel,
this would not likely be as large a benefit as smoking cessation. Adding metformin is not indicated
with a hemoglobin A1c of 6% and would not be expected to reduce his MI risk. Coenzyme Q10 and

omega-3 fatty acids have not been shown to reduce global cardiovascular risk.

40. A 63-year-old woman with a history of hypertension presents with an acute coronary

syndrome. Coronary angiography demonstrates a 90% lesion in her posterior descending artery.

She subsequently undergoes stent placement.

Labs include peak troponin of 1.2 ng/ml and hemoglobin A1c of 5.6%. Fasting lipid profile includes
total cholesterol of 160 mg/dl, high-density lipoprotein (HDL) of 38 mg/dl, low-density lipoprotein
(LDL) of 67 mg/dl, and triglyceride (TG) of 240 mg/dl.

Which of the following is the most appropriate lipid management strategy?

a. Fenofibrate 145 mg qd.

b. Fish oil 2 g qd.

c. Ezetimibe 10 mg qd.

d. Niacin 500 mg qd.

e. Atorvastatin 80 mg qd.

40. d.
Niacin 500 mg qd.
It is important to obtain a fasting lipid profile on all patients presenting with acute coronary

syndrome. High-intensity statin therapy is indicated for all patients ages <75 years with

atherosclerotic cardiovascular disease (ASCVD), even with a "normal" LDL. Despite low HDL and
elevated TG, initial therapy with statin is preferred over the other options.

When added to statin monotherapy, the addition of niacin or fenofibrate has not been shown to

reduce CV events despite improvement in lipid profile. While ezetimibe as add-on therapy to statin
reduced CV events in the IMPROVE-IT trial, it should not be used as monotherapy in a patient who

is statin naive. To date, fish oil has not been definitively shown to reduce CV events in patients with
ASCVD and should not be used over statin therapy.

41. You are counseling a 58-year-old man prior to discharge from the hospital following an acute
myocardial infarction. He received primary percutaneous coronary intervention with a drug-eluting
stent to the left anterior descending artery. Left ventricular ejection fraction is 55%. He had no known
medical history prior to admission. His current medications include atorvastatin 80 mg,

lisinopril 5 mg, metoprolol succinate 25 mg, aspirin 81 mg, and ticagrelor 90 mg bid.

In addition to reviewing the importance of taking all his medications, which of the following

recommendations is the most appropriate?

a. Limit total calories to <2000 per day.

b. Reduce saturated fats, trans fatty acid, and cholesterol intake.

c. Recommend antioxidant vitamin supplements.

d. Increase monounsaturated fat intake and reduce cholesterol intake.

e. Limit sodium intake to <2000 mg per day.

41. b.
Niacin 500 mg qd.
Dietary modification is recommended for all patients with acute coronary syndrome (ACS).

Reducing saturated fats to <7% of total calories, eliminating or reducing trans fatty acids, and
limiting dietary cholesterol to <200 mg daily are all likely to improve cardiovascular health.

Limiting total calories alone is not specific enough to reduce saturated fat intake. Sodium

restriction to <2000 mg/day is not indicated, as he does not have left ventricular systolic

dysfunction or heart failure, although lowering sodium intake by 1000 mg/day from baseline or to

approximately 2400 mg/day with the DASH diet may reduce hypertension.

Antioxidants, including vitamin E, vitamin C, and beta carotene, have not been shown to confer any
benefit in secondary prevention of ACS. Increased monounsaturated fat alone will not ensure a

reduction in dietary saturated fat and trans fatty acid intake.

42. A 72-year-old man with persistent atrial fibrillation presents to the emergency room with a

non–ST-segment elevation myocardial infarction. He has a history of hypertension and diabetes.

The patient undergoes angiography, and a drug-eluting stent is placed in the left circumflex to treat a
hazy area of acute plaque rupture.

Which of the following is the best maintenance therapy for this patient?

a. Aspirin 81 mg, clopidogrel 75 mg, and oral anticoagulation.

b. Aspirin 325 mg.

c. Aspirin 325 mg and clopidogrel 75 mg therapy.

d. Clopidogrel 75 mg.

e. Aspirin 325 mg and oral anticoagulation.

42. a.

Aspirin 81 mg, clopidogrel 75 mg, and oral anticoagulation.

In the presence of atrial fibrillation and an acute myocardial infarction, the addition of an oral
anticoagulant remains controversial following percutaneous coronary intervention. The addition of
an oral anticoagulation should be guided by use of the CHA2DS2-VASc score. Individualization can

also be determined by the HAS-BLED scoring system for risk of bleeding.


43. A 70-year-old man on the trauma service is being evaluated because of a positive troponin.

He was in a motor vehicle accident yesterday and fractured his femur. He underwent surgical open

reduction and internal fixation this morning. He has diabetes and hypertension, but no known

coronary artery disease. There was no steering wheel contact; he was wearing his seat belt. He has no
symptoms.

His examination is normal except for pallor and his now casted left leg. His electrocardiogram

shows sinus rhythm and nonspecific T-wave flattening over the precordial leads. His laboratory

tests are notable for a hemoglobin of 7.5 mg/dl. The troponin is 1.2 ng/dl.

Which of the following is the most appropriate next step?

a. Enoxaparin 40 mg daily.

b. Aspirin 325 mg daily.

c. Clopidogrel 300 mg loading dose.

d. Heparin 60 U/kg bolus.

e. Ticagrelor 180 mg loading dose.

43. a.
Enoxaparin 40 mg daily.
This patient has an asymptomatic rise in his troponin in the absence of symptoms and nonspecific

changes on his electrocardiogram. Moreover, he has undergone significant trauma and is very

anemic. The low-grade troponin is most likely due to demand ischemia. The clinical scenario is not
consistent with an acute coronary syndrome and therefore does not require treatment for an acute

coronary syndrome (e.g., antiplatelet agents, intravenous antithrombins). His greatest risk is deep
venous thrombosis, and he should receive deep vein thrombosis prophylaxis (e.g., enoxaparin).

44. A 50-year-old woman presents with chest pain. It occurs with exertion and is relieved by rest.

Her history is notable for hypertension; her father died suddenly at 55 years of age of unknown

causes. She completes 6 minutes of a Bruce protocol stress test with myocardial perfusion (MIBI)

that demonstrates 2 mm ST-depressions in the lateral leads and a reversible defect in the

inferolateral wall. Her blood pressure and heart rate responses are normal. She undergoes

coronary angiography that is notable for a left dominant coronary circulation with a 70% mid left
anterior descending (LAD) lesion and luminal irregularities of the first obtuse marginal branch.

Fractional flow reserve of the LAD lesion is 0.85.

Which of the following is the most likely cause of her chest pain?

a. Coronary vasospasm.

b. 70% mid LAD lesion.

c. Pericarditis.

d. Coronary dissection.

e. Microvascular angina.

44. e.
Microvascular angina.
Coronary microvascular dysfunction is an important prognostic factor in a wide range of diseases.

In this case, microvascular angina is the manifestation of her atherosclerotic coronary artery

disease. Evidence for ischemia includes her electrocardiographic ST depressions and stress MIBI

changes. Pericarditis would not produce the ST changes or the stress MIBI changes. Coronary

vasospasm generally is not provokable with exercise stress testing and typically occurs in the early
morning hours; it classically involves the epicardial coronary arteries, although microvascular

spasm has been speculated. Coronary dissection usually presents as a myocardial infarction, and

the diagnosis is generally apparent on coronary angiography, although intravascular ultrasound and
ocular coherence tomography may be required for definitive diagnosis.

45. A 45-year-old woman presents to your office with a 6-month history of chest pain. She

reports episodes of chest pain lasting 10-15 minutes that occur with exertion. The pain radiates to her
left shoulder and she becomes short of breath. She has no family history of coronary artery

disease and she has smoked for 10 years. She does not have a history of diabetes or hypertension.

Her physical examination is unremarkable, except for a body mass index of 32. A 12-lead

electrocardiogram (ECG) is normal. A previous cardiologist had performed a treadmill stress test,
during which she developed chest pain and a 2 mm ST-depression in leads V5 and V6. She undergoes

diagnostic cardiac catheterization, and her coronary arteries are free of stenosis except for a 20%

lesion in the circumflex. Echocardiogram shows normal left ventricular and right ventricular (RV)
function and normal RV systolic pressure.

What is the most likely cause of her symptoms?

a. Gastroesophageal reflux disease (GERD).

b. Stress-induced cardiomyopathy.

c. Chronic thromboembolic pulmonary hypertension (CTEPH).

d. Pericarditis.

e. Coronary microvascular dysfunction.

45. e.
Gastroesophageal reflux disease (GERD).

Her history is consistent with angina. Although she does not have any significant coronary artery
stenosis expected to lead to flow limitation, coronary microvascular dysfunction is the most likely
cause of her symptoms. This syndrome occurs in women more than men, can present with classical

angina, and can be associated with ST depression provoked by exercise, and it occurs in the

absence of significant epicardial coronary stenoses.

CTEPH is not typically associated with anginal type symptoms or with exercise-induced ischemia.

Stress-induced cardiomyopathy (Takotsubo) results in wall motion abnormalities on

echocardiography, usually apical ballooning. There is no evidence of pericarditis, because her ECG

is normal and chest pain is only exertional. While GERD is a common cause of chest pain, it would
not result in exercise-induced symptoms or ECG changes.

46. A 50-year-old woman presents to your office for a second opinion. She reports a 3-year history
of exertional chest discomfort. Her previous cardiologist had performed a treadmill stress test during
which she developed a 1.5 mm ST-depression in 3 leads, after which he recommended

the patient undergo a cardiac catheterization. Her coronary arteries had no epicardial stenoses and the
cardiologist attributed the chest pain to anxiety. The patient continues to have exertional chest pain,
and she is willing to undergo additional noninvasive diagnostic testing, because she remains
concerned about this symptom. Her physical examination is unremarkable.

Which of the following noninvasive diagnostic tests do you recommend?

a. Computed tomography (CT) angiography of the coronary arteries.

b. Exercise thallium.

c. Resting transthoracic echocardiogram.

d. Assessment of coronary artery calcium by CT.

e. Repeat stress treadmill with electrocardiogram (ECG) monitoring.

46. b.
Exercise thallium.
The patient may be presenting with coronary microvascular dysfunction. Patients with this

condition can develop ischemia in the absence of significant epicardial coronary artery stenosis.

Such patients may have a reversible perfusion defect as assessed by stress thallium perfusion

study, though absence of this finding does not exclude that diagnosis. There would be little utility in
repeating the treadmill test with ECG monitoring, because this was already positive. Neither CT

angiography or assessment of coronary artery calcium would be helpful given the recent

demonstration of no significant stenosis at left heart catheterization. A resting transthoracic

echocardiogram would not establish the diagnosis of coronary microvascular dysfunction.

47. A 48-year-old woman presents with exertional chest pain with associated dyspnea. Following

a stress perfusion study showing a reversible defect consistent with ischemia, she undergoes

cardiac catheterization, including assessment of hemodynamics. Her pulmonary artery pressure

(PAP) is 30/15/22 mm Hg and cardiac index is 2.5 L/min/m2. Coronary angiography reveals no

significant epicardial stenoses. The ratio of the coronary flow following administration of

adenosine, to that at baseline, is 2. A catheter is placed into her left ventricle, and the left

ventricular end-diastolic pressure (LVEDP) is 12 mm Hg.

Which of the following diagnoses would be most consistent with these findings?

a. Coronary microvascular dysfunction.

b. Noncardiac chest pain.

c. Pulmonary embolism.

d. Heart failure.

e. Pulmonary hypertension.

47. a.
Coronary microvascular dysfunction.
A coronary flow reserve (CFR) of 2.5 is abnormal and consistent with coronary microvascular

dysfunction. Her LVEDP is low, which is not consistent with heart failure. Her mean PAP is 25 mm

Hg, so she does not have pulmonary hypertension. While we cannot exclude pulmonary embolism

to be the cause of her symptoms, she does not have associated pulmonary hypertension. The

finding of a low CFR makes coronary microvascular dysfunction a more likely explanation. The

patient has abnormal CFR and a positive stress test, making noncardiac chest pain less likely.

48. You are asked to consult on a 55-year-old man in the emergency department with chest pain.

He describes a dull chest discomfort that started while he was climbing the attic stairs to put away
some boxes about 2 hours ago. He was sweating and breathing heavily from the work when the

pain started. The discomfort became so intense that he thought he was going to pass out and

gradually resolved over 10 minutes when he sat down to catch his breath. Medical history includes
hypertension managed with amlodipine and gastroesophageal reflux disease managed with

omeprazole.

His electrocardiogram is shown in Figure 1. Blood work drawn on arrival in the emergency

department included chemistries, which were normal, and cardiac troponin I, which was

undetectable.

Which of the following laboratory tests should be obtained within the next 6 hours?
a. Creatine kinase-myocardial band.

b. High-sensitivity cardiac troponin I.

c. Heart-type fatty acid binding protein.

d. Myoglobin.

e. Lactate dehydrogenase.

48.

b.

High-sensitivity cardiac troponin I.

Injury to cardiac myocytes results in loss of cell membrane integrity and subsequent diffusion of
intracellular proteins into the extracellular space, where they are detectable by laboratory

assays on plasma. These biomarkers of myocyte injury include myoglobin (present in all muscle
cells), lactate dehydrogenase (present in all muscle cells), creatine kinase (present in all muscle cells
but with subtypes that are more prevalent in cardiac muscle cells), heart-type fatty acid

binding protein (a subtype of a low molecular weight protein specific to cardiac muscle), and

cardiac troponin I (present only in cardiac muscle cells). Troponin T may be expressed by some

skeletal muscle cells, but in most clinical settings will have similar specificity to troponin I.

The high specificity of cardiac troponin for cardiac muscle injury and predictable kinetics make it the
preferred biomarker to confirm or exclude the diagnosis of myocardial infarction. Cardiac

troponins generally will be detectable first in the blood 2-3 hours after the event in 80% of

patients and within 6 hours of the event in essentially all patients. Serial measurements, as in

this patient with undetectable troponin immediately after the event, are therefore critical for

ruling out the diagnosis of myocardial infarction and for ruling in the diagnosis by demonstrating the
characteristic rise and fall of troponin in the blood.

49. A 60-year-old man with a 60 pack-year history of smoking presents to the emergency

department with 2 hours of crushing substernal chest pain. His electrocardiogram shows 3 mm ST

elevation in the anterior leads. He is taken urgently to the cardiac catheterization laboratory.

Which of the following strategies would require additional antithrombotic therapy during primary

percutaenous coronary intervention (PCI)?

a. Unfractionated heparin.

b. Bivalirudin.

c. Fondaparinux.

d.

Unfractionated heparin with eptifibatide.

e.

Unfractionated heparin with abciximab.

49. c.
Fondaparinux.
Fondaparinux is not recommended as the sole anticoagulant during primary PCI due to an

increased risk of catheter thrombosis (Class III recommendation). Unfractionated heparin with or

without concomitant intravenous glycoprotein IIb/IIIa therapy (including abciximab and

eptifibatide), and bivalirudin, are acceptable anticoagulant therapies during primary PCI.

50. A 31-year-old man with no significant medical history presents to the emergency department

with sudden-onset severe substernal chest pain that began 15 minutes prior to presentation. His

friend admits that they had been smoking marijuana that was laced with cocaine. The patient

appears somewhat disheveled and irritable. His pupils are dilated, heart rate is 105 bpm, and blood
pressure is 163/106. He was started on nitroglycerin, but has persistent chest pain. An

electrocardiogram shows ST elevations in leads V3-V6.

Which of the following medications would be appropriate to administer?

a. Flumazenil.

b. Lorazepam.

c. Naloxone.

d. Metoprolol.

e. Dabigatran.

50. b.
Lorazepam.
Benzodiazepines with or without nitroglycerin may be used to manage hypertension and

tachycardia in patients who present with non-ST elevation acute coronary syndromes (NSTE-ACS)

and signs of acute cocaine or methamphetamine intoxication. For STE-ACS, lorazepam would be

the most reasonable choice of the options listed, although the primary focus would be on

emergent coronary angiography and percutaneous coronary intervention, if indicated.

Per the NSTE-ACS guidelines, "Beta-blockers should not be administered to patients with ACS with a
recent history of cocaine or methamphetamine use who demonstrate signs of acute intoxication

due to the risk of potentiating coronary spasm." Naloxone is used to treat opioid overdose and
flumazenil is used to treat benzodiazepine overdose, so neither would be appropriate here.

Dabigatran is not indicated for the management of ACS.

51. A 67-year-old woman with history of diabetes and asthma is admitted to the cardiac intensive care
unit after percutaneous intervention with drug-eluting stent placement for an ST-elevation

myocardial infarction (STEMI) involving the right coronary artery. She has not had any recurrent

chest discomfort since her procedure. Her vital signs are temperature 37.5°C, heart rate 116 bpm,
blood pressure 153/73 mm Hg, respiratory rate 24 breaths/minute, and oxygen saturation 96%.

Physical examination is notable for an irregularly irregular heart rhythm, no jugular venous

pressure, and moderate expiratory wheezing. Electrocardiogram demonstrates atrial fibrillation

with a rapid ventricular response, but isoelectric ST segments and no Q waves. Her medications

include aspirin, clopidogrel, and atorvastatin.

Which of the following would be the next best medication to consider?

a. Spironolactone.

b. Metoprolol.

c. Amiodarone.

d. Diltiazem.

e. Nitroglycerin.

51. d.
Diltiazem.
Certain calcium channel blockers may be useful for the treatment of angina, hypertension, and

atrial tachyarrhythmias in select patients after STEMI when beta-blockers (or nitrates) are

ineffective, not tolerated, or contraindicated and in patients who have no signs of congestive heart
failure, left ventricular dysfunction, or atrioventricular block. In this patient with recent STEMI, atrial
fibrillation with rapid ventricular response, and elevated blood pressure, diltiazem would be a good
choice to slow her ventricular rate and help control blood pressure. There is no evidence of active
ischemia, and thus immediate electrical cardioversion is not necessary.

Metoprolol is contraindicated given the history of asthma and evidence of active bronchospasm.

The duration of atrial fibrillation is unknown, thus amiodarone would not be a good choice at this
point, as it also can cause cardioversion and other attempts at rate control have not been made.

Nitroglycerin is not ideal in the setting of tachycardia and would not help to control heart rate.

Aldosterone antagonists are recommended for patients with STEMI who are already receiving an

angiotensin-converting enzyme inhibitor and beta-blocker and who have a left ventricular ejection
fraction ≤40% and either symptomatic heart failure or diabetes. This patient would not qualify for an
aldosterone antagonist at this time.

52. A 51-year-old woman is admitted to the cardiac intensive care unit (ICU) immediately

following primary percutaneous coronary intervention for an anterior wall myocardial infarction

(MI). She was intubated for respiratory distress en route to the hospital and no history is available.

Current vital signs are pulse 108 bpm, respiratory rate 12 breaths/minute with 100% fraction of

inspired oxygen from the ventilator, blood pressure 98/70 mm Hg, and oxygen saturation 99%.

Weight is 240 lbs. Physical exam reveals acanthosis of the neck and bilateral rales; the peripheral
pulses are weak.

Lab exam results were remarkable for random glucose 264 mg/dl; there was no anion gap.

Creatinine and other chemistries were within normal limits. An infusion with insulin is started for
control of hyperglycemia.

Which of the following is the most appropriate target of glucose in this patient?

a. 181-220 mg/dl.

b. <80 mg/dl.
c. 221-280 mg/dl.

d. 81-110 mg/dl.

e. 111-180 mg/dl.

52. e.

111-180 mg/dl.

Nearly one-quarter of patients with ST-elevation MI (STEMI) have diabetes, and up to one-third of
those may be diagnosed at the time of hospitalization for STEMI. Diabetic patients have higher

mortality from STEMI, and hyperglycemia is a predictor of early mortality. However, there is a U- or
J-shaped relationship between serum glucose and death, with the risk of mortality similar for either
hypoglycemia (random glucose <70 mg/dl) or hyperglycemia (variously defined as random glucose

>140, >180, or <200 mg/dl).

The NICE-SUGAR trial randomized 6,104 ICU patients to either intensive blood glucose control

(target blood glucose 81-108 mg/dl) or conventional blood glucose control (target blood glucose

<180 mg/dl) and found lower mortality (24.9% vs. 27.5%, p = 0.02) for the conventional glucose
control strategy, with fewer episodes of hypoglycemia (0.5% vs. 6.8%, p < 0.001).

Therefore, "modest" rather than "intensive" glucose control, that is, target <180 mg/dl with avoidance
of hypoglycemia, is the current recommendation for critically ill patients with STEMI.

53. A 68-year-old Caucasian man is seen in the clinic for cardiovascular risk assessment. He has a
history of hypertension and type 2 diabetes . He has never smoked. His medications include

metformin 500 mg bid and lisinopril 10 mg qd. His blood pressure is 130/80 mm Hg. His

examination is otherwise unremarkable. His lipid profile shows total cholesterol 161 mg/dl, high-

density lipoprotein 45 mg/dl, low-density lipoprotein 80 mg/dl, and triglyceride 180 mg/dl. His

hemoglobin A1c is 6.4%.

Which of the following is the most appropriate next step?

a. Start glipizide 5 mg qd.

b. Start fenofibrate 145 mg qd.

c. No additional therapy.

d. Start atorvastatin 40 mg qd.


e. Increase lisinopril to 20 mg qd.

53. d.
Start atorvastatin 40 mg qd.
Initiating statin therapy is appropriate for diabetics 40-75 years of age with low-density lipoprotein
cholesterol >70 mg/dl. In this patient whose 10-year risk of atherosclerotic cardiovascular disease is
11.2% based on pooled cohort equation, high-intensity statin therapy is recommended. In the

ACCORD trial, intensifying glycemic control to target A1c <6% was associated with higher rates of
hypoglycemia and increased mortality and did not significantly reduce major cardiovascular events.

Similarly, the addition of fenofibrate was not associated with significant reduction in rate of

cardiovascular events. This patient's blood pressure is at goal; up-titration of lisinopril is not
indicated.

54. A 74-year-old man with a history of hypertension is brought to the emergency department

with chest pain and an acute inferior wall myocardial infarction. His only medication on admission is
lisinopril 40 mg daily. He undergoes drug-eluting stent placement without complications and is
placed on aspirin 81 mg daily, clopidogrel 75 mg daily, atorvastatin 80 mg daily, and metoprolol 50

mg twice daily. Prior to discharge, he experiences similar chest pain and diaphoresis. His

electrocardiogram shows ST elevation in the inferior leads. Stent thrombosis is diagnosed at

catheterization and successfully remedied.

Which of the following factors may have contributed to development of subacute stent

thrombosis?

a. Inadequate dosing of aspirin.

b. Heparin-induced thrombocytopenia.

c. Nickel allergy.

d. Resistance to clopidogrel.

e. Resistance to aspirin.

54. d.

Resistance to clopidogrel.

On March 12, 2010, the Food and Drug Administration approved a new label for clopidogrel with a
“boxed warning” about the diminished effectiveness of clopidogrel in patients with impaired ability
to convert the drug into its active metabolite. Patients with decreased cytochrome P450 2C19
(CYP2C19) function because of genetic polymorphisms metabolize clopidogrel poorly and have
higher rates of cardiovascular events after acute coronary syndrome and percutaneous coronary
intervention (PCI) than patients with normal CYP2C19 function. The warning also notes that tests are
available to identify patients with genetic polymorphisms and that alternative treatment strategies
should be considered for patients who are poor metabolizers. The clopidogrel boxed warning leaves
the issue of whether to perform CYP2C19 testing up to the individual physician. It does not
specifically require genetic testing or other changes in evaluation or treatment and does not imply
that there are solid evidence-based reasons for such actions. Rather, it serves to inform clinicians of
genetic variations in response to clopidogrel and to emphasize that clinicians should use this
knowledge to make decisions about how to treat individual patients. At the present time, the evidence
base is insufficient to recommend routine genetic testing in patients undergoing PCI. There may be a
potential role for genetic testing for patients undergoing elective high-risk PCI procedures (e.g.,
unprotected left main, bifurcating left main, or last patent coronary artery).

The recommended maintenance dose of aspirin is 70-100 mg following stent implantation. Higher
doses have been shown to increase bleeding risk without reducing major adverse cardiac events. The
prevalence of aspirin resistance is controversial, but has not been linked to stent thrombosis. Heparin-
induced thrombocytopenia increases thrombosis risk in general, but has not been linked to subacute
stent thrombosis. Most modern stents are constructed of cobalt-chromium alloys rather than with
nickel titanium; furthermore, there is no proven association between nickel allergy and stent
thrombosis.

55. A 69-year-old man is hospitalized for primary percutaneous coronary intervention for an ST-

segment elevation myocardial infarction. His medical history includes hypertension and a previous
ischemic stroke. His current medications include aspirin 81 mg, carvedilol 12.5 mg bid, lisinopril 20

mg daily, and atorvastatin 80 mg daily.

Which of the following best explains the benefits of ticagrelor over clopidogrel?

a. Ticagrelor is associated with lower rates of side effects.

b. Ticagrelor is associated with lower stroke risk.

c. Ticagrelor is associated with lower rates of stent thrombosis.

d. Ticagrelor is associated with fewer drug-drug interactions.

e. Ticagrelor is associated with lower bleeding risk.

55. c.

Ticagrelor is associated with lower rates of stent thrombosis.

In the PLATO trial, ticagrelor was associated with lower rates of death, stroke, and stent thrombosis
compared with clopidogrel in acute coronary syndrome patients. However, non-coronary artery

bypass grafting major bleeding rates were higher with ticagrelor compared with clopidogrel,

although overall bleeding rates were similar. Ticagrelor and clopidogrel were associated with a
similar risk of stroke, although there were more hemorrhagic strokes with ticagrelor (0.2% vs.

0.1%). Ticagrelor is metabolized through cytochrome P450 3A and cannot be combined with strong

inducers or inhibitors of that enzyme. Ticagrelor is associated with more side effects, including
dyspnea and bradycardia. Because of increased bleeding when combined with higher doses of

aspirin, ticagrelor should only be combined with aspirin 81 mg.

56. A 65-year-old retired biology professor presents for cardiac catheterization in the setting of a
recently positive stress test. You discuss the procedure in detail and obtain informed consent for
coronary angiography plus percutaneous intervention if needed. He has been reading about

antiplatelet therapy and asks if you will screen him for genetic mutations related to clopidogrel
metabolism.

Which of the following is appropriate routine use of genetic testing to screen patients treated with
clopidogrel after percutaneous coronary intervention?

a. Recommended in those receiving drug-eluting stents only.

b. Recommended, as the evidence suggests that this reduces adverse outcomes.

c. Recommended in those with concomitant atrial fibrillation.

d. Recommended in patients who develop a rash after taking clopidogrel.

e. Not recommended, as the evidence to support this is insufficient.

56. e.

Not recommended, as the evidence to support this is insufficient.

Clopidogrel is a prodrug that requires conversion into an active metabolite. The major enzyme

involved in the generation of the active metabolite is cytochrome P450 2C19 (CYP2C19). Among

several common polymorphisms of CYP2C19 are loss of function mutations that result in a

premature stop codon and a truncated protein. In theory, patients with these polymorphisms who

are taking clopidogrel would have a reduced amount of the active metabolite present for platelet

inhibition.

Because of concerns over reduced activity in these patients, the Food and Drug Administration

issued a boxed warning for clopidogrel in 2010. Multiple studies since then have evaluated the

relationship between the presence of these polymorphisms and adverse outcomes in patients, and
the data are inconclusive. Therefore, the routine use of genetic screening in patients undergoing
percutaneous coronary intervention is not recommended.

Genetic testing for CYP2C19 mutations has nothing to do with allergy or rash to clopidogrel.

Screening is not recommended in patients receiving bare metal or drug-eluting stents. The

presence of atrial fibrillation does not affect the recommendation against screening.

57. A 45-year-old man presents to the emergency department with crushing substernal chest

pain and ST elevation in the inferior leads on the electrocardiogram. His blood pressure is 185/100

mm Hg and his heart rate is 100 bpm. The cardiac catheterization laboratory is activated, aspirin 325
mg is administered orally, and a heparin bolus and drip are started. The patient is anxious and reports
no medical history, no medication allergies, and he takes a multivitamin daily and sildenafil 100 mg
as needed for erectile dysfunction (most recently the evening prior to presentation). A

urinary toxicology screen returns that is positive for cocaine.

Which of the following medications would be appropriate to treat the patient's hypertension, while
waiting for coronary angiography?

a. Sodium nitroprusside.

b. Nitroglycerin.

c. Lorazepam.

d. Nifedipine.

e. Metoprolol.

57. c.
Lorazepam.
Cocaine-induced chest pain may be related to myocardial infarction (MI), aortic dissection, or

anxiety. Among those with MI, this may be due to coronary vasospasm and/or thrombotic

occlusion of one or more coronary vessels. Patients with acute coronary syndromes and severe

hypertension upon presentation should be treated appropriately with aspirin, heparin (assuming

aortic dissection has been ruled out or is considered unlikely), and benzodiazepenes to control

central sympathetic outflow and anxiety.

Beta-blockers are contraindicated in cocaine-related MI, as they may result in increased coronary
vasospasm and decreased coronary blood flow due to unopposed alpha activity. Combined alpha-and
beta-blocker agents, such as labetalol or carvedilol, may be acceptable alternatives. Short-

acting nifedipine is contraindicated in acute MI. While nitrates to control blood pressure and chest
pain would be appropriate in cocaine-associated MI, they are contraindicated in this patient due to
recent use of sildenafil.

58. A 55-year-old man presents to the emergency department with progressive fatigue and

shortness of breath for the past 3 weeks. In the past 2 days he has also developed some lower

extremity edema and feels short of breath in bed at night.

His medical history is notable for deep vein thrombosis (DVT), for which he is currently maintained
on apixaban, and he has not missed any doses. He had a recent echocardiography, demonstrating a

left ventricular ejection fraction of 35%. On examination, his heart rate is 165 bpm and irregularly
irregular. His blood pressure is 105/75 mm Hg, jugular venous pressure is 7 cm, and there are

bibasilar crackles on lung exam.

Cardiac exam is notable for the irregularly irregular rhythm, without appreciable murmur, rub, or
gallop. Extremities are warm with 1+ bilateral edema to his calves. There are no palpable cords or
focal tenderness. Electrocardiogram reveals atrial fibrillation with a rapid ventricular response, and
nonspecific T-wave changes. Laboratory values show a normal troponin, normal basic metabolic

panel, and an elevated brain natriuretic peptide.

Which of the following treatments is contraindicated?

a. Amiodarone.

b. Esmolol.
c. Diltiazem.

d. Dofetilide.

e. Digoxin.

58.

c.
Diltiazem.
This patient is presenting with signs of acute heart failure in the setting of atrial fibrillation with a
rapid ventricular response. Rate control is an important step in management, in addition to

diuresis in light of his signs of volume overload. All of the listed agents may provide rate control and
are indicated for the acute treatment of supraventricular tachycardia. However,

nondihydropyridine calcium channel antagonists are contraindicated in the setting of left

ventricular systolic dysfunction or congestive heart failure, and should be avoided. Cautious use of
beta-blockers is more appropriate, although monitoring for exacerbation of cardiac function is also
warranted when they are used in this setting.

Amiodarone use may be appropriate acutely in this patient, because despite his unknown

duration of atrial fibrillation, he is already anticoagulated for a history of DVT. Therefore, the risk of
stroke from cardioversion is low. Dofetilide is not contraindicated, because he has been

anticoagulated, but will not help with rate control.

59. A 76-year-old man is brought to the emergency room by ambulance with an acute anterior wall
myocardial infarction. He has a history of hypertension and permanent atrial fibrillation for which he
takes metoprolol succinate 100 mg daily and warfarin to maintain an international

normalized ratio (INR) between 2.0 and 3.0. His creatinine clearance is 65 ml/min. A sirolimus-

eluting stent is placed in his proximal left anterior descending coronary artery. He was given aspirin
and clopidogrel at loading doses, and the interventional cardiologist recommends continuing

aspirin at 81 mg daily and clopidogrel 75 mg daily.

Which of the following medical therapy adjustments should also be made?

a. Warfarin should be discontinued.

b. Change warfarin to rivaroxaban 15 mg daily.

c. Change warfarin to dabigatran 75 mg twice daily.

d. Target INR should be 1.5-2.0.

e. Target INR should be 2.0-2.5.

59. e.

Target INR should be 2.0-2.5.

Due to his elevated CHA2DS2-VASc score (4, due to age, hypertension, and vascular disease) and
CHADS2 score (2, due to age and hypertension), anticoagulation with warfarin or a novel oral

anticoagulant is indicated. Setting a target INR below the therapeutic window is not advised.

Setting the target INR at 2.0-2.5 in the lower therapeutic range is recommended to decrease the

risk of bleeding while keeping the risk of stroke and systemic emboli low. There is no current

recommendation to use non-vitamin K oral anticoagulants as part of a triple therapy strategy.

60. A 60-year-old woman with a history of nicotine dependence, hypertension, diabetes, and

right common iliac artery stenting presents to the emergency department with a 3-day history of

mild exertional substernal chest pain relieved by rest. Her brother underwent coronary artery

bypass graft surgery at age 55 years. She takes aspirin 81 mg daily, clopidogrel 75 mg daily,

metoprolol 50 mg twice daily, lisinopril 20 mg daily, and metformin 500 mg twice daily.

In the emergency department, her blood pressure is 120/70 mm Hg and her heart rate is 65 bpm.

Electrocardiogram shows normal sinus rhythm with evidence of left ventricular hypertrophy.

Laboratory values reveal of creatinine 1.2 mg/dl, and the initial troponin is elevated. Sublingual
nitroglycerin completely resolves her chest pain. She is admitted to the inpatient ward overnight, with
a plan for coronary angiography in the morning.

Which of the following medications is most appropriate, in addition to aspirin 324 mg?

a. IV nitroglycerin.

b. IV abciximab.

c. No additional therapy.

d. Intravenous (IV) unfractionated heparin.

e. IV fibrinolytics.

60. d.

Intravenous (IV) unfractionated heparin.

This patient presents with a non-ST-elevation myocardial infarction (NSTEMI). Her TIMI risk score
is

>3. The correct answer is IV unfractionated heparin (Class I, Level of Evidence B).

In patients with NSTEMI, anticoagulation in addition to antiplatelet therapy is recommended for all
patients, if there are no contraindications. Anticoagulant options include subcutaneous enoxaparin 1
mg/kg every 12 hours (reduce dose to 1 mg/kg subcutaneously daily if creatinine clearance is <30

ml/min). IV bivalirudin 0.1 mg/kg loading dose, followed by 0.25 mg/kg/hr can be administered in

patients being managed with an early invasive strategy, that is, within 24 hours (Class IB).

Fondaparinux 2.5 mg subcutaneously daily can be administered either for the duration of the

hospitalization or until percutaneous coronary intervention (PCI) is performed (Class IB). IV

fibrinolytics are contraindicated in NSTEMI (Class III). Glycoprotein IIb/IIIa inhibitors can be

considered, in addition to aspirin and P2Y12 inhibitors in high-risk NSTEMI patients. IV


nitroglycerin is recommended for persistent ischemia, heart failure, or hypertension (Class IB).

61. A 60-year-old man with a history of hypertension and diabetes mellitus was admitted to your

hospital for management of a non–ST-segment elevation myocardial infarction (NSTEMI). His

coronary angiogram revealed three-vessel coronary artery disease. Coronary artery bypass graft

(CABG) surgery was recommended. His medications include aspirin 81 mg daily, clopidogrel 75 mg

daily, enoxaparin subcutaneous 70 mg twice daily (weight is 75 kg and creatinine is normal), and

metoprolol 25 mg orally twice daily. His clopidogrel is to be discontinued 5 days prior to surgery.

Which of the following is the appropriate timing to discontinue his enoxaparin prior to surgery?

a. 36 hours.

b. 9 hours.

c. 6 hours.

d. 1 hour.

e. 12 hours.

61. e.
12 hours.
The correct answer is 12 hours. The half-life for enoxaparin is 4-6 hours. Enoxaparin should be

discontinued 12- 24 hours prior to CABG, and substituted with intravenous unfractionated heparin.

Compared with unfractionated heparin, the use of enoxaparin <12 hours prior to CABG is

associated with lower postoperative hemoglobin and a higher risk of blood transfusion.

62. A 56-year-old man with a history of nicotine dependence, diabetes, and hypertension

presents to the emergency department (ED) with a 10-hour history of severe exertional shoulder

pain. His father underwent three-vessel coronary artery bypass graft (CABG) surgery at age 60

years.

The patient takes aspirin 81 mg daily and metformin 500 mg twice daily. In the ED, his blood

pressure is 100/70 mm Hg, and heart rate 90 bpm. His electrocardiogram (ECG) is shown (Figure

1).

His initial troponin is negative. His pain does not improve with sublingual nitroglycerin. Aspirin 324

mg orally, intravenous (IV) unfractionated heparin, and clopidogrel 600 mg are initiated. Your

facility is a percutaneous coronary intervention (PCI)-capable hospital.

Which of the following is the most appropriate next step?


a. Intra-aortic balloon pump (IABP).

b. IV metoprolol.

c. IV Reteplase.

d. IV tenecteplase.

e. Emergent PCI.

62. e.
Emergent PCI.
The correct answer is emergent PCI. His ECG shows an ST-elevation myocardial infarction (STEMI),

which is defined by the ESC/ACCF/AHA/World Heart Federation Task Force as a new ST elevation at

the J-point in at least two contiguous leads of ≥2 mm in men or ≥1.5 mm in women; in leads V2 -

V3 and/or 1 mm in other contiguous chest or limb leads. New left bundle branch block is considered
an STEMI equivalent, and may interfere with ST-elevation analysis.

Reperfusion therapy should be administered to all eligible patients with STEMI with symptom onset
within the prior 12 hours (Class IA). Primary PCI is the recommended method of reperfusion when

it can be performed in a timely fashion by experienced operators (Class IA). Fibrinolytics, such as
tenecteplase and reteplase, are not correct because the question states that the hospital is PCI-capable.
If the hospital is not PCI-capable, and the patient is greater than 120 minutes away from a PCI-capable
center, then fibrinolytic therapy would be indicated. IABP is not indicated, because the patient is
hemodynamically stable and needs urgent PCI. IV metoprolol is not the best answer,

because the patient has ST elevation and needs urgent reperfusion.

63. A 59-year-old farmer with a history of hypertension and hyperlipidemia presents with crushing
substernal chest discomfort that started approximately 1 hour prior to arrival in the local emergency
department. An electrocardiogram demonstrates 2 mm ST-elevations in V1-V3 with

reciprocal ST depressions in the inferior leads. He continues to have 7/10 chest pain. The closest
percutaneous coronary intervention-(PCI) capable facility is 3 hours away. He has no

contraindications to thrombolysis, so intravenous alteplase is administered. His chest discomfort and


ST elevations resolve, and he remains hemodynamically stable.

Which of the following is the best next step in this patient's management?

a. Emergent coronary angiography.

b. Transfer to facility with cardiac surgery capability.

c. Transfer to facility with PCI capability.

d. Echocardiogram.

e. Admit to the intensive care unit (ICU) for close monitoring.

63. c.

Transfer to facility with PCI capability.

Patients who present to a non–PCI-capable hospital with ST-elevation myocardial infarction


(STEMI) within 12 hours of symptom onset should be immediately transferred to a PCI-capable

hospital for primary PCI, if the time from first medical contact to intervention will be less than 120

minutes. If the anticipated time exceeds this threshold, then thrombolysis within 30 minutes of

arrival should be considered if there are no contraindications.

For patients receiving fibrinolysis, transfer to a PCI-capable facility is reasonable even when they are
hemodynamically stable and have clinical evidence of successful reperfusion therapy, as in this case
(Class IIa). In this patient population, angiography should be performed when logistically

feasible at the receiving hospital (ideally within the first 24 hours), but not within the first 2-3 hours
following fibrinolysis because of increased bleeding risk. This “pharmacoinvasive” approach has

been demonstrated in clinical trials to decrease the risk of recurrent MI and death compared with a
conservative strategy. Although cardiac surgery backup may be necessary, the patient primarily

needs PCI capability. Admission to the ICU and echocardiography are warranted, but should not

delay transfer to a PCI facility.

64. An 82-year-old active, thin woman with a history of remote tobacco use and hypertension

presents with new-onset chest pain. The symptoms began 2 weeks prior, initially occurring with

exertion climbing a flight of stairs, but gradually have became more progressive. On the day of

presentation, she developed an episode of chest discomfort at rest and her husband brought her to the
emergency department for evaluation.

On arrival, she was pain free and her electrocardiogram demonstrated inferolateral T-wave

inversions. She was admitted to the hospital where serial cardiac biomarkers revealed moderately

elevated troponin levels. Her other blood work was normal, aside from a mild chronic anemia.

Based on these findings, an early invasive strategy was planned with cardiac catheterization the

next day.

Which of the following management strategies would offer the best risk profile in this patient?

a. Aspirin, clopidogrel, and enoxaparin.

b. Aspirin, clopidogrel, and unfractionated heparin.

c. Aspirin, prasugrel, and unfractionated heparin.

d. Aspirin, clopidogrel, unfractionated heparin, and glycoprotein (GP) IIb/IIIa receptor inhibitor.
e. Aspirin, prasugrel, unfractionated heparin, and GP IIb/IIIa receptor inhibitor.

64. b.

Aspirin, clopidogrel, and unfractionated heparin.

In patients with non-ST-elevation acute coronary syndrome (NSTE-ACS), bleeding is associated with
worse clinical outcomes. Therefore, the choice of antiplatelet and antithrombotic agents must be

considered carefully. In fact, the ACUITY trial suggested that the mortality risk associated with
significant bleeding may be at least as great as that related to the myocardial infarction.

This patient has multiple bleeding risk factors, including female sex, age, low body weight, and

anemia. Of the available choices, the combintation of aspirin, clopidogrel, and unfractionated

heparin would likely incur the lowest bleeding risk.

All NSTE-ACS patients should receive aspirin. Compared with clopidogrel, prasugrel is associated

with higher rates of bleeding and should also be avoided in patients >75 years old or low body
weight, as well as those with a history of prior cerebrovascular accident. The addition of GP IIb/IIIa
receptor inhibitors upstream of percutaneous intervention would significantly increase the

bleeding risk and has not been shown to have significant clinical benefit. Enoxaparin was associated
with increased major bleeding compared with unfractionated heparin in NSTE-ACS patients treated

with an early invasive strategy in the SYNERGY trial.

65. A 66-year-old male smoker with a history of diabetes mellitus, hyperlipidemia, and

gastroesophageal reflux disease (GERD) presents with central chest discomfort occurring at rest.

The pain began nearly 10 hours before, but he believed it was related to a severe flare of his GERD

and initially avoided seeking medical attention until the following morning. On arrival in the

emergency department, his electrocardiogram (ECG) demonstrated Q waves with ST elevations in

V2-V3. However, he is now free of chest pain. Urgent coronary angiography demonstrated occlusion

of the mid left anterior descending (LAD) without collaterals.

Which of the following strategies is the best next step to reduce the risk of cardiac rupture related to
myocardial infarction?

a. Intra-aortic balloon pump.

b. Glycoprotein (GP) IIb/IIIa receptor inhibitor.


c. Angiotensin-converting enzyme inhibitor (ACEI).

d. Percutaneous coronary intervention.

e. Beta-blocker.

65. d.
Percutaneous coronary intervention.
All patients who present with ST-elevation myocardial infarction (STEMI) within 12 hours of

symptom onset should receive urgent reperfusion therapy. The risk factors for free wall rupture are
female sex, older age, presence of Q waves on ECG indicating completed infarct, LAD involvement,

absence of collaterals, no prior history of MI, hypertension during the acute phase of STEMI, use of
steroids or nonsteroidal anti-inflammatory drugs, and fibrinolytic therapy greater than 14 hours

after symptoms onset.

Reperfusion therapy within 12 hours can decrease this risk. An intra-aortic balloon pump and GP

IIb/IIIa receptor inhibitors do not decrease the risk of free wall rupture. Medical management with
ACEIs and beta-blockers are likely warranted in this patient, but would be instituted after

reperfusion therapy when the patient is hemodynamically stable.

66. A 69-year-old woman with hypertension and hyperlipidemia presents to the emergency room

with acute shortness of breath. Two nights ago she had the onset of flu-like symptoms. She has no
other significant past medical history. On exam she is sitting upright. Her blood pressure is 80/50

mm Hg, her heart rate is 110 bpm, and her oxygen saturation is 88% on 6-liter nasal cannula. Her

jugular venous pressure is 15 cm. Lung exam reveals rales to the mid lung fields bilaterally.

Cardiovascular exam reveals tachycardia without murmur or rub appreciated. Extremities are cool.

Electrocardiogram shows a 1 mm ST elevation in leads II, III, and a ventricular fibrillation.

Which of the following is the most likely explanation of her clinical presentation?

a. Contained free wall rupture.

b. Right ventricular (RV) infarct.

c. Ventricular septal rupture.

d. Papillary muscle rupture.

e. Giant cell myocarditis.

66. d.
Papillary muscle rupture.
Mechanical complications of acute myocardial infarction (MI) that can result in shock include

papillary muscle rupture, ventricular septal rupture, free wall rupture, and RV infarct. In this patient
presenting with an inferior MI and acute pulmonary edema, the most likely etiology would be

papillary muscle rupture and severe mitral regurgitation. In the acute setting, a murmur may not be
heard. This typically occurs from 2 to 7 days post MI. Ventricular septal rupture may result in a
similar presentation, but usually occurs at 3-5 days and occurs more commonly with anterior MIs.

RV infarct typically presents with hypotension but clear lung fields. Giant cell myocarditis would not
be expected to be associated with inferior ST elevations.

67. A 75-year-old woman presents to the emergency department (ED) 4 days after the onset of chest
pain. She lives in a remote area and did not initially seek medical attention. Today her family visited
her and found her barely responsive in her home. Upon arrival to the ED, she is cool, clammy, and
confused.

Her blood pressure is 80/50 mm Hg, heart rate is 110 bpm in sinus rhythm, and oxygen saturation is
90%. She has bibasilar fine rales, and elevated jugular venous pressure and a thrill felt along the left
sternal border. On auscultation, there is an audible grade 4/6 holosystolic murmur and an S3. She has
no edema. Her initial 12-lead electrocardiogram reveals evidence of an acute anterior myocardial
infarction (MI). Her initial cardiac enzymes are abnormal.

An echocardiogram confirms your suspicion of an acute ventricular septal defect (VSD) with anterior
akinesia and left ventricular ejection fraction of 30%. She is taken quickly to the cardiac
catheterization laboratory, where a total occlusion of the proximal left anterior descending is found,
along with a 70% lesion in a large circumflex obtuse marginal. Urgent surgical consultation is
obtained.

Which of the following is the best timing to consider operative repair of the VSD?

a. In more than 28 days.

b. The risk is unrelated to the time period after the infarction.

c. In 15-28 days.

d. In 7-14 days.

e. In less than 7 days.

67.

e.
In less than 7 days.
The correct answer is less than 7 days. In post MI VSD, overall surgical mortality is 42.9%,

according to the Society of Thoracic Surgeons (STS) database. The mortality rate is highly

associated with the timing of surgery post MI. In the STS database, if repair was performed in

less than 7 days post MI, operative mortality was 54.1%, while it was 18.4% if surgery was more

than 7 days post MI.

Additional independent risk factors for mortality include female sex, presence of shock, older

age, preoperative balloon pump, hemodialysis, moderate to severe mitral regurgitation, and

whether the surgery is a re-do cardiac surgery.

The remaining answers are incorrect because the highest mortality is seen early in the post MI

course. The progressively lower odds of mortality is due to subselection of a healthier cohort

and healing of the myocardium with time, which improves surgical outcomes. Percutaneous

closure of the VSD may be considered in high-risk patients who do not require additional surgical
procedures, such as coronary bypass or valve replacement, within the acute time frame.

However, the issue of post-MI friable myocardium still applies to this intervention.

68. A 55-year-old man is shoveling snow when he is seen grabbing his chest and falls to the ground.
Bystander cardiopulmonary resuscitation (CPR) is performed until emergency medicine

services (EMS) arrives. An automated external defibrillator (AED) is attached and shock is advised
and delivered. CPR is continued and there is no return of spontaneous circulation. Awaiting

advanced cardiac life support (ACLS), the AED advises another shock, which is administered and

followed by return of a palpable carotid pulse. He is intubated on scene, after the administration of 5
mg of midazolam. Prior to intubation, there is no witnessed spontaneous limb movement.

In the hospital emergency department, an electrocardiogram (ECG) is performed that

demonstrates 2 mm inferolateral ST-segment depressions. Norepinephrine is initiated for

hypotension and cardiogneic shock. His vital signs are stable and he is cooled to a temperature of 35
degrees Centigrade. On neurologic examination, the pupils are fixed and nonreactive. He has no
spontaneous movements, nor does he withdraw or grimace to noxious stimulus. He is

overbreathing the ventilator. Aspirin is administered through an orogastric tube. A noncontrast


head computed tomography is unremarkable.

Which of the following is the best next step in this patient's management?

a. Immediate coronary angiography.

b. Coronary angiography only if patient can follow commands.

c. Coronary angiography only if cardiac-specific troponin is elevated.

d. Coronary angiography only if ST elevations develop on ECG.

e. Coronary angiography only if pupils become reactive to light.

68.

a.
Immediate coronary angiography.
Out-of-hospital cardiac arrest is often caused by acute coronary syndromes. Patients with ST-

elevation myocardial infarction (STEMI) by ECG after return of spontaneous circulation should go

emergently for coronary angiography. Similarly, patients with non-STEMI or other evidence of

acute coronary syndromes with electrical or hemodynamic instability should go emergently for

coronary angiography.

Recommendations for emergent coronary angiography are independent of neurologic status, as

the neurologic exam is unreliable early post arrest. The administration of sedatives or paralytics
further confounds neurologic prognostication. The absence of spontaneous movement post

arrest should not be interpreted as poor neurologic prognosis. Similarly, pupillary reactivity early
post arrest lacks sensitivity or specificity for a poor neurologic outcome. Neurologic

prognostication is improved at 72 hours post return of spontaneous circulation. If the patient

was cooled, neurologic examination and prognostication should be delayed until 72 hours post

rewarming.

69. During acute coronary syndromes, there is thrombus formation at the site of plaque rupture.

Which of the following patient factors contributes to the formation of thrombus on a ruptured

coronary plaque?

a. A thick fibrous cap.

b. Laminar flow in the culprit vessel.

c. Cigarette smoking.

d. Branch point coronary anatomy.

e. Increased high-density lipoprotein cholesterol (HDL-C).

69. c.
Cigarette smoking.
Cigarette smoking is the correct answer, because it has been shown to correlate with increased

systemic thrombotic propensity in patients. Increased thrombotic propensity is one of three factors in
Virchow’s triad that contributes to thrombus formation in acute plaque rupture. Laminar flow

would not increase the risk of thrombosis. Branch point areas may have increased plaque

formation, but not necessarily plaque rupture. Thickness of the fibrous cap correlates with risk of
plaque rupture, but not directly with thrombus formation. Increased HDL-C does not increase foam

cell formation as opposed to increased low-density lipoprotein cholesterol.

70. A 66-year-old man presents with substernal chest pain suspicious for myocardial ischemia

that began approximately 45 minutes ago, but has now subsided. He has a past history of heavy

smoking and carries the diagnosis of chronic obstructive pulmonary disease, for which he is being
treated with inhalers that he intermittently uses. He has been noncompliant with medications in

the past and has been homeless at various times in his life. He has hypertension and borderline

hyperlipidemia.

His examination reveals normal blood pressure with a heart rate of 102 bpm. His oxygen saturation is
89%. He has reduced breath sounds throughout, but a normal cardiac exam. His initial

electrocardiogram (ECG) reveals sinus tachycardia with left ventricular hypertrophy and secondary
repolarization changes.

Which of the following is the most sensitive and specific laboratory test (or tests) for the diagnosis of
myocardial infarction (MI) in this situation?

a. Cardiac-specific troponin plus creatine kinase-myocardial band (CK-MB).

b. Cardiac-specific troponin plus D-dimer.

c. Cardiac-specific troponin plus N-terminal pro-brain natriuretic peptide (NT-proBNP).

d. Cardiac-specific troponin plus CK total.

e. Cardiac specific troponin only.

70. e.
Cardiac specific troponin only.
In the appropriate clinical context, an MI is diagnosed when there is an elevation of cardiac-specific
biomarkers >99% above the upper reference limit and at least one of the following: 1) symptoms of
ischemia, 2) new or presumed significant ST-segment–T wave (ST–T) changes or new left bundle

branch block (LBBB), 3) development of pathological Q waves in the ECG, 4) imaging evidence of

new loss of viable myocardium, 5) a new regional wall motion abnormality, or 6) identification of an
intracoronary thrombus by angiography or autopsy.

The definition revolves around an elevation in cardiac-specific troponin (the preferred biomarker),
as it is the most sensitive and specific marker of myocardial necrosis. If troponin is unavailable, CK-
MB would be the next most sensitive biomarker and may be used. Creatine phosphokinase (CPK) is

nonspecific and released by skeletal and cardiac muscle breakdown; therefore it is unreliable as a
biomarker of myocardial necrosis. NT-proBNP is a biomarker that reflects myocardial wall stress

and has been shown to be related to prognosis in acute MI, but is not a specific marker of

myocardial necrosis. D-dimer is nonspecific, although very sensitive for thrombus, and does not

reflect myocardial necrosis.

71. A 28-year-old woman who is 32 weeks pregnant presents to the emergency department with

1 hour of chest pressure and dyspnea. She has no prior history of cardiac disease. She is tachycardic
with a heart rate of 120 bpm and a blood pressure of 150/60 mm Hg. Her oxygen saturation is 95%

on 2 liters of nasal cannula supplemental oxygen. She has a 1/6 early peaking systolic murmur

across the precordium. Her jugular venous pressure is elevated. There are faint rales at the bases of
her lungs bilaterally. Her electrocardiogram (ECG) demonstrates sinus tachycardia at 120 bpm with 3
mm of ST-elevation in leads V1-V4.

Which of the following is the most likely etiology for her presentation?

a. Coronary artery dissection.

b. Pneumothorax.

c. Amniotic fluid embolism.

d. Pulmonary embolism.

e. Eclampsia.

71. a.
Coronary artery dissection.
The correct answer is coronary artery dissection. There is a risk for coronary artery dissection

during pregnancy and shortly after delivery in the postpartum period. It is a rare, but well

recognized complication of pregnancy. Her presentation is most consistent with this diagnosis

given the clinical features of chest pain, mild pulmonary edema on examination, and ST-segment

elevation in the left anterior descending territory.

The ECG findings and clinical features are not characteristic for pericarditis. There is a risk of
pulmonary embolism during pregnancy and in the postpartum period due to the hypercoagulability

of pregnancy. However, the features of her presentation are not consistent with this diagnosis.

Likewise, pneumothorax is also on the differential diagnosis for chest pain and dyspnea, but her

physical exam is not consistent with this, because there is no history of trauma and there is no

increased risk of pneumothorax during pregnancy. Amniotic fluid embolus would occur postpartum

and presents with shock. Eclampsia presents with significant hypertension and seizures and would

not cause ST elevation.

72. A 78-year-old man is seen in the office 2 weeks after an anterior ST-segment elevation MI

(STEMI). He underwent primary percutaneous coronary intervention with a drug-eluting stent to

the left anterior descending. His ejection fraction is 40%. His medical history is significant for a
transient ischemic attack (TIA) 5 years ago, chronic kidney disease stage 3b, and peripheral arterial
disease with mild intermittent calf claudication. He was discharged on dual antiplatelet therapy

with ticagrelor 90 mg twice daily and aspirin 81 mg daily.

Which of the following is a contraindication to vorapaxar in this patient?

a. History of cerebrovascular event.

b. Concomitant use of ticagrelor.

c. Chronic kidney disease.

d. Age greater than 75 years.

e. Recent STEMI.
History of cerebrovascular event.

Vorapaxar is an antiplatelet drug that inhibits PAR-1, the thrombin receptor on platelets. The TRA
2°P-TIMI 50 (Thrombin Receptor Antagonist inSecondary Prevention of Atherothrombotic Ischemic

Events Thrombolysis in Myocardial Infarction) trial randomized 26,449 patients who had a history

of MI, ischemic stroke, or peripheral arterial disease to receive vorapaxar (2.5 mg daily) or

matching placebo. Cardiovascular death, MI, or stroke was significantly lower in the vorapaxar

group compared with the placebo group. Moderate or severe bleeding occurred in 4.2% of patients

who received vorapaxar and 2.5% of those who received placebo.

There was an increase in the rate of intracranial hemorrhage in the vorapaxar group. Patients with a
history of stroke had an excessive risk of intracranial hemorrhage. Therefore, the drug is

contraindicated in patients with a history of a TIA or stroke. In the TRA 2°P-TIMI 50 trial, the use
andtype of P2Y12 inhibitor was left to the discretion of the physician. There are no specific data on
the risk of vorapaxar in patients treated with clopidogrel compared with more potent agents. A

small number of patients (0.7%) received prasugrel during the study. Ticagrelor is not

contraindicated, but may alter the efficacy or safety. Because of liver metabolism, vorapaxar can be
used in patients with chronic kidney disease.

73. A 58-year-old man presents to your office to establish cardiovascular (CV) care. He has a

history of myocardial infarction (MI) treated with a drug-eluting stent 2 years ago. At that time, he
had single-vessel disease and a preserved ejection fraction. He was treated with dual antiplatelet
therapy for 12 months and he has been on low-dose aspirin since that time. He has diabetes

requiring oral medication and hypercholesterolemia. He has no history of stroke or bleeding. He is


asymptomatic and has not had recurrent ischemia.

Which of the following do you recommend to decrease the risk of major adverse CV events?

a. Clopidogrel 75 mg daily.

b. Ticagrelor 90 mg twice daily.

c. Prasugrel 10 mg daily.

d. Ticagrelor 60 mg twice daily.

e. Prasugrel 5 mg daily.

73. d.
Ticagrelor 60 mg twice daily.
The PEGASUS-TIMI 54 (Prevention of Cardiovascular Events in Patients with Prior Heart Attack

Using Ticagrelor Compared to Placebo on a Background of Aspirin Thrombolysis in Myocardial

Infarction 54) trial randomly assigned in a 1:1:1 double-blind fashion, 21,162 patients who had an MI
in the past 1 to 3 years to ticagrelor 90 mg twice daily, ticagrelor 60 mg twice daily, or placebo.

Ticagrelor reduced the rate of the primary efficacy endpoint (death, MI, or stroke), with Kaplan-

Meier rates at 3 years of 7.85% for ticagrelor 90 mg twice daily, 7.77% for ticagrelor 60 mg twice
daily, and 9.04% for placebo group. For ticagrelor 90 mg versus placebo, the hazard ratio (HR) was
0.85 and the confidence interval (CI) was 0.75 to 0.96 (p = 0.008), and for ticagrelor 60 mg versus
placebo the HR was 0.84 and the CI was 0.74 to 0.95 (p = 0.004). The rates of TIMI major bleeding
were higher with ticagrelor (2.60% with 90 mg and 2.30% with 60 mg) than with placebo (1.06%).

74. A 74-year-old woman is referred to you with indigestion, belching, and mild exertional

“discomfort,” but no chest pain. She is overweight and has hypertension, diabetes (non-insulin

dependent), and sleep apnea. There is a family history of coronary artery disease (CAD) in an older
brother who had successful percutaneous coronary intervention (PCI).

Her current medications include chlorthalidone 25 mg, lisinopril 40 mg, metformin 500 mg twice

daily, and a multivitamin. Her baseline labs are normal, including her lipid profile and HbA1c. Her
electrocardiogram (ECG) is normal. Physical examination reveals a body mass index of 34.5, heart

rate of 78 bpm and regular, blood pressure of 135/85 mm Hg, and oxygen saturation of 94%. She

has an auscultatory S4 and a soft grade 2/6 aortic flow murmur, but no bruits, and a negative

cardiac exam otherwise.

Which of the following do you recommend as the next step in her work-up?

a. Nuclear exercise stress test.

b. No further cardiac workup necessary.

c. Echocardiogram.

d. Gastrointestinal (GI) referral and possible endoscopy.

e. Coronary computed tomographic angiography (CTA).

74. a.
Nuclear exercise stress test.
While she does not have classic angina, she has multiple features that make her presentation

suggestive of atypical angina. Atypical angina may present as indigestion, belching, and exertional
dyspnea, and it is more common in women, older patients, and those with diabetes.

Her baseline ECG is normal and the first test to be considered should be a stress ECG with imaging.

In women, the use of imaging improves diagnostic accuracy when compared with standardized ECG

stress testing. An echocardiogram to assess left ventricular function might be useful, but her exam
and ECG is normal. GI endoscopy might be a consideration as well, but the exertional component to
her chest “discomfort” warrants further evaluation. Coronary CTA and a calcium score might also

be helpful, because a negative study at her age would suggest there is little chance that CAD is

present. Because angina can be difficult to diagnose in this population, some effort to exclude a
coronary source should be undertaken.

75. A 57-year-old man is referred to you with shortness of breath. He has not been followed

regularly and he says he was told he had a murmur as a teenager, but that he would likely

“outgrow it.” He works in heating and air conditioning and has a fairly physical work environment.

He does not smoke and is on no medication.

Which of the following could be considered an etiology for cardiac-related dyspnea?

a. A left-to-right intracardiac shunt Qp/Qs of 2.1:1.

b. Pulmonary capillary wedge pressure (PCWP) of 11 mm Hg.

c. A distal left circumflex coronary lesion of 60%.

d. Cardiac index of 3.0 L/m2.

e. Mean pulmonary artery pressure of 28 mm Hg during exercise.

75. a.

A left-to-right intracardiac shunt Qp/Qs of 2.1:1.

The correct answer is a left-to-right cardiac shunt with its associated volume overload.

Cardiac-related dyspnea can occur when there is an elevated PCWP with resultant stimulation of

the interstitial J receptors and/or pulmonary edema. Usually there is associated orthopnea or
paroxysmal nocturnal dyspnea (PND), which occurs 2-4 hours after the onset of sleep and subsides

minutes after sitting upright or standing. The PCWP of 11 mm Hg is normal in this situation.

Patients with low cardiac output (cardiac index of 3.0 L/m2) experience dyspnea on exertion and

fatigue in the absence of orthopnea or PND. Dyspnea may also be due to ischemia during exercise,

but it is very unlikely to be associated with the degree of coronary disease that jeopardizes only a
small amount of myocardium as noted.

76. The jugular venous pressure (JVP) reflects the right atrial (RA) pressure. It is a critical

component of the cardiac examination, as an elevation suggests an elevated RA pressure and has

prognostic importance.

Which of the following is correct regarding the assessment of an elevated JVP?

a. It is best measured as the vertical distance above the clavicle in the upright position.

b. It is best measured as the vertical distance between the top of the pulsation and the angle of c. Louis.

d. It can best be assessed with the patient lying at a 20 degree elevation.

e. The degree of elevation cannot be reliably assessed during atrial fibrillation.

f. Either the external or internal jugular veins can be assessed reliably.

76. a. It is best measured as the vertical distance above the clavicle in the upright position.

The height of the JVP is reflective of RA pressure. The best answer is to measure the height of the
JVP in the upright position above the clavicle. Using that as a reference point is effective, because the
RA pressure must be at least 10 cm high for the waveforms to be visible in that position. Thus, any
evidence of waveforms in the upright position means the JVP is elevated.

While either the external or the internal jugular can be used, the jugular vein valves may interfere
with accuracy and the internal jugular vein is preferred. Using the angle of Louis (the sternal

inflection point where the manubrium meets the sternum), a distance >3 cm has been considered

abnormal. However, the distance from the sternal angle and the RA is quite variable among

patients, and this leads to an underestimation of the venous pressure. Atrial fibrillation will affect the
waveforms observed, but the total height of the JVP column will still reflect mean RA pressures.

The height of the JVP cannot be reliably assessed with the patient in a semisupine position.

76. A number of early diastolic heart sounds may be audible in certain patients. These include an S3 in
young patients and in older patients suffering from heart failure, a pericardial knock in those with
constrictive pericarditis, and a mitral opening snap in patients with rheumatic mitral stenosis.

The timing from S2 of each of the early diastolic heart sounds varies, depending on the origin of the
sound.

Which of the following is the correct sequence one could expect these heart sounds to occur if they
were present, from a timing perspective in early diastole?

a. Opening snap, followed by pericardial knock, followed by an S3.

b. Opening snap, followed by an S3, followed by pericardial knock.

c. Pericardial knock, followed by an S3, followed by opening snap.

d. Pericardial knock, followed by opening snap, followed by an S3.

e. S3, followed by an opening snap, followed by a pericardial knock.

77. a.

Opening snap, followed by pericardial knock, followed by an S3.

These early diastolic sounds reflect the hemodynamic changes in early diastole. The opening snap is
a high-pitched sound occurring shortly after S2 and reflects the abrupt opening of a mobile but

stenotic mitral valve. The interval between the S2 and opening snap decreases as the left atrial (LA)
pressure rises (worsening mitral stenosis). It is the first sound heard in early diastole, as it reflects the
mitral valve opening prior to the initiation of left ventricular (LV) filling. The second possible sound
is the pericardial knock. It occurs once the mitral valve is open and is a high-pitched sound reflecting
the impact of the cessation of rapid filling on the LV wall. In constriction, the LV fills rapidly from
the elevated LA pressure, to the point it cannot expand further due to the restriction placed on it from
the constrictive pericarditis.

An S3 is a low-pitched sound that reflects the rapid filling phase of diastolic LV filling and occurs
later than the impact sound from a pericardial knock. An S3 may be normal in children, adolescents,
and young adults, but implies heart failure in adults. It can also be heard in severe mitral

regurgitation. Thus, the correct answer is opening snap, followed by pericardial knock, followed by
an S3.

78. The Valsalva maneuver can help distinguish the origin of some cardiac murmurs and is known

to be abnormal in heart failure. For patients with an intact autonomic nervous system, when the

Valsalva maneuver is performed there are four phases evident from the hemodynamic tracings.

Which of the following occurs during phase 3 of the Valsalva maneuver?

a. The systemic pressure declines and the heart rate (HR) increases.
b. The systemic pressure increases due to increased blood pressure return.

c. The systemic pressure increases with slowing of the HR.

d. The systemic pressure declines in response to a decrease in intrathoracic pressure.

e. The systemic pressure increases transiently due to an increase in intrathoracic pressure.

78. d. The systemic pressure declines in response to a decrease in intrathoracic pressure.

The four phases of the Valsalva maneuver are shown in Figure 1, where the relative HR and aortic

pressure are outlined. In phase 1, with the initiation of the Valsalva, the intrathoracic pressure
increases and blood is forced out of the pulmonary circuit. There is an increase in left ventricular
(LV) filling and stroke volume and the systemic blood pressure rises. In phase 2, with the

maintenance of the Valsalva, the LV filling declines as the increased intrathoracic pressure impedes
venous return and stroke volume drops, resulting in vasoconstriction with a compensatory

tachycardia.

In phase 3, with release of the Valsalva, the aortic pressure declines further in response to the
decreased intrathoracic pressure and decreased left atrial return associated with the increased

aortic volume and the delay before venous return catches up to the LV. In phase 4, the LV preload
increases with the return of the venous flow; stroke volume increases rapidly into the

vasoconstricted aortic system and the blood pressure overshoots and the baroreceptors are

stimulated with a subsequent reduction in the HR. Some have noted that when the ratio of the the

fastest HR (during Valsalva) divided by the slowest HR (upon release) is <1.4, there is evidence for
autonomic impairment.
(Figure 1)

79. The use of the Valsalva maneuver can help differentiate the origin of certain cardiac

murmurs. The four phases of the Valsalva maneuver also may be abnormal in patients with

autonomic dysfunction and in heart failure.

Which of the following summarizes the abnormality in the phases of the Valsalva maneuver that

occurs in heart failure?

a. There is an increase in BP with little change in heart rate (HR) during phase 2.

b. There is a decrease in blood pressure with no change in HR during phase 3.

c. There is an increase in BP and bradycardia during phase 4.

d. There is an increase in blood pressure (BP) and bradycardia during phase 2.

e. There is a decrease in BP with little change in HR during phase 2.


79. d. There is an increase in blood pressure (BP) and bradycardia during phase 2.

There is an increase in BP with little change in HR during phase 2 of the Valsalva maneuver in

patients with significant heart failure. This is shown graphically in Figure 1. As opposed to the
normal phase 4 of the Valsalva, in patients with severe heart failure there is a “square wave”

response with an initial increase in BP (phase 1) that remains elevated during the entire strain

period (phase 2) and returns to resting levels in phase 3. This is thought to be due to maintenance of
the left ventricular (LV) filling during the strain phase, despite the decrease in venous return. This is
presumably due to the increased central blood volume that serves as a reservoir to maintain LV

preload. This can be detected with BP monitoring during Valsalva or by an abnormal response as

assessed by the Korotkoff sounds as shown in Figure 2. In the latter figure, the loss of the

overshoot in BP has been correlated with loss of systolic function and the square root response has
been correlated with an elevated pulmonary capillary wedge.

(Figure 1)
(Figure 2)

80. A 65-year-old woman with hypertension is admitted to a rural hospital with chest pain for 1

hour and an ST-segment elevation myocardial infarction (STEMI). She immediately undergoes

therapy with tenecteplase.

On arrival to the tertiary referral center, she is comfortable without chest pain, her

electrocardiogram is now normal, and she is hemodynamically stable. Her exam is only remarkable

for S4.

Which of the following is the next most appropriate step?

a. Emergent coronary angiography.

b. Coronary computed tomography angiography.

c. Elective coronary angiography.


d. Cardiac magnetic resonance imaging.

80. c.
Elective coronary angiography.
Assessment of clinical reperfusion following lytic therapy for STEMI is essential to guiding the

urgency of subsequent coronary angiography. Resolution of symptoms, improvement of ST-

segment elevation by at least 50%, and reperfusion arrhythmias (e.g., accelerated idioventricular
rhythm) are indicative of epicardial coronary patency. If there is clinical uncertainty about the status
of reperfusion, emergent coronary angiography is warranted (e.g., rescue percutaneous

coronary intervention). Most patients following STEMI treated with lytic therapy will undergo

elective coronary angiography to define the coronary anatomy within a day of presentation (Class

IIa). In high-risk scenarios like shock, heart failure, or recurrent chest pain, urgent coronary

angiography is recommended (Class I).

81. A 49-year-old man with no significant medical history presents with 1 week of intermittent

chest pain that worsened over the 2 hours prior to presentation.

Blood pressure measures 84/43 mm Hg with a pulse of 113 bpm. Oxygen saturation on 2 liters of

nasal cannula is 91%. He is tachycardic with an S3 and rales on lung exam. Initial electrocardiogram
demonstrates ST-segment elevation (STE) in leads V1-V5.

The patient undergoes urgent cardiac catheterization with placement of a drug-eluting stent in the
proximal left anterior descending artery. His blood pressure improves post revascularization and he
is admitted to the coronary care unit. A post-percutaneous coronary intervention echocardiogram

shows severe hypokinesis of the anterior and anterolateral wall. Left ventricular ejection fraction is
25%.

On day 2 of hospitalization, the patient develops dyspnea and the rhythm seen in Figure 1.

Which of the following best describes the incidence and degree of myocardial injury associated

with the rhythm seen in Figure 1 in an anterior as compared with inferior myocardial infarction

(MI)?
a. The incidence during MI is lower, the degree of myocardial injury is lower, and the location of
block is at the AV node.

b. The incidence during MI is greater, the degree of myocardial injury is lower, and the location of
block is at the atrioventricular (AV) node.

c. The incidence during MI is the same, the degree of myocardial injury is greater, and the

location of block is below the AV node.

d. The incidence during MI is lower, the degree of myocardial injury is greater, and the location of
block is below the AV node.

81. d. The incidence during MI is lower, the degree of myocardial injury is greater, and the
location of block is below the AV node.

The rhythm strip reveals the development of complete heart block (CHB) occurring in the setting of
recent anterior wall MI. This is a serious complication of anterior MI, which occurs in roughly 1% of
cases. CHB in anterior MI is associated with poorer short-term outcomes, including survival, as

compared with anterior MI patients who do not develop CHB.

The incidence of CHB in inferior distribution ST-elevation (STEMI) is higher, approximately 4%,
and may be transient. CHB occurring in inferior STEMI patients is associated with a higher location
of AV

block (typically the AV node), whereas in anterior wall MI, the location of block is usually below the
AV node, resulting in higher long-term block.
The degree of myocardial injury in anterior wall STEMI exhibiting CHB is greater than the amount
of myocardial injury observed in inferior STEMI complicated by CHB.

82. A 75-year-old woman with diabetes is admitted to the coronary care unit after a drug-eluting stent
(DES) placement in her mid right coronary artery following an acute inferior wall myocardial
infarction (MI). Earlier, she had received 600 mg of clopidogrel in the emergency department and

had been administered a bolus of bivalirudin followed by an intravenous infusion. She has a history
of a prior ischemic stroke and has been a lifelong asthmatic.

Her electrocardiogram shows sinus rhythm at 50 bpm, and inferior Q waves with almost complete

resolution of her inferior ST elevation. Her body mass index is 20 kg/m2, blood pressure is 148/70

mm Hg, and lungs are clear to auscultation. Cardiac examination reveals a fourth heart sound and a
loud aortic component of the second heart sound.

At discharge, which of the following medications is the most appropriate choice?

a. Ticagrelor.

b. Ticlopidine.

c. Prasugrel.

d. Cangrelor.

e. Clopidogrel.

82. d.
Cangrelor.
The choice of antiplatelet agents following DES placement for acute MI should be guided by specific
patient characteristics outlined by clinical trials. Prasugrel is contraindicated in this patient due to her
size, age, and previous stroke. Ticagrelor is not preferred due to the history of asthma and

bradycardia on exam. Ticlopidine is no longer a first-line choice due to its numerous adverse

effects, including rash, diarrhea, and neutropenia. Cangrelor is only available intravenously.

83. Mr. K is a 76-year-old man who presents to the emergency department. He complains of epigastric
pressure, shortness of breath, nausea, and fatigue, all of which suddenly started about 2

hours earlier. He has been in good health in the past and his history is remarkable only for diabetes
treated with oral hypoglycemic medication. He is clinically stable with a blood pressure of 112/78

mm Hg, heart rate of 75 bpm, S02 of 96%, and temperature of 36.4°C. His electrocardiogram

(ECG) is normal. An initial set of cardiac biomarkers turns out to be negative. After a tablet of
sublingual nitroglycerin, he says that his symptoms are slightly improved.

Which of the following is the most appropriate next step for management?

a. Transesophageal echocardiography.

b. Outpatient stress test.

c. ECG with posterior leads.

d. Ventilation/perfusion scan.

e. Coronary computed tomography angiography (CTA).

83. c.
ECG with posterior leads.
This patient presents with strong pretest probability for an acute ischemic syndrome, although

the ECG is normal. Posterolateral infarctions are often electrocardiographically silent and should be
considered in these situations. Presence of ST-segment elevation in V7-V9 will confirm the diagnosis
of true posterior ST-segment elevation myocardial infarction and the need to perform urgent

reperfusion.

Outpatient stress testing would not be appropriate in a patient with high pretest probability of an acute
ischemic syndrome. Coronary CTA performs best in intermediate-risk patients. The pretest

probability for pulmonary embolism and aortic disease is sufficiently low such that they would not be
appropriate when the most likely diagnosis is still an acute coronary syndrome.

84. Which of the following phrases best describes tenecteplase?

a. Nonantigenic, administered as a single bolus, fibrin specific.

b. Nonantigenic, administered as two boluses 10 minutes apart, fibrin specific.

c. Antigenic, administered as a single bolus, fibrin specific.

d. Antigenic, administered as two boluses 10 minutes apart, not fibrin specific.

e. Nonantigenic, administered as single bolus, not fibrin specific.

84. a.

Nonantigenic, administered as a single bolus, fibrin specific.

Tenecteplase, or TNK, is a fibrin-specific, nonantigenic fibrinolytic. Although its administration is


weight based, its main advantage is that it can be administered as a single bolus. Reteplase, or

Retavase, has limited fibrin specificity. It is given as two boluses 10 minutes apart, but does not have
to be adjusted for weight. Tissue plasminogen activator has moderate fibrin specificity, but is
administered as a drip. Streptokinase has no fibrin specificity and is highly antigenic.

85. A 63-year-old man presents with chief complaints of chest heaviness. He tells you about an

episode in the morning that subsided spontaneously after 10 minutes, and a new episode 2 hours

before presentation that lasted about 30 minutes. Now he is asymptomatic, but his wife insisted on
bringing him to the emergency department. He has been diabetic for the last 15 years and treated

with metformin and glimepiride. He has also been prescribed amlodipine to control his blood

pressure.
His blood pressure is now 145/90 mm Hg, his heart rate is 62 bpm, and creatinine is 1.12 mg/dl.

The physical exam is unremarkable. He smokes about 10 cigarettes per day. His electrocardiogram

is normal and on the first set, his troponin T is 0.06 ng/ml (99th percentile, 0.01 ng/ml). You

calculate a GRACE score of 106 and TIMI score of 3.

Which of the following is the optimal management for this patient?

a. Emergent coronary angiography.

b. Coronary angiography within 24 hours.

c. No further diagnostic testing.

d. Coronary computed tomography angiography.

e. Stress test.

85. b.
Coronary angiography within 24 hours.
The risk profile of this patient does not warrant an emergent catheterization. The 2011 focused

update of the Guidelines for the Management of Patients With Unstable Angina/Non-ST-Elevation

Myocardial Infarction (NSTEMI) considers an early invasive management strategy within 24 hours

reasonable in the case of high-risk criteria or clinical instability. This patient is diabetic and has
positive troponin, both of which favor an invasive approach within 24 hours.

This patient has a confirmed NSTEMI, and a stress test to provoke ischemia in this setting is not
diagnostically useful and is unsafe. Coronary computed tomography angiography is predominantly

useful when there is diagnostic uncertainty and the pretest probability is either low or

intermediate.

86. A 50-year-old man with diabetes mellitus and a history of cigarette smoking underwent

percutaneous coronary intervention of his mid left anterior descending artery 2 months ago. At the
time of his procedure, he was noted to have a total cholesterol of 350 mg/dl, low-density

lipoprotein cholesterol (LDL-C) of 180 mg/dl, high-density lipoprotein cholesterol (HDL-C) of 20

mg/dl, and triglycerides of 280 mg/dl. He was placed on atorvastatin 40 mg daily, and repeat

laboratory tests obtained today show a total cholesterol of 200 mg/dl, LDL-C of 120 mg/dl, HDL-C

of 23 mg/dl, and triglycerides of 250 mg/dl. The patient is compliant with his medical regimen.

Which of the following is the most appropriate next step for this patient?

a. Add niacin.

b. Increase atorvastatin dose to 80 mg daily.

c. Add omega-3 fatty acids.

d. Add ezetimibe.

e. Add gemfibrozil.

86. b.

Increase atorvastatin dose to 80 mg daily.

For individuals with established coronary artery disease and diabetes, an LDL-C reduction of at least
50% is the recommended therapeutic strategy. Although administration of a statin reduced the
patient's LDL-C, since it is not at goal, the statin dose should be increased.

Gemfibrozil is used primarily to treat hypertriglyceridemia, and an increased incidence of myositis is


observed when it is used concomitantly with a statin. The addition of niacin to a statin is

attractive because it may further increase HDL and reduce triglycerides, but it has not been shown to
reduce cardiovascular events. Ezetimibe and omega-3 fatty acids are not indicated without

maximizing statin therapy.

87. Which of the following findings is most useful for establishing or excluding the diagnosis of ST-
segment elevation myocardial infarction (STEMI)?

a. Normal cardiac troponin 1 hour after symptom onset.

b. ECG with 100% ventricular pacing and 3 mm of ST-segment elevation in precordial leads.

c. ECG with 1 mm of ST-segment elevation in leads V2-V3 in an asymptomatic 44-year-old man.

d. Electrocardiogram (ECG) with left bundle branch block (LBBB) and 2 mm of concordant ST-

segment elevation in two leads with a positive QRS deflection.

87. d. Electrocardiogram (ECG) with left bundle branch block (LBBB) and 2 mm of concordant
ST-segment elevation in two leads with a positive QRS deflection.

Neither cardiac troponin nor creatine kinase-myocardial band (CK-MB) are sufficiently sensitive

within the first 1-2 hours after symptom onset to exclude a diagnosis of MI.

The multisociety Task Force for the Universal Definition of MI has defined the diagnostic criteria for
STEMI (at the J point) as follows: ≥2 mm in men or ≥1.5 mm in women in leads V2-V3 and/or ≥1
mm

in at least two other contiguous chest leads or two limb leads. Moreover, particularly in young

men, a multisociety scientific statement recommends a higher threshold of ≥2.5 mm in leads V2-V3.

The Sgarbossa criteria for diagnosis of MI emphasize concordant ST-segment elevation in two leads
with a positive QRS as most indicative of STEMI in the setting of an LBBB. It is challenging to

diagnose STEMI in the setting of ventricular pacing.

88. A 60-year-old businessman has a history of obesity, with a body mass index of 32 kg/m2. His

history includes prior cigarette smoking, hypertension, and hyperlipidemia. Six months ago, he

decided to start climbing three flights of stairs to his office each morning. When performing this
activity, he notes a pressure sensation localized to his substernal region that resolves within 2-3
minutes once he sits down at his desk. He describes this symptom to his physician, who orders an

exercise treadmill test. The results are as follows: exercise duration 8 minutes of the Bruce

protocol; the patient requests to stop because of his chest pressure; exercise electrocardiogram

(ECG) is negative for ischemia.

Based on the ECG stress results, his post-test probability of coronary disease is which of the

following?

a. >90%.

b. 60-90%.

c. 0-30%.

d. 30-60%.

88. a.

>90%.

A 60-year-old man with typical angina has a pretest probability of coronary artery disease (CAD)

that is >90%. According to Bayes Theorem, a negative exercise ECG shifts this probability very little,
and the post-test probability of CAD remains >90%. This patient’s Duke treadmill score can be

calculated as 0: 8 minutes Bruce protocol – 0 (for no ST-segment depression) – 8 (4 x 2, for test-


limiting angina). A Duke treadmill score of zero places a patient into the intermediate-risk category
for cardiac death, where annual cardiac mortality ranges between 1-3%.

89. Ionizing radiation can result in cellular DNA injury. A stochastic injury from medical radiation
exposure results in which of the following consequences?

a. Carcinogenesis.

b. Nausea and vomiting.

c. Skin injury.

d. Hair loss.

e. Cellular death.

89. a.

Carcinogenesis.
Stochastic injuries can occur after a single exposure to the cell. It is therefore not threshold

dependent. The DNA injury that results does not kill the cell, but rather the cell lives on mutated.

This can eventually lead to cancers later in life. A deterministic injury results when the cell dies from
the DNA damage. If enough cells of an organ die, then the organ becomes dysfunctional. This

is seen most often in skin damage, but can result in hair loss, or if extensive (such as acute radiation
injury), can result in sloughing of the gastrointestinal tract. Deterministic injury only occurs after a
threshold level of radiation exposure has occurred.

90. A 65-year-old active woman has a history of type 2 diabetes and uncomplicated inferior wall

myocardial infarction treated with lytic therapy therapy 10 years ago. She has experienced a dozen
episodes of interscapular back and chest discomfort over the past month. This discomfort usually

occurs while gardening. She has tried nitroglycerin twice and thinks that it has helped. Her

electrocardiogram (ECG) shows old Q waves in the inferior leads and is otherwise normal.

Which of the following is the procedure of choice for this woman?

a. Standard treadmill test.

b. Coronary angiogram.

c. Regadenoson single-photon emission computed tomography (SPECT).

d. Dobutamine ECG.

90. a.

Standard treadmill test.

This patient is describing atypical angina and is therefore a candidate for noninvasive assessment,
rather than proceeding directly to coronary angiography. The multisociety stable ischemic heart

disease guideline recommends consideration of three criteria for deciding between the standard

treadmill test and stress imaging: a history of prior coronary revascularization, ability to adequately
exercise, and findings on a resting ECG.

ECG abnormalities that warrant proceeding with stress imaging instead of the standard treadmill

test include pre-excitation, left bundle branch block, paced ventricular rhythm, or >1 mm of ST-
segment depression. The presence of Q waves is not an indication to proceed with stress imaging

instead of the standard exercise treadmill test. Although this patient has risk factors that place her at a
higher risk (i.e., history of established coronary artery disease and the presence of diabetes), the
standard treadmill test would still be the procedure of choice in this setting, according to the
guideline.

91. Which of the following procedures results in the highest potential effective radiation dose

being delivered to the patient?

a. Technetium-99m sestamibi rest-stress scan.

b. Thallium-201 rest-stress scan.

c. Diagnostic coronary angiogram.

d. CT coronary angiogram.

e. Coronary calcium computed tomography (CT).

91. b.

Thallium-201 rest-stress scan.

Diagnostic coronary angiography, technectium-99m scans, and CT angiography (if the newest

techniques are applied) result in similar effective dose exposure to patients. Increased dose occurs
with percutaneous coronary intervention because of the time, angulated views, the use of

magnified views, etc. Thallium-201 has a long half-life and slow excretion, resulting in the greatest
exposure compared to the other agents.

92. A 65-year-old man presents for cardiac computed tomography (CT) due to a myocardial

perfusion study in which the imaging and electrocardiogram findings are discordant.

Which of the following presenting conditions should lead to cancellation of the cardiac CT order?

a. Use of sildenafil 24 hours ago.

b. Body weight 42 kg/m2.

c. Sinus bradycardia at 40 bpm.

d. Atrial fibrillation, rate 80-110 bpm.

92. d.

Atrial fibrillation, rate 80-110 bpm.

Atrial fibrillation due to the extreme R-R variability causes motion artifacts on cardiac CT that lead to
an unacceptable rate of nondiagnostic examinations. Although some scan modes can enable
moderately accurate cardiac CT angiography in the setting of atrial fibrillation (e.g., helical CT

without tube current modulation or single-beat wide-area detector CT), atrial fibrillation represents a
strong contraindication to the test. Bradycardia is not a contraindication. Increased body mass index
can lead to noisy cardiac CT images and will necessitate higher radiation exposure. Sildenafil use
contraindicates use of nitroglycerin, but scanning can proceed without its use.

93. You are called to see a patient in the emergency room 2 days after percutaneous coronary

intervention, who has low-grade fever, nausea, diffuse pruritus, and a maculopapular rash on the

trunk and upper extremities. The patient was initiated on clopidogrel after the procedure.

Which of the following is your next step in management of this patient?

a. Intravenous (IV) immunoglobulin (IG).

b. IV steroids.

c. Diphenhydramine therapy.

d. Stop the clopidogrel.

93. c.
Diphenhydramine therapy.
Late contrast reactions are T-cell mediated and are more common with nonionic dimers. If a

patient has a prior history of such a reaction, choosing a contrast other than a nonionic dimer, such as
a nonionic monomer (iopamidol), is advised. This is a self-limiting condition and symptomatic

treatment is all that is required. This is not a reaction to the clopidogrel and thus should not be
discontinued.

94. In a patient who receives enoxaparin with the electrocardiogram shown in Figure 1, which of

the following conditions would indicate antifactor Xa (anti-Xa) level monitoring?

a. Liver disease.

b. Concomitant warfarin

therapy.
c. Renal insufficiency.

d. Diabetes.

e. Stroke.

94. c.
Renal insufficiency.
The patient’s degree of renal dysfunction warrants anti-Xa monitoring. According to the

current American Heart Association/American College of Cardiology guideline, low molecular

weight heparin agents should be avoided in patients with severe renal dysfunction (creatinine >2.5

in men, >2.0 in women). In patients with moderate renal dysfunction, dose adjustment should be
considered and anti-Xa monitoring may be necessary.

Enoxaparin is not hepatically metabolized; liver function testing abnormalities do not necessitate dose
adjustment or anti-Xa monitoring. Dose adjustment and anti-Xa monitoring are indicated in

patients with a weight exceeding 100 kg, but at weights below that standard dosing is appropriate and
anti-Xa monitoring is not necessary, independent of body mass index.

While the use of chronic warfarin causes concern for the possibility of increased bleeding risk in the
setting of enoxaparin, it does not affect dosing or the role of anti-Xa monitoring. Management of
enoxaparin in the cardiac catheterization laboratory is based on the time from the last dose

administered, not anti-Xa level.

95. A 79-year-old man presents to his local community hospital with substernal chest pain lasting 3
hours. His resting electrocardiogram (ECG) shows 3 mm ST elevation across the anterior

precordium with presence of QS complexes in V1-V4. His resting heart rate is 96 bpm, and his blood
pressure in the right upper limb in the supine position is 100/70 mm Hg.

Physical examination reveals an apical third heart sound, a soft systolic murmur in the aortic area that
increases with expiration, and crepitations in bilateral lung bases. His respiratory rate is 20

breaths/minute, and a pulse oximetry on room air is 92%. The emergency department physician

activates the catheterization laboratory planning for primary percutaneous coronary intervention

(PCI) and places a nasal cannula delivering oxygen at 4 L/min.

Which of the following medical interventions is the most appropriate?

a. Reteplase.

b. Diltiazem.

c. Enalaprilat.

d. Isosorbide.

e. Furosemide.
95. e.
Furosemide.
The use of intravenous (IV) enalaprilat in the CONSENSUS 2 trials was not associated with a

mortality benefit. This lack of benefit was attributed to hypotension in the early postreperfusion
setting. The borderline blood pressure, high resting heart rate, and evidence of pulmonary

congestion in this elderly patient with an extensive anterior myocardial infarction (MI) on ECG are
likely due to a significant acute decrease in stroke volume.

Treatment with IV beta-blockade as noted in the COMMIT 2 trial increases the risk of precipitating
cardiogenic shock in this setting and should be avoided. The dosage of unfractionated IV heparin in
the setting of ST elevation MI should be weight-adjusted with a bolus dose of 60 U/kg not to

exceed 4000 U, and an infusion rate of 12 U/kg not to exceed 1000 U/h.

96. A 78-year-old woman presents with resting chest pain and anterior T-wave inversions on

electrocardiogram (ECG). Her cardiac enzymes are normal. She has no cardiac history, no history of
diabetes, and no known allergies. She is treated with aspirin and subcutaneous enoxaparin and

remains chest pain free. However, 6 hours after admission, she develops massive hematemesis.

Which of the following is the next best step in her management?

a. Fresh frozen plasma (FFP).

b. Cryoprecipitate.

c. Protamine.

d. Lepirudin.

96. c.
Protamine.
Although protamine incompletely reverses the effect of enoxaparin on factor Xa, it does reverse its
effect on thrombin (factor IIa), and as such, is indicated for patients with bleeding complications due
to enoxaparin. Adverse reactions to protamine can be seen among patients who are allergic to

fish or who have received pork insulin. Cryoprecipitate and FFP do not reverse the effects of

indirect thrombin inhibitors such as heparins. A direct thrombin inhibitor, such as lepirudin, is
contraindicated in the presence of active bleeding.

97. A 52-year-old man develops intermittent chest pain while on a business trip and presents to

the emergency department for evaluation. He does not have diabetes and is a nonsmoker. His

outpatient medical regimen includes atorvastatin 40 mg, and lisinopril 20 mg daily. His exam is

normal. His electrocardiogram (ECG) shows nonspecific flattening of the T waves in the lateral

leads, and serial cardiac enzymes are normal. He is treated with aspirin and unfractionated heparin
for 24 hours and his chest pain has not recurred. He is anxious to return home and asks if he can catch
his flight later that afternoon.

Which of the following is most appropriate for the care of this patient?

a. Exercise treadmill test.

b. Coronary angiography.

c. No further testing prior to travel.

d. Heparin for additional 24 hours.

97. a.

Exercise treadmill test.

The patient presents with a symptom complex consistent with possible unstable angina. However,

there are no objective signs of ischemia, the clinical presentation is low risk given his young age, his
cardiac enzymes are normal, and there is an absence of diagnostic ECG changes. In low-risk

patients, an initial conservative (ischemia-guided) approach is reasonable in which the patient is


treated medically and undergoes risk stratification prior to discharge. Although 48-72 hours of

medical therapy prior to stress testing is recommended when definite acute coronary syndrome

(ACS) is present, stress testing can be performed safely earlier in low-risk patients with possible
ACS, and exercise stress testing provides a prudent solution that meets the wishes of the patient and
avoids excessive testing. Exercise stress testing prior to discharge would be recommended

because the patient has planned air travel.

98. A 60-year-old woman with atypical chest pain undergoes a treadmill stress test. The stress

test findings are:

 Heart rate response: 64-127 bpm

 Blood pressure (BP) response: 132/66 to 174/80 mm Hg

 Symptoms: Mild fatigue at peak

 Electrocardiogram (ECG): 2 mm of horizontal ST-segment depression in leads V5-V6 at peak

exercise

 Exercise duration: 3.0 minutes on Bruce protocol

 Perfusion images: Reversible apical ischemia

Otherwise, it was a normal study.

Which of the following findings best correlate with a highest risk for cardiac death?

a. Perfusion ischemia.

b. Poor functional capacity.

c. Hypertensive BP response.

d. Heart rate response.

e. ECG ischemia with 2 mm ST-segment depression.

98. b.
Poor functional capacity.
Poor functional capacity is the best predictor of cardiac death. Perfusion images or ECG ischemia
predict ischemic events, but are less predictive of death than exercise duration. Neither her heart rate
response nor BP response is particularly concerning.

99. A 65-year-old woman with a history of coronary artery disease is admitted with a 3-day

history of chest pain, nausea, and vomiting. Her electrocardiogram shows ST elevation in inferior
leads. She is taken to the cardiac catheterization laboratory and undergoes percutaneous coronary
intervention of the right coronary artery. In the cardiac care unit, she is somnolent, and her blood
pressure is 88/60 mm Hg, heart rate is 92 bpm, and oxygen saturation is 96% on room air. Jugular

venous pressure is 10 cm H2O. Lung fields are clear, and cardiovascular examination has normal

S1and S2 without murmurs appreciated. Extremities are cool.

Which of the following is the most likely diagnosis?

a. Papillary muscle rupture.

b. Ventricular septal defect (VSD).

c. Free wall rupture.

d. Right ventricular (RV) infarct.

e. Pulmonary embolism.

99. d.

Right ventricular (RV) infarct.

The patient has signs of RV involvement in her inferior myocardial infarction (MI) with hypotension
and evidence of poor perfusion. With hypotension and inferior MI, clinical suspicion of RV

involvement should be high. While other etiologies of her hypotension could include over-sedation,
bleeding, and cardiogenic shock, management is quite different for an RV infarct and needs to be

identified early.

Papillary muscle rupture and VSD are later mechanical complications of MIs. A post-MI VSD may be

accompanied by a thrill along the left sternal edge in 90% of cases, and the murmur would be

holosystolic. Free wall rupture is more common in anterior MIs and is more commonly seen in first
presentation of coronary disease and in elderly women. Pulmonary embolism is less likely given her
normal oxygen saturation.
100. A 58-year-old man presents to a community hospital 90 minutes from the nearest cardiac

catheterization laboratory with 2 hours of substernal chest pain. He is noted on electrocardiogram to


have marked ST elevation in leads V1-V5, with reciprocal ST depression in the inferior leads,

consistent with an evolving anterior ST-segment elevation myocardial infarction (STEMI).

Two years previously, he underwent an aortic valve replacement with a St. Jude mechanical

prosthesis, due to chronic aortic insufficiency. Prior to the aortic valve replacement, his coronary
angiogram revealed a 30% stenosis in the proximal left anterior descending coronary artery, which
was not bypassed. He is taking warfarin, and 3 days ago his international normalized ratio (INR) was
3.2.

Which of the following is the most appropriate treatment in this patient's care?

a. Aspirin, clopidogrel, intravenous unfractionated heparin, and tenecteplase.

b. Aspirin, clopidogrel, half-dose tenecteplase, followed by transfer for PCI.

c. Aspirin, clopidogrel, and transfer for primary percutaneous coronary intervention (PCI).

d. Aspirin plus tirofiban.

100. c. Aspirin, clopidogrel, and transfer for primary percutaneous coronary intervention (PCI).

Transfer to a primary PCI-capable facility is indicated if first medical contact to device time is <120

minutes, as in this case. Transfer should also be considered for those at high risk of bleed or signs of
shock. His risk for bleeding is higher with thrombolytics given his chronic anticoagulation. Even if
there was a time delay for primary PCI, one might consider transfer since bleeding risk increases
with a higher INR and thrombolytics. Aspirin plus tirofiban is not definitive reperfusion. Facilitated
PCI has not been demonstrated to improve outcomes.

101. An 82-year-old man presents with a non-ST elevation myocardial infarction, complicated by

congestive heart failure. His initial laboratory values are notable for a serum creatinine of 3.0

mg/dl, a creatine kinase-myocardial band of 147 mg/dl, and a cardiac troponin I of 23 mg/dl. He is
treated with aspirin, heparin, and eptifibatide and undergoes cardiac catheterization on hospital Day 2.
He has successful percutaneous transluminal coronary angioplasty and stenting of a 90%

stenosis in the proximal left anterior descending artery, and initially does well.

After the procedure, unfractionated heparin was discontinued, and the patient was maintained on

aspirin and a tirofiban infusion, which finished 1 hour ago. Sheaths were successfully removed soon
after the procedure. Later that evening, he is noted to be confused and agitated. On examination, his
heart rate is 116 bpm, blood pressure 88/58 mm Hg, and respirations are 20 breaths/minute.
His peripheral pulses are intact, and there is no groin hematoma. His electrocardiogram (ECG) is

unchanged.

In addition to intravenous saline, which of the following is the next best step in his care?

a. Urgent repeat coronary angiography.

b. Abdominal/pelvic computed tomography.

c. Platelet transfusion.

d. Intravenous Solu-Medrol.

101. b.

Abdominal/pelvic computed tomography.

The most likely diagnosis is retroperitoneal hemorrhage, as a complication of cardiac

catheterization and antiplatelet therapy. Importantly, the patient has a significant renal

dysfunction, which will potentiate the antiplatelet effects of tirofiban. Since tirofiban and

eptifibatide are reversible inhibitors of the glycoprotein IIb/IIIa receptor and have a short half-life,
platelet infusions are rarely necessary or helpful in patients receiving these agents.

In contrast, abciximab binds avidly to the receptor and thus has a very prolonged effect on platelet
aggregation (despite low free abciximab levels). Thus, platelet transfusions may be considered in
patients with bleeding complications following abciximab administration. The presentation is not

consistent with allergic reaction, therefore, steroids are not indicated. Repeat ECG is unchanged,
therefore a repeat catheterization is not indicated.

102. A 54-year-old man is brought to the emergency department (ED) by his coworker with acute-
onset dyspnea and syncope. He lost consciousness at work for approximately 15 seconds, but was

spontaneously revived after falling to the floor. His medical history includes hypertension, active
smoking, diabetes, and chronic kidney disease. He took an aspirin while on route to the ED.

His heart rate is 92 bpm and regular, blood pressure is 156/88 mm Hg, and respiration rate is 24

breaths/minute. Jugular venous pressure is normal, lung fields have scant bibasilar crackles, and he
has no cardiac murmur. Initial cardiac enzymes are negative, blood urea nitrogen is 42, and

creatinine is 2.6 mg/dl. Electrocardiography shows normal sinus rhythm with 2-4 mm ST elevation

in V2-V4. Symptoms began 45 minutes before arrival. In traffic, the nearest percutaneous coronary
intervention (PCI)-capable facility is 2.5 hours away at this time of day. Helicopter transport is not
available due to ocean fog.

Which of the following treatment strategies would you choose prior to transfer?

a. Tirofiban, half-dose tenecteplase, IV UFH, aspirin.

b. Full-dose tenecteplase, intravenous (IV) unfractionated heparin (UFH), aspirin, clopidogrel.

c. IV UFH, aspirin, clopidogrel.

d. Abciximab, half-dose tenecteplase, IV UFH, aspirin.

e. Full-dose tenecteplase, IV low molecular weight heparin (LMWH), aspirin, clopidogrel.

b.

Full-dose tenecteplase, intravenous (IV) unfractionated heparin (UFH), aspirin,


clopidogrel.
Given the time constraints associated with transfer for primary PCI (i.e., first medical contact to
device time >120 minutes), full-dose lytic therapy with transfer is the single best choice (Class I). IV

UFH is preferred over LMWH in the setting of renal insufficiency of this magnitude. Combination

reperfusion therapy with abciximab and half-dose tenecteplase is not recommended by the current

ST-segment elevation myocardial infarction (STEMI) guideline. The use of heparin, aspirin, and

clopidogrel alone is not appropriate for treatment of a STEMI.

103. A 74-year-old man presents with pain in the toes of his right foot. The pain is most severe at
night when he is in bed, such that he often sits up and dangles his foot over the side of the bed for
relief.

On examination, there is a fissure at the tip of the right great toe that is tender to touch. There is mild
bilateral ankle edema. The dorsalis pedis and posterior tibial pulses are not palpable in either foot.

Which the following diagnostic tests should be performed to determine appropriate treatment for

this patient?

a. Magnetic resonance (MR) angiogram.

b. Venous duplex ultrasound.

c. Foot X-ray.

d. Computed tomography (CT) angiogram.

103. a.

Magnetic resonance (MR) angiogram.

This patient presents with symptoms and findings of peripheral arterial disease (PAD). Ischemic

pain in the right foot is exacerbated when he is in bed, so he dangles the foot to improve perfusion
pressure (via gravitational effects) and reduce his pain. The finding of a skin lesion is consistent with
severe ischemia. The ankle swelling reflects the prolonged periods of putting his feet in a

dependent position; it is not a consequence of venous thrombosis. Therefore, a venous ultrasound

examination is not required.

A foot X-ray might useful to detect osteomyelitis, but these are not diagnostic for PAD. An

arteriogram is indicated to define his vascular anatomy and plan a revascularization procedure. MR
imaging is preferred (Class I, Level of Evidence A) versus CT angiogram (Class IIb, Level of
Evidence B) to diagnose the anatomic location of the lesion.

104. A 24-year-old woman with mixed connective tissue disease is admitted with weakness and

dizziness. Her blood pressure is 105/70 mm Hg, neck veins are elevated, chest is clear, cardiac exam
is normal, and there is no edema. She is referred to the cardiac catheterization laboratory. An intra-
arterial catheter yields the pressure tracing shown in Figure 1.

Which of the following hemodynamic abnormalities is demonstrated?

a. Pulsus parvus et tardus.

b. Pulsus paradoxus.

c. Wide pulse pressure.

d. Pulsus alternans.

104. b.
Pulsus paradoxus.
There is marked (>10 mm Hg) variation in the systolic arterial pressure. This finding indicates
exaggerated variation in systolic arterial pressure with the respiratory cycle, and is known as pulsus
paradoxus.

Pulsus parvus et tardus, a low pulse pressure with a slow upstroke, is observed in severe aortic

stenosis. Pulsus alternans, in which there are alternating strong and weak beats with alternating higher
and lower systolic arterial pressure, is seen in severe left ventricular systolic dysfunction. A wide
pulse pressure is observed in aortic regurgitation.

105. A 78-year-old woman with an anterior ST-segment elevation myocardial infarction (STEMI) is

treated with fibrinolysis, low molecular weight heparin (LMWH), aspirin, and clopidogrel, beginning
42 minutes after onset of pain. She is transported to the nearest percutaneous coronary

intervention center. Two hours later, she develops severe headache and is less responsive. A head
computed tomography scan shows a right frontal lobe hemorrhage.

In addition to neurosurgical consultation, which of the following treatments would you

recommend?

a. Epsilon aminocaproic acid.

b. Platelet transfusion.

c. Aprotinin.

d. Vitamin K.

e. Desmopressin acetate (DDAVP).

105. b.
Platelet transfusion.
This woman has suffered a fibrinolytic-related intracranial hemorrhage. Risk factors for bleeding
after thrombolytics include age, sex, small body size, and wide pulse pressure, in addition to the
particular agent used.

Recommended treatments include:

Cessation of fibrinolytic, antithrombin, and antiplatelet therapies

Protamine to reverse the effects of heparin or LMWH

Fresh frozen plasma to provide clotting factors V and VIII

Prothrombin complex concentrate to provide additional clotting factors

Platelets

Elevation of the head of the bed, hyperventilation, mannitol, and neurosurgical evacuation if

indicated

The other choices listed are not recommended. DDAVP is not indicated in the treatment of

bleeding after thrombolytics. Aprotinin, an antifibrinolytic agent, was withdrawn from the US

market in May 2008 due to adverse findings in a study of patients undergoing cardiac surgery.

Epsilon aminocaproic acid is not approved for this indication. Vitamin K is used for reversal of

bleeding related to vitamin K antagonists such as warfarin.

106. A 62-year-old woman is admitted with progressive angina, ST depression in leads V4-V6, and a
cardiac troponin value that peaks at twice the upper reference limit. Invasive therapy is

recommended, but declined by the patient. She stabilizes after treatment with intravenous
heparin, aspirin 81 mg, atorvastatin 80 mg, and metoprolol 25 mg bid. At the time of discharge, she is
chest-pain free while walking around the unit. Her echocardiogram demonstrates normal left

ventricular function.

Which of the following do you recommend for medical therapy, in addition to the medications

listed above?

a. Cilostazol 100 mg twice daily for 30 days.

b. Enoxaparin 1.0 mg/kg twice daily for 14 days.

c. Ticagrelor 90 mg BID for 14 days.

d. Clopidogrel 75 mg daily for 1 year.

e. Prasugrel 10 mg daily for 14 days.

106. c.

Ticagrelor 90 mg BID for 14 days.

For patients with unstable angina/non-ST-segment elevation myocardial infarction (NSTEMI) for

whom a conservative strategy is chosen, aspirin (75-162 mg) is continued indefinitely (Class I).

Antithrombin therapy with either unfractionated heparin, enoxaparin, or fondaparinux can be

given for the duration of hospitalization or up to 8 days (Class I). Based on the CURE trial, the
unstable angina/NSTEMI clinical practice guideline recommends clopidogrel 75 mg daily for at least
30 days and ideally for up to 1 year after presentation (Class I).

Cilostazol has been used as an alternative to thienopyridine therapy in allergic patients after

percutaneous coronary intervention, but has not been studied in this setting. Prasugrel has not

been studied in a conservative management strategy. Ticagrelor is an option for conservative

management therapy, but the duration should be for 1 year. Enoxaparin might be used, but not for

14 days.

107. A 62-year-old man with diabetes and dyslipidemia is hospitalized with recurrent angina 4

months after percutaneous coronary intervention (PCI) with deployment of a bare-metal stent for

treatment of a 90% mid left anterior descending stenosis. During his previous admission, he

developed a deep vein thrombosis and was treated with intravenous unfractionated heparin. His
platelet count fell from 223,000 to 62,000, and a platelet factor (PF4) assay at time of discharge was
positive. He was discharged on warfarin, aspirin, clopidogrel, atorvastatin, and metoprolol.

He was readmitted with chest pain and ST-segment depression 3 months later. His hepatic function

and cardiac enzymes are normal, but he has stage III kidney disease. His pain resolves and his

electrocardiogram normalizes after 2 hours.

A repeat cardiac catheterization reveals a 70% in-stent stenosis.

Which of the following medications would you consider for adjunctive therapy at the time of

repeat PCI?

a. Unfractionated heparin.

b. Bivalirudin.

c. Fondaparinux.

d. Enoxaparin plus a glycoprotein IIb/IIIa receptor antagonist.

107. b.
Bivalirudin.
This man developed heparin-immune thrombocytopenia during his first admission. Unfractionated

heparin and low molecular weight heparin are contraindicated in this situation. Upstream use of

fondaparinux has been associated with higher rates of catheter thrombus at time of PCI,

necessitating the use of heparin.

108. Which of the following is the correct order of events in the ischemic cascade?

a. ECG changes, perfusion defects, systolic dysfunction, diastolic dysfunction, chest pain.

b. Diastolic dysfunction, perfusion defects, ECG changes, chest pain, systolic dysfunction.

c. Perfusion defects, diastolic dysfunction, systolic dysfunction, electrocardiogram

(ECG) changes, chest pain.

d. Perfusion defects, diastolic dysfunction, systolic dysfunction, chest pain, ECG changes.

e. Diastolic dysfunction, systolic dysfunction, perfusion defects, chest pain, ECG changes.

108. c. Perfusion defects, diastolic dysfunction, systolic dysfunction, electrocardiogram (ECG)


changes, chest pain.

Understanding the order of events that occur in the ischemic cascade is important in understanding
how stress testing can detect subclinical disease. Single-photon emission computed tomography

(SPECT) myocardial perfusion imaging (MPI) unmasks exercise- or pharmacologic-induced

differences in perfusion, revealing subtle patterns of perfusion defects and early atherosclerotic
disease. This explains why nuclear perfusion-based techniques, which typically detect abnormalities
in coronary flow reserve even in the absence of ischemia, are more sensitive than contractile

dysfunction-dependent techniques, like stress or dobutamine echocardiography, which require

subendocardial ischemia to be abnormal. ECG changes and chest pain are relatively late

presentations of ischemia, which explains why the sensitivities of coronary artery disease detection
are lower than with imaging techniques.

109. A 60-year-old man underwent bare-metal stenting of a discrete left anterior descending

artery lesion after non-ST-segment elevation myocardial infarction and was placed on aspirin and

clopidogrel. One week after the procedure, he presents with a lower gastrointestinal bleed and is
found to have a previously undetected colon cancer that requires resection. Because of the
extensive surgical exploration that is anticipated, the surgeon requests that the clopidogrel be

stopped prior to surgery.

Which of the following is the most appropriate length of clopidogrel therapy in this setting?

a. 9 months.

b. 12 months.

c. 1 week.

d. 6 months.

e. 4 weeks.

109. e.
4 weeks.
The issue in this clinical situation is a tension between stent thrombosis, which is reduced by dual
antiplatelet therapy, versus periprocedural bleeding, which is accentuated with antiplatelet

therapy. For truly elective yet major surgery, studies suggest that waiting for 4-6 weeks after bare-
metal stenting allows the coronary stent to endothelialize so that perioperative stent thrombosis off
dual antiplatelet drugs is rare. Operating on dual antiplatelet therapy would expose the patient to
unacceptable bleeding risk. To stop clopidogrel now would increase the risk of stent thrombosis.

Heparin has not been shown to reduce the risk of early stent thrombosis. The risk of waiting 6

months to resect the colon cancer outweighs the benefits of 6 months of dual antiplatelet therapy.

110. In a patient with hypertrophic obstructive cardiomyopathy, which of the following

combinations is true of the post-extrasystolic beat?

a. The gradient increases and the pulse pressure decreases.

b. The gradient and pulse pressure both decrease.

c. The gradient decreases and the pulse pressure increases.

d. The gradient and pulse pressure both increase.

110. a.

The gradient increases and the pulse pressure decreases.

This is the Brockenbrough sign. The decrease in pulse pressure following a premature ventricular

contraction is caused by a reduced stroke volume that results from increased dynamic obstruction, as
evidenced by an increase in the gradient.

111. A 56-year-old woman presents for evaluation of 8 months of shortness of breath. Her exam

shows blood pressure 150/80 mm Hg, heart rate 78 bpm, and her body mass index is 40 kg/m2.

Jugular venous pressure is mildly elevated with clear lungs. No murmur is heard. She has an

S4 gallop and trace peripheral edema.

A transthoracic echocardiogram is performed, but is of limited quality due to body habitus. Her left
ventricular ejection fraction appears grossly normal. She undergoes right and left heart

catheterization for further evaluation of her symptoms. There is no significant coronary artery

disease on angiography. Right heart catheterization shows the following: right atrium 10 mm Hg,
pulmonary artery 55/25 mm Hg (mean 35), and pulmonary capillary wedge pressure 23 mm Hg

with prominent V waves.

Which of the following is the most likely cause of her symptoms?

a. Precapillary pulmonary hypertension.

b. Mitral regurgitation.

c. Obesity.

d. Heart failure with preserved ejection fraction (HFpEF).

111. d.

Heart failure with preserved ejection fraction (HFpEF).

The hemodynamics shown are consistent elevated filling pressures in a noncompliant left atrium.

This constellation of symptoms and hemodynamic findings are most compatible with HFpEF.

Mitral regurgitation would be expected to be associated with a systolic murmur in most cases.

While obesity may be a cause of her symptoms, it would not necessarily explain the

hemodynamics. Precapillary pulmonary hypertension would result in a significant transpulmonary

gradient that is not present here.


112. A 44-year-old woman is in the coronary care unit following presentation with an inferior

myocardial infarction. She was treated with primary angioplasty and stenting to a lone proximal

right coronary artery (RCA) stenosis.

Upon returning to the coronary care unit, she develops mild palpitations. She has no further chest
discomfort. Her blood pressure is 110/70 mm Hg and heart rate is 90 bpm. The 12-lead

electrocardiogram is shown in Figure 1.

Which of the following is the best management for this rhythm?

a. Atropine.

b. Synchronized direct current cardioversion.

c. No therapy required.

d. Repeat percutaneous coronary intervention (PCI).


e. Lidocaine.

112. c.
No therapy required.
The rhythm strip shows a sinus beat with inferior Q waves followed by an accelerated

idioventricular rhythm. In the context of PCI to the RCA, this represents a reperfusion rhythm; this is
self-limiting and does not require treatment.

113. A 17-year-old is referred by his pediatric cardiologist for advanced cardiovascular imaging
after a classic tetralogy of Fallot repair at the age of 2 months. His exercise tolerance is normal, and
he is not cyanotic. An echocardiogram reveals right ventricular enlargement and pulmonic

regurgitation.

Which of the following is most appropriate at this time to further assess the status of his congenital
heart disease?

a. Radionuclide angiography.

b. Computed tomographic angiography.

c. Magnetic resonance imaging (MRI).

d. Transesophageal echocardiography.

e. Right-heart catheterization.

113. c.

Magnetic resonance imaging (MRI).

Cardiac MRI is the only test that can provide all the components needed to quantify all the

parameters necessary, including right ventricular and right atrial volumes, valvular regurgitation, and
residual shunting. The right ventricular structures are difficult to quantify by echocardiography, and
computed tomography does not provide dynamic flow information. Right-heart catheterization

will not provide volumes or accurate quantification of valvular regurgitation.


114. A 25-year-old college basketball player is referred to see you due to an abnormal screening
electrocardiogram (ECG) obtained during basketball practice (Figure 1). He has no complaints, has a
normal physical examination, and has no family history of arrhythmias or sudden death.

Which of the following is the next most appropriate step in his management?

a. Repeat electrocardiogram (ECG).

b. Cardiac catheterization.

c. Echocardiogram.

d. Stress test.

e. Holter monitor.

114. a.

Repeat electrocardiogram (ECG).

This ECG is normal except for limb lead reversal. This can be seen by the negative P wave QRS
complex and T wave in lead I, as well as a total lack of voltage in lead II. It can be repeated with the
limb leads correctly placed to document the lack of underlying pathology. Other imaging or

diagnostic studies are not indicated in an asymptomatic patient with an otherwise normal ECG

without a family history of arrhythmias or premature death.

115. Which of the following patients has the highest probability of coronary artery disease (CAD)?

a. A 65-year-old woman with atypical angina and a positive exercise ECG.

b. A 65-year-old man with typical angina and a negative exercise electrocardiogram (ECG).

c. A 45-year-old man with atypical angina and a positive exercise ECG.

d. A 45-year-old woman with typical angina and a positive exercise ECG.

115. b. A 65-year-old man with typical angina and a negative exercise electrocardiogram (ECG).

According to the Bayes Theorem, the post-test probability that a patient has CAD depends on both

the pretest probability of the patient having CAD, and the results of the exercise test (Figure 1).

Pretest probability of CAD depends on age, sex, and character of chest pain, and can be determined
for an individual patient from published tables.

For patients at the extreme (low <10%, high >90%), the results of the exercise test shift post-test
probability of CAD very little (<10%). The 45-year-old man with atypical angina and a positive
exercise ECG, still has a post-test probability of CAD that exceeds 85-90%, because pretest

probability is so high. This patient has a pretest probability of CAD that exceeds 90%.

The other patients described all have a pretest probability of CAD of approximately 50%. A positive
exercise test shifts their post-test probability to approximately 75-80%.
(Figure 1)

116. A 48-year-old African American man presents to your office with palpitations. He is otherwise
healthy and has no other complaints. His physical examination is unremarkable. His

electrocardiogram shows sinus rhythm with a PR interval of 230 msec. On Holter evaluation, he has
6,000 multifocal premature ventricular contractions (PVCs) over a 24-hour period. Transthoracic

echocardiography reveals normal biventricular function without valvular abnormalities.

Which of the following is the next best step in his care?

a. Left-heart catheterization.

b. Cardiac computed tomographic angiography.

c. Cardiac magnetic resonance imaging (MRI).

d. Single-photon emission computed tomography (SPECT) myocardial perfusion imaging (MPI).

e. Transesophageal echocardiography.
116. c.

Cardiac magnetic resonance imaging (MRI).

This patient's clinical presentation is most consistent with sarcoidosis. Given his race, high PVC

burden, and conduction system disease, excluding this diagnosis is essential. Cardiac MRI and

fluorodeoxyglucose (FDG) PET are two modalities that can help establish the diagnosis of sarcoid.

He has no cardiovascular symptoms of ischemia. Thus, cardiac catheterization and SPECT MPI are

not indicated. Transesophageal echocardiography will not add additional information.

117. A 75-year-old woman was admitted 4 days after the onset of substernal chest pain. The patient is
an active smoker and has a history of high cholesterol, hypertension, and diabetes. On admission, she
was free of chest and epigastric pain and had a blood pressure of 142/70 mm Hg with a heart rate of
89 bpm.

Laboratory testing revealed that the troponin I was elevated at 20 ng/ml and the creatine kinase-
myocardial band was within normal limits. The patient's electrocardiogram had Q waves in leads II,
III, and aVF and there were no other ischemic changes. Diagnostic angiography demonstrated an
occluded right coronary artery (RCA), nonobstructive left anterior descending artery disease, and a
70% proximal left circumflex artery.

The patient was managed conservatively and medical therapy was implemented. Two days later in the
cardiac telemetry unit, the patient became acutely hypoxic, hypotensive, and required intubation. On
examination, she appears acutely ill with bilateral rales and a loud systolic murmur is present, which
was not heard on admission.

Her blood pressure is 85/60 mm Hg and her heart rate is 110 bpm. An echocardiogram reveals an
acute ventricular septal defect (VSD). An intra-aortic balloon pump is placed and her hemodynamics
stabilize.

Which of the following is the next appropriate management step?

a. PCI of the RCA.

b. Implant a left ventricular assist device (LVAD).

c. Stabilize for 4 weeks, then cardiovascular surgery.

d. Urgent surgical VSD repair.

e. Percutaneous coronary intervention (PCI) of the left circumflex artery.

117.

d.
Urgent surgical VSD repair.
Post-infarct VSDs typically occur 2-8 days after a myocardial infarction (MI). This is often after a
first MI. Medical management of post-MI VSDs have very high mortality rates (>90%) and even

with cardiac surgical or percutaneous repair, the mortality rates are >50%. Patients who survive do
better with early surgical VSD repair, as the ruptured site can expand abruptly. Percutaneous

closure is an emerging potential option, but early results suggest that residual shunting persists.

As device development improves, this may become a viable option. PCI alone would not correct

the hemodynamic situation, and an LVAD will not address the shunting problem.

118. A 45-year-old woman presents to the emergency department with 2 weeks of intermittent rest
substernal chest pain, which she describes as sharp. It is not provoked by exercise and has

lasted up to 30 minutes. There are no cardiac risk factors, her electrocardiogram (ECG) and cardiac
enzymes are normal, and she is able to exercise.

Which of the following is most appropriate in this patient's care?

a. Coronary angiogram.

b. Exercise stress echocardiogram.

c. Cardiac computed tomography (CT) angiography.

d. Exercise myocardial perfusion imaging.

e. Exercise stress test.

118. e.
Exercise stress test.
In this low-risk patient with a normal ECG and ability to exercise, exercise stress testing is the most
appropriate first step. Cardiac CT angiography may be helpful in patients who are at intermediate
risk. This patient with atypical nonexertional chest pain is low risk. With a baseline normal ECG,
stress testing with additional imaging should be reserved for equivocal exercise stress testing

results, inability to exercise, or high-risk exercise features.

119. An 89-year-old woman residing at a nursing home comes to the emergency department of your
hospital with a 24-hour history of dyspnea and confusion. The patient has significant dementia, as
well as hypertension, heart failure with preserved ejection fraction, and insulin-dependent diabetes.
She is agitated and disoriented, and unable to answer questions.

Her electrocardiogram demonstrates a heart rate of 72 bpm, with a normal axis, PR interval of 155
msec, a narrow QRS complex, and inferior ST elevation. Her troponin T is 1.6 ng/ml and her
creatinine is 2.8 mg/dl; her creatinine clearance is calculated as 25

ml/min. She weighs 60 kg.

Initial treatment and response: Emergency Medical Service has already given the patient 325 mg of
aspirin. As you are preparing to wheel the patient to the cardiac catheterization laboratory, her
daughter arrives, who holds medical power of attorney. She tells you that her mother has a Do Not
Resuscitate, Do Not Intubate (DNR/DNI) document and would not want to undergo a procedure.

However, she agrees to the administration of fibrinolytic therapy.

In addition to giving fibrinolytics and aspirin 81 mg qd, which of the following do you recommend
as additional antiplatelet and anticoagulant therapy?

a. Give a clopidogrel dose of 75 mg orally, followed by 75 mg daily. Also initiate unfractionated


heparin with weight-based dosing.

b. Give a clopidogrel load of 300 mg orally, followed by 75 mg daily. Also initiate enoxaparin 60 mg
subcutaneously twice daily.

c. Give a clopidogrel dose of 75 mg orally, followed by 75 mg daily. Also initiate enoxaparin 60 mg


subcutaneously twice daily.

d. Give a clopidogrel load of 300 mg orally, followed by 75 mg daily. Also initiate unfractionated
heparin with weight-based dosing.

e. Give a prasugrel load of 60 mg orally, followed by 10 mg daily. Also initiate unfractionated


heparin with weight-based dosing.

119. a. Give a clopidogrel dose of 75 mg orally, followed by 75 mg daily. Also initiate


unfractionated heparin with weight-based dosing.
The correct answer is to give 75 mg of clopidogrel, followed by 75 mg a day, followed by the use of
unfractionated heparin. In this elderly patient with central nervous system symptoms and renal

dysfunction, adjunctive antithrombotic therapy is limited following fibrinolytic therapy.

Prasugrel is not recommended in patients over 75 years of age. Enoxaparin dosing is complex in

patients with chronic kidney disease and every 24-hour dosing is recommended if creatinine

clearance is <30 cc/min. In patients over the age of 75 years, no loading dose of clopidogrel should
be used. Fondaparinux is also contraindicated if the creatinine clearance is <30 cc/min.

120. A 62-year-old woman presented to the emergency department with 2 hours of crushing
substernal chest pain. An electrocardiogram obtained demonstrates sinus rhythm at 95 bpm with

1-2 mm of ST elevation in leads II, III, and aVF. Her medical history is significant for dyslipidemia
and a possible transient ischemic attack. Her current medications include simvastatin. The patient
receives aspirin and is taken to the catheterization laboratory.

Which of the following agents confers the greatest survival advantage?

a. Prasugrel 60 mg orally.

b. Enoxaparin 1 mg/kg.

c. Clopidogrel 600 mg orally.

d. Intravenous heparin 70 U/kg.

e. Ticagrelor 180 mg orally.

120. e.

Ticagrelor 180 mg orally.

Either clopidogrel or ticagrelor are Class I recommendations. Prasugrel should be administered

after the anatomy is known and it is certain that the patient will be undergoing percutaneous

coronary intervention (PCI). In patients with non-ST-elevation acute coronary syndrome (NSTE-
ACS) and high-risk features (e.g., elevated troponin) and not adequately pretreated with clopidogrel
or ticagrelor, it is useful to administer a glycoprotein IIb/IIIa inhibitor (abciximab, double-bolus
eptifibatide, or high-dose bolus tirofiban) at the time of PCI (Class I).

Ticagrelor therapy was associated with less cardiovascular mortality than clopidogrel in the PLATO

trial. Prasugrel therapy was associated with fewer ischemic events than clopidogrel in the TRITON

trial, but mortality did not differ between treatment groups. Pretreatment with heparin or
bivalirudin has never been shown to reduce death.

121. There are multiple centers in Gotham City that perform >500 percutaneous coronary

interventions (PCIs) per year. Two of these centers spend significant resources advertising their
expertise in radial access.

PCI performed via radial access has a demonstrated mortality benefit compared with femoral

access when performed for which of the following indications?

a. Non–STEMI.

b. Assessment of valvular heart disease.

c. Chronic stable angina.

d. Unstable angina with negative biomarkers.

e. ST-segment elevation myocardial infarction (STEMI).

121. e.

ST-segment elevation myocardial infarction (STEMI).

The mortality benefit of radial access PCI has been demonstrated specifically in STEMI. While there
are multiple other benefits of radial access over femoral access, including greater patient

satisfaction and lower access site complications such as bleeding, mortality benefit has not been
demonstrated in the other clinical situations listed.

122. A thin 62-year-old man is referred by the patient’s family practitioner for preoperative

assessment of a large (5.8 cm) abdominal aortic aneurysm. He has a medical history of chronic

obstructive pulmonary disease (COPD) treated with inhalers, a left hemispheric stroke with little
residual, diabetes requiring insulin, and chronic renal insufficiency. The patient has one-block

claudication.

On exam, his vital signs are normal, and he has no evidence of cardiac murmurs, rubs, or gallops.

Chest examination reveals bilateral wheezes. There is no lower extremity edema. The

electrocardiogram (ECG) is normal.

Which of the following would be your next step in this man’s preoperative evaluation?

a. Dobutamine echocardiography.
b. Exercise ECG stress testing.

c. Resting echocardiogram.

d. No additional studies are indicated.

e. Pharmacologic stress testing using regadenoson.

122. a.
Dobutamine echocardiography.
A vasodilator study would not be wise in this patient, because there is evidence of wheezes on the lung
exam.

Adenosine may induce bronchoconstriction and respiratory compromise by activating the A2B
and/or

A3 receptors, resulting in mast cell degranulation, immunoglobulin E, and histamine release.


Regadenoson is an A2A-receptor agonist, which theoretically does not appreciably agonize A2B
and/or A3 receptors, but as per the American Society of Nuclear Cardiology imaging guidelines for
nuclear cardiology procedures, patients with bronchospasm are considered a contraindication to the
use of regadenoson, because there are inadequate data regarding its safety with active wheezing. In a
recent publication of 356 patients with asthma or COPD undergoing stress testing with regadenoson,
however, there was no evidence of severe bronchoconstriction; despite this, the authors still advised
caution in these patients.

Exercise ECG stress testing is unlikely to have diagnostic value because the likelihood of achieving
an adequate heart rate and blood pressure response is low due to his disabling claudication.

His abdominal aneurysm is considered large. Aneurysm diameter is the most important factor
predisposing to rupture. Based on this patient’s aneurysm diameter, his annual rate of rupture is 3-
15%, as per the Joint Council of the American Association for Vascular Surgery and Society for
Vascular Surgery. This patient should undergo repair of his aneurysm.

According to the guideline on perioperative cardiovascular evaluation and management of


noncardiac surgery patients, those with high cardiovascular risks with functional status <4 metabolic
equivalents should have stress testing if it would alter preoperative management by the identification
of severe coronary artery disease. A dobutamine echocardiogram is therefore the best available test
in this situation.

The patient does not require a resting echocardiogram, as there are no signs of fluid overload and the
cardiac physical exam is normal. In the absence of history of heart failure or ischemic heart disease,
there would be no indication for the resting echocardiogram.

123. A 51-year-old woman is diagnosed with breast cancer. She will undergo a mastectomy with
adjuvant chemotherapy with doxorubicin hydrochloride. The hospital protocol requires the patient

to have periodic evaluations of cardiac ejection fraction (EF).

An echocardiogram is performed and her EF cannot be evaluated despite the use of echo contrast

due to body habitus. The patient is extremely claustrophobic. The electrocardiogram (ECG) reveals
criteria for left ventricular (LV) hypertrophy.

Which of the following is the best imaging alternative in the care of this patient?

a. A gated single-photon emission CT (SPECT) study.


b. A radionuclide ventriculography (RVG).

c. Computed tomography (CT) angiogram.

d. A 3D echocardiogram.

e. Cardiac magnetic resonance imaging (MRI).

123. b.

A radionuclide ventriculography (RVG).

As cardiologists, our role is to provide the oncologist with the necessary information to make
therapeutic decisions. It is up to the oncologist to decide if an alternative to the doxorubicin
hydrochloride is as effective in this type of malignancy.

Gated-equilibrium RVG is a procedure in which the patient’s red blood cells (RBCs) are
radiolabeled and ECG-gated cardiac scintigraphy is obtained. Single or multiple measurements
of LV and/or right ventricular function are determined. Data are collected from several hundred
cardiac cycles to generate an image set of the beating heart that is presented as a single,
composite cardiac cycle. The method provides the clinician with regional and global wall motion,
cardiac chamber size and morphology, and ventricular systolic and diastolic function, including
LV and right ventricular EFs. The most common clinical settings in which RVG may be useful
include: known or suspected coronary artery disease, distinguishing systolic from diastolic
causes of congestive heart failure, evaluating cardiac function in patients undergoing
chemotherapy, and assessing ventricular function in patients with valvular disease. There are
certain potential sources of error for this technique, which include red blood cell labeling
(certain medications alter the labeling efficiency of the isotope), patient positioning, gating
errors, and image statistics. Since this technique involves radiation, its use has decreased (but it
is still a viable alternative to other modalities).

Cardiac MRI would have been a good alternative, because it is considered a gold standard for
evaluation of the EF, but the patient is claustrophobic, which is a major limitation of this
technique.

LV chamber and mass quantification have been studied extensively using 3D echocardiography.
This method has been shown to be more accurate and reproducible than 2D, but the accuracy of
the volume calculations is highly dependent on image quality.

The patient had a 2D echocardiogram performed, but image quality was poor despite the use of
echo contrast. A 3D echo would have the same issues.

The accuracy of EF measurement from gated-SPECT has been validated with respect to a
variety of radiopharmaceuticals and against a large number of gold standards (MRI, 3D
multigated acqusition scan, electron beam CT, etc.). EF can be underestimated in patients with
LV hypertrophy, though, because algorithms are either calibrated for the range of thickness
most typically encountered in clinical practice or assume a fixed myocardial thickness. The
reported discrepancy in EF may be up to 4 points.
This suggests the RVG would provide a more accurate EF determination in this setting.

124. For a patient with aortic stenosis and 3+ aortic regurgitation (AR), you are asked to provide an
aortic valve area (AVA) to help decide about surgical intervention.

The following data are obtained at cardiac catheterization:

 Peak aortic gradient 50 mm Hg

 Mean aortic gradient 36 mm Hg

 Fick cardiac output 3 L/min

 Thermodilution cardiac output 3.2 L/min

Based just on these data, which of the following is correct regarding the determination of the

estimated AVA?

a. It cannot be determined in the catheterization laboratory because of significant AR.

b. The AVA will be falsely smaller using the Fick cardiac output.

c. The AVA can be determined using the thermodilution cardiac output.

d. The AVA can be determined using the Fick cardiac output.

124. a.

It cannot be determined in the catheterization laboratory because of significant AR.

One can estimate the AVA by use of the Hakki equation. It is defined by:

flow past the valve (L/min) / square root of the mean gradient (mm Hg). The Fick cardiac output

and the thermodilution cardiac output describe the forward flow through the heart to the

periphery (actually pulmonary flow, but it should equal systemic flow if no shunt). The flow past the
aortic valve, though, includes both the forward flow and the regurgitant flow due to the AR,

and this is what is needed to do the calculations.

In this case, the estimated AVA would be: 6 L/min / 6 mm Hg or an AVA of 1.0 cm2. Using the Fick

output, the flow would erroneously be less than actual, resulting in a falsely low AVA.

125. A 75-year-old man with a medical history of hypertension, hyperlipidemia, diabetes, and a

remote history of neck injury after a fall, presents to the emergency department with vertigo and
blurred vision that lasted 20 minutes and resolved spontaneously. His symptoms started
immediately after turning his head to the right. Physical examination is normal except for a bruit on
the left side of his neck. Noncontrast computed tomography (CT) of the brain is normal.

Which of the following is the most appropriate test for this patient?

a. Dix-Hallpike test.

b. Carotid Doppler ultrasonography.

c. Cerebral angiogram.

d. Head and neck CT angiography (CTA).

125. d.

Head and neck CT angiography (CTA).

The patient’s symptoms are consistent with posterior circulation cerebrovascular syndrome (or

vertebrobasilar insufficiency). Typical symptoms of posterior circulation ischemia can include

vertigo, presyncope, syncope, diplopia, blurred vision, perioral numbness, tinnitus, ataxia, or

bilateral sensory deficits. His symptoms began with turning his head. Posterior circulation ischemia
can be caused by vessel impingement from the transverse processes in the cervical spine that

occurs with head movement. Given the patient’s history of neck injury, it is possible that an

unrecognized neck fracture of the transverse process could have occurred. Vertigo that occurs with
head movement is typically due to benign paroxysmal positional vertigo and can be assessed using

the Dix-Hallpike maneuver. However, blurred vision is unlikely to occur with benign paroxysmal

positional vertigo, thus the response Dix-Hallpike test is incorrect.

Despite having a left carotid bruit, the patient's symptoms are not consistent with anterior

circulation ischemia. For vertebral artery disease, the sensitivity and specificity of CTA or magnetic
resonance angiography is 94% and 95%, respectively, and significantly better than carotid Doppler
ultrasonography (sensitivity = 70%). Therefore, carotid Doppler ultrasonography is incorrect.

Cerebral angiography should be reserved for patients that require revascularization based on

noninvasive imaging. Thus, cerebral angiogram is incorrect. CTA is the correct answer.

126. You are the preceptor of a cardiology clinic in a university hospital setting. One of the

residents is evaluating a middle-aged woman with intermediate risk for having coronary artery

disease and an abnormal resting electrocardiogram (ECG). She has poor echocardiographic
windows and requires a nuclear stress test.

Which of the following protocols results in the LEAST radiation exposure to the patient?

a. One-day rest/stress with technetium-99m.

b. Stress only with technetium-99m.

c. Dual isotope (thallium and technetium).

d. PET scan using rubidium-82.

e. A positron emission tomography (PET) scan with fluorodeoxyglucose (FDG) and ammonia.

126. e. A positron emission tomography (PET) scan with fluorodeoxyglucose (FDG) and
ammonia.

Ammonia-13 (N-13) is a blood flow tracer and has a half-life of 10 minutes. It is generated in a
cyclotron and the average effective dose resulting from one of these studies is 2 mSv, making this
stress protocol the correct answer. The dosimetry is relatively low due to the short half-life of N-13
and the low energy of the emitted positrons: 1.48 mSv in 20 mCi (usually cardiac studies utilize 10-20
mCi/study). The 1-day rest/stress protocol with technetium is linked to an average effective dose of
about 12 mSv. This protocol is one of the most frequently used protocols in single-positron emission
computed tomography (SPECT).

Rubidium-82 is a perfusion isotope used in PET. The half-life is 75 seconds and is produced in a
generator. The average effective dose generated by this protocol (rest/stress) is about 15 mSv. In an
average study, 1.75-7.5 mSv in 60 mCi (usually cardiac studies utilize 20-40 mCi/study) is used.

The purpose of a rest image on a nuclear stress test is to define reversibility of stress-induced
perfusion abnormalities. In more than 50% of the patients, rest imaging is redundant, and does not
provide additional diagnostic or prognostic information. The average effective dose obtained from a
stress-only study using technetium-99m is 9 mSv. The ideal candidate for this study has a low pretest
probability for coronary artery disease, weighs between 120-250 lbs, and has had a normal stress test
in the past 3 years. Potential drawbacks include the inability to assess ventricular size and function,
both at rest and after stress. Transient ischemic dilation will not be seen, but the likelihood is very low
in patients with normal stress perfusion. There are prognostic and long-term data that suggest that this
approach is effective. This would be a reasonable alternative in the absence of a PET camera.

The average effective dose obtained from a dual isotope study is 35 mSv, which is one of the reasons
(if not the main reason) why this test is rarely used anymore. The half-life of thallium is 73.1 hours
and the patient is given between 2.5-4 mCi per study in addition to the technetium. Thallium thus
results in the highest radiation exposure.

127. During a routine cardiac catheterization procedure, which of the following will DECREASE the
total X-ray dose produced by the X-ray system during a procedure?

a. Using magnified views.


b. Panning over the bony structures compared with the lung fields.

c. Using angulated views.

d. Using lower framing rates.

e. Increasing the source-to-image distance.

127. d.
Using lower framing rates.
Only lowering the acquisition framing rate will help lower the X-ray dose among these scenarios.

The X-ray exposure equation must be satisfied for each frame imaged, and while the dose per

frame may be similar, reducing the number of frames per second reduces the overall dose

delivered during a case.

X-rays are divergent, and increasing the source-to-image distance results in magnification of the
image onto the face of the image detector and also loss of X-rays that diverge outside the face of the
image receptor. Since there are fewer X-rays that strike the image detector due to this loss

outside, more X-rays are needed to produce a satisfactory image on the detector, and the dose

necessarily increases.

Similarly, magnified views are created by using less of the face of the image receptor and, again to
produce a satisfactory image, more X-rays are needed. Panning over bony structures compared

with the lung fields also requires more X-rays because of the differences in density of the materials
and the resultant effect on attenuation of the X-ray beam. Angulated views result in an increase in the
source-to-image distance and require a greater X-ray dose.

128. A 68-year-old man (weight 87 kg) with a history of type 2 diabetes, hypertension,

gastroesophageal reflux disease, and seizure disorder presented to the emergency department

(ED) with complaints of chest pain and diaphoresis. The patient first noticed the onset of the chest
pressure the night prior to admission while he was walking on his treadmill. The pain resolved with
rest and recurred while he was getting dressed on the morning of admission.

In the ED, laboratory data were notable for a serum creatinine of 1.9 mg/dl and cardiac biomarkers,
including a troponin T of 0.6 ng/ml and a creatine kinase-myocardial band of 24 U/L. The

electrocardiogram revealed 1 mm of new ST depression in the inferior leads. Current home

medications include simvastatin 40 mg daily, lisinopril 10 mg daily, lansoprazole 30 mg daily, and


phenytoin 300 mg at bedtime. The patient was scheduled for a cardiac catheterization in the

morning.

Which of the following medications would preclude the use of ticagrelor therapy in this patient?

a. Phenytoin.

b. Lisinopril.
c. Simvastatin.

d. Lansoprazole.

e. Metoprolol.

128. a.
Phenytoin.
Ticagrelor is metabolized by the hepatic cytochrome P450 (CYP) 3A4 enzyme. Based on this (and

the observation in pharmacokinetic studies of reduced antiplatelet effect when ticagrelor was

given concomitantly with drugs that induce the CYP 3A4 isoenzyme such as rifampin), the

manufacturer currently recommends avoiding the use of ticagrelor with strong inducers of the CYP

3A4 enzyme, including rifampin, phenytoin, carbamazepine, phenobarbital, and dexamethasone.

129. A 62-year-old financial planner with a history of hypertension, diabetes, and peptic ulcer

disease comes to your office for evaluation. She describes nausea and abdominal discomfort that

has been bothering her for several months. She reports that this has been increasingly bothersome to
her despite frequent use of antacids. She notes some relation to consumption of large meals or at the
beginning of exercise. It lasts for several minutes then abates spontaneously. She denies

dyspnea, orthopnea, palpitations, or lightheadedness. She has no other medical conditions.

On physical exam, there is point tenderness located over the left sternum, but otherwise is normal.

Her resting electrocardiogram is normal.

Which of the following options is the most appropriate next step?

a. Cardiac catheterization.

b. Reassurance.

c. Referral for gastrointestinal source of symptoms.

d. Exercise stress testing.

e. Echocardiography.

129. d.
Exercise stress testing.
This woman has atypical symptoms, but has several features consistent with ischemic heart disease
and has risk factors for coronary artery disease. The relation of symptoms to exercise and after

meals is consistent with angina. While typical angina pain is described as heaviness, squeezing, or
tightness, atypical symptoms are common in women and the elderly. Nausea and difficult to

characterize discomfort is common in these groups.

In the WISE (Women's Ischemic Syndrome Evaluation) study, 65% of women with ischemia

presented with atypical symptoms. Pain reproduced by palpation is suggestive of a musculoskeletal


etiology, but does not exclude the possibility of her other symptoms being related to ischemic

heart disease. She has diabetes, and gastroparesis may be contributing.

The next appropriate study would be stress testing to assess for coronary artery disease.

130. A 77-year-old man comes to the office to discuss an upcoming cardiac catheterization

procedure that will be performed for angina that is refractory to medical therapy. He has

hypertension, hyperlipidemia, and no other chronic medical conditions. His laboratory tests include
hemoglobin 12 g/dl, creatinine 1.1 mg/dl, and international normalized ratio (INR) 1.0.

You tell him the most common complication of cardiac catheterization with coronary angiography

is which of the following?

a. Vascular access complication.

b. Anaphylactoid reaction to contrast agent.

c. Stroke.

d. Myocardial infarction.

e. Acute renal failure.

130. a.
Vascular access complication.
Myocardial infarction, stroke, and death are infrequent complications with a frequency of <0.1%.

More common complications are tachy- or bradyarrhythmias, acute renal failure, and vascular

complications. Vascular access site bleeding is the most common complication of cardiac

catheterization, occurring in 1.5-2.0% of patients. The Registry of the Society for Cardiac

Angiography and Interventions reported 519 major contrast media complications (0.23%) with four

deaths (1 in 55,000) in 222,553 patients undergoing selective coronary cineangiography.

While the radial approach has gained wider acceptance and has fewer bleeding complications

compared with the femoral approach, the incidence of radial artery occlusion is approximately 1-

4%, but this is generally asymptomatic. Although major access site complications are rare,

atrioventricular fistula, nerve injury, radial artery dissection and perforation, hematomas, and

pseudoaneurysm still occur with the radial approach at a rate that exceeds the other listed

complications.

131. An 82-year-old man with diabetes, hypertension, hyperlipidemia, and stable angina comes to

your office for routine follow-up. He has no change in his angina pattern. He plays tennis 3 times per
week with no symptoms. His blood pressure is 140/95 mm Hg and heart rate is 67 bpm. His

glycated hemoglobin is 7%. Urinalysis shows 1+ proteinuria. He is currently only on metoprolol 25

mg bid, atorvastatin 20 mg, and aspirin 81 mg.

Which of the following should be initiated?

a. Sliding scale insulin.

b. Metformin 500 mg daily.

c. Lisinopril 5 mg daily.

d. Hydrochlorothiazide 12.5 mg daily.

e. Clopidogrel 75 mg daily.

131. c.
Lisinopril 5 mg daily.
Angiotensin-converting enzyme (ACE) inhibitors are indicated in many patients with stable ischemic
heart disease, especially those with diabetes or left ventricular dysfunction. Weight loss, aerobic
exercise, an American Heart Association Step II diet, and ACE inhibitors decrease the risk of

microvascular complications and potentially cardiac events in patients with diabetes. Tight glycemic
control does not reduce the risk of macrovascular complications in patients with type 2 diabetes; it
may actually increase the risk of cardiovascular death and complications. According to the

guidelines, the goal blood pressure is <140/90 mm Hg in patients with diabetes or chronic kidney
disease. An ACE inhibitor or angiotensin-receptor blocker is first-line therapy in diabetes to prevent
progression of renal disease.

132. A 78-year-old man with peripheral arterial disease is referred to your clinic for preoperative
evaluation prior to femoral-popliteal bypass. He currently can only walk 1-2 blocks due to

claudication. He underwent percutaneous coronary intervention with a bare-metal stent for single-

vessel coronary disease affecting the right coronary artery 2 years ago. He denies angina or

shortness of breath.

A persantine sestamibi stress test demonstrates a small area of ischemia at the apex without

angina. Medications include aspirin 81 mg daily and metoprolol tartrate 50 mg twice daily. Blood

pressure is 112/72 mm Hg, and heart rate is 62 bpm. Creatinine is 1.5 mg/dl. His low-density

lipoprotein is 65 mg/dl, high-density lipoprotein is 45 mg/dl, and triglycerides are 100 mg/dl.

Which of the following do you recommend?

a. Add nitrates.

b. Increase beta-blocker.

c. Proceed with surgery.

d. Coronary revascularization.

e. Full-dose aspirin.

132. c.
Proceed with surgery.
This patient is at elevated risk for perioperative cardiac events for a moderate-risk surgery, but in the
face of appropriate medical therapy including aspirin and beta-blockade, there is no evidence that
revascularization will mitigate this risk. Results from the randomized, prospective CARP

(Coronary Artery Revascularization Prophylaxis) trial demonstrated that coronary artery

revascularization before elective major vascular surgery did not improve long-term survival or alter
early postoperative outcomes, including death, myocardial infarction, and length of the hospital

stay among patients with stable coronary artery disease. This patient’s heart rate suggests that he is
well beta-blocked, and you would not change the dose just prior to surgery. The patient should
proceed with surgery on current medical therapy. There is no evidence that an increased dose of

aspirin or nitrates will reduce perioperative events. The small amount of ischemia would not justify
revascularization.

133. A 47-year-old male smoker with hyperlipidemia calls for emergency medical services after
developing substernal chest pressure while watching television. Emergency medical technicians

arrive and administer sublingual nitroglycerin, which completely relieves his symptoms.

Upon arrival to the emergency department, the patient notes recurrent chest pressure with pain,

and his electrocardiogram shows 2.5 mm ST-segment elevation in V2-5. Both resolve with

administration of sublingual nitroglycerin. He is given aspirin, heparin, morphine, and intravenous


nitroglycerin. Coronary angiography reveals minimal nonobstructive plaque in the left anterior

descending artery and right coronary artery. Toxicology screening is negative.

In addition to smoking cessation, which of the following is the next most appropriate course of

action?

a. Metoprolol.

b. Niacin.

c. Omega-3 fatty acids.

d. Prazosin.

e. Diltiazem.

133. e.
Diltiazem.
While the clinical findings suggest coronary spasm, the diagnosis is assumed, but not proven, as

spasm was not actually observed. Coronary vascular smooth muscle hyper-reactivity is a key factor
in the pathogenesis of coronary artery spasm. Cigarette smoking is a major risk factor for variant
angina and endothelial dysfunction; patients should be counseled to avoid tobacco use.

Diltiazem, a nondihydropyridine agent, is often first-line therapy for coronary spasm, but if

symptoms recur, spasm testing with intracoronary acetylcholine is indicated.

Treatment with either niacin or omega-3 fatty acids is not indicated here, as they have not been

shown to improve coronary spasm. Patients should not be treated with beta-1 and beta-2 selective

agents, because of the possibility of unopposed alpha-adrenergic activity. If beta-blockade is

needed for management, the new agents with vasodilator activity (nebivolol, carvedilol, etc.) would
be better choices.

134. A 62-year-old man comes to your office with 3 months of dyspnea on exertion. He has a

medical history of hypertension treated with a calcium channel blocker, hyperlipidemia treated

with atorvastatin, and long-standing asthma well controlled with theophylline. He also has been

complaining of severe knee pain that he mitigates with nonsteroidal anti-inflammatory drugs.

On exam, his blood pressure is 98/65 mm Hg. His cardiovascular exam is unremarkable and lungs

are clear to auscultation bilaterally. His electrocardiogram (ECG) shows sinus rhythm with first-

degree atrioventricular (AV) block.

You decide he needs a pharmacologic nuclear stress test.

Which of the following factors would contraindicate using adenosine as your stress of choice?

a. Use of calcium channel blocker.

b. Low normal blood pressure.

c. First-degree AV block on ECG.

d. History of asthma.

e. Theophylline use.
134. e.
Theophylline use.
Adenosine is a vasodilator agent that works by producing stimulation of A2A receptors.

Methylxanthines (caffeine, theophylline, and theobromine) are competitive inhibitors of this effect,
which require withholding prior to testing. Second- or third-degree AV block is a contraindication;
first-degree is not. Systolic blood pressure <90 mm Hg is another contraindication.

Asthmatic patients with ongoing wheezing should not undergo adenosine stress testing. However,

it has been reported that patients with adequately controlled asthma can undergo an adenosine

stress test and can have pretreatment with two puffs of an inhaled bronchodilator. This patient had
clear lungs on auscultation. Another contraindication to adenosine is unstable acute myocardial

infarction or acute coronary syndrome. The patient was admitted and had negative cardiac

enzymes.

135. A 45-year-old man discharged from the hospital 6 weeks ago following an admission for
unstable angina during which he undergoes stenting of the right coronary artery presents to the

clinic. He reports no history of diabetes, smoking, hypertension, or a family history of premature


heart disease. Discharge daily medications include metoprolol succinate 25 mg, aspirin 81 mg,

atorvastatin 20 mg, and clopidogrel 75 mg. His body mass index is 27 kg/m2, waist circumference is
42 inches, and blood pressure is 135/85 mm Hg.

His laboratory results (on admission) were:

 Total cholesterol 230 mg/dl

 Triglycerides 350 mg/dl

 High-density lipoprotein 35 mg/dl

 Low-density lipoprotein 125 mg/dl

 Fasting glucose 99 mg/dl

Which of the following is the most appropriate next step for this patient?

a. Add fenofibrate.

b. Increase atorvastatin to 80 mg.

c. Continue current therapy.

d. Add ezetimibe.
135. b.
Increase atorvastatin to 80 mg.
Current guidelines recommend that this patient with atherosclerotic cardiovascular disease should be
on a high-dose statin for maximal effect. He had elevated triglycerides on admission, but these should
be reassessed on a higher-dose statin and with lifestyle changes prior to addition of any

other pharmacologic therapy such as fenofibrate. This patient is not on a maximal dose of statin.

Therefore, addition of ezetimibe at this time is not appropriate.

136. A 43-year-old man is referred for dyslipidemia. He has no history of hypertension or diabetes.

His father had a myocardial infarction at the age of 38. The patient is a nonsmoker who exercises
regularly and eats a vegetarian diet.

Body mass index is 20 kg/m2, waist circumference is 30 inches, blood pressure is 110/80 mm Hg,

and heart rate is 48 bpm.

The patient’s internist provides the following lab results:

 Total cholesterol 395 mg/dl

 Low-density lipoprotein (LDL) 321 mg/dl

 High-density lipoprotein 54 mg/dl

 Triglycerides 100 mg/dl

Which of the following is the appropriate next step?

a. Exercise treadmill stress test.

b. Coronary artery calcium score.

c. Statin therapy.

d. Genetic testing.

136. c.
Statin therapy.
For an initial evaluation of hyperlipidemia, testing for secondary causes of elevated cholesterol is
usually indicated, as disorders such as hypothyroidism, nephrotic syndrome, and medications can

be secondary causes of dyslipidemia and should be screened for too. In this patient who is likely to
have the heterozygous form of familial hypercholesterolemia, the most appropriate next step is

treatment with a statin.

Familial hypercholesterolemia is an autosomal-dominant disorder, caused by defective LDL-

receptors. Total cholesterols are usually in the 300-500 mg/dl range in many patients. Tendon

xanthomas on the Achilles tendon and extensor tendons of the hands are often present. These

patients carry a risk of premature coronary heart disease, as evidenced by the family history for this
patient. The prevalence is approximately 1:500 in the United States. This patient should be

started on lipid-lowering therapy along with continuation of therapeutic lifestyle changes, diet, and
physical activity. Furthermore, lipid testing is recommended for offspring.

Stress testing in this asymptomatic patient is not indicated. Calcium scoring does not change the
management in this patient, as he is already in one of the four statin treatment groups.

137. A 69-year-old woman sees you in the office for evaluation of fatigue with exertion for the past 6
months. She says that when she is climbing stairs, especially if she is carrying something, she is more
tired than usual. Symptoms resolve within 5 minutes of rest. She has hypertension, treated

with losartan and a diuretic. She reports numerous food allergies and she once had an episode of

wheezing and nasal congestion after taking aspirin.

You order a nuclear stress test, which reveals a small, mild reversible inferior perfusion abnormality
with a normal left ventricular ejection fraction.

Which of the following should be added to her medical regimen?

a. Prasugrel.

b. Cilostazol.

c. Dipyridamole.

d. Clopidogrel.

e. Ticlopidine.

137. d.
Ticlopidine.
Ordinarily, aspirin (75-162 mg/day) would be recommended for antiplatelet therapy to prevent

myocardial infarction in patients with established atherosclerotic disease. However, this patient has an
indication of an aspirin allergy. Aspirin therapy is therefore contraindicated. Clopidogrel has proven
efficacy for prevention of ischemic events based on the CAPRIE trial, and therefore can be used in
place of aspirin.

Cilostazol is a phosphodiesterase-3 inhibitor that acts as both an arterial dilating agent and an
antiplatelet agent; it can increase symptom-free walking distance in patients with intermittent

claudication, but has no proven benefit in preventing ischemic coronary events. Dipyridamole in

combination with aspirin has been shown to reduce subsequent strokes and transient ischemic

attacks in patients with cerebrovascular disease, but not ischemic coronary events. Ticlopidine is an
adenosine diphosphate (ADP)-receptor inhibitor similar in mechanism of action to clopidogrel, but
has more serious side effects, including neutropenia and thrombotic thrombocytopenic purpura,

thus has been essentially replaced by clopidogrel. Prasugrel is an ADP-receptor inhibitor that is
approved for patients with acute coronary syndrome. This patient has chronic angina.

138. A 67-year-old man is seen in the cardiology clinic for a routine follow-up visit. He currently has
no symptoms of angina, dyspnea, orthopnea, or ankle swelling. He plays golf for exercise, saying

that he walks all 18 holes four times a week. He suffered an inferior wall myocardial infarction 4

years ago, which was treated with primary percutaneous coronary intervention. Nonobstructive

stenoses were noted in the left anterior descending artery at the time, but a total occlusion of the right
coronary artery was deemed to be the culprit lesion and received treatment with a drug-eluting stent.
He received medical therapy for lipids and hypertension as well as dual antiplatelet therapy for 1 year
following his myocardial infarction. An echocardiogram 1 year ago demonstrated normal left
ventricular systolic function. He is currently taking aspirin 81 mg daily,

hydrochlorothiazide 25 mg daily, atorvastatin 40 mg daily, and lisinopril 20 mg daily.

Which of the following interventions should you recommend at this time to reduce morbidity and

mortality for this patient?

a. Depression screening.

b. Cardiac rehabilitation.

c. Aldosterone antagonist therapy.

d. Influenza vaccination.
e. Beta-blocker therapy.

138. d.
Influenza vaccination.
Vaccination against influenza has been shown to reduce morbidity and mortality in both patients

with acute coronary syndrome and in patients with prior cardiovascular events. Beta-blocker

therapy has not been studied in patients with a normal ejection fraction more than 3 years after a
coronary event. Depression screening may be important for quality of life, assuming supports to

treat depression are available, but has not been proven to reduce morbidity and mortality among

patients with coronary artery disease. Aldosterone antagonists are only proven to be beneficial in
patients with reduced ejection fraction following coronary events. Cardiac rehabilitation is

important following acute events and for the management of symptoms including angina, but

neither apply to this patient.

139. A 61-year-old woman presents for follow-up after a recent hospitalization for a non-ST-

segment elevation myocardial infarction (NSTEMI). She has type 2 diabetes, hypertension, and

stage 2 chronic kidney disease. Six weeks ago, she went to the emergency department with chest

pressure and underwent coronary angiography, which revealed a focal stenosis in the left anterior
descending coronary artery, for which she received a single drug-eluting stent. Her medications

include aspirin 81 mg daily, clopidogrel 75 mg daily, and simvastatin 20 mg daily, glargine insulin,
metoprolol succinate 25 mg qd, amlodipine 5 mg qd, and lisinopril 20 mg qd.

Today, she reports feeling well, has had no recurrent chest pressure or dyspnea, and is

participating in cardiac rehabilitation. Her blood pressure is 122/76 mm Hg and heart rate is 68

bpm. Her lipid profile shows total cholesterol of 210 mg/dl, low-density lipoprotein of 120 mg/dl,
high-density lipoprotein of 40 mg/dl, and triglycerides of 200 mg/dl.

Which of the following is the most appropriate next step, in addition to lifestyle modification?

a. Change simvastatin to atorvastatin 40 mg.

b. Add ezetimibe 10 mg daily.

c. Add fenofibrate 145 mg daily.

d. Add fish oil 3 g daily.

e. Increase simvastatin to 40 mg.


139. a.

Change simvastatin to atorvastatin 40 mg.

This patient has established coronary artery disease and had a recent acute coronary syndrome.

High-intensity statin therapy should be continued in all patients with NSTEMI and with no

contraindications to its use (Class I, Level of Evidence A). Replacement of simvastatin with

atorvastatin 80 mg daily is supported by several large randomized trials. High-intensity statin

therapy used in randomized controlled trials includes atorvastatin 40-80 mg and rosuvastatin 20-40

mg. Adding ezetimibe, fenofibrate, fish oil, and niacin is not indicated, as the primary goal is high-
intensity statin therapy.

140. A 43-year-old man without any significant medical history presents for an office visit to

discuss prevention of cardiovascular disease (CVD). He is a nonsmoker. He takes no medications

and exercises 2-3 times per week at the local gym.

On physical examination, his weight is 220 lbs, blood pressure is 136/84 mm Hg, and pulse is 76

bpm. His body mass index is 33 kg/m2 and waist circumference is 40 inches. His physical

examination is otherwise unrevealing. Laboratory studies reveal a total cholesterol of 190 mg/dl,
low-density lipoprotein cholesterol of 130 mg/dl, high-density lipoprotein cholesterol of 39 mg/dl,
triglycerides of 209 mg/dl, and a fasting glucose of 110 mg/dl.

Which of the following is the most appropriate next step?

a. Start aspirin.

b. Recommend lifestyle modification.

c. Start statin.

d. Measure carotid intima-media thickness (CIMT).

e. Obtain exercise stress test.

140. b.
Recommend lifestyle modification.
This patient's risk of atherosclerotic CVD (ASCVD) is low (<5%). Current CVD risk guidelines stress
the importance of lifestyle modifications to lower CVD risk. This includes eating a heart-healthy diet,
regular aerobic exercises, maintenance of desirable body weight, and avoidance of tobacco

products.

Due to his low risk, he would not benefit from statin or aspirin therapy. Further risk assessment may
be considered if a treatment decision is uncertain. Assessment of one or more of the

following–family history, high-sensitivity C-reactive protein, coronary artery calcium score, or


ankle brachial index—may be considered to inform treatment decision -aking (Class IIb, Level of
Evidence B). However, measurement of CIMT is not recommended for risk assessment for ASCVD
(Class III,

Level of Evidence B). In a low risk, asymptomatic patient, stress testing is inappropriate.

141. A 67-year-old Spanish-speaking man from Mexico comes to the hospital for evaluation of chest
pain that occurred previously with exertion and now at rest. He has a history of hypertension,
hyperlipidemia, and diabetes. His admission electrocardiogram was notable for 1 mm ST

depression in the inferior leads. His initial troponin level was 0.2 ng/dl (normal <0.04 ng/dl). His
chest pain was relieved in the emergency room with sublingual nitroglycerin and morphine.

You believe that the next appropriate step is coronary angiography and would like to discuss the

risks/benefits of this invasive procedure with him in order to obtain consent. He speaks broken

English, but appears to understand much of what you say.

Which of the following is the most appropriate method to communicate this information to the

patient?

a. Having his bilingual daughter translate, who is with him now.

b. Giving him a copy of the consent form in Spanish to read on his own.

c. Referring him to your colleague who speaks Spanish and who will be on service tomorrow.

d. Explaining in English and having him nod to ensure understanding.

e. Requesting and waiting for a hospital interpreter in person or by phone

141. e.

Requesting and waiting for a hospital interpreter in person or by phone


In cases of patients whose English skills are limited, an official medical interpreter is the best choice
for conveying accurate medical information, especially when it concerns an invasive procedure with
risks of serious complications. A family member is generally only acceptable in emergencies or if
the patient specifically requests him or her to interpret for a small matter. Minors (<18 years old)
should be avoided, and the age of the patient's daughter is unknown. Relying on nodding to convey
understanding of the nature of a complex procedure is not sufficient in this patient with limited
English skills. Reading and signing the consent form alone is not adequate for obtaining informed
consent and provides no opportunity for the patient to ask questions. Delaying care of a non-ST-
elevation myocardial infarction by a day or more to involve your colleague is providing suboptimal
care and not an acceptable option.

142. A 68-year-old man with a history of ongoing tobacco use, hypertension, and a 4.2 cm

abdominal aortic aneurysm (AAA) is seen for scheduled follow-up. His AAA has been followed with

yearly ultrasounds and is stable in size. He is retired and walks for 30 minutes each morning with his
dog. Medications include lisinopril, aspirin, and a multivitamin.

On examination, his blood pressure is 128/82 mm Hg and pulse is 88 bpm. There is a soft

abdominal bruit and an accentuated abdominal aortic pulsation. A fasting lipid profile reveals total
cholesterol of 220 mg/dl, high-density lipoprotein (HDL) of 31 mg/dl, and triglycerides of 316

mg/dl.

Which of the following is the most appropriate recommendation?

a. Losartan.

b. Fenofibrate.

c. Atorvastatin.

d. Fish oil.

e. Niacin.

142. c.
Atorvastatin.
This patient presents with dyslipidemia with elevated total cholesterol and triglycerides and low HDL.
The first step in management of dyslipidemia includes assessing patient risk. This patient’s AAA
places him in the highest risk category for future cardiac events. Other coronary risk

equivalents include the presence of peripheral arterial disease, diabetes, or cerebrovascular

disease.

Given his high risk, moderate- to high-intensity statin therapy is recommended. Fenofibrate is

effective at lowering triglycerides. However, in this patient, the non-HDL cholesterol should be

treated first. If triglycerides remain elevated despite non-HDL cholesterol at goal, therapy directed
toward hypertriglyceridemia is reasonable, although concurrent fibrate and statin therapy increase the
risk of statin-induced myopathy. Fish oil can also be useful in management of

hypertriglyceridemia, but should not be used as the initial therapy in this case. Niacin is effective at
raising HDL. However, the incremental value of raising low HDL with pharmacotherapy has not

been proven in several large randomized trials, and is an area of ongoing study and debate. Finally,
addition of a second antihypertensive agent is not indicated, as his blood pressure is well controlled
to <130/80 mm Hg.

143. A 48-year-old man presents for follow-up. He feels well and recently completed his first 5k road
race. His medical history includes dyslipidemia managed with atorvastatin. Physical

examination is normal. He obtained laboratory work at a recent “employee health fair,” which he

brings for review. The total cholesterol is 160 mg/dl, high-density lipoprotein cholesterol is 48

mg/dl, and triglycerides are 102 mg/dl. The high-sensitivity C-reactive protein level is within the
normal range. Serum chemistries including creatinine are normal. Dipstick urinalysis is negative for
blood, protein, and glucose. Serum creatine kinase (CK) level is 301 U/L (reference range, 26-192).

Liver function tests are normal.

Which of the following is the best next step in management of this patient?

a. Decrease frequency of atorvastatin to every-other-day dosing.

b. Decrease the dose of atorvastatin and add ezetimibe.

c. Stop atorvastatin and prescribe cholestyramine.

d. Stop atorvastatin and prescribe rosuvastatin.

e. Continue atorvastatin and check CK in 3 months.


143. e.

Continue atorvastatin and check CK in 3 months.

This patient is asymptomatic and presents with a slightly increased CK level while on statin therapy.

Guidelines recommend checking a single CK level upon initiation of statin therapy to obtain a

baseline value for comparison, should myopathic symptoms develop. CK should be rechecked with

any new muscle symptoms, but routine monitoring of CK in asymptomatic patients is not

recommended. Statin therapy should be stopped in patients with severe myopathic symptoms

regardless of CK level and if the CK is greater than 10-times the upper limit of normal. For this
asymptomatic patient with a slightly elevated CK, no change in therapy is needed and he can be

followed serially. The elevations in CK are exacerbated by his training/running.

Cholestyramine can be used in lieu of statin therapy in statin-intolerant patients, as there are data for
an improvement in mortality in the primary prevention setting. Rosuvastatin and pravastatin

are both more hydrophilic than atorvastatin, and in some patients with statin myalgia (not

myopathy), transition to a more hydrophilic statin can result in symptom improvement. Similarly,

for statin-intolerant patients, an every other day dosing regimen can allow for some statin therapy.

A combination of simvastatin and ezetimibe was not superior to simvastatin alone in a randomized

controlled trial, and no change in therapy is indicated for this patient.


144. A 55-year-old woman with stable anginal chest pain on no cardiac medication underwent a

computed tomography (CT) coronary angiogram to assess for possible coronary artery disease

(Figure 1). Her electrocardiogram (ECG) showed no acute ischemic changes and is normal. Her

blood pressure is normal and she has no diabetes.

Based on the CT image shown in Figure 2, which of the following is the next best step?

Figure 2

a. An exercise ECG stress test.

b. Cardiac catheterization.

c. Begin medical therapy for coronary disease.

d. An exercise nuclear stress test.

144. c.
Begin medical therapy for coronary disease.

The CT suggests a mid left anterior descending artery lesion and single-vessel disease. She needs
initial treatment with medical therapy for stable angina before further testing is indicated.

145. A 58-year-old man presents to the emergency department with a 5-hour history of crushing,
substernal chest pain. The patient history is significant for type 2 diabetes, hypertension, dyslipidemia,
and coronary artery disease with stenting of his proximal left anterior descending artery 3 years ago.

Admission electrocardiogram shows anterior ST-segment elevation, including ST elevation in lead


aVR. The patient is referred for emergent cardiac catheterization. Initial coronary angiography shows
a 95% ostial left main stenosis with thrombus. An intra-aortic balloon pump (IABP) is inserted and
successful stenting of the left main is performed with 0% residual stenosis and TIMI 3 flow
throughout the left coronary system. The right coronary artery is widely patent. The IABP is
continued at a 1:1 counterpulsation ratio.

One hour after the procedure, the patient develops atrial fibrillation with a rapid ventricular response.
The patient remains asymptomatic. Hemodynamic variables after percutaneous coronary intervention
(PCI) are shown in the table in Figure 1.

Which of the following choices would be the best next step in the management of this patient?
a. Add flecainide.

b. Repeat coronary angiography.

c. Add amiodarone.

d. Insert an Impella 5.0 left percutaneous device.

e. Add dobutamine.

145.

c.
Add amiodarone.
The IABP is a catheter-mounted balloon that augments pulsatile blood flow by inflating during

diastole, which displaces blood volume in the descending aorta and increases mean aortic

pressure, thereby augmenting coronary perfusion. Upon deflation, during systole, the IABP

generates a pressure sink, which is filled by ejecting blood from the heart. The net effect of IABP

therapy is an increase in mean arterial pressure and augmented ventricular stroke volume. For

this reason, proper IABP function depends on native contractile function of the heart. During

periods of tachycardia, the IABP is unable to adequately inflate, and therefore cannot augment

mean arterial pressure.

Thus, pharmacological management of reducing the heart rate is a reasonable next step to

optimize IABP function and stabilize the patient’s hemodynamics. Escalating mechanical support

to an Impella, which requires surgical vascular access, is not required. There is no indication that the
patient has developed recurrent thrombosis of the left main artery. Initiation of inotropes

and vasopressors may worsen the tachycardia and lead to further decompensation. Direct

current cardioversion may be appropriate but is not listed as an option. Flecainide is

contraindicated in patients with coronary artery disease.

146. A 65-year-old woman was admitted with an anterior ST-segment elevation myocardial

infarction (STEMI). She was taken urgently to the catheterization laboratory, where she underwent
placement of two bare-metal stents to the proximal and mid-left anterior descending coronary

artery. Forty-eight hours later, she complains of shortness of breath and chest pain of a different
quality than her presenting symptoms. On examination, her blood pressure is 80/57 mm Hg, heart

rate is 102 bpm, and oxygen saturation is 94% on room air. Her jugular venous pressure (JVP) is

elevated to the angle of the jaw. Lung exam is clear.

What is the most likely finding on cardiovascular examination?

a. Loud pansystolic murmur.

b. Muffled heart sounds.


c. Displaced point of maximal impulse.

d. Loud S4.

146. a.
Loud pansystolic murmur.
The patient is expected to have muffled heart sounds secondary to free wall rupture and increased
pericardial fluid. This diagnosis is consistent with elevated JVP and clear lungs, and occurs more
frequently in the setting of an anterior wall MI. It is considered a surgical emergency with a high
mortality.

A loud pansystolic murmur at the lower left sternal border, accompanied by a thrill, is consistent with
ventricular septal defect and is usually accompanied by pulmonary congestion. The treatment

is urgent surgery. There is a higher mortality in patients with inferior MI compared with those with
anterior MI. A fourth heart sound may be heard in the setting of an acute MI, but given signs

consistent with pericardial effusion, this is likely not to be audible. A displaced apex is most

consistent with chronic systolic heart failure.


147. A 57-year-old man has had 2 years of progressively worsening angina with exertion. He was

initially managed with medical therapy, but has had progressively worsening angina despite

maximization of two antianginal therapies. A stress test showed moderate exercise tolerance, and

nuclear imaging suggested a large area of inferior ischemia involving >10% of the left ventricular
muscle mass. A diagnostic cardiac catheterization was performed and showed a chronic total

occlusion of the right coronary artery, as shown in Figure 1.

Given his lifestyle-limiting symptoms despite maximal medical therapy, you have referred him for

percutaneous coronary intervention (PCI). You have obtained careful consent and have advised him

of the risk of radiation-induced skin injury. You have assured him that you will take every

precaution to minimize the radiation exposure to the patient during what may be a lengthy

procedure.
Which of the following techniques will increase radiation exposure to the patient?

Figure 1

a. Use of less magnified views.

b. Use of fluoroscopy storage instead of cine storage.

c. Use of collimation.

d. Steep imaging intensifier angles.

e. Use of fluoroscopy frame rate of 7.5 frames per second (FPS).

147. d.

Steep imaging intensifier angles.

Patients with cardiovascular disease are exposed to a wide array of diagnostic and therapeutic
procedures that result in a significant cumulative exposure of ionizing radiation. There is a growing
awareness of this cumulative exposure, and every effort should be made in the catheterization
laboratory to minimize the radiation exposure to the patient (and the operator). The majority of PCI
procedures can be completed, as most modern imaging equipment in the catheterization laboratory
has a fluoroscopy storage function where a maneuver performed under fluoroscopy can be
permanently stored and recorded. In many cases, the fluoro image will provide diagnostic grade
imaging so that cine-angiography will not be necessary and further radiation exposure can be
reduced. On average, cine radiation doses can be as much as 20-times higher than fluoroscopy doses.

Reducing the frame rate for image acquisition will also significantly reduce radiation exposure
during fluoroscopy.

While many operators prefer using 15 FPS, a frame rate of 7.5 FPS is sufficient in most patients for
effective imaging and will reduce radiation use by 50% compared to 15 FPS. Collimation and use of
lower magnified imaging fields are also critical for reduction of radiation exposure.

Use of steep imaging angles will increase the radiation exposure to the patient. With steep angulation,
fewer X-rays reach the image intensifier, resulting in feedback to the radiation generator to generate
greater energy in order to provide a sufficient image as the source-to-image distance is increased.
Every effort should be made to use angles as shallow as possible during PCI, and to use multiple
angles so as not to concentrate the radiation beam to one area.
148. A 76-year-old man with atrial fibrillation presents to clinic with a 2-year history of chest pain at
rest located in the mid-epigastrium radiating to the mid-substernal region. Computed

tomography (CT) angiography of the chest reveals the finding shown in Figure 1.

Which of the following is the abnormality shown in Figure 1?

Figure 1

a. Primary cardiac tumor.

b. Coronary artery aneurysm.

c. Pericardial cyst.

d. Atrial appendage thrombus.

148. b.

Coronary artery aneurysm.

The abnormality shown on the CT image has peripheral calcifications with central contrast and

surrounding soft tissue density. It is continuous with the atherosclerotic coronary artery and

consistent with coronary artery aneurysm, rather than a pericardial cyst or primary cardiac tumor.

Neither of these would be expected to have peripheral calcification. Pericardial cysts are generally
benign and usually right-sided, often in the atrioventricular groove.

The abnormality is anterior to the left atrium and not consistent with an atrial appendage

thrombus.

149. A 75-year-old man is found by his family, unresponsive and crumpled over in the bathroom.

Emergency medical service is called, but no bystander cardiopulmonary resuscitation (CPR) is

attempted. Upon arrival 7 minutes later, emergency medical service personnel begin CPR, and after 2
minutes the initial rhythm is ventricular fibrillation. After a 20-minute period of CPR effort,
including epinephrine and repeated shocks, spontaneous circulation is established. He is intubated.

Upon arrival at the emergency department, the patient has a weak pulse and blood pressure of

95/65 mm Hg. He remains unresponsive, without spontaneous movement. The monitor reveals a

wide QRS complex tachycardia at 100 bpm.

Which of the following is the next most appropriate postresuscitation therapy?


a. Discuss with the family withdrawal of therapy.

b. Therapeutic hypothermia.

c. Electroencephalogram.

d. Cardioversion for wide QRS tachycardia.

149. b.
Therapeutic hypothermia.
This patient’s prognosis is poor but not futile. Since he remains comatose, hypothermia is the best
chance to limit long-term neurological damage. Long-term survival can be improved with

hypothermia, generally from previous improvement rates of 1-2% to an improvement rate of 5-

20%. Recent data suggest this may not be the case in children. It is important to realize that in order to
achieve these improved survival rates, adequate time after rewarming must be allowed

before reliable prognostication can be done and further efforts terminated. There is no need for

cardioversion of his postresuscitation wide QRS rhythm since it is providing a current acceptable
blood pressure and most certainly represents a transient epinephrine/norepinephrine response. An

electroencephalogram may be important later to help assess his neurologic status.

150. A 42-year-old man with a long-standing human immunodeficiency virus (HIV) infection comes

to your office out of concern of his cardiovascular risk. He is on antiretroviral drugs. He smokes half
a pack per day and drinks 1-2 drinks per week. Pertinent routine tests show a blood pressure of

127/88 mm Hg, low-density lipoprotein 177 mg/dl, high-density lipoprotein 27 mg/dl, and

triglycerides 334 mg/dl.

Which of the following is an appropriate therapy for this patient, in addition to smoking cessation?

a. Ezetimibe 10 mg a day.

b. Begin 2 g fish oil daily.

c. Rosuvastatin 20 mg a day.

d. Atorvastatin 10 mg a day.

e. Fenofibrate 150 mg a day.

150. d.
Atorvastatin 10 mg a day.
HIV-infected individuals have high rates of smoking, and smoking cessation is perhaps the most

modifiable risk factor to minimize cardiovascular disease risk in these persons. While this patient
requires antiretroviral drugs and needs to improve his lipid profile, smoking cessation would have
the most immediate effect on his overall health and cardiovascular disease risk. He warrants

therapy for his hyperlipidemia. Because of the interaction with antiretroviral drugs, a low-dose

statin is recommended.

151. A 57-year-old man with a history of hypertension, hyperlipidemia, and a prior stent to the

right coronary artery is seen for routine follow-up. Although he does not exercise regularly, he

denies anginal symptoms during daily activities. He drinks one glass of wine daily and does not

smoke. His medications are aspirin 81 mg qd, metoprolol succinate 25 mg qd, lisinopril 10 mg qd,

and atorvastatin 40 mg qd. His blood pressure is 112/75 mm Hg. His most recent lipid panel shows

a low-density lipoprotein (LDL) of 68 mg/dl and high-density lipoprotein (HDL) of 25 mg/dl. He

wants to know how to best improve his cardiovascular risk profile.

Which of the following is the most appropriate recommendation?

a. Add colesevelam 3750 mg qd.

b. Start a regular aerobic exercise program.

c. Add niacin 500 mg bid.

d. Increase daily alcohol intake to 3-4 drinks.

e. Change to simvastatin 80 mg daily.

151. b.

Start a regular aerobic exercise program.

Abundant epidemiologic evidence supports the relationship between increased physical activity

and reduced cardiovascular risk. Currently, the best advice for patients is to perform regular

aerobic exercise, which has beneficial effects on cardiovascular health. While niacin is effective at
raising HDL levels, recent trials have shown that the addition of niacin fails to reduces
cardiovascular events in patients with well-controlled LDL levels on background statin therapy.

The AIM-HIGH trial tested the effects of adding niacin for patients with low HDL levels already on a
statin. Despite increasing HDL levels, there was no benefit with niacin for reducing cardiovascular
events and there was a question of a signal of increased stroke.

The HPS2-THRIVE trial also examined the effects of niacin/laropiprant in patients with well-

controlled LDL levels on a statin. There was no difference in rates of cardiovascular events, and
niacin treatment was associated with more side effects. Given these results, it is not clear that niacin
provides any additional benefit in a patient such as this who already is on a statin and at target LDL.

Initiation of simvastatin at 80 mg or uptitration to 80 mg dose is contraindicated in patients due to


risks of muscle injury. Colesevelam is a bile acid sequestrant that has primarily LDL-lowering
effects.

While moderate alcohol intake (especially red wine) may have beneficial cardiovascular effects,

consuming more than 1-2 drinks daily has been shown to have detrimental effects on health.

152. A 45-year-old man presents with substernal chest discomfort that comes on predictably with
walking two blocks. The discomfort is alleviated with rest. He has a long-standing history of

hypertension and was diagnosed with diabetes and dyslipidemia 6 months earlier. Current

medications include metoprolol, lisinopril, simvastatin, and baby aspirin.

A stress myocardial perfusion imaging study is ordered. He is unable to complete stage 2 of the

standard Bruce protocol due to recurrence of chest discomfort. At peak stress and 2 minutes into

recovery, 1.5-2 mm ST-segment depression is noted in the anterolateral leads. His imaging study is
inconclusive due to submaximal workload.

The most appropriate next test that also minimizes radiation exposure would be which of the

following?

a. Pharmacologic myocardial perfusion imaging.

b. Diagnostic cardiac catheterization.

c. Coronary computed tomography (CT) angiography.

d. Dobutamine stress echocardiography.

e. Stress echocardiography.

152. b.
Diagnostic cardiac catheterization.
This patient has a high pretest likelihood of coronary artery disease based on his risk factors and
characteristics of typical angina. Electrocardiographic changes during peak stress are suggestive of
inducible ischemia, even though imaging was “inconclusive.” Risk-benefit ratio must be taken into
account with diagnostic imaging studies. Although repeating the stress test would probably confirm
the clinical suspicion, he will still need further evaluation of the coronary arteries.

Coronary CT angiography is an excellent tool in patients with equivocal or nondiagnostic stress

tests. However, in patients with a high pretest likelihood of coronary artery disease, coronary CT

angiography is considered an inappropriate test, without added value of layered testing. Among

the choices provided, triage to diagnostic cardiac catheterization with the option for

revascularization based on anatomy would be the most appropriate decision in order to avoid

additional radiation exposure.

153. A 48-year-old man with a history of hypertension comes to your clinic to establish care. He
states that he was told a few years ago that he had high cholesterol but he did not take further

action. He does not smoke and drinks alcohol rarely. He states he eats a balanced diet with limited
carbohydrates. His only medication is amlodipine. His blood pressure is 129/81 mm Hg, heart rate

is 76 bpm, and the remainder of his exam is unremarkable. His body mass index is 24.8 and his

glycated hemoglobin is normal.

A fasting lipid panel from this morning reveals a total cholesterol of 210 mg/dl, triglycerides 630

mg/dl, high-density lipoprotein (HDL) 42 mg/dl. His low-density lipoprotein (LDL) cannot be

calculated. His thyroid-stimulating hormone is normal.

In addition to recommending therapeutic lifestyle changes, which of the following is the next most
appropriate step in his care?

a. Recheck lipids in 3 months.

b. Start low-dose aspirin.

c. Switch amlodipine to hydrochlorothiazide.

d. Start atorvastatin.

e. Start fenofibrate.
153. e.
Start fenofibrate.
The patient has very high triglyceride level, as defined by a level >500 mg/dl in the Adult Treatment
Panel III guidelines. Such a severe elevation in triglycerides can lead to pancreatitis, so priority
should be made to reduce levels as quickly as possible with effective triglyceride-lowering

medications. Appropriate first-line choices include fibrates and niacin. Statins are less efficacious on
this front, and would only be considered for LDL-lowering purposes once triglyceride levels are

below 500 mg/dl.

Therapeutic lifestyle changes should be recommended, but are not adequate to prevent

pancreatitis in such a setting. Certain cardiovascular medications, such as hydrochlorothiazide, can


increase triglyceride levels, thus would be detrimental in this case. Amlodipine has no such effects.

The calculated LDL value in a lipid panel will be falsely low in the setting of elevated triglycerides,
thus cannot be properly interpreted. Alternative approaches are repeating the lipid panel when

triglycerides are better-controlled, measuring direct LDL, or following a non-HDL cholesterol

treatment goal.

154. A 50-year-old man with hypertension comes to your office for consultation on management of
his cardiovascular risk. He has no symptoms suggestive of cardiovascular disease. His

hypertension has been treated with hydrochlorothiazide and lisinopril, and his treated resting

blood pressure is 130/80 mm Hg. His fasting lipid profile reveals a total cholesterol of 170 mg/dl,
low-density lipoprotein (LDL) of 110 mg/dl, high-density lipoprotein of 45 mg/dl, and triglycerides
of 75 mg/dl. His fasting glucose is 90 mg/dl. He does not smoke cigarettes. He has no family history
of early-onset atherosclerotic cardiovascular disease (ASCVD).

Which of the following therapeutic options is supported by the risk profile?

a. Omega-3 fatty acids.

b. Statin therapy.

c. No lipid-lowering medications.

d. Nicotinic acid.

e. Fibrate therapy.

154. c.

No lipid-lowering medications.
The 2013 American College of Cardiology (ACC)/American Heart Association (AHA) cholesterol

guideline identified four major statin benefit groups. Using the 2013 ACC/AHA algorithm, this

patient does not have clinical ASCVD, LDL ≥190 mg/dl, or diabetes. Using the Pooled Cohort

Equation, his 10-year risk of coronary death, nonfatal myocardial infarction, fatal stroke, and

nonfatal stroke is 3.7%. This risk level falls below the 7.5% threshold at which the benefit of statin
therapy significantly exceeds the risk of incident diabetes. While further risk assessment may be
considered, such as coronary artery calcium scoring, based on the available risk profile, no lipid-
lowering medications are recommended. There is no randomized clinical trial evidence that

initiation of a fibrate, niacin, or fatty acids would be beneficial in primary prevention in this patient.

155. A 65-year-old woman with hypertension comes to your office to seek consultation on

management of her cardiovascular risk factors. She has no history or symptoms suggestive of

cardiovascular disease. Her hypertension has been treated with hydrochlorothiazide and lisinopril and
her resting blood pressure is 130/80 mm Hg. On fasting laboratories, she has a total cholesterol of
190 mg/dl, low-density lipoprotein (LDL) of 120 mg/dl, high-density lipoprotein (HDL) of 40

mg/dl, triglycerides of 150 mg/dl, and glucose of 90 mg/dl. She currently smokes half a pack per

day of cigarettes.

After counseling her on smoking cessation and discussing the risks and benefits of testing, you

recommend that she consider which of the following?

a. Obtain an exercise stress test.

b. Measure high-sensitivity C-reactive protein (hs-CRP) level.

c. Start moderate- to high-intensity statin therapy.

d. Measure glycated hemoglobin (HbA1c) level.

155. c.

Start moderate- to high-intensity statin therapy.

The 2013 American College of Cardiology (ACC)/American Heart Association (AHA) cholesterol

guideline identified four major statin benefit groups. Using the 2013 ACC/AHA algorithm, this

patient does not have clinical atherosclerotic cardiovascular disease, does not have familial

hyperlipidemia with an LDL above 190, and does not have diabetes. She is between the age of 40
and 75 years, and therefore, the Pooled Cohort Equation should be used to estimate her 10-year

atherosclerotic risk. This calculated to a 10-year atherosclerotic risk of 15.4%, well above the

recommended 7.5% risk threshold for initiating statin therapy, and therefore, additional testing

with hs-CRP is not needed.

She has already had appropriate screening for diabetes with a fasting glucose level, therefore,

HbA1c is not necessary. The patient has no symptoms suggestive of ischemic heart disease, and

therefore, stress testing in this asymptomatic patient is not appropriate.

156. A 55-year-old woman is seen for a second opinion on her lipid management. She has a recent

diagnosis of non–insulin-dependent diabetes and no known history of cardiovascular disease. At

the time of the diagnosis of her diabetes 6 months ago, her fasting lipid profile revealed a total
cholesterol of 200 mg/dl, low-density lipoprotein (LDL) of 110 mg/dl, high-density lipoprotein of 45

mg/dl, and triglycerides of 225 mg/dl. Although pharmacotherapy for her lipids was recommended

by her primary care physician, she opted for lifestyle changes first.

After 6 months of therapy for her diabetes and diet modification, her lipid profile is unchanged. Her
body mass index is 28 kg/m2 and her glycated hemoglobin is 6.9%. She has a blood pressure of

135/80 mm Hg and does not smoke. No proteinuria noted on urinalysis.

In addition to continued lifestyle modification with regular exercise, diet, and modest weight loss for
a body mass index goal <25, which of the following do you recommend as the next step for this
patient?

a. Amlodipine 5 mg.

b. Atorvastatin 20 mg.

c. Niacian ER 500 mg bid.

d. Fenofibrate 145 mg.

e. No change in medications at this time.

156. b.
Atorvastatin 20 mg.
The 2013 American College of Cardiology (ACC)/American Heart Association (AHA) cholesterol

guideline identified four major statin benefit groups. Using the 2013 ACC/AHA algorithm, this

patient has diabetes with a Pooled Cohort Equation estimated 10-year atherosclerotic risk of 4.4%.

As per the 2013 ACC/AHA algorithm, in diabetics (40-75 years, LDL 70-189 mg/dl), with a 10-year

atherosclerotic cardiovascular disease risk <7.5%, a moderate-intensity statin should be initiated.

This patient's blood pressure is <140 mm Hg, and antihypertensive medication is not indicated. The
2013 guideline did not find data supporting the routine use of nonstatin drugs combined with

statin therapy to reduce further atherosclerotic cardiovascular events. Because her risk is <7.5%, a
high-intensity statin is not indicated.

157. A 56-year-old African American woman with a history of hypertension treated with

hydrochlorothiazide, and a family history of premature coronary disease presents to your clinic.

She is a current smoker. Her blood pressure is 140/70 mm Hg, heart rate is 55 bpm, and body mass

index is 28.9.

Her lipid profile after a 10-week very low fat diet with increased physical activity is as follows:

 Total cholesterol 240 mg/dl

 Triglycerides 450 mg/dl

 High-density lipoprotein (HDL) 50 mg/dl

 Low-density lipoprotein (LDL) 145 mg/dl

Which of the following is the first therapy to initiate?

a. Omega-3 fatty acids.

b. Fibrate.

c. Statin.

d. Nicotinic acid.

157. c.
Statin.
The 2013 American College of Cardiology (ACC)/American Heart Association (AHA) cholesterol

guideline identified four major statin benefit groups. Using the 2013 ACC/AHA algorithm, this

patient does not have clinical atherosclerotic cardiovascular disease, does not have familial

hyperlipidemia with an LDL above 190 mg/dl, and does not have diabetes. She is between the age

of 40 and 75 years, and therefore, the Pooled Cohort Equation should be used to estimate her 10-

year atherosclerotic risk. This calculated to a 10-year atherosclerotic risk of 10.6%, and is probably
even higher given her family history of premature coronary artery disease, thus warrants treatment
with a statin to reduce this risk.

158. Effective dose-reduction strategies include appropriate patient selection.

For which of the following patients is coronary computed tomography angiography (CCTA)

appropriate?

a. A 68-year-old man with high risk for coronary disease with acute chest pain.

b. A 50-year-old man unable to undergo risk assessment.

c. An asymptomatic 59-year-old man with a history of coronary artery bypass grafting surgery 2

years ago.

d. A 51-year-old woman with low risk for coronary disease and typical angina.

e. A 45-year-old male executive with intermediate risk for coronary disease and worsening chest

pain with activity.

158. d.

A 51-year-old woman with low risk for coronary disease and typical angina.

Appropriate patient selection is an important first step in dose reduction. In general, due to its high
sensitivity and negative predictive value, CCTA performs well in ruling out coronary artery disease
in low-to-intermediate risk individuals. In patients with a high likelihood of coronary disease

needing triage to invasive assessment, CCTA results in unnecessary radiation exposure with little
added value. In such patients, CCTA is considered inappropriate.

159. A 53-year-old woman presents to the heart failure clinic for evaluation and management. She
experiences angina on minimal exertion despite medical therapy. She had an implantable
cardioverter-defibrillator (ICD) inserted 3 months after her most recent myocardial infarction (MI).
She had suffered two prior MIs and had a percutaneous coronary intervention (PCI) of the left
anterior descending (LAD) and right coronary artery. Her most recent ejection fraction is 25% with a
dilated left ventricle and no valvular abnormalities. Her electrocardiogram shows normal sinus
rhythm with a QRS of 110 msec and Q waves in the inferior leads and precordial leads.

A coronary angiogram shows patent right and left circumflex coronary arteries with a subtotal highly
calcified occlusion of the proximal LAD. The distal LAD is collateralized from the right and left
circumflex coronary arteries.

PCI was not technically feasible. She undergoes rest/4-hour redistribution thallium. Results show 60%

redistribution in the distal half of the anterior wall and septum.

Which of the following is the next best step in this patient's care?

a. Surgical revascularization.

b. Biventricular ICD.

c. Intensify medical treatment.

d. Transplant evaluation.

e. Cardiac magnetic resonance imaging (MRI).

159. a.
Surgical revascularization.
This patient is presenting with ischemic cardiomyopathy and New York Heart Association class III-IV

heart failure. She has advanced left ventricular dysfunction. On the single-photon emission

computed tomography images, there is evidence of thallium uptake at 4 hours in her LAD

distribution. Therefore, this area may improve with revascularization.

In a meta-analysis comparing all imaging modalities that can identify hibernating myocardium and

predicting recovery of regional function after revascularization, positron emission tomography

(PET) using fluorodeoxyglucose (FDG) was the most sensitive, while dobutamine was the most

specific, but when global recovery after revascularization was analyzed, thallium and PET were very
similar.

She does not meet criteria for upgrading to a biventricular ICD based on her QRS. An MRI would

not provide further information and is contraindicated in patients with an ICD. If she does not

improve with revascularization, then cardiac transplant should be considered.

Treatment of triglycerides and non-HDL is secondary to initiating a statin, and there has been no
evidence to support treatment with a fibrate or nicotinic acid to optimize non-HDL levels.

160. A 68-year-old woman with hypertension and diabetes is being referred to the nuclear

laboratory for a myocardial perfusion study. The nurse asks you to choose the imaging protocol for
the patient. The patient’s body mass index (BMI) is 38 kg/m2. The patient walks her dog routinely.

The patient lives close to the clinic where you would perform the test and is retired.

Which of the following protocols would optimize patient images?

a. One-day low-dose rest/high-dose stress with technetium.

b. Two-day high-dose rest/stress imaging with technetium.

c. One-day rest/stress dual-isotope imaging.

d. One-day low-dose stress/high-dose rest with technetium.

e. One-day stress/redistribution/reinjection with thallium.

160. b.
Two-day high-dose rest/stress imaging with technetium.

Ideally, stress and rest imaging with technetium-99 agents should be performed on two separate

days to avoid having residual activity from the first study contaminating the second study in

overweight patients (i.e., >250 lbs. or body mass index >30 kg/m2) or in women where significant
breast attenuation is anticipated. A low dose of technetium-99 radiotracer may result in

suboptimal images and a 2-day imaging protocol is preferred in this patient population.

The effective dose of radiation is higher with dual-isotope and should be avoided. The rest-stress
protocol is a good option if the patient could not have a 2-day protocol. The patient does not

require an assessment for myocardial viability. The low-dose stress should not be performed in this
population because of the potential poor quality images on the stress study.
Hear
tFai
lur
eandCar
diomyopat
hies

QUESTI
ONS:

1. A35- year-ol
dmanpr esent
sf orevaluat
ionofhear
tfail
ure.Hi
swor kuphasincludedan
echocardiogram t
hatdemonstrat
esadi lat
edcar di
omyopathywithanej ecti
onfract
ionof25%,
l
eftventricul
ar(LV)end-
diastol
icdimensionof6. 5cm,andmi ldmitralregurgi
tati
on.Cardi
ac
cathet
erizati
ondemonstr
atedangiographicall
ynormalcor
onaryarter
ies. 

Laborator
yr esult
s,i ncl
udingthyroid-sti
mul at
inghormone,humani mmunodefi
ciencyvir
us,
completebloodcel lcount,
metabolicpanel ,andfer
ri
ti
n,ar
eallwithi
nnor
malli
mits.Hisbrot
her
wasr ecentl
ydi agnosedwi thheartf ail
ure,hisfat
herunderwentaheartt
ransplant,andhis
pater
nalgrandfatherdiedsuddenlyatt heageof30ofa“ hear
tattack.

Whichoft
hef
oll
owi
ngi
srequi
redt
omakeadi
agnosi
soff
ami
li
alcar
diomyopat
hyi
nthi
s
pat
ient
?

a. Echocardiographyofhisfi
rst-
degr
eerelat
ives.
b. Endomy ocardialbi
opsy.
c. Cardiacmagnet icr
esonanceimaging(MRI).
d. Genetictesti
ngofhisbrother.
e. Nof urt
herdiagnosti
cstudies.

ANSWER:

1. e. Nof
urt
herdi
agnost
icst
udi
es.

Thedi agnosisoff ami li


alcar di
omyopat hyst i
llremai nsaclinicaldiagnosisandr equir
est hr
ee
generati
onst obeaf fect ed.Itisusual l
yaut osomaldomi nant.Genet i
ct esti
ngcanbeusef ulbut
i
snotcur rentl
ynecessar ytoestablishthedi agnosi sofaf amili
alcardiomyopat hy.Mut ati
onsin
sarcomer i
cproteins,t hecyt oskeleton,calcium fluxpr ot
eins,andmet aboli
cpat hwayshaveal l
beendescr i
bed.Ifgenet ictesti
ngi sconsider ed( alt
houghnotr equir
edf ordiagnosis),t
hemost
aff
ected proband shoul d be tested.Ther e ar e no specifi
c pat hologicfindings atbi opsy,
alt
hough genet ict esting ofcar diomyocyt es coul d be performed.Si mil
arly,there are no
pathognomoni ccar diacMRIpat t
ernsf orfami li
alcardiomyopat hy.Althoughechocar di
ography
ofallhisfir
st-degreer elat
iveswoul dbeadvi sabl e,i
tisnotr equiredt omaket hediagnosisof
famili
alcardi
omyopat hy.

2. Whi
chofthef
oll
owi
ngechocar
diogr
aphi
cfi
ndi
ngsi
smostconsi
stentwi
thadi
lat
ed
car
diomyopat
hy?
 

  Endsyst
oli
c(mm) Enddi
ast
oli
c(mm) Post
eri
orwal
l(mm) Sept
alwal
l(mm)
A.2.
0 5.
0 0.
8 0.
8
B.2.
0 3.
5 2.
0 2.
0
C.5.
0 6.
5 1.
0 1.
0
D.1.
0 3.
5 1.
0 2.
5
E.2.
0 6.
0 1.
4 1.
4
a.RowA.
b.RowC.
c. RowD.
d.RowB.
e. RowE.

ANSWER

2. b. RowC.

Dil
atedcar diomyopathies( opti
onC)ar echaracteri
zedbyani ncr
easedend- diastol
icvolume,
decreasedej ect
ionfraction,andi ncr
easedeccent r
icmyocar di
almass.Opt i
onA i snor mal
dimensi ons.OptionB i sconsi stentwithar estr
ictivecardiomyopat hywithl eftventr
icul
ar
hypertrophy (e.
g.,amyl oid).Opt ion D demonstrates a smal lventricl
e with supranormal
contractil
it
y and asymmet r
ic hypertr
ophy (e.
g.,hyper t
rophic cardiomyopathy).Opt i
on E
demonst r
atesalargehear tbutnor malcontr
acti
li
tywi thconcentri
cleftventr
icularhypert
rophy
consistentwithanathlete’sheart.

3. A58- year
-ol
dwomanpr esentswit
hacuteshor t
nessofbreathandchestpain.Shehad
j
ustlefthomeaf t
err
ecei
vingacal
lthatherhusbanddiedsuddenlyatwork.
 Shehasneverbeen
hospit
alizedandhasnocardi
ovascularhi
stor
yorriskfact
ors.Shelooksanxiousonexam and
di
aphor eti
c. 

Herbl oodpr essurei s170/80mm Hg,herhear trateis110bpm,andherr espir


atoryr at
eis18
breaths/mi nute.Venouspr essur
esar enotel evat
ed;lungsarecl ear
;S4 on cardiacexam.
Abdomenandext remiti
esareunremar kable.Tr
oponi nIi
s1.0ng/dl;B-t
ypenatriur
et i
cpeptide
(BNP)i s500 pg/ ml.Herel ectr
ocardiogram showsdeep ant er
iorT- waveinver si
ons.Her
echocar diogram demonst r
atesleftventri
cular(
LV)dysfuncti
onwit
hanej ecti
onfraction(EF)of
30%.Car diaccatheteri
zati
onshowsnor malcoronariesandLVapicalaneurysm.Thepat i
entis
start
edonmet oprolol.

Whi
choft
hef
oll
owi
ngt
her
api
esi
smostappr
opr
iat
eint
hispat
ient
?

a. Apixi
ban.
b. Colchi
cine.
c. Sacubit
ril
/val
sart
an.
d. Implant
ablecardi
over
ter
-def
ibr
il
lat
or(
ICD)i
mpl
ant
ati
on.
e. Lisi
nopril
.

ANSWER:

3. e. Li
sinopr
il
.

Thispati
entmostl
ikelyhasTakotsubocar diomyopat hy,whichi sclassi
cal
lyseeni
nmi ddle-
agedwomenwhoexper i
encesevereemot ionaldi st
ress,althoughanysi tuati
ont
hatcoul d
resul
tinamassivecat echol
aminereleasecanr esultinthi
st ypeofmyocar di
ali
nsult
.The
ventr
icul
arwal
lmot i
onabnor mali
tyisdisti
nctivewi t
hpr eservationofthebaseandal arge
segmentofapi
calball
ooning.Othert
ypesofwal lmot ionabnor mali
tieshavebeendescri
bed,
butaregenerall
ynotincoronar
yvasculardi st
ri
buti
ons.Characteri
sti
cal
ly,t
heBNPelevat
ionis
greatl
youtofproporti
ontothet
roponi
nr elease,whichisgener
allymodest.Despi
tet
heseverit
y
oft he wal
lmot ion abnor
mali
ty,myocar dialscarri
ng is di
ff
icultto documentbycar di
ac
magnet i
cresonance.Commonly,compl
et enormalizati
onofLVf uncti
onisexpect
ed.

Alt
hough there are no speci f
ic guidelines to di
rectt herapy,medi cati
ons thatt argett he
neurohormonalaxi s and car di
ac r emodel ing are appr opr i
atei nt his pat
ientpopul ati
on.
Therefor
e, angiot
ensin- conver t
ing enzyme ( ACE) i nhibitors woul d be i ndicated her e.
Anti
coagulati
onwi t
hnovelant icoagulant s,suchasapi xibani npat i
entswi thapicalaneurysms,
hasnotbeenst udied.Ther eisnoevi denceofper i
carditis;therefore,t
reatmentwi thcolchicine
i
snoti ndi
cated.Sacubit
r il
/valsartanhasnotbeenst udi edi npat i
entswi t
hacutehear tfail
ureor
i
nt hosewhohavenotpr eviouslyt oleratedACEi nhibitors.I CDi mplantati
onisnoti ndicated
heresincethisisthefirstdemonst rationofl ow EFint hispat i
entandmyocar dialrecover yi s
common.

4. A25- year-
oldmanpr esentsforevaluat
ionofhypert
rophiccardiomyopathy(HCM) .He
hasnoexer t
ionalsymptoms.Hi sf
atherdiedsuddenlyattheageof40year s.AHol t
ermoni t
or
showsoccasi onalpremat
urevent r
icul
arcompl exes(PVCs),butnononsust ainedventr
icul
ar
tachycar dia(NSVT).OnaBr uceprotocolstr
esstest,
hehasSTdepr essionswithexer
ciseanda
hypot ensiver esponseto exerci
se.Theaor t
icoutfl
ow vel
ocityis4 msec;t heseptalwall
thicknessi s30mm.

Whi
choft
hef
oll
owi
ngi
sthemostappr
opr
iat
ether
apy?

a. Al
coholsept alabl
ati
on.
b. Surgicalmyect omy.
c. Me t
opr ol
ol.
d. Disopyrami de.
e. Animpl antablecardi
over
ter
-def
ibr
il
lat
or.

ANSWER:

4. e. Ani
mpl
ant
abl
ecar
diover
ter
-def
ibr
il
lat
or.

Theassessmentofsuddendeat hr
iski
nHCM i spar toft
herout
ineevaluati
onofapatientwi
th
thi
sdiagnosis.Thefi
vetradi
ti
onalri
skfact
orsincludesyncope,
familyhist
oryofsuddendeat
h,
sever
eleftventr
icul
arhypert
rophy(
30mm) ,hypotensionwit
hexerci
se,andNSVT. 

Interesti
ngly,aseverer esti
nggr adi
enthasnotbeenest abli
shedasar iskfactorforsudden
cardiacdeat h(SCD).Lategadol i
nium enhancementsuggest i
veoffi
brosi
sandscaroncar di
ac
magenet i
cr esonanceimagi ngmayal sobepr edi
cti
veofSCD overandabovecl ini
calri
sk
factors.Inthispati
ent,
SCDpr eventi
onisparamountsincehehast hreeri
skfactorsforSCD,f
or
exampl e,fami l
yhistor
y,sever eleftventr
icul
arhypertrophy,andahypot ensi
ver esponseto
exercise.ThePVCsandr est
inggradientar
enotpredi
ctive. 

Medicaland/orsurgi
calther
apyaregeneral
lyreservedforsymptomati
cpat
ient
sandmost
aut
hor i
ti
esfavormyectomyingoodsur
gicalcandidatesoverabl
ati
on,i
fmedi
calt
her
apyf
ail
s
tor
elievesymptoms.
5. A60-year -
oldmani sadmit
tedtot hehospi
talwi
thdyspneaandabdomi nalbloati
ng.He
hasbeenadmittedtwot i
mesthisyearforacutedecompensatedheartfai
lur
e.Therehadbeen
nopri
orcar
diovascularhi
stor
y.Heistakingfur
osemide,di
goxi
n,warfari
n,andmetoprolol

Onexami nation,
heappear schroni
cal l
yil
l.Hisbloodpressureis95/75mm Hgandhear tr
ateis
95bpm wi thani rr
egul
arr hyt
hm.Thecent r
alvenouspr essureistothemandi bl
eintheseated
posit
ion;thecar oti
dupstrokesareweak.Ral esar epresentinthel ungbases.Auscultat
ion
notablef
orr espi
rophasicholosyst
olicmur muroverprecor di
um andS3 gal
lop.Theabdomeni s
di
stended;li
verisenlarged.Theextremiti
esar ecool
,andt helegsareedematous. 

TheB-typenatri
uret
icpepti
de( BNP)is1400pg/ ml,f
erri
ti
nis89mcg/ L,t
ransferri
nsaturation
i
s32%,bl oodureani t
rogenis60mg/ dl
,andcr eati
nineis2.5mg/ dl
.Anel ectrocar
diogram
showsatri
alfi
bri
ll
ati
on,ri
ghtwardaxis,andnarrowQRSwi thprecordialQRSampl i
tudeof4mV.
Anechocardi
ogram demonst r
atesanej ecti
onfract
ionof50%wi thleftventr
icularhypertr
ophy
(LVH;20 mm post eri
orwal l
),moder atemi t
ralregurgi
tati
on,tricuspidregur gi
tati
on,smal l
peri
car
dialef
fusi
on,andseverebiatr
ialenlar
gement .

Whi
choft
hef
oll
owi
ngi
sthemostl
ikel
ydi
agnosi
s?

a. Cardi
acamyl oi
dosi s.
b. Hypert
rophiccardiomyopathy.
c. Heartf
ail
urewi t
hpr eser
vedeject
ionf
ract
ion(
HFpEF)
.
d. Constr
ict
ivepericardi
ti
s.
e. Hemochromat osis.

ANSWER:

5. a. Car
diacamyl
oidosi
s.

HFpEFi sadi agnosisofexclusi


on.Thispat i
enthassever ebiventricul
arheartfai
lur
e,alow
bloodpr essur ei ntheabsenceofvasodi l
ators,andaver yelevatedBNPdespi t
et henormal
eject
ion f raction.Moreover,the elect
rocardiogram shows low vol t
age and an anteri
or
myocar dialinfarcti
ondespi
tetremendousLVH, noanter
iorwallmotionabnor mali
ty,
andsevere
biatr
ialenlar gement.Thesefindi
ngsar eallhighlysuggest
iveofar estr
ict
ivecardi
omyopathy
(e.g.
,amyl oidosis).
 

Hemochr omat osi


sdoesnottypi
cal
lypr esentwit
har estri
cti
vecardi
omyopathy,butratherwi
th
adi l
atedcardiomyopathy.I
naddit
ion,thet r
ansf
er r
insat urati
onof32%i snotconsistentwi
th
thisdiagnosis.Theveryelevat
edBNPi snotchar acteri
sticofeit
herconstri
cti
onorHFpEF.
Hypertrophiccardi
omyopathycanpr esentinit
sl atest ageswi t
hsevereheartfai
lur
e,butthe
eject
ionfracti
onisgeneral
lyi
mpair
ed, alt
houghthehear tmaynotbedilated.

6. A 60-year-ol
d woman pr esentst o your off
ice on advi
ce ofher brother
,who
recommendst hatshebeeval
uat edforhypertr
ophiccardiomyopat
hy(HCM).Shebringscopies
ofhi s medi
calr ecor
ds.He had an out -of
-hospitalcardi
ac arr
estand for
tunatel
y was
resusci
tat
edsuccessfull
y.Hi
sechocar di
ogram demonst r
atedHCM wit
ha1. 8cm ventricul
ar
septum.Shehasnothadgenot ypingper f
ormed. 

Yourpat
ienti
sasympt
omat
icandherexam,i
ncl
udi
ngcar
diovascul
ar,i
sunr
emar
kabl
e.You
obtainanechocardi
ogram,whichistechni
cal
lyverydiff
icul
t,butyouareconf
identt
hather
septum i
snotthi
ck,al
thought
heotherlef
tvent
ri
cularwall
sarenotwellseen.
 
Atthispoi
nt,
yourecommendshehavewhi chofthefol
lowing?

a. Cardiacmagnet i
cresonancei
maging(MRI
).
b. Exercisestr
esstest.
c. 24-hourHol t
ermonitor
.
d. Genetictest
ingusinganHCM panel.
e. El
ectrophysiol
ogystudy.

ANSWER

a. Car
diacmagnet
icr
esonancei
magi
ng(
MRI
).

HCM hashet
erogeneousphenot ypi
cexpr essi
on.Alt
houghyoucanexcl udesept alhyper
trophy,
shemaymanifestHCM throughhypertrophyinotherpart
sofherleftvent
ricle.Cardi
acMRIhas
aClassIi
ndi
cati
ontoassessf orHCM whenechocar di
ographyi
sinconclusiveofthediagnosis.
Nei
theraHol
ternorexer
cisestresstestwi l
lconf
ir
m orref
utethediagnosisofHCM.  

I
fgenetictest
ing i
s desir
ed,itwould be more appropri
atet otestt
he brot
herwho has
documentedHCM,andt hengenotypeyourpat
ientf
orani dent
if
iedmutat
ion.Fur
thert
est
ingi
s
i
ndicat
edbecauseyourpati
entcouldbeatri
skforsuddendeathifsheisf
oundtohaveHCM.

7. A17-year-ol
dwomanwhoi sahi ghschoolsoccerpl
ayerpresent
st otheemergency
room wi
threport
sofchestpainfor1week.Shereport
shavingalow-gradefeveranddi
arr
hea.
Sheisschedul
edtoplayinherteam’sst
atechampionshi
psoccergamenextweek.  

Herphysi calexaminati
onshowshert obeinnoappar entdistressthoughsomewhatanxi ous,
herpulsei s100bpm wi t
hsomepr emat urebeat
s,andherbl oodpr essur
ei s110/70mm Hg.
Shehasanor malS1 andS2 withnoS3 ormur mur,herneckvei nsar enotdi st
ended,andt he
remainderofherexami nationisunr
emar kabl
e.Herelectrocardiogram isnotableforpremature
ventr
icularcont r
acti
onsand l ow volt
age.Hercar diact roponi nisundet ectabl
e.A limited
echocardiogram intheemer gencyroom showsal eftventricularejecti
onfracti
onof50%wi th
noper i
cardialef
fusi
on.

Whi
choft
hef
oll
owi
ngi
syourbestcour
seofact
ion?

a. Dischar
gehomeonl ow-dosebet
a- bl
ocker
s,butadviseshenotplayinthesoccergame.
b. Dischar
gehomewi t
hnomedi cat
ionsandcleartoplayinthesoccergame.
c. Admi ttothe hospit
alforfurt
herdi agnost
ictest i
ng,incl
uding cardi
ac magneti
c
r
esonancei
magi
ng(
MRI
).
d. Admi
ttoobser
vat
ionst
atusover
nightanddi
schar
gei
fser
ialcar
diacenzymesr
emai
n
negat
ive.
e. Di
schar
gehomeonl
ow-
dosebet
a-bl
ocker
swi
thcl
ear
ancet
opl
ayi
nthesoccergame.

ANSWER

c. Admi
ttot
he hospi
talf
orf
urt
herdi
agnost
ict
est
ing,i
ncl
udi
ng car
diac magnet
ic
r
esonancei
magi
ng(
MRI
).

Myocar di
ti
scanpr esenti nvariablef ashion.Shef ul
fil
scriteriaproposedbyt heEuropean
SocietyofCar di
ologyf ormyocar diti
s.Sheshoul dnotpar ticipateincompeti
ti
vesportswith
possibl
eacut emyocar dit
is,evenwhi leonbet a-bl
ockers.Furtherdiagnost
ict
esti
ngtoconfir
m
whethershehasmyocar dit
isiswar ranted,anditwouldber easonabl et
oobtai
nacar di
acMRI
wit
hgadol i
nium.Anendomyocar dialbiopsyshouldalsobeconsi der
ed. 

8. A17-year-oldmalehighschoolstudenti
nDall
as,Texas,whowaspr evi
ouslyhealt
hy
present
edwithacut esyst
oli
cheartf
ail
ureintheset
ti
ngofaf ebr
il
eill
nessinthesummer .He
doesnotdri
nkal cohol
,smoke,orusedrugsandisnotsexual
lyacti
ve.Endomyocar
dialbi
opsy
confi
rmedt
hedi agnosisofmyocardi
ti
s.

Whi
choft
hef
oll
owi
ngi
sthemostl
ikel
yvi
ralet
iol
ogyofhi
sil
lness?

a. I
nfl
uenzaB.
b. Par
vovirusB19.
c. CoxsackieB.
d. Humani mmunodef
ici
encyvi
rus(
HIV)
.
e. Adenovir
us.

ANSWER

c. Admittothe hospi
talf
orf
urt
herdi
agnost
ict
est
ing,i
ncl
udi
ng car
diac magnet
ic
resonancei
magi
ng(MRI).

Myocar di
ti
scanpr esenti nvariablef ashion.Shef ul
fil
scriteriaproposedbyt heEuropean
SocietyofCar di
ologyf ormyocar diti
s.Sheshoul dnotpar ticipateincompeti
ti
vesportswith
possibl
eacut emyocar dit
is,evenwhi leonbet a-bl
ockers.Furtherdiagnost
ict
esti
ngtoconfir
m
whethershehasmyocar dit
isiswar ranted,anditwouldber easonabl et
oobtai
nacar di
acMRI
wit
hgadol i
nium.Anendomyocar dialbiopsyshouldalsobeconsi der
ed. 

9. Youareevaluati
nga25- year-ol
dAf r
ican-Ameri
canmani nt heemergencydepart
ment
(ED)
.Hehadbeenwat chinghisnephewpl ayfootbal
landhadsyncopeandcol l
apsedwhenhe
waswalking up t
hestadium steps.When t hepar amedicsarri
ved,theynoted hewasi n
i
nter
mit
tenthi
gh-gradeatri
ovent
ri
cular(AV)blockandtranspor
tedhimtoyourhospital
.

Anelectrocar
diogram i
ntheEDconfir
mstheirf
indi
ngs.I
nt al
kingwithhim,herepor
tshehas
beenmor eshor tofbr
eaththepast2monthscomparedwithhisbaseli
ne.Onexaminat
ion,he
hadjugularvenouspressur
eof12cm andanaudibl
eS3.ChestX-rayshowedcardi
omegalybut
noinfi
lt
rates.

Youobt
ainanechocardi
ogr
am,whichr
eveal
shislef
tvent
ri
cul
ar(LV)eject
ionf
ract
ioni
s25%
wit
hanLVdiast
oli
cdimensi
onof6.
5cm.Cardi
acenzymesarenegat
ive.

Whi
choft
hef
oll
owi
ngdi
agnost
ict
est
swoul
dyounextr
ecommend?

a. Exer
ciseper
fusi
oni
magi
ng.
b. 48-hourHoltermoni tor
.
c. Endomy ocardialbi
opsy.
d. Lef
t-heartcatheter
izati
on.
e. El
ectrophysi
ology( EP)test
ing.

ANSWER

9. c. Endomyocar
dialbi
opsy.

Thepr esentationofacutesyst oli


chear tfail
urewi thhigh- gradeAVbl ockr aisesthespecterof
myocar di
tiscausedbygi antcel lmyocar di
t i
sorani nf
il
trati
vepr ocesssuchassar coi
dosis.The
AHA/ ACC/Eur opeanSociet yofCar di ologysci entif
icst atementont heroleofendomyocar dial
biopsyi nt hemanagementofcar diovascul ardi seasehasa Cl assIr ecommendat ion f
or
endomyocar di
albiopsyf ornew- onsethear tfai
lureof2weekst o3mont hsdur at
ionwi t
ha
dil
atedLVandnew vent ricul
arar rhyt hmi as,second-ort hi
rd-degreehear tblock,orfail
ureto
respondt ousualcar ewithin1t o2weeks.Atage25,cor onaryarterydiseasewoul dbemuch
l
essl ikel
y.Ther eisnoi ndicati
onf oradi agnosticEPst udyor48- hourHol termonitorinthe
setti
ngofacut emyocar diti
s. 

10. A 38- year-ol


d woman pr esentstot he emergency depar
tmentwi t
h 5 days of
progressiveexer
t i
onaldyspnea,fati
gue,andmildl
owerext r
emityedema.Shehadprevi
ously
beenf eel
ingwell.Hermedi calhi
stor
yi snot
abl
eforhypothyr
oidi
sm secondar
ytoHashi
moto's
thyroi
diti
s.Heronl ymedi cati
onisl evot
hyr
oxi
ne.Herhear trat
ei s105bpm andherblood
pressureis92/68mm Hg.  

Onphysi calexamination,ther
ei smoder atejugularvenousdi stent
ion,t
achycardicbutregular
rhythm withoccasionalprematurebeat s, anS3, bi
basil
arral
es, andcoolperi
pher alext
remit
ies.
Shei sadmi t
tedtot hecar di
aci ntensivecar eunitforfurtherwor kup.Anechocar diogram
demonst ratessevereleftventr
iculardysf unctionwithanej ectionfract
ionof23%.Thenur se
notif
iesyout hatthepat i
entishavi ngmul tiplerunsofmul ti
focalnonsustai
nedvent ri
cular
tachycardia.

Whi
choft
hef
oll
owi
ngi
sthemostl
ikel
yet
iol
ogyoft
hepat
ient
'scar
diomyopat
hy?

a. Cardiacamyloi
dosis.
b. Takotsubo(str
ess)cardi
omyopat
hy.
c. Chagasdisease.
d. Cardiacsar
coidosi
s.
e. Giantcel
lmyocarditi
s.

ANSWER:

10. e. Gi
antcel
lmyocar
dit
is.

Thispatient'
spr esentat
ion i snotableforr apidl
ypr ogressi
veheartfai
luresymptomsand
ventr
icul
arar rhythmi as.Takot subo (st
ress)car di
omyopat hytendsto presentin an ol
der
pati
entpopulat i
on( cl
assicallypostmenopausalwomen)andmaybepr ecededbyanacut e
emotionalorphysi calstress;neitherisment ionedher e.Cardi
acamyloi
dosisalsot endsto
aff
ectol derindividuals.Al though cardiac sarcoi
dosis may be associ
ated both withlef
t
ventr
icul
ar dysf unct i
on and ar r
hythmias,i ti s usuall
y seen inindi
viduals wit
h other
manifestat
ionsofsarcoi
dosi
s.Chagasdiseaseisani
mportantcauseofcar
diomyopathyin
Centr
al and Sout h Amer i
ca, usual
ly devel
opi
ng year
s or decades after i
nfect
ion
wit
h Trypanosomacruzi.
 

Giantcellmyocardit
isiscommonl yseeni nyoungeri ndi
vidual
s,i
sr api
dlyprogressi
ve,andi s
often associ
ated wit
h ventr
icul
ararrhythmias.Ther eisa hi gh f
requencyofaut oi
mmune
disordersini
ndivi
dualswithgiantcel
lmyocar di
ti
s.Thus,gi antcel
lmyocar dit
isisthecorrect
opt i
on.Endomyocardialbi
opsyisindicat
edf orpatientswithrapi
dlyprogressi
ve,unexplai
ned
cardiomyopathyasgiantcel
lmyocarditi
smayber esponsivetoimmunosuppr essi
on. 

11. Whi
choft
hef
oll
owi
ngi
sTRUEr
egar
dingt
heepi
demi
ologyofhear
tfai
lur
e(HF)
?

a. Atage40year
s,t
heli
fet
imeriskofdevel
opi
ngHFis20%.
b. Thebenef
it
sofbloodpressurecontroli
nreduci
nginci
dentHFar
eseenpr
imar
il
yin
youngerandmi ddl
e-agedadults,withnosi gnif
icanteffectamongadults>80year
sof
age.
c. Theinci
denceandpr eval
enceofHFar eincreasi
ng, butonlyintheWesternhemi
spher
e
andinhigh-
income/developedcount r
ies.
d. HFamongadul ts<50yearsofagei smor ecommoni nwhi test
hanblacks.
e. Amongadults>65year sofage,theincidenceofHFi shigherinwoment hanmen.

ANSWER

11. a. Atage40year
s,t
hel
if
eti
mer
iskofdevel
opi
ngHFi
s20%.

HFamongadul t
s<50year sofagei smor ecommoni nblackst hanwhi t
es.Thei nci
denceofHF
i
smor ecommoni nolderment hanolderwomen.I ntheHyper tensi
onint heVer yEl
derlyTr i
al,
amongadul ts>80yearsofagewi t
habasel i
nesystol
icbloodpr essure=160mm Hg, tr
eat ment
wit
hindapami deorperindopri
lwasassoci atedwit
ha64%r elativeri
skr educti
oninther ateof
HF.Althoughtheeti
ologyofHFvar i
esaroundtheworld,HFisawor ldwidepandemi cwithbot h
i
ncreasinginci
denceandpr evalence,especi
all
yintheelderl
y.Atage40year s,t
heli
fet
imer i
sk
ofdevelopi
ngHFi sapproximately20%.Ther i
skofnewHFatage80year sisalso20%. 

12. Amongi ndi


vidual
snewlydi
agnosedwi
thhear
tfai
lur
e,t
he5-
yearmor
tal
it
yrat
eis
cl
osestt
owhi
chofthefol
lowi
ng?

a. 20%.
b. 40%.
c. 50%.
d. 30%.
e. 60%.

ANSWER: 

12. c. 50%.

The 5-
year mor
tal
it
yrat
e among i
ndi
vidual
s newl
y di
agnosed wi
th hear
tfai
lur
eis
appr
oxi
matel
y50%.

13. Hear
tfai
lur
e(HF)r
epr
esent
sAmer
ica'
slar
gestdi
agnosi
s-r
elat
edgr
oup,l
eadi
ngt
othe
f
actt
hatmor
eMedi
car
edol
lar
sar
espentonHFt
hanonanyot
herdi
agnosi
s.

Which ofthe f
oll
owi
ng condi
ti
ons r
epr
esentt
he hi
ghestpopul
ati
on at
tr
ibut
abl
eri
sk f
or
devel
opingHF?

a. Micr
oalbuminuri
aandl eftvent
ri
cularhypert
rophy.
b. Di
abetesandobesi t
y.
c. Hypert
ensionandcor onaryart
erydisease.
d. Smok i
ngandchr oni
cki dneydi
sease.
e. Peri
pheralar
ter
ialdi
seaseandhypot hyr
oidi
sm.

ANSWER: 

13. c. Hyper
tensi
onandcor
onar
yar
ter
ydi
sease.

Alloftheriskf
actorslist
edhavebeenassoci
atedwit
hHFinselectpopulat
ions.Thegr
eatest
ri
skf act
orsfordevelopmentofHFacrossallpopul
ati
onsi
ncl
udehypertensionandcoronary
arter
ydisease.

14. A 56- year-


old patient wit
hl ong-standing heartf ailur
e( HF) from nonischemi
c
cardiomyopathypr esent
st ot heemergencyr oom with dyspneaand pr esyncope.Hehas
recentl
yexper i
enced shocksf rom hi
si mplantablecardi
over ter-
def
ibri
ll
ator(ICD)and now
presentswit
hdyspneaatr est,dif
fi
cul
tyconcentrati
ng,anddecr easeduri
neout put.
 

Onexami nat
ion,heisi
nmoder atedistr
esssecondarytobreat
hlessness.Hi
sbloodpressur
eis
88/62mm Hgandhi sheartr
ateis100bpm.Jugul arvenouspressureisel
evat
ed,andhehasa
prominentS3 gal
lop.Dist
alextremit
iesarecoolt ot ouch,and thereis2+ pit
ti
ng edema.
Creat
ininei
selevatedt
o2.1mg/ dlfr
om abaseli
neof1. 3mg/dl.

Whi
choft
hef
oll
owi
ngi
sthenextmostappr
opr
iat
ether
apy?

a. Dobutamine.
b. Bet
a-blockers.
c. I
ntr
avenous( IV)f
lui
ds.
d. Ni
troprussi
de.
e. Amiodarone.

ANSWER: 

14. a. Dobut
ami
ne.

Thepat i
entisin car
diogenicshockwi t
h evidenceofend- organ hypoper f
usi
on (el
evated
creati
nineandchangesinment alstat
us).Despi
tehypotension,therei
sevi denceofanincrease
inintr
acardiacfi
ll
ingpressures,andt herapyshouldfocusonbot hincreasingperf
usionand
decreasingvolumeoverload.Bet a-
blockersarecontr
aindicatedbecauseofshockdespi tehis
ventri
cularect
opyandICDshocks.  

Nit
roprusside mi
ghtbe appropr
iatein a hypotensi
ve pati
entifi nvasi
ve hemodynami cs
demonst r
ateaseverel
yelevat
edsystemicvascularresist
ance.Inthiscase,theeti
ologyof
hypotensi
oni suncl
ear,andempi r
icni
tr
oprussideint hissett
ingcouldwor senthe 
cli
nical
situati
on. I
Vfluidswouldnotbehel pfulsincet heover allpictureisoneofvolumeover loadand
reduced car diac output.His nonsust ained vent riculart achycar
dia and subsequentI CD
dischargesar emor elikel
yar eflecti
onof  
hiswor seningHF.I Vamiodaronelikel
ywi l
lwor sen
hisHFduet obet a-
blockeref
fect .
 Thebestopt i
oni sdobut ami nebecauseitmostr apidlywill
i
mpr ovehiscar di
acoutputandaddr esshi scar di
ogeni cshock.
15. A42- year-ol
dmal epatientwi thahi storyofhyper t
ensi onandrecentdiagnosisofhear t
fail
ure( HF)pr esent
st oyourof ficef orfur t
hereval uat ion.Hehasal eftventri
cul
arej ect
ion
fracti
on( EF)of35%.Hi smedicalr egimeni ncludescar vedilol25mgbi d,li
sinopri
l10mgdai l
y,
furosemi de20mgdai l
ywhennecessar y,andamul t
ivitami n. 

Pri
orwor kup f
orhis cardiomyopathy di
d notshow a r ever sible cause,and coronar
y
angi
ogr aphydemonst
ratednocoronar yl
esi
ons.Hehasnopr oteinur ia,hemogl
obinA1cis5.2,
andl ow-densi
tyli
poprotei
nwasf oundtobe80mg/ dl
.Hecur rent l
yhasNew Yor kHear t
Associati
on(NYHA)classIsympt oms,anormalphysi
calexami nat ion,andbloodpressur
eof
128/92mm Hg.

Whichofthefol
lowingisthenextbestst
epi
nmanagementpr
iort
oeval
uat
ionf
ori
mpl
ant
abl
e
car
diover
ter
-def
ibri
ll
ator(
ICD)?

a. Eplerenone25mgdai l
y.
b. Digoxin0. 125mgdai ly.
c. As pi
rin81mgdai ly.
d. Increaselisinopr
ildose.
e. Ros uvastati
n10mgdai ly.

ANSWER: 

15. d. I
ncr
easel
isi
nopr
ildose.

Inthispati
entwithrecentlydiagnosedcar diomyopathy,opti
mizati
onofmedi calt
herapyshould
beper f
ormedpr i
ortoconsi derati
onforICDpl acementforprimarypreventi
onofsuddencar diac
death.Eplerenonehasbestbeenst udiedi nthepost- myocardi
alinfarcti
onpopul at
ionand
chronicHFpat i
entswi thNYHA cl assI I-I
Vf uncti
onalclass(thi
spat ienti sclassI)
.Int he
absenceofanot herindicat
ion,therei
snoevi denceofbenefitf
oruseofst ati
nsoraspi r
ini
nt he
HFwi thpreservedEFpopul ati
on.Ther oleofdi goxi
nint heHFwi thpr eservedEFpopulati
on
remainscont r
oversi
al,butmayr educehospi t
ali
zat
ions.

16. A62- year-


oldmanpresentstothehospi
talwith2weeksofprogr
essi
vemid-ster
nal
chestpr
essurewhenwalki
ngatanor malpaceonlevelgr
ound.Pr
iort
o1mont hago,hewas
worki
ng12hour s/
dayasaroofer
.Hisvi
talsi
gnsarebloodpr
essur
e123/84mm Hgandhear t
rat
e88bpm.  

Onexam,hehasnosi gnsoff l
uidoverl
oad.Laboratori
esarenor malwi t
ht heexceptionofan
el
evatedB- typenatri
ureticpepti
de.Hisechocar di
ogram showsgl oballeftventri
cular(LV)
hypokinesi
swi t
hanLVej ect
ionfr
acti
on(LVEF)of20%, LVdiastol
icdiamet erof6cm, andmi l
d
concentri
cLVhyper tr
ophy.Coronaryangiographydemonstratesmul ti
vesselcoronarydisease
wit
hf ocalpr
oximalstenosesof70%i nthecir
cumflex,70%i
nt heleftanteri
ordescendingar t
ery,
and80%i ntherightcoronaryart
ery.
 

Hei
sst
art
edonaspi
ri
n81mg,
lisi
nopr
il10mg/
day,
ator
vast
ati
n80mgPOqhs,
andcar
vedi
lol
12.
5mgbi
d.Af
tersever
alweeksont
her
apy,
hisexer
ti
onalangi
naper
sist
s.

Whi
choft
hef
oll
owi
ngi
sthenextst
epi
nmanagementoft
hispat
ient
?

a. LVas si
stdevi ce(LVAD)implant
.
b. Coronaryarterybypasssurgery(CABG).
c. Nuclearvi
abi l
itystudy.
d. Hearttr
anspl antli
sti
ng.
e. Magne t
icresonancei maging(MRI)vi
abil
it
yst
udy.

ANSWER: 

16. b. Cor
onar
yar
ter
ybypasssur
ger
y(CABG)
.

Theuseofcor onar yrevasculari


zation(surgicalorper cutaneous)i
naddi ti
ont oguideli
ne-
di
rectedmedi caltherapyi npat ient
swi thhear tf
ail
ure( HF)andr educedLVEFshoul dbe
i
ndividual
ized.Inthiscase, i
tisaCl assIrecommendat i
ont oproceedwithrevascul
ari
zati
onf or
i
mpr ovementofangi nalsympt omsi n a pat i
entwith HF and suit
ablecor onar
yanat omy.
Furt
hermor e,surgicalrevascul
arizati
onisr easonabletoimpr ovemorbidi
tyandcardiovascular
mortali
tyforpat i
entswi t
hsever eLVdysf unction(EF<35%) ,HF,andsigni
fi
cantcoronaryartery
di
sease( ClassIIa).
 

Myocardialvi
abil
it
ytesti
nghasar oleincer t
ainpat i
ents.However ,inthiscase,i ti
sunlikelyto
changemanagement .Further,inananal ysisoft heSTI CH (
Sur gicalTr eatmentforIschemi c
HeartFail
ure)t
ri
alofsurgicalr evasculari
zati
oni npatientswit
hsever eLVsyst ol
icdysf
unct i
on,
theassessmentofmyocar dialviabil
itydidnoti dentifypat
ientswi thadi f
fer
enti
alsur vival
benefi
tfrom CABG,as compar ed wi t
h medi calther apyalone.Ref erralforadvanced HF
ther
apiesincl
udingLVADandhear ttransplantisreasonableinpat ientswi thsevereLVsyst olic
dysf
unctionwhohaveexhaust edconvent ionaltherapiesincl
udinggui deline-
dir
ectedmedi cal
ther
apyandconvent i
onalcar diacsurgery.Inthiscase, suchar ef
erralwoul dbepremature.

17. A 58- year-ol


d man wi th a strong family histor
y ofhear tfail
ure( HF)has an
echocar diogram per formedbyhi spr i
mar ycarephysician.Theechocardiogram showsal ef
t
ventricul
arej ect i
onfract i
on(EF)of35%.Hei sanact ivenonsmokerwhoexer ci
sesr egul
arl
y
withoutdi ff
iculty.Exami nati
onisunremar kabl
eexceptf orabloodpressureof144/ 92mm Hg
andhear trateof74bpm.Labor atorytestsareunremar kabl
e.Heprefer
snott otakeanymor e
medi cati
onsandcont rolhypert
ensionwi t
hdietandexercise.

Whi
choft
hef
oll
owi
ngi
syourr
ecommendat
ionf
ort
hispat
ient
?

a. Nome dicalt
herapyunl
essHFsymptoms.
b. Angi
otensin-
convert
ingenzymei
nhi
bit
orsandbeta-blocker
s.
c. Angi
otensin-
convert
ingenzymei
nhi
bit
orsanddigoxin.
d. Nome dicalt
herapyunl
essEFremai
nsdepressedat90days.
e. Cal
cium channelbl
ocker
.

ANSWER: 

17. b. Angi
otensi
n-conver
ti
ngenzymei
nhi
bit
orsandbet
a-bl
ocker
s.
Asympt omaticdecr easesinEF( StageBHF)shoul dbet reatedwi thangiotensin-convert
ing
enzymei nhi
bitortherapyandabet a-bl
ockertherapyasanaddi tionaldrug.Therapyshoul dnot
bedel ayedwhi l
eawai ti
ngi mprovement .Whil
eappr opri
ateforhyper t
ension,calcium channel
blockerther
apyi snotindicatedasaf ir
st-l
inetherapyf
orpatientswithreducedEF.  

18. A68- year


-oldmal epati
entwithahistor
yofhypert
ension, di
abet
es,andmorbi
dobesity
present
stoyourcli
nicwithcomplaint
sofdyspneaonexerti
onf orthepast2months.Hedenies
chestdiscomfor
t,orthopnea,paroxysmalnocturnaldyspnea,l owerext
remit
yswel l
ing,or
abdominalbl
oati
ng.Her eport
sheisr el
ati
vel
ysedentar
yandhasbeenmor esosincetheonset
ofsymptoms.Hehasa20pack- yearhist
oryofsmokingandqui t20year
sago. 

Onexami nati
on, hisbloodpr essur eis150/ 90mm Hgandhi shear trateis78bpm.Hi soxygen
saturationi s98%onr oom airatr estandwi thambul ati
on.Thej ugularvenouspr essureis6cm
H2O.Thecar diacexami nati
onr evealsar egularr ateandr hythm wi t
hagr ade2/6hol osystoli
c
mur muratt he rightl owerst ernalborder .The abdomen i s notdi st
ended wi t
houtany
appreci ableasci t
es.Thel ungsar ecleartoauscul t
ation.Theext r
emi t
iesar ewarm andwi thout
edema.Theel ectrocar di
ogram demonst ratesl eftvent ri
cularhyper t
rophy( LVH)byvol tage
cri
teria.Theechocar diogram repor tsnormalbi ventricularfuncti
on,mi ldLVH,mi ldleftatri
um
enl
ar gement ,and mi ld mitralr egurgi
tati
on.A phar macol ogi
c nucl earper f
usion study is
negat i
vef ori nducibleischemi a.Amonghi slabor atoryevaluati
on,hi sN- ter
minalpro–B- type
natri
ur eti
cpept ide( NT-proBNP)i s100pg/ ml.

Whichofthef
oll
owi
ngoft
hef
oll
owi
ngi
smostl
ikel
ytobehel
pfuli
nest
abl
ishi
ngt
hecor
rect
di
agnosi
s?

a. Cardiacmagnet i
cr esonanceimagi
ng.
b. Transesophagealechocar di
ogram.
c. Ri
ght -heartcatheterizat
ionwit
hexerci
se.
d. Coronaryangi ography.
e. Treadmi l
lstresstest.

18. c. Ri
ght
-hear
tcat
het
eri
zat
ionwi
thexer
cise.

Hear tfail
urewi t
hpr eser
vedeject
ionf
ract
ion( HFpEF)ist hecli
nicaldi
agnosis.Pat i
ent
soft
en
presentwi thsignsandsympt omsofcongesti
on.Typicalfeatur
esonechocar diographyi
ncl
ude
LVH,l eftatri
alenlargement
,andabnormalE/ e'values.Thenat r
iuret
icpept
idel evelsmaybe
onlymi l
dlyelevated.Hehasnor malNT- pr
oBNPl evel
s,butt hi
scanbeconf oundedbyhis
mor bidobesity. 

Inthiscase, thedi agnosi


sismoredi f
fi
cultasthi
spatientiseuvol emiconexami nat i
onandhas
sympt oms onl y with exer
ti
on.Ther ei s a wi
derdi f
ferentialdiagnosis,i
ncludi ng chr
onic
obstructi
vepul monar ydisease(given hissmoking history)and decondi t
ioning (gi
ven his
sedentaryl i
festyl
e and mor bi
d obesity).Inthese cases,r i
ght-heartcat
heter i
zati
on may
demonst ratenor malhemodynami csatr est
,butanexagger atedincreaseinfil
lingpressures
wit
hexer ci set hatmaybeaccompani edbyabl unt
edi ncreasei ncar di
acoutput ,whichhelp
establi
sht hedi agnosisofHFpEF. 

Whileacardi
acmagneticr
esonancemaybeusefultoeval
uat
ecardi
acfuncti
onandassessfor
scarandotherpot
ent
ialet
iol
ogi
esofHFpEF,i
tsuti
li
tyi
nmakingadiagnosisofHFpEFi
slow.A
tr
ansesophagealechocar
diogr
am is unli
kel
yt o yiel
d addi
ti
onalinformati
on.Coronary
angi
ogr
am maybehel pfuli
fhisshort
nessofbreathi
sthoughttobeanangi
nalequi
val
ent
,but
hi
snegati
veperfusi
oni magingmakest hi
sl essli
kel
y.Ast andar
dtr
eadmil
ltesti
sli
kel
yt o
i
nducedyspnea,
butthiswil
lnotident
if
yeti
ology.
 

19. A54- year


-oldmanwi thahi stor
yofhear tfai
lur
ewi t
hpr eser
vedej
ect
ionf r
act
ionis
ref
err
edtoyouf orrefr
act
oryvolumeover load.Onexami nat
ion,hehasanelevatedjugul
ar
venouspr
essure,
clearl
ungs,
regularheartr
atewithS4,
andpi t
ti
ngedemat ot
het
highs. 

Your evi
ew hi sdi agnosti
cst udiesincludi
nganel ectr
ocar diogr am (ECG)t hatdemonstr ates
sinusr hyt
hm, l
owvol t
ageinthel i
mbl eads,andnonspeci f
icl ateralST-Twavechanges, andan
echocar di
ogr am t hatdemonst ratesconcent r
icleftvent
ricul arhyper t
rophy,al ef
tventri
cul ar
eject
ion fraction of50%,sever ebi atr
ialenlargement,and an est i
mat ed pulmonar
yar tery
systolicpressureof50mm Hg.Apr iorri
ght
- heartcat
heterizat i
ondemonst r
atesequali
zati
onof
diastoli
cpr essures, aprominentYdescenti nther i
ghtatri
altr acing,anda" square-
root"signi n
therightvent ri
culartraci
ng.

I
nadditi
ontoi
ntensi
fyi
ngt
hepat
ient
'sdi
uret
icr
egi
men,whi
choft
hef
oll
owi
ngshoul
dbe
per
for
med?

a. Comput edtomogr aphycoronaryangiogr


am.
b. Coronaryangi ogr
aphy.
c. Endomy ocardialbiopsy.
d. Chestcomput edtomographywithoutcont
rast
.
e. Exerci
senucl earperfusi
onimaging.

ANSWER: 

19. c. Endomyocar
dialbi
opsy.

This pati
ent '
s exam and di agnost i
c studies are highly suggestive of an infil
tr
ati
ve
cardi
omyopat hy resulti
ng i nar estri
cti
ve cardi
omyopat hy.Despi tet hi
ckened walls on
echocardi
ogr aphy,thel owvol t
agei nt helimbleadsont heECGi ssuggest i
veofaninfil
tr
ati
ve
process.Bi atri
alenl argementwi thoutval vulardi
sease i s also suggestive ofrest
ri
cti
ve
physiol
ogy.Hi sr i
ght-heartcatheterizati
onisconsist
entwi theitherarestr
icti
veorconstri
cti
ve
processgivenequal izationofpr essures,rapidYdescent,andt hesquar erootsignsuggesti
ng
earl
yequalizat i
onofpr essuresbet weent heatri
aandventricl
es. 

Givent hisconstell
ati
onoff i
ndings,amyloidcar diomyopat hywoul dbehi ghl
ysuspect ed.The
goldst andardfordi agnosisisendomyocar dialbi opsy.Gi venthedi f
fusenatureofamyl oid
deposition,di
agnosticyiel
dfrom biopsyishigh.I naddition,ti
ssueallowsforthedeterminati
on
ofamyl oidsubt ypeandpot ential
lyaff
ectsmanagement .Whi l
ecar di
acmagnet i
cr esonance
maybehi ghl
ysuggest i
veofamyl oidversusot herpot enti
aletiol
ogies,inthi
srel
ativel
yyoung
pati
ent, wherethereisahi ghsuspici
onforcardi acamyl oid,atissuediagnosisi
sbestt ogui de
therapy.

Eval
uationf ori
schemi cdisease,suchasexer ci
senucl earperfusionimagingandcor onary
angi
ogr aphy,i
sl essl
ikel
ytobehel pfulint
hesett
ingofot herf
indingssuggestiveofinfil
trat
ive
heartdisease.Anoncont rastchestcomput edtomographywoul dbeusef ult oevaluatet he
peri
cardium inthesetti
ngofconstricti
veperi
car
dit
is,butinthi
scase,t heevidencepoi nt
st oa
myocar
dialpr
ocess.
 

20. A75- year-oldmanwi t


hahi storyofhypert
ensionanddi abet
esisseenbyyoui nfol
low-
up.Athisl astvisit
,hecompl ai
nedofpr ogr
essi
veshor t
nessofbr eath,ort
hopnea,andlower
extr
emityswel li
ng.Hewashyper tensive,andtherewasevi denceofvolumeover l
oad.Hewas
star
tedonl i
sinopri
l10mgandf urosemi de20mgdai ly.Anechocar di
ogram demonstrat
ed
normalbi ventri
cular f
uncti
on,2+ t r
icuspidregurgit
ation,and moder ateleftventri
cular
hypert
rophy.At readmil
lnucl
earperfusionstudydemonst ratednofi
xedorrever
sibl
edefects.
 

Onexami nati
ontoday,hi
sbloodpressur
eis150/95mm Hg,hi shear
trat
eis72bpm,andhis
j
ugularvenouspressureis8cm H2O.Hehasar egul
arheartr
atewit
hgrade2/6hol
osyst
oli
c
murmuratt heri
ghtlowerst
ernalborder
,hi
slungsareclear
,andhehastracel
owerext
remit
y
edema.

Whi
choft
hef
oll
owi
ngi
sthemostappr
opr
iat
einhi
scar
e?

a. I
ncreasel isinopril
.
b. Adddi goxin.
c. Adds pironol act
one.
d. Addme toprol ol
.
e. Addval sartan.

ANSWER: 

20. a. I
ncr
easel
isi
nopr
il
.

Nopr ovent herapi esexisttor educemor bidi


tyormor tal
it
yint hegr oupofpat i
entswi t
hheart
fai
lur
ewi thpr eservedej ecti
onf ract
ion(HFpEF) .Guideli
nesrecommendcont rol
li
ngcongest ion
wit
h di ureti
cs,r uling outi schemia,and cont rol
li
ng blood pr essure and rate cont r
ol(or
considerati
on ofcar diover
sion)when at ri
alf ibri
ll
ati
on is present.A r ecentst udy ofa
pharmacol ogicagentf orHFpEFwasTOPCAT( TreatmentofPr eservedCardiacFunct ionHeart
Fai
lureWi thanAl dosteroneAnt agonist
),whichl ookedatspironolactoneversusplacebowi t
ha
pri
mar ycomposi teout comeofcar diovasculardeath,abortedsuddencar diacdeat h,orheart
fai
lur
ehospi tali
zat ion.Thisstudywasneut r
al. 

Ofnote,givensigni
ficantdif
fer
encesi neventrat
esi nNor
thAmer i
cacomparedwit
htherestof
thewor l
d,asubgr oupanal ysisofpat i
entsinNor thAmeri
cademonst r
atedabenef
itforthe
aldost
erone antagonistoverpl acebo.However ,since t
his anal
ysi
s was posthoc,t he
hypothesi
st hataldosteroneantagonistsar eeff
icaci
ousinthetreat
mentofHFpEFr emains
unproven.Therefor
e, t
reatmentofhishypertensi
oni st
hepri
marygoal.
 

Thelow doseofli
sinopr
ilshoul
dbet it
ratedbef or
et headditi
onofasecondagent .Bet
a-
bl
ocker
sar ef
ourt
h-li
netherapyforthet r
eat mentofhypertensi
on,andangi
otensi
n-r
ecept
or
bl
ocker
sshouldnotbeaddedtoangiotensin-convert
ingenzymeinhi
bit
ors.
 

21. Youar eadmitti


nga65-year-oldwomant othehospitalbecauseofdyspnea,orthopnea,
andedema.Shei swell-
knowntoyourpr act
icebecauseofl ong-standi
nghear tf
ailur
ewi t
h
eject
ionfract
ion(EF)
 of45%andhyper tensi
on.Hermedi cat
ionsincludeli
sinopr
il2.
5mgdai l
y,
carvedi
lol12.
5mgbi d,andfur
osemi de40mgdai l
y.Vit
alsignsaret emperatur
e36.9°C,heart
rate110bpm, bloodpressur
e160/100mm Hg, and 
oxygensaturati
oni s88%onr oom air

Herphysicalexam i
sr emarkablef
orelevat
edj ugul
arvenouspr essureandanS3 gal
lop.She
has3+edemat oherkneesandoverthesacr um.Extremi
tiesarewar m.Labor
ator
yevaluati
on
i
ssignif
icantforsodium 134mEq/ L,creat
inine2.0mg/ dl(baseli
ne1.2mg/ dl
),andB- t
ype
nat
ri
uret
icpeptide3400pg/ml .

Whi
choft
hef
oll
owi
ngf
indi
ngsi
nthi
spat
ienti
sthemostl
ikel
yet
iol
ogyofherr
enalf
ail
ure?

a. Angiot
ensin-
converti
ngenzyme(
ACE)i
nhi
bit
oruse.
b. Hypoxemia.
c. Lef
tventri
cul
arejecti
onf
racti
on(
LVEF)
.
d. Lowc ar
diacout
put .
e. Venouscongesti
on.

ANSWER: 

21. e. Venouscongest
ion.

Cardi
orenalsyndrome,especial
lytype1( acut
e)car
diorenalsyndr
ome,compli
cates25-30%of
hospit
alizat
ionsforacutedecompensat edhear tf
ail
ure.Changesassmal las0. 3mg/ dlin
ser
um cr eati
nine have been associ
ated wi t
h prol
onged hospit
ali
zat
ion and increased
mortal
ity. 

Multi
plef act
ors have been post ul
ated to contr
ibut
et othis syndrome,incl
udi ng venous
congest
ion,lowr enalper
fusion,anddysfunctionalaut
oregul
ati
onoft heki
dney.Int hi
spat i
ent
,
ther
eisclearevidenceofvenouscongest i
onandvol umeoverl
oad.Herphysicalexam doesnot
suggestlow cardiacoutput.Further
,int heESCAPE( EndovascularTreat
mentf orSmal lCore
andProximalOccl usi
onIschemi cStroke)tr
ial
, t
herewasnocor r
elati
onbetweenr enalfunct
ion
andcardiacindex. 

LVEFi snotconsider
edar i
skfact
orf orcardi
orenalsyndrome,andcardi
orenalsyndromei s
seeni ncasesofhear
tfail
urewithpreservedorreducedEF.Whi l
eACEinhi
bitor
scandecr ease
glomer ul
arfi
lt
rat
ion r
ate,they do notdecr ease renalblood fl
ow and arether
efore not
responsibl
eforrenalfai
lur
ei nthi
spat ient
.Hypoxemi aini sol
ati
onisnotacauseofr enal
fail
ure.
 

22. Youar epl anningtodischar gea55-year-oldmanf rom thehospi


talaft
ertreat
mentfor
acut edecompensat edheartfail
ure( HF)
.Hewasadmi t
ted5daysagowi t
hdyspnea, decr
eased
exercisetol
erance,andedema.Thi swashissecondadmi ssionforHFwit
hinayear .Hismedial
historyincl
udesanant eri
orwal lmyocardialinfar
cti
on3year sagowi t
hal ef
tventri
cul
ar
ejectionf
ract
ionof30%, hypert
ension,andat
r i
alfi
bri
ll
ati
on,whi chisnowratecontr
oll
ed. 

Hehasasi ngl
e- l
ead,pri
marypr eventi
on implantablecardioverter
-defi
bri
ll
ator
.Dur
ing the
hospit
ali
zati
on,hisweighthasdecreased7pounds, andhi ssymptomsofdyspneaandf at
igue
arenow consistentwithNew YorkHear tAssociat
ionf uncti
onalclassII.Curr
entmedicat
ions
i
ncludecarvedil
ol,l
isi
nopr
il
,spi
ronolactone,
furosemi de,at
orvastat
in,aspi
ri
n,andwarf
arin.

I
naddit
iontogui
deli
ne-di
rect
edmedi calt
herapy,
whi
choft
hef
oll
owi
ngi
nter
vent
ionshasbeen
pr
oventohelpr
educerehospi
tal
izat
ionsforHF?
a. Exerci
setrai
ningpr ogram.
b. Enroll
mentinadi seasesuppor tgr
oup.
c. Cogniti
vebehavioraltherapy.
d. Standardi
zeddiseaseeducat i
on.
e. Telemonit
ori
ngofsympt oms.

ANSWER: 

22. d. St
andar
dizeddi
seaseeducat
ion.

HFpat i
entsr emai ni navul ner ableperi
odf ol lowingdi schargef r
om t hehospi tal
.Impr oved
communi cati
onbet weencl iniciansandnur ses,medi cationr econci l
iation,careful
lypl anned
transit
ionsbet ween car eset t
ings,and consi stentdocument ation ar eexampl esofpat i
ent
safetystandar dst hatshouldbeensur edforpat ientswithHFdi schar gedf r
om thehospi t
al(see
theHFgui deli
ne) .Theaddi ti
onofa1- hournur seeducat or-deli
ver edt eachi ngsessionatt he
ti
meofhospi taldi scharger esultedinimpr ovedcl i
nicalout comes,i ncr easedself-careand
treatmentadher ence,andreducedcostofcar e.Educat i
onwhi l
ei nthehospi t
alal
ongwi thear l
y
foll
ow- upandenr oll
menti nanout pati
entdi seasemanagementpr ogram shoul dbeusedt o
address pat i
ent -specifi
c bar ri
erst o guideline- di
rected medi calt her apy and bar ri
erst o
behavioralchange, asdescribedi ntheHFgui del ine.

Randomi zed,control
ledtri
alsoft el
emonitori
ngandexer ci
set rai
ninghavenotpr ovent hat
theseint
erventi
onsr educereadmissi
ons,althoughexercisetr
ainingmayhaveot herbenef i
cial
eff
ectsforHFpat ients,i
ncl
udingimprovedheal t
h- r
elat
edqualit
yofl if
e.Enr
ollmenti ndisease
supportgroupsmayhavepsychosoci albenefit
sf orpati
entswithmanychr oni
cdi seases,but
hasnotbeenpr ovent or
educereadmissionsinprospectivecl
inicalt
rial
s.Cognit
ivebehavi oral
therapyi
snotat reatmentforHF. 

23. You ar easked to seea 59- year-


old man recent
lydi
agnosed wit
h non-Hodgki
n’s
l
ymphomai n consult
ati
on regar
ding thecardi
acrisksofanthracycl
inechemother
apy.He
speci
fi
cal
lyasksaboutcardiactoxici
tyoftheproposedchemotherapyregi
menbecausehi s
fat
herdi
edsuddenlyofaheartatt
ackathisage.

Whichofthef
oll
owi
ng,i
fpr
esent
,mostl
ikel
ywoul
dincr
easet
her
iskofant
hracycl
ine-
induced
car
diomyopat
hy?

a. Personalhi
stor
yofper i
cardit
is.
b. Personalhi
stor
yofradiati
ont reat
ment.
c. Presenceofsel
enium defi
ciency.
d. Presenceofhematologi
cmal ignancy.
e. Familyhist
oryofheartf
ail
ure.

ANSWER: 

23. b. Per
sonalhi
stor
yofr
adi
ati
ont
reat
ment
.

Riskf actor
sforanthracycl
ine t
oxi
cit
yinclude:1)t ot
alli
fet
ime dose ofanthracycl
ine;2)
i
ntravenous bol
us administ
rati
on;3)highersingl
e doses;4)historyofpr i
ormedi ast
inal
i
rradiati
on;5)useofot herconcomit
antagentsknownt ohavecardiot
oxicef
fects,suchas
cyclophosphami
de,trast
uzumab,andpaclit
axel;6)femalesex;7)underlyi
ngcardiovascular
disease;8)extr
emesofpat i
entage(
bot
hver
yyoungandol
dage)
;and9)i
ncr
easedl
engt
hof
ti
mesi nceanthracycl
inecompl
eti
on.
 

Per icardit
isisgeneral
lysel f
-li
mitedandnotspeci fical
lydemonstrat
edt oincreaser i
sksof
car diotoxi
cit
y of chemot herapy. Tot aldose of chemotherapeuti
c agent and t ype of
chemot herapeuti
cagentarebot himpor tantf
act
orsinthedevel
opmentofcar di
omyopat hy,but
typeofcancerbei ngtreat
edbeyondt heagentusedi snot.Caseser i
eshavesuggest eda
relationshipbetweenselenium defici
encyanddevelopmentofcardiomyopathy,butthesehave
notbeenval idat
edinprospectivestudies. 

24. A72- year


-oldwomani sreferr
edt oyourclinicforconsultat
ionregar
dingthecar di
ac
ri
sksofchemot herapyf
orherbreastcancer.Shehasahi storyofhyper
tensi
oncontroll
edwith
li
sinopr
ilandamlodipi
neandcoronaryarterydi
seasef orwhi chsheunderwentstent
ingseveral
yearsago.Shehasneverhadacl i
nicalmyocardi
alinfarct
ion. 

Ar ecentechocar di
ogram document edlef
tventricularejecti
onf ract
ionof55%.Sher ecentl
y
wasdi agnosedwi thbr eastcancer,andpathologicr esult
si ndicat
edt hetumorwast ripl
e
negative fort he est
rogen r ecept
or,the pr
ogest erone receptor,and t HER2/
he  neu 
gene.
Therefore,heroncologistrecommendsacombi nationofchemot herapeuti
cagent
s,incl
uding
anant hracycl
ine.

I
nadditi
ontol
imi
tingthetot
aldoseofant
hracycl
ineagents,
whichofthefol
lowi
ngst
rat
egi
esi
s
mostl
ikel
ytomit
igateherri
skofdevel
opi
nganthracycl
inecardi
otoxi
cit
y?

a. Useofradiat
ioninaddi t
iontochemot her
apy.
b. Addi
ti
onofcycl ophosphami detothechemot herapyregi
men.
c. Addi
ti
onofpr ednisonetothechemot herapyregimen.
d. Admini
strati
onofbol usrathert
hancont i
nuousant hr
acycli
nei
nfusi
ons.
e. Admini
strati
onofal i
posomalant hracycl
inepreparat
ion.

ANSWER: 

24. e. Admi
nist
rat
ionofal
iposomalant
hracycl
inepr
epar
ati
on.

Thelar
gestsingleriskfact
orf
ordevel
opingcardi
otoxi
cit
yiscumul ati
vedoseofant
hracycli
ne.
2
Thecumulat
ivepr obabil
it
yofdoxor
ubi
cin-i
nducedheartfai
lur
e i
s 3-5%wit
h400mg/ m, 7-
26%
at550mg/m2, and18- 48%at700mg/m2. 

Otherriskfactor
sf oranthr
acycli
net oxicityinclude:1)i ntravenousbol usadmi nistr
ati
on,2)
highersingl
edoses,3)hi stor
yofpr iormedi astinalir
radiation,4)useofot herconcomi t
ant
agentsknown t o havecar di
otoxi
cef fect s,such ascycl ophosphami de,trast
uzumab,and
pacli
taxel
,5)femalesex,6)underl
yingcar diovasculardisease, 7)extremesofpatientage(both
veryyoungandol dage),and8)incr
easedl engthoft i
mesi nceant hracycl
inecompl eti
on.

Preventionst
rat
egiesincl
udeco-admi ni
strat
ionofcardi
oprotect
antssuchasdexr azoxane,use
ofli
posomalanthracycli
nepreparat
ions,anduseofpr ol
ongedcont i
nuousinfusionsratherthan
boluses.Angiotensi
n-convert
ing enzyme inhibit
ors and beta-bl
ockers also may have a
protect
iveeff
ectbutarenoteffecti
veinallpatient
s.Predni
soneandot hersteroi
ds,whi chare
partofmanychemot herapyregimens,donothaveani nt
eracti
onwi t
hant hracycli
nesi nthe
myocar dium.
 
25. A 55- year-ol
d man withi schemi
c car
diomyopat
hy pr
esent
s wi
th compl
aint
s of
pal
pit
ati
ons,f
atigue,andheadaches.Heusuall
ycanexerciseupto30mi nut
esatat ime
wal
kingonatreadmillbr
iskl
ywi
thoutsympt
oms. 

Curr
entdai
lymedi
cat
ionsi
ncl
udecarvedi
lol12.
5mgor
all
ybi
d,l
isi
nopr
il10mg,aspi
ri
n81mg,
at
orvast
ati
n80mg,andhydr
ochl
orot
hiazi
de25mg. 

Exami nati
onr evealshisbloodpressureis131/ 65mm Hg, pulseis79bpm r egular,
andoxygen
saturati
onis96%onr oom air
.Hisbodymassi ndexis34.Car diacauscult
ati
onr evealsasof tS1,
physiologi
callyspl i
tS2,andasof tapi calS3.Nomur murswer eheard.Lungswer eclear.
Extremiti
esr evealmi ni
maledema.An el ectrocardiogr
am r evealssinusr hythm wi t
hr ight
bundlebr anchbl ockandQRS dur ationof121msec.Hi smostr ecentechor evealedl ef
t
ventri
cular ejection fract
ion of34%.I nterrogati
on ofhi s singl
e chamber i mplantable
cardiovert
er-defibri
ll
ator(I
CD)revealssever alelevatedventri
cularhigh-rat
eepi sodesthatar e
ir
regular.Not herapieshavebeendelivered.

Whi
choft
hef
oll
owi
ngi
sthenextbestst
epi
nthi
spat
ient
'seval
uat
ion?

a. Invasi
veelectrophysi
ologicstudy.
b. Upgradet odual-chamberI CD.
c. Echowi thdyssynchronyanal ysi
s.
d. Noc t
urnalpolysomnogr aphy.
e. My ocardi
alperfusi
onimagi ngstudy.

ANSWER: 

25. d. Noct
urnalpol
ysomnogr
aphy.

Ther ei
sahi ghpreval
enceofsleepdi
sorderedbreathi
ng(eit
herobstructi
vesleepapnea[ OSA]
orcent r
alsleepapnea)inpatientswit
hcongest iveheartfai
lure(approximatel
y60%) .Thi s
patient
'ssymptomsar ehighl
ysuggesti
veofsl eepapnea( headaches/f
atigue).I
nadditi
ont o
theassociat
ionbetweencongesti
veheartf
ailur
eandOSA, OSAi salsothoughttobeapot ential
tri
ggerofatri
alfi
bri
ll
ati
on(AF)insomeaf f
ectedindivi
dual
s.Thispatient
'sdeviceint
err
ogation
i
ssuggest i
veofAF. 

Thetreatmentofsl
eepapneai sdependentupont henat ureoft heapnea.OSAsar ebestt reated
withCPAP.Cent r
alapneasar ebestt r
eatedwi thopt i
mi zingvolumest atusandhear tfai
lure
ther
apy.Whileaddinganat ri
alleadmayhel pquant ifyhisbur denofAF,thisdoesnotappeart o
beindicat
edatthepr esentti
me.Echowi thevaluationofdyssynchr onyisofverylimitedut il
it
y
andnotwi delyacceptedf ordecisionmaki ngf orupgr adi ngtoacar diacresynchronization
ther
apydevice.Therei snoi ndicati
onf orelectrophysiol ogictest
ingint hi
spat i
ent,andhi s
symptomsar enonangi nal
.Ther ef
ore,thecor r
ectopt ioni ssleepstudyt oevaluat
ef orsl eep
apnea. 

26. A69- year-


oldmanwi t
hahi st
oryofpr i
ormyocardi
alinfarcti
onpresentsafterr ecent
hospi
tal
izat
ion forcongestiveheartf
ail
ure( CHF)
.Echocardiographyduring hospi
t ali
zation
demonstrat
edwor senedlef
tventr
icul
arfuncti
onwi t
hanej ecti
onf ract
ionof30%.Cor onary
angi
ography demonst r
ated no fl
ow-li
miti
ng or obstr
uctive lesi
ons. At dischar ge,hi s
medi
cati
onsincl
udedat
orvast
ati
n80mg,
aspi
ri
n81mg,
li
sinopr
il40mg,
andcar
vedi
lol25mg
t
wicedai
ly.
 

Laborator
yvaluesincl
udest
abl
ecreati
nineof1.
4mg/dlandpotassi
um of4.
4mEq/L.Because
ofnocturnalapneathatwasnotedduri
nghishospi
tal
izat
ion,heunder
wentout
pati
entsleep
studythatwasconsist
entwi
thcent
ralsl
eepapnea(
CSA).

Whichofthefoll
owi
ngt her
api
eshasbeendemonst
rat
edt
obeoft
hegr
eat
estbenef
iti
nthe
t
reatmentoft
hispat
ient
'sCSA?

a. Ref
erralforuvulect
omy.
b. Addi
ti
onofspi ronolact
one.
c. Cont
inuousposi ti
veairwaypr
essur
e.
d. Addi
ti
onoft heophylli
ne.
e. Noct
ur naloxygentherapy.
ANSWER: 

26. b. Addi
ti
onofspi
ronol
act
one.

Therear everylimiteddat asuppor ti


ngtheuseofsuppl ementalt
herapi
esf orthetreatmentof
CSA/ obstr
ucti
vesl eepapneai npat i
entswithCHF.Whi l
eli
mitedtr
ial
shavesuggest edabenef i
t
fortheophylli
neandoxygent herapy,thesedat aareveryl i
mit
ed.TheCANPAP ( Canadian
ContinuousPosi t
iveAi rwayPressur eforPati
entsWithCentr
alSleepApneaandHear tFai
lur
e)
studyshowedsomebenef iti
nt er
msofi mprovementoflef
tventr
icul
arejecti
onfractioninthe
treat
edar m ofthest udy.However, at2yearsoffoll
ow-up,mortal
it
yandtransplantratesinthe
twogr oupswer esimi l
ar. 

Cardiacr esynchronizati
ont her apyandat ri
alpacingar enotconclusivelyi ndicat
edi nt his
population. Hence, t he gui deline continues to r ecommend f i
rst-l
ine t herapy to be
compr ehensive and pr ogressive managementofCHF medi caltherapy int hese patients.
Additi
onofal dosteroner eceptorantagoni stinthissympt omati
cpatienti sindicatedashi s
ser
um cr eat
ini
nei s<2. 5mg/ dlandpot assium islessthan5.0mEq/ L.Car efulmoni t
ori
ngof
potassium andr enalf uncti
oni si mport
antwi t
hiniti
ati
onofaldoster
oner eceptorantagonist.
Uvulectomyi snoti ndi
catedinpat i
ent
swi thCSA.  

27. A44- year-


ol dmanhasa5- yearhistor
yofnonischemiccar diomyopathy.Hepresents
toyouroff
icefoll
owi nghisthir
dhospi t
ali
zati
oninthepastyearforvolumeoverload.Heisshort
ofbreath with mi nimalexer t
ion,butappear s euvol
emic on exam.Hi s echocardi
ogram
demonstratesaleftventri
culareject
ionfr
actionof20%,t
heleftventr
icularcavi
tysizei
sdilat
ed,
andtherearenomaj orvalvul
arabnor mal
iti
es.

Pr
ognosi
sint
hispat
ienti
sbestdet
ermi
nedbywhi
choft
hef
oll
owi
ng?

a. Ri
ght-heartcat
heterizat
ion.
b. Cardi
opulmonaryexer ci
setest
ing.
c. 6-minut
ewal ktest
.
d. Seatt
leHeartFail
ureScor e.
e. Cardi
acmagnet icresonanceimaging(
MRI
).
ANSWER: 

27. b. Car
diopul
monar
yexer
ciset
est
ing.

Prognosisin heartf
ail
ureisbestdet ermined byassessmentoffuncti
onalcapaci
ty.The
acceptedindi
cati
onforheartt
ranspl
antationisanoxygenconsumpt
ionof<14cc/ kg/
min(or
12cc/kg/minint hepr
esenceofbeta-blocker
s)measuredduri
ngacardiopul
monaryexerci
se
test

TheSeat t
leHeartFai
lureScoreisknownt ounder esti
mat eri
skinpati
entswi t
hadvancedhear t
fai
lur
e,suchast heonedescr ibedi
nt hisquest i
on.Whi l
ethe6-minutewal ktestdoesconvey
prognosti
cinformati
on,itisnotasi nfor
mat i
veascar di
opulmonarystresstestingandisnot
usedasani ndicat
ionfortr
ansplant
ation.CardiacMRImaybehel pf
ulinidenti
fyi
ngt heeti
ology
ofnonischemiccardiomyopathyandcanconveypr ognosti
cinf
ormati
onbasedont heamount
ofdelayedenhancement,butisnotaswel lval
idatedascar di
opul
monar ystr
esst esti
ng.
 

28. A 62- year


-ol
d woman with chr
onic heartfai
lur
e wit
hr educed ej
ecti
on fr
acti
on i
s
transfer
redtothecoronar
ycareuni
t.Shewasst artedonmilr
inonewhenshewasadmi tt
edto
thefloor3daysagowher eshewasawaiti
nghear tt
ranspl
ant
ation.Apulmonaryart
erycat
het
er
i
spl aced. 

Thehemodynami
csar
e(i
nmm Hgwher
eappr
opr
iat
e):

 Ri
ghtatr
ialpressure:20
 Pul
monar ycapil
lar
ywedgepr essure:30
 Pul
monar yarter
ypr essur
e:60/30/40
 Car
diaci
ndex:1. 5L/ mi m2
n/
 Bl
oodpressure:80/ 50/65

Whi
choft
hef
oll
owi
ngi
sthenextmostappr
opr
iat
est
ep?

a. I
sosor bidetri
nit
r at
e.
b. Referralforaleftvent
ri
cul
arassi
stdevi
ce(
LVAD)
.
c. Removef rom tr
ansplantl
ist
.
d. Phe nylephri
ne.
e. Si
ldenaf il
.

ANSWER: 

28. b. Ref
err
alf
oral
eftvent
ri
cul
arassi
stdevi
ce(
LVAD)
.

Themostcommoni ndicati
onf oranLVADi sasabr i
dget ohear ttr
anspl antati
on.Thi spati
ent
remai nsinseveredecompensat edhear tfail
uredespi tei nt
ravenousi notropicsuppor t(i.
e.,
mi l
ri
none). Because t he t i
ming of t ransplantati
on cannot be det ermi ned, adequate
hemodynami csupportofthepat i
entmustbemai ntai
nedi nor dertoopt imizet hepreoperati
ve
stateofthetranspl
antcandi dat
e.Phenylephrinewi l
lprovidevasoconst r
icti
on, butwillnottr
eat
thel ow outputstate.Sil
denaf i
lcan increaset hepul monar ycapi l
larywedgepr essureby
decreasingthepulmonar yvascularresist
anceandover whelmt heleftventricularcompliance
from theincr
easedflowacr ossthepulmonar ycirculati
on.Ni t
rateswoul dbepoor l
ytoler
atedin
thesetti
ngofhypot ensi
on.Removalf
rom t
het r
anspl
antli
sti
spri
maril
yindi
cat
edwhenthe
patientis nota tr
ansplantcandi
dat
e due t
o sever
e comor
bidi
ti
es t
hatcompromi
se t
he
transpl
antprocedur
e.

29. A24- year-oldwomanpr esentswit


h2weeksofpr ogr essiveshor t
nessofbreath,fat
igue,
ort
hopnea,andsyncopeaf t
erar ecentvirali
ll
ness.Thei ni
ti
alechocar di
ogram evaluati
on
demonstrat
esanor malheartsizeandleftventri
cularejecti
onf ractionof20%.Despi teinit
ial
att
emptsatst abili
zat
ion ofacutedecompensat ed heartf ai
lurewi t
h medicaltherapy,the
pati
ent’
sconditi
oncont i
nuestodet er
ior
ate.Onexami nation,herbl oodpr essurei
s82/ 67mm
Hgwi t
hahear trateof115bpm.Herpul monar yar t
eryoxygensat urati
onf el
lfr
om 45%t o37%
aft
er12hoursofhi gh-
doseintr
avenousinotropes.

Hercr
eati
ninei
s2.6mg/dl
,andsheisol
igur
ic.Herment
alst
atusdecl
ines.Shei
sint
ubat
ed
andpl
acedonextr
acor
por
ealmembr
aneoxygenati
on.

Whi
choft
hef
oll
owi
ngi
sthenextmostappr
opr
iat
edi
agnost
ict
est
?

a. Endomy ocardialbiopsy.
b. Coronar
yangi ogr ams.
c. Cardi
acmagnet icresonanceimaging(MRI
).
d. Geneti
ct est
ing.
e. Cardi
accomput edt omography(CT).

ANSWER: 

29. a. Endomyocar
dialbi
opsy.

Thispat i
enthascar diogenicshocknotr esponsi
vet omedicalther apyandi sappropri
atel
y
placedonmechani calsupport.Endomyocar di
albi
opsyshouldbeconsi deredinpati
entswi t
h
rapidlypr ogressi
veclini
calhear tfail
ureandi sespecial
lyhelpfult oevaluat
eforgi antcell
myocar di
tis,whichmayr espondtoi mmunosuppr essi
vetherapies.CardiacCT, 
cardiacMRI ,
andcor onaryangiogramsar econtrai
ndicatedbecauseofacut er enalfai
lur
eandwoul dnot
provideadef ini
ti
vediagnosis.Genet i
ctestingisnotusefulwithoutapr i
orfamil
yhi st
oryof
i
diopat hiccardiomyopathy. 

30. A 64- year


-ol
d man wi thi schemic car di
omyopathy is admit
ted wi t
h wor seni
ng
short
nessofbr eat
handl owerextremityedema.Hei sonfurosemide40mgperday, aspir
in81
mg,atorvastati
n80mgatbedt i
me,andl i
sinopr
il40mgperday.Onexami nati
on,heisi nno
acutedistr
ess.Hi sbloodpr essureis115/ 75mm Hg,hear trateis72bpm,andoxygen
satur
ati
onis96%.Hi sjugularvenouspr essureiselevat
ed,l
ungsar ecl
ear,andheartisregul
ar
wit
hnor malS1 andS2 
andgr ade3/ 6hol osystol
icmur muratapex.Ext
remi t
iesarewarm wi t
h
2+edema.  

Anelect
rocar
diogr
am showsnormalsinusrhythm wit
hQwavesinV1-V4.QRSdur ati
onis110
msec.Echodemonstr
ateseject
ionfr
acti
on( EF)of30%wit
hlef
tvent
ri
cul ar(LV)end-di
ast
oli
c
di
mensionof6.0cm,moderate-
sever
emi t
ralregur
git
ati
on(
MR),andl
eftatri
alenlar
gement.

Whi
choft
hef
oll
owi
ngi
sthemostef
fect
ivewayt
odecr
easeMR?

a. Dobut
ami
ne.
b. Mit
ralvalverepair.
c. Car
diacr esynchroni
zati
ontherapy-
def
ibr
il
lat
or(
CRT-
D).
d. Sur
gicalventricul
arreduct
ion.
e. Car
vedilol.

ANSWER: 

30. e. Car
vedi
lol
.

Secondar yMR i scommonandassoci atedwi thwor seout comesi n  heartfail


ur e(HF)wi th
reducedEF.Thepr imaryappr oacht othetreatmentofMRi nt hi
ssettingi nvolvesopt imizat
ion
ofmedi calt herapy f or reduced EF.The i mprovementi nr emodel ing associ ated with
opti
mi zati
onofmedi cati
onwi llresul
tindecr easedMR.CRT- Dalsowi llreducef unctionalMR
duet oimpr ovedr emodeling, buti nthiscase,CRT- Disnotindicatedbecauset hepat ientisnot
onopt imalmedi caltherapyandhasanar r
owQRS.Sur gi
calther
apiesf ortheMRi nt hissett
ing
havenotbeenpr ovent oimpr ovemor tal
it
yandar eonl yusedasapal li
ati
onf orrefract
ory
sympt oms.  
Inot
ropesi nchr oni cHFar eonlyusedaspal l
iati
onorasabr idget oadvanced
therapies.
 Theaddi tionofbet a-blockerstot hispatient'
sregimenof fer sthebestst rategyto
i
mpr ovehisMRt hroughLVr emodel i
ngandpr ovidesmor tal
it
ybenef i
t.
 

31. Whi
choft
hef
oll
owi
ngi
sthel
ikel
ihoodofsur
vivalat5year
saf
terhear
ttr
anspl
ant
ati
on?

a. 90%.
b. 70%.
c. 60%.
d. 80%.
e. 50%.

ANSWER: 

31. b. 70%.

Sur vivalafterhearttranspl
antat
ionhasi ncreasedsteadi
lysincethe1980s.Accor dingtothe
Internat i
onalSociet
yofHear tandLungTr ansplantat
ion,t
hesurvivalaf
terhear
ttransplant
ati
on
is90%at1year ,
70- 75%at5year s,
and50%beyond10year s.Incontr
ast,inthe1980s, t
he5-
year sur vival r
ate af t
er hearttransplantati
on was 65%. The medi an survi
val after
transpl antati
onint he1980swasappr oximately8yearsandi scurrent
lyapproximately11
year s. 

32. Whi choft


hef
oll
owi
ngi
sthemostcommoncauseofdeat
hint
hef
ir
stmont
haf
terhear
t
tr
anspl
ant
ati
on?

a. Malignancy.
b. Rej
ect i
on.
c. I
nfection.
d. Cardiacall
ograf
tvascul
opat
hy.
e. Pri
mar ygraftf
ail
ure.

ANSWER: 
32. e. Pr
imar
ygr
aftf
ail
ure.

Themostcommoncauseofdeat hinthefi
rst30dayspostcardi
actranspl
anti
spri
mar
ygraft
fai
lur
e,def
inedasdysfunct
ionoft
hetransplant
edorganwi
thinthefi
rst24hour
s.Ri
skf
actors
forthedevelopmentofpri
marygraftdysfunct
ionmayincludeolderdonorageandlonger
i
schemicti
mes.  

Whiletherei sasignif
icantriskofinfect
ionimmedi at
elypostt r
anspl
ant,i
tisnotascommona
causeofdeat hwithi
n30daysaspr i
marygr af
tdysfunction( 13%vs.39%,respect
ivel
y).Acute
cel
lularrejecti
onisseeni nthef i
rstyearposttransplant,buthyperacuterej
ect
ionimmedi at
ely
aft
ersur geryisrar
elyani ssue.Malignancyandvascul opathyar enotcommoncausesofdeat h
wit
hint hefirst30days,butar esi
gnifi
cantlong-t
erm compl icati
ons. 

33. A57- year-oldmanpr esent


sf orhi
st hi
rdhear tfai
lure(HF)hospital
izat
ionint hepast
year.Hehadal argeant eri
ormyocardiali
nfar
ction(MI)3year sagoandhasanej ecti
onf r
acti
on
of20%.Hi s medi calregimen incl
udes met oprol
olext ended-rel
ease 100 mg once dai ly,
li
sinopr
il40mgdai ly,
furosemide120mgt wi
cedai l
y,atorvastat
in80mgoncedai ly,aspir
in81
mgoncedai ly,andwar fari
n5mgdai l
y. 

He had an i mplantabl
e cardi
overt
er-def
ibri
ll
ator(ICD)pl aced 1 yearafterhi
s MI .His
el
ectrocardi
ogram showsat r
ialf
ibri
ll
ati
onandaQRSdur ati
onof110msec.Hi sbloodurea
nit
rogeni s80mg/ dl
,andhi screati
nineis1. 9mg/ dl.Echoshowsadi l
atedlef
tventri
cle,
moder at
ely severe mi t
raland tri
cuspidr egurgi
tat
ion,leftatri
alenlar
gement,and large
hypokinet
icrightventri
cle.

Whi
choft
hef
oll
owi
ngi
sthemostappr
opr
iat
enextst
ep?

a. Cardi
acresynchronizati
ontherapy.
b. Ref
erralf
ormitralvalverepl
acement.
c. Ref
erralf
orhearttransplant
.
d. Hydral
azi
ne.
e. Dobutamine.

ANSWER: 

33. c. Ref
err
alf
orhear
ttr
anspl
ant
.

Ani mportantaspectofchr oni


cHFmanagementi stherecogni
ti
onofadvancedHFandt he
tr
igger
sf or ref
err
alf or advanced therapies such as hearttranspl
ant
ati
on.Fr equent
hospit
ali
zations,car
diorenalsyndrome,r ecurr
entICD shocks,and poorfunct
ionalstatus
despit
eopt i
malmedi calther
apyareconsideredmilest
onesofworseni
ngadvancedHFandar e
associat
edwi thhi
ghmor tali
ty.
 

34. A 55- year


-oldmanwi thchroni
chear
tfai
lur
e( HF)withreducedej
ect
ionf
ract
ioni
s
admitt
edwit
hper si
stentdyspnea.Duri
nghi
shospit
ali
zati
on,hi
ssympt omsar
eref
ract
oryt
o
i
ncreasi
ngdosesofdiuret
ics.
 

Onexami nati
on,hi
sbodymassi ndexis38kg/m2,bl
oodpressur
eis90/
70mm Hg,andhear t
rat
eis100bpm.Hi svenouspressur
ecannotbeassessed.Hisl
ungsar
eclear
,heartsounds
aredi
stant,andabdomenisobese.Hisext
remi
ti
esareedematouswi
thbr
awnydiscol
orat
ionof
t
heover
lyi
ngski
n. 

Hislabor
atori
es(mg/dl)ar
enot
abl
eforasodi
um of134,
pot
assi
um of5.
0,bl
oodur
eani
tr
ogen
of70,andcreati
nineof2.
5.

Whi
choft
hef
oll
owi
ngi
sthenextmostappr
opr
iat
est
ep?

a. Cardi
opulmonar yexercisetesti
ng.
b. Echocar
diography.
c. Pulmonaryarterycatheteri
zati
on.
d. ChestX-r
ay.
e. Brai
nnatri
ureticpeptidelevel.

34. c. Pul
monar
yar
ter
ycat
het
eri
zat
ion.

Thispatientwithacut edecompensatedhear tfail


ure(ADHF)i sfai
li
ngtoi mprovewit
hempi ri
c
diuret
icmanagementandhasanuncl earhemodynami cpr of
il
efrom bedsideassessment .
Althoughal ow out putsyndromei spossible,themostcommonhemodynami cabnormalit
y
associatedwi t
hADHFi sanel evat
edcentralvenouspr essure.I
nordertodeterminethecause
oft his pat
ient'
sr efract
ory HF and cardiorenalsyndr ome,invasive measurementoft he
hemodynami csar erequired.However,ther outi
neuseofpul monaryar t
ery(PA)catheter
-
guidedtherapyf orADHFi snotrecommendedasnot edi ntheESCAPEt r
ial

Thepr imaryindicati
onsforPAcat heterizationinHFi ncludediagnost i
cuncer taintywithrespect
tot hehemodynami cprofi
le,conf irmat ionofWor ldHeal thOr ganizati
onGr oup1pul monary
arteri
alhypertension,andinpr eparationf oradvancedt herapi
es,suchashear ttransplantat
ion
orvent ri
cularassistdevices.Theot herchoi cesar elimitedbyt heirindir
ectassessmentof
hemodynami cs,whi charecommonl yi nsensiti
vetosubt l
echangesi nhemodynami cstatus.B-
typenat r
iuret
icpept ide(
BNP)canbef alselylowinobesi t
yduet oi ncreasedcl earanceofBNP
byendopept i
dases.Echocar diographyr equirescertain assumpt ionsi n ordert o determine
absolute r at
her t han relative hemodynami c values ( e.
g., assessment of pul monary
hypertension).

35. Admi ni
str
ation ofwhich ofthe fol
lowi
ng agent
s shoul
d be accompani
ed by an
assessmentofl
eftvent
ricul
ar(
LV)funct
ion?

a. Dexrazoxane.
b. Suni
tinib.
c. Radi
at i
on.
d. 5-Fl
uorouracil(
5-FU)
.
e. Cort
icoster
oids.

ANSWER

35. b. Suni
ti
nib.

Cancerchemot her
apyi scommonl yassociat
edwi t
hcardiovascul
aradver
seeff
ects.Alt
hough
anthracycli
nes(e.
g.,doxor
ubici
n)andt r
astuzamab( Hercept
in)ar
egenerall
ywell
-recogni
zed
agentst hatcausemyocar di
aldysfunct
ion,manyot heragentsrequi
reclosecardi
ovascul
ar
surveil
lance.Suni
ti
nib(Sutent)
,at yr
osi
neki naseinhi
bitor
,iscommonl yusedforrenalcel
l
carcinomaandgast r
icst romaltumor sandmaycauseLVdysf uncti
on,part
icul
arl
yi fpat ient s
experiencehyper tensiondur i
ngt her
apy.Dexrazoxaneisusedt opr ot
ectagainstanthracycl i
ne
cardiotoxici
tythroughf reer adi
calscavenging.Radiati
onmaypr oduceaccelerat
edcor onar y
art
ery di sease,val vulari nj
ury,and per i
cardi
tis,butgener all
y only pr
oduces syst oli
c LV
dysfunction int he set ting ofr eceivi
ng anothercardiotoxic agentsuch as doxor ubi cin.
Surveill
anceechocar diographyi sgenerall
ynotneededdur ingchestr adiat
ionadministrationi n
theabsenceofsympt oms.St eroidshavenodi rectcardiotoxicit
y.5-FUmaycausecor onar y
vasospasm, butdoesnotgener all
ycausedi r
ectmyocardialinjury. 

36. A45- year-ol


dmanwaspr evi
ousl
yhospit
ali
zedforbiopsy-
provenacutelymphocyti
c
myocar dit
is.Initi
alechocar diogr
am demonst r
ated bi
ventr
icul
ar dysfunct
ion wi
thal ef
t
ventr
icularejecti
onf r
acti
on(LVEF)of25%andnor malcavi
tydimensions.Hewasdi schar
ged
withNewYor kHear tAssoci
ation(NYHA)classI
Isymptoms. 

Ona6- mont hfoll


ow-upvisi
t,heendor sesNYHAcl assI Isympt
oms.Anechocardi
ogr
am is
obtainedthatdemonst r
atesrightventri
culardysfuncti
on,anLVEFof20%,andamoder at
ely
di
latedleftvent
ricl
e.Hiscurr
entmedi cati
onsincludel i
sinopri
l20mgqd,car
vedi
lol25mgbid,
andspi r
onolact
one25mgqd.
 
Whichoft hefol
lowingisthemostappr opri
atenextstepinmanagementofthi
spatient
?

a. Ref
erralf
orleftvent
ricul
arassi
stdevi
ce(LVAD)pl
acement.
b. St
artrosuvastati
n.
c. St
artwarfarin.
d. St
artprednisone.
e. Ref
erralf
orimpl ant
ablecardi
over
ter
-def
ibr
il
lat
or(
ICD)pl
acement
.

ANSWER: 

36. e. Ref
err
alf
ori
mpl
ant
abl
ecar
diover
ter
-def
ibr
il
lat
or(
ICD)pl
acement
.

Pat
ient
swit
hnonischemiccar
diomyopat
hywit
hEF<35% andNYHAclassIIorIIIsymptoms
whoareongui
del
ine-di
rect
edmanagementf
oratl
east6mont
hshavesur
vivalbenef
itwi
thICD
pl
acement

Therei sl it
tl
eevidencet hata  denovo  presentationofvi r
alorl ymphocyt i
cmyocar dit
isi s
responsive t o st
eroids.However ,evidence suggest sthatsome pat i
ents wi th chronic
i
nflammat ory myocar dit
is such as ongoi ng l ymphocyt i
c myocar dit
is may r espond t o
i
mmunosuppr essi
on,especi all
yt hat subgroup wi t
hout evidence ofcar di
otropic viral
ly
medi at
edmyocar dit
is.Ingener al,pati
entspr esentingwithfulminantlymphocyticmyocar dit
is
presentmor edramat i
call
yandar emor el i
kelytoneedmechani calsuppor t
,butar ealsomor e
l
ikelytor ecovermyocar dialfunctionversust heacut epresentat
ion.Giantcellmyocar dit
isisa
parti
cularly mor bid i nflammat ory myopat hy. Whi l
e i t may r espond t o mul ti
ple
i
mmunosuppr essi
veagent s,transplant
-freesur vivalisparti
cularl
ylow;t hus,diagnosisand
earlytr
eat mentisrequired. 

Therearenodatatosuppor
ttheuseofwarf
ari
ninpati
entswit
hdil
atedcardi
omyopat
hyi
nthe
absenceofat
ri
alfi
bri
ll
ati
on,
LVthrombus,
orotheri
ndi
cati
onforant
icoagul
ati
on.
 

St
ati
nshavenotdemonst
rat
edbenef
iti
npat
ient
swi
thnoni
schemi
ccar
diomyopat
hy.LVADs
ar
eindi
cat
edf
ormor
eadvancedhear
tfai
lur
e. 

37. A 78- year


-old woman presentsto youroffi
ce f
orconsul
tat
ion.She complains of
dyspneawi
thexert
ionandlowerextremi
tyswelli
ng.Hermedi
calhi
stor
yincl
udesdiabetesand
dysl
ipi
demi
a. 

Onexami nat
ion,shehasanel evatedjugularvenouspul sat
ion,ar egul
arrat
eandr hyt
hm,faint
crackl
esatt hel ungbases,and2+l owerext remit
yedema.Shehashadanechocar di
ogram
thatdemonstratesanor mall
eftvent
ri
cularejecti
onfracti
on( EF).Youtel
lherthatshehasheart
fail
urewith preserved EF(HFpEF).Sheasksyou f oraddi t
ionalinf
ormati
on regar
ding her
diagnosi
s.

Whi
choft
hef
oll
owi
ngbestdescr
ibeschar
act
eri
sti
csofHFpEFi
nthegener
alpopul
ati
on?

a. Malesexisar i
skfact
orforHFpEF.
b. Mortal
it
yissimi l
artoHFwi threservedEF(HFr
EF).
c. Angi
otensin-converti
ngenzymei nhibi
tor
sareassoci
atedwi
thr
educt
ioni
nmor
tal
it
y.
d. Overal
lprevalanceofHFpEFi sdeclini
ng.
e. Approxi
mat el
y25%ofpat ientswithHFhaveHFpEF.

ANSWER: 

37. b. Mor
tal
it
yissi
mil
art
oHFwi
thr
eser
vedEF(
HFr
EF)
.

PatientswithHFpEFver susanHFr EFar eaboutequali nprevalence,withsi milarmortal


it
y
outcomes.HFpEFpat ient
s,however ,aremor eoft
enf emale,older,andhypert ensive.Wi
ththe
agingoft hepopulation,thepr eval
enceofHFpEFi sincr
easing.Ot herassociatedconditi
ons
seenwi thHFpEFi ncludeobesi ty,atr
ialfi
bri
ll
ati
on,dysl i
pidemia,anddi abetes.Whi leuseof
medi cat
ionsfortr
eatmentf orbloodcontrolandvolumemanagementi sappropr i
ate,atpr
esent,
therearenodr ugsthathavepr ovenbenefitoncardiovascul
arout comes. 

38. A65- year


- ol
dwomanpr esent
stoyouwi thcomplai
ntsofdyspneaonexerti
on.She
deni
es dyspnea atrest
,or t
hopnea,orparoxysmalnoctur
naldyspnea.She deni
es lower
ext
remi
tyedema.Hermedi calhi
storyi
sonl
ynotablef
orhyper
tensi
onandhyper
li
pidemia.
 

Onphysi calexaminati
on,herbloodpr essurei s154/ 90mm Hgandot her
wiseunr emarkable.
HerN- ter
minalpro–B-typenatriuret
icpept ideis150pg/ ml.Anexerci
semyocar dialperf
usion
i
magi ngstudyisperformed.Sheexer ci
ses4mi nutesonast andardBruceprotocol,andstops
duet odyspnea.Herst r
esselectrocardiogram iswi t
houtischemicchangesandt heper f
usion
i
magesdonotshow evi denceofi nfarctori schemi a.Yoususpectshehashear tfail
urewith
preservedeject
ionfr
acti
on( HFpEF)andor derar ight-hear
tcathet
eri
zat
ionwi t
hexercise.

Whi
choft
hef
oll
owi
ngi
sthemostl
ikel
yfi
ndi
ngwi
thexer
cise?

a. Inappr opr
iat
eincreaseinheartrat
e.
b. Exagger at
edincreaseinpulmonaryart
erypressur
es.
c. Increaseinthemi xedvenousoxygensatur
ation.
d. De creaseinpulmonar ycapi
ll
arywedgeV-waveamplit
ude.
e. Sy st
emi chypotension.
ANSWER: 

38. b. Exagger
atedi
ncr
easei
npul
monar
yar
ter
ypr
essur
es.

HFpEFi sadi ffi


cul tdiagnosi st omakeandi soccasi onal l
yoneofexcl usionifpat i
entsarenot
clearl
y congest ed atr est.Whi lear i
ght -
heartcat heteri
zation may demonst r
ate normal
hemodynami cs,ar ight- heartcat heteri
zati
onwi t
hexer cisecanof t
enconf i
rm asuspect ed
diagnosi
sbyshowi nganexagger atedelevat i
oninsyst emicbl oodpressureandi nt
racardi
ac
fi
lli
ng pressurer esponset o exercise.Forpat i
entswi th normalhemodynami csatr est,a
pulmonar yarterysyst olicpr essurewi thexerciseof>45mm Hgsuggest sadi agnosisofHFpEF
withhighsensi ti
vi t
y.Of tentimes,augment ationincar diacout putisbluntedandi nsuff
ici
entto
supporttheexer ciseandl owert hansi milarpatientswi t
houtHFpEF,l ikel
yr el
atedt olackof
systol
icanddi ast oli
cr eserve.HFpEFi softenassoci atedwi thlackofchronotropicresponse. 

39. Tr
oponi
nisnecessar
yforwhi
choft
hef
oll
owi
ngf
unct
ionswi
thi
nthehear
t?

a. Mitochondri
alel
ectront r
ansport.
b. Propagati
onoftheact ionpotential
.
c. DNAr epli
cat
ionandmyocar dialcelldivi
sion.
d. Cardiomyocytecontracti
onandr elaxat i
on.
e. Maintenanceofcellmembr anef l
uidity.

ANSWER: 

39. d. Car
diomyocyt
econt
ract
ionandr
elaxat
ion.

Myosin,actin,t
ropomyosin,and t
hetroponi
nsaresarcomericprotei
nsthatinteractwi
th
cal
cium andadenosinetri
phosphat
etoproducemyocar
dialcontr
act
ionandrelaxati
on.None
oft
heot heranswerchoi
cesarecorr
ect
.

40. Exci t
ati
on-cont
ract
ioncoupl
ingdescr
ibesthepr
ocessbywhi chelect
ri
calact
ivat
ionof
thecar
diomyocyt
eviaanacti
onpotenti
alr
esult
sinmyocar
dialcont
ract
ion.

Whi
choft
hef
oll
owi
ngoccur
sfi
rstwi
thi
nthepr
ocessofexci
tat
ion-
cont
ract
ioncoupl
ing?

a. Entryofcalci
um i
ntot hecell
.
b. Displacementoftr
opomyosi nf rom acti
n-bindingsit
es.
c. Releaseofcalci
um from thesar coplasmicreti
culum.
d. Mov ementofmyosinheadsal ongt heactinfi
lament.
e. Dissociat
ionofcal
cium from troponin.

ANSWER: 

40. a. Ent
ryofcal
cium i
ntot
hecel
l.

Entryofcalci
um intothecelloccur
sf i
rst
.Subsequent l
y,t
herei
sr el
easeofcalci
um f rom t
he
sarcoplasmicreti
culum (cal
cium-i
nduced calci
um r el
ease)
.Cal cium binds t
ot roponin C,
i
nducing conformati
onalchangest hatdisplacetr
opomyosi nfrom t heacti
n-binding si
tes.
Therei s cr
oss bri
dge formati
on bet
ween myosi n and act
in with adenosi
ne tri
phosphat e
hydrolysi
s.Thepowerst r
okeconsistsoft hemovementofmyosi nheadsalongt heacti
n
fi
lament .Adenosi
netr
iphosphate-bi
ndi ngisneededf orreleaseofmyosinheadsfrom t
heactin
fi
lament .Thiscycl
econt i
nuesunt i
lintracell
ularcalcium level
sfalll
ow enoughthatcalci
um
dissociatesfr
om tr
oponin,causi
ngar eturntot heori
ginalconformati
onoftropomyosi
nwher e
i
tbl ocksmyosi nfr
om accessingtheact in-bi
ndi ngsi
tes.Thismar kstheendofthecontr
acti
le
cycle.

41. Fi gur
e1depict
sapressur
e-vol
umel
oopoft
hel
eftvent
ri
cleatr
est(
cur
veA)andaf
ter
ani
nter
venti
on(
curveB).

Whi
choft
hef
oll
owi
ngi
smostl
ikel
ytor
epr
esentt
hei
nter
vent
iont
hatwasadmi
nist
ered?

a. Metoprolol.
b. Norepi
nephr ine.
c. Phenylephrine.
d. Fur
osemi de.
e. Dobutami ne.

ANSWER: 

41. e. Dobut
ami
ne.

Pressure-vol
umel oopsdepi ctchangesi n vent
ri
cularpressureagai nstvent r
icularvol
ume
duri
ngasi nglecardiaccycle.Thest rokeworkoft hevent r
icl
ei srepresentedbyt hearea
enclosedbyt hepr essur
e-volumel oop.Changesi nstr
okewor karei nfl
uencedbypr el
oad,
aft
erload,andint
ri
nsiccontracti
li
tyofthemyocardium.Thepr essure-
volumel oopcanbeused
todescribethecompl i
anceandener geti
cstat
eoftheventri
cle.CurveBdemonst ratesashif
tin
theend- systol
icpr essur
e-vol
umer elati
onshi
p( ESPVR)t hatisconsistentwit
hani ncreasei n
cont r
acti
li
ty.Al soseeni sani ncreasei nstr
okevol ume( t
hedifferencebetweent heend-
diastoli
candend- systol
icvol
umes) .Theshif
tf r
om cur veAt ocurveBi sconsi
stentwi t
ht he
effectsofdobut amine,whichactsprimaril
yasabet a-1adr ener
gicagoni st
.Theend- diastoli
c
volumei sr educed,becausethei ncreasedcontract
il
ityoftheleftvent r
icl
ehasr esul
tedi na
lowerend- systoli
cvolumebeforeonsetofdiastoli
cfi
ll
ing. 

Fur osemide woul d show a decr ease in preload,but woul d not affect contractil
ity.
Phenyl ephri
nei sadi rectalphaagonistthatresultsinar t
eri
alvasoconstr
ict
ionandi ncreased
afterload,butdoesnotaf fecttheESPVR ( contractil
it
y).Norepi
nephri
nehaspr edomi nant ly
alpha- 1 adrenergic acti
vit
yal ong withal owercomponentofbet a-1 adrenergic act
ivity,
resulting i
ni ncreased afterl
oad and a smal lincreasei n cont
ract
il
it
y.No i ncr
easei nt he
afterloadisseeni nthisdiagram.Metoprololwouldr esul
tinadecreaseincontracti
li
ty,depicted
byacl ockwiseshift
/lesssteepESPVRcur ve.

42. Youar eseeinga63- year


-ol
dwomani ncl i
nicforfollow- upaftershewashospi tali
zed
l
ast week wi t
h an ant erior wal
lST- segment myocar di ali nf
arcti
on ( MI)
.Dur i
ng t he
hospit
ali
zation,sher eceivedt wodrug-eluti
ngst ent
st oherl eftanteriordescendingar t
ery.
Bipl
ane vent ri
cul
ogr aphy at t he ti
me of pr imar y percut aneous cor onaryi nter
vent i
on
demonstratedanterior,l
ateral,andapicalhypokinesi
swi t
hl ef tventri
culareject
ionfractionof
40%.Herel ectr
ocardiogram i sshowninFi gure1.Hercar diacmedi cati
onsincludeaspi rin81
mg,clopidogrel75mg,met oprol
olsuccinate100mg,l isi
nopr i
l10mg,spi r
onolactone25mg,
andatorvastati
n80mg.

Whichoft
hef
oll
owi
ngneur
ohor
monesi
smostl
ikel
ytopr
eventmal
adapt
iver
emodel
ingi
nthi
s
pat
ient
?
a.Endotheli
n-1.
b.Norepinephri
ne.
c. Al
doster
one.
d.Atri
alnatri
uret
icpepti
de.
e. Ar
gini
ne-vasopressi
n.

ANSWER: 

42. d. At
ri
alnat
ri
uret
icpept
ide.

Acommonset tingformyocar di
alremodel ingisMI ,especiall
yoft heanteri
orwal l
.Persist
ent
acti
vationoft hesympat heticnervoussyst em,partl
ymedi at
edt hroughnorepi
nephr i
ne,aswell
asact ivati
onoft her eni n-
angiotensin-al
doster
onesyst em resultinincreasedendot heli
n-1
product i
on,col l
agen t urnover,reduced ni t
ri
coxi deact ivi
ty,and cell
ularapopt osi
s.These
cell
ularphenomena t ranslat
ei ntoi ncreased ventricularsize and spher i
cit
y,decr eased
contractil
eper formance,andevent uall
yhear tfai
lurei nmanypat i
ent
s.Theneur ohormonal
mediator s(norepinephrine,angiotensin,al
dosterone,andvasopr essi
n)arealltar
getsofdr ugs
thatreduceorevenr ever seremodel i
ngandl eadtoi mprovedcl i
nicalout
comesaf terMI. 

Endot
hel
ini
sact
ivat
edbyangi
otensi
nandr
educedbyangi
otensi
n-conver
ti
ngenzymei
nhi
bit
or
andangi ot
ensi
n- r
eceptorbl ockert herapy;specif
icendothel
ininhi
bit
orshavebeenshownt o
reduceremodelinginani malmodel saswel l.Natri
ureti
cpepti
desar evasodi
lat
orsthatalso
maydi rectl
yinhibitmyocyt ehyper tr
ophy.Ani malmodel sand smal lhuman studi
eshave
demonst r
atedreducedr emodel ingaf terMIwi t
hi nf
usionsofbothbrainandat r
ialnat
ri
ureti
c
pepti
des.However ,theonecl inicalstudytodat edidnotfindanychangeincli
nicalout
comes
suchasdeat horrehospital
ization.

43. A68- year-


oldwomanpr esent
st oyourcl inicforevaluationofherpr ogressivelower
extr
emityedemaanddyspneaonexer tion.Hermedi calhist
oryincludeshypert
ension,diabetes,
andchronickidneydi
sease.Sheisonl isinopri
l,hydrochlorot
hiazide,andinsul
in.Transthoraci
c
echorevealslef
tventr
icul
ar(LV)eject
ionf ract
ionof65%, moder ateconcentr
icLVhyper trophy,
normalvalvul
arfunct
ion,pul
monar yarterysystoli
cpr essureof45mm Hg, andE/ e'rat
ioof18.

Whicht
hefol
lowingther
api
eshavebeenprovenef
fect
ivet
oreducehermor
tal
it
yfr
om herhear
t
f
ail
urewit
hpreservedej
ect
ionf
ract
ion(
HFpEF)?

a. Not herapyr educesmortal


it
y.
b. Diureti
cs.
c. Beta-blocker s.
d. Angiotensin- conver
ti
ngenzymei
nhi
bit
ors.
e. Spironolactone.

ANSWER: 

43. a. Not
her
apyr
educesmor
tal
it
y.

Unfort
unat
ely,noneofthecurrentther
api
esusedforheartf
ail
urewithreducedejecti
onf r
acti
on
havebeenshownef f
ect
iveforHFpEF.Cli
nicalt
ri
alsi
nHFpEFhavepr oducedneutralresult
sto
date,andt
reatmenti sl
argelydirect
edtowardassoci
atedcondit
ions( e.
g.,hyper
tension)and
symptoms(e.g.,edema).
 

44. A40- year-oldmani sreferr


edforeval
uationofexerti
onalchestpai nandshor t
nessof
breat
h.Hismedicalhist
oryisnotablef
orhypercholest
erol
emia,forwhi chhetakesrosuvast
at i
n
20mgqhs.Hei sal if
eti
menonsmokeranddoesnotdr inkal cohol.Ther
ei snohi stor
yof
hypert
ensi
on.Thef amil
yhistor
yi snot
ableforhismat er
nalgrandfatherandhisbrotherhaving
devel
opedheartfail
ure(HF)i
nt hei
r50s. 

Hisexaminati
onreveal
sabloodpressureof110/
70mm Hg,ahear tr
ateof80bpm,jugul
ar
venousdist
enti
onat16cm H20,cl
earlungsonauscul
tat
ion,anS4 
gall
op,andmi
ldper
ipher
al
edema. 

Anelectrocardi
ogr am revealssinusrhythm,withgeneral
lylowQRSvol tageandnoevi denceof
apr i
ormyocar dialinfarcti
on.Hisechocar di
ogram showsmoder ateconcentri
cleftvent ri
cul
ar
(LV)hyper t
rophywi t
hanej ecti
onf r
action(EF)of50% andnoval vul
arabnormal i
ties.Init
ial
l
aboratoriesarenot ableforasodi um l
evelof132mEq/ L,serum creati
nineof1.8mg/ dl ,
andan
N-terminalpro-brainnat r
iuret
icpepti
de( NT-proBNP)of1800pg/ ml .Hi
scompl et
ebl oodcount
wasnor malwi t
hanor malwhi tebloodcountdiffer
ent
iati
on. 

Atcar
diaccat
heteri
zat
ion,hi
scoronar
yar t
eri
esarenor
mal
.Thepressur
emeasur ement
sare:
meanrightat
ri
al,10mm Hg;r i
ghtventr
icl
e,55/
14mm Hg;pulmonaryart
ery,55/25mm Hg
wit
hmeanof35mmHg;andpulmonar
ycapi
ll
arywedge,
25mm Hgwi
thaVwaveof32mm Hg.
Cardi
acout
putwas4.
0L/
min.

Whichofthefol
lowi
ngi
sthemostr
easonabl
euni
fyi
ngdi
agnosi
sfort
hispat
ientwi
thHFand
pr
eservedEF?

a. Amy loi
dosis.
b. Glycogenst oragedisease.
c. Fabry'sdisease.
d. Loeffl
erendocar di
ti
s.
e. Sarcoidosis.

ANSWER: 

44. c. Fabr
y'sdi
sease.

ThecorrectanswerisFabry'sdisease,analpha-gal
act
osi daseAdef i
ciency.Thispresentat
ion
wit
hHFi nthesett
ingofreasonablypreservedEFassociatedwi t
hchestpai n,normalcoronary
art
eri
es,LVhyper tr
ophy,andr enaldysfuncti
onismostt ypicalofFabry'sdisease.Thefami l
y
hi
stor
yi sconsi st
entwi t
h at ransmission ofan X-l
inked disorder
.Si ncet hebrotherand
maternalgrandf
atherwerealsoaffect
ed, hi
smothermayhavebeenanunaf f
ectedcarri
erofthe
mutati
on.Ar epl
acementtherapyisavailabl
eatahighcost . 

Theechocar diographi cfindingsandr enalinsuffi


ciencyar eatypicalforpat ientswi t
hLoef f
ler
endocar dit
is,whi ch isnoti nherit
ed,and i schar acteri
sti
cal
lyassoci ated wi t
h endocar dial
thickeningandmur althrombiaswel lasper ipher
aleosi nophi
li
a.Car di
acsar coidosisisal soan
acqui r
edr athert hani nher i
teddi sorder,andi scommonl yassociatedwi t
hat rioventr
icularblock
orvent r
icularar rhythmi asi naddi ti
ont oHF.Whi l
efami li
alamyloidosisrelatedt oelaborationof
mut antt r
anst hyretinal somaybeassoci atedwi t
hcar diacandr enalinvolvement ,autosomal
domi nanti nher i
tancei st ypicalandusual lymani f
estsatanol derage.Pat i
ent swithgl ycogen
storagedi seasest ypicall
ypr esentwi t
hHFatayoungage,andevenwi thappr opri
atet herapy,
rarelysurvivet oadul thood.

45. A50- year


-ol
dmanwi thnosi gnif
icantmedi calhi
storypresent
swit
hpalpi
tat
ions.On
examinat
ion,
hehasami l
dlyelevatedjugularvenouspressureat10cm H20,
afai
ntparast
ernal
l
ift
,andaprobabl
eri
ghtventr
icular(RV)S3 
gallop.

Hisel ectr
ocar diogr am isnot ablef orsinusrhyt
hm,wi thani ncompl eter ightbundl ebr anch
block.AHol termoni torrevealsf requentr unsofnonsustai
nedvent riculart achycardiawi t
ha
lef
tbundl e br anch bl ock pattern.Cor onaryangi
ographyr eveals no si gnificantepicardial
coronar yarterydi sease.Echocar di ographysuggestsRVcavitydilat
ionandmoder at
etri
cuspid
valver egurgitati
on.Magnet icr esonancei magi
ngi ssuggesti
veoff at t
yi nfil
trationoftheRV,
consistentwi thar rhyt
hmogeni cRVcar diomyopathy(ARVC).

Whi
choft
hef
oll
owi
ngpr
otei
nsmostl
ikel
yhasadef
ecti
nit
sgeneencodi
ng?

a. Lami nAandC.
b. Tafazzin.
c. My osinheavychai
n.
d. Plakoglobi
n.
e. Dyst
rophi
n.

ANSWER: 

45. d. Pl
akogl
obi
n.

Thecor rectansweri splakoglobin.ARVCi sanaut osomaldomi nantfamil


ialdi seaseoft he
desmosome,l i
nkedt omut ati
onsi nseveralgenesencodi ngdesmosomalpr oteins,including
j
unctionalplakoglobin.Myosinheavychai nmut at
ionsmaybeassoci at
edwi thbot hdi l
ated
cardi
omyopat hyandhyper trophiccardi
omyopat hy.Similar
ly,mutati
onsindyst rophinmaybe
associat
edwi t
hmuscul ardyst r
ophyanddi l
atedcar di
omyopat hy,andmutat i
onsi nt afazzin
havebeenassoci atedwith dil
atedcardiomyopathyandl eftventri
cul
arnoncompact i
on,but
neit
heristypicalofARVC.

46. A 62-year-
oldmanwi t
hahistor
yofhypert
ensi
onisref
erredwit
hadecreasedlef
t
vent
ri
cul
arej
ect
ionfr
act
ion(
LVEF)
.Hehasahi
stor
yofpoorcompl
iancet
ohismedi
cat
ions.
 

Hisli
stedmedicat
ionsi
ncludeval
sart
an80mgdai
ly,at
enol
ol50mgdail
y,andf
urosemide40
mgdai ly.Hi
sbloodpressureis130/85mm Hgandhisheartr
atei
s80bpm andr egul
ar.
Cardi
acexaminati
onisnormal.

Echocar
diogr
aphyr
esul
tsshowamoder
atel
ydi
lat
edLV,
LVhyper
trophy,
andanLVEFof30%.

Which oft
he f
oll
owing i
sthe mostappr
opri
ate nexttherapeut
ic step,based on cur
rent
gui
deli
nesf
ort
hetreat
mentofasympt
omati
csystol
icvent
ricul
ardysfuncti
on?

a. Changeatenololtocarvedil
ol.
b. Beginadiuret
ic.
c. Changevalsartantoli
sinopri
l.
d. Referf
orprophylacti
cimplantabl
ecar
diover
ter
-def
ibr
il
lat
or(
ICD)
.
e. Beginanaldosteroneblocker.

ANSWER: 

46. a. Changeat
enol
olt
ocar
vedi
lol
.

Thecorrectansweri st ochanget hebeta-blockertoapr ovenone.Al t


houghat enol
olisan
ef
fect
ivebet a-
blocker,
ithasnotbeenst udi
edi nheartf
ailur
e.Theref
ore,t
heef f
icacyandother
cl
ini
call
yrelevantissues,suchast ar
getdose,ar enotknown.Most ,butnotall
,beta-bl
ockers
havebeenf oundtobeef fect
iveinchr
onicsystoli
cheartfail
ure.Gui
deli
nesrecommendt heuse
ofagent
st hathavebeenst udiedincl
ini
calt
rials.
 

Angiot
ensin- r
eceptorbl ockersappeart obeatl eastasef f
ecti
veasangi otensi
n-conver t
ing
enzymei nhibit
orsandar eacceptablechoi ces,parti
cularlyinpatient
sal r
eadyr eceivi
ngt hem
fort
het r
eatmentofhyper tensi
on.Aldosteronebl ockadeisnotr ecommended, unlessadequat e
l
aboratoryfollow-upcanbeensur ed.Diur eti
csanddi goxinareusef ulonlyf orsympt omat i
c
heartfail
ure.The i ndications f
oran I CD,based on t he Amer i
can College ofCar diology
Foundati
on/ Amer i
canHear tAssoci
ation/Hear t
hRhyt hm Soci ei
tyguideli
nes,i ncl
udeanLVEF
<35%wi t
hNewYor kHear tAssociati
oncl assII-I
IIheartfail
ure(ClassI,LevelofEvidence[ LOE]
B).Thepat i
enti scurrentlyasymptomat i
c,butanI CDcoul dbeconsidered( ClassIIb,LOEC) .
Thebestanswert
houghi
stoensur
ethepat
ienti
songui
del
ine-
dir
ect
edmedi
calt
her
apy.
 

47. A62- year-


oldAfr
ican-Americanmanwi thahi stor
yofhypert
ensionwasr efer
redwitha
decreasedl
eftventri
cul
areject
ionfr
action(LVEF).Hedeniesdyspnea,f
atigue,chestpain,li
ght-
headedness,orsyncope.Heseeshi sint
erni
stsporadical
lyandhasnotf oll
owedt heprevi
ous
recommendat i
ont oseeacar diol
ogist.Hel i
ves50mi lesawayandhasdi f
ficul
tygetting
transpor
tat
ion. 

Hismedicati
onsi ncludevalsart
an80mgdai ly,at
enolol50mgdaily,andf
urosemide,which
wasdisconti
nuedi nt hepast.Hisbloodpressureis130/85mm Hg,hishear
trateis80bpm
andregular
,hiscar diacexaminati
oni snormal,andhehasnosi gnsofheartfai
lur
eorfluid
over
load.Echocardiographyresult
sshow amoder at
elydi
lat
edLV,withanLVEFof30%.You
swit
chhisatenololtoanotherbeta-bl
ocker

Hefai
lstokeephi s1-monthscheduledfol
low-upappointment.Sixmonthslater,hepresents
wit
hincreaseddyspnea.Hisint
erni
stmadef ur
thermedicat
ionchangesandr eferredhim back
t
oyou,st i
llsympt
omatic.Hisdail
ymedicat
ionsarelong-acti
ngmet opr
ololsuccinate100mg,
l
isi
nopri
l20mg, andfurosemide40mg.Hehasgai ned6poundssi ncehispri
orvisit.
 

Hisbloodpressureis120/80mm Hg,hi sheartr


ateis50bpm andregul
ar,hi
sjugularvenous
pressur
eismi l
dlyelevat
ed,andasof tS3 gal
lopi
spresent.Hehast
raceedema.Hislaboratory
resul
tsaresodium 138mEq/ L,potassium 4.
8mEq/ L,bl
oodureani
tr
ogen32mg/ dl,creat
inine
1.2mg/ dl
,andB-typenatri
uret
icpeptide150pg/ml.

Whi
choft
hef
oll
owi
ngi
sthemostappr
opr
iat
enextst
ep?

a. I
ncreasehisli
sinoprildose.
b. I
nit
iateanaldosteronebl ocker
.
c. Ref
erforcardi
act ransplant.
d. I
nit
iatehydral
azineandi sosorbidedi
nit
rat
e.
e. I
ncreasehismet oprololdose.

ANSWER: 

47. d. I
nit
iat
ehydr
alazi
neandi
sosor
bidedi
nit
rat
e.

Thecor rectansweristoaddhydr al
azineand i
sosor bidedinitrat
eandt i
tr
atetothetar
getblood
pressur e.The combi nati
on ofhydr alazi
ne and i sosorbide dinit
ratel owers mort
ali
ty and
decreaseshear tfail
urehospital
izat
ionsi nAfrican-Amer icanpat i
ent swithchronicsystolic
heartf ailur
e( New YorkHear tAssociati
onclassI II-
IV)whoar ealreadyonopt imalmedi cal
therapy.The pat ient’
sr est
ing heartr at
e of50 bpm makes i ncreasing his bet
a-bl
ocker
impr actical
.Analdosterone-r
eceptorblockerrequirescar efulfol
low- up,whichisprobl
emat ic
inapat ientwhomi ssesappointments.Transplantati
onispr ematurei npati
ent
swhohavenot
yetexhaust edmedicalt
herapy. 

48. A74- year-


oldwomanwi t
hnoni schemicdilat
edcardiomyopathyisseen1weekaf ter
hert hi
rd hospi
tal
ization i
n 6 monthsf orprogressi
vewei ghtgain,peripheraledema,and
reduced uri
neout put,each ti
mer api
dl yresponsivetointravenousfurosemi de.Di
schar
ge
medicati
onswer el i
sinopri
l20mgdai ly,carvedi
lol25mgt wicedail
y,spironolact
one25mg
dail
y,andfurosemide120mgt wicedai
ly.
Sincedi
scharge,
shehasgai
ned10pounds.Herdaught
erconf
ir
msshei
scompl
iantwi
thal
lof
hermedicat
ionsandsodi
um r
est
ri
cti
on.

Whi
chofthefol
lowi
nginter
venti
onsismostl
ikel
ytobesuccessf
uli
nrest
ori
ngf
lui
dbal
ance
wi
ththemi
nimum adver
sepotent
ial
?

a. Substi
tutetorsemideforfur
osemi
de.
b. I
ncreasespi ronol
actonedose.
c. I
ncreasefurosemi dedose.
d. I
ncreaselisinopri
ldose.
e. Addme tolazone.

ANSWER: 

48. a. Subst
it
utet
orsemi
def
orf
urosemi
de.

Thecorrectansweri stochanget otorsemide.Thegreat


eroralbioavai
labi
li
tyoftorsemide( e.g.,
approachingi tsi
ntravenousaction)provi
desbenef i
tsoverfurosemide,whichhassi gnif
icant
vari
ableor alabsorpti
on.Alloft
heot herchoicesaremor eli
kelytoproduceunt owardadver se
eff
ects,suchashypokal emia(
e.g.,metol
azone),wit
houtsigni
ficantcl
ini
calbenefi
t,part
icular l
y
si
ncet hepat ient
’sbasel i
nemedi cat
ionsareal r
eadyatthet argetdosesusedi nt heclinical
tr
ial
s. 

49. A 73- year


-oldwomanwi thhyper t
ension,di
abetes,andhear tfai
lurewi t
hpreser ved
ejecti
onf r
acti
onpr esent
st ot heemergencydepar t
mentwi t
ha2- weekhistoryofprogressive
dyspnea on exer tion,wei ghtgai n,and lower extremit
y edema.A di agnost
ic cardiac
catheter
izati
on 1 year ear l
ier demonstrat
ed normalcor onary ar
teri
es.Her out patient
medi cat
ionsi ncl
udel i
sinopril5 mg oncedai l
y,metoprol
olsuccinate100 mg oncedai ly,
furosemide40mgoncedai l
y,andmet for
min500mgt wicedaily.
 

Herphysi
calexaminati
onisnotabl
eforajugularvenouspressureof14cm ofwat er
,bibasi
lar
i
nspir
ator
yrales,
and2+l owerextr
emit
yedemat othekneesbil
ater
ally.HerchestX-rayshows
mil
dcardi
omegal ywit
hpulmonaryvascul
arredist
ri
buti
onandasmal llef
tpleur
aleff
usion.
 

Sheisadmi
tt
edt
othehospi
talser
vicewi
thadi
agnosi
sofacut
edecompensat
edhear
tfai
lur
e
(ADHF)
.

Whi
chofthefol
lowingabnormal
it
iesi
st hemostimport
antpredi
ctorofhospi
talmor
bidi
tyand
mor
tal
it
yinthi
ssituat
ion,
inaddi
ti
ontolowsystol
icbl
oodpressure?

a. Serum creati
nine.
b. Heartr
ate.
c. Oxygensat ur
ation.
d. Serum sodium.
e. Hematocr i
t.

ANSWER: 

49. a. Ser
um cr
eat
ini
ne.
Thecor r
ectansweri sthecr eati
nine.Cl i
nicalinformationobtainedatthet i
meofadmi ssioncan
helpt opr edi
ctout comesdur i
ngt hehospi tali
zationandf ol
lowingdischarge.TheADHERE
(AcuteDecompensat edHear tFailur eNationalRegi str
y)foundt hatthesinglebestpredictorfor
mor tal
itywasanel evatedbl oodur eani t
rogen( ≥43mg/ dl)
,foll
owedbyl owSBP( <115mm Hg) ,
and hi gh ser um cr eatini
ne ( ≥2.75 mg/ dl)
. A subsequentmodelf rom t he OPTI MIZE-HF
(OrganizedPr ogram t oInit
iateLifesavi ngTr eatmenti nHospi t
ali
zedPatientsWi t
hHear tFail
ure)
registr
yal sofoundSBP ( ≤100mm Hg)andser um creati
nine( ≥2.
0mg/ dl)tobepower ful
predictors ofi n-hospitalmor tality. 
Hyper t
ensi on is associated wit
hi mpr oved outcomes.
Anemi aandhyponat remiaar ecommoni npat ientswithadvancedHFandpr edictadverselong
-ter
m out comesandnotshor tter m.Tachycar di
ahasnotbeenshownt opr edi
ctei t
herin-
hospi t
alorpost di
schar gemor t
alityi npatientswi thADHF.  

50. A56- year-oldmanst atus/postanter


iormyocardialinfarcti
onwi t
hanej ect
ionfract
ion
of25% andmi ldrightventri
culardysfuncti
onpresent
sur gentlytocl i
nicwithcomplaintsof
ort
hopnea,par oxysmalnoct urnaldyspnea,andabdomi nalbl oati
ngf or2days.Her ecentl
y
tr
aveledtovisi
tfami l
yforthehol i
daysandadmi t
tedtohavingdi ffi
cult
ymai nt
aini
ngthesodium
andf l
uidrestr
icti
on,althoughhewasadher entwit
hhi smedi cati
ons.Uponr etur
nhome,he
notedthathisweighthadi ncreased10poundsandhi slegswer eswollen. 

Hisout pati
entmedi calregi
meni ncludesenalapril10mgtwi cedail
y,car vedil
ol12. 5mgt wice
daily,spi
ronolactone25mgoncedai l
y,fur
osemi de80mgt wicedaily,
at orvastati
n40mgonce
daily,
andaspi ri
n81mgoncedai ly.Hedoesnotsmokeci garettesordrinkalcohol .Hisphysical
exami nati
onreveal sabloodpr essureof130/80mm Hg,hear trat
eof88bpm,j ugularvenous
pressureof14cm H2O,decr easedbr eat
hsoundsatt hel eftbase,andasof thol osystoli
c
mur muratt hel eftl
owersternalborder.Theliveredgei
spal pablethreef i
nger breadthsbelow
ther i
ghtcostalmar gi
n,andthel owerextremiti
esarewarm wi th1+pi tt
ingedemat ot hemi d
calvesbilat
erall
y. 

Youdeci
detoadmitthi
spat
ientdir
ectl
ytothei
npat
ientcar
diol
ogyser
vicef
ormanagementof
acut
edecompensat
edhear
tfail
ure(ADHF)
.

Whi
ch ofthe fol
lowi
ng woul
d be the saf
estand mostef
fect
ive i
nit
ialor
derf
orvol
ume
management
,inaddit
iont
oa2-li
terf
lui
drestr
ict
ion?

a. Bedsidevenovenousul
traf
il
tr
ati
onat100ml/h.
b. Furosemide80mgi nt
ravenousbol
usevery12hour
s.
c. Furosemide40mgi nt
ravenousbol
usevery12hour
s.
d. Bedsidevenovenousul
traf
il
tr
ati
onat200ml/h.
e. Bume tani
de2mgor all
yevery12hours.

ANSWER: 

50. b. Fur
osemi
de80mgi
ntr
avenousbol
usever
y12hour
s.

Thecor r
ectansweri sl argerdosesofi ntr
avenous( I
V)f ur
osemi de.Thispatientpresents
urgent
lywit
hsympt omsandsi gnsofcongest i
onandnor malsyst emicperfusi
on.Thel i
kely
preci
pit
antt
ot hi
sadmi ssionisnonadherencewiththesodi um andf l
uidrest
ri
cti
on,whichoften
canoccurduringtr
avelawayf rom home.Whenpat ientsareadmi tt
edtot hehospi
talwit
hADHF,
i
niti
ati
ng an eff
ecti
vedi uresi
swi thIVl oop diur
eticsiscr i
ticaltol oweri
ng cardi
acf i
ll
ing
pressur esandr eli
evingcongest i
vesympt oms.Accor dingt otheACCF/ AHA2013Gui deli
nef or
theManagementofHear tFai l
ure, 
ifpati
entsareal r
eadyr eceivi
ngl oopdiureti
cs,t heini
tialIV
doseshoul dequalorexceedt hechr oni
coraldose( f
urosemi de80mg, not40mg) .Bumet anide
i
sa mor epotentl oop di ureticthan furosemide,but2 mg t wicea dayi sequi valentt o
furosemi de80mgt wiceadayandshoul dbegi venintravenously,perguideli
nes.Ther ecent
DOSE( Diur
eti
cOpt i
mi zat
ionSt r
ategiesEvaluat
ioni nAcut eHear tFail
ure)studyshowedno
benefitofacont inuousi nf usion, 
andwhi lepil
otst udiessuggestt hatearl
yul trafi
lt
rat
ioni s
associ atedwithgr eaterfl
uidr emovalt hanmedi caltherapyal one, 
theHeartFailureSoci etyof
Amer icagui del
inesr aiseconcer nsaboutsaf et
y,cost,andneedf orspeciali
zedmedi caland
nursingsuppor t. 

51. An83- year-oldmanwi t


hatri
alf
ibr
il
lat
ionandnoni schemicdi
latedcardi
omyopat hyis
admi t
tedt othehospi talwithacutedecompensatedhear tfai
lur
e(ADHF) .Hi
smedi calhist
ory
i
ncludest ype2di abetes,chronicki
dneydisease,andgout .Hisadmissionbl
oodpr essureis
90/
76mm Hg,andhi shear tratei
s94bpm andi rregul
ar.Bumetani
dei sadmini
steredasa4
mgi ntravenous(IV)bol usevery12hours.

Despi
tehigh-
doseIVdiuret
ics,hisur
ineoutputr
emains<70ml/h,hiswei
ghtdecreases<0.
5kg,
andhisserum creat
ini
neincreasesf r
om 1.4mg/dlto2.1mg/ dloverthefir
st48hour s.I
n
addi
ti
on,shortr
unsofnonsustainedventr
icul
art
achycar
diaar
enot edontel
emetry.

Whi
choft
hef
oll
owi
ngwoul
dbear
easonabl
enextst
epi
nthemanagementoft
hispat
ient
?

a. Bedsidevenovenousul traf
il
trat
ionat200ml /h.
b. Dopami ne2mcg/ kg/minbycont i
nuousinfusion.
c. Mil
rinone0. 25mcg/ kg/minbycont i
nuousinfusion.
d. Ni
troprusside20mcg/ minbycont i
nuousinfusion.
e. Obtainright
- hear
tcatheter
izati
onbeforechangingther
apy.

ANSWER: 

51. b. Dopami
ne2mcg/
kg/
minbycont
inuousi
nfusi
on.

Thecor r
ectansweri stoassesshemodynami cs.Cardior enalsyndr ome, orco- existentcar di
ac
andr enaldysf unct i
on, i
scommoni npat i
ent swi thHF.  
Fur ther mor e,dur inganHF
hospit al
izati
on, 15- 30%ofpat i
entswi lldevelopwor seni ngr enalf unct ion.Thi spat i
enthas
severalindependentr i
skfact orsforthedevel opmentofwor seningr enalf unct ion,including
olderage, di
abet es,andbasel i
ner enaldysf unct i
on.Di uret i
cr esi
st ancemaybecont ri
but i
ngt oo.
Addingal ow- dosei notrope( dopami ne)ori nodi lat
or( mi lr
inone)hasnotbeenshownt o
 
i
mpr oveshor t
-term outcomesandmayexacer batethispat i
ent ’
sar rhyt hmi as. Empi ri
c
admi nistrati
onofni tr
oprussidewi thoutar terialmoni toringmaybedi ffi
cul tgivent hepat i
ent’
s
relat
ivelylowbl oodpr essure.TheCARRESS- HF( Cardior enalRescueSt udyi nAcut e
Decompensat edHear tFail
ur e)tr
ialtestedt hesaf etyandef ficacyoful trafilt
rationasar escue
strat
egyi nthisset t
ingandf oundnocl ini
calbenef i
twi thwor seningr enalf unct ioncompar ed
withst eppedphar macol ogiccar e. 
Whi letherei snor olef orr outinepul monar yar t
er y
cathet er
izationint hemanagementofADHF, theAmer i
canCol l
egeofCar diol
ogy
Foundat ion/Amer i
canHear tAssoci ati
on2013Gui del
inef ortheManagementofHear tFai l
ure
suggest st hathemodynami cmeasur ement smayhel pi nt hechoi ceandt itr
ationofpar enteral
agent s,especi all
yinpat i
entswhoar er ef
ractor ytoinitialther apy, i
ncl udinghi gh-doseor
combi nat i
ondi uretics(Table1) .
52. A71- year-ol
dwomanpr esent
st otheoutpati
entcli
nicwi thexerti
onaldyspneafort he
past6months.Thereisnoorthopneaorchestdiscomfort
.Shei sal i
fet
imenonsmokerandhas
hi
stor
yofhypertensi
onandpar oxysmalatri
alf
ibri
ll
ati
on.Currentmedi cati
onsincludeli
sinopri
l
20mgandat enolol50mg.Herhear trateis55bpm andr egular,herbloodpressureis165/ 90
mm Hg,andherjugularvenouspressureis12cm. 

Cardiopulmonar y exami nat


ion is unr
emar kable,and ther
ei s 1+ pret
ibi
aledema.Chest
radi
ogr aphyisunr emar kabl
e.Echocardiographyshowsnor mallef
tventr
icl
esizeandmass,
witheject i
onfract
ionof66%.Ther eismi l
dl ef
tatri
alenl
argementandmi l
d( Gr
adeI)di
ast
olic
dysfunct i
on,withanest imatedpulmonar yar t
erysystol
icpressur
eof45mm Hg.Mi l
dmitral
i
nsuf f
iciencyandmi ld-
moder at
etri
cuspi
di nsuff
ici
encyarepresent
.

Whi
choft
hef
oll
owi
ngdoyour
ecommend?

a. St
artdigoxi
n0.125mgqd.
b. St
artsil
denafi
l20mgt i
d.
c. I
ncreaseatenololt
o100mgqd.
d. St
artchlort
hali
done25mgqd.
e. St
artbosentan62.5mgbid.

ANSWER: 
52. d. St
artchl
ort
hal
idone25mgqd.

Thecor rectanswerist ostartchlor


thal
idone.Basedont hepatient’shi
stor
y,demogr aphics,
exam, andechocar diogram, t
hemostl i
kelydiagnosisisheartf
ailurewit
hapr eser
vedejecti
on
fr
action( HFpEF).Pulmonar yhypert
ensioniscommonl yseeninHFpEFandi sr el
atedt
o
elevatedleft
-heartfi
ll
ingpressures.Thefir
ststepi str
eatmentoft heunder
lyingcauseoft he
HFpEF, thatis,
thesyst emicelevati
oninbloodpr essure.
 

Inthissituation,t
henextopt i
oni stoaddadi uret
icsuchaschl or t
hali
done.Increasingthe
atenololwoul dnotbeasef fecti
veandherhear tr
atesarelowal ready.Whilestudiesare
underway, thereisnocur r
entevi denceforpulmonary-specif
icvasodilator
sinHFpEF, andther
e
i
spot entialforwor seningleft
-sidedcongest i
onwi t
hisolatedpulmonar yvasodilati
onwi t
h
bosent anorsi l
denaf i
linHFpEF.Di goxinwasnotassoci atedwithi mprovedoutcomesi nan
ancil
laryanal ysi
soft heDI Gtri
al.Digoxinisusefultohelpcontroltheventri
cularresponsein
atri
alfibr
ill
ation,butthepat i
entisi nsi
nusr hyt
hm.  

Whi l
ethereisnopr oventherapyforHFpEF, theAmer icanCollegeofCar di
ology
Foundation/AmericanHear tAssociat
iongui delinerecommendscont rolofbloodpr essureand
volumest at
us(Classl )
,andbot hcanbeef f
ect ivelyaccompl i
shedbyaddi ngchl ort
halidoneto
theregimen.Inananci ll
aryanalysisf
rom theALLHAT( Ant
ihypert
ensiveandLi pid-Lowering
TreatmenttoPreventHear tAttack)tr
ial
,chlorthalidonewasassoci atedwithal oweri nci
dence
ofnew- onsetHFpEF, al
thoughef f
ectsonpr eval entHFpEFmor bi
dit
yandmor tali
tyhavenot
beenassessed. 

53. A50- year


-oldmanpr esent
st otheoutpati
entcli
nicwi
thcomplai
ntsofdyspneaand
fat
iguewi t
h anyact i
vit
y.Hehasa di st
ant20- packperyearsmoking hi
story,and mi l
d
control
ledhypert
ension.Onexami nati
on,hi
shear trateis60bpm andr egular,hi
sbl ood
pressureis105/85mm Hg,andhi sjugul
arpressureis20cm wit
hprominentVwaves.Hi s
hearthasar egul
arrateandr
hythm,wi t
hathi
rdsoundandgr adeIhol
osyst
oli
cmur muratthe
l
eftster
nalborder.
 

Liveredgeispulsat
il
eandpalpabl
eatfourf
ingerbr
eadt
hsbel
owthecost
almargi
n,andt
herei
s
a positi
ve flui
d wave.Lowerext r
emiti
es are coolwit
h 3+ edema.Chestfil
m shows
cardiomegalyandpulmonar
yvenoushypertensi
on. 

Theel
ectr
ocardi
ogram (
Figur
e1)showsnor mall eftvent
ri
cul
ar(
LV)si
zeandf
unct
ion,wi
th
moder
ateconcent
ri
cLVhypertrophyandbi
atr
ialenlar
gement.

Whi
choft
hef
oll
owi
ngwoul
dyour
ecommend?
a. I
nvasi vehemodynami csatrestandwit
hexerci
se.
b. Startli
sinopri
l5mgperday.
c. Coronar yangiography.
d. Cardiopulmonar yexerci
setesti
ng.
e. Ri
ght -heartcatheter
izat
ionwithvent
ri
cularbi
opsy.

ANSWER: 

53. e. Ri
ght
-hear
tcat
het
eri
zat
ionwi
thvent
ri
cul
arbi
opsy.

Thecor r
ectanswerisright-heartbi
opsyf orhi
ghsuspicionofamyl oi
dheartdiseasegivenhi
s
symptoms, age,
andl owvol tageonelectrocar
diogram despi
tehisLVhyper t
rophy.Thi
s
suggestsaninfi
lt
rat
ivedisease.Hehashear tfai
lur
ewithsignif
icantvol
umeover l
oad,
as
evi
dencedbysever ejugulardistent
ion,pulsat
il
ehepatomegaly,ascit
es,edema, andathir
d
heartsound. 

Cardiopul
monar yexercisetest
ingandinvasi
veexer
cisehemodynami csarever
yusefult
o
assessforheartfai
lurewithpreservedej
ecti
onfr
act
ion( HFpEF)whenhist
oryandexam
fi
ndingsareequivocal.Inthispat
ientwhois50yearsold,themostli
kel
ydiagnosi
sisli
ght-
chain(AL)
-typeamyl oid. 

Angi
otensi
n-conver
ti
ngenzymeinhi
bit
orshavenotbeenshowntobeeff
ect
ivei
nHFpEFor
i
nfi
lt
rat
ivecardi
omyopat
hies,
andgivenhisl
owbloodpressur
e,t
hiscoul
dcausesever
e
hypotensi
onorazotemia.Thereisnot
hinginthehi
stor
ytosuggestacti
veischemiaint
he
presentat
ion;t
her
efor
e,coronaryangi
ographywoul
dnotbethefir
sttestofchoice.
 

54. A68- year-ol dwomanpr esentstot heemer gencyr oom wi t


hsever eor thopneaf ort he
past12hour s.Shehasahi storyofhyper tension,ost eoarthr
iti
s,andbor der l
inedi abetes.
Examinationr evealsi r
regularrateandr hythm,wi t
hahear trateof115bpm,bl oodpr essureof
2
190/100 mm Hg,j ugularpr essureof14 cm,and bodymassi ndexof38 kg/ m .Chest
auscult
ationshowsbi l
ateralrales,whilet hepr ecordialexam r eveal sani rr
egul arandr apid
heartr
atewi thdi stanthear tsoundsandnodi scerniblegal l
op.Thepat ienti
sgiveni nt
ravenous
fur
osemide,di lt
iazem,ni tr
oglycerininfusions,and hepar i
n,wi th pr omptimpr ovementi n
symptoms.Hert ransthoracicechocar diogr am showsnor malbi ventri
cularsizeandf unct i
on,
mildmitralregur gitati
on,andmi ldlef
tatrialenlargement .Theel ectrocardi
ogram showsat rial
fi
bri
ll
ati
onandvol t
agecr iteri
aforleftventr
icularhyper trophy. 

Fort
y-ei
ghthour slater
,shei sonor allosar
tan,di
lt
iazem,ri
var
oxaban,andf ur
osemide.Her
heartr
ateisnowi r
regularat70bpm,bloodpressurei
s130/65mm Hg, and venouspressur
eis
normal,butshesti
llcomplainsofmoderatedyspneaonexert
ion.

Whi
choft
hef
oll
owi
ngwoul
dyour
ecommend?

a. Switchdilt
iazem t
oatenol
ol.
b. Transesophagealechocar
diogr
am-guidedcar
diover
sion.
c. Continuouspositi
veair
waypressur
e(CPAP).
d. Holtermoni t
or.
e. Adddr onedarone.

ANSWER: 

54. b. Tr
ansesophagealechocar
diogr
am-
gui
dedcar
diover
sion.

Thisisatypicalpresent ationofacut ehyper tensi vepulmonar yedemai nawomanwi thhear t


fail
urewithpreservedej ect i
onf r
action( HFpEF) .Shepr esentedinr apidatri
alfi
bril
lati
on, which
mayhavepr ecipi
tatedt hedecompensat ion.Thesepat ientstypicallyrespondfairl
yqui cklywit
h
goodcont rolofbloodpr essur eandvent ricularr esponse.Atrialfi
bril
lati
onisextremely
commoni nHFpEF, notedi n30- 40%ofpat i
ents.I ncontrasttohear tfail
urewithreduced
ejecti
onfracti
on(HFr EF) ,therearenot ri
alscompar i
ngr ateversusr hythm contr
oli nHFpEF.
However ,becauset herei sthepot entialforincr easedrel i
anceonat ri
alsystol
eintheset tingof
diastol
icdysfunction,theACC/ AHAgui deliner ecommendsst rongconsi derat
ionof
cardiover
sionforat ri
alfibril
lati
on( Classl l
b,LevelofEvi denceC) .

Becauset hisisthefir
stepisodei nthispati
ent,t
hel i
kel i
hoodt ohaveadur abl erest orat i
onof
sinusrhythm isenhanced.Ther eisnoevi dencet osuppor tatenol oloverdil
tiazem orl isinopril
overlosartani nHFpEF,asnot reatmentshavebeenpr ospect ivelyshownt obeef fecti
vei nl arge
randomizedt rial
s.Thepat i
ent’
sbodyhabi t
usandat r
ialfibr i
ll
ationincreaset her iskofsl eep-
disorder
edbr eathing,andthisshouldbeeval uated,butsheshoul dnotbest artedonCPAPi n
theabsenceofadi agnosisofsleepapnea.AHol termoni torcanbeusef ultodet ermi ne
adequater atecontrol
,butthispatientissymptomat icdespi t
er at econtr
ol.Ther efor e,an
attemptatr estor
ingsinusrhythm isappr opr
iat
e.Dr onedar onei snotindicatedi npat i
ent swi th
decompensat edheartfai
lurewithinthepast4weeks, andi sunl ikel
ytocar diover tthispat ient. 
55. A25- year
-oldcoll
egestudentwasplayi
ngwit
hhisyoungnephew,whohadafeverand
aner yt
hematousrash.Subsequenttot hatvi
sit
,hedevel
opsdyspnea,rash,andf
ever.An
echocardi
ogr
am reveal
edhisej
ectionf
racti
ontobe25%.

Whi
choft
hef
oll
owi
ngi
sthemostl
ikel
ycauseofmyocar
dit
isi
nthi
spat
ient
?

a. Mumps .
b. Al
cohol.
c. Chagasdisease.
d. Parvovi
rusB19.
e. Adenovir
us.

ANSWER: 

55. d. Par
vovi
rusB19.

Themostcommonvi ralet
iol
ogyofmyocarditi
satpresenti
sparvovi
rus.Rashesar
ecommon
withal
ltypesofvi
ruses,andaresuggesti
veofavirus-r
elat
edmyocarditi
sinthi
sscenari
o.
Adenovir
usandmumpsmaycausemyocar di
ti
s,butarerar
eeti
ologi
es.Thereisnoreduvi
id-
organi
sm exposur
et osuggestChagas,whichisendemictoCentr
alandSouthAmer i
ca. 

56. A55- year-


oldtruckdri
verwhoseonlymedicalhi
stor
ywasunt reatedhyper
tensi
onwas
di
agnosedwi t
hdilat
edcar di
omyopathyandNew Yor kHeartAssoci
ationclassII
Isymptoms.
Hisendomyocardi
albiopsyisposi
ti
veforl
ymphocyti
cmyocar di
ti
s.

I
naddi
ti
ont
obet
a-bl
ocker
sanddi
uret
ics,
whi
choft
hef
oll
owi
ngi
sappr
opr
iat
einhi
scar
e?

a. Prednisone.
b. Angiotensin- conver
ti
ngenzymei
nhi
bit
or. 
c. Betainterferon.
d. Spir
onol actone.
e. Me t
hot r
exate.

ANSWER: 

56. b. Angi
otensi
n-conver
ti
ngenzymei
nhi
bit
or. 

Themainstayoft reatmentformyocardit
isisst
andardheartf
ail
uret
herapy.Diur
eti
ctherapy
al
oneisinsuff
icient.Thereisnoevidencethatr
outi
neimmunosuppressionisofbenefi
t.Ther
e
aresomedatat hatpat i
entswithchroni
clymphocyti
cmyocardit
iswit
houtevidenceofaviral
et
iol
ogymaybenef i
tfr
om steroi
dtherapy,butt
hisi
snotfir
st-
li
netherapy.
 

57. I
nwhi
choft
hef
oll
owi
ngcl
ini
calscenar
iosshoul
danendocar
dialbi
opsybeper
for
med?

a. A20-
year
-ol
dwomanwi
thacut
ehear
tfai
lur
e,ej
ect
ionf
ract
ion(
EF)of25%,andt
hir
d-
degr
eeat
ri
ovent
ri
cul
arbl
ock.
b. A 65-
year
-ol
d man wi
th cr
ushi
ng chest pai
n,or
thopnea,l
eg edema,and an
el
ect
rocar
diogr
am showi
ngaQRSdur
ati
onof150msec.
c. A65-
year
-ol
dmanwi
that
ypi
calchestpai
nanddel
ayedgadol
ini
um enhancementi
n
t
helater
alwall
s.
d. A35-year-
oldmanwhocomplai
nsofacut
epl
eur
it
icchestpai
nwi
thanor
malej
ect
ion
f
ract
iononechocar
diogr
am.

ANSWER: 

57. a. A20-year
-ol
dwomanwi thacutehear
tfai
lur
e,ej
ect
ionf
ract
ion(
EF)of25%,
andt
hird-
degreeat
ri
ovent
ri
cularbl
ock.

Thecor rectansweri sthepatientwi thacutehear tfail


ure, at
ri
oventr
icularblock,andr educed
l
eftventricularEF.I
tiscrit
icaltoident i
fycausesofl i
fe-threat
eningmyocar diti
swhosecour se
couldbeal teredbyearlyinterventi
on, suchasgi antcel lmyocar di
ti
soreosi nophil
ic
myocar diti
s.Thepat i
entpresent i
ngt otheemer gencydepar t
mentwi thcrushingchestpai n,
orthopnea, etc.
,hascoronarydi seaseunt i
lprovenot her wise.Thepat i
entcompl ai
ningofacut e
pleuri
ti
cchestpai nwi t
hanor malEFonechocar diogr am hasnoevi denceofmyocar dial
dysfunction.Thepat i
entwithat ypicalchestpainanddel ayedgadolini
um enhancementi nthe
l
at er
alwal l
shasevi denceofcor onar ydisease.
 

Endomyocardi
albi
opsyisaClassIrecommendat ioninpatientswhodevelopnew-onsethear
t
fai
lur
eandheartbl
ock.Inthi
ssit
uati
on,i
nfil
trat
ivedisorder
si ncl
udi
ngsarcoidneedtobe
consider
ed,
andthediagnosi
sisbestestabl
ishedbybi opsy.

58. A43- year


-ol
dwomanwi tha6-yearhist
oryofnonischemicdilat
edcardi
omyopat
hy,
paroxysmalatri
alfi
bri
ll
ati
on,andanimplant
abledefi
bri
ll
atorisref
erredtothehear
tfail
ure
cl
inic.Shehashadprogressi
vesympt
omsdespiteopt
imalmedicalt
herapy.

I
nr evi
ewinghermedi calhist
ory,whi
choft
hef
oll
owi
ngwoul
dbeassoci
atedwi
thawor
se
pr
ognosisforherhear
tfail
ure?

a. Sinusbr adycardi
a.
b. Lowc holest
erol.
c. Femal esex.
d. Obe si
ty.
e. Hy pouricemia.

ANSWER: 

58. b. Lowchol
est
erol
.

Lowercholester
olhasbeenl i
nkedt oworseprognosisi
nhear tfai
lurepati
ent
sandi sli
kely
rel
atedtopoornutri
ti
onalstatus(cardi
accachexia)andelevatedcircul
ati
nginfl
ammat or
y
cyt
okines.I
ngeneral,pat
ientsrequir
inghigherdosesofloopdi ur
eti
cshavewor seoutcomes,
sothefactthatt
hecasepat i
entdoesnotr equir
eloopdiureti
cspor t
endsabet t
erprognosi
s. 

The“ obesi
typaradox”suggest
sthatobesepati
entshavebett
eroutcomescomparedwit
hthei
r
thincounter
part
s.Hyperuri
cemiahasbeenassociatedwithworseoutcomes,
ashasrest
ing
sinustachycar
dia.Womenwi thheartf
ail
urehaveabetterpr
ognosisthanmen.
59. An 86- year-old woman wi th noni schemic cardiomyopat hy and type 2 diabet
es
complicatedbychr onickidneydisease(creat i
nineclear
ance22ml /min)i
shospi t
ali
zedwith
worsening cr eati
nine,decr eased appeti
te/ f
ail
uret ot hrive,cooll ower extr
emiti
es,and
confusionbyf ami l
yr epor
t.Herelect
rocar
di ogram showedar i
ghtbundlebranchblock(RBBB)
wit
haQRSdur ati
onof136msec.Homemedi cati
onsincludel i
sinopr
il20mgdai l
y,carvedi
lol
12.5mgbi d,
andspi ronolact
one25mgdai ly. 

ASwan- Ganzcatheterizat
ionisperfor
med,whi chreveal
sar i
ghtatr
ialpressureof24mm Hg,
pulmonarycapi
ll
ar ywedgepr essure(PCWP)of34mm Hg,andcar diacindexof1. 4L/min/m2.
Milr
inoneisi
nstit
utedwi thadecr easeofPCWPt o22mm Hgandani mpr ovementincardi
ac
2
i
ndext o1.9L/mi n/m .Ment at
ioni mprovesandshei seati
ngandabl et ointer
actwithher
famil
y.However,wi t
hin24hour sofweani ngtheinotr
ope,thepat
ientisnot edtobeconfused
andhercreat
ini
nei ncreasedfurt
her.

Whi
choft
hef
oll
owi
ngwoul
dbet
henextst
epf
ort
hispat
ient
?

a. I
nit
iationofanangi ot
ensin-receptorblocker(
ARB).
b. I
nit
iationofdialysis.
c. Continuousinfusionofinot r
ope.
d. Useofi nt
ermittentintr
avenous( IV)inotr
ope.
e. I
mpl antat
ionofacar di
acr esynchronizati
onther
apy(CRT)
.

ANSWER

59. c. Cont
inuousi
nfusi
onofi
not
rope.

Thedecisi
ontousechronici
nfusionsofinot
ropi
ct her
apyiscli
nicalandmustt
akeinto
accountpat
ientpr
efer
encesandcost .Themajorreasontoini
ti
atetherapyi
spal
li
ati
on.
Mortal
it
yisveryhi
ghinthispati
entcohort.
 

Implant
ationofaCRThasnotbeenr igorousl yst
udi edininotrope-dependentpat ientsandis
notindi
cat edinpatient
swi t
hanRBBBandQRSdur ati
onof<150msec.Useofi ntermitt
entIV
i
notropicagent sisaClassI IIrecommendat ion;therear enodoubl e-bl
indedcl i
nicaltri
alst
o
supportthepr act
ice.Dialysi
sshoul donl ybeconsi deredunderext enuatingcircumst ances
duri
ngt er
mi nalcare.Dialysiswi l
lnotchanget henat uralhi
storyofhear tfai
lur
ei nthissett
ing.
Theaddi t
ionofanARBwhent hepat ientisal r
eadyonanangi otensi
n-conver t
ingenzyme
i
nhibit
orandspi r
onolactonei saCl assIIIrecommendat i
on.
 

60. Accor di
ngt otheFr amingham Hear
tSt
udy,t
heli
feti
mer i
skatage40year
sfort
he
devel
opmentofheartf
ail
ureforbothmenandwomeniswhichoft
hefol
lowi
ng?

a. 1i
n5.
b. 1i
n2.
c. 1i
n20.
d. 1i
n15.
e. 1i
n10.

ANSWER: 

60. a. 1i
n5.
Accor
dingt
otheFramingham Hear
tSt
udy,
thel
if
eti
mer
iskatt
heageof40year
sfort
he
devel
opmentofhear
tfai
lur
eforbot
hmenandwomenis1in5.

61. A67- year


-oldmanwi t
hatri
alfi
bri
ll
ati
onandnoni schemicdil
atedcardi
omyopat
hyi s
admitt
edtot
hehospitalwit
hacutedecompensatedheartfail
ure(ADHF).Hi
smedicalhi
story
i
ncludest
ype2diabetesandchroni
ckidneydisease.Hi
sadmi ssi
onbloodpressur
eis82/ 66
mm Hgandhisheartr
ateis94bpm andir
regul
ar. 

A uri
nar ycatheterispl acedf oraccur ateassessmentoffluidbalance,andbumet ani
dei s
administer
edasa4mgi ntr
avenous( IV)bol
usevery12hours.Despitehigh-doseIVdiur
etics,
hi
sur ineout putr emains<70mlperhour ,hisweightdecreases<0.5kg,andhi sserum
creat
inineincreasesfr
om 1. 4mg/ dlto2.1mg/dloverthef
ir
st48hour s.Inaddit
ion,
shortruns
ofnonsustainedventri
cul artachycardiaarenot
edont el
emetr
y.

Whi
choft
hef
oll
owi
ngwoul
dbear
easonabl
enextst
epi
nthemanagementoft
hispat
ient
?

a. Mil
rinone0.25mcg/ kg/minbycont i
nuousinfusion.
b. Sendbloodf orB-t
ypenat r
iuret
icpepti
demeasur ement .
c. Pulmonaryar t
erycathet
erforassessmentofcent ralhemodynami
cs.
d. Bedsidevenovenousul t
raf
ilt
rat
ionat200mlperhour .
e. Nit
roprussi
de20mcg/ minbycont i
nuousinfusion.

ANSWER: 

61. c. Pul
monar
yar
ter
ycat
het
erf
orassessmentofcent
ralhemodynami
cs.

Cardiorenalsyndrome,orco- exi
stentcardi
acandr enaldysf
uncti
on,iscommoni npatient
s
withHF.Fur ther
more,duri
ngaHFhospi tal
izat
ion,15-30%ofpat i
entswil
ldevelopwor seni
ng
renalfuncti
on.Thispati
enthassever alindependentr i
skfactor
sforthedevelopmentof
worseni ngrenalf
uncti
on,includi
ngol derage,diabetes,andbaseli
nerenaldysfuncti
on.Diuret
ic
resi
stancemaybecont ri
butingaswel l,andthenextst epinmanagementi scri
tical
,but
empiric. 

Addingalow-doseinot
ropeorinodil
atorhasnotbeenshownt oimproveshort
-term out
comes,
andmayexacer bat
ethi
spati
ent'sarrhyt
hmi as.Empiri
cadmi ni
str
ati
onofnit
roprussi
dewi t
hout
art
erialmoni
tor
ingmaybediff
icultgi
vent hepatient
'srel
ati
velyl
owbl oodpr
essure.

Thesafetyandeffi
cacyoful
traf
il
tr
ati
onasar escuestrat
egyi
nt hi
ssetti
nghasnotbeent est
ed,
andcli
nicalr
eportsrai
seconcernsaboutf
urtherworseni
ngofrenalfunct
ion.Inaddi
ti
on,the
CARRESS- HFtri
alshowednobenefitoful
tr
afil
tr
ati
oninpati
ent
swi thcardior
enalsyndr
ome
duetoADHF.  

Whi l
ether
ei snor oleforrouti
nepulmonaryar t
erycatheter
izati
oninthemanagementofADHF,
the2013HFgui delinegivesaClassII
arecommmendat i
onf ori
nvasi
vehemodynamic
moni t
ori
nginpat i
entswi t
hacuteHFwi t
hper si
stentsymptomsdespi teempi
ri
cadjust
mentof
standardtherapieswi t
hwor seni
ngrenalfuncti
on.

62. A53- year


-ol
dmanwi t
hischemiccar
diomyopat
hyandaleftventr
icul
ar 
eject
ionfract
ion
(EF)of26% i
shospit
ali
zed wi
th acut
edecompensated hear
tf ai
lur
e( HF),hist hi
rd such
admi
ssi
oni
nthel
ast6mont
hs.
 

Medicat
ionsi
ncl
udeaspir
in81mgqd,warf
ari
n,furosemide40mgqd,car
vedi
lol25mgbi
d,
l
isi
nopri
l20mgqd,spi
ronol
act
one25mgqd,andatorvast
ati
n40mgqd.
 

Hehadadual
-chamberi
mpl
ant
abl
ecar
diover
ter
-def
ibr
il
lat
or 
(I
CD)i
mpl
ant
ed18mont
hsago.
 

Onphysi
calexaminati
on,hi
sheartrateis 
75bpm and  ir
regular.Hisbloodpressur
eis 110/75
mm Hg.Jugul
arvenouspressur
eis 15cm H20. 
Ther earecracklesovertwo-
thirdsofbothlung
f
iel
ds.Hehasa+S3  gal
lopand 
grade3/ 6holosystoli
cmur muratt heapex.Abdomenshows
mil
dhepatomegal
y. 
Hehas 3+bil
aterall
owerextremityedema.  

Laborat
oryresul
ts 
i
nclude:troponi
n I0. 54 ng/
ml,B-type natr
iur
eti
c pept
ide 1523 pg/
ml,
creat
ini
ne 1.
8 mg/dl,and inter
nati
onalnor mali
zed r
atio 2.
3.Compl et
e blood cel
lcount
shows hemogl
obi
nof7. 0 
g/dl

Recent 
glycat
edhemoglobin 
is7.3%. 

Hi
select
rocar
diogr
am (
ECG)shows at
ri
alf
ibr
il
lat
ionwi
thhear
trateof75bpm,and 
l
eftbundl
e
br
anchblockwit
hQRSdurat
ionof165msec(nochangefr
om pri
orECG).

Af
terdi
uresi
s,whi
choft
hef
oll
owi
ngi
sthenextbestst
epi
nhi
smanagement
?

a. Er
ythropoi
etin.
b. Ref
erforventri
cularassistdevi
ce.
c. Per
cutaneousmi t
ralval
verepair.
d. Addangiotensi
n- r
eceptorblocker.
e. Upgradetoabi ventri
cul
arICD.

ANSWER: 

62. e. Upgr
adet
oabi
vent
ri
cul
arI
CD.

AnacuteHFadmi ssionof fersanimportantopportunityt


oadjustoralmedi cati
ons,toaddress
probl
emsassoci atedwi thr educedEFsuchasar rhythmiaandthromboembol icri
sk,andto
managecomor biditi
esi ncludingdiabet
es,anemia, andchroni
ckidneydi sease.Thispati
entis
onguidel
ine-di
rectedmedi caltherapy,
incl
udingbet a-bl
ocker
,angiotensin-conver
tingenzyme
(ACE)i
nhibi
tor,
andal dactone.However ,hi
stherapyi snotopt
imizedbecausehehasaCl assI
i
ndicat
ionforbiventri
cul arpacing.

Mit
ralval
verepai
risaClassII
bindicat
ionformi
tr
alr
egurgi
tat
ionaft
eral
lothert
herapi
esar
e
i
mplemented,i
ncl
udingbivent
ri
cularpaci
ng,whi
chcanreducethesever
it
yofmitral
r
egurgit
ati
on.

HeisonACEinhibi
torsandaldactone.Thus,theadditi
onofanangiotensi
n-receptorblocker
wouldputhi
m atsignif
icantr
iskofhyperkal
emia.Alt
houghr epeat
edhospit
alizat
ionssuggest
hehasadvancedHF, youwoul dupgradetoabiventr
iculardevi
cepri
ortoconsiderati
onofmor e
advancedt
herapi
essuchasvent r
icul
arassi
stdeviceort r
anspl
ant.
 

Whil
eanemiaisassociatedwithworseout
comesi
nHFpat
ient
s,cl
ini
calt
ri
aldat
ahavenot
shownbenef
itf
rom erythropoi
eti
nther
apy.
 
63. A35- year-ol
dmanwi thanonischemi
ccardiomyopathyof10yearsdurat
ionwit
hal eft
ventri
culareject
ionfract
ion(
LVEF)of15% wasreferr
edtoyourcar di
actranspl
antcent
erf or
progressivesymptomsofhear tf
ail
ure.Aspar
tofthisevaluat
ion,heunderwentari
ght-heart
catheter
izati
on. 

Inthecat heter
izati
onlaborat
ory,hi
sini
tialhemodynamicsincludedar i
ghtatr
ialpressur eof  
15
mm Hgandpul monarycapill
arywedgepr essure(PCWP)of 35mm Hg.Thecar diologistwas
carefultodef l
atetheballoonfoll
owingmeasur ementofthePCWP.Addi ti
onalmeasur ement s
2
i
ncludedacar diacindexbyt her
modiluti
onof2. 4L/min/m  andaFi ckcardiacindexof1. 5
L/min/m2.Oxygen consumpt ion wasmeasur ed f
ortheFickcar diacindex.Syst oli
cbl ood
pressurewas80mm Hgandsyst emicvascul arr
esi
stancewas1800dyne- sec-cm5.Bl ood
obtainedf r
om t heri
ghtatri
um hadasat urati
onof48%,andf rom thepulmonar yartery( PA),a
saturati
onof35%.  

Thepat ientwasstart
edonsodium ni
tr
oprussideat0.5mcg/kg/
minandtransf
erredbackt o
theintensivecar
eunit(I
CU),wher
eanothersampleofPAbloodwasobtai
ned1houraf terthe
i
niti
almeasur ement,andfoundtohaveasat urat
ionof95%.Thepat
ient
’scli
nicalcondit
ion
remainedunchanged.

Whi
choft
hef
oll
owi
ngi
sthemostl
ikel
yexpl
anat
ionf
ort
hechangei
nPAsat
urat
ion?

a. Theri
ghtheartcat
hetercreatedaventri
cul
arseptaldef
ect.
b. Thei
niti
alPAsaturat
ionwaser r
oneousl
ylow.
c. Thepati
enthascyanidetoxicit
y.
d. ThesampleofPAbl oodint heI CUwaserroneouslyobt
ainedf
rom t
hepr
oxi
malpor
tof
t
her
ighthear
tcat
het
er.
e. Ther
ighthear
tcat
het
ermi
grat
eddi
stal
lydur
ingt
ranspor
tbackt
otheI
CU.

ANSWER: 

63. e. Ther
ighthear
tcat
het
ermi
grat
eddi
stal
lydur
ingt
ranspor
tbackt
otheI
CU.

Itwouldbeunl i
kelyforthepatienttodevel opsodi um ni
tr
opr ussi
detoxicit
yorani ntr
acar di
ac
shuntwithin1hour .Dependingonwher ebloodi ssampledint heri
ghtat r
ium,it
ssat urati
on
canbehi gherthanthePA.Typi cal
ly,infer
iorvenacavabl oodhasahi gheroxygensat urati
on
thansuperiorvenacavaorcor onarysi nusblood.Evenifbloodwer esampl edfrom thepr oxi
mal
port,i
twouldbeunl ikel
yforrightatrialorsuper i
orvenacavabl oodtohaveasat urati
onashi gh
as95%.Ri ghtheartcatheter
scanmi gratedistal
lyduri
ngt r
ansportintothewedgeposi ti
on
evenwi t
hthebal l
oondef l
ated.Ifthisoccur s,aswasl i
kel
yi nthiscase,thebloodsampl ewill
refl
ectpulmonaryvenousO2  sat ur
ation.

64. A 56- year-


old woman wi th a hi
storyofbr eastcancerwho is curr
entl
yreceivi
ng
doxorubicinchemot her apyisref
erredtothecar di
ologycli
nicbyheroncol
ogistf
orchangesin
l
eftventricul
arejectionf ract
ion(LVEF)not edbyser i
almulti
gatedacqui
sit
ion(MUGA)scans.
Thepatientisduet or eceiveaddi
tionaldosesofchemot her
apy,pendi
nghercardi
aceval
uation.
Herbasel ineLVEFpr iortodoxorubicintherapywas64%,andhermostr ecentLVEFaftera
cumulativedoxorubicindoseof400mg/ m2 isnow58%.  

Thepatientappearswell
,isinnodist
ress,
anddeni
escomplai
ntsofexer
ti
onalorrestdyspnea,
chestpain,noct
urnaldyspnea,orl
owerextr
emi
tyswel
li
ng.Onexaminati
on,herbl
oodpr essur
e
i
s122/ 70mm Hg,herpul
seis88bpm andr egul
ar,andherjugul
arvenouspressurei
s5cm
H2O.Cardi
acandpulmonaryexaminat
ionsarenormal.A12- l
eadelect
rocar
diogram shows
normalsi
nusr
hythm wi
thnonspeci
fi
cSTandTabnor mali
ti
es.

Whi
choft
hef
oll
owi
ngwoul
dyour
ecommend?

a. Troponinlevel.
b. Disconti
nuedoxorubici
n.
c. Serum B-typenatr
iuret
icpept
ide.
d. Echocardiogram.
e. Continuedoxorubi
cin.

ANSWER: 

64. e. Cont
inuedoxor
ubi
cin.

Chemot herapy- i
nducedcar diomyopat hyhasbeenvar iablydef i
ned, butadefinit
iont hatisoften
referencedi sonedevel opedbyt hecardiacr eviewandeval uationcommi t
teeover seeing
trastuzumabt ri
als.Thecommi tt
eedef inedcar diacdysf uncti
oni nt
heset t
ingofchemot herapy
useasanyoneoft hefollowing:1)car diomyopat hychar act
erizedbyadecr easei ncar di
ac
LVEFt hatwasei therglobalormor esevereint hesept um;2)sympt omsofcongest ivehear t
fail
ur e(CHF) ;3)associat
edsi gnsofCHF, includingbutnotl i
mi tedtoS3 gall
op, t
achycar dia,or
bot h;and4)decl ineinLVEFof≤5- 55%wi thaccompanyi ngsignsorsympt omsofCHFora
decl i
nei nLVEFof≤10- 55%wi t
houtaccompanyi ngsignsorsympt oms.However ,recent
evidencesuggest sthatchangesi nlongitudinalstrai
nmaybemor esensiti
vet oearly
myocar dialdamage.  

Althoughther estingEFislowert hanpr iorEF,i


tisstilli
nt henormalrangeandt hepat i
entby
histor
yandcl inicalexaminationhasnosi gnsofhear tfailur
e.Therearenocl eargui del
inesas
touseofbi omar kerstoguidemanagementi nasympt omat icpati
entswithnor malEF
undergoingchemot herapy,andunl esst heMUGAscanwasofquest i
onablequal i
ty,an
echocardiogram woul dnotbei ndicated.Basedont heFoodandDr ugAdmi nistr
ation-approved
l
abel i
ngfordoxor ubici
n,anEFdr opof≥10%t oanabsol utelevel<50%shoul dprompt
disconti
nuationofchemot herapyandconsi derat
ionfori nit
iat
ionofangiotensin-converti
ng
2
enzymei nhibit
or sorbeta-blockers.Afteracumul ati
vedoseof500mg/ m, repeatevaluati
on
shouldbeconsi der edforeveryaddi t
ional50mg/ m2 ofdoxor ubi
cinadministered.

65. A63- year-


oldCaucasi anwomani sevaluatedinthehear tf
ai l
urecli
nicf
orconsi
deration
ofadvanced t her
apies,including possi
blehear ttransplantat
ion.Shehasal ong-standing
hist
oryofi schemiccar di
omyopat hywi thal eftventri
cularejectionf r
acti
onof15% despi t
e
ther
apywi thbeta-bl
ockers,angiot
ensin-converti
ngenzymei nhi
bitors,al
doster
oneantagonists,
anddi uret
ics.Shei saf or
mersmokerwi tha60pack- yearhistoryofsmoki ng,andhas
hypert
ensionanddi et-control
leddiabetes.Shehadabi ventri
cularimplantabl
ecardiovert
er-
defi
bri
llat
orplaced2year sago. 

Herpr imary cardiol


ogistwas concerned aboutherwor sening f
uncti
onalstatus,as she
becomesdyspnei cwi t
hmi ni
malact i
vit
y.Sheper f
ormed" poorl
y"ona6- mi
nutewal ktest
,
pri
mar i
lylimited by dyspnea.A car di
opulmonar y exer
cise testreveal
s peak VO2 of9.9
ml/kg/min.Sheal sohadar i
ght-hear
tcatheter
izat
ion,whichdemonst rat
edacardiacindex(CI
)
of1.8.Shewaspl acedoni nt
ravenousmilri
noneandherCIi mpr oved.
Whi
choft
hef
oll
owi
ngi
sthenextbestst
epi
nhercar
e?

a. Referfortr
ansplant
.
b. Ri
ghthear tcat
heteri
zation.
c. Continuemedicaltherapy.
d. Destinat
ionlef
tventri
cularassi
stdevi
ce.

ANSWER: 

65. a. Ref
erf
ort
ranspl
ant
.

Pati
ent
sgenerallyareconsideredcandidatesforhearttransplantationwhent hei
rpeakVO2 i
s
<14ml/kg/
min( or<12ml /kg/minonbet a-bl
ockers).Shehaspr ogressivesymptomsdespite
opt
imalmedicaltherapy.Shemaybeacandi dat
ef ordestinati
onl eftvent
ricul
arassi
stdevi
ceif
sheisdeemednott obeat ranplantcandidat
e,butsheshoul dunder goaf ullwor
kupto
det
erminebestavail
abletherapy.Ar epeatri
ght-heartcatheter
izationwi l
lnotaddtohercare.
 

66. A50- year -


ol dmani sr eferr
edt oyourcl i
nicf orevaluat
ionofexertionaldyspnea.Over
thepast3mont hs, hehasnot edi ncreasingshortnessofbr eathduringdailyacti
vit
ies,suchas
walking shortdistancesorcar ryi
ng lightbundles.Hehasno pr iorhistoryofmyocar dial
i
nfarcti
onanddeni eschestpai n.Hi spastmedi calhistoryisnotableonlyforhypertensionand
hypercholest
erolemi a.Hisonl ymedi cat
ionsareat enolol25mgoncedai l
yandsi mvast at
in20
mgoncedai l
y.Hedr inks t
wo gl assesofwi newithdi nnereachni ghtanddeni esuseoftobacco
oril
li
citdrugs.Hisfami l
yhistoryi snotableforapat ernalunclewhodi edsuddenlyinhis50s. 

Hisexaminationisnotablef orbloodpr essure170/ 80mm Hg,pul se80bpm,mi ldjugular


venousdist
ent i
onat12cm H2O,cl earlungst oauscul t
at i
on,andmi l
dper ipher
aledema.S1 is
normal,
 
S2 i
spar adoxi
cal
lyspl it
,andt her eisasof t S3 gall
opwithagr ade2/ 6holosyst
olic
murmuratt heapex.Hiselectrocar
diogram reveal
ssi nusr hythm,wit
hl eftbundl
ebranchblock
(LBBB)andaQRSdur ati
onof150msec.Echocar diogr am reveal
sadi latedlef
tventr
icl
ewi th
eccent
ri
chyper tr
ophy,
anteriorhypokinesis,andej
ectionf r
acti
on(EF)25%.

Whi
choft
hef
oll
owi
ngf
act
orsi
sthemostl
ikel
yet
iol
ogyofhi
scar
diomyopat
hy?

a. Alcoholuse.
b. Fami li
aldil
atedcardi
omyopat
hy.
c. Cor onaryart
erydisease(
CAD).
d. Hy pertensi
on.
e. Viralmyocar di
ti
s.

ANSWER: 

66. c. Cor
onar
yar
ter
ydi
sease(
CAD)
.

Thispatientpresentswi thnew- onsetheartfai


lure(HF)inthesetti
ngoflef
tventr
icular(
LV)
cavit
ydi l
ati
onandr educedEF.Ther egionalwallmotionabnormalit
yandLBBBi nt hepresence
ofcardiacriskfact
or s,includi
nghyper t
ensionandhyper li
pidemia,makeCADt hemostl i
kely
eti
ology.Hypertensioni tself
,ifuncontroll
ed,mayl eadtothedevelopmentofLVhyper tr
ophy
andHF, wi
theitherpr eservedorr educedEF.Thef amilyhi
storyofsuddendeathhintsata
potent
ialgeneti
ccontr
ibutortohisi
ll
ness,butgi
ventheri
skf act
orsf
orCAD,thi
sismoreli
kely
acauseofsuddencar diacdeath.Myocardit
isi
snotsuggestedspeci
fi
cal
lybythehi
stor
y,buta
typi
calvir
alprodromeisfrequent
lyabsent.Modestal
coholconsumpti
onisunli
kel
ytobe
associ
atedwi t
hthedevelopmentofcardiomyopathy.

67. A55- year-


oldmanwi t
hahistoryofaleftanteri
ordescendinginf
arcti
on( l
eftventr
icul
ar
eject
ionfr
acti
on[LVEF]of15%,andLVend- diast
olicdimensionof7. 8cm)i nt hesetti
ngof
cocaineuse12yearspriort
opresentat
ionisadmi t
t edwithincreasi
ngshor t
nessofbr eathin
thesett
ingofnoncompli
ancewit
hdietaryrecommendat ions.Hisbloodpressureonadmi ssion
i
s90/ 50mm Hg. 

HisB- t
ypenat r
iuret
icpepti
deis2235pg/ mlandserum sodi
um is128mEq/L.Hemoglobi
nis
7.9 g/dl
.He was di ur
esed and sodi
um nor mali
zed t
o 136 mEq/L.He did nottol
erat
e
angiot
ensin-convert
ing enzyme (ACE)inhi
bit
ors due t
o symptoms ofdi
zziness.He was
ther
eforedischargedoncarvedi
lol3.
125mgor al
lybidandfur
osemide80mgorall
ybid.

Whi
choft
hef
oll
owi
ngi
sassoci
atedwi
thawor
sepr
ognosi
sfort
hispat
ient
?

a. Fur
osemidedose>60mgbi d.
b. Ser
um sodium <140mEq/L.
c. Hypot
ension.
d. Anemia.
e. Lef
tvent
ricul
ardiast
oli
cdi
mensi
on>7.
0cm.

ANSWER: 

67. c. Hypot
ensi
on.

Inpati
entswi t
hhear tfailurewi t
hreducedEF, t
her ear eanumberofadver seprognosti
c
i
ndicator
s, andthispatienthasmanyoft hem,includinghyponat remia,anemia,diur
eticdose,
anddilat
edLV.However ,noneoft hem isassociatedwi thashi ghamor tali
tyrat
east hatseen
i
npat i
entswhocannott olerateACEinhibit
orsduet oeitherhypotensi
onorr enalf
ail
ure
(ci
rcul
atoryrenalli
mi t
ation) .Inanobservati
onalst udy,patient
swi thadvancedheartfailure
whoar edischargedoffACEi nhi
bit
orsduet ocirculatoryrenalli
mitati
onhaveamor tal
ityrateof
57%at8. 5mont hs.

68. A55- year


-ol
dtruckdr i
verwhoseonl ymedicalhistor
yi ncl
udesunt r
eatedhypertensi
on
presentswit
h3weeksofNewYor kHeartAssociati
onclassIIIheartfai
lur
e( HF)symptoms.An
echocardi
ogram demonstratesl eftventr
iculardil
atati
on with an eject
ion fract
ion of26%.
Cardiaccat
heteri
zat
ionisnegativeforobstructi
vecoronaryarterydi
sease.

I
naddi
ti
ont
otr
eat
mentofhi
sHF,
whi
choft
hef
oll
owi
ngi
sthenextbestst
epi
nhi
scar
e?

a. Continuecurrenttherapy.
b. Leftventr
icul
arassi stdevice.
c. Endoc ar
dialbiopsy.
d. Implantablecardiovert
er-defi
bri
ll
ator(
ICD)
.
e. Prednisone.
ANSWER: 

68. a. Cont
inuecur
rentt
her
apy.

Themai nstayoftr
eatmentf ormyocardit
isisst
andar
dHFt herapy.I
mmunosuppr essi
onmaybe
ofbenefi
tforgiantcel
lmyocar dit
is,
buthis3-weekpresentati
oni si
nconsi
stentwi thgi
antcell
.
Endomyocar di
albiopsyisindicat
edfordiagnosi
sifi
tinf
luencesmanagement .Thepatienti
s
notacandidateforICDbecausehehasnotbeenopt imallymedi cal
lytr
eat
ed.Althoughsome
pati
entswithacutemyocar diti
smayr equir
emechanicalsuppor t
,nothi
nginthisquesti
on
suggeststhatthepatienti
si l
ltot
hatextent.
 

69. Thel ef
tvent
ri
cul
arandsyst
emi
car
ter
ialpr
essur
etr
aci
ngsshowni
nFi
gur
e1ar
e
obt
ained.

I
nwhi
choft
hef
oll
owi
ngcondi
ti
on(
s)ar
ethesef
indi
ngsexpect
ed?

a. Supraval
vul
araort
icstenosi
s.
b. Subaorti
cmembrane.
c. Val
vularaor
ti
cstenosis.
d. Hypertr
ophi
cobstruct
ivecar
diomyopat
hy(
HOCM)
.
ANSWER: 

69. d. Hyper
trophi
cobst
ruct
ivecar
diomyopat
hy(
HOCM)
.

Inthepost extrasystol
icbeat(foll
owingavent r
icul
arpr emat urebeat),ther
eisani ncreasei nt
he
pressuregr adient.Thisfi
ndingisobser vedinaor t
icst enosis( AS),subaorti
cmembr ane, and
HOCM, anddoesnothel pdi st
inguishamongt hethree.However ,i
nHOCM, t
heincreased
contracti
litythatcharacter
izesthepost ext
rasystoli
cbeatcausesgr eaterapposit
ionoft he
i
nterventricularseptum totheant eri
orleaf
letofthemi tralvalve,i
ncreasingthedegr eeof
obstructi
on.Thi seffectoverwhelmsanybenef itobtainedf rom thehi gherend-diastol
icvolume
thatcharact eri
zesthepost ext
rasystoli
cbeat .Therefore, t
hest rokevolume, andhence, the
aorti
cpul sepr essurefal
ls.Thisphenomenoni sknownasaBr ockenbr oughsign. 

I
nval vul
arAS, supr
aval
vul
arAS, 
andsubaor t
icmembr ane,t
heincr
easedprel
oadinthe
postextr
asystoli
cbeati
ncr
easesstr
okevolume,result
inginbot
hani ncr
easeingr
adientduet
o
thehigherfl
owandani ncr
easeinaort
icpulsepressur
eduet ot
hehigherstr
okevolume.

70. Youar ewalki


ngthr
oughthecar
diaccathet
eri
zat
ionl
abor
ator
yandseeat
raci
ngofr
ight
atr
ial(
RA)pr
essur
eonthemonit
or(Fi
gure1).

Whi
choft
hef
oll
owi
ngi
sthemostl
ikel
ydi
agnosi
s?
a. Pulmonaryhypert
ensi
on.
b. Peri
cardi
altamponade.
c. Constr
icti
on.
d. Tri
cuspidregur
git
ati
on.

ANSWER: 

70. b. Per
icar
dialt
amponade.

TheRApr essur
eishigh(mean=appr
oximatel
y15mm Hg, wi
thnormal=6mm Hg).Ther
eare
normalAwaves, Xdescent
s,andVwaves,
buttheYdescenti
sblunt
ed.Thi
sist
ypi
call
yseeni
n
tamponade. 

Inconst
ri
cti
veperi
cardi
ti
s,t
heRApressuremaybeel evat
ed,butthewavefor
msshowst eepX
andYdescents.RApressur
emaybehighi npulmonaryhypert
ension,
butapulmonaryar
ter
y
tr
acingi
salsoneededtomakethi
sdiagnosis.Youwouldnotexpecttohaveablunt
edY
descent

Inthesett
ingoft
ricuspi
dregur
git
ati
on,
theRAt
raci
ngwi
lldemonst
rat
eal
argeCVwavedur
ing
ventr
icul
arsyst
ole. 

CannonAwavesareobser
vedwhenthereisanarrhyt
hmi
acharact
eri
zedbysi
multaneous
depol
ari
zat
ionoft
heatr
iaandvent
ri
cles.Pr
ominentXandYdescentsar
eseeninperi
cardi
al
const
ricti
on.Lar
geV,
or,
mor
eaccur
atel
y,r
egur
git
ant(
CV)wavesar
eobser
vedi
ntr
icuspi
d
regur
gitat
ion.

71. Yourpat ientwithprevi


ouslystableNewYor kHeartAssociat
ionclassIIheartfai
lur
eand
ast
hmar eturnstoyourof f
icewithorthopnea,par oxysmalnoct ur
naldyspnea,andnoct uria.He
hasbeent akingfurosemide40mgdai ly,met oprololsucci
nate50mgdai l
y,li
sinopri
l20mg
dai
ly,anddigoxin0.125mgdai l
y.Themet oprololwasbegun3weeksago.Onexam,hi shear t
rat
ei s110bpm,bl oodpressureis100/ 70mm Hg,andvenouspr essureis15cm ofwat er.
Cardi
ovascularexam revealsamitralregurgi
tationmur murthatislouderthanpreviousl
yhear d.
Hehasanor malS1, S2,andanS3 isheard.Hisl ungsar ecl
ear,andthereisnoper i
pheraledema.

Whi
choft
hef
oll
owi
ngi
sthenextbestst
ep?

a. St
artaldost
eroneantagonist.
b. I
ncreaseli
sinopri
l.
c. I
ncreasehisfurosemide.
d. Decreasehi
sbet a-bl
ocker.

ANSWER: 

71. c. I
ncr
easehi
sfur
osemi
de.

Volumeretenti
onisacommoncompl i
cati
onofbet a-bl
ockert
herapyandshoul dbetreated
wit
hani ncreaseindiuret
ict
oachievecli
nicaleuvolemi
a.Generall
y,beta-blockerdosesdonot
needtobedecr easedunlessther
eishypotensionordiuret
icr
esistance.Aldosterone
antagoni
stsandl i
sinopri
lwouldnotbeexpectedtoachieveapromptandadequat ediur
esis.

72. A 55- year-oldmanwi thischemi csystol


ichear tfai
lur
e( HF)wi thbasel
ineeject
ion
fract
ionof30%i sbeingseeni nfol
low- upafterarecentadmissionfordecompensatedHF.He
reportsthatheget sshor tofbreathwal kingupt hethreestepsi ntohishouse.Hisdai l
y
medicat i
onsi
ncludemet oprol
olsuccinate200mg, li
sinopri
l20mg, furosemide120mg,aspiri
n
81mg, andat
orvastati
n40mg.  

Onexam,hi sbloodpressureis95/ 65mm Hgandhear trateis62bpm.Hi sjugularvenous


pressur
eis7cm H2O.Hi sl ungsar ecl
ear
,andcar di
ovascularexaminati
onr evealsnormal
S1 andS2 
andgrade2/ 6holosystol
icmurmur.Hisextr
emi ti
esshow tr
aceedema.Sodi um i
s
136mg/ dl,pot
assi
um is4. 2mEq/ L,andserum creati
nineis1. 4mg/ dl
.Electr
ocardiogram
showsnormalsinusrhythm withevi
denceofpr
iorant
eri
ori nf
arct
ion.

Whi
choft
hef
oll
owi
ngshoul
dbeaddedt
ohi
smedi
cat
ionr
egi
men?

a. Warfar
in.
b. Angi
otensin-
receptorbl
ocker(
ARB)
.
c. Amiodarone.
d. Al
dosteroneantagonist
.

ANSWER: 

72. d. Al
dost
eroneant
agoni
st.
Thepat i
enti
stolerati
nganangi otensin-
conver
ti
ngenzymeinhibi
tor,
soaddi ti
onofanARBor
substit
uti
onofanARBi snotindicated.Hisr
enalf
uncti
onandpotassium level
swoul dall
owfor
additi
onofaldosteroneantagonistinthi
spati
entwit
hclassI
IIheartf
ail
ure.Ami odar
one,
war f
ari
n,andserum level
sofdigoxinhavenotbeenshownt oextendsurvivalorimprovehear
t
fai
luremorbi
dity.

73. A33- year-


oldwomanwi t
hahistoryofperi
part
um cardi
omyopat hy(PPCM)comest o
youst at
ingthatshewant stohaveanotherbaby.Anechocar
diogram i
nt heoff
icer
eveal
sthat
hercur r
entleftventri
cularej
ect
ionfr
act
ion( LVEF)i
s55%.Sheisonl i
sinopri
l20mgadayand
carvedil
ol12.5mgt wiceaday.

Whi
choft
hef
oll
owi
ngi
scor
rectr
egar
dingherr
iskwi
thsubsequentpr
egnancy?

a. Herr
iskofrecur
renthear
tfai
lur
ewi
thasubsequentpregnancyi
sapproxi
matel
y20%.
b. She shoul
d conti
nue curr
entmedi
cat
ions dur
ing pregnancytoreduce t
he r
iskof
r
ecur
renthear
tfai
lur
e.
c. Sheisnotatriskofr
ecurrentheartf
ail
ure.
d. Recurr
entpregnancyisassociat
edwi t
h80%mat
ernalmor
tal
it
y.

ANSWER: 

73. a. Herr
iskofr
ecur
renthear
tfai
lur
ewi
thasubsequentpr
egnancyi
s
appr
oxi
mat
ely20%.

Inallwomenwi thahi storyofPPCM, anextensi


vedi scussionabouttherisksofasubsequent
chil
disimportant.Thispat i
ent'
sLVEFhasr etur
nedt onor mal,butshesti
llhasar i
skofrel
apse
withsubsequentpr egnancy.Basedonast udyof44womenwi thahistor
yofPPCM, women
whor ecover
edtheirLVEFhada21%r i
skofrecurrencewithsubsequentpr egnancy,butt
here
werenomat ernaldeat hs.Inthepati
entswithpersist
entLVdysf uncti
on,therewasa44%r i
skof
recurr
entheartf
ailureanda19%r iskofmaternalmor tal
it
ywi t
hsubsequentpr egnancy.

Bothangiot
ensi
n-convert
ingenzymeinhi
bit
orsandangiot
ensin-
recept
orbl
ockersar
e
ter
atogeni
candtheref
orenotsafedur
ingpregnancy;medi
cat
ionsshouldberevi
ewedifa
pati
entdeci
destoproceed.

74. A 45- year-ol


d man wi th known hyper trophic obstr
uctive cardi
omyopat
hy and
signifi
cantoutf
low tr
actgr adi
enti
sadmi tt
edt othei ntensi
vecareuni twit
hhypot
ensi
onaft
er
receivi
ngspinalanesthesia.Hewasgi ven3liter
soff luidinthepost-anest
hesi
acareuni
tand
stil
lremainshypotensive.Hehasalsodevelopedralesonexam.
 
Whi chofthefol
lowingwoul dyourecommendt or ai
sehi sbloodpressure?

a. Dobut ami ne.


b. Epinephr i
ne.
c. Phe nylepinephr
ine.
d. Dopami ne.
e. Isoprotenol.

ANSWER: 
74. c. Phenyl
epi
nephr
ine.

Whent r
eati
nghypotensioninpati
entswi thobst
ructivecar di
omyopathy,inot
ropesthat
i
ncreasecontr
acti
li
tyshouldbeavoided, astheywillincreasethel
eftventri
cularout
fl
owt r
act
gr
adientandreducecardiacoutput
.Instead,agentst hatincr
easeaf
terloadandhavel i
tt
le
ef
fectonmyocardialcontr
acti
li
tyshouldbepr ef
erent i
all
yused. 

Phenylepi
nephr
inehaspot
entalpha-adrenergi
cacti
vit
yandlit
tl
ebeta-
adrenergi
cacti
vity,
and
i
stherefor
eagoodchoicei
nt hi
sset t
ing.Al
loftheotherchoi
ceshavesi
gnif
icantbet
a-recept
or
act
ivi
tyandwoul dwor
senthepatient
'soutfl
owt r
actgradi
ent
.

75. A30- year-oldmancomest oyourcl


ini
cwit
ha1- yearhistor
yofexer t
ionalchestpai
n,
dyspnea,andpal pitat
ions.Hehashadaknownhear tmurmurf orseveralyears.Hedoesnot
takeanymedi cationsorsupplementsanddeni
esanyrecr
eationalsubstanceabuse.Thereare
nocardiacri
skf actorsorhist
oryofpr
ematur
ecardi
acdeathinthefamily.
 

Hisphysi
calexam demonstrat
esaheartrat
eof56bpm andbloodpressurei
nt hel
eftar
m of
115/78mm Hg.Agr ade3/6di amond-
shapedsystol
icmurmuri
sappr eci
atedovertheleft
l
owersternalbor
der;i
tislouderduri
ngthestr
ainphaseoft
heValsal
vamaneuverandsof ter
wit
hsquatti
ng.

Forwhichofthef
oll
owi
ngr
iskf
act
orsi
sitr
easonabl
etoi
mpl
antani
mpl
ant
abl
ecar
diover
ter
-
def
ibr
il
lat
or(I
CD)?

a. Fami l
yhistor
yofsyncope.
b. Leftventri
cul
ar(LV)wal lt
hicknessof>30mm.
c. Re pet
iti
venonsustainedVT( NSVT)onHoltermonitor
.
d. Inducibl
eventri
culartachycardi
a(VT)onelect
rophysi
ologyst
udy(
EPS)
.

ANSWER

75. b. Lef
tvent
ri
cul
ar(
LV)wal
lthi
cknessof>30mm.

I
npat i
entswithhypert
rophiccardi
omyopathy( HCM)andaper sonalhistoryofsuddencardiac
deat h(SCD),vent
ri
cularfi
bri
ll
ati
on,orhemodynami call
ysignifi
cantVT, implantat
ionofanICD
i
saCl assIrecommendat i
on.However,guidelinessurroundingt hedecisionforpri
mary
preventionI
CDsi nthesetti
ngofHCM ar edrivenpredomi natelybyexper tconsensus.Cl
assI I
a
recommendat i
ons(LevelofEvi
denceC)includef i
rst-degr
eer elati
vewithSCD, maximum wal l
thickness>30mm, andoneormor erecentsyncopalepi sodes.

I
nHCM pat i
entswithNSVTorabnor malbl oodpressureresponset oexerci
se,otherr
iskfact
ors
mustbepr esenttowar r
antimplant
ationofanI CD.Theseot herr
iskfactorsincl
uderesti
ngLV
outfl
owt r
actobstruct
ion>30mm Hg, gadoli
nium enhancementr epresenti
ngmyocar di
al
fi
brosisbycardiacmagneticresonance,andLVapi calaneurysm.

Thereisnorol
eforEPSinthedecisi
onaboutpri
marypreventi
onICDsinHCM.Fami
lyhi
stor
yof
syncopeisnotanindi
cati
onforICD(asopposedtof
amilyhist
oryofSCD)
.NSVTonHolt
er
monitorr
equir
esotherri
skfact
orstobepresent
,asdescr
ibedabove.
 
76. A55- year-ol
dwhi tewomanwi t
ha4- mont hhistoryofnoni schemiccar di
omyopat hy
comesi nforfoll
ow- upofherhear tfai
lure.Shewal ksone- halfmilebeforesheget sshor tof
breath,anddeni esdi zzi
ness,palpi
tat
ions,orthopnea,orpar oxysmalnoct urnaldyspnea.Her
l
eftvent r
icul
arejecti
onf r
actionwhenassessedl astmont hwas32%.Onexami nat
ion,herblood
pressurei s104/70mm Hg,andpul se72bpm.Herj ugularvenouspr essur
ei snotel evated,
normalS1  andS2 withoutmur mursorgal lop,andt hereisnoper ipheraledema.Medi cat
ions
i
ncludel i
sinopri
l5mgdai ly,car
vedil
ol12.5mgt wicedaily,andfurosemide40mgt wicedai l
y.
Herser um creat
inineis2.5mg/ dl
.

Whi
chi
sthenextmostappr
opr
iat
est
epi
nhercar
e?

a. Adds pironol
actone12. 5daily.
b. Decreaset heli
sinoprilt
o2.5mgaday.
c. I
ncreasecar vedil
olto18. 75mgt wicedail
y.
d. Adddi goxin0.125dai l
y.
e. Addisor di
l20mg/ hydral
azine37.5mgt hreet
imesdai
ly.

ANSWER: 

76. c. I
ncr
easecar
vedi
lolt
o18.
75mgt
wicedai
ly.

Beta-blockerdoseshoul dbeescal atedtoachievet argetdoses( e.g.,


carvedilol25mgt wice
dai
lyormet oprololsuccinat
e200mgdai l
y)intheabsenceofcont rai
ndicati
on.Spironol
actone
shouldnotbest artedgivenhercr eatini
neof2.5mg/ dl.Digoxinwoul dnotbei ni
ti
atedgi
venher
cl
assI Isympt omsbef or
eoptimizationofot hermedi caltherapy.Isordil
-hydralazi
neisnot
i
ndicatedi nclassIIwhi t
epatients.Ther eisnor easont oreducet hedoseoft heangi ot
ensin-
convertinginhibi
tor .
 

77. A75- year


- ol
dmanwi t
ha10- yearhist
oryofnonischemiccar di
omyopathy(
eject
ion
fr
acti
on25%)i sadmi t
tedwit
hacut
edecompensat edheartf
ail
ure.Hehasbeenunablet
oaf f
ord
hismedicati
onsf orthelastmont
h.Hehadgai ned30poundsovert heprecedi
ng2weeks.
Overni
ght,hewasgi venonedoseofi
ntravenous(I
V)fur
osemide60mgandhemade3l it
ersof
uri
ne. 

Onexami nat
ion,hi
sbloodpressur
ewas105/ 78mm Hg,pul sewas90bpm andr egul
ar,and
respir
atoryratewas20br eat
hs/minute.Hedidnotappearacut el
yil
l.Hi
sj ugularvenous
pressurewasatt heangleoft
hejaw,lungshaddimini
shedbreathsoundsatt
her i
ghtbase,and
hehadasof tS3.Hehadasci t
es,andt her
ewas3+pi t
ti
ngedemat othethigh.Laborat
ory
analysi
sshowedanel evat
edN- t
erminalpro–B-t
ypenatri
ur et
icpepti
de(2500pg/ ml),and
hyponatremia(130mmol /L)
.Hiscr
eatini
newaselevat
edat2.6mg/ dl.

Whi
choft
hef
oll
owi
ngwoul
dyour
ecommendf
ort
hispat
ient
?

a. Mil
rinone.
b. ContinueIVfur
osemi
de.
c. Carvedil
ol.
d. Nesiri
ti
de.
e. Metolazone.
ANSWER: 

77. b. Cont
inueI
Vfur
osemi
de.

Thepatienti
srespondi
ngappropriat
elytoIVdi ur
eti
csandt heseshouldbecont i
nuedsi
ncethe
pat
ientismarkedlyvol
umeoverloaded.Neithernesir
iti
denormi lr
inonehavebeenshownt o
i
mpr oveoutcomesinpati
entswithdecompensat edhear tf
ail
ure.Beta-blocker
sshouldnotbe
i
nit
iatedinpati
entswhoareacutelydecompensat ed,butshouldbeconsi deredoncethepati
ent
i
scompensat ed.Metol
azoneisnotindicat
ed, asthi
spat i
enti
sr espondingwelltofur
osemide. 

78. A76- year-oldmanwi thischemiccar diomyopathyand r


ecur r
entadmi ssionsf orhear
t
fail
ure despite excell
entmedi caland di etary compli
ance presentst o yourof f
ice f
ora
consul t
ati
on.Hei saccompani edbyhi swi feanddaught er.Hedeniesangi nalsympt omsand
under wentcor onary angiography 1 yearago t hatshowed pat entbypass  gr
afts.He has
dyspneawi thmi nimalexer t
ion.Hisangi ot
ensi n-
convert
ingenzymei nhibi
tor(ACEI)andbet a-
blockerhavebeendownt i
tr
atedovert helastfewmont hs,andult
imatelyhadt obedi sconti
nued.
Hehasani mplantablecardiovert
er-defi
bri
llator(I
CD)inplaceandr ecei
vedashock2mont hs
agof orventri
culartachycardia.
 

Onexami nat i
on,hisbl oodpressureis80/60mm Hg, hispulse98bpm, andhi sr
espir
atoryrate
is16br eaths/mi nute.Hei snotacut el
yill
.Hehasmi l
dbi t
empor alwast i
ngpresent.Jugular
venouspr essur eis12cm,auscul t
at i
onrevealsS1 andS2  wi
thgr ade2/ 6mur murofmi t
ral
regurgitat
ion at t he apex.Hi sliver is not distended,and he has 1+ edema.Hi s
electr
ocar diogram showssi nusrhythm,withar ightbundlebranchbl ock( RBBB),QRSof110
msec,wi th deep Q wavesacr osst heprecordium.Echocar di
ogr am showsl eftventri
cular
diastol
icdi mensi on( LVDd) 
tobe 7.
8cm, l
eftventr
icul
arejecti
onfracti
on( LVEF)20%,moder ate
mi t
ralregurgitati
on( MR) ,
andhi sri
ghtventri
cularfuncti
onwasmi ldlydepressed.

Whi
choft
hef
oll
owi
ngwoul
dbeappr
opr
iat
einhi
scar
e?

a. Mit
ralval
ver epai
r.
b. Upgradefr
om I CDtocardi
acresynchr
oni
zati
ont
her
apydef
ibr
il
lat
or(
CRT-
D).
c. Repeatcoronaryangiogr
aphy.
d. Desti
nati
onvent r
icul
arassi
stdevice(
VAD).
e. Metoprol
olsuccinate12.
5mg/ day.

ANSWER: 

78. d. Dest
inat
ionvent
ri
cul
arassi
stdevi
ce(
VAD)
.

Thepat i
enthasmul ti
plemarker sofadvancedhear tf
ail
ure,i
ncl
udingr ecurr
enthospi
tali
zati
on,
i
ntolerancetoACEIandbet a-blocker,ICDshock,andcachexia(manifestasbitemporal
wasting).Heappearstohavegoodf amil
ysupportandtobecompl iant.Overall
,hei
sa
candidateforadvancedheartfailuretherapi
es.Hi
sagei sabovethecut offf
orconsi
derati
onfor
cardiactranspl
antat
ion,buthemi ghtbenefi
tfr
om desti
nati
on-t
herapyVAD.  

I
niti
ati
onofbet
a-bl
ocker
swouldnotbeindicat
edatthi
spoint
,becauseher
emains
decompensat
ed.Amitr
alval
verepai
rwouldnotbethepref
err
edtreat
mentgi
venhisi
schemi
c
cardi
omyopathywithamar kedlyenlar
gedLVDd, onl
ymoder ateMR,andhistwopriorcor
onary
art
erybypassgraf
ts.Bi
ventricul
arpacingisunli
kel
ytobenefithi
m,gi
venhisQRSi s110msec
i
nanRBBBpat t
ern.Thepatientdoesnothavechestpain,hadacardiaccat
heter
izati
onwith
patentgr
aft
s1yearago, andar epeatcoronar
yangiogr
am isnotindi
cated.
 

79. A40- year-oldmanwi t


ha5- yearhistor
yoff amil
ialdil
atedcar di
omyopathyandhear t
fai
lur
e( HF)presentst ot he emergency departmentwi t
hi ncreased short
ness ofbreath,
parti
cul
arl
ywhenhet ri
est osleepatni ght.Hi
smedi calhist
oryissi gni
fi
cantforanxi
ety.On
examinati
on,
hewasi nnoacut edist
ress,hadbloodpressureof110/ 80mm Hg, hadapulseof
98bpm, andwasaf ebri
le.Hisbodymassi ndexwas25kg/ m2. 

Hisjugul
arvenouspulsationswerevi si
blej
ustbelowhi searlobeswhi l
ehewassitt
ingupright
.
Oncar di
acauscult
ati
on, t
herewasnoS3  ormur mur.Auscultati
onrevealedcl
earl
ungfiel
ds.His
abdomenwasbeni gnwithouttendernessorflui
dshift.Hisextremit
ieswerewarm andwithout
peri
pheraledema.Hi selectr
ocardiogram showedsi nusr hythm,leftvent
ri
cul
arhypertr
ophy,
andnoot herabnor
mal i
ty.ChestX-rayshowedmi ldcar di
omegal y.

Whi
choft
hef
oll
owi
ngi
sthel
ikel
yet
iol
ogyofhi
ssympt
oms?

a. Sl
eepapnea.
b. Myocar
diali
nfar
cti
on.
c. Decompensat
edHF.
d. Pul
monaryembolus.
e. Pani
catt
ack.

ANSWER: 

79. c. Decompensat
edHF.

Thepat i
entpr esentswithorthopnea, whichisamar kerofelevatedfil
li
ngpressuresi npatient
s
withHF.Hi sjugularvenouspr essurewasel evatedwi t
hpulsationsvisi
blebyhisear lobesinthe
uprightposit
ion.Itisimportantt orecognizethatrales,l
egedema, andpulmonar yedemaon
chestX- r
aycanal lbeabsenti npat i
ent
swi thdecompensat edHF.Hi sbodymassi ndexwas
normal ,
maki ngsleepapneal esslikel
y.Giventhathehadobj ecti
veevidenceofel evatedjugul
ar
venouspr essure,itisl
ikel
ythathi ssympt omswer esecondar ytodecompensat edHFandnota
panicattack,despitehishistoryofananxi etydisorder.

80. A 25- year


- old Af ri
can- Amer i
can man wi t
h a 3- year history of noni schemic
cardiomyopat hypresent stoyourof f
iceforfoll
ow- up.Her eports 
shortnessofbreat hwit hl
ess
thanonef l
ightofst airs.Hiscur rentmedi calr egimeni ncludescar vedil
ol25mgt wicedaily,
aldactone 25 mg dai l
y,l i
sinopril40 mg t wi ce dai ly,furosemi de 80 mg t wice daily,and
potassium 40mEqt wicedai ly.Hishomesyst olicbl oodpr essuresar ebetween145and160
mm Hg.Onexami nat i
on,heappear scomf ortabl e.Vitalsignsar ebloodpr essure150/ 100mm
Hg,pul se78bpm,r espirat
oryr ate16br eaths/mi ntut e.Perti
nentf i
ndingsonhi sexami nati
on
i
ncludedcl earlungf i
elds,jugularvenouspr essur eof8cm,anor malS1 andS2 wi t
houtS3 or
mur mur ,and hisl owerext remiti
esar ewar m wi thoutedema.Hi sbasicmet aboli
cpanel
revealedanor malpot assium andcr eatini
neval ues.  

Whi
choft
hef
oll
owi
ngi
smostappr
opr
iat
einhi
scar
e?
a. Dil
ti
azem sustai
ned-r
elease120mgdaily.
b. CoenzymeQ10100mgdai ly.
c. I
sosorbidedini
tr
ate20mg/ hydral
azi
ne37.
5mgt
hreet
imesdai
ly.
d. Candesart
an4mgdai l
y.
e. Cl
onidine0.1mgt wi
cedai l
y.

ANSWER: 

80. c. I
sosor
bidedi
nit
rat
e20mg/
hydr
alazi
ne37.
5mgt
hreet
imesdai
ly.

TheA- HeFT(African-Amer i
canHear tFail
ureTr i
al)trialdemonst rat
edsignif
icantreduct i
onin
mortal
itywhent hecombi nationofisosorbi
dedi nitrate-hydr
al azi
newasi ncorporatedint ot
he
medicalregi
menofsel f-
ident if
iedAfri
can-Amer i
canswi t
hsyst ol
icheartf
ail
ur eandNewYor k
HeartAssociat
ioncl assIII-I
Vsympt oms.Thecombi nationofanangi ot
ensin- convert
ing
enzymei nhi
bit
or, al
dactone, andanangi otensin-recept orbl
ockershoul dnotbeusedt oget
her
duetother i
skofhyper kalemi a.Theuseofanondi hydr opyr
idinecalci
um channelbl ocker
(har
m)ornut ri
ti
onalsuppl ement s(nobenefit
)car ryacl assIIIrecommendat ion.Clonidinehas
notbeenadequat el
ytestedi nsystoli
cheartfail
ure. 

81. A25- year -


oldmandevel opeddyspneaonexer ti
on2mont hsago.Eval
uationbyanother
physici
ani ncludedanel ectr
ocardiogram wit
hleftbundlebr anchblockat160msecandan
echocardi
ogr am thatshowedal eftventri
cul
areject
ionfract
ionof25%wi thglobalhypoki
nesis
andnoval vularabnormalit
ies.Anexer ci
seperf
usionstudydemonst rat
ednoischemi a.Hewas
star
tedonenal apr
il5mgbi dandf urosemide20mgbi d.
 

Youal sot r
eathisfather,whohasanoni schemiccar diomyopathy;thus,t
hepat ientpr
esentst
o
yourof f
iceforasecondopi nion.Her eport
songoi ngdyspneaatone- hal
fmi le.Hedeniedany
chestpai n,pal
pitat
ions,syncope,or thopnea,par oxysmalnocturnaldyspnea,orl egedema.On
examinat i
on,hewasi nnoacut edistress.Hisbl oodpr essur
eis110/ 80mm Hg, hispul
seis72
bpm,andhi srespirati
onis16br eaths/minute.Ther ewasnoj ugularvenousdi st
enti
onor
hepatojugularrefl
ux.AuscultationrevealedS1 andS2  arenormalwithasof tS3 andnomur mur.
Extr
emi ti
eswer ewar m,andt herewasnoedema.Your evi
ewedt heout si
deechocar di
ogram
andagr eewi t
ht hepreviousinterpr
etat i
on.

Whi
choft
hef
oll
owi
ngdoyour
ecommend?

a. Ref
erforcardiacresynchronizati
ont her
apy-def
ibr
il
lat
or(
CRT-
D).
b. Per
form anendomyocar dialbiopsy.
c. St
artcarvedi
lol3.125mgbi d.
d. Ref
erforimplantablecardiovert
er-defi
bri
ll
ator(
ICD).
e. Ref
erforcoronaryangiography.

ANSWER: 

81. c. St
artcar
vedi
lol3.
125mgbi
d.

Thepatientpr
esent
swithnew- onsetsyst ol
icheartfai
lur
e,possi
blyfr
om afamili
al
cardi
omyopathy,gi
venhisfat
her’sil
lness.Hecur rentl
yisNewYor kHeartAssociat
ioncl
assI
I,
appearscompensatedonexami nat
ion, andaddingabet a-bl
ockertohisangi
otensin-
convert
ingenzymeinhi
bit
orwoul dbei ndicated.
 
Anendomyocar di
albiopsyshoul dnotbeper for
medi nther out
ineeval
uati
onofpatientswit
h
heartfail
ure(Cl
assIIIi
ndicati
oni nthehear tf
ail
ureguideline).BothICDandCRT- Dshouldbe
i
mpl antedfoll
owingat ri
alofguidel
ine- basedmedicaltherapy, whichwoul
dincludebeta-
bl
ocker s,f
orthispharmacologicaltherapycouldpromot er everseremodel
ingandthereby
obviat
et heneedforeitherICDorCRT- D.Givenhisage, absenceofchestpain,andnegative
str
essper fusi
onstudy,coronaryangi ographyisnotindicated. 

82. A56- year-oldmanwi t


hani schemi ccar di
omyopat hyandsympt omati
chear tfailur e
presentstoyourof ficeforaconsul tati
vevi sitafterar ecenthospit
alizationforheartfai
lure.He
underwentcor onar yarterybypassgr afting2year sagoi nt hesett
ingofanacut emyocar di al
infar
cti
on.Hehasbeenhospi tal
izedt wi cei nt helast  
3mont hsf oracut eheartfail
ureand
ventri
cular tachycar di
a( treat
ed wi thi mpl antable car di
over
ter-defibri
ll
ator [
ICD]shock) .
Echocardiographicassessmentdur ingt hel asthospi tali
zati
onshowedl ef
tventri
cularejection
fract
ion(LVEF)of  
15%wi thmoder atemi tralr egurgit
ationandLVend- diastol
icdi
mensi onof  
7
cm.Dur ing the hospi tali
zati
on f or vent ricul
ar t achycardi
a,cor onary angiography had
demonst rat
edpat encyofhi sbypassgr aftsandher epor t
snochestpai n. 

Her eportsdyspneawi th minimalexer t


ion such aswal ki
ng t othebathroom.Her eports
compliancewi t
hhismedi cati
onandpr escribedlow-sodium dietandstoppedt obaccouseat
theti
meofhi smyocar dialinf
arct
ion.Thisisconf i
rmedbyhi swi fewhoaccompani eshimt o
thi
svisit.Duetosympt omat ichypotensi
onandacut ekidneyinjurydur
ingthel astadmissi
on,
hisangiotensi
on-converti
ngenzyme( ACE)inhibi
torandaldactonehavebeendi sconti
nued,but
heistakingcarvedi
lol3.125mgPObi d.
 

Onexami nation,his 
bloodpr essure i
s92/ 71mm Hg,hispul
seis100bpm,andhi srespi
rator
y
2
rateis16br eaths/minute;bodymassi ndexi
s24kg/ m ;heisinnodist
ress.Jugularvenous
pressureis14cm,auscul t
ationrevealsanS3 wi
thgrade3/6mur murofmit
ralregurgi
tat
ionat
theapex.Hisl i
verisnotdi stended,andhehas1+edema.Cr eat
ini
neis1.
3mg/ dl.Youperform
acardiopulmonar yexercisetestandf i
ndhehasapeakoxygenconsumpt i
onof10ml /kg/
mi n.

Hieel
ect
rocardi
ogram shows 
normalsi
nusrhythm,ant
eri
orQwaves,andaQRSdur
ati
onof
100msec. 
Right
-hear
tcathet
eri
zat
iondemonst
ratest
hefol
lowi
ngr
esul
ts:
 

 Ri
ghtatr
ialpressure:12mm Hg
 Pul
monar yarter
ypr essur
e(PAP):45/20mm Hgwi
thameanPAP32mm Hg
 Pul
monar ycapil
lar
ywedgepr essure:25mm Hg
 Car
diacoutput:4L/ min
 Car
diaci
ndex:1. 8L/ mi m2
n/

Att
hispoi
nt,
whi
choft
hef
oll
owi
ngwoul
dbeappr
opr
iat
e?

a. Ref
erhimforupgradetobi ventr
icularICD.
b. Ref
erhimforcardi
actranspl ant
ation.
c. I
nit
iat
ether
apywi t
hmi l
rinone0. 25mcg/ kg/min.
d. Ref
erhimf orconsider
at i
onofal eftvent
ri
cularassi
stdevi
ce(
LVAD)asdest
inat
ion
t
her
apy.
e. Ref
erhi
mforconsi
der
ati
onofmi
tr
alval
ver
epai
r.
ANSWER: 

82. b. Ref
erhi
mforcar
diact
ranspl
ant
ati
on.

Thepat ienthasmul tiplemar kersofadvancedhear tfai


lur e,i
ncl udingr ecur renthospi talizat
ion,
nar r
owpul sepr essur e,restingt achycar dia,i
ntolerancet oACEi nhibitor ,
dilatedvent ri
cl e,and
electri
calinst abil
ity.Car diopulmonar yexerci
set estingcanpr ovidei mpor tantpr ognost ic
i
nf ormation, andapeakoxygenconsumpt ion<14ml /
kg/ mi nsuggest s>50%1- yearmor tal
it
y
andconsi derat i
onofcar diact ranspl antati
on.Heappear st ohavegoodf ami lysuppor tandt o
becompl iantanddoesnotappeart ohavemedi calcont raindicationst ot ransplantationsuch
asongoi ngsubst anceabuse, pul monar yhyper t
ensi on, advancedage, obesi t
y,orpoorr enal
function.Ther efore, hewoul dnotbeconsi deredf ordest inationt herapyLVADi mplant ation,
whi chisgener all
yr eservedf ornont ransplantcandi dates.Hei snotacandi datef orcar diac
resynchr onizationt herapybecausehi sQRSdur ationi s<120msec.Ami t
ralvalver epai rwould
notbet hepr eferr
edt r
eat mentgi venhi sischemiccar diomyopat hy, EF<30%wi t
hamar kedly
enlargedLVdi astoli
c- dimensi on, andonl ymoder atemi tr
alregur gitation.Al thoughi not ropi
c
suppor tmaybeusedaf terlisti
ngf orcardiactranspl antation, thereisnour gentneedf ort hi
s
therapyint heabsenceofend- organdysf unction. 

83. A45- year-


oldmanwi thahistoryofi
schemiccardi
omyopathyandal ef
tventr
icul
ar
ej
ectionfr
action(LVEF)of25% presentsforr
outi
nefoll
ow-up.Hedeniesor
thopnea,edema,
paroxysmalnocturnaldyspnea,
orangina.Hehasdyspneawhenwalkingt
woblocksorupone
fl
ightofst
air
s. 

Onexamination,hi
sbloodpr
essureis145/90mm Hgandhi shear
trateis80bpm.Hisj
ugul
ar
venouspressureis7cm withouthepato-j
ugul
arr
efl
ux.Hehasal ateral
lydi
spl
acedpoi
ntof
maximum impulse(PMI),
noS3,cl
earlungs,
andnolowerext
remit
yedema.  

Hi
scurr
entmedi cati
onsincl
udecarvedi
lol25mgt wicedai
ly,
li
sinopri
l40mgdai l
y,f
urosemide
40mgtwicedai l
y,aspi
ri
n81mgdai l
y,andsimvast ati
n40mgdai ly.Hi
slabor
atorydatareveal
anN-t
erminal-probrai
nnatri
uret
icpept i
de(NT- pr
oBNP)levelof1232pg/ ml,sodi
um of134
mmol/
L,potassium of4.
2mEq/ L,
andcr eati
nineof1.4mg/dl.

Whi
choft
hef
oll
owi
ngwoul
dbet
hemostappr
opr
iat
eforhi
scar
e?

a. Addaml odipi
ne10mgdai l
y.
b. I
ncreasefurosemideto60mgt wicedail
y.
c. Adds pi
ronolact
one12.5mgdaily.
d. Addlosartan25mgdai ly.
e. Addisosorbidedini
tr
ate10mgpl ushydral
azi
ne12.
5mgt
hreet
imesdai
ly.

ANSWER: 

83. c. Addspi
ronol
act
one12.
5mgdai
ly.

Thispati
enthasNewYorkHeartAssoci ati
on(NYHA)classIIIsympt omsandareducedLVEF.
Aldost
erone-r
ecept
orant
agoni
stsar erecommendedi npat i
entswi t
hNYHAclassII-I
Vheart
fai
lur
ewi t
hanLVEFof35%orless, unl
esscont r
aindi
cated,t
or educemorbidi
tyandmor t
ali
ty.
Thispati
ent’
screat
ini
neandpotassium level
sarewellbelowther ecommendedthreshol
dsf or
i
nit
iat
inganaldost
eroneantagoni
st.Cr
eat
ini
neshoul
dbe<2.
5mg/
dli
nmenand<2.
0mg/
dli
n
women, andpot
assium <5mEq/L.
 

Althoughhehasdyspneawi thexert
ionandanelevat
edNT-proBNP,heappear seuvolemicon
exami nat
ion.Thus,
incr
easingfur
osemidewouldbenotbeappropri
ate.Calci
um channel
blockingdrugsarenotrecommendedasr out
inetr
eatmentf
orpati
entswithheartfai
lureand
reducedEF. 

Acombi nati
onofisosorbidedini
trateandhydr alazineisrecommendedf orself
-report
ed
Afri
can-Americanpatientswit
hNYHAcl assI I
I-I
Vhear tfail
ureandreducedEFonopt i
mal
ther
apywi t
hanangi otensin-
conver ti
ngenzymei nhibi
tor(ACEI)andbet
a-blocker,orf
orany
pati
entwithsymptomat icheartf
ailurewit
hr educedEFwhocannotbegi venanACEIor
angiot
ensin-r
eceptorblocker(ARB) .However,thispatientisCaucasi
anandi staki
nganACEI,
makingthischoicelessappropriate. 

TheadditionofanARBt oanACEIandbeta-
blockerf
oraper
sist
entl
ysymptomati
cpatientwi
th
heartfai
lureandreducedEFshouldonl
ybeconsider
edwhenanaldost
eroner
eceptor
antagonistisnoti
ndicat
edortol
erat
ed.

84. Youhavef oll


owedan89- year
-ol
dpat i
entformanyyearsinyourpractice.Hehashad
stableangina,whi chhasbeenmanagedmedi call
y.Hehasbeent r
eatedforpr ost
atecancer
,
hypertensi
on, hyperl
ipi
demi
a,andhashaddi abetescontr
oll
edwit
hmedi cat
ions.Hei sbr
ought
tot hehospitalbyambul anceafterst
affathisassistedli
vingf
acil
it
ynot edt hathelooked
"gray.

Uponar r
ivaltot hehospi t
al,hei sment ati
ngandcompl ai
nsofchestpai n.Hehasabl ood
pressureof82/ 40mm Hg, heartrateof100bpm, andanoxygensaturat
ionof89%on6l it
ersof
nasalcannul a.Hislungsar econgest edonexam andyouhearanS3  aswel lasaholosystol i
c
mur mur.Hi sextremitiesarecool.Hisel ect
rocardi
ogram demonstr
ates4mm STel evationi n
theant eri
orpr ecordium.You r ecommend ur gentcardi
accatheteri
zati
on,butyourpat ient
ref
usesi nvasivepr ocedures,avi ew hehasexpr essedtoyoui nthepast .Youcont actt he
assist
edl i
vingfacil
ityanddet er
mi nethatthepati
enthasahome" donotr esusci
tat
e"order.

Whi
choft
hef
oll
owi
ngi
syournextst
epi
nthemanagementoft
hispat
ient
?

. Admi
tthepat
ientt
othegener
almedi
cineuni
t,pr
ovi
desuppor
ti
vemeasur
es,consul
t
pal
li
ati
vecar
eteam.
b. Pr
oceedt
othecat
het
eri
zat
ionl
abor
ator
yforur
gentper
cut
aneouscor
onar
yint
ervent
ion
wit
ht wophysicianconsent ssi
ncehisconditi
onisli
fe-t
hreat
ening.
c. Contactthepsychiatri
stoncallforur
gentevaluat
ionofpati
entcompetency.
d. Cal
lhi sspouset or equestherconsentf ort heproceduregiventhelif
e-t
hreat
ening
nat
ureofhi ssi
tuation.
e. Documentt hatthepat ientisnotcompetentandobt ai
nawi tnesstosigntheconsent
andpr oceedwit
ht heprocedureunderemer gencycir
cumstances.

ANSWER: 

84. a. Admitt
hepatienttot
hegener
almedi
cineuni
t,pr
ovi
desuppor
ti
vemeasur
es,
consul
tpal
li
ativecar
eteam.
Thi
spat ienthassignsofseverecardiacdysfuncti
onandacutemyocar dialinf
arcti
on.Whi l
e
car
diaccat het
erizati
onandrevasculari
zati
onar ei
ndi
catedandnecessar ytostabi l
izethe
pat
ient,hedoesnotwantsuchpr ocedures.Despit
ederangementinvitalsigns,thereisno
evi
dencet hatthepatienti
snotcompet enttomakemedi caldeci
sions.Hisdecisiont oavoid
i
nvasivemedi caltherapi
esshoul
dbehonor ed.

Alt
houghgr avel
yil
l,
heappearscompetent,
sother
eisnoindi
cati
ontocallpsychi
atr
y.Nei
ther
spousalconsentnortwophysi
cianconsent
sshouldover
ri
dehispref
erence.I
tisnot
appropri
atetofi
ndawi t
nesstosignt
heconsentbecausehei
scompet enttomakehisown
deci
sions. 

85. Youar econsul t


edtoseea36- year-
oldAf r
ican-Amer icanwomanont hepostpart
um
unitforevaluati
onofdyspneaandedema.Shehasahi st oryofchr onichyper t
ensionand
cocaineabuse.Twodaysago,shedel i
veredtwi ni nfants,andsi ncet henhasdevel oped
worseningdyspnea,or t
hopnea,andlowerextremityedema.Herbl oodpr essureis110/82mm
Hg,herhear trat
eis118bpm,andheroxygensat urati
oni s90%on6l i
terspernasalcannul a.
Sheappear sfati
gued,isinmoderaterespi
ratorydistress,andhasj ugularvenousdi st
entionto
themandi ble.Cardiacauscultat
ion r
evealstachycar dia,r egularr hyt
hm,and an S3  gall
op.
Cracklesar
ehear dinthelowerhalfoft
helungf i
eldsbi l
aterally.Theext r
emi t
iesarecooland3+
pit
ti
ngedemai spresentuptotheknees. 

Laborator
yf i
ndingsr evealasodi um of132mmol /L,potassi
um 4.2mmol /L,creati
nine 
1.7
mg/ dl
,magnesi um 2.
4mg/ dl,aspart
atetransaminase32U/ L,alani
netr
ansaminase28U/ L,
hemoglobin11.4g/ dl,andpl atel
etcountof280, 000/ mm3.Anechocardiogr
am isper f
ormed
showingnor mallef
tvent r
icul
arsize,butseverel
ydepr essedsystol
icf
unct
ion,est
imatedat10-
15%wi thglobalhypokinesis.Cardiaccatheteri
zat
ionshowsnor malcor
onari
es,withelevated
2
bivent
ri
cularfi
ll
ingpressures,andcardiacindexof1.2L/ min/
m.

Whi
choft
hef
oll
owi
ngi
sLEASTl
ikel
ytobeofpot
ent
ialt
her
apeut
icbenef
iti
nthi
scondi
ti
on?

a. Anti
coagul ati
on.
b. Angiotensin-convert
ingenzymeinhi
bit
or(
ACEI
).
c. Considerationforcardiactr
anspl
ant
ati
on.
d. Magne sium.
e. Mechani calcir
culat
or ysuppor
t.

ANSWER: 

85. d. Magnesi
um.

Thiscasei
ll
ustr
atesthepresentati
onofawomanwi t
hper i
partum cardiomyopathy(PPCM).
Advancedmaternalage,
mul t
iparousst
atus,
mult
i-f
etalgest
ation,
Af r
icandescent,hist
oryof
hypert
ensi
on/pr
e-eclampsia,andcocai
neabusehavebeenr eport
edt obepot enti
alr
isk
fact
ors.
 

Magnesium i
samainst
ayoft herapyf
orpre-eclampsia,butnoti
nperi
part
um car
diomyopat
hy.
Furt
hermore,
magnesi
um levelsarenormali
nt hiscase;ther
efor
e,magnesi
um
repl
acement/
suppl
ementati
oni snoti
ndicat
ed. 

Convent
ionalt
her
api
es,
suchasbet
a-bl
ocker
sandACEI
s,shoul
dbei
nit
iat
edoncet
hepat
ient
i
sdeemedhemodynami
cal
lyst
abl
e. 

Advancedheartfai
lur
etherapi
esoflef
tvent
ri
cul
arassi
stdevi
cesandcar
diact
ransplant
ati
on
arepotent
ialt
herapeut
icopti
onsforsomepati
ent
swi t
hsever
ePPCM wit
houtrecoveryof
vent
ri
cularfunct
ion.

Duet othehighi ncidenceoft hromboembol ism i nPPCM, anti


coagulati
oni sat herapeut i
c
considerati
on, especiall
yduringpr egnancyandi nthefirst6-8weekspost par t
um duet o
hypercoagulabi l
it
ydur i
ngtheset i
meper iods.Dat afrom smallstudieshavedemonst ratedt
hat
additi
onofbr omocr ipti
netost andar dheartfailuret her
apiesmayi mprovel eftventr
icular
ejecti
onfractionandcomposi teclinicaloutcomesi nwomenwi t
hPPCM.Thepost ulated
mechani sm ofact i
oni sthr
oughsuppr essionofpr olacti
n, whi
chi nits16-kDaant i
-angiogeni
c
andpr oapoptot i
cform mayl eadtoendot helialinfl
ammat i
on,impairedcardi omyocyt e
met abol
ism, andr educedmyocar dialcontractioni nPPCM.However ,t
histher apyisnotyet
recommended.

86. A79- year-ol


dwomanpr esent
stothecl i
nicwithshor t
nessofbr eathonexer ti
on.She
hasNew Yor kHeartAssociat
ionclassIIheartfail
ure.Hermedi calhistoryincludesahear t
murmur, poor l
y controll
ed hyper t
ensi
on, and hyper l
ipi
demi a. A t wo- di
mensional
echocar
diogram showspr eser
vedejecti
onfract
ion,moder at
eleftventr
icularhypertr
ophy,mil
d
aort
icstenosis,andanormalpulmonaryarterypressure.Theimagesf r
om t heechoar eshown
i
nFigures1, 2,and3.

Whi
choft
hef
oll
owi
ngbestdef
inest
hegr
adeofdi
ast
oli
cfunct
ion?
 
 
Fi
gur
e1
Fi
gur
e2
Fi
gure3
a. Gr
adeI
II
b.
b. Gr
adeI
.
c. Gr
adeI
I.
d. Gr
adeI
II
.

ANSWER: 

85. d. Magnesi
um.

Thiscasei
ll
ustr
atesthepresentati
onofawomanwi t
hper i
partum cardiomyopathy(PPCM).
Advancedmaternalage,
mul t
iparousst
atus,
mult
i-f
etalgest
ation,
Af r
icandescent,hist
oryof
hypert
ensi
on/pr
e-eclampsia,andcocai
neabusehavebeenr eport
edt obepot enti
alr
isk
fact
ors.
 

Magnesium i
samainst
ayoft herapyf
orpre-eclampsia,butnoti
nperi
part
um car
diomyopat
hy.
Furt
hermore,
magnesi
um levelsarenormali
nt hiscase;ther
efor
e,magnesi
um
repl
acement/
suppl
ementati
oni snoti
ndicat
ed. 

Convent
ionalt
her
api
es,
suchasbet
a-bl
ocker
sandACEI
s,shoul
dbei
nit
iat
edoncet
hepat
ient
i
sdeemedhemodynamical
lyst
abl
e. 

Advancedhear
tfai
lur
ether
api
esofl
eftvent
ri
cul
arassi
stdevi
cesandcar
diact
ranspl
ant
ati
on
arepot
entialt
herapeut
icopt
ionsf
orsomepat
ient
swi
thsever
ePPCM wi
thoutr
ecover
yof
vent
ri
cularfunct
ion.

Duet othehighi ncidenceoft hromboembol ism i nPPCM, anti


coagulati
oni sat herapeut i
c
considerati
on, especiall
yduringpr egnancyandi nthefirst6-8weekspost par t
um duet o
hypercoagulabi l
it
ydur i
ngtheset i
meper iods.Dat afrom smallstudieshavedemonst ratedt
hat
additi
onofbr omocr ipti
netost andar dheartfailuret her
apiesmayi mprovel eftventr
icular
ejecti
onfractionandcomposi teclinicaloutcomesi nwomenwi t
hPPCM.Thepost ulated
mechani sm ofact i
oni sthr
oughsuppr essionofpr olacti
n, whi
chi nits16-kDaant i
-angiogeni
c
andpr oapoptot i
cform mayl eadtoendot helialinfl
ammat i
on,impairedcardi omyocyt e
met abol
ism, andr educedmyocar dialcontractioni nPPCM.However ,t
histher apyisnotyet
recommended.

87. A 25- year-old woman pr esentst othe emergency departmentwi th progressively


wor seni
ngdyspneaovert hepastweek.Sympt omswer einit
ial
lyassoci
atedwithaf ever,
rigors,
andnasalcongest ionf ort hefir
st2days.Thepat i
enthasnoknownpr i
ormedi calhistory.
Admi ssi
onel ectrocardiogr am isnor
mal.Thepat i
entisafebril
e.Herheartrat
eis100bpm.Her
bloodpr essur eis90/ 70mm Hg.I nit
ialphysi
calexami nati
onshowedanel evatedjugul ar
venouspr essure,bi l
ateralr al
es,anS3/ S4 
summati
ongal lop,and2+pi t
ti
nglowerext remi ty
edemabi laterall
y. 

Admission echocardi ogram shows a l eftventri


cular( LV) 
ejecti
on fract
ion of20%,gl obal
hypokinesis,novalvul ardisease,andnoper icardi
alef fusi
on.Ri ghtvent r
icul
arfuncti
oni s
normal.I ntr
avenous di ureti
cs arei niti
ated wi t
h minimalr esponse.Wi t
hin 24 hours of
admission,t he pat
ientdevel ops hypot ension and pr ogressive heartfail
urer ef
ract
oryt o
i
notropesandvasopr essors.Thepatienti sref
erredforacut emechani calci
rculat
orysupport
.A
prei
mpl anttwo-dimensi onalechocar
di ogram oftheLVi sshown( Fi
gur e1).

Whichoft
hef
oll
owi
ngper
cut
aneousmechani
calsuppor
tdevi
cesi
scont
rai
ndi
cat
edi
nthi
s
pat
ient
?
Fi
gur
e1

a. TandemHeart.
b. I
ntra-
aorti
cballoonpump( I
ABP)
.
c. I
mpella.
d. Veno-ar
teri
alextracor
por
ealmembr
aneoxygenat
ion(
VA-
ECMO)
.

ANSWER: 

87. c. I
mpel
la.

Thi
spati
enthasanapi
calLVthr
ombusseenonechocar
diogr
aphy.Ofthesuppor
tdevi
ces
l
ist
ed,
theImpell
aispl
acedi
ntheLVvi
aretr
oaor
ti
capproach,andther
efor
e,maycause
embol
ism oft
heLVthr
ombus.

Intr
a-aor
ticbal
looncounterpul
sat
ionpumpsareplacedi
ntothedescendi
ngaor t
aanddispl
ace
bloodvolumefrom thedescendi
ngaort
abyinfl
ati
ngduri
ngdiast
oleanddeflat
ingduri
ng
systol
e.Anabsolutecontr
aindi
cati
ontotheI
ABPismoderateorsevereaort
icregur
git
ati
on.

TheTandemHear tdevi
ceisanextracor
por
ealcent
rif
ugalpumpt hatusestwopercut
aneously
del
iveredcannulastopumpoxygenat edbl
oodfrom t
heleftat
rium t
othesystemicci
rcul
ati
on
vi
at hefemoralarter
y.Forl
efthear
tsuppor
t,t
hedevicerequir
esat r
ans-sept
alpunct
urefor
del
iver
yoft
he21- Fr
enchinf
lowcannul
aandaseparat
ear
ter
ialaccesssi
tefora15-,
17-,or19
-Fr
enchart
eri
alcannul
a.Anabsolut
econt
rai
ndi
cat
iontot
heTandemHear tdevi
ceisl
eftat
ri
al
thr
ombus.

TheImpell
apumpsareaxial
-fl
owcathet
ersthatdi
rectl
ydispl
acebloodfrom t
heLVintothe
ascendi
ngaort
a.Thedevi
cescanbeplacedviafemoralart
eri
alaccessinret
rogr
adefashion
acrosst
heaort
icval
veint
othelef
tventr
icl
e.Anabsolutecont
rai
ndicat
iontotheImpel
lapumps
i
sLVt hr
ombus.

VA-ECMOisplacedvi
aper
ipheralcannul
asinthevenousandar t
eri
alsyst
em.A
cont
rai
ndi
cat
iontoVA-ECMOi ssevereperi
pheralvascul
ardi
sease.
 
Mi
scel
laneous
Quest
ion:

1. A28-year-ol
dmanpr esentst oyourofficeforgeneticcounseling.Hehasaknown
hist
oryofaort
icdi
latati
on,
mi tralvalveprol
apse, andocul arlensdisplacement
.Onexami
nati
on,
younotepectusexcavatum andmar kedjointlaxit
y.Hei saccompani edbyhiswif
ewhois
normalappeari
ngandhasnof ami lyhist
oryofcar diovasculardisease.Theywantt
oknowthe
li
kel
ihoodofthei
rchildr
eninher i
ti
ngt hei
rfather'scardi
ovascul arconditi
on.

Whi
choft
hef
oll
owi
ngi
syourr
esponse?

a. Lowr
isk,
ashi
sdi
seasei
scausedbymul
ti
plegenes,
wit
hgene-
envi
ronment
i
nter
act
ions.
b. Unli
kel
y,ashi
sdi seasei scausedbyanaut osomalrecessivegene.
c. 0%ofhisdaughters, ashisdiseaseiscausedbyanX- l
inkedrecessivegene.
d. 50%ofhischi
ldr
en, ashisdiseaseiscausedbyanaut osomaldomi nantgene.
e. 100%ofhisdaught ers,ashisdiseaseiscausedbyanX- li
nkeddomi nantgene.

Answer
:

1. d. 50%ofhi
schi
ldr
en,
ashi
sdi
seasei
scausedbyanaut
osomaldomi
nantgene.

Thi spat ienthasMar f


ansyndr ome, whichi soneoft hemostcommonMendel i
andi sor ders,
wi thapr eval
enceofappr oximat ely1in5, 000indivi
duals.Marfansyndr omei scausedby
mut ati
onsi nthefibril
li
n-1geneandi si nheri
tedinanaut osomaldomi nantfashion.Ther efore,
50%oft heof f
spri
ngofanaf f
ectedpar entwoul dbeexpectedtoi nheritt
heabnor malgene.
Ot hercar diovasculardiseasest hatdemonst r
ateMendel i
aninher i
tanceincludefami lialdilat
ed
car diomyopat hy,whi chhasdi f
ferentmodesofi nherit
ancebasedont hegenemut ati
on
i
nvol vedbutmostcommonl yisaut osomaldomi nant;f
amili
alhyper tr
ophiccardiomyopat hy,
whi chhasaut osomaldomi nanti nheri
tancecausedbyoneof>900i denti
fi
edmut ati
onsi none
oft he14genesencodi ngcomponent soft hesarcomere;andl ongQTsyndr ome, whi ch
typical l
yshowsanaut osomaldomi nantmodeofi nherit
ancebutwi t
hvar i
ablepenet rance.
Thesedi seaseswi thMendel ianinher i
tanceshoul dbecont r
astedwi thcommon, compl ex
diseasessuchasat heroscleroti
ccar diovasculardisease,whichhasmul ti
plegenesconf erri
ng
risk, wit
honl ymodestef fect,oft
enwi t
hmul t
ipl
egene- geneandgene- envir
onmenti nteract i
ons.

Quest
ion:

2. A27-year-oldwomanpr esent stoyouroffi


cef orgenet
iccounseling.Herf
atherdiedof
anaor t
icdissect
ionatage45year s.Herolderbrotherhasbeendi agnosedwi t
hMar fan
syndrome.Thepat ientisrecent
lymar ri
edandt hi
nki ngofhavingchildr
en.Shedeniesany
histor
yofeyepr oblems.Onexami nat
ion,shehasapect usexcavatum andstri
aeonher
abdomen, butotherwisehasnoscol i
osisorarachnodact yl
yandhasanegat i
vewristand
thumbsi gn.Echocardiogr
am showsnor malaorti
cr ootandascendi ngaortasi
ze,wit
hout
evidenceofmi t
ralortri
cuspidvalveprolapse.

Sheisi
nter
est
edinundergoi
nggenet
icscr
eeni
ngt
oknowwhetherherfut
urechi
ldr
enwoul
dbe
atr
iskf
orMarfansyndr
omeandwantstoknowhowli
kel
ygenet
icscreeni
ngtest
saret
opick
upt
hedi
sease.
 

Your
espondt
hatgenet
ict
est
ingwi
lli
dent
if
yacausat
ivemut
ati
oni
nwhi
choft
hef
oll
owi
ng?

a. Appr
oxi
mat el
y30%ofcases.
b. Appr
oxi
mat el
y70%ofcases.
c. <10%ofcases.
d. >90%ofcases.
e. Appr
oxi
mat el
y50%ofcases.

Answer
:

2. d. >90%ofcases.

Mar fansyndr omei saconnect i


vet issuedi sorderchar act er
izedbycar di ovascular( aorti
c
dil
at at i
onanddi ssect ion,mitralandt ricuspi dvalvepr olapse) ,ocular(lensdi splacementand
retinaldet achment ),andmuscul oskel etal(jointlaxi t
y, l
ongboneover gr owt h,scoliosis,pect us
excavat um orcar inatum)mani f
est ations.Ther evisedGhentscor eprovi desascor i
ngsyst em
tohel pmakeacl ini
caldi agnosi sofMar fansyndr ome.Ourpat i
enthasaGhentscor eof3( 2
point sf orpect usexcavat um and1poi ntforski nst ri
ae) .Whi leshehasaf amilyhi stor
yof
Mar fansyndr ome, herscor eof3i sbel owt hesyst emi cscor eof7neededt odiagnoseMar fan
syndr ome.Mar f
ansyndr omei sinher it
edi nanaut osomaldomi nantfashi on,andi scausedby
mut ationsi nt hef i
brill
in-1extracel l
ul armat ri
xpr ot ei
ngene.Upt o30%ofcasesdonothave
affect edpar ent sandar efel
ttor epr esent  denovo  mut ations.Genet i
ct estingisavai l
able,and
thel ikelihoodoff indingacausat i
vemut ationi sappr oximat ely95%i npat ientswi thMar fan
syndr ome.Genet ictest i
ngisnotr equi redf orconf irmi ngdi agnosi sint hei ndexcase( whichi s
madecl inically), butcanbehel pfulf orot herf amilymember swhoneedt obescr eened.These
resul tscanhel pdet ermi newhet hert heyneedt ohavel ongi t
udi nalcli
nicalmoni tor i
ngor
whet hert heycanber eassuredt hatt heyhavenoti nheritedt hepat hologi cmut ation.

Quest
ion:

3. A35-year-ol
dmanpr esent
stoyouroff
iceforadvi
ceregar
dinggeneticscr
eeni
ng.His
40-year-ol
dbr ot
herwasdi agnosedrecentl
ywithanidi
opathi
cnonischemiccardi
omyopathy.
Hisfatherhadahear tt
ransplantatage48.Hehasr eadontheInt
ernetthatgenet
ict
est
ingis
avail
ableandiswonder i
ngi fiti
scorr
ectforhi
m.

Whi
chofthefol
lowi
ngbestrepr
esent
sthesensi
ti
vit
yforcur
rentgenet
ict
est
ingf
orpat
ient
s
wi
thfami
li
alcar
diomyopat
hy?

a. 10-20%.
b. <10%.
c. 30-40%.
d. >90%.
e. 50-60%.
Answer
:

3. c. 30-
40%.

Fami li
aldilat
edcardi
omyopat hy(FDC)isacl i
nicaldiagnosi swheni di
opathicdil
ated
cardiomyopat hyoccursi natleasttwocloselyrelatedfami lymember s.About80-90%ofcases
areinheritedintheautosomaldomi nantfashi
on.I tisrarelyinherit
edi nautosomalrecessiveor
X-li
nkedf ashion.Mutationsi n>30geneshavebeenf oundt ocauseFDC.I fgeneti
ctesti
ngi s
considered, i
tshoul
dfirstbeper for
medi nthemostcl earlyaffectedper soninthefamil
y.FDCi s
ahet erogeneousgeneticdi sease,wit
hvar i
ablepresent ati
ons, r
educedpenet r
ance,and
dif
ferentmodesofi nheritance.Theknownmut at
ionsaccountf oronl y40-50%ofcases;t hus,
theroleofr outi
negenetictest i
ngisunclear.

Quest
ion:

4. Youar easkedtoconsultona45- year-ol


dmanhospi tal
izedwithaf i
rstepisodeofhear t
fail
ure.Hehasnoknownpastmedi calhi
story.Headmitstoasedent arylif
estyl
e.Exami nati
on
disclosesbloodpressureof118/70mm Hg, el
evatedj
ugularvenouspr essure,clearlungfi
elds,
normal lyspli

S1 
andS2,and2+edemat otheleveloft
hepost er
iorthi
ghsbi l
aterall
y.His
electrocardi
ogram i
sshowni nFigure1.Hisechocardi
ogram demonst ratedconcent ri
clef
t
ventricular(
LV)hypert
rophywithwallthi
ckness1. 5cm andincreasedLVmassi ndex;normal
systolicfunct
ionwithLVeject
ionfract
ionis60%.

Whi
choft
hef
oll
owi
ngi
sthemostl
ikel
ydi
agnosi
s?
a. Hyper t
ensiveheartdi
sease.
b. Athl
ete'sheart.
c. Hyper t
rophiccardi
omyopathy.
d. Cardiacamyl oi
d.
e. Fabry'sdisease.

Answer
:

4. c. Hyper
trophi
ccar
diomyopat
hy.

Hyper t
rophi
ccardi omyopat hymustbedi ff
erentiatedfrom inf
ilt
rativeandmet aboliccondi t
ions
thatalsocancausel eftvent r
icularwallthickening, suchasFabr y'sdi seaseorPompe' s
disease.Thi
spat ienti
sol derthanexpect edf orpresentati
onf orFabr y'sdisease, whi chtypical
ly
presentswithneur ol
ogicsignsandhear tdiseasei nchi l
drenandadol escents.Thedi agnosis
woul dbemadewi thalphagal actosidaseact ivit
yandendomyocar dialbiopsy.Amyl oidosis
woul dbeexpectedt ocausenor malorl owvol t
ageont heelectrocar diogram witht hi ckened
wallsonechocar diographyandwoul dbeconf i
rmedwi thmyocar dialbiopsy.At hlete'shear t
changescani ncludewal lthickeningandi ncreasedchambersi zeandi sseeni nhi ghl ytrai
ned
athlet
es,whichthispat i
entisnot .Thispat ientdoesnothaveahi storyoforcur rent
hypertensi
ontoexpl ai
nhyper tensivehear tdisease.

Quest
ion:

5. A22- year
-oldcoll
egeat hleteisbeingeval uatedafteranepisodeofsyncopeonhi sway
toclass.Hehasnosi gnif
icantpastmedi calhist
or yanddeni estobacco,drug,andsoci al
alcoholuse.Hei sonnomedi cat i
onsanddeni esuseofanyher balornutri
tionalsupplement s.
Hisfami lyhistoryi
sunr emar kabl e.Hisexami nationreveal
sbl oodpressureof110/ 68mm Hg,
heartrate66bpm, bodymassi ndex21, andbeni gnexami nat i
on.Echocardiogram r
eveals
normall eftventri
clesizeandt hickness,normalr ightventr
icle,andnovalvularabnormal i
ties.
Elect
rocar diogram showssi nusr hyt
hm andQTi nter
valof492msec.Toxi cologyscreeningi s
negative.

I
naddi
ti
ont
oini
ti
ati
onofbet
a-bl
ocker
s,whi
choft
hef
oll
owi
ngi
smostappr
opr
iat
einhi
scar
e?

a. Car diacmagnet icresonancei


maging.
b. Electrophysiologi
cst udy.
c. Impl antablelooprecorder.
d. Ge neticscreening.
e. Impl antati
onofi mplantabl
ecar
diover
ter
-def
ibr
il
lat
or.

Answer
:

5. d. Genet
icscr
eeni
ng.

Geneticanalysisisimportantt
oidentif
yal ongQTsyndromemutati
ontoal
lowforri
sk
str
atif
icat
ionandt herapeuti
cdecisionsandt oeval
uat
efamil
ymemberstoident
if
ysilent
carr
iers.I
mpl antablecar
diover
ter
- defi
bri
ll
atori
mplant
ati
onshouldber
eser
vedforthosewho
hadpr evi
ouscar diacarr
est(Cl
assI )andcanbeconsideredi
nthosewhohavesyncopeor
ventri
culartachycardi
awhi l
eonbeta-bl
ockers.Hehadnotbeentreatedyetwithbet
a-bl
ockers.
Patientsundergoingmoni t
ori
ngshouldhaveexter
nalmonitori
ngpriort
oimplantabl
eloop
recorders.El
ectrophysi
ologi
cstudyandmagnet i
cresonanceimagingarenotindi
catedatthi
s
ti
me.Thi spatientalsoshouldunder
golif
estyl
echanges,i
ncludi
ngavoidanceofcompetit
ive
sportsandavoi danceofdrugsthatprol
ongQT.

Quest
ion:

6. A45-year
-oldwomanpr esentstoyouroff
iceforadviceregar
dingherri
skofdeepvein
thr
ombosis(DVT).Herbrot
herhadaDVTandwasdi agnosedashavi ngfact
orVLeiden.She
wantstoknowherriskofhavi
ngt hediseaseandwhet hersheshouldbetaki
nglif
elong
anti
coagul
ati
on.

Youadvi
set
hatf
act
orVLei
deni
swhi
choft
hef
oll
owi
ng?

a. X-l
inkeddominantwit
hlowlif
eti
mer i
skofDVT.
b. Aut
osomalrecessivewit
hhighli
fet
imeriskofDVT.
c. Aut
osomalrecessivewit
hlowlif
eti
mer i
skofDVT.
d. Aut
osomaldomi nantwit
hlowlif
eti
mer i
skDVT.
e. Aut
osomaldomi nantwit
hhighli
fet
imeriskofDVT.

Answer
:

6. d. Aut
osomaldomi
nantwi
thl
owl
if
eti
mer
iskDVT.

FactorVLei denisanaut osomaldomi nantgeneticdisorderinwhi chamut at i


onoff act
orV
cannotbei nacti
vatedbyact i
vatedpr otei
nC, r
esult i
nginani ncreasedriskoft hrombosis.
Therefore,herri
skofhavi ngthemut ationis50%.Whi l
eappr oxi
mat ely20%ofpat i
entswithDVT
willbefoundt ohavefactorVLei den,theli
feti
mer iskofDVTi napat i
entwi thfactorVLeidenis
onlyappr oximat
ely5%.Thepr evalenceoft hefactorVLei denmut at
ionisappr oximatel
y5%i n
CaucasianAmer i
cans,2%i nHispanicAmer i
cans, 1%inbot hAfricanAmer icansandNat i
ve
Amer i
cans, and0.5%i nAsi anAmer i
cans.Whi lethispatientshouldnotbepl acedon
prophylacticant
icoagulation,l
if
est yl
emodi fi
cation,suchasmai ntai
ningnor malbodywei ght
,
avoidanceofsuppl ementalest r
ogen, andavoi danceoft obaccopr oducts,shouldbe
encouraged.

Quest
ion:

7. A78- year-
oldmani sadmi t
tedt othecoronarycar euni taft
erpr esenti
ngwi t
hanacut e
anter
iormyocar di
alinfarcti
on.Heunder goessuccessf ultreatmentofapr oximall ef
tant eri
or
descendingocclusi
onwi thadr ug-eluti
ngst ent
.Af ol
low- upechocar diogram r evealsant er
ior
andapicalhypokinesis,withanej ectionfracti
onof40%.Hi spastmedi calhistoryisnot ablefor
hypert
ension,hyperl
ipidemi awithhismostr ecentlow-densi tyli
poprotein120mg/ dl,r
ecurrent
ki
dneyst ones,andpr i
orgast r
ointestinal(
GI )bl
eedingfrom duodenalul cers.I
naddi t
iont o
conti
nuinghishomemedi cati
onsofaspi r
in81mg, atorvastatin40mg, andallopur i
nol,his
l
isi
noprilisi
ncreasedt o20mgdai l
y,andhei sstar
tedoncl opidogrel75mgdai l
y.

Whichofthefol
lowi
ngi
scor
rectr
egar
dinghi
str
eat
mentf
oll
owi
ngper
cut
aneouscor
onar
y
i
ntervent
ion(
PCI)?
a. Routi
negenet i
ctestingforclopi
dogr
elmetabol
ism i
srecommended.
b. Cl
opidogrelshouldbechangedt opr
asugr
el.
c. Prot
onpumpi nhi
bitor(PPI)ther
apyshoul
dbeadded.
d. Pl
atel
etfuncti
ont estingisrecommended.
e. Aspi
rinshouldbei ncreasedto325mgdaily.

Answer
:

7. c. Pr
otonpumpi
nhi
bit
or(
PPI
)ther
apyshoul
dbeadded.

In2010, theFoodandDr ugAdmi nistrati


onappr ovedanewl abelf orclopidogr elwi t
haboxed
war ningaboutt hedi mi ni shedef fectivenessofcl opidogr elinpat ientswi thimpai r
edabi lityto
conver tt hedr ugintoi tsact ivemet abol i
te.Pat ientswi thdecr easedcyt ochromeP4502C19
(CYP2C19)f unctionbecauseofgenet icpolymor phi smsmet abol izecl opidogr elpoor l
yandhave
higherr at esofcar di
ovascul arevent saf teracut ecor onar ysyndr omeandPCIt hanpat ient swith
normalCYP2C19f unct i
on.Thewar ningal sonot est hatt estsar eavai lablet oi dentifypat ients
withgenet i
cpolymor phi smsandt hatal ternat ivet reat mentst rategiesshoul dbeconsi der edfor
patient swhoar epoormet abolizers.Thecl opi dogr elboxedwar ningl eavest hei ssueofwhet her
toper form CYP2C19t est i
ngupt ot hei ndividualphysi cian.Itdoesnotspeci fical l
yrequi re
genet i
ct est i
ngorot herchangesi neval uationort reat mentanddoesnoti mpl yt hatt herear e
solidevi dence- basedr easonsf orsuchact ions.Rat her ,itservest oinform cl iniciansofgenet ic
variationsi nresponset ocl opidogr elandt oemphasi zet hatcl i
niciansshoul duset his
knowl edget omakedeci sionsabouthowt ot r eati ndi vidualpat i
ent s.Atthepr esentt ime, the
evidencebasei sinsuf ficienttor ecommendr out inegenet ictestingi npat ientsunder goi ngPCI .
The2011ACCF/ AHA/ SCAIPCIgui del i
net her ef oregi vest her outinecl i
nicaluseofgenet i
c
testingf orclopidogr elmet abolism aCl assI IIindi cat ion( nobenef it).Similarly, routi
nepl atel
et
funct i
ont est
ingal soisnotr ecommended.  

TheCOGENTt rialrandomi zedpat i


entswi t
hdualant iplatelettherapy( DAPT)t ocl opidogreland
omepr azol
eorcl opidogrelandpl acebo.Althought herewasnodi f
ferencei ncar di
ovascul ar
event sbetweent het wogr oups,GIevent swer ehalvedi nt hoser andomi zedt oomepr azole.Iti
s
reasonabl etocar eful
lyevaluatetheindicationforPPIt her apyinpat ientstreatedwi th
clopidogrelbasedont hepr esenceorabsenceoft her i
skf actors.Theneedf orGIpr otection
i
ncr easeswi t
ht henumberofr i
skfactorsforbleeding.Pr iorupperGIbl eedingi sthest rongest
andmostconsi stentri
skfact orf
orGIbl eedingonant iplatelettherapy.Pat ientswi t
hacut e
cor onarysyndromeandpr iorupperGIbl eedingareatsubst antialcar di
ovascul arrisk,soDAPT
withconcomi t
antuseofaPPImaypr ovidetheoptimalbal anceofr iskandbenef i
t.Prasugrelis
relati
velycontraindicatedandhasabl ackboxwar ningi npat ientsabovet heageof75year s, a
priorhistoryofstroke,orwi thweight.Aspirinatdoses>100mgi snotr ecommendedaspar tof
DAPTf oll
owingPCI .

Quest
ion:

8. Upto30%ofbi cuspidaorti
cvalve(BAV)patientswil
lhaveafirst
-degr
eerelati
vewit
h
associ
atedaort
opathy.Thisincl
udescoar ctat
ionoftheaort
aoranascendi nganeurysm.At
ti
mes,theaort
opathymaybedet ectedindependentofthepresenceofaBAV.

Whichoft
hefoll
owi
nggenet
icabnor
malit
iesi
srecognizedasbei
ngr
esponsi
blef
ort
he
devel
opmentofaBAVandit
sassoci
atedaort
opathy?
a. Transf
orminggrowthfactor
-beta1(TGF-
beta1)mutat
ion.
b. NOTCH1ge nemut at
ion.
c. Noge nehasbeenidenti
fi
ed.
d. Absenceofanabnormal i
tyofoneoftheXchromosomes.
e. Fi
bril
li
n-1mutat
ion.

Answer
:

8. c. Nogenehasbeeni
dent
if
ied.

Thecor rectansweri sthatnogeneti


cmut ati
onhasbeeni dent
ifiedtodat e.Whileitisknown
thatNOTCH1mut ationsareassoci
atedwi thearlydevelopment aldefectsintheaor ti
cvalveand
possiblyaBAV, thepr esenceofthi
smut ationalsohasbeenl i
nkedt ohypopl asti
cl ef
theartand
tocalcium deposi ti
oni nboththeaorti
cval veandaor ta.Thefi
br il
li
n-1mut ati
oni sassociated
withtheMar f
ansyndr ome.Theabsenceoranabnor mal i
tyintheXchr omosomei sassociated
withTur ner'
ssyndr ome.Aorti
caneurysmsmaybepr esentinbot htheMar fanandTur ner'
s
syndrome.Tur ner'ssyndromepat i
entsfrequentl
yhaveabi cuspi dvalveaswel lasaor ti
c
coarctation.TheTGF- beta1genemut at
ioni sassociatedwithLoeys- Dietzsyndr omeand
multipl
eaor ti
caneur ysms.

Quest
ion:

9. An18- year-ol
dmani srefer
redtoyouroffi
ceforaf ami
lyhist
oryofhyper
trophi
c
cardi
omyopat hy(HCM) .Heplayshighschoolsport
s,isasymptomatic,hasnoothermedical
hist
ory,andhasaf amilyhi
storyofHCM.Hi sfat
herwasdi agnosedwithHCM atage30year s,
andatage45year sisasymptomaticonabet a-bl
ocker.Thepati
enthasnoheartmur murat
restorwithValsal
va,andhiselectr
ocardi
ogram isnormal.Yourecommendgenet ict
esti
ng.

Thei
nter
valf
orcl
ini
calscr
eeni
ngf
orHCM i
nthi
spat
ientdependsonwhi
choft
hef
oll
owi
ng?

a. I
ndependentofgenet i
ctest
ing.
b. Predi
ctedri
skofsuddendeat h.
c. Genotypeofthepatient
.
d. Phenotypeofthefather.
e. Genotypeofthefather.

Answer
:

9. c. Genot
ypeoft
hepat
ient
.

Hyper trophiccardiomyopat hy( HCM)i scausedbyanaut osomaldomi nantmut ati


oni ngenes
encodi ngsar comer epr oteinsorsar comere-associ
atedpr oteins.Geneticorcl i
nicalscreening
ofal lfirst-
degr eefami l
ymember sofpat i
entswithHCM i sindicatedinthosewi thunrecognized
disease.Rel ati
veswi thover tHCM wi l
lhavethesamepat hogeni cHCM mut ati
onast heindex
pat i
ent .Thesemut ationcar r
iersshouldbeevaluatedbyphysi calexaminat i
on,
electrocar diography, andt wo- dimensionalechocardi
ography.I fHCM isi dentif
ied,these
i
ndi vidual sshoul dunder gor i
skstrati
fi
cati
on.Genetictesti
ngpr ovi
desdef i
niti
vedi agnosisof
affect edgenet icstatus, andcur rentl
yisusedmostef fecti
velyi ntheidentif
icati
onofaf fected
relativesi nfami l
iesknownt ohaveHCM.Mut at
ion-negati
vef ami l
ymember sandt heir
descendant shavenor iskf ordevelopingHCM anddonotneedf urt
hereval uati
on.The
phenotypi
cexpr
essi
onofHCM isvar
iabl
e,evenamongf
ami
lymember
swi
tht
hesame
genotype,
anddoesnoti
nfl
uencescr
eening.

Quest
ion:

10. A47- year-ol


dmanwi t
ht hehumani mmunodef ici
encyvirus(HIV)presentswithan
acut eanteri
ormyocar diali
nfarction.Hehasnoot hersi gnif
icantpastmedicalhist
ory.Heison
highlyact i
veantir
etr
oviraltherapy, butnoothermedi cat i
ons.Heunder goesasuccessf ul
per cutaneouscoronaryinterventionofhisproximallef tanteri
ordescending.Hislaborator
y
valuesatt hetimeofpr esentati
oni ncludeHbA1cof5. 9%, CD4countof330, andadet ectable
viralload.Totalchol
esterolis195mg/ dl
,l
ow- densit
yl ipoprotei
ncholester
olis110mg/ dl
,hi
gh
-densi tyl
ipoprot
eincholesterolis40mg/ dl
,andt r
igl
ycer idesare225mg/ dl
.

Whi
choft
hef
oll
owi
ngi
sthemostl
ikel
yexpl
anat
ionf
orhi
sacut
eevent
?

a. Anti
ret
rovi
ralplat
eletacti
vat
ion.
b. Vi
ralmyocardialt
oxicit
y.
c. Vi
ralendothel
ialt
oxicit
y.
d. Vascul
arinfl
ammat ion.
e. Anti
ret
rovi
ral-
inducedvasospasm.

Answer
:

10. d. Vascul
ari
nfl
ammat
ion.

Car di
ovascularcompl icationsar emor ecommoni nHI V- i
nfectedindivi
dualsthani nage-
mat cheduni nfectedindividual s.Ant i
ret r
ovir
alther apyr educest heriskofcardiovascul ar
compl icati
ons, suggest i
ngt hatvi ralreplicat
iondi rectlyori ndi
rect
lycausesvascul ardisease.
Long- term ef
fectiveant i
retrovi ralther apydoesnotf ul
lyr estor
evascul arheal
th, andtreated
adul t
scont i
nuet ohavehi gher -than- expectedr atesofdi seaseprogression.Althought his
excessr iskdur i
ngt herapyi sl ikelyduet omul ti
plef actors, agrowingbodyofevi dence
suggest sthatchr onicinflammat ion, whichper sistsdur ingef f
ecti
veant i
ret
roviralther
apy, i
s
directl
yandcausal lyassoci atedwi thvascul ardysf unctionandt heaccel er
ateddevel opmentof
atherosclerosi
s.LowCD4count s, asevi dencedbyt heD: A:Dstudy,arear i
skf orcardiovascul
ar
diseasei nHIVpat i
ents.Vi remi aaf fectst hemyocar dium andcancausecar diomyopat hy,but
woul dnotbeexpect edt ocauseat hrombot i
ceventorvasospasm.

Quest
ion:

11. A55- year-


oldmanwi t
hthehumani mmunodefi
ciencyvir
us(HIV)pr
esentsfor
eval
uat
ionofhi
sriskofcor
onaryar
ter
ydisease.Hehasneversmokedandhi stot
alchol
est
erol
i
s134mg/ dl
.Hedoesnothavehypert
ension.Heexer
cisesregul
arl
y.

Whi
choft
hef
oll
owinghasbeenshowntoi
mprovecardiovascul
arr
iskpr
edi
cti
oni
npat
ient
s
wi
thHI
V,i
naddi
ti
ontotradi
ti
onalFr
ami
ngham r
iskf
actors?

a. Durat
ionofdiabet
es.
b. CD4c ounts.
c. Useoffusioninhi
bit
ors.
d. Useofintegr
aseinhibi
tor
s.
e. Tr
igl
ycer
idel
evel
.
Answer:

11. b. CD4count
s.

TheFr ami ngham modelforglobalcar diovasculardisease( CVD)r i


skmaysi gnif
icantl
y
underest i
mat et
helikel
ihoodofCVevent sinHI V-inf
ectedpat ient
s.Abet t
ermodelhasbeen
deri
vedf rom theD:A:D(DataCol l
ect i
ononAdver seEvent sofAnt i-
HI VDrugs)study, whi
chalso
consider sCD4cellcountsandadmi nist
rati
onofant iret
roviralagents.TheD: A:
Dmodeli s
basedonobser vati
onsin32,663HI V-positivepersonsf rom 20Eur opeancount ri
esand
Australi
awhower efreeofCVDatst udyent ryandwhopr ovidedthoroughi nfor
mat iononCV
ri
skfact ors.Therat
eofacomposi teCVDendpoi nt,
compr i
singmyocar dialinf
arcti
on(MI),
str
oke, coronaryart
eryrevascularization,caroti
dendar terectomy,oranyCVDdeat h,was
assessedover186, 364.5person- years. 

FullandreducedD:A:Dmodel
swer edevelopedtocalcul
ateesti
mat ed5-yearrisksofCVD.
Therewer e1,
010compositeCVDevents,givi
ngarateof5.42per1, 000person- years.Near
ly
one-halfoft
heeventswereMIs(493/1010),f
oll
owedbyst roke(295/1010),angioplasty
(129/1010),
coronaryar
ter
ybypass,carot
idendart
erectomy,andot herevents.
 

Thef i
nalfullD:A:Dmodeli ncludedtradi
ti
onalCVDr iskf
actors,
aswel lasCD4counts,
combi nati
onant ir
et r
oviraltherapy(cART),cumulat
iveexposuretoprotease-andnucleosi
de-
reverse-t
ranscriptasei nhibit
ors,andcurrentuseofabacavir
.Othersingl
e-classt
herapi
eswere
notassociat edwithi ncreasedr i
sk.Thereducedmodeldoesnoti ncludeuseofART.Dur at
ion
ofHIVi nf
ectionisassoci atedwithhigherri
skofeventsinthi
sstudy.Dur at
ionofdi
abeteswas
evaluatedint hi
sst udy.

Quest
ion:

12. A32- year-oldunrestrainedpassengerisbroughtt otheemer gencyr oom f


oreval uat
ion
aft
eranaut omobi l
eaccident .Heisaler
tandor i
ented, andhei scompl ai
ningofleftlegpai nand
chestpainafterairbagdepl oyment .Hi
sbloodpressur e145/ 75mm Hg, andhi shear trate105
bpm.Lungsoundsar eclearandequalt hr
oughout .Cardiovascularexam reveal
sdi stanthear t
soundswi t
houtr ub.Otherfindingsarechestwallwi t
hmi lderythemaandear l
ybruising, anda
ri
ghtfemurwi thobviousdef ormityandnormalper ipheralpulses.ChestX- r
ayrevealscl ear
l
ungf i
elds,normalmedi asti
num, andfr
actureofther i
ght10thand11t hri
bs.

Whi
choft
hef
oll
owi
ngi
sthebestnextst
epi
nthecar
diaceval
uat
ionoft
hispat
ient
?

a. Evaluatet
roponinandmyogl obi
n.
b. Transesophagealechocardi
ogram.
c. Cardiaccomput edtomographyangi
ogr
aphy.
d. Transthor
acicechocardiogr
am.
e. 12-leadelect
rocardi
ogram

Answer
:

12. e. 12-
leadel
ect
rocar
diogr
am

Thor
aci
cinj
uri
esar
ethet
hir
dmostcommoni
njur
iesi
npol
ytr
aumapat
ient
s.The12-
lead
electrocar di
ogram ( ECG)i sthebestnextst epinthemanagementoft hepat i
entt oruleout
evidenceofmyocar dialinjur
y.Al thoughECGchangesar enonspeci f
icandar enotr egardedas
ast r
ongi ndicatorofmyocar dialcontusioni ntr
aumacases, t
heycani ndi
cat ecardiac
i
nvol vementandcer t
aincompl i
cati
ons,suchasr uptureofthecoronaryar t
eriesleadingto
massi vemyocar di
alinf arcti
on( MI )
.Amet a-analysisbyMaenzaandcol l
eaguesshoweda
concor dancebet weeni mpor tantcardiaccompl i
cat i
onsandabnor malECGf indings.Cardiac
contusi onsanddi rectinjurywi thdissectionoft hecor onaryarter
iesandsecondar yMImustbe
routi
nel yconsi deredi naddi tiontoacut eMI .Alongwi t
htypicalaccompanyi nginjuries,
arrhythmi asof t
enoccuri nconj uncti
onwi thcar diaccontusions.Attenhoferandcol leagues
foundECGchangesi n32%ofpat ientsafterbluntt horaci
ctrauma, pri
marilytachycar di
a,ST
segmentchanges, andbundl ebr anchbl ocks. 

Myogl obinisexpect edtobeabnor mal.Determinati


onofmyocar dialmar kersincasesofsever e
thoracictraumadoesnotper mi tanyval i
ddiff
erenti
ati
onoftheet iol
ogy.Car diacenzymel evel
s
serveasani ndicat
orofcar diacpat hology,thoughindivi
duallaboratoryvaluesar eless
signif
icantindet er
mi ni
ngadi agnosist hanthepat i
ent
'scourseandcl inicalconditi
on.
Echocar di
ogr aphycanbeusef ultoevaluatewallmot i
onabnor malit
ies,pericardi
alinvolvement,
andaor ta,buttheECGshoul dbeper formedf i
rst.Si
milar
ly,
car di
accomput edt omogr aphy
angiographycanbeusef ultoeval uat
ecor onaryvesselsandaor ta,butECGshoul dbe
performedf ir
st. 

Quest
ion:

13. A68- year-


oldwomanwhohasbeenondi al
ysisf
ormanyyear sisadmi tt
edfr
om home
wit
hdi zzinessandhypotensi
on.Sheunder wentherrout
inedi
alysisyester
day.Shehadbeen
welluntil3hour spri
ort
oadmi ssi
on,whenshehadashaki ngchillandcomplainedof
headache, butonlyvaguechestdiscomfort
.Shehaslong-st
andingdiabetes,butnoprior
cardiachistoryexceptf
orl
eftbundlebranchblock(
LBBB)onherel ectr
ocardi
ogram (ECG).She
i
sal ertbutanxiousandmildl
ydyspneic. 

Herbloodpr essureis85/60mm Hg, temperat


ur eis38. 2degr eesCent i
grade,andhear tr
ateis
103bpm andr egular
.Herlungsrevealafewmi norcr acklesbilaterall
y.Hercar diac
examinationrevealsthefuncti
oni
ngshunti nt
hel eftarm, mi l
dj ugularvenousdi stenionwi t
ha
posit
ivehepatojugularref
lux,anS4 
andS3,butonlyagr ade1/ 6aor ticfl
owmur mur .Thereisno
edema, butshei scoolandclammy.AnECGi sobt ainedr eveali
ngt heLBBB, butnonew
changessi ncetheECGobt ai
ned1yearago.Theacut emyocar dialinfarct
ion(MI)pr otocoli
s
acti
vated,andyouar easkedt oconsideremergentcar diaccat heteri
zation.

Whichoft
hefoll
owi
ngwoul
dmostl
ikel
yresul
tint
heneedf
orur
gentcar
diaccat
het
eri
zat
ioni
n
t
hissit
uat
ion?

a. Elevat
edcreati
ninekinase-
myocardi
alband.
b. Ne wwallmotiononherechocardi
ogram.
c. Elevat
edD- di
mer .
d. S3 gal
loponexami nat
ion.
e. Elevat
edtroponi
nT.
Answer
:

13. b. Newwal
lmot
iononherechocar
diogr
am.

Thecor rectansweri snewwal lmotionont heechocar diogr am.Shei sf ebr i


leandot hercauses
ofherhypot ensionandshockshoul dbequi ckl
yexcl udedpr i
ortoconsi der ationf orcardi
ac
catheteri
zat i
onandcor onaryangi ography.Herpr esentationi spossibl yr elatedt osepsisor
shock, andyourdeci sionmustt henr elyonwhet herthishasl edtoaMI .Shei scol dand
cl
ammy, suggest i
ngt hecausei scardi ogenic.Thet r
oponi nTandcr eat inineki nasemaybe
el
evatedi npat ientsondi al
ysisandcanbeconf usingint hisset t
ing.Theel evat edwhi t
eblood
countismor el
ikelyassoci atedwi thheri nfecti
ouspr ocesst hanapot ent i
alMI .TheS3 gall
op
suggest sleftventri
cul ardysfunction, butdoesnothel pdi ff
er ent
iateacauseofherhypot ension
i
nt hi
sset t
ing.Theechocar diogram sept alwallmot i
onmaybeaf fectedbyt heLBBB, butthe
presenceofanewwal lmot i
onabnor mal it
yisofconcer nandi sthest rongesti ndicati
onofa
myocar dialeventr equi r
ingur gentintervention.

Quest
ion:

14. A75- year


-oldwomanpr esent st othehospi t
alafter2- 3daysoff eel i
ngunwel l,
fati
gue,
nausea, andvomi ti
ng.Herbl oodpr essur ei
s70/ 40mm Hgandt heelectrocar diogram
demonst r
atedQwavesandSTel evationsinleadsV1- V4 
andST- segmentdepr essionsi nthe
i
nf er
iorleads.Sheistakent ot hecar diaccathet eri
zati
onl aborat
or yandf oundt ohavea
thrombot i
cocclusionofthepr oximall eftant
eriordescendi ngartery.Percut aneouscor onary
i
nt er
ventionisperformedwi thpl acementofadr ug-eluti
ngst ent.Ventr
icul ography
demonst r
atesanteri
orhypoki nesi sandapi caldyskinesiswi thanej ect
ionf ractionof35%on
dopami ne.Laterthateveningt hepat i
entbecomessuddenl ypale,diaphor etic,andi ncreasingl
y
hypotensive.Anewsyst olicmur muri shear datt heleftl
owerst ernalborder .Nor epinephrineis
start
ed.Ast atechocardiogram i sor dered.

Whi
choft
hef
oll
owi
ngi
sthemostl
ikel
ycauseofhersuddendet
eri
orat
ion?

a. Acut
emi t
ralregurgi
tati
on.
b. Lef
tventr
icul
arpseudoaneur ysm f
ormat
ion.
c. Vent
ri
cularseptaldefect
.
d. Myocardi
alfreewallruptur
e.
e. Ri
ghtventri
culari
nfarcti
on.

Answer
:

14. c. Vent
ri
cul
arsept
aldef
ect
.

Whi l
elesscommont hani npriorer as,mechanicalcompl i
cationsaf t
erat r
ansmur almyocar di
al
i
nfarction(MI )sti
lloccur ,
especi allyinthosewithal ate-presentingMI .I
nt hi
spatient,sudden
hemodynami cdet eri
or ati
oni ntheset ti
ngofal ate-presenting,leftanter
iordescending(LAD) -
rel
atedMIwi thanewwel l-heardmur murismostconsi stentwi thanacut eventr
icularseptal
defect.Whi l
eLADi nf
ar cti
oncanbeassoci atedwi t
hacut emi tralregurgi
tati
on(MR) ,the
mur murofacut eMRi squitedifficulttoheargivenrapi dequal i
zationofchamberpr essures,
andt hislesi
oni susual lyassociat edwi t
hflashpulmonar yedemaandhypoxemi a.Inaddi t
ion,
ther
ei sdualbl
oodsuppl
ytot
heanterol
ater
alpapi
ll
arymuscle,whi
let
hepost
eromedi
al
papil
larymusclehasasi
ngl
ebloodsupplyf
rom whateverar
ter
ysuppl
iest
heinf
eri
orwal
l.
 

Freewal lr
upt
ureismostcommonl yassociatedwit
hlate-presenti
nglef
tci
rcumflex-
relat
edMI,
al
thoughi tmaybeassoci at
edwi t
hinfar
cti
oni nanyterri
tor
y.Thisisusual
lyacatastr
ophic
compl i
cati
onassociatedwitheff
usiononechocardiogram, unl
esscontai
nedbyt he
pericar
dium.Rightventr
icul
arinf
arcti
onisaconsequenceofpr oxi
maltomidrightcoronary
arteryoccl
usi
onandwoul dnotsuddenlypresenti
nal atefashion.

Quest
ion:

15. A72- year-oldmanwi thahi st


oryofcor onar yar t
erydisease(CAD) ,pri
orper cutaneous
coronaryi nt
ervention( 16mont hsago)onaspi ri
n81mgdai l
yandcl opidogrel75mgdai lywas
admittedt ohospital.Hehasnochestpai n.Hehaschr onicatri
alfi
bril
lati
on(AF)forwhi chhe
takeswar fari
n.Hewasadmi t
tedfordizzi
nessandf oundt ohaveanacut egastrointesti
nal
bleedandhypot ension.Hi swarfari
nandbet a-blockersar eheld.Heisr esuscit
atedwi thbl ood
productsandf r
eshf rozenplasmai sadmi ni
stered.Anur gentendoscopydemonst r
ateda
gastri
cul certhatiscaut eri
zed.Atthet i
meofadmi ssionhewast achycardicandanemi c.An
elect
rocar di
ogram ( ECG)demonst ratedAFwi t
har apidvent r
icul
arresponseandi nferolateral
downsl oping1.5mm ST- segmentdepr essions.Att het i
meofadmi ssion,atroponinTi s
checkedandr eturnsat0. 08ng/ ml.Sixhoursl aterthet r
oponi nTreturnsat0.13ng/ ml( cut-of
f
foracutemyocar dialinfar
cti
oni s0.1ng/ml ).

Whichoft
hef
oll
owi
ngi
sthebestnextbestst
epi
nthecar
diovascul
armanagementoft
his
pat
ient
?

a. Resumebeta-bl
ocker
s.
b. Pr
oceedwi t
hcoronar
yangiography.
c. Resumeaspiri
nandclopi
dogrel.
d. I
niti
ateahepari
ndri
p.
e. Resumewar f
ari
n.

Answer
:

15. a. Resumebet
a-bl
ocker
s.

Thispatientl
ikel
yhasat ypeIImyocar dialinfarction(MI )duetot heacut
ebl oodloss,
hypotension,andt achycardi
aincreasi
ngmyocar dialoxygendemandanddecr easi
ngoxygen
deli
very.Hehasnochestpai n.Thispr esentationi snotconsi stentwit
hat ypeIMI( acute
coronarysyndrome) .Theut i
li
tyofchecki ngcar diacbi omarkersint hi
ssetti
ngwher etherei
sa
cl
earet i
ologyforthepat i
ent'
ssympt oms( acut ebloodl oss,hemodynami call
ysignifi
cant
bleedi
ng)i sl
ow, yetfrequent
lyresult
si ncar diologyconsul tat
ion. 

Themosti mpor tantconsiderat


ioni storestoreadequateoxygenat iont othemyocar di
um
throughbl oodr esusci
tati
on,and, oncest able,
resumptionofr atecont rolmedicationsforat
rial
fi
brill
ation.Asthistroponinelevationisnotr el
atedtoacutepl aquer upture,thereisnoutil
it
yf or
i
ni t
iati
onofant icoagulati
onorcor onaryangi ography.Whileresumpt ionofaspi rin81mgdai ly
i
sappr opriat
eaf t
ergastroint
estinal(GI)i
nterventi
on,giventhepat ient'
sr equir
ementf or
war far
in,theclopidogrelshouldbedi scontinuedgivenhisrecentGIbl eedandl engthoftime
sincehi sstentplacement .Whilewar f
arin(ortarget
edant i
coagul ants)shoul deventuall
ybe
r
esumedforst
rokeprophyl
axi
s,t
heper-dayriskofst
rokeissmallanditwouldber
easonabl
e
t
oholdr
esumptionofanti
coagul
ati
onuntili
tiscl
earthepati
enti
snol ongerbl
eedi
ng.

Quest
ion:

16. A42- year


- ol
dwomani sbeingtreat
edwi t
hchemot herapy,i
ncludingdoxor ubicin,f
or
recentlydiagnosedmet astat
icosteosarcoma.Asidefrom legpainatthesi t
eofhert umor ,she
hasbeenwel landhasnosympt omswi thphysicalexert
ion.Shehadbeenexer cisi
ngr egularl
y
untilherrecentdiagnosis.Heroncologi
stperf
ormedabasel i
neelectr
ocar di
ogram thatwas
normal ,aswellasamut li
gat
edacqui st
ionscanthatshowedal ef
tventri
cularejecti
onf racti
on
of65%.

Whichofthefol
lowi
ngstagesofhear
tfai
lur
e,byAmer
icanCol
legeofCar
diol
ogy/
Amer
ican
HeartAssoci
ati
oncri
ter
ia,
doesshehave?

a. St
ageE.
b. St
ageA.
c. St
ageD.
d. St
ageC.
e. St
ageB.

Answer
:

16. b. St
ageA.

Whil
ethispat
ientcurrentl
yhasanor malhear
t,shei
sbeingtr
eatedwit
hanant hr
acycl
ine
chemother
apeuticagent.Exposuret
opotenti
all
ycar
diot
oxiccancert
herapyagent
sis
cl
assi
fi
edasst ageAhear tfai
lur
e(HF).

St
ageA:AthighriskforHF,butwit
houtstruct
uralheartdi
seaseorsympt
omsofHF.
St
ageB:Str
ucturalheartdi
sease,
butwithoutsignsorsympt omsofHF.
St
ageC:Str
ucturalheartdi
seasewithpr
iororcurrentsymptomsofHF.
St
ageD:Refr
actoryHFr equi
ri
ngspecial
izedinter
venti
ons.

Ther
eisnoAmer
icanCol
legeofCar
diol
ogy/
Amer
icanHear
tAssoci
ati
onst
ageE.

Quest
ion:

17. A42- year-


oldwomani sevaluatedforacutelef
tlowerextremit
yischemia.Herpast
medi
calhist
or yi
ssigni
fi
cantforhypert
ensionandchr onicmyelogenousleukemia.Her
medi
cati
onsi ncl
udeamlodipi
ne,hydrochlorot
hiazi
de,ethi
nylestr
adiol
/norgest
rel
,andnil
oti
nib.
Sheunder
goessuccessfullef
tfemoralarter
yembol ectomywithrestor
ationoffl
ow.

Whi
choft
hef
oll
owi
ngi
sthemostl
ikel
ycont
ri
but
ingf
act
ort
othedevel
opmentofhert
hrombus?

a. Oralcontracepti
ve.
b. Tyrosinekinaseinhibi
tor.
c. Hypertension.
d. Vascularspasm.
e. Atheroscl
er ot
icplaqueruptur
e.
Answer
:

17. b. Tyr
osi
neki
nasei
nhi
bit
or.

Certai
nBCR- ABLt yrosi
neki naseinhibitorsareassociat
edwi thani ncr
easedri
skoft hromboti
c
events.Thesecond-andt hird-generationagents,suchasnilotinibandponat i
nib,havea
higherriskofthiscompl i
cat i
on.Ponat i
nibhasblackboxwar ningr egar di
ngcardiovascul
ar,
cerebrovascular,andperipher alvasculararter
ialthr
ombosis.Al thought hi
spati
enthas
hypertension,atherage, t
her i
skofpl aqueruptureandper i
pher althrombosisi
sl essli
kely.
Spasm issi mil
arlylessli
kelyt ooccuri nthispatient
.Oralcontracept i
vesaremostcommonl y
associatedwi t
hvenoust hr ombosi s.

Quest
ion:

18. A54- year-ol


dmanwi t
hmet astat
icprostatecancerunder
goesradi
calprostatect
omy.
Heisstart
edonadj uvantther
apywiththeandr ogen-depri
vat
ionagentl
euprol
ide.Hehasno
hist
oryofhyper
tension,di
abetes,
orhyperli
pidemia.Hei sanonsmoker.Arecentli
pidprofi
le
showedat ot
alcholester
olof166mg/ dlandlow- densit
yli
poprot
einchol
ester
olof85mg/ dl
.

Whi
choft
hef
oll
owi
ngi
sthemostappr
opr
iat
eti
mingf
orf
utur
ecar
diovascul
arr
iskassessment
?

a. 2year
s.
b. 1month.
c. 1year
.
d. 5year
s.
e. 3months.

Answer
:

18. e. 3mont
hs.

Patient
sr eceivi
ngandr ogen-depri
vati
ontherapyshouldbeevaluatedper
iodi
cal
lyforblood
pressure,li
pids,andfastingglucose.I
ngeneral,
patient
sshouldbeseenbyt hei
rphysician
within3-6mont hsofinit
iati
onoftherapybecausethefi
rstef
fectsofandr
ogen-deprei
ation
therapytypicall
yoccuri nthi
st i
mef r
ame.

Quest
ion:

19. A75- year-oldmanhasahi storyofcoronaryart


erydisease,stat
us/postpercutaneous
coronaryi
nterveniontotheri
ghtcoronar yar
tery2yearsagof orexert
ionalangina,mil
dCl assII
systol
icheartfai
lure,di
abet
es,hyperl
ipidemia,andarecentdiagnosisofmul t
iplemyeloma.  

Heret
urnstothecardi
ologycl
ini
ctodayafterarecentemer
gencyroom vi
sitf
orr
ightcal
fpai
n
andswell
ing.Hewasdiagnosedwithadeepvei nthr
ombosisoft
herightl
owerextr
emityand
i
nit
iat
edonant i
coagul
ationwi
thri
varoxaban. 
Hiscardi
ovascularmedicat
ionsincludemet oprol
olsuccinat
e25mgoncedai l
y,met
for
min
1000mgoncedai ly,
andali
rocumab( proprotei
nconvertasesubti
li
sin–kexintype9[
PCSK9]
i
nhibit
or),
ivabradi
ne5mgt wi cedai
ly,sacubitr
il
/val
sart
an50mgt wicedaily,
andaspir
inonce
dail
y.Hismultipl
emyelomai sbeingtreat
edwi thpredni
sone,thal
idomide,andasneeded
ondansetronfornausea.
 

Hisbloodpr essureis120/70mm Hg,heartrate70bpm, oxygensaturati


on99%onr oom ai
r.
Onexami nat
ion,hisjugul
arvenouspr
essureis2cm abovet hecl
avi
clewhi l
elyi
ngat45
degrees.Nor malS1andS2 wit
hnomur murs,rubs,
orgallops.Lungsarecleartoauscult
ati
on
bi
lat
er al
ly.Hehasasymmet ri
cedemainther i
ghtlowerextremit
ywithintactpul
ses.

Whichoft
hefol
lowi
ngmedi
cat
ionsi
sassoci
atedwi
thi
ncr
easedr
iskofvenous
t
hromboemboli
sm?

a. Sacubit
ril
/val
sart
an.
b. I
vabradine.
c. Ondansetron.
d. Ali
rocumab( PCSK9)i
nhi
bit
or.
e. Thali
domi de.

Answer
:

19. e. Thal
idomi
de.

Thecor rectansweri st hali


domi de.Orali
mmunomodul ator
ydrugs, suchast halidomideand
l
enalidomi de, haveadocument edassoci
ationwi thincr
easedr i
skf orvenoust hromboembolic
events.Theot hermedi cati
onsar enotassociatedwi thincr
easedr i
skoft hromboembol i
sm.
Pati
entswi t
hmul tiplemyel omaar egenerall
yol derandmayhavepr e-exist
ingcar diac
conditi
ons.Ther efor e,theymayber eferr
edt othecar di
ologyclinicforf oreval uati
onoflower
extr
emi tyedema, asi spr esentinthiscase,orfordyspneai ntheeventofvenous
thr
omboembol i
sm l eadi ngtopul monaryembol ism.Hence, cardiologistsshoul dbeawar eof
thi
spot entialsideef fectoft heseagentsusedt otreatmul t
ipl
emyel oma.Nei therivabradi
neor
sacubitri
l/valsartanhavebeenshownt oincreaser i
skofvenoust hromboembol ism.

Quest
ion:

20. Your eviewanarti


clet
hatsuggestsanovelserum biomarkerexcl
udesthepresenceof
cor
onaryar
terydi
sease(CAD).I
nthearti
cle,ser
um samplesofthenovelbiomarkerare
cor
rel
atedwit
hsubsequentangi
ographi
callydefi
nedpresenceorabsenceofCAD.

Ther
esul
tsoft
hisst
udyar
edepi
ctedi
nFi
gur
e1.

Whichoft
hef
oll
owi
ngi
sthecor
rectnegat
ivepr
edi
cti
veval
ue(
NPV)oft
hebi
omar
keri
nthi
s
st
udy?
a. 67%.
b. 15%.
c. 90%.
d. 20%.
e. 53%.

Answer
:

20. c. 90%.

Thecor
rectansweri
s90%.

NPViscalcul atedbythef ormula:(TrueNegat ives)


/(TrueNegat i
ves+FalseNegati
ves)
.
Inthiscase45/ (5+45)=0. 9or90%.
67%represent sthesensitivi
tyofthet est:(Tr
uePositives)/
(TruePositi
ves+Fal
se
Negat
ives).
10/(10+5)=67.
53%represent sthespecifici
tyofthet est:(Tr
ueNegat ives)
/(TrueNegati
ves+False
Posi
ti
ves).
45/(45+40)=0. 53.
15%represent stheprevalenceoft hedi sease:(Tot
alwi thDisease)
/(Tot
alSampl
e).
15/100=0. 15.
20%r epresentsthepositi
vepredi
cti
vevalueofthetest
:(Tr
uePositi
ves)
/(TruePosi
ti
ves
+FalsePosi ti
ves).
10/(
10+40)=0. 2.
Consequentl
y, fr
om thisstudy,wewouldconcludethatthi
sbiomarkerhasahighNPV, and
thosepati
entswhoar ebiomarkernegati
veareveryli
kelyt
onothaveangiographical
lypr
oven
CAD.

Quest
ion:

21. Youar easkedt oseea19- year- ol


dmanaspar tofhi
sscreeningphysicalpri
orto
playi
ngcol l
egebasket ball
.Hehasnopastmedi calhistoryandhasbeenpl ayi
ngbasket ballat
thehighschooll evelfor4year s.Herepor tsnochestpai nordyspnea, andcankeepupwi thhi
s
teammat esduringdr i
ll
sandcondi ti
oni ngexercises.Hehasnopal pitat
ions.Hedoesnot
smoke.Hi sfamilyhi storyisnegati
vef orarrhythmia, congenit
alheartdisease,aneur
ysm, or
unexplaineddeat hs.Onexam, heis6f eet4i nchest allandweighs190pounds.Vi t
alsignsar e
heartrate55bpm, bloodpr essure115/ 65mm Hg, andoxygensat urati
on99%.Jugul arvenous
pressureisnormalandhehasnocar diacmur mur s.Lungsar eclearandhehasno
muscul oskel
etalabnor mali
ti
es.

Whi
choft
hef
oll
owi
ngst
udi
esdoyouper
for
m bef
orecl
ear
inghi
mforspor
tspar
ti
cipat
ion?

a. El
ectrocardiogr
am.
b. ChestX- r
ay.
c. Echocardiogram.
d. Nofurthertesti
ngneeded.
e. Exerci
setreadmill
test
ing.

Answer
:

21. d. Nof
urt
hert
est
ingneeded.

Cardiovascularscreeningofathletesi
sperfor
medinonlythreecountr
ies:theUni t
edSt at
es,
It
aly,andI sr
ael.I
nt heUnitedStates,t
hescreeni
ngconsist
sofat hor
oughhi storyanda
compl etephysicalexam.Ital
yhasadvocat edfort
headditi
onofanelectrocardiogram (
ECG),
andIsr aelhasadpot edthi
sstrategytoo.Ther
eissti
llaf
airamountofdebat eaboutt heopti
mal
str
ategy.  

Whenconsi deri
ngt headditi
onoft heECG, onemustconsidertheresourcesi
nvolvedandthe
i
mpactoff alsel
yposi ti
vetest
ing.Af ewimpor t
antpoi
ntst
hatsuppor tt
heuseofhi stor
yand
physicalonlyare:1)t heuseofECGsasascr eeni
ngtesti
nheal t
hypopulati
onshasnotbeen
proven,2)thebenef it
sofECGscr eeninghaveonlybeendemonst rat
edinonedat abasefr
om a
smallregioni nIt
aly,and 3)theser esul
tshavenotbeenrepli
catedinotherareas.
 

Ther ef
ore,thecur
rentrecommendat i
onsint heUni
tedStat
esforscreeningofathletesi
ncl
udea
thoroughhi st
oryandphysi calexaminati
on( Ameri
canHeartAssociati
on's14-pointscr
eeni
ng
guideli
nes;Amer i
canAcademyofPedi at
rics’Pr
epart
ici
pat
ionPhysicalEvaluati
on).Mass
screeningofyoungat hlet
eswi thECGsi snotrecommended.Similarl
y,i
nanasympt omati
c
patientwithnosignif
icantfamilyhist
ory,exerci
setr
eadmil
ltest
ing,chestX- r
ay,and
echocar di
ographyarenotnecesssar y.
Quest
ion:

22. A19- year


- ol
dmancomest oyouf orasecondopi nionaboutpar ti
cipati
ngincollege
basketball.Hi sbrotherwasr ecentlydiagnosedwi t
hhyper tr
ophi ccardi
omyopat hy(HCM)af t
er
hehadchestpai nwhi lerunning.Thepat ienthasbeengenet icall
yscreenedandi sfoundt o
havehi sbr other'
sdi sease-causingmut ation.Yourpat i
entunder wentechocar di
ography, which
wasnor mal .Therewasnol eftventri
cularhyper tr
ophyandnor est
ingorprovokableleft
ventr
icularout fl
owt ractgradient.Cardiacmagnet i
cresonancedemonst ratednomyocar dial
fi
brosis.Yourpat ienthasneverhadsympt omsandhaspl ayedonhi shighschoolbasket ball
team wi t
houti nci
dent .Thereisnof ami l
yhi storyofsuddencar di
acdeath.Hisprimarycar e
physicianr ecommendedt hathenotpar t
icipateincoll
egeat hleti
cs.Thepat i
entisanxi
oust o
playandcomest oyouf orasecondopi nion.

Whi
choft
hef
oll
owi
ngdoyour
ecommend?

a. Al
lowhimtoparti
cipateaf
teri
niti
ati
onofbeta-
blockers.
b. Al
lowhimtoparti
cipateaf
ter48-hourHolt
ermonitor
ingshowsnoar rhyt
hmi
a.
c. Cl
earhimforcompetiti
vesport
s. 
d. Al
lowhimtoparti
cipateaf
teri
mpl ant
ati
onofcardi
overter
-def
ibr
il
lat
or.
e. Noclear
anceforpart
ici
pati
ngincompet i
ti
vesport
s.

Answer
:

22. c. Cl
earhi
mforcompet
it
ivespor
ts.
 

Patient swi t
hadiagnosisofhyper t
rophi ccardiomyopat hy(HCM)andcl inicalexpressi onofthe
phenot ypeshouldnotpar t
ici
pateincompet it
ivesports,exceptf orlow- i
ntensityactivities,
i
ndependentofage, sex,magnitudeofl eftventri
cularhypertr
ophy( LVH) ,specif
icmut ation,
presenceorabsenceofLVout f
lowtractobst ructionatrestorexer cise,fi
br osi
soncar di ac
magnet i
cresonance(CMR) ,absenceofpr i
orsympt oms, andhi storyofmyect omyorsept al
ablation.Phar macologicagentsandi mpl antablecardiover
ter-def i
bril
lat
or sshouldnotbe
uti
li
zedf orthesolepurposeofpar t
ici
pat ioninhi gh-i
ntensit
yspor t
s.Pat i
ent swhoar egenot ype
-posi t
iveandasympt omaticwithnoevi denceofLVHbyechocar di
ogram andCMRi nt he
absenceofaf amil
yhistoryofHCM- relatedsuddendeat hhavenocont r
aindicati
ont o
participation(Cl
assIIa).

Quest
ion:

23. An85- year-oldwomani sseeni nthehospi talforapr eoperativecardi


acevaluat
ion.She
wasadmi t
tedear l
ierinthedaywi t
haf emor alheadf racturesustaineddur i
ngawi t
nessed
mechani calf
al lonherwaybackf r
om thegym.Hercar diacexami nationisnotabl
eforanor mal
S1 andsoftS2, anda3/ 6systoli
cmur murt hatr adi
atest ohercar oti
dsand2/ 6api
calsystol
ic
mur mur.Herj ugul arvenouspr essureisnotel evated.Theel ectrocardiogr
am showsshei sin
si
nusr hythm, withleftventr
icularhypertr
ophy.Labor at oryvaluesrevealtroponi
nis0.01ng/ ml
,
creati
nineis1. 7mg/ dl,andbrainnatri
ureticpept i
deis109pg/ ml.

Theor
thopedi
csur
geonwoul
dli
ket
otakehert
osur
ger
yassoonaspossi
ble.Abedsi
de
echocar
diogr
am i
sobt
ained(
Videos1and2)
.

Whi
choft
hef
oll
owi
ngi
sthemostappr
opr
iat
enextst
ep?
 

ht
tp:
//cdn.
mycr
owdwi
sdom.com/acc/questions/9dd124a7-3ae1-
4cf6-bc9f
-
1cf066238bf4.mp4
ht
tp:
//medi
a.mycrowdwi
sdom.
com.
s3.amazonaws. com/ acc/accsap9/
Questi
on1524%20Vi
deo
2.
mp4

a. Per
for
mr ightandleftheartcathet
eri
zati
on.
b. Per
for
m coronarycomput edt omographyangiography.
c. Ref
erforvalver
eplacementpr iort
ohipsurgery.
d. Per
for
m myocar di
alperfusionimaging(si
ngle-photonemi
ssi
oncomput
edt
omogr
aphy)
.
e. Pr
oceedwi t
hhipsur gery.

Answer
:

23. e. Pr
oceedwi
thhi
psur
ger
y.

Thispat i
enthaswhatappearstobeasympt omaticsever
eaor t
icst
enosis.Whil
ethemost
approriatel
ong-ter
mtherapyfortheaorti
cstenosi
smayi nfactbeaort
icvalverepl
acement
(ei
thertranscut
aneousorsurgical
),shehasahipfract
urethatmaywellrequir
emor eurgent
att
ention. 

Af r
ankdi scussionwi t
ht hepat i
entandwi tht
hesur gicalandanesthesiat
eamsi sappr opri
ate.
Stresstestinginanact ivewomenf orpreoperati
ver i
skassessmentisunneccessar y.I
nvasive
cardiaccatheterizati
on,includi
ngcor onaryangiographyandmeasur ementofi nt
racardiac
pressures,maybehel pfulformanagi ngahemodynami cal
lyunstabl
epatient
,butwoul dnotbe
neededi nthispat i
ent(asidefrom cardiaccatheter
izati
ontodef i
necoronaryanatomypr iort
o
valvereplacement ).Simil
arly,acoronarycomput edt omorgraphyangiogr
aphywoul daddl it
tle
tothispatient'
smanagement .

Quest
ion:

24. A70- year-


oldreti
redinternistpresent
st oyourofficeaskingforyouropinionabouther
ri
skofhavingacardi
accompl i
cationfrom anupcomi ngf emoral-popli
tealbypasssurger
y
bei
ngperformedtorel
ievesever eclaudicati
on.Indiscussingthis,shetellsyouthatsherecal
ls
usi
ngtheGol dmanri
skscor ewhenshepr act
icedtoassessoper at
iverisk.

Whichoft
hefoll
owingwouldbet
hemostaccur
atechar
act
eri
zat
ionofmul
ti
var
iabl
etool
sfor
pr
eoperat
iver
iskassessment
?

a. Val
idat
edmul
ti
var
iabl
eri
sk-
predi
cti
ont
ool
srecei
veaCl
ass1i
ndi
cat
ioni
nthe
pr
eoper
ati
veassessmentofnoncar
diacsur
ger
y.
b. Nat
ri
uret
icpept
idet
est
inghassuppl
ant
edt
heuseofmul
ti
var
iabl
eri
skt
ool
sint
he
pr
eoper
ati
veset
ti
ng.
c. TheGol
dmanr
iskscor
eremai
nst
hepr
efer
redmul
ti
var
iabl
escor
eforpr
eoper
ati
ver
isk
assessment.
d. Dobutaminestr
essechocar
diogr
aphyhassuppl
ant
edt
heuseofmul
ti
var
iabl
eri
skt
ool
s
i
nthepr
eoper
ati
veset
ti
ng.
e. Sever
almul
ti
var
iabl
eri
skscor
esar
enowavai
labl
eforpr
eoper
ati
ver
iskassessment
.

Answer
:

24. e. Sever
almul
ti
var
iabl
eri
skscor
esar
enowavai
labl
eforpr
eoper
ati
ver
isk
assessment
.

Ther
eareseveralmult
ivari
abl
etoolsavail
ableforpreoper
ati
veriskassessment.Theseincl
ude:
1)Ameri
canCollegeofSurgeonsNationalQuali
tyImprovementProgram (ACS-NSQIP)surgi
cal
ri
skcal
culat
or,
2)RevisedCardiacRiskIndex(RCRI),and3)VascularSt
udyGr oupofNew
Engl
and(VSGNE)r i
skindex.
 

TheACS- NSQIPof fersaWeb- basedcal culat


or(www. r
iskcal
culator.
facs.
org).Recent
guideli
nesst at
et hataval i
datedr i
sk-predicti
ontoolcanbeusef ulinpredicti
ngtheriskof
perioperati
vemaj oradver secardiacevent sinpati
entsundergoingnoncar diacsurgery(Class
II
a,notCl assI).Neitherdobutami nestressechocar di
ographynornat ri
ureti
cpeptidetesti
ng
hasr epl
acedt heser iskscores.Inpatientswithoutacar di
achistorywhohaveno
cardiopulmonar ysympt oms, nofurtherr i
skstrat
if
icat
ionisnecessary.TheGol dmanr iskscore
hasbeensuppl antedbyt hesemor emoder ntool
s.

Quest
ion:

25. A60- year -


oldmanhasahi storyofabaremetalst
entimplant
edinhismidlef
tant
eri
or
descendi
ng3year sagoforangina.Sincethenhehasbeenchest-pai
nfree.Heissomewhatof
avaguehistorian, buthi
swife,whoisi ngoodhealt
h,conf
ir
mshewasabl etokeepupwit
hher
astheywalkedt hr
ought heshoppingmal lovert
heholi
dayseason,andoftenhel
pswit
h
vacuumingthei rhouse.Heisanavi dgolferandcanwalkan18-holecourse. 

Onemont hago,hehadmil
dhemopt ysi
sandwasf oundtohavea1-cm lungcancerintheright
upperlobe.Hi
soncol
ogi
strepor
tsthatheisanexcell
entcandi
dat
eforcurat
iveresect
ion.The
pati
entnowpresent
stoyouforpreoper
ati
veclear
ance. 

Onexami nati
on,hi
sbloodpr essureis120/80mm Hg, heartrate74bpm, andbodymassi ndex
35kg/m2.Hehasnor malS1 andS2 heartsoundswit
hnoS3  ormur mur,cl
earlungs,noedema,
andotherwisehasanormalexami nati
on.Onelect
rocardi
ography, t
hereisl
eftvent
ricul
ar
hyper
trophywi t
hstr
ain,butotherwiseiti
snormal.Hiscreati
nineleveli
s1.2mg/ dl
.

Whi
choft
hef
oll
owi
ngwoul
dyour
ecommendaspar
tofyourpr
eoper
ati
vepl
an?

a. Left
-heartcatheteri
zat
ion.
b. Nofurthertesti
ngisneeded.
c. Dobutaminest ressechocardi
ography.
d. Exerci
setreadmi l
lstr
esstest.
e. Exerci
senucl earperf
usiontest
.

Answer
:

25. b. Nof
urt
hert
est
ingi
sneeded.
TherecentAmer i
canCol legeofCar diology/Amer icanHear tAssociati
onguideli
nerecommends
nofurt
herpreoperativetesti
ngi fpati
ent shavemoder ateorgr eat
er(≥4METS)f uncti
onal
capaci
ty.Addit
ionall
y,nof urtherpreoper at
ivetestingisadvisedifitwil
lnotchange
managementdeci sions.Inthiscase, eveni ft
hepat i
entwasf oundt ohavecoronarydisease,
rout
inecoronaryrevascularizati
onbef orenoncar diacsur ger
yexclusivel
ytoreduce
peri
operat
ivecardiacevent sisNOTi ndi cat
ed( ClassI I
I:nobenefi
t).

Quest
ion:

26. A29- year


-ol
dwomanwi thahistor
yoftr
icuspi
datresi
awhounder wentaFontan
procedur
eyearsagoisconsi
der
ingbecomingpregnantagai
nsthercar
diol
ogist
'sadvi
ce.

I
fshei
sabl
etosust
ainapr
egnancyi
ntot
het
hir
dtr
imest
er:
 

Whi
choft
hef
oll
owi
nghemodynami
cchangeswi
llmostl
ikel
yoccuri
nthi
spat
ient
?

a. Systemicvascul arresi
stancewi lldecrease.
b. Fontanejecti
onf racti
onwi l
lincrease.
c. E/E'rat
iowi l
ldecrease.
d. Leftventr
icularmasswi lldecrease.
e. Leftventr
icularejecti
onfract
ionwi l
ldecrease.

Answer
:

26. e. Lef
tvent
ri
cul
arej
ect
ionf
ract
ionwi
lldecr
ease.

Manywomenwi thst ructuralheartdiseasereachr eproduct


iveageandcont emplate
mot herhood.Pregnancyi nducesandr equiresmaj orhemodynami cchanges.Pregnantwomen
withstructuralhear tdiseasemayhavear educedcar diacreserve.Cardiacoutput,
stroke
volume, lef
tventricularmass, andE/E' r
ati
oi ncr
easesi gni
fi
cantly,andejecti
onfracti
onand
fr
act i
onalshor t
eningdecr easedur i
ngpr egnancy.Cor net
teandcol leaguesshowedt hatt
here
wasast ati
sti
callysi gnif
icantdiff
erenceinej ect
ionfracti
on,fr
actionalshort
ening,andE/E'r
ati
o
beforeandaf terpr egnancy.

Quest
ion:

27. A36- year-oldwomanunder goesinvitr


ofer
til
izati
onandbecomespregnantwi
thtwins.
Herpregnancyi scompl i
cat
edbygest ati
onalhyper
tension.Shei
str
eatedwit
hbeta-bl
ocker
s
anddeli
verst woheal t
hysons.Duri
ngt henextmonths, herbl
oodpr
essurer
eturnst
onormal,
andshenol ongert akesanymedicati
ons.

Whi
choft
hef
oll
owi
ngi
srecommendedf
orherf
oll
ow-
up?

a. Surveill
anceechocar di
ogr
aphy.
b. Ongoi ngcoronaryarter
ydiseaser
iskf
act
orscr
eeni
ng.
 
c. Dai
lyaspi ri
n.
d. Cardiacrehabil
itat
ion.
e. Geneticscreening.
Answer
:

27. b. Ongoi
ngcor
onar
yar
ter
ydi
seaser
iskf
act
orscr
eeni
ng.
 

Womenwi thahistoryofgest
ati
onalhypert
ensionorpr e-ecl
amspi aareatatwofoldincreased
ri
skofcardi
ovasculardi
seaseandshouldhaveaggr essiveri
skf actorscr
eeni
ngf orcoronary
art
erydi
seaseastheyage.Nei t
hergeneti
cscreeni
ngf ormut at
ionsorechocardiographyar e
i
ndicat
ed.Alt
houghhercar di
ovascul
arri
skiselevat
ed, prophyl
act i
caspiri
nisnotindicated.

Quest
ion:

28. A27- year


-oldwomani sseenint heemergencydepar t
ment(ED)forseveraldaysof
progressivedyspnea, orthopnea,paroxysmalnocturnaldyspnea,
andper i
pheraledema.She
hadasi mi l
arpresentationtotheED3weeksagoandwasgi venaprescri
pti
onfor40mgdai l
y
offurosemideandmet oprol
ol12.5mgt wicedai
ly.Sheis25weekspr egnant.Vi
talsignsare
heartrate102bpm, bloodpr essure100/60mm Hg, andoxygensaturat
ion100%on2l i
ter
s
nasalcannul aoxygen. 

Physicalexami nati
ondemonst r
ateselevatedjugularvenouspr essuretot heangleofthe
mandi blelyingat45degr ees,aregulartachycardi
cr hythm wi
tha1/ 6hol osystoli
cmur murat
theapexwi tha2/ 4diastolicmurmuratt heapex;anopeni ngsnapi spresent .Ther
eare
bil
ateralralesfortwo- t
hirdsofthepost eri
orlungfiel
dswi thtr
aceper ipheraledema.
Echocar di
ogr am revealsnor mallef
tventri
cularfunctionwithameanmi tr
alvalvegradi
entof16
mm Hgwi t
ht ri
vialmitr
alr egur
git
ation.Valveismobi lewithminimalcal ci
ficati
on.The
esti
mat edr ightventri
cularsystol
icpressureis50mm Hg.

Whi
choft
hef
oll
owi
ngi
sthebestnextst
epi
nhermanagement
?

a. Del
iverynow, t
henmitralvalverepl
acementsurgeryshort
lyt
her
eaft
er.
b. Mit
r alval
verepl
acementsur gery.
c. Percutaneousmitr
alcommi ssurotomy(val
vuloplast
y).
d. Percutaneousmitr
aledge-to-edgemitr
alvalverepair(
Mit
ra-
Cli
p).
e. Cont i
nuemedicalt
herapy.

Answer
:

28. c. Per
cut
aneousmi
tr
alcommi
ssur
otomy(
val
vul
opl
ast
y).

Percutaneousmitralval
vecommi ssur
otomy(valvul
oplasty)i
saCl assIIar
ecommendat i
onin
theabsenceofsignifi
cantmitralregur
git
ati
onandf avorableanatomicfeat
uresinthe2014
Amer i
canCollegeofCardiol
ogy/ Ameri
canHear tAssociat
ionvalvul
arheartdiseaseguidel
ines
f
orpr egnantpati
entswithClassI I
IorI
Vheartfail
uresympt omsdespitemedicaltherapy. 

Cardiothoraci
csurgerycarri
esa30%r iskoff et
allossandi sthereforeavoi
dedduring
pregnancy, i
fpossi
ble.Deli
veryandthensur gerywoul dnotbeopt i
malmanagementatt his
point
, unlessforaspecif
icobstetr
ici
ndicati
onduet oissuesrelatedtoprematur
it
y.Her
sympt omshavepr ogresseddespitedi
uretictherapy,thus,conti
nuedmedi calt
herapywould
notbet hebestchoice.Percutaneousmitraledge-to-edgemi tr
alvalverepai
r(Mit
raCli
p)isnot
i
ndi
cat
edast
hepat
ienthasmi
tr
alst
enosi
snotmi
tr
alr
egur
git
ati
on.

Quest
ion:

29. A32- year-ol


dwomanwi t
hahi st
oryofbicuspidaort
icvalvepresentsf
orevaluationof
aorti
cstenosis.Sher eport
ssomer ecentdyspneawi t
hexerti
on,buthasnothadsyncopeor
chestpain.Echocardiographyrevealsnormalleftventr
icul
arfuncti
on,andameangr adient
acrossheraor t
icvalveof52mm Hg.Herascendi ngaortaisnormalinsize.Shei
sr ecently
mar r
ied,andisanxioust ostar
taf amily.

Whi
choft
hef
oll
owi
ngdoyour
ecommend?

a. Exercisetreadmil
ltest
ing.
b. Surgicalaorti
cvalverepair
.
c. Aorti
cval verepl
acementpr i
ortopregnancy.
d. Transcatheteraorti
cvalverepl
acementpriort
opr
egnancy.
e. Noint er
ventionpri
ortopregnancy.

Answer
:

29. c. Aor
ti
cval
ver
epl
acementpr
iort
opr
egnancy.

ThispatienthasaCl assIindi
cati
onforaor t
icvalver epl
acement(AVR)becauseshei s
sympt omat i
cwithsevereaort
icstenosis(AS)andwant stobecomepr egnant
.Valveselect
ion
i
nt hi
spopul ati
oniscontroversi
alandallpatientsshouldbecounsel edbycar
diol
ogistswith
experti
sei nval
vularheartdi
seaseaboutt herisksandbenef i
tsofallopt
ionsf
oroperative
i
ntervention,i
ncl
udingmechani calprost
hesis,bioprosthesi
s,andtheRossprocedure.
 

AVRi sanapplicabl
eopt
ionforsomepatientswit
haorti
cvalveregur
gitat
ion,
butisnotan
optioninpat
ientswit
hAS.Transcat
heterAVRisnotapprovedforpati
entswithbicuspi
dval
ve,
andt hi
spati
enthasaverylowSocietyofThoraci
cSurgeonsscore.Treadmil
ltesti
ngisnot
recommendedbecauseshei ssymptomat i
cwithsever
eAS.

Quest
ion:

30. A38- year-ol


dmanwi
thmetastat
ict
esti
cularcancerunder
goessurgi
calresect
ion,
foll
owedbyt
reat
mentwit
haci
spl
atin-
basedregimen.Hehasnoot hersi
gnif
icantmedical
histor
y.

Whi
choft
hef
oll
owi
ngi
sthemostl
ikel
ycar
diovascul
arcompl
icat
ionofhi
schemot
her
apy?

a. Venoust hrombosi s.
b. Arter
ialthrombosis.
c. Cardiomyopat hy.
d. Premat ureatheroscl
erosi
s.
e. Hyper t
ension.
Answer
:

30. e. Hyper
tensi
on.

Thegreatestcardi
ovascul arr
iskofplatinum-basedchemother
apyisassociat
edwiththe
devel
opmentofhyper tension.Ar t
eri
althromboti
criski
sassoci
atedwithtyr
osineki
nase
i
nhibit
orssuchasni l
otinib.Cardiomyopat hi
esaremostcommonl yassoci
atedwit
h
anthr
acycli
ne-basedther apy.Venoust hrombosismayberelat
edtothehypercoagul
abil
it
y
associ
atedwi t
hunderlyingmal ignancy.

Quest
ion:

31. Youar eseei ngi ntheoffi


cea45- year
-oldwhi temal enonsmokerwi thtype2diabetes
andhumani mmunodef i
ciencyvir
us( HIV).Hehasat otalcholesterolof190mg/ dl,l
ow-densi
ty
l
ipoprot
eincholesterolof101mg/ dl,andasyst ol
icbloodpr essur eof125mm Hgonno
medicat
ionsforhyper tension.HisHIVi swellcontroll
edonapr oteasei nhi
bit
or-basedregi
men
wit
hanundet ectableviralload.Previouslyhi
sHI Vhadnotbeenwel lcontr
oll
ed.Youwoul dli
ke
himtobeonopt i
malmedi cati
ont oreducehisr i
skf orcardiovascular(CV)events.

Whichofthefoll
owingchangesi
npharmacot
her
apydoyour ecommendasthebestst
rat
egy
f
orreduci
ngt heCVr i
skforat
her
oscl
erot
icCVdi
seaseevent
swhi l
emai
ntai
ningef
fi
cacyof
t
reatmentforHIV?

a. Changeanti
retr
oviralagentt
oanon-nucleosi
derever
set
ranscr
ipt
asei
nhi
bit
or.
b. I
nit
iat
efenofi
brate145mgperday.
c. Changeanti
retr
oviralagentt
oani
ntegraseinhi
bit
or.
d. I
nit
iat
eatorvast
atin10mgperday.
e. I
nit
iat
esimvastati
n40mgperday.

Answer
:

31. d. I
nit
iat
eat
orvast
ati
n10mgperday.

I
niti
at i
ngmedi cati
ons,li
kestat
insandf i
shoi l,
whicharedesignedtoi mprovel
ipi
dsand
tr
iglyceri
des,isli
kelyabett
eroptionfordecreasingCVrisk,r
atherthanchangingantir
etr
ovi
ral
s.
Proteaseinhibit
ordrugsworsenlipidprofi
les,moresothanintegraseinhibi
tor
sornon-
nucleoside-r
everse-tr
anscri
ptaseinhibi
torregimens.However,switchingfr
om astabl
e
proteaseinhibit
orregi
ment oanal t
ernati
ver egimenmaybeassoci atedwi t
hvir
ologi
c
breakthrough. 

Inar andomi zedtr


ialofli
pidlower i
ng,pravast
atinandbezaf i
br at
epr ovedsigni f
icantlymor e
eff
ectivei nthemanagementofhi ghlyacti
veant ir
etroviraltherapy- r
elatedhyper l
ipidemi athan
switchingt her
apyfrom proteaseinhibit
orstonevi rapineoref avirenz.However ,
large-scaleCV
outcomespr event
iondataar elacking.Simvastatinandl ovastati
nar econtraindicatedf oruse
withpr oteaseinhi
bitor
s.Preferr
edst art
ingdosesofast atininHI Vpat i
entstakingpr otease
i
nhibitorr egi
mensi ncl
udeat or
vastatin10mgdai lyorr osuvast ati
n10mgdai ly)Inthisvignett
e,
thepat ientwouldmeetgui deli
nesforamoder ate-intensitystatinbyvi r
tueofhi sdiabet es.
Quest
ion:

32. A36- yearoldwomanpr esent


stoyouf orcar
diol
ogyconsul
tat
ion.Her42-yearol
d
brot
herwasrecent
lydiagnosedwithhyper
trophiccar
diomyopat
hy(HCM) ,andhetoldhert
hat
sheneededt
obescr eenedbyacar di
ologi
st.
 

Thepati
ent’
sbr otherwasdi agnosedafterar
outi
neelect
rocardi
ogr
am (ECG)showedlef
t
vent
ri
cular(LV)hypertrophywithastrai
npatter
n.Herbrot
herhadanechocar di
ogr
am
conf
ir
mi ngthediagnosi s,
andheunder wentgenet
iccounsel
ingandtesti
ng.Hewastoldthat
nopathogenicmut ati
onswer eident
if
ied. 

Thepati
enthasnosympt oms.Sheexerci
sesbywal kingatabriskpacefor30minutesdai
ly
wit
houtli
mitat
ionsandworksful
lti
measahi gh-
schoolalgebrateacher
.Shetakesno
medicat
ionsandhasnopastmedi calhi
stor
y.Herfatherdiedattheageof42inacaraccident
.
Hermotherisali
veandwell
,andshei snotsurei
fhermot herorhertwosister
shavebeen
scr
eened.Onexaminati
on,herheartr
ateis72bpm andherbl oodpressurei
s108/70mm Hg.
Shehasnomur mur
s. 

Youperform a12-l
eadECGt hatshowsnormalsinusrhyt
hm, withnormalaxi
sandinter
vals
andnoST- Tchanges.Youper f
orm anechocar
diogr
am thatshowsnor malLVfunct
ion,noLV
hyper
trophy,nor
malvalvul
arfuncti
on,andnopulmonaryhypertensi
on.

Whi
choft
hef
oll
owi
ngt
est
sdoyouadvi
se?

a. 24-hourambul at
or yECGmoni t
oring.
b. Cardiacmagnet i
cr esonanceimagi ng.
c. Not esti
ngnow;r epeatechocardiogram i
n5year
s.
d. Geneticcounseli
ngandt est
ing.
e. Exercisest
resstest.

Answer
:

32. c. Not
est
ingnow;r
epeatechocar
diogr
am i
n5year
s.

Hypertrophiccardiomyopat hy(HCM)iscausedbyanaut osomaldomi nantmut ationingenes


thatencodesar comer eprotei
nsorsar comere-associatedpr oteins.Geneticand/ orclinical
screeningofallfi
rst-degreefamilymember sofpatientswi thHCM i simpor tanttoi dentif
ythose
withunr ecogni
zeddi sease.Ont hebasisoffamil
yhi st
or y,cli
nicalscreening,andpedi gree
analyses, t
hepatternofinherit
anceisascertai
nedtoi dent i
fyandcounselr elat
ivesatr i
sk.
Becausef amili
alHCM i sadomi nantdisorder,
theriskthatanaf fectedpatientwilltransmi t
diseaset oeachof f
springis50%.  

Whenapat hogenicmut at
ionisi denti
fi
edinanindexpatient,t
hegenet i
cstatusofeachf amil
y
membercanber eadi
lyascer t
ained.BecauseHCM mut ationsarehighlypenetrant,amutati
on
conveyssubstanti
al(
95%)r iskoveral if
eti
mefordevelopingcli
nicaland/orphenotypic
evi
denceofHCM.However ,
whent hereisnoi
dentif
iabl
epat hogenicmutati
oni ntheindex
pati
ent,
therei
snoi ndicat
ionf orfurt
hergenet
ictesti
ngofot herfamil
ymember s. 
Exer cisest resst estingisusef ulinpat i
entswi t
hHCM t oassessf unctionalcapaci tyand
responset ot herapy, performr i
skst r
atif
icati
onf orsuddencar di
acdeat h,andt oassessf oran
exerci se- induceddynami cleftventri
cularout f
lowt ractgr adient
.However ,thispatienthasno
evidenceofHCM onechocar diogram, sotherei snoi ndicat i
onforstr
esst est i
ng.Car diac
magnet icr esonancei maging( MRI)isusef ulinpatientswi thHCM whent heechocar di ogram is
i
nconcl usi vef orthedi agnosisormor einformat i
onr egardi ngthemitralval veappar at usis
requi red.However ,cardiacMRIcur rentl
ydoesnotpl ayar oleinthescr eeni ngoffirst-degr ee
rel
at ivesofpat i
entswi thHCM.Twent y-four-hourambul at oryel
ectr
ocar diographicmoni tori
ng
i
sr ecommendedf orpat i
entswi thHCM whohavepal pit
at ionsorli
ghtheadedness( ClassI )
,as
ani nitialscr eeningf orvent r
iculartachycardiatodet ermi net heneedf orplacementofan
i
mpl ant edcar di
over ter-
def i
bri
ll
ator(ClassI )
,andever y1- 2yearstor eassessr i
skofsudden
cardi acdeat h( ClassI I
a).However ,allofthesei ndicati
onsar especif
ictopat i
entswhoal ready
carryadi agnosi sofHCM, andt heyarenoti ndicatedaspar toft
hescr eeningal gorithm f or
unaf fect edf irst-degreer elati
ves. 

Seri
alechocardiogramsar ereasonabl
e(ClassII
a)forcli
nical
lyunaffectedpatient
swhohavea
fi
rst
-degreerel
at i
vewi t
hHCM whengenet i
cstatusi
sunknown.Suchf oll
ow-upmaybe
consider
edevery12t o18mont hsforchil
drenoradolescentsfrom high-ri
skfamil
iesandever
y
5year sf
oradultfami l
ymember s.Thus,notesti
ngnowandaf oll
ow-upechocar diogr
am i
n5
yearsisthecorr
ectr esponse.

Quest
ion:

33. A60- year-


oldmancol l
apsesatal ocalfit
nesscent er
.Bystandershavewi tnessedthe
col
lapse,911iscalled,andchestcompr essionsar eini
ti
ated.Paramedicsar r
ivewi t
hin5
minutes.Ventri
cul
arf i
bri
ll
ati
on( VF)i
sseenont hemoni tor,a200Jbi phasicshocki sdeli
ver
ed,
andchestcompr essionsareresumedf or2mi nutes.VFremains,anda200Jbi phasicshockis
del
ivered,f
oll
owedbyanot her2mi nut
esofchestcompr essions.VFst i
llr
emai nsont he
monitor.

Whi
choft
hef
oll
owi
ngshoul
dbet
henextst
epi
nmanagement
?

a. Hol
dcar di
opulmonaryresusci t
ati
onandi nt
ubatethepat
ient
.
b. Est
abli
shIVaccessanddel iverami odar
one300mg.
c. Del
iveranotherimmedi
ate(st acked)shock.
d. Del
iverepi
nephr i
ne1mgi ntraosseousandcont i
nuechestcompressi
ons.
e. Est
abli
shintravenous(
IV)accessanddel i
verl
idocai
ne1mg/ kg.

Answer
:

33. d. Del
iverepi
nephr
ine1mgi
ntr
aosseousandcont
inuechestcompr
essi
ons.

Accordingt o2010advancedcar di
aclifesupportguideli
nes,thenextstepinmanagementi
sto
del
iverepinephrinebyeitheranIVorintraosseousroute,butavoidint
errupti
oninchest
compr essions.Vasopressi
nisanalternati
ve.Avasopr essorshouldbegi venbef
ore
anti
arrhythmictherapyisini
ti
ated.
PERI
CARDI
ALDI
SEASE

1. A49-
year
-ol
dmanwi
thahi
stor
yofhyper
tensi
onandobesi
typr
esent
stot
he
emer
gencydepar
tmentf
oll
owi
ngar
ecentvi
ralsyndr
ome.Onexami
nat
ion,
hei
spal
e
andappearsuncomfor
table,wi
thel
evatedj
ugularvenouspressur
e,clearlungf
iel
ds,
andnocardiacmurmurs,rubs,
orgall
ops.Hi
ssystoli
cbloodpressurefall
s20mm Hg
wit
hinspi
rati
on.Hi
ssaturati
onsare91%onr oom ai
r. 

Anelect
rocar
diogram demonstr
atessinustachycar
diaandbor der
li
nelowvol tages,
buti
sotherwi
senor mal.Labor
atori
esretur
nwi t
hhemat ocri
t29%,potassium 3.3
mEq/L,andcreati
nine1.0mg/dl.Whil
eadditi
onaleval
uationisbei
ngpur sued, a
nursecheckingonthepat
ientobservesthathesuddenlybecomesunr esponsi
ve.
Sinustachycar
diai
snotedont hecardi
acmoni t
or,butheiswithoutdetect
ablepul
se
andtherefor
ecardi
opulmonaryresusci
tati
onispromptlyi
niti
ated.

Whi
choft
hef
oll
owi
ngi
sthemostl
ikel
yet
iol
ogyoft
hepat
ient
’scar
diacar
rest
?

a. Pul
monar yemboli
sm.
b. Hypokalemia.
c. Anemia.
d. Myocardialinf
arct
ion.
e. Cardi
act amponade.

ANSWER

E.Car
diacTamponade

Inpatientswithtachycar
dia, jugulardistenti
on,orpulselesselectri
calacti
vity,cardi
ac
tamponadeshoul dbehighlysuspect ed.Whencar diacar r
estoccur ssecondar yto
tamponade, retur
nofspont aneousci rculat
ionisoft
ennotpossi bleunti
lperforming
emergentper icar
diocent
esi stoabr uptlydecreasepericardialpr
essur esandal low
vent
ricularfi
ll
ing.Alt
houghaddi ti
onalcausesofpul selessel ect
ricalacti
vit
yi nthi
s
pati
entmustbeconsi dered, basedont heinfor
mationpr ovided,theotheranswer
choi
cesar el
essl i
kel
yetiologiesthancar di
actamponade.
2. A64- year-ol
dmanpr esentswit
hnewonsetofpal pitati
ons,substernalchest
discomf ort
,anddyspnea.Hef el
twellunti
labout5daysagowhenhet houghthehad
theflu.Hei sonnomedi cati
onexceptforover-the-count
erpseudoephedr i
neand
i
bupr ofen.Onexami nat
ion,heisanxious,
inmoder atedistress,andtachypneic.He
hasahear trateof122bpm, bl
oodpressureof90/ 55mm Hg, andtemper atureof
37.2°C.Hisl ungsarecl
ear,andther
ei sanelevatedjugularvenouspr essure.A
pulsuspar adoxusismeasur edat18mm Hg.Hei scoolandcl ammy.

Per
ti
nentl
aborator
ydatarevealahemat
ocr
itof28%,
whi
tebl
oodcel
lcountof12,
500,
andcr
eati
nineof1.2mg/dl.
 

Anelectrocardi
ogram reveal
sanarrowcomplext achycar
dia.Abedsi de
echocardiogram conf
ir
msamoder ateperi
car
dialeff
usionwithrightventr
icul
ar
diast
oli
ccol l
apse,substanti
almi
tr
alinfl
owvariabil
it
y,andinfer
iorvenacavapl et
hor
a.

Whi
choft
hef
oll
owi
ngi
sthenextbestst
epi
nthi
spat
ient
’smanagement
?

a. Dopami neinfusion.
b. Peri
cardiocentesis.
c. Dir
ectcur r
entcar di
oversi
on.
d. I
ntravenousfur osemide.
e. Bedsideadenosi netesti
ng.

ANSWER

B.Per
icar
diocent
esi
s

Thepat i
entisincar diactamponadeandcar diogenicshockandi sinurgentneedof
defi
nit
ivetreatmentwi t
hemer gentper i
cardiocentesi
stor eli
eveper i
cardi
alpressur
e
andimpr ovethehemodynami cinstabili
ty.Whilesorti
ngouthi stachycardiaand
i
mpr ovinghishear tratewillbenecessar y,dir
ectcurrentcardioversi
onwoul dnot
l
ikel
yimpr ovehi sacut ehemodynami cpicture,andheneedsf ir
sttohavet he
peri
cardialfl
uidremoved.Dopami nemayhel phisbloodpr essureatt heexpenseof
i
ncreasinghist achycar di
a.Diur
eticslikelywoul dworsenhi slowout putstat
e.

3. A57- year-ol
dwomanwi t
hal ongsmoki nghist
orypresentswi t
hfever,mal
aise,
anddyspnea.Onexami nat
ion,hervi
tal
sincludetemperatur
e37. 9°C,hear
trate110
bpm, andbloodpressure98/64mm Hg.Scat t
eredwheezesar enotedonl ung
auscultat
ion.Heartsoundsaredist
antbutregularandtachycardicwithnomur murs,
rubs,orgal
lops.Anel
ect
rocar
diogram demonstrat
essinust
achycardiaandl ow
volt
ages.Thepulsuspar
adoxusismeasur edat17mm Hg.Ast atechocar di
ogram i
s
perfor
medandshowsnosi gnif
icantperi
cardi
aleff
usi
onandnor malleftventr
icul
ar
functi
on.

Whi
choft
hef
oll
owi
ngi
sthemostl
ikel
yexpl
anat
ionf
orherpul
suspar
adoxus?

a. Chronicobstruct
ivepulmonar
ydi
seaseexacer
bat
ion.
b. Thyrotoxi
cosis.
c. Restri
cti
velungdi sease.
d. Pneumomedi ast
inum.
e. Anorexianervosa.

ANSWER

A.Chr
oni
cobst
ruct
ivepul
monar
ydi
seaseexacer
bat
ion

Inpat i
ent swi thsuspect edper icar di
altamponade, document ati
onofanel evat ed
pulsuspar adoxusi sakeyexami nat i
onfinding.Pul suspar adoxusismeasur edusing
slowdef lationofamanualbl oodpr essurecuf fandcal culati
ngt hepressur e
diff
erencebet weent hef ir
stauscul tatedKor otkoffsoundst hatoccuronl yi n
expirati
onandt hepr essur eatwhi chKor ot
kof fsoundsar eaudi bleforallhear tbeats.
Normal ly,thepul suspar adoxusi s<10mm Hg.I nper i
cardialtamponade, const rai
nts
onvent ricularfill
ingleadt or espi rophasi ci
nt erventriculardependencet hatenhances
fl
uctuationsi nleftventri
cul arst rokevol ume.However , anelevatedpulsuspar adoxus
i
snotl imi tedt oper i
cardialt amponade, butcanal sobeobser vedi npati
ent swi t
h
obstructivel ungdi sease, obesi ty,obst r
uctivesl eepapnea, andt ension
pneumot hor ax.Inthispat ient ,t
heai rtr
appingandi ncreasedwor kofbreat hinglead
towi deswi ngsi nintrat
hor aci cpr essureandcr eatet heel evatedpulsuspar adoxus.

4. A52- year-ol dwomanwi t


hahi st
oryofmetastaticbreastcanceron
chemot herapypr esentswithsl
owlyprogressi
vedyspneaonexer ti
onoverthepast
month.Shedeni esfeverorchestpain,buthasbeeni ncreasi
nglyfat
igued.On
physicalexam, herhear tr
ateis115bpm wi t
hbloodpr essure102/70mm Hgand
ther
ei sevidenceofel evat
edjugularvenouspressure. 

Achestcomputedtomographi
c(CT)scandemonst r
atesa“l
argeper
icar
dialeff
usi
on”
prompti
nganurgentbedsi
deechocardi
ogram thatr
eveal
snormalbi
ventr
icular
funct
ionanda1.7cm ci
rcumfer
enti
alef
fusion.
I
naddit
iont
or i
ghthear
tdi
astol
iccol
lapse,whichoft
hefol
lowi
ngechocar
diogr
aphi
c
f
indi
ngswouldbemostsuggesti
veofpericar
dialt
amponade?

a. I
nferi
orvenacavacol l
apse.
b. Mitr
alDoppl erfl
owvar iat
ion40%.
c. Paradoxicalint
erventr
icularseptalmot
ion.
d. Atr
ialseptaldefectfl
owr eversal.
e. Tri
cuspidDoppl erinf
lowvar i
ation40%.

ANSWER

B.Mi
tr
alDoppl
erf
lowvar
iat
ion40%

Thispat i
enti ssuf fer
ingf r
om per i
cardialtamponadeandr equi resur gentdiagnost i
c
andt herapeut icper i
cardiocentesis.I
tisessent i
altounder standt hef undament al
echocar diographi cfi
ndingsoft amponade, ofwhicht hehal l
mar kisrightheart
diastoli
cchambercol lapseduet ocompr essionoft hecar diacchamber sbyel evated
pericardialpressur eleadingtoi mpairedf i
ll
ing.I
naddi t
ion, patientsintamponade
oftendemonst rateexagger at
edf l
owvar iati
onacr osst heat rioventri
cularvalvesasa
resultofr espirophasicvent ri
cularinter
dependence, defi
nedas>30%mi t
ralf
low
variati
onand>60%t ri
cuspi dfl
owvar i
ati
on, andinferiorvenacavadi lat
ationreflecti
ng
mar kedlyel evatedcent ralvenouspr essures.Paradoxi calint er
vent r
icularseptal
mot ionandat rialseptaldefectfl
owr eversalarenotchar act eri
sticfi
ndi ngsin
tamponade.

5. A49- year-
oldwomanwi t
hahistor
yofsystemiclupuserythematosuspresent
s
foreval
uationofint
ermit
tentshar
pchestpainforthepast3days.Sher eport
sthe
discomfortcameonsuddenl y,
isl
ocatedattheanteri
orchestwallwi
thr adi
ati
onto
theshoulders,
andfeelsworsewithdeepbreathi
ng.Onexami nati
on,t
hehear ti
s
regul
arwit
hasof tsystol
icmurmuratthelef
tst
ernalbor
der
.Anelectrocar
diogr
am
(ECG)demonstr
atessinustachycardi
awit
hmi l
ddiff
useSTelevat
ion.AchestX-ray
showsclearl
ungf i
eldsandcardiomegal
y.

Whi
choft
hef
oll
owi
ngi
sthenextbestdi
agnost
icmodal
it
y?

a. Cardiacmagnet i
cr esonancei
magi
ng(
MRI
).
b. High-sensiti
vitytroponin.
c. Coronaryangi ography.
d. Echocar di
ogram.
e. Exercisetr
eadmi lltest.
ANSWER

B. Echocar
diogr
am

Peri
cardialef
fusioncanbecausedbyanydi sorderoftheper i
cardi
um andis
suspectedbyhi story,
physi cal
,ECG,andchestX- r
ayfindings.Thispat
ientdescr
ibes
thecharacter
ist
icshar p,pleuri
ti
csubst er
nalchestdiscomf ortassoci
atedwith
peri
cardiali
nfl
ammat ion.Oftenthediscomfortisalsowor sewhenl yi
ngdownand
i
mpr ovedwhensi tt
inguprightandforwar d,
andmayr adiatetotheshouldersdueto
i
nvolvementoft hephr enicnerveasittraversesthepericardi
um.  

Examinati
onmayr evealapericardi
alfri
ct i
onr ub,whichisahi ghlysensiti
vefi
nding.
Earl
yinthediseasecour se,t
heECGof tenshowsdi f
fuseSTel evat
ion(concaveup)
andPRdepr essioninmostl eadsexceptf orleadaVR, whichmayr evealPRelevati
on
andSTdepr ession.However ,
afterthefirstweek,thechar acteri
sti
cSTandPR
changesmaynor malize.Whil
et hechestX- r
ayisoftenwit houtacuteabnor mali
ti
es,
l
argerperi
cardialeff
usions(>200mlf luid)maydemonst rat ecardi
omegal y.
 

Asthispat i
entpresentswi thacl i
nicalsyndr omesuggest i
veofper i
cardialdisease
andenl argementoft hecardiacsilhouet t
eonchestX- rayi mplyinganunder lyi
ng
peri
car dialeff
usion,anechocar di
ogr am isthenextbestst epint hepat i
ent’
s
eval
uat ion.High-sensiti
vitytroponi nwoul dbehelpf
ultoeval uat eforevidenceof
myocar dialinfl
ammat i
onbuti ssecondar ytoechocardiography, whichcandet ermi ne
thesizeandhemodynami csi gnif
icanceoft heperi
cardialeffusion.Thepat ientisl ess
l
ikel
yt ohaveobst ructi
vecor onarydi sease,andtheref
or e,exercisetreadmillstress
test
ingandcor onar yangiographyar enotindicat
ed.CardiacMRImaybebenef icial
oncet hedi agnosisisestablished, butwoul dnotbethef ir
st-li
net est.

6. A50- year
-oldwomanpr esent swi t
hevaluat i
onofcar diomegalyonr outinechest
X- ray.Shei sacti
vewi t
houtsympt oms.Herr eviewofsympt omsisnot abl
eforcold
i
nt olerance.Herhistoryisnotablef orhypothyroidism anddi abet
es.Shei stakingno
medi cat i
ons.Onexami nati
on, herbl oodpressurei s140/ 100mm Hg, heartratei
s50
bpm, andt emperatureis35°C.Shei swearingawi g.Thereisnoj ugularvenous
distention,andlungsar eclear.Ther eisnoadenopat hypalpable.Breastexami nat
ion
i
snor mal .Heartsoundsaredi stant.Abdomeni sobese, andt her
eis1+l ower
extremi tyedema.Ther earebrawnyski nchangesoverherl owerextremiti
es.

Anel
ect
rocar
diogr
am (
Figur
e1)andechocar
diogr
am (
Figur
e2)ar
eshown.
Whi
choft
hef
oll
owi
ngi
sthenextmostappr
opr
iat
edi
agnost
icst
ep?

Fi
gur
e1.

Fi
gur
e2.
a. Thyroid-st
imulatinghormone.
b. Cardiaccathet
er i
zati
on.
c. Chestcomput edt omography.
d. Cardiacmagnet icresonancei
magi
ng(
MRI
).
e. Peri
car di
ocentesis.

ANSWER

A. Thyr
oid-
sti
mul
ati
nghor
mone.

Themanagementofper i
cardialeffusionsdependsupont heetiology, i
ts
hemodynami cconsequences, andt hepat i
ent'
ssympt oms.Int heabsenceof
symptomsorhemodynami csequel ae,thet r
eatmentshoul dbebasedupont he
underlyi
ngetiol
ogy.Inthiscase, thepat ient'
sunder lyi
nghypothyr oidism likely
explai
nshermyxedemat ousst ateandconsequentl argeperi
car dialeffusion.Since
shehasnosympt omsandt herei snoevi denceforhemodynami ccompr omi seby
examinationorecho, t
hereisnoi ndi cati
onf orpericardi
ocentesisf ortherapeut i
c
reasons.Otherconsiderati
onsi ncludeaut oi
mmunecondi ti
ons, medi cati
ons( e.
g.
hydral
azine),Dr
essler'
s,radiati
on, uremi a,malignancy,andinfections. 

Mostper icar di
alef f
usionsareeit
herviralori
diopat
hicineti
ology.Di
agnosti
c
peri
car di
ocent esiscanbeconsi deredwhent hereisconcernformali
gnancyor
nonvirali
nf ections.However ,
pericar
dialti
ssuegenerall
yprovidesgr
eaterdi
agnosti
c
uti
li
ty.Car diaccat heteri
zat
ionisnotli
kelytobeusefulintheabsenceofsymptoms, a
normalcar diovascularexaminati
on,andnoevi denceforhemodynami ccompromise
onecho.Car di
acMRIwouldbel i
kelytoonlyconf
irmtheechocardiogr
aphicfi
ndings;
therearenoMRIf i
ndi
ngsthatarepathognomonicofaspecifi
cdiagnosis.A
screeni
ngchestcomputedtomogr aphyformali
gnancycouldbeofusei fthecl
ini
cal
diagnosisofhypot
hyr
oidi
sm isnotconfi
rmedbyt hyr
oid-
sti
mul at
inghormone.

7. A65- year-
oldmanunder goespericar
diocentesi
sforalargeeffusion.Fluid
analysisi snotableforadenosi
nedeami naselevel100U/L,protei
n3g/ dl
,glucose30
mg/ dl,lactatedehydrogenase(LDH)120mg/ dl
,andspecif
icgravit
y1. 020.Numer ous
atypicalandmesot heli
alcel
lsareseen.Theserum glucoseis120mg/ dl,pr
ot ei
nis4
g/dl,lactatedehydrogenase(LDH)is150mg/ dl,
andwhi t
ecountis15, 000wi tha
normaldi f
ferent
ial
.

Wi
thwhi
choft
hef
oll
owi
ngar
ethesef
indi
ngsmostconsi
stent
?

a. I
diopathic.
b. Hypothyroidism.
c. Malignant.
d. Tuberculosis.
e. Autoimmune.

ANSWER

D.Tubecul
osi
s

Whenper icardialfl
uidisobt ai
neddur ingper i
cardiocentesis,i
tshoul
dber outi
nel
y
sentforbot hchemi calandcyt ologicalexami nation.However ,theet
iologyof
peri
cardialeffusionsisrarelyestabl i
shedt hroughexami nati
onofpericardialf
lui
d;
sampl i
ngoft heper i
cardium itselfistypicall
yneeded.Unf ort
unatel
y,mal i
gnant
eff
usionsr arelydemonst ratemal ignantcells.Ahi ghtri
glyceri
decontentofan
eff
usionischar act
eri
sti
cofmyxedema.Anadenosi nedeami naselevelof>40U/ L,
el
evatedpr otein,andLDHl evelsar esuggest i
veoft uberculosis.

8. A21- year
- ol
df emal estudentwithrecentupperrespirat
orysymptomspr esent
s
compl ai
ningof12hour sofleft
-sidedchestpain.Thepai nisdescri
bedasshar p,
worsenswi t
hbr eathing,andimpr oveswithleaningforward.Thereisnochangewi t
h
eat
ing.Shehaspr eviouslybeenact i
ve,exerci
singregular
lywithoutsymptoms.Her
el
ectrocardiogram (ECG)i sshowni nFi
gur e1.

Whi
choft
hef
oll
owi
ngwoul
dbet
henextappr
opr
iat
edi
agnost
ict
est
?
a. Hepatobil
iaryult
rasound.
b. Comput edtomogr aphyangi
ographyofchest
.
c. Esophagogast r
oduodenoscopy.
d. Exerci
sest r
esstest.
e. Echocardi
ogr am.

ANSWER

E. Echocar
diogr
am

Thispat i
entpr esentswi thclassicsymptomsofpericardi
tiswi t
hpl euri
ti
cshar pchest
pain,betterleaningf orward.HerECGshowst ypi
caldif
fuseSTel evati
ons,wi t
hPR
depressional sonot ed.Anechocar di
ogr
am wouldbehel pful,bothinconf i
rmingt he
diagnosisofper i
carditi
sbypr esenceofapericar
dialeff
usion( seeni napproximat el
y
60%ofpat ients)andi nruli
ngoutt amponade.Withthispatient'
scl assi
csympt oms
andECGf indings, t
hereisnor easonatthi
sti
met opursueal ternateeti
ologieswith
esophagogast roduodenoscopy, st
resst
esti
ng,hepatobil
iaryultrasound,orcomput ed
tomogr aphyangi ographyoft hechest.

9. A60- year
-ol
dwomanwi t
hdiabet
esmel l
it
usandahistor
yofmant elradi
ati
on
forl
ymphomaatage15present
stoyouroff
iceforeval
uat
ionofpr
ogressivedyspnea
onexer
ti
on.Onexami
nati
on,herbl
oodpressureis90/70mm Hgwithapul sus
paradoxusof8andahear trat eof80bpm.Shehasj ugul
arvenousdistensionwit
h
posit
iveKussmaul’ssign,
clearlungs,normalhear
tsoundswithoutmur mur,pal
pabl
e
enl
argedliverwit
hnoappr eciableasci
tes,and2+pit
ti
ngedemabi l
ateral
ly.An
el
ectrocar
diogram demonstratesinf
eri
orQwavesandl owQRSvol tage.

Whi
choft
hef
oll
owi
ngi
sthemostl
ikel
ydi
agnosi
s?

a. Peri
cardialeffusion.
b. Tri
cuspidst enosis.
c. I
schemi ccar diomyopat hy.
d. Constr
ictiveper i
cardit
is.
e. Pulmonar yhyper tensi
on.

ANSWER

D.Const
ri
cti
veper
icar
dit
is

Basedont hehi st
oryandphysi calexami nation, themostl ikel
ydi agnosi si s
const ri
ct i
veper i
cardi
tis.Constrict
iveper icardit
isf rom chr onicinflammat ioncauses
i
mpai redf il
li
ngoft hevent ri
clesandr educedvent r
icularfunction.I ncreased
pericar dialthicknessi sgenerall
ypr esentbutnotr equi r
edf orthedi agnosisor
treatment .Historycanr evealcommonr iskf actors, i
ncludingpr iorcomput ed
tomogr aphysur gery,chestradiati
on, andr ecurr entper i
car dit
is.Onhi st
ory, pat
ients
mayr epor tfatigue,edema, dyspnea, andabdomi nalswel l
ing.Typi cal l
y,thereisa
l
ongdel aybet weent hei ni
ti
alper i
cardialinflammat ionandconst ri
ct i
vedi sease.On
exami nat ion,Kussmaul ’
ssignandot herf indingsofr i
ghthear tfai
lur earepr esent.
 

Dif
ferentialdiagnosisincludesr i
ghthear tfail
uresuchasf rom acut eri
ghtventri
cul
ar
myocar dialinfarct
ion,tri
cuspidst enosis, orrestri
cti
vecar diomyopat hy.
Echocar diographyisusef ulf
ordi agnosi s, canassessf orthickenedorcal ci
fied
pericardium, andcanexcl udeot hercauses.Car di
accat heterizati
onhas
hemodynami cwavef ormschar acteri
sticofconst ri
cti
onandi susef ulf
or
dif
ferentiati
ngr estr
icti
vecardiomyopat hy.Pat ient
swi thprevi ouschestradiati
onare
atincreasedr iskforcoronaryar terydiseaseandval vularhear tdi
sease,butthat
woul dnotr esultintheexami nationfindingsseeni nthispat ient.Asympt omatic
pericardialeffusi
onwoul dbeexpect edt obeassoci atedwi thapul susparadoxus.

10. A57- year-ol


dwomani sseeingyouf orthefi
rstt
imeandhasbeenr efer
red
becauseofdyspneaandedema.Shehadr adi
ati
ontherapytoherchestforHodgkin'
s
lymphomawhenshewas17year sold.Shehasdonewel lunti
labout6monthsago,
whenshenot edtheonsetofexer
cise-rel
atedshort
nessofbr eathandmildbil
ater
al
l
owerextr
emit
yedema.Overthelastf
ewweeks,t
hishaswor senedandshenowhas
pi
tt
ingedematohermi
dcalves.Shedeni
espalpi
tat
ions,
fever,cough,
orabdomi
nal
bl
oati
ng. 

Onexami nat
ion, sheappearswel lwit
habl oodpressur
eof120/80mm Hgwi th
minimalchangewi t
hinspir
ation.Herheartr at
eis78bpm andr egular
.Herlungsare
cl
ear.Herjugularvenouspr essureiselevatedtoherearlobeswhilesit
ti
ngupright
andtherearepr ominentwavef or
msobser ved.Therearenomur mursandnoext ra
cardi
acheartsounds.Herabdomeni snor malexceptforali
veredgefeltt
wof i
ngers
bel
owt hecostalmar gin.Theedemai sevi dent.
 

Youor deranelectr
ocar diogr
am thatreveal
snormalsi nusr hythm andborderl
inelow
volt
age,butisotherwisenor mal.Anecho/ Doppl
erisobt ai
nedandt heresul
tsare
showni nFigure1andVi deo1.Perti
nentadditi
onalresultsincludeamaxi mal
tr
icuspidregurgi
tantjetveloci
tyof3.0msec.Leftventricular(LV)andrightventr
icul
ar
systol
icfuncti
onar enor mal.

Basedonthecl
ini
calhist
oryandechocar
diogr
ams,
whi
choft
hef
oll
owi
ngi
sthemost
l
ikel
ycauseofherr
ightheartf
ail
uresympt
oms?

Fi
gur
e1.
Vi
deo1.

htt
p:/
/media.
mycrowdwisdom.com.s3.
amazonaws.com/
acc/
quest
ions/
asset
_per
ica
rdi
al/
1754b2ae-
b00e-4571-aca2-
49b1f2747c77.
mp4

a. Restri
cti
vecar di
omyopathy.
b. Radiati
on-inducedtri
cuspi
dregurgi
tati
on.
c. Chronicthromboembol i
cpulmonarydisease.
d. Constri
cti
veper i
cardi
ti
s.
e. Pulmonar yfi
brosi
sandcorpulmonal e.

ANSWER

D.Const
ict
ivePer
icar
dit
is
Thecor rectansweri sconst
rictivepericarditi
s.Theevidenceofr ighthear tfail
urein
associat ionwi t
ht heechocar diographicf i
ndingsconfirmt hediagnosi s.I
n
const r
iction, t
herightatri
alpr essuresar eelevatedandt hereisveryr apidear l
yfill
ing
ofther ightvent r
icle(rapi
dydescent ).Ther eisoftenevidenceofasept al"bounce, "
correspondi ngtot hehighimpact  earl
yr apidfi
lli
ngoftheLVbef orethel i
mi tsofthe
LVdi mensi onresul t
sinstoppageofi nfl
owt ot heLV.Pat i
entswi t
hconst ricti
ve
pericarditisusuallyhavenor malpul monar ypressures,anda>25%r eductionint he
mitralinfowDoppl ervelocit
ieswi thinspirati
on.Thevi deoandf iguredemonst rate
thesef i
ndi ngs. 

Thel ackofpul monaryhypert


ensioninthiscaseisi nconsistentwit
ht hediagnosesof
rest
ricti
vecar di
omyopathy,chronicthr
omboembol i
cpul monar ydisease,andcor
pulmonal e.Radiat
ionheartdi
seasecani ncludethef ormat i
onoffi
brosi st
hataffects
theper i
cardium,myocardium,conductionsystem, orcar di
acvalves.Itcanalsor esult
i
nsi gnif
icantlunginj
ury.Whil
et heseareallrel
evanti nthiscase,t
hef indi
ngsont he
echo/ Dopplerconfi
rmthediagnosisofconst ri
cti
veper i
car di
ti
s.

11. A26- year-oldmanpr esentswi t


hchestpai n.Heexper i
encedanupper
respirat
oryinfection2weeksago.Fort hepastweek, hehashadchestpai nt hatgets
wor sewi t
hinspirati
onandsupi neposi t
ion.Therei snoexerti
onalquali
tytot hepain;
otherwise,onseti sunpredi
ctable.Hehashadnof evers,
chil
ls,orsweats.Hehasno
cough.Onexami nati
on,helookswel l
.Hisbloodpr essur
eis118/ 70mm Hg, hisheart
rateis70bpm, andhei safebril
e.Lungsar eclear.Ani nt
ermitt
entperi
cardialrubis
noted.ChestX- rayi snormal.Hiselectr
ocardiogram intheoffi
ceshowssi nusr hyt
hm,
buti sotherwisenor mal.

Whi
choft
hef
oll
owi
ngi
sthemostappr
opr
iat
ether
apeut
icst
rat
egyatt
hist
ime?

a. I
buprofenf
or8- 12weeks;colchi
cinefor1-2weeks.
b. Predni
sonefor8-12weeks;colchici
nefor1-2weeks.
c. Oxycodonefor1-2weeks;prednisonefor8-12weeks.
d. I
buprofenf
or1- 2weeks;col
chici
nef or8-12weeks.
e. Predni
sonefor1-2weeks;oxycodonef or8-12weeks.

ANSWER

D.I
bupr
ofenf
or1-
2weeks;col
chi
cinef
or8-
12weeks.
Thisyoungmanhasper i
car diti
s.Itismostl ikelyidiopat hi corviralinli
ghtofhi s
recentupperr espiratoryinfection.Thepr edominantt her apeuticconcer nsarer el
iefof
sympt omsandpr eventi
ngr ecur rence.Nonst eroidalant i- i
nflammat orydrugs( NSAI Ds)
areveryeffectiveati mmedi at epainr eli
efandshoul dbeconsi der edfi
rstli
ne.The
additi
onofcol chicinehasbeenshownt opreventr ecurr encesi nsever alprospective
randomizedt ri
als.Pr ednisonehasl it
tleroleinthepr imar ycontempor ary
managementofper i
carditi
sandmaybedi ff
icul
tt oweanoverext endedper iodsof
ti
me, notwithstandingt hesi gnifi
cantsi deeffectsofchr oni csteroids.Narcoticshave
nospecificant i
-infl
ammat or yactionsandar einferiortoant i
-i
nflammat ory
approaches.NSAI Dsshoul dbeusedf orsympt om sever ity,gener al
lyabout2weeks.
Colchici
neshoul dbecont i
nuedf or2- 3mont hst opr eventr ecurr
ence.

12. A20- year


-ol
dmanpr esent
swit
ha1- weekhi
stor
yofrhi
niti
s,myalgi
a,f
ever
,and
chestpain.Hisexaminati
onisnot
abl
eforaneasi
lyaudi
bleper
icar
dialf
ri
cti
onruband
isother
wi seunremarkabl
e.

Whichofthefol
lowi
ngagent
smaycont
ri
but
etoani
ncr
easedr
iskofr
ecur
rent
per
icar
dit
is?

a. Aspiri
n.
b. I
bupr ofen.
c. Corti
cost er
oids.
d. Colchicine.
e. I
ndomet hacin.

ANSWER

C. Cor
ti
cost
eroi
ds

Thi
spatientmostli
kelyhasvi r
alpericardi
ti
s.Whi
lecorti
costeroi
dsmaybeef f
ecti
ve
att
reat
ingacutepericardi
ti
s,theyshouldnotbeusedasf irst
-li
netherapyasthey
mayleadt oanincr
easedr i
skofr ecurr
ence.Theotheragentsdonoti ncr
easetherisk
ofr
ecurrence,
andinf act,
colchici
nehasbeenshownt oreducetheriskofrecurr
ent
per
icar
diti
s.
13. A65-
year
-ol
dwomani
sbr
oughtt
othecat
het
eri
zat
ionl
abf
ort
reat
mentof
car
diact
amponade.Shei
suncomf
ort
abl
eandsi
tt
ingat45degr
eesont
hegur
ney.
Herbl oodpressur
eis100/85mm Hg, herheartr
ateis110bpm, andherbreat
hingi
s
rapid.Thevenouspressurei
svisi
bleatthejaw,r
alesarepresenti
nthelungbases,
heartsoundsar edi
stant
,andther
earenopr ecordi
alimpul
ses.Herpulsesare
thready.

Underf
luoroscopi
cguidance,
aperi
cardi
ocentesisneedl
eisplacedint
heepigast
ri
um
anddir
ectedposter
ior
lytowardt
helef
tshoulder.Wit
hneedleaspir
ati
on,bl
oodis
obt
ained.

Whi
choft
hef
oll
owi
ngi
sthenextmostappr
opr
iat
est
ep?

a. Transducepressuref r
om per
icar
diocent
esi
sneedl
e.
b. Sendbloodforanal ysi
s.
c. Placeperi
car
di aldrai
n.
d. Perfor
mr i
ght-heartcathet
eri
zati
on.

ANSWER

A.Tr
ansducepr
essur
efr
om per
icar
diocent
esi
sneedl
e.

Duringur gentper icardiocentesi s,eit


herechocar diogr aphicorf l
uor oscopi cguidance
canbeusedt ofaci l
it
atet hepr ocedur e.Whent heaspi ratedf l
uidi ssangui nous, i
tmay
beduet oeitherhemoper i
cardi um (e.g.,
foll
owingat r
ialf i
brill
ationabl ation)or
myocar dialpunct ure,awel l
-recogni zedcompl icationofper i
cardiocent esis.Themost
expedi t
iousmet hodf ori dentifyingthesour ceoft hesangui nousaspi r
at eisto
tr
ansducet hepr essur efrom theneedl e.Inthecaseofmyocar dialpunct ure,the
wavef ormt ransducedwi llbechar acteri
sti
cofint ravent ri
cularpr essur e;ifinthe
peri
car dialspace, thewavef or m wi l
lnotdemonst ratet hewavef ormsofi ntracardiac
pressur es.Echocar diographicgui dancemaydecr easet hispar ticularcompl i
cationof
peri
car diocentesiswhencompar edwi t
hf l
uoroscopi c- guidedper i
cardi ocentesis.
Placingadr ai
nwi thoutconf irmat ionoft helocationoft heneedl eiscont raindicated.
Theot herdi agnost i
copt ionswoul dnotbet i
mel yint hisur gentsi tuati
on.

14. A60-
year
-ol
dobesemal
esmokerpr
esent
swi
thsubst
ernalpl
eur
it
icchestpai
n.
Onemont hago, herecall
sasever eepi sodeofhear tburnaftereati
ngal ar
gemeal .His
wiferecallshi
m sweat i
ngpr ofusel
y.Hef el
tpoor lyandf ati
guedf orthedayafter.I
n
thepastweek, hi
schestpai nhasbeenwor sewhi l
esupineandwi thdeepinspirat
ion.
Hishistoryisnotabl
ef orpoorlycont r
olleddi abetes,hypertensi
on, anddysli
pidemia.
Onexami nati
on,heisuncomf or
table.Hisbl oodpr essureis160/ 110mm Hg, hi
s
heartrateis85bpm, andhi stemperatur ei
s37. 9° C.Jugularvenousdi st
enti
oni snot
visi
ble.Lungsar ecl
ear .Scrat
chymur muri saudi ble.Abdomeni sobese.Venous
stasi
schangesar eappar entoverthelowerext remities.

Hi
sel
ect
rocar
diogr
am i
sshown(
Figur
e1)
.

Whi
choft
hef
oll
owi
ngwoul
dbet
henextmostappr
opr
iat
edi
agnost
icst
ep?

a. Anti
nuclearanti
body.
b. Venti
lat
ion-perf
usionscan.
c. Coronar
yangi ography.
d. Chestcomput edtomography.
ANSWER

C. Cor onaryAngiography
Thi spatientpresentswi t
hsympt omsconsi stentwi t
hper i
carditi
s,butalsohas
car di
ovascul arri
skfactorsforcoronaryart
erydi sease.Thehi storyofheartburn,
diaphor esis,andfeel
ingpoor l
yforadayi sstrongl ysuggest i
veofamyocar di
al
i
nf arcti
on( MI)athome, forwhichhedi dnotseekmedi calat t
ention.The
electrocardiogram i
salsosuggest iveofapr i
orant eri
orMI .Thisclini
calscenarioi
s
highlysuggest i
veofpostinf
arcti
onper i
cardit
is(e.g.,Dressler'
ssyndrome) .Dressl
er'
s
syndr omehasbecomeanuncommoncompl icationofST- segmentel evati
onMIi n
themoder ner aduetot herapidtri
ageandr eperfusiont r
eatmentf orMI. 

Abr oaddi f
ferent i
alshouldal waysbeentert
ainedwhenadi agnosisofper icardi
ti
sis
suggest edbycl inicalevaluati
on.Otherdi
agnost i
cconsi
derati
onsi ncludepul monary
embol ism withr esulti
ngpl euri
sy,
aorti
cdissecti
onwithproxi
malext ensi
onr esul
ti
ng
i
nper icardi
albl ood, andacut ecoronar
ysyndromes.Inthispatient
, anunr ecogni
zed
MIist hemostl i
kel ydiagnosisandisbestconf i
rmedwithcoronaryangi ography.I
f
coronar yangiogr aphyi snotdiagnosti
c,t
heot herdi
agnosti
copt i
onswoul dbe
reasonablet opur sue.

15. A45- year-ol


dmani sadmi tt
edt ot hehospi t
alaf t
ersuf f
eri
ngsuddencar diac
arrestathiswor kpl
ace.Acowor kersuccessf ull
yut i
li
zedthewor kplaceautomat ed
externaldefibr
il
lat
or,whi
chr ecordedvent r
iculartachycardia.Hishospit
alevaluati
on
revealsapr evi
ouslyundiagnosednoni schemi cdilat
edcar diomyopathy.Heist r
eated
forsignsandsympt omsofhear tfail
ure, andonhospi t
alday3her ecei
vesan
i
mpl antablecardiover
ter
-defibr
il
lator(ICD) .
 

Thatnightyouarecall
edbythefloornursewhoisconcernedthathi
sbloodpressur
e
hasdroppedandhi shear
trat
ehasi ncreased.Hecomplai
nsofchestti
ghtnessand
short
nessofbreath.Hehasclearlungs,buthisj
ugul
arvenouspressur
eiselevat
ed.
Hi
sbl
oodpr
essur
eis80/
50mm Hgandhi
shear
trat
eis120bpm.

Hi
select
rocardiogr
am showsnoi
schemi
cpat
ter
ns.Anechocar
diogr
am r
eveal
sa
smal
lperi
cardialef
fusi
on. 

Youtr
ansferhimtothei
ntensi
vecar
eunitandplaceacentr
alvenouscat
het
eri
nhi
s
ri
ghti
nter
naljugul
arvei
ntomeasurehi
srightheartpr
essur
es.
 

Youobt
aint
her
ightat
ri
al(
RA)pr
essur
etr
aci
ngf
rom yourcent
rall
ine(
Figur
e1)
.

Whi
choft
hef
oll
owi
ngi
sthemostappr
opr
iat
enextst
ep?

a. Pericar
diocentesis.
b. Thrombol yt
ictherapy.
c. Bloodtransfusion.
d. Aggressi
vediur
esis.
e. Surgi
calper
icar
dialwindow.

ANSWER

A.Per
icar
diocent
esi
s

TheRApr essuretracingi sconsi stentwithcardiact amponade,andt hepatientneeds


pericardi
ocentesi
s.Thi sscenariodescr ibesapr obabl eri
ghtventri
cular(RV)
perforati
onfrom theICDpr ocedur e.Becausei tisacut e,t
heamountofper i
cardi
al
fl
uidmaybesmal l.TheRAt racingshowsabl untedydescentconsi stentwithcardi
ac
tamponade.Thi sisduet othei nabi l
it
yoft heRVt of il
linear
lydiast
oleoncet he
tri
cuspidvalvehasopenedast heRVdi astoli
cpr essureisbeingaffectedbythe
elevatedpericar
dialpr essure. 

Hissyst emicbl oodpr essur eisl


ow, andapar adoxicalpul semaybedi f
fi
culttodet ect.
Althoughabl oodt r
ansf usionmi ghtbehel pf ulinthisinstance,itshouldnotbet he
fi
rststepaf t
err ecognizingcar di
act amponadei spr esent.Ifanythi
ng,heneedsf l
uids
andi notropicsuppor t.Aggr essi
vedi uresiswoul dr educepr eloadandmaywor senhi s
signif
icanthemodynami ccompr omi sebyr educingvent r
icularfil
li
ng.Thrombol yt
ic
therapyhast hepot enti
alt oworsent hesi tuation.Sur gi
calconsul t
ati
onfora
pericardialwindowmaybeconsi der
ed, butt hef i
rststepshoul dbeper i
cardiocentesis,
ast hi
si susuallyallthati srequi
r edtost abili
zet hepat i
entandr e-establ
ishhis
hemodynami cstatus.

16. A20- year


-oldwomanunder goessur gicalr
epai rofalargesecundum at r
ial
septaldef
ectviaamedi anster
notomy.Twoweeksl ater
,shepr esentswithcli
nical
si
gnsandsympt omsofl ar
geperi
cardialeff
usion.Sheunder goesper icar
diocentesi
s,
whichrevealsopalescentmil
kyfl
uid,
whi chbyanal ysisishighint r
igl
yceri
desand
prot
ein.

Whi
choft
hef
oll
owi
ngi
sthemostappr
opr
iat
eini
ti
alr
ecommendat
ioni
nthi
spat
ient
?

a. Di
etwi
thmedi
um-
chai
ntr
igl
ycer
ides.
b. Colchicine.
c. Steroids.
d. Surgicalthoraci
cductl
igat
ion.
e. Pericardiect
omy.
ANSWER

A. Di
etwi
thmedi
um-
chai
ntr
igl
ycer
ides.

Thecor rectanswerf orachyl opericardialeffusioni stot rydi et


aryeffortsbef ore
surgery.Chyl oper i
cardium i sararecompl icat i
onofcar diacsur geryandt endst obe
rel
at edtodi sruptionoft horacicductt ri
butaries.Per i
cardi ocentesisisi ndicatedin
thesepat i
ent s.Becausemedi um chai ntri
glycer i
desar eabsor beddi rectl
yi ntothe
portalsystem andnott hroughl ymphat i
cs,al ow- fatdiett hatincludesonl yt hesefats
i
st hemostappr opriatefirstli
neoft r
eatment .I nthemaj orityofcases, thisal ongwit
h
drainageoft hefluidwillallowcl osur eofthef istul
ousconnect ionandavoi dmor e
i
nvasi vether apies.Surgicalper i
car dialwindowandl igationandr esect i
onoft he
thoracicductj ustabovet hedi aphr agm havebeenr epor tedasef fectivetreatment
optionsint hiscondi ti
on, butusual lyar ereser vedforper sistentorr ecur r
entchyl ous
pericardi
alef fusions.Phar macol ogi ctherapiesar enoti ndi catedint het r
eat mentof
thi
scondi ti
on.

17. A70- year-


oldwomanwi t
hahi st
oryofhypert ensi
onanddysl i
pidemiapresents
wit
hst utteri
ngsubst er
nalchestpai noverseveraldays.Ont hedayofadmi ssi
on,she
devel
ops10/ 10chestpai natrest.Herinit
ialvi
talsar eabl oodpressureof160/90
mm Hgwi t
hahear trateof40bpm i ncompl etehear tblock.Hercardiacenzymesar e
posit
ive.Herelectrocardiogr
am (ECG)demonst ratesdiffuseST-segmentdepr ession.
Sheisdi agnosedwi thanon- STel evati
onmyocar di ali
nfarcti
on,
andi sr
eferr
edtot he
cardi
accat heteri
zati
onl aborat
ory. 

Atempor ar
ypacingwireisimplanted.Diagnosti
cangiographyviather i
ghtfemoral
ar
teryreveal
sobstruct
ivediseaseinther i
ghtcoronar
yartery.Percutaneouscoronary
i
ntervent
ion(PCI)i
spursuedandi sdiff
icultduetothetor
tuosityofthetargetvessel
.
Postprocedure,
thepatienti
schest-painfree.Unfr
acti
onatedhepar i
nandt icagr
elor
ar
eadmi nist
eredfol
lowingPCI. 

Thepatientisadmit
tedtothecoronar
ycareuni t
.Approxi
mat el
y10-12hour slater
,
thenursecallst
oinfor
m youthatthepati
entishypot ensi
ve.Thepatientreport
smi ld
dizzi
ness,short
nessofbreat
h,chestpai
nwi thdeepi nspir
ati
onandleaningf orward,
andmi l
dnausea.Shehasasupi nebloodpr essureof80/60mm Hg, wi t
hapul serate
of80bpm ( vent
ri
cul
arl
ypaced).Thecatheterizati
ongroinsit
elooksokay.

Whi
choft
hef
oll
owi
ngi
sthenextbestst
epi
nhermanagement
?
a. Incr
easepaci
ngrate.
b. ECG.
c. Echocar
diogr
am.
d. Computedtomographyoft
hechest
.
e. Computedtomographyoft
heabdomen.

ANSWER

C. Echocar
diogr
am

Thecor r
ectansweri sanechocardiogram.Thi spati
enthasdevel opedat raumati
c
i
at rogenicper
icardialef
fusi
onandt amponadeasal atecompl i
cationofPCIort he
tempor arypacemakeri mplant
ation.Althoughtheincidenceislowf ol
lowi ngPCI(0.2-
0.4%) ,pr
omptr ecogniti
onisessentialsothatpreventi
onoft hetransitionf rom
effusiontotamponadecanbeat tempt ed(e.g.
,byreversinganticoagulat ion)ort
he
hemodynami ccollapsecaneitherbeaver tedortr
eated( per
icardiocent esis).
 

Thepr esentat
iondependsonseveralfactors,i
ncludi
ngthestr
uctur
ethati
s
perfor
ated,thedevi
cethatcausedtheper forat
ion,t
hebasel
inehemodynamicstat
us
ofthepat i
ent,
andthelevelofant
icoagulationpresent
.Symptomscanbefair
ly
nonspecifi
c. 

I
nt hi
svignette,r
iskfactor
sincludethediffi
cul
tyofthePCIandt hetemporarypacer
pl
acementi nassociationwit
ht heuseofpot entanti
coagul
ant s.Thispati
entli
kel
y
developedadi st
alcoronarywireperfor
ationorar i
ghtvent
ricularperf
orati
on
unrecognizedatthetimeofthepr ocedure,butthentheper
icar di
aleff
usionworsened
i
nt hesett
ingofcontinuedanticoagulat
ion. 

AlthoughanECGcanbeconsi der
ed, sheisbei ngpacedandhersympt omsar eoutof
propor ti
onf oranischemi ccauseoft hehypot ensi
on.Similar
ly,thereissomeover l
ap
i
nsympt omat ol
ogywi t
ht hedevelopmentofacut epulmonaryembol i
sm, butthi
sis
l
oweront hedi f
fer
entialinwhatwasot herwiseanambul ator
ypat ient
.Hersympt oms
al
socoul dbeamani festati
onofretroper i
tonealhemor rhage(aconcer nthatwould
promptr eturntothecar diaccatheterizati
onl aborator
y),butinthissettingan
emer gencyechocar diogram shouldbet hef i
rststudyordered.Admi nisteri
ng
i
ntr avenousf lui
dsisessent ialtotempor aril
yr esuscit
atethepatient.I
ncr easi
ngthe
paci ngratewoul dlikelynotbeofmuchval ue.

18. A24- year-ol


dmanpr esent
stot
heemergencyroom withtheonsetofchestpai
n
over1day.Hedescr i
besthepainasshar
pandstabbing,wit
hsomer adiat
iontohis
lef
tshoul
der.Hisbreathi
ngalsoisal
it
tl
eshall
ow.Her epor
tsfeverwi
thchill
sfort
he
past24hours.Heiscur r
entl
yonnomedicat
ionsandhasnoknownal l
ergi
es. 

Onadmi
ssi
on,
hei
sfebr
il
eto37.
9ºC.Hi
swei
ghti
s65kg.Hi
sbl
oodpr
essur
eis
140/92mm Hg, heartrateis104bpm andregul
ar,andrespir
ator
yratei
s22
breat
hs/min.Hisphysicalexaminati
onisnor
mal,exceptforathr
ee-component
peri
cardi
alrub.Hi
sel ect
rocardi
ogram i
sshowninFigure1.

Whi
choft
hef
oll
owi
ngi
sthenextbestst
epi
nmanagement
?

a. I ndomethaci
n50mgbidwithcolchicine0.6mgdai lyfor6weeks.
b. Aspi r
in800mgtidf
or6weekswi thcol chi
cine0.6mgbi dfor3months.
c. Aspi r
in325mgbidwit
hcolchi
cine0. 6mgbi df
or4weeks.
d. Pr edni
sone60mgdail
ywit
hcol chicine0.6mgdai l
yfor6weeks.
e. I bupr
ofen800mgti
dfor7-
10dayswi thcolchi
cine0.6mgbi df
or3
mont
hs.
ANSWER

E. I
bupr
ofen800mgt
idf
or7-
10dayswi
thcol
chi
cine0.
6mgbi
dfor3mont
hs.

Thecorr
ectopti
oni sibuprofen800mgt i
df or7-
14dayspluscolchi
cine0.
6mgbi d
for3months.Thispatienthasclassi
csympt omsofacut
eperi
cardit
isconf
irmedby
hisel
ect
rocardi
ogram wi t
hSTel evat
ionandPRdepressi
oninmul ti
plel
eads.Thefi
rst
-li
net
reatmentishigh-doseaspirin(
650-975mgt i
d)ori
buprofen(800mgt i
d)for7-
10days,al
ongwi thcolchici
nefor6mont hs. 

I
nt heICAP( I
nvest i
gat iononColchicineforAcut ePericar
diti
s)trial
, addi
ngcol chici
ne
for3mont hst ost andar danti
-i
nfl
ammat oryregimens( ei
therhigh- doseaspi ri
nor
nonsteroidalant i-infl
ammat orydrugs[NSAI Ds])reducedther elati
ver i
skofi ncessant
orrecurrentper icar dit
isby44%.Thest andar dhigh-doseanti-infl
ammat ori
eswer e
usedonl yf orthef irst7- 14days,withtaperingbyt heendof3- 4weeks.Thebenef i
ts
ofcolchicinewer ei ndependentoft heuseofei theraspiri
norNSAI Ds.Pr ednisone
generall
yi sr eservedf orthoseall
ergictoori nt
olerantofaspir
inorNSAI Ds, and
i
ncreasest her i
skofr ecur r
entperi
cardit
is.Apr otonpumpi nhibitorshoul dbeadded
totheant i
- infl
ammat oryregimen.

19. A50- year-oldwomanwi t


hahi storyofbr eastcancer,di
abetes,hypertensi
on,
andcor onaryarterydiseasepresentst otheemer gencyroom withprogressive
shortnessofbreath.Shehadbeenf eel
ingwel lunt i
labout6mont hsagowhenshe
noti
cedshehadbegunt oloseweightwi thouttryingtodoso.Shenowcompl ai
nsofa
2-weekper i
odwi t
hwor seningdyspneaevenatr est.Hermedicati
onsinclude
metformin500mgbi d,l
isi
nopril20mg, aspi
ri
n81mg, clopi
dogrel75mg, metoprol
ol
25mgbi d,l
evothyroxine125mcg/ kg,andchl orthali
done25mgeachday.  

Onexami nat
ion,herbloodpressureis90/70mm Hgandhear trat
eis100bpm.She
isaf
ebril
e.Herbodymassi ndexis28.3.Shehasclearl
ungfi
elds,
butherj
ugul
ar
venouspressureiselevat
edabovet heclavi
cleupr
ight,
butt
hereisnoedema.The
heartsoundsaredistant,
butnomur mur sareaudi
ble.

Anelectrocar
diogram i
snormalexceptf
orl
owvoltage.HerchestX-
rayi
sclear
exceptforcardi
omegaly.Youobtai
nanechocar
diogram thatr
eveal
sevi
dencefora
peri
cardialef
fusi
on(Figur
es1and2) .
Fi
gur
e1.
Fi
gur
e2.

Basedont hepr
evi
ousi
nfor
mation,
whichofthefoll
owingchar
act
eri
sti
csoft
he
present
ati
onwouldwar
rantur
gentper
icar
diocentesi
s?

a. Thefactt
hatthecausei slikel
ymal ignancy.
b. Thehemodynamici nstabil
ity.
c. Thel
ikeli
hoodthi
sisper icardialhemorrhage.
d. Theechocardi
ographicfindings.

ANSWER

B. Thehemodynami
cinst
abi
li
ty

Thecorrectopti
oni
sthehemodynami cinst
abil
it
y.Whiletheechocardiogram reveal
s
ther
eisnotonlyperi
car
dialef
fusi
onpresentbutalsorightvent
ri
cul
ardi astol
ic
coll
apse(Figur
e3)and>25%mi tr
ali
nfl
owvar i
abil
ityconsist
entwit
ht amponade, thi
s
wouldnotdictateanur gentresponseift
hepat i
entwer ehemodynami cal
lyst
abl e.Her
weightlossandlargeef f
usionsuggestthi
spr ocesshasbeengoi ngonf oralongt i
me
andthecausei shighlyli
kelyrelat
edtoherpr i
ormal i
gnancy.Whil
eitispossiblethe
charact
eroftheper i
cardialfl
uidishemorrhagic(especi
all
ygivenit
slikel
ycauseand
theuseofdualant i
platel
ettherapy),
thi
sshoul dhavenobear i
ngont heurgencyofa
peri
cardialt
ap.

Fi
gur
e3.
20. A52- year
-oldmanpresent
stotheemergencyroom wi
thabdomi nalpainand
bl
oati
ng,lowerextremi
tyedema,anddecr
easedappetit
eofseveralmonthsdurati
on.
Onexaminat i
on,heisf
oundtohavemarkedjugul
arvenousdistenti
on,asci
tes,
and
per
ipher
aledema.

Echofindi
ngsar
eshown(
Figur
es1,
2,3,
4).Whi
choft
hef
oll
owi
ngi
sthemostl
ikel
y
diagnosi
s?

Fi
gur
e1.
Fi
gur
e2.
Fi
gur
e3.
Fi
gur
e4.

a. Restri
cti
vecardi
omyopathy.
b. Chronicobst
ruct
ivepulmonarydi
sease(
COPD)
.
c. Constri
cti
veperi
cardi
ti
s.
d. Dil
atedcardi
omyopathy.

ANSWER

C. Const
ri
cti
veper
icar
dit
is

Theinferiorvenacava( IVC)isdilat
ed, asignofel
evatedr i
ghtatri
alpressure.Mitral
inf
lowvar iabi
li
ty(i
nspirator
yf al
loft hepeakEwavevel ocityof>25%ont hefir
stbeat
fol
lowingt heonsetofinspirati
on)isar efl
ecti
onofthedi ssoci
ati
onofi nt
r at
horacic
andintracardiacpressures.Oni nspirat
ionthewedgepr essurefal
ls,butleft
ventr
icular(LV)diastol
icpressur eremainsconstant
,hencet heleftatr
ial
/ LVpressure
gradientdecreases.Expirator
yfl
owr eversalinthehepat
icveinsisalsoaref
lect
ionof
thedissociat
ionofpr essuresdescr
ibedear l
ierandventr
iculari
nter
dependence.The
l
ow- veloci
tysuperiorvenacava( SVC)flowwi thminimalrespir
ator
ychangeisalso
characteri
sti
cofconst ri
ction.
 

COPDandast hmaexacer bati


onbot hhavemi tr
alinfl
owvari
abil
it
y,butt hesedonot
conformt othefi
rstbeatfol
lowingtheonsetofi nspi
rati
onandexpir
ation.Int hese
scenarios,t
hereismar kedrespi
ratoryvar
iat
ioni nSVCf l
ow.I
nrestri
ctive
cardi
omyopat hy,ther
ei snovariabi
li
tyofmit
ralandt ri
cuspi
dinf
lows, noexpi rat
ory
fl
owr eversali
nthehepat i
cveinsandt hemitrale'vel
ocit
yismarkedlydecr eased.
Pul
monar
yCi
rcul
ati
onDi
sor
der
s
Quest
ion:

1. A65- year -oldmanwi thsystemi chypertensionandpar oxysmalat r


ialfi
bril
lati
onis
hospitalizedf orshor tnessofbr eath.Onphysi calexam, hisHRi s75, BPi s155/88, andoxygen
saturationi s95%onr oom air.Hiscar diovascularexam isnot ablef oraJVPof10cm H20,
heartexam i sr egul arwi thaf aintS3,S4, andaI I/
VIholosystolicej ecti
onmur muratt heapex.
Cracklesar ehear di nbot hlungbases, andhehast racebilaterallowerext r
emi t
yedema.Hi s
echocar diogr am demonst r
atesanEFof65%, LVH, mil
d-moder atemi tr
alregurgitat
ion,and
pulmonar yar terysyst oli
cpr essur eesti
mat edat48mmHg.Hi sECGdemonst r
atesnor mal
si
nusr hythm wi thl eftventri
cul arhypertrophy.ChestCTshowedmi ldbibasil
arpulmonar y
i
nfil
trates, butot herwi senor mall ungpar enchymaandnoPE.Bedsi dePFTswer enor mal
exceptf ormi ldr educt ionindi f
fusioncapaci t
y.BNPi s135ng/ L.Hei scurrentl
ytaki ng
met prololsucci nate50mgdai ly,amiodar one200mgdai l
y, l
asix20mgdai l
y,andwar f
ari
n.

Whati
sthenextappr
opr
iat
est
epi
nthemanagementofhi
spul
monar
yhyper
tensi
on?

a. Si
ldenafil20mgt hreeti
mesdai l
y.
b. Disconti
nueami odarone.
c. I
ncr easelasi
xto40mgdai ly.
d. Ri
ghthear tcatheteri
zat
ion.
e. Pulmonar yventi
lati
on-perf
usionscan.

Answer
:

1. c. I
ncr
easel
asi
xto40mgdai
ly.

Whilerecentevi
dencepointstoani ncreasei
nthenumberofpatientswit
hWHOGr oup1
pulmonaryarter
ialhypert
ensi
on( PAH), t
henumberofpati
entswithpulmonar
yhypert
ensi
on
(PH)relat
edtochroniclef
theartdisease(WHOGr oup2)orchronichypoxi
cst
ates(WHOGroup
3)isfargreat
er.

Whilet hi
spatientdoeshavemoder atepul monaryhypertensionbyechocr it
eria,hepresents
withmul ti
plesignsandsympt omsofl eftheartf
ail
ure,andthedi agnosti
ctesti
ngper f
ormedso
farclearl
ysuggest sthathehasincreasedpulmonar yvenouspr essureli
kelyrelatedt
ochr oni
c
hyper t
ension,l
eftventri
cul
arhypert
rophy, anddiast
olicdysfunct ion.Thefi
rststepistomanage
hisincreasedpulmonar yvenouspressur ewit
hadditionaldiuresis.

Intheevaluati
onofpul monar yhypert
ension( PH),i
tisimport
anttofi
rstevaluateforl
eftheart
diseaseorlungdisease.Iftheseareconf i
rmedandt r
eatedwit
houtimprovement ,orofthe
sympt omsar eoutofproporti
ont othedegr eeofdisease,t
henaddit
ionalevaluati
onis
war r
anted.Ruli
ngoutchr onicthromboembol icdiseaseisanappropri
atenextstep.Oncet hese
stepshavebeencompl et
ed, i
fthepatientremainswi t
hPHt henadditi
onaltesti
ngi swarrant
ed
tofurthereval
uateforpul
monar yart
erialhypert
ension.

Forthi
spati
ent,
rightheartcathet
eri
zati
onisnotnecessar
yatt hisst
ageoftheeval
uat
ionunt
il
thepati
enti
sfurt
herdiuresed.AV/ Qscani
salsonotyetindicated;t
hepat
ienti
s
anti
coagul
atedmakingthromboembol icdi
seaselessli
kel
y,andt hemorel
ikel
ycauseofhi
s
i
ncreasedpulmonar
ypr essur
eshouldfir
stbetr
eated,aspulmonaryedemacanr esul
tin
abnormalV/Qresul
ts.Amiodaroneisunl
ikel
ytobecont r
ibut
ingher
einlightofthenormallung
parenchymaandrel
ativel
ynormalPFTs:themildl
yreducedDLCOmayal sobeseenduet o
pul
monar yedema.I
twoul dbeinappropr
iat
etobegintreatmentwi
thsil
denafi
lforPAHunt i
lthe
di
agnosisofPAHwer emade.

Quest
ion:

2. A49- year-oldwomanpr esent stoyourcl inicforeval uationofpr ogressivedyspneaon


exerti
onoverl ast2year s.Herhi stor yissignifi
cantf orobesi tytreat edwi t
h
fenfl
uramine/ phentermine20year sago, remotet obaccouse, andr emot eDVTassoci atedwith
tobaccouseandor alcontraceptivepi ll
s.Onphysi calexami nat ion,hear tratei
s72bpm, BP
142/85, Pulseoximet ryis95%onr oom ai randBMIi s45.Shei sinnoacut edistress.
Pulmonar yexami nat i
onisnot ablef orabi basil
arcr ackleswhi chcl earwi t
hcough.Car diac
exami nati
onr evealsanor malS1/ S2 withaI I
/VIholosyst ol
icej ectionmur muratherapex.Ther e
i
snoRVheave.Herext r
emi t
iesar ewar m, andtherei snoedemaorcl ubbing.HerECG
demonst ratesnormalsi nusr hythm wi thbor derl
inel owvol t
ageQRS.Echocar diogram i sli
mit
ed
butshowsmi l
dlydilatedRVwi t
hmi ldl
yr educedf unct i
on,traceTR, normalLVf unctionand
PASPest i
mat edat25mmHg.HerV/ Qscanshowedsmal lper fusiondef ectint her i
ghtlower
l
obe.HerPFTsshowedmi ldl
yr educedDLCOandmi ld-moder at erestricti
on.Ri ghthear t
catheteri
zationreveals:RA4mm Hg, RV30/ 4mm Hg, PA33/ 16mean22mmHg, PCWP12
mmHgandLVEDP11mmHg.HerCOi s5.4L/ min.

Whi
choft
hef
oll
owi
ngi
sthenextbestst
epi
nhercar
e?

a. Startsi
ldenaf
il
b. Vasodilat
ortest
ing
c. Referforbari
atr
icprogram
d. Pulmonar yangi
ogram
e. Repeatechocardiogram i
n6mont
hs

Answer
:

2. c. Ref
erf
orbar
iat
ri
cpr
ogr
am

Thispat i
enthassympt omsofwor seni
ngdyspneaonexerti
on,andamedi calbackgroundof
priorDVTwi thl
ikel
yPE, signi
fi
cantobesi
ty,
hyper
tensi
on,andprioruseof
fenfl
uramine/phenter
mi ne.Theinci
denceofPAHisapproxi
mately0.005%iffenf
lur
ami nei
s
usedf orgreat
erthan3mont hs.

Eval
uat
ionf
orpul
monar
yhyper
tensi
oni
swar
rant
edher
ebasedonhersympt
omsandhi
stor
y.

Cardi
accat heteri
zationi nthispat i
entdemonst
rat
edthatwhileshehasmodestsyst
emicand
pulmonarypr essureel evation,shedoesnotmeetcrit
eri
aforthediagnosi
sofpul
monary
hyper
tension:thisisdef inedatameanPAP>25mmHgatr estint
hesetti
ngofanor
mal
pulmonarycapi l
larywedgepr essure(3Woodunit
s.Hersympt omsmaybeduet oherobesi
ty
andtheref
or ewei ghtlossi ntervent
ionsarei
ndi
cated.
 

Acut
evasodil
atort
est
ingshouldbeperf
ormedinalli
diopat
hicpulmonar
yart
erialhyper
tensi
on
(I
PAH)pat
ient
swhomi ghtbeconsi
der
edpotent
ialcandi
datesforl
ong-t
ermtherapywithoral
cal
cium channelbl ockers.IPAHpat i
entsinwhom chr oniccalcium channelblockertherapy
wouldnotbeconsi dered,suchast hosewi thovertrightheartfail
ureorhemodynami c
i
nstabil
it
y,neednotunder goacut evasodi latort
esting.Thedef ini
ti
onofanacut eresponsethat
maywar rantiniti
ationoflong-termt herapywi thoralcalcium channelblockersisadecr easein
mPAPofatl east10mm Hgt oanabsol utemPAPofl essthan40mm Hgwi thoutadecr easein
car
diacout put.Acut evasodil
atortestingisnoti ndi
cat ed,andmaybehar mful,inpatientswit
h
si
gnifi
cantl
yel evatedleftheartfi
ll
ingpr essures.

Tr eatmentwithendot hel i
nr eceptorant agoni stsorphosphodi est
eraseinhibi
torsisoften
i
ndi catedinpatientswhohavenegat iver esponset oacut evasoreacti
vetesti
ngandar ei nlower
riskcategori
es.TheSUPER- 1(Sil
denaf i
lUsei nPul monar yAr t
eri
alHypertension)studywasa
randomi zed,doubl e-blind, placebo- cont rolledt r
ialthatassigned278pat ient
swi thPAH( either
IPAHorPAHassoci atedwi thconnect ivet i
ssuedi seaseorwi threpair
edcongeni talsystemi c-t
o
-pul monaryshunt s)topl aceboorsi ldenaf il(20,40, or80mg)or all
y3timesdai lyfor12weeks.
The6MW t estincreasedf rom basel inei nal lsildenaf i
lgroups,withmeanpl acebo- corr
ect ed
treatmenteffectsof45, 46,and50m f or20- ,
40- ,and80- mgdosesofsi l
denaf i
l,r
espectively.
Thi sappearedt obeent i
relyr el
atedt oi mpr ovement swi thacti
vetherapy,astherewasl ittle
changei n6- minut ewal ktesti nthepl acebogr oup.Al lsil
denafi
ldosesreducedt hemPAPand
i
mpr ovedfunctionalcl ass.Thei ncidenceofcl inicalwor seni
ngdi dnotdif
fersignifi
cantl
y
bet weenthepat ientstr eatedwi t
hsildenaf i
lver suspl acebo.

Patientswi t
hsuspectedPAHshoul dunder goeval uati
onf orchronicthromboembol i
c
pulmonar yhypertension(CTEPH) .Thescreeningt oolofchoicef orCTEPHi sventi
lat
ion
perfusionscanning.Ifindicati
veofCTEPH, apul monar yangiogram shoul dbeper f
ormed.
Patientsareconsideredt obecandi dat
esforpul monar ythromboendar terectomy(PTE)ift
hey
havesur gicall
yaccessiblediseaseandpr esentaccept ablesurgicalr
isk.ThegoalofPTEi sto
removesuf f
ici
entmat eri
alfrom t
hepul monaryar t
eriestosubst anti
all
yl owerPVRandi mprove
cardiacout put
.Thiscompl exandli
fe-savi
ngt herapyisbestper formedathi gh-
volume
centers. 

Repeatechocar
diogr
am i
n6mont
hsi
snotwar
rant
edi
ntheabsencechangei
ncl
ini
calst
atus.

Quest
ion:

3. A34- year-ol
dwomancomest oyourclinicforevaluationofdyspneaonexer t
ion.She
hasahi storyofGERD, andapriorDVTduringr ecoveryfrom kneesur gery.Sher epor t
st hat
overt hepastyearshehasnot i
cedincr
easingfat i
gue.Sheusedt ojoglastsummerbutst opped
i
nt hef al lduet oincr
easingproblemswithRaynaud' si nherhands.Sheswi t
chedt oani ndoor
tr
eadmi llbuthasbeenget ti
ngmor eshortofbreathwi ththisandhast ostopear lier.Her
medi cat ionsi ncl
udelansoprazoleandanor alcontraceptive.Herphysicalexam i snot ablefora
HR80, BP135/ 82,pulseoxygenof97%onr oom air.Herpul monar yexami nati
oni s
unremar kable.Cardi
acexami nati
onisnotableforapr omi nentS2att heapex, andal eft
parast ernalheave.HerJVPi s6cm H20.Shehasnol owerextr emit
yedema, not ender nessor
palpabl evenouscor ds.Herhandsar epuff
yandt heski nonherdi stalf
ingersisshi ny.HerECG
i
sshowni nFi gur
e1.
Whati
sthemostappr
opr
iat
enextt
estf
orhereval
uat
ion?

a. Echocardi
ogram
b. Exerci
sestr
esstest
c. V/Qscan
d. Ri
ghtheartcatheter
izati
on
e. Pulmonaryfuncti
ont est
s

Answer
:

3. a. Echocar
diogr
am

Thispati
enthasphysicalexami nat
ionandECGf i
ndingsconsist
entwithrightventri
cular
hypert
rophyandpulmonar yhyper t
ension(l
oudP2) .Inaddit
ion,shehasahi storyandphysi
cal
exam f
indingssuggesti
veofsyst emicscler
osis.Giventhesefi
ndings,
therei ssi
gnif
icant
concernforpul
monar yart
erialhypert
ension(group1pul monaryhypertension).

Giventhehi ghcl
inicalsuspicionf orpul monar yhypertensionandt hediagnosisofr i
ght
ventri
cularhypert
rophyi nthispat ient,anechocar diogram isthemostappr opriatenexttestto
confir
mt hediagnosisandeval uatet hedegr eeofsever it
y.AV/ Qscanmaybeconsi deredat
somepoi nt,butdespiteherpr iorDVTaf tersurgery,chronicthromboembol i
cdiseaser emai ns
lessli
kelythanpulmonar yar t
er i
alhyper tensionassoci atedwithsystemicsclerosisbasedon
herhistory.Pul
monar yfunctiont estsar eunlikel
yt obesi gnif
icant
lyabnormalinher ,and
theref
orear eunl
ikelytoaidindet er miningpr ognosisort reat
mentf orher
.Anexer cisestress
testissi milarl
yunlikelytobehel pfulinhereval uati
on, ashersympt omsar eunli
kelyt
obedue
toischemi aorarrhythmias.A6- mi nute-walktestwoul dbeabet t
ertoolatthisstagetohelp
stratif
yherr i
sk.Arightheartcat heterizati
onisoftenindicatedwhent hediagnosisof
pulmonar yar t
eri
alhypertensioni ssuspect edorknown, butitismor eappropriat
et of
ir
st
pur suemor enon-invasivetesting.Wi thabet t
erunder standingofherdi agnosisanddisease
sever i
ty,ar i
ghthear tcathet
erizationmaybemor eusef uliftheresponset oacutevasodil
ator
s
i
seval uated,whichcant hengui det herapy.

Thereare5groupsofpul monaryhypert
ensionascl assi
fiedbyt heWHO:gr oup1, pulmonary
art
eri
alhypert
ension;group2,pulmonaryhypertensionduet olefthear
tdisease;group3,
pulmonaryhypertensi
onassociat
edwithlungdi seasesand/ orhypoxemia;group4, pulmonary
hypert
ensionduet ochroni
cthromboti
cand/ orembol i
cdi sease(CTEPH);andgr oup5,
miscel
laneous(sarcoi
dosis,l
ymphangiomatosis,compr essionofpulmonaryvessel s)
.

Apr i
mar ypul monaryar teriopathyoccur smostcommonl yi npati
entswi t
ht heli
mi ted
cutaneousf orm ofsystemi csclerosis,former l
yref
erredtoast heCREST( calci
nosis, Raynaud’s,
esophagealdysf unction, sclerodactaly,telangect
asias)variant
.Alt
houghataut opsy, 65%t o
80%ofi ndividual
shavehi stopathologicalchangesconsi stentwit
hPAH, lessthan10%devel op
cl
inicall
yappar entdisease.Sur veil
lanceechocar diographysuggestst hatthereisasubst ant
ial
prevalenceofmi ldtomoder atePHi nconnect i
veti
ssuedi seasepatients.However , t
he
managementandnat uralhi st
oryofsuchpat i
entshasnotbeenwel lstudied.Histology
consistentwi t
hPAHhasal sobeenobser vedinsystemiclupuserythemat osus, mixed
connect i
vetissuedisease, andr heumat oidar t
hri
ti
s.

I
naddi t
iont othescleroderma/ connecti
vet i
ssuespectrum ofdiseases,gr
oup1PHal so
i
ncl udes:i
di opathi
c,famili
al(usuallyassociatedwit
hbonemor phogeni
cprotei
nreceptor-2
[
BMPR2]aut osomaldomi nantmut ati
ons),congeni
tal(systemic-to-pul
monaryshunts),por
tal
hyper t
ension, HIVi
nfection,drugsandt oxi
ns, andotheril
lnessessuchashemogl obinopathies,
glycogenst oragediseases, heredit
aryhemor rhagi
ctelangiect
asia,andchronic
myel oprol
iferati
vedisorder.

Quest
ion:

4. A28yearol dwomanpr esent


stoyourcl i
nicwitha6monthhistor
yofprogressi
ve
dyspneaandchr onicfat
igue.Pastmedicalhi
storyisremar
kabl
eforarthr
it
is.Hercurr
ent
medicati
onsincludeibuprofenasneeded.Sher epor
tsexer
ti
onaldyspneawhenwal ki
ng
dist
ancesgreaterthan100f eet

Onexam, bloodpr essureis112/ 65.Pulsei s76andr egular.Room air02 sat


ur at
ionis93%.BMI
2
i
s20kg/ m .Car diacauscultati
onr evealsasof tsystolicfl
owmur muratt herightsternalborder
.
P2 
componentoft hesecondhear tsoundi saccentuat ed.Lungexam i sclearbi l
ater
all
y.EKG
demonst r
atessinusr hythm wit
hi ncompl eteRBBB.Tr ansthoraci
cechocar diogram reveals
normalleftvent
ricularsyst
olicfunction,moder ateTRandami l
dlyenlargedrightventri
cleand
ri
ghtatr
ialenlar
gement .Ri
ghtvent ri
cularsystoli
cpressur esi sesti
mat edat59mm Hg.
Agit
atedsalinei
nj ecti
onwasnor malwi thnoevi denceofshunt .

Whi
choft
hef
oll
owi
ngi
sthenextbestst
epi
nhermanagement
?

a. Car
diacMRI
b. Myocardi
alPer
fusi
onI
magi
ng
c. Ri
ghtheartcardiaccat
heteri
zati
on
d. PEprot
ocolCTangi ogram ofthechest
e. Noct
urnalpolysomnography

Answer
:

4. c. PEpr
otocolCTangi
ogr
am oft
hechest

Thedi
agnosisofpulmonar yhypert
ensionencompassesWHOcl assesI-V,ofwhi
chtypeIis
pul
monaryarter
ialhypertensi
on(PAH).Becauseofheryoungage,lackofcomorbi
dit
iesand/or
si
gnsofl
eftheartdisease,wesuspectPAHi nthi
spati
ent.

RHCi sindicatedi npat i


entswithsuspectedpulmonar yar terialHTNaspar tofthewor kupof
elevatedrighthear tpressuresdetect
edonechocar di
ogr aphy.Al thoughcar di
acMRIcan
provideanat omi cinformation,i
twil
lnotconfir
mt hedi agnosi sofPAH.Myocar dialperfusi
on
imagingi snoti ndicatedasshei snothavingact iveangi naandherpr etestprobabili
tyofCADis
low.CTangi ographymaybehel pf
ultofurt
hereluci datetheet i
ologyofpul monar yHTNwhen
signif
icantthromboembol i
cdiseaseissuspected.However ,thechr onicit
yoft hispati
ent's
sympt omsmakeVQscanmor eappropriat
efort heeval uationofchr onicthromboembol i
c
pulmonar yhyper tension.Sleepstudymaypr ovidei nformat ionaboutet i
ologyofpul monar y
hypertension,RHCr epresentstheonlywayt oobt aindi r
ectmeasur ement sofpul monary
vascularresistanceandt heref
oreestabl
ishadi agnosi sofPAH.

Quest
ion:

5. A72yearol dmanwi thahist


oryofhypertensi
onandpr ostat
ecancerpr esentsfor
eval
uationofdyspnea.Threeyear
sago, thepati
entunderwentarobot i
cprostatect
omyt hat
wascompl i
catedbysepsisandapr ol
ongedI CUcourse.Atthatti
me, tr
ansthoracic
echocardi
ogram perf
ormed,demonstratednormalLVsystoli
cfunction.Fol
lowingdischarge
fr
om thathospit
ali
zati
on,hehashadpr ogressi
vedyspneaonexer ti
on.

Heunder wentmyocar
dialperf
usi
onstudywhichreveal
ednoevi denceofischemia,
butshowed
evidenceofRVhypert
rophyandRVenlargement.Repeatechocardiogr
am reveal
snormalLV
functi
on,moderat
eRVenl ar
gement,
moder at
etri
cuspidregur
gitat
ionrightvent
ri
cul
ar
hypertr
ophyandPASPof55mmHg.  

Uponexam t oday,hisbloodpr essuremeasured145/76,pulse78andr


egul
arandO2 sat
urat
ion
=93%onr oom air.Physicalexam demonstratesaquietpr
ecordi
um wi
thnomur mur
s.S2 
is
spl
itphysiol
ogically.P2 
i
sl oud.Lungsarecl
ear .Ther
eisnoedemaperi
pheral
ly.Hei
snoton
anycardiovascularmedi cati
ons. 

Whi
choft
hef
oll
owi
ngi
sthenextmostappr
opr
iat
est
epi
nhi
scar
e?

a. Exerci
seechocardiogr
aphy
b. Nocturnalpol
ysomnogr aphy
c. HIVserologi
es
d. Pulmonaryventil
ati
onper f
usi
onscan
e. Coronaryangiogr
aphy
Answer
:

5. d. Pul
monar
yvent
il
ati
onper
fusi
onscan.

Thecor rectansweri spulmonar yventilati


onperfusionscan( VQscan) .Thispat i
ent'
shi st
oryof
cancerandpr ol
ongedhospitali
zationsuggest sthathemayhavedevel opedpul monar y
embol icdi sease.VQscansar ethescr eeningstudyofchoi cet oexcludechr onic
thromboembol i
cpulmonarydiseaseasacauseofpul monar yhyper t
esion.Hissci nti
graphic
fi
ndi ngsar ehighlysuggest
iveofpul monar yHTN( RVH/RVenl ar
gement )
.Exercise
echocar diographywoul dhavelimitedut il
it
yinthiscir
cumst ance,ashisMPIhasexcl uded
signifi
canti schemiaburden.HIVt est
ingi sreasonable,butgi venhisclini
calhistory,chronic
thromboembol i
cdiseaseseemsmor el i
kely.Coronaryangiogr aphyisnoti ndicatedashi s
stresst estshowednoevi denceofi shchemi aandhi sexertionalsympt omsar emor elikel
y
relatedt opul monar yhyper
tension.Noct urnalpolysomnogr aphymaybeconsi deredonce
chr onict hromboembol icdi
seasei sexcl uded.

Quest
ion:

6. A39yearoldwomanseeksasecondopi nionregar
dingherdyspnea.Oneyearago,she
developedincr
easingdyspneaonexer t
ionwi
thoccasionalpalpit
ati
ons.Shewasrefer
redtoa
cardiol
ogistwhoperformedastressechocar
diogram.Sherelatesthati
treveal
edshehadhigh
pressuresinherheart.Shecur
rentl
ytakesdi
lt
iazem 120mgdai l
yandaspiri
n81mgdai l
y.

Physi
calexam r
eveal
sabl oodpressureof108/64andapul seof63bpm.O2 
sat
urat
ionon
room ai
rwas93%.Cardiacexam reveal
edapr ominentsecondheartsoundandaI/I
Vdiast
oli
c
murmuratthebaseoftheheart.Shehasmoder atepret
ibialedema.

Herest
imatedpul
monaryarteri
alsyst
oli
cpressur
eonherrecentechowas76mmHg.Ther
e
wasnoevidenceofi
nter
atr
ialshuntbyagit
atedsal
inecont
rast

Whi
choft
hef
oll
owi
ngi
sthenextbestst
epi
nthi
spat
ient
'smanagement
?

a. Myocar
dialPerfusionImagingSt udy.
b. Tr
ansesophagealechocar di
ogram.
c. Lef
tandrightheartcatheter
izat
ion
d. PEprot
ocolCTangi ogr
aphyoft hechest.
e. Ri
ghtheartcatheteri
zati
onwi t
hvasodilat
orchal
lenge.

Answer
:

6. e. Ri
ghthear
tcat
het
eri
zat
ionwi
thvasodi
lat
orchal
lenge.

Thenextbestst
epint hi
spati
ent
'seval
uat
ionwouldberighthear
tcathet
eri
zati
onwi t
hplanned
vasodi
lat
orchal
lenge.Hersympt
omsandpr iori
magingstudi
esallsuppor
tthediagnosi
sof
pulmonar yhyper tension.TheACCF/ AHAgui deli
nesf ort hetr
eatmentofpul monar y
hypertensionf ocusont heimpor tanceofobt aini
ngar ightheartcardiaccat heteri
zati
ont o
confir
mt hedi agnosi sofpul monar yHTN.Thi sshoul dincludemeasur ement sofoxygen
saturati
onsandr ighthear tchamberpr essuresincludingt hepulmonar yarteryandcal culati
ons
ofcardiacout put/indexandpul monar yvascularresistance.Atthet i
meofr ightheartcardiac
catheteri
zation,ther esponset oacut evasodi l
atorsubst ancesshoul dbeassessedaswel l.
Furtherevaluat i
onf orcoronar yarterydiseasewi thmyocar di
alperfusionimagi ngorl eftheart
cathisnoti ndicatedatt histimegi venhernor malst ressechocar diogram andherl owpr etest
probabili
ty.CTangi ographyoft hechestmaybehel pfulindeterminingt heet i
ologyofher
suspectedpul monar yHTN, butthedi agnosisofpul monar yhypertensionneedst obe
confir
medi nvasively.TEEmaybeabl etoidentif
yani nt r
acardiacshuntt hiswoul dnotconf irm
thediagnosi sofpul monar yhyper tension.

Quest
ion:

7. A65- year-oldmanpr esentst otheemergencydepar tmentwith2daysofchestpai n


anddyspnea.Hehasahi stor
yofhyper t
ension,
dysl i
pidemia,andcoronaryar terydisease
tr
eatedwi thcor
onar yart
erybypasssur ger
ysever alyearsago.Hei safebri
le,buthisoxygen
satur
ationis92%onr oom air
.Jugul arvenouspr essureis6cm H2O.Hi slungsar eclearto
auscultat
ionbil
aterall
y.Hiselect
rocar di
ogram i
sonl yr emarkabl
eforsinust achycardi a.
Cardiacbiomarkers,incl
udingcardiact r
oponin,arenor mal.HischestX-rayi sinter
pr etedas
normal.

Basedonthepat
ient
’spr
esent
ati
onandr
iskf
act
ors,
whi
choft
hef
oll
owi
ngi
sthemostl
ikel
y
di
agnosi
s?

a. Gastr
oesophagealrefluxdi
sease.
b. Pulmonaryembol i
sm (PE).
c. Congesti
veheartfai
lure.
d. Pneumoni a.
e. Acutecoronarysyndrome.

Answer
:

7. b. Pul
monar
yembol
ism (
PE)
.

PEi sthecor r
ectchoicebecauset hepat i
ent’
scl i
nicalpresentat
ionofchestpai n,dyspnea,
hypoxemi a,andsinust achycardi
ai nt hesetti
ngofanor malchestX- r
aysuppor t
st he
diagnosisofPE.Fur t
her more,patientswi t
hatherot hr
ombot iccardiovasculardiseaseand
associatedriskfact
orshaveani ncr easedincidenceofvenoust hromboembol ism.Pneumoni a
i
si ncorrectbecauseoft helackoff ever,physi
calexami nationsignsofconsol idation,andchest
X- r
ayfindingsdonotsuppor tadi agnosisofpneumoni aorhear tfail
ure.Gastroesophageal
refl
uxdiseaseshoul dnotr esul
tinhypoxemi a.Acut ecoronarysyndr omei sincorrectbecause
thepat i
entishypoxemi c,withacl earlungexami nat i
onandchestX- ray,andhasno
electr
ocar di
ographi
cf i
ndingsofi schemi a.

Quest
ion:

8. A54-year
-ol
dwomanpr esent
st otheemergencydepart
mentwit
h12hoursofpl
euri
ti
c
pai
n,cough,
anddyspnea.Shehasahi stor
yofhypert
ensi
ont r
eatedwi
that
hiazi
dedi
uret
ic.
Sheisaf
ebril
eandhasanoxygensaturati
onof99%onr oom ai
r.Herel
ect
rocardi
ogr
am
demonstr
atessi
nusrhyt
hm withnoabnormalit
ies.Car
diacbiomarker
s,i
ncludi
ngcardi
ac
tr
oponi
n,arenor
mal.HerchestX-r
ayisinter
pretedasnormal.

Whi
choft
hef
oll
owi
ngi
sthemostappr
opr
iat
enextst
epi
ntheeval
uat
ionoft
hispat
ient
?

a. D-dimerenzyme- l
inkedimmunosorbentassay(ELI
SA)t
est
.
b. Magne ti
cr esonancepulmonaryangiography.
c. Contrast-
enhancedchestcomput edt omography.
d. Transthoracicechocardiogr
aphy(TTE).
e. Venti
lati
on- perf
usionlungscan.

Answer
:

8. a. D-
dimerenzyme-
li
nkedi
mmunosor
bentassay(
ELI
SA)t
est
.

TheD- dimerELISAt estist hecor r


ectchoice, becauset hecl inicalprobabi l
ityofpul
monar y
embolism (PE)basedonherpr esentationisl owt omoder ate.Init
ialevaluationwithD- di
mer
ELISAtesti
ng, i
fnormal ,avoi dst heneedf orfur t
hercost lydiagnost icimagi ngbecauseoft he
test
’shighnegat i
vepr edictiveval ue(99%) .Inaddi t
ion,D- dimert estingwhenused
appropri
atelyavoidsexposi ngt hepatientt oionizi
ngr adiationandi ntr
avenouscont r ast
requi
redforchestcomput edt omogr aphyeval uati
onf orPE.Thechoi cesofachestcomput ed
tomography, l
ungscan, ormagnet i
cresonanceangi ographyar eincorrectbecauset hepatient
doesnothaveahi ghenoughcl inicalprobabi l
ityofPEt owar rantproceedi ngdirect
lyt oi
magi ng.
TheTTEopt i
onisincor r
ectbecauseTTEi snotsensi ti
vef orthedi agnosi sofPE.

Quest
ion:

9. A48-year-
oldmanpr esentstotheemer gencydepart
mentwith3hour sofpr
ofound
dyspneaandl i
ghtheadednessand2daysofr i
ghtlegpainandswell
ing.Hehasahi st
oryof
diabetesandcoronaryarter
ydiseasewithapr i
orlargeant
eri
ormyocardiali
nfar
cti
onandal ef
t
ventri
cularej
ecti
onfract
ionof30%.Hei safebri
lebuthasabl oodpr
essureof70/30mm Hg
andanoxygensat ur
ati
onof84%onr oom air

Hiselectr
ocar diogram demonst ratessinustachycardiaat120bpm andnoot herchanges
compar edwi t
hapr iortracing.Chemi strypanel,
compl et
ebloodcount , andcoagulati
ontests
arenormal .Car di
act roponinTi s2ng/ ml(normal<0.2ng/ml).Hi schestX- r
aydemonst rat
es
cardi
omegal ywi t
hcl earlungf i
elds.Bedsideechocar di
ogram revealsasever el
ydil
ated
hypokineti
cr i
ghtvent r
icle.Contrast-enhancedchestcomput edt omogr aphydemonst r
ates
l
argebi l
ateralmai npul monar yembol iwit
hsevereri
ghtventri
cul arenlargement.Heis
i
mmedi at
elyst art
edoni ntravenousunf racti
onatedhepari
nasabol usf oll
owedbyi nf
usion.

Whichoft
hefol
lowi
ngi
sthemostappr
opr
iat
enextst
epi
nthemanagementoft
hispat
ientwi
th
pul
monaryemboli
sm?

a. Cont
inuesystemi
canticoagulati
onwithunfr
acti
onat
edhepar
in.
b. Ref
erthepati
entf
oropensur gi
calpul
monaryembolect
omy.
c. Admini
sterr
ecombinanttissue-pl
asminogenacti
vat
or100mg.
d. Ref
erthepati
entf
orul
trasound-
faci
li
tated,
cat het
er-
dir
ect
ed,
low-
dosef
ibr
inol
ysi
s.
e. I
nsertaret
ri
evabl
einf
eri
orvenacava( I
VC)fil
ter.

Answer
:

9. c. Admi
nist
err
ecombi
nantt
issue-
plasmi
nogenact
ivat
or100mg.

Administr
ationofti
ssue-pl
asminogenactivatori
sthecor r
ectopti
on,becausethepati
enthas
suff
eredmassi vepulmonaryemboli
sm, andisatincreasedri
skofdeathfrom ri
ghtventr
icul
ar
(RV)fai
lur
ei fr
eperf
usionther
apyisnotinstit
uted.Hehasnoevi denceofacti
vebleedi
ng,
thr
ombocyt openia,
orothercont
rai
ndicati
ont ofi
bri
nolyti
ctherapy.
 

Referri
ngthepat i
enttocatheter-
di r
ectedfibrinolysi
sorsur geryisincorrectbecauset hepatient
doesnothaveanycont rai
ndicati
onst of i
brinolyti
cther apy.Cat het
er-basedorsur gical
pulmonar yembolectomygener allyisreservedf orpatientswi thcontrai
ndi cati
onstof i
bri
nolyti
c
therapyorcatheter-
basedt echniques.Insertionofar et r
ievableIVCf i
lt
erisi ncor
rectbecause
IVCf i
lt
erplacementwillnotreversetheRVpr essureover loadthatisresultinginRVf ail
ureand
hypotension.Simplyconti
nuinghepar inther apyi si
ncor rectbecausesyst emi canti
coagulation
alonewi l
lnotimproveRVf ail
urer api
dlyenought oavertfurtherhemodynami ccol
lapseand
possiblydeath.

Quest
ion:

10. TheFi f
thWor l
dSymposi um onPul monar
yHypertensi
on( WSPH)in2013out l
inedthe
fi
vemajorcategori
es(Groups)ofpathologi
cmani f
est
ati
onsofpulmonar yar
ter
ialhypert
ension
(PAH)i
nanef for
ttobettercat
egori
zet hemanycausesofelevatedpul
monar yart
erypressures.
Whilet
hedemogr aphics,manif
est
ations,andoutcomesmayvar y,t
hosecausescategorizedin
Group1refl
ectedthepresenceofanunder lyi
ngpulmonaryvascul
opathy.

Whichoft
hefoll
owi
ngassociat
ionswithpul
monar
yar
ter
ialhyper
tensi
onhasbeencl
assi
fi
ed
asGroup1intheWSPHclassif
icat
ion?

a. Sar
coidosi
s.
b. Sl
eepapnea.
c. Chroni
cthromboemboli
cdi
sease.
d. Mit
ralval
vestenosi
s.
e. Ei
senmengersyndrome.

Answer
:

10. e. Ei
senmengersyndr
ome.

Thecor r
ectanswerisEi senmengersyndr ome.Group1PAHi ncludest hosewithevidencefor
pulmonar yhypert
ensionandanunder l
yingvasculopat
hy.Group1i ncludesidi
opathicPAH,
heri
tablePAH( themostcommonmut ationbeingbonemor phogenet icprotei
nreceptortype2
[
BMPR2] ),
drug-ortoxin-inducedPAH( part
icul
arl
ybyappet i
tesuppr essantdrugs),andthose
associatedwithconnectivetissuedi
sorders(par
ti
cular
lyscleroderma) ,human
i
mmunodef i
ciencyvir
us(HI V)inf
ecti
on,port
alhypert
ensi
on,andschi
stosomiasi
s,andthose
relat
edtocongeni t
alheartdisease.Whenthereispul
monaryhypert
ensionandar esul
tantr
ight
-to-l
eftshuntwithcyanosis,thecomplexisref
erredt
oastheEisenmengercomplexor
syndrome. 

Thef i
veWSPHgr oupsare:Group1, pul
monar yveno-occlusivediseaseandpul monary
capil
laryhemanigomatosis;Group2, PHduet oleft
-heartdisease, suchasmi t
ralst
enosis;
Group3, PHduet olungdiseaseorhypoxi a,
suchassl eepapnea;Gr oup4, PHduet ochroni
c
thr
omboembol i
cdisease;Group5, PHduet ounclearormul ti
factorialmechanisms,suchas
sarcoidosi
s,chr
onicmyel opr
olif
erati
vediseases,orglycogenstor agediseases.

Quest
ion:

11. A55- year-oldsedentarywomanpr esent sincli


nicwi thworseni
ngshortnessofbreath
overthepastseveralmonths.Shehasahi stor yofhypertension,
diagnosedat30yearsofage,
andcutaneoussarcoidosi
s.Hercur r
entmedi cationsarelisi
nopril20mgdailyand
hydrochl
orot
hiazi
de25mgdai l
y.Herbloodpr essureis128/ 70mm Hgandhear tr
ateis75
bpm.Physicalexaminati
onr eveal
sanel evatedj ugularvenouspr essur
eandahol osystol
ic
murmur ,
bestheardattheleftlowersternalbor der. 

Anechocar diogram orderedbyherprimarycarephysicianrevealedami ldlydil


atedatriaand
ri
ghtventricl
e(RV),mi l
dconcentri
cleftventr
icul
arhypertr
ophy, andinterventri
cularseptalwall
fl
att
eninginsyst ol
ewi thabnormalmot i
on.Ther i
ghtandleftventri
cularsystoli
cf uncti
onwas
normal.Therewasmoder at
etri
cuspidregurgit
ati
onsecondar ytoadi l
atedt r
icuspidvalve
annulus.TheRVsyst oli
cpressur
e(RVSP)wasest i
mat edtobe40mm Hggr eaterthantheright
atr
ialpressure,butmaybeunder esti
mat ed.Therewasmi ldl
ydilatedinferi
orvenacava.

Whi
choft
hef
oll
owi
ngi
sthenextbestst
epi
nthi
spat
ient
'scar
e?

a. Nuc l
earperfusi
oni magi ng.
b. Ri
ght -hear
tcatheterizati
on.
c. Cardiacmagnet i
cr esonancei maging(MRI )
.
d. Transesophagealechocar di
ogram (TEE).
e. Comput edtomogr aphi cangiography(CTA).

Answer
:

11. b. Ri
ght
-hear
tcat
het
eri
zat
ion.

Thispati
enthaspulmonar yhyper t
ension(PH) .Thepresenceofaf latt
enedintervent
ri
cular
septum anddil
atedri
ghtatrium andvent ri
cleinthesettingofshortnessofbreatharemost
consist
entwit
hPH.Al thoughechocar diographycanof tenesti
mat etheRVSP, asur r
ogatefor
pulmonarysystol
icpressurewhennopul monar yvalvepat hol
ogyexists,t
hedi agnosi
sofPH
mustbemadevi ari
ght-heartcatheter
izati
onanddi rectmeasur ementofpulmonar yarteri
al(
PA)
pressur
es. 

TheRVSPi nthispatienti
sslightl
yelevated,
butcoul
dbeanunder est
imationofthetrue
pulmonarypressure.Theref
ore,thenextstepshoul
dbetoreferthepati
entforr
ight-
heart
cathet
eri
zat
iont odetermi
nePApr essure.Ameanpulmonarypressure>25mm Hgonr ight
-
heartcat
heteri
zati
ondocument sthepresenceofPH. 
CTAwoul dbeusef ulwhent hereisasuspi cionofpul monar yembol ism ( PE).Inthiscase,the
chronicityoft hepat i
ent'ssympt omsgoagai nstanacut ePE.Chr oni ct hromboembol i
c
pulmonar yhyper tension(CTEPH)canpr esentmor echronically,butt hegol dstandardfor
diagnosi ngCTEPHi sviaaVQscan, onlyaf terPHi sdocument edvi ar i
ght -heartcathet
eri
zation.
CardiacMRIi shelpfulfortheexactquant ifi
cat i
onoftheejectionf ract i
on, toevaluate
myocar di alfi
brosis,toassessPAsi ze,andt oquant i
fythetricuspidr egurgitati
on,butitwould
notbeabl et odiagnosePH.TEEwoul dnotof feraddi
tionalinformat i
ont odi agnosePH.
Improvedvi suali
zat i
onoft hetricuspidval vei sunnecessary,becauset heet iol
ogyi sdi
lat
ionof
theannul usrelatedt ori
ght-sideddilatati
onf rom PH.Finall
y, nuclearper fusionimagingwoul d
behel pfultoassessi schemi aiftheshor tnessofbr eathwast houghtt obeanangi nal
equivalent ,butthef i
ndingsonechocar diogr am makePHmuchmor el ikely.

Quest
ion:

12. A43- year-oldwomanwi thafamilyhistor


yofpul monaryhyper tension(PH)isseenin
cl
inicforacompl aintofdyspneaonexer t
ion.Shet akesadailymul tivi
tamin.Curr
entvitalsi
gns
areapul seof90bpm, bl
oodpr essur
e110/ 76mm Hg, respi
ratoryrate14, andoxygen
saturat
ion99%onr oom air.Herexaminati
oni sremarkableforpromi nentAwavesi nthe
j
ugularveins,apar ast
ernalli
ft
,asystoli
cmur mural ongtheleftsternalborder,
andar ight-
si
dedS4  sound.Thereisnohepat omegalyandonl yt r
aceankleedema.  

Herechocardiogram r
evealsnormalleftvent
ricularsizeandf uncti
on.Ther i
ghtventri
cular(RV)
basaldi
amet eris4.
1cm, andtheRVf r
eewal lthicknessis8mm.Tr icuspidannularplane
syst
oli
cexcur si
onis2.0cm.Ther eisseptalflatt
eningi nbot
hsyst oleanddi ast
ole.Thereis
moderatetr
icuspidregurgit
ati
on(TR),withapeakTRj etvel
ocityof3.9m/ sec.Theinferi
or
venacavaisnor malinsi
ze, butdoesnotchangesi gnif
icant
lywi t
hrespirat
ion.Thereisno
evi
denceofashunt . 

Right-
heartcathet
eri
zationr evealstheri
ghtatr
ial(RA)pressur
eis9mm Hg, RVpr essure68/9
mm Hg, pulmonaryartery(PA)pr essure67/29mm Hgwi thameanof42mm Hg, and
pulmonarycapill
arywedgepr essureis12mm Hgmeasur edatend-expir
ati
on.Fickcardiac
2
outputwas4.5L/ min(cardiaci ndex2.8L/min/
m) .Fol
lowingachallengewithi
nhal ednit
ri
c
oxide,t
hePApr essureis47/ 23mm Hgwi thameanof31mm Hg, andtheFickcardiacoutput
i
s5. 0L/min(cardi
acindex3. 1L/min/m2).

Whichofthef
oll
owi
ngt
her
api
esi
sthemostappr
opr
iat
efori
nit
ialt
reat
mentofherpul
monar
y
hyper
tensi
on?

a. Sil
denaf il
.
b. Riociguat.
c. Epopr ostenol
.
d. Nifedipine.
e. Bos entan.

Answer
:

12. d. Ni
fedi
pine.

Thecor
rectansweri
sacal
cium channelbl
ocker
.Thi
spat
ienthasPH(
li
kel
yWor
ldHeal
th
Organization[WHO]Gr oup1givenherf amilyhi
stor
y)withoutsignsofr i
ghtheartfail
ure.Her
ri
ght-heartcat het
eri
zati
ondemonstratedadr amaticacuteresponset oavasodilatorchall
enge,
usuall
ydef inedasaf al
linmeanpulmonar yart
erysyst
olicpressureofatleast10mm Hgt oa
val
uel esst han40mm Hg, wi
thoutadr opincardi
acoutput,inresponsetoacut e
administrationofapulmonaryvasodilatorsuchasinhalednitr
icoxide,i
ntravenous(IV)
epoprostenol ,ori
ntr
avenousadenosine. 

Anacut er esponsei sobservedonl yi


nami nori
tyofGr oup1PHpat ients.Anacut er esponse
predict schr onicresponsetot herapywi thacalcium channelbl ocker ,whichhasf ewersi de
effectst hanot hermor especifictherapiesforPH.Ni fedipi
neandaml odipinearemost
commonl yused.Ver apamilshoul dbeavoi dedbecauseofi tsnegat iveinot r
opicef f
ects.
Prost anoi dssuchasI Vepopr ostenol,
whi chrequiresacont inuousI Vpump;endot heli
n- 1
ant agoni stssuchasbosent an, whichcancausehepat i
cdysf unction;phosphodi esterase
i
nhi bitorssuchassi ldenaf
il,whichcancausef lushingandepi staxis;orguanyl atecycl ase
stimul atorssuchasr ioci
guat, whichcancausesi gnifi
canthypot ensi on, wouldallbe
reasonabl echoi cesinapat i
entwhodi dnothaveanacut eresponset ovasodilators,orf or
pat i
ent swhodonothavei mpr ovementi nsympt oms( toNewYor kHear tAssociat i
oncl assIor
IIstatus)wi t
hcal ci
um channelbl ockers.

Quest
ion:

13. A35- year-oldAf


rican-Ameri
canwomanpr esentswit
h6mont hsofwor seni
ng
exertionaldyspnea.Shedeni eschestpai
n,hypertension,di
abetes,hyperli
pidemia,
jointpai
ns,
historyofpulmonar yembol i
,orsmokingcigar
ettes.Onexami nation,herbloodpressureis
135/ 72mm Hg, pulse86bpm, andt
emperature98. 6ºF.Shehasj ugularvenousdistenti
on,
clearlungf i
elds,aprominentsecondcomponentofS2, andnoedema.  

AchestX- rayrevealspromi nentpulmonaryar t


eri
es,noinf
ilt
rates,andami ldl
yenl ar
gedright
atri
um (RA)andr i
ghtventricle(RV).Herel
ectrocardi
ogram revealsborderlinerightaxis
deviati
onandani ncompl eterightbundlebranchblockinnor malsinusr hythm.Her
echocardiogr
am showednor mallef
tventr
icularcontr
acti
li
tywithanej ectionfractionof65%,
RAandRVenl argementwi that ri
cuspidannularplanesystoli
cexcur si
onof14mm, andan
esti
mat edpulmonar yart
ery( PA)systol
icpressureof70mm Hgmeasur edbyt ricuspi
d
regurgi
tati
onjetveloci
ty. 

Youorderaright
-heartcat
het
eri
zat
ionandthemeanRApressur
ei s6mm Hg, RVpressureis
73/
9mm Hg, PApressurei
s73/18mm Hg( meanof38mm Hg),andpulmonarycapil
lary
wedgepressurei
s13mm Hg.Sher espondsfavor
abl
ytoanit
ri
coxideinhal
ati
onchallenge.

Inaddi
ti
ontoini
ti
alt
her
apywi
thacal
cium channelbl
ocker
,whi
choft
hef
oll
owi
ngshoul
dbe
consi
dered?

a. I
sosorbidedinatrat
e/hydr
alazi
ne.
b. Warfar
in.
c. I
vabradine.
d. Metoprololsucci
nate.
e. Sacubi
tril
/valsar
tan.

Answer
:
13. b. War
far
in.

Thecor rectansweri swar fari


n.Thepat i
entmostl ikelyhasGr oup1i diopathicpulmonar y
arterialhyper t
ension(IPAH) ,althoughot hercausesmustbeexcl uded.Ant i
coagulantshave
beenst udiedinfouruncont r
olledobser vationalser i
es, twopr ospecti
veandt woretrospective,
primar il
yinI PAHpat i
ent s.Ani mpr ovementi nsur vivalwasobser vedinal lf
ourwi t
ht heuseof
war fari
n.Mostgui del
inesr ecommendwar farinanticoagul ati
ontitr
atedt oani nt
ernational
nor mal i
zedr ati
oof1.5- 2.5inpat i
entswi t
hI PAH.Ther earenodat atosuppor ttheother
optionsi nt hi
ssituati
on.Met oprololsuccinat e,i
vabr adine,sacubitri
l/
valsartan,andisosor bide
dintrate/hydralazi
nehavebeenshownt obehel pfulwi thleftheartfai
lureandr educedej ecti
on
fr
act ion,buthavenotbeenst udi edinGr oup1pul monar yarteri
alhyper t
ension.

Quest
ion:

14. A51- year


-ol
dwomanwi t
hahi stor
yofsystemi csclerosispresent
stoyouroff
icef
or
worseningdyspneaonexer ti
on.Shereportsthatwithevent hemostmi ni
malofphysi
cal
acti
vit
y,suchasgettingdressed,shebecomesmar kedlyshor tofbr
eath.Onexaminat
ion,she
hasanel evat
edjugularvenouspressure.Shehasapr ominentP2 andar i
ght-
sidedpr
ecordial
heave.Anechocardiogram suggestsapulmonar yarterysystoli
cpressureof70mmHg.  

Youor deraright -heartcat


heterizationwithpulmonar yvasor eact
ivit
ytest i
ng.Herrightatri
al
(RA)pressurei s14mm Hg, andt her i
ghtventr
icul
arsyst ol
icandend- diastoli
cpressuresare
68and14mm Hg, respect
ivel
y.Herpul monaryarter
ialpressur ei
s68/ 28mm Hgandt he
pulmonar ycapi l
larywedgepr essur eis12mm Hg.Thecar diacout putis4.8L/ minandt he
cardiacindex( CI)is2L/ min(aver ageofthreethermodiluti
onmet hoddet erminati
ons).Her
hemodynami csshownosi gnif
icantchangeaf teradministr
ationofni t
ricoxi de(NO)ateither10
partspermi l
lion( PPM)or20PPM.

Whi
choft
hef
oll
owi
ngi
sthebesti
nit
ialdr
ugt
her
apyf
ort
hispat
ient
?

a. Oraltadal
afi
landoralambri
sent
an.
b. I
ntravenousepoprost
enol.
c. Oraldil
ti
azem.
d. I
ntravenousepoprost
enolandoralambr
isent
an.
e. Oraltadal
afi
l.

Answer
:

14. d. Intravenousepopr ostenolandor alambr isentan.


Thispat ienthashi gh-riskprecapil
larypulmonar yhypertensi
on.Themostappr opri
ate
tr
eatmentopt i
ongi venherhi gh-
risk(highRApr essure,lowCI )andWor ldHealt
hOr gani
zati
on
(WHO)f unctionalclass4st atus,i
sacombi nati
onofi ntr
avenouspr ostacycli
nandanor al
endothel inantagoni st
.Thecombi nat i
onofor albosentanandi nt
ravenousepopr ost
enolhas
beenst udi edandappear stohavehemodynami cadvant agesasupf r
ontt her
apyforsevere,
high-
riskpul monar yarteri
alhypertension(PAH) .Basedont heEur opeanSoci et
yof
Cardi
ol ogy/ EuropeanRespi rator
ySoci ety(
ESC/ ERS)2015gui del
inef orpulmonary
hypertensi on,thi
si saCl assIIar
ecommendat ion. 

Theupf
rontchoi
ceofdualtherapyfort
hiscohor
tist
heini
ti
alrecommendationintheESC/ERS
tr
eat
mentalgori
thm f
ort
hishigh-ri
skcohort
.Ahigh-
dosecalci
um channelblocker
,suchas
dil
ti
azem,wouldbeindicat
edonlyinpati
entswhor espondtoNOvasor eacti
vit
ytest
ing.Even
amongpat i
entswhodor espond,hi
gh-dosecalci
um channelblocker
sarerecommendedonl y
fori
diopat
hic,
heri
tabl
e,ordrug-i
nducedPAH, not
but  connect
iveti
ssuedisorder-
rel
atedPAH. 

Upfr
onttreat
mentwi thoralagentsislessidealgiventhehigh-r
isknatureofthi
spatient'
s
hemodynami cf
indingsandWHOf uncti
onalclass.Ini
ti
almonotherapywithintr
avenous
epoprost
enolwouldbeaconsi derat
ion,butthefir
st-l
inerecommendat i
onforthispati
entis
combinati
ontherapyofintravenousepopr ost
enolandor alambrisent
an.

Quest
ion:

15. A26- year-


oldwomanwi t
har ecentdiagnosi sofpulmonaryhyper t
ension(PH)isunder
yourcare.Echocardiographyshowsnor mall eftventri
cular(
LV)systol
icfuncti
on, normall
eft
atr
ial(LA)vol
umes, severePH, andpreservedr ightventr
icul
ar(RV)systoli
cfunction.A
venti
lati
on-perf
usion(VQ)scanr evealsnoevi denceoft hromboembolicdisease.Shehasno
evi
denceofcongeni talheartdi
sease,li
verdisease, humani mmunodef i
ciencyvirus(HIV)
,
hemat ol
ogicorrheumat ologi
cdisease,orsleep- di
sorderedbreat
hing.
 

Youperfor
m ar i
ght-hear
tcatheter
izati
onthatreveal
sthesepressures:ri
ghtat r
ial5mm Hg,RV
90/5mm Hg, pulmonaryarter
y(PA)90/ 33/52mm Hg, andpulmonar ycapil
larywedge8mm
Hg.Thecardiacindexis2.8L/min/m2 (car
diacoutput5L/min).Thereisnoi ntr
acardi
acshunt
.
Aft
er10mi nutesofinhal
ednitr
icoxideat40par t
spermi l
li
on,themeanPApr essurei
s
88/31/
50mm Hgandt hecardi
acout putisunchanged.

Whi
choft
hefol
lowi
ngist
heappr
opr
iat
ecl
assi
fi
cat
ionaccor
dingt
otheFi
ft
hWor
ldSymposi
um
onPul
monar
yHypert
ensi
on?

a. Gr
oup1.
b. Gr
oup2.
c. Gr
oup3.
d. Gr
oup5.
e. Gr
oup4.

Answer
:

15. a. Gr
oup1.

Thewomani nt hi
svignet
tehasGroup1pul monaryar
ter
ialhypert
ension(PAH).A
compr ehensivewor kupdoesnotrevealot
hercausesofPH.Herhemodynami csrevealsever
e
PHt hatisunr esponsivet
oavasodilat
orchall
enge(i
naposi t
ivestudy,t
hemeanpul monar y
art
erypr essuremustber educedby10mm Hgt oavalueof40mm Hgorl ess).
 

Group1PAHi seit
heridiopathic,her
it
able,drug-andtoxin-r
elat
ed, orassociatedwi t
h
connecti
veti
ssuedisease,portalhyper
tension,HIV,orcongenit
aldisease.Gr oup2i s
secondaryt
oleft
-heartdisease.Group3i ssecondarytolungdiseaseorhypoxi a.Group4is
duetothromboembolicandot herpulmonar yarter
yobstruct
ivedisease.Gr oup5i sunclear
and/ormult
if
actori
aleti
ology,suchashemat ologi
corsystemicinfl
ammat orydisorderssuch
assarcoi
d.

Quest
ion:
16. A35- year-oldwomanwi thidi
opathi
cpulmonaryarter
ialhyper
tensi
on( PAH)(Gr
oup1)
i
sunderyourcare.Atdi
agnosis,shehadapositi
vevasodil
atorstudywithadropinhermean
pul
monaryarter
y(PA)pressurefrom 50mm Hgto38mm Hg, withapreservedcardi
acindexof
2.
6L/min/m2.Shewaspr escr
ibedlong-act
ingni
fedi
pine.
 

Sixmont hslater,atar epeatr


ight-
hear tcat
het er
izat
ion,herPApr essuresar
eincreasedtoa
2
meanof49mm Hg, withacardi
acindexof2. 2L/min/ m .Otherwi
se,shehasWor l
dHeal t
h
Organi
zat i
oncl ass2sympt oms,a6- mi nut
ewal kdistanceof450m, andapeakVO2  of15
ml/kg/minonacar diopulmonaryexercisetest.Shehaspr eser
vedri
ghtventr
icul
arfuncti
onon
echocardiogram.Youdeci detoini
tiat
ecombi nati
ont herapy.

Whi
chofthef
oll
owi
ngi
sthemostappr
opr
iat
ether
apy,
inaddi
ti
ont
ochangi
ngni
fedi
pinet
o
ambr
isent
an?

a. Epoprostenol.
b. Aml odi
pine.
c. Tadalafi
l.
d. Verapami l
.
e. Treprost
inil
.

Answer
:

16. c. Tadal
afi
l.

Thispat i
enti samongt hemi nor i
tyofpat i
entswi t
hPAHwhor espondt oapul monary
vasodi latorstudy.Overt ime, however ,herPAHhaswor senedandshei sinneedoff urt
her
therapy.Agent sapprovedbyt heFoodandDr ugAdmi ni
strati
onf orthispati
entincl
ude
prost acycli
ns, endothelinrecept orantagonists,andagentstar get i
ngt henit
ri
coxidepat hway
(phosphodi esterasetype-5i nihibit
orsandsol ubleguanylycycl asei nhibi
tor
s).Verapimili
s
gener allyavoi dedinpat i
entswi thPAHowi ngt oitsnegati
veinot r
opicef f
ect
s.Further,t
his
pati
enti snotr espondingt ocal cium channelblockers.Therefor e,verapamili
snotan
appr opr i
atechoi ce.Recentevi dencesuppor tstheuseofambr isentanandt adalafi
linde
novo  ClassI Ipulmonar yhyper tension.

Quest
ion:

17. A35- year-


oldwomanwi th6mont hsofdyspneaevenwhendr essingwhoexper i
enced
twoepi sodesofsyncopeunder wentanechocar diogram thatrevealedseverepul monar y
hypertension(PH).Sheult
imatelyunder wentr i
ght -
heartcatheteri
zationrevealingapulmonar y
capil
larywedgepr essureof10mm Hg, rightatrial(RA)pressure18mm Hg, pulmonaryar t
ery
meanpr essureof70mm Hg, cardiacout putof5L/ min,andpulmonar yvascularresi
stanceof
12Woodsuni t
s.Therei
snor esponset oinhaledni tri
coxide.Furtherevaluati
ont hatexcluded
othercausesofPHl edtoadiagnosi sofidiopathicpulmonar yarteri
alhypertension.Shecomes
toyouf oranopi ni
onregardi
ngt herapeuticopti
ons.

Whi
choft
hef
oll
owi
ngt
her
api
eswoul
dyouadvi
se?

a. Epoprost
enolandambri
sent
an.
b. Sil
denafi
landri
oci
guat
.
c. Si
ldenafi
l.
d. Ambrisent
anandsi
ldenafi
l.
e. Ambrisent
anandri
ociguat
.

Answer
:

17. a. Epopr
ost
enolandambr
isent
an.

Thecor r
ectansweri sepoprost
enolandambr i
sentan.Thi
spatienthasseveralhi
gh-risk
featur
es,i
ncludingNewYor kHeartAssoci
ati
onclass4dyspnea, syncope,andaRApr essure
>15mm Hg.Ot herhigh-
riskf
eaturesarea6-minutewalkdist
ance<300m, peakVO2 <12
ml /
kg/min,cardiacindex≤2L/min/ m2,
andri
ghtventri
cularej
ecti
onf r
acti
on<35%oncar diac
magnet i
cresonance.  

Thet r
eatmentofpul monar yar
teri
alhypertensi
onshouldbetail
oredtot hesever
ityofi
ll
ness.I
n
suchhi gh-ri
skpat ient
s,epoprostenolshouldbeincl
udedininit
ialt
reatmentplans.Alt
hough
highestriskpatientarenotwel lr
epresentedinrandomizedcl
ini
caltri
als,exper
topini
onfavors
upfrontcombi nationtherapyoftwot othreedrugs,i
ncl
udingepoprostenol.

Quest
ion:

18. A42- year-oldmanpr esent


stoyouroffi
cet odiscusstr
eatmentforhisrecently
diagnosedidiopathi
cpul monar yart
eri
alhypert
ension( I
PAH).Hewasdi agnosedaf t
er
developingprogressivedyspneaonexer t
ion,andechocar di
ographydemonst r
atednor mallef
t
ventri
cular(
LV)funct ion,el
evatedpulmonaryarterypressur
e,amoder at
elydil
atedright
ventri
cle(RV)withRVhyper t
rophyandmoder atelyreducedfuncti
on,andsigni
ficanttr
icuspid
regurgi
tati
onwi t
hadi lat
edrightat
ri
um (RA).
 

Hispriorworkuphasdemonst ratednoevi denceofchroni cthromboembol i


cdisease,anormal
sl
eepst udy,andnegativetesti
ngf orhumani mmuodef iciencyvirusandconnect i
vetissue
diseases.Hecurr
entlygetsshor tofbreathwhenhepl ayswi thhiskidsandisnol ongerableto
mowhi slawn.Hi
sphysi calexami nati
onisnot abl
eforanaccent uat
edP2,amidsyst oli
c
ejecti
onmur mur,andleftparasternalheave.Hehasdi stendedj ugul
arveinsandt r
acel ower
extremityedema,buthisextremi t
iesarewar m andtherei snocyanosisorclubbing. 

Heper for
msa6- minut
ewal kt
esti
nyouroffi
ceandisabletowalk350met er
s.Aright-heart
catheteri
zat
iondemonstr
atesLVenddiast
oli
cpressureof13mm Hg, meanpulmonar yarter
y
pressureof38mm Hg, wit
hacardiaci
ndexof2.5L/min/m2,andapulmonaryvascular
resi
stance(PVR)of5Wooduni t
s.Hehasanegativeresponsetovasodi
lat
orchal
lenge.

Whi
choft
hef
oll
owi
ngi
nit
ialt
her
apeut
icappr
oachesi
sappr
opr
iat
e?

a. Ri
ociguatandsil
denafi
l.
b. Si
ldenafi
l.
c. Amlodipine.
d. Ambr i
sentanandtadal
afi
l.
e. At
rialseptost
omy.
Answer
:

18. d. Ambr
isent
anandt
adal
afi
l.

Thecor r
ectanswerisambr i
sentanandtadalafil
.Si
ldenaf
ilhasbeenst udi
edinarandomized,
placebo-cont
rol
ledtrialandwasf oundtoimpr ove6-minutewalkdistanceandhemodynami cs,
butnotthesecondar yendpointofti
met oclini
calworsening.Gi
vent hi
spatient
’ssever
it
yof
disease,t
reat
mentwi thsildenaf
ilal
oneisnotadequat eandcombi nationt
herapywithan
endotheli
nreceptorantagonistoraprostanoidshouldbeconsidered.

Theseverit
yofi ll
nessi nIPAHi sani mportantf
actorindetermini
ngtherapy.Thi
spatient
demonstratesfindingsofatl eastmoder at
elyseverePAH( NewYorkHeartAssociat
ion[NYHA]
cl
assII-
IIIheartfail
ure, 6-
mi nutewalkdistanceof350m, RVhypertr
ophywi t
hreducedf uncti
on,
andPVRof5Wooduni ts)
.Therear edatashowingthatt heini
ti
alcombinati
onofambr i
sentan
andtadalafi
lli
ssuper iortoambr isentanortadal
afil
laloneinNYHAf uncti
onalcl
assIIandI II
PAHpat i
ents.Ther ef
or e,t
hisini
tialcombinati
ontherapywoul dbeagoodchoi ceforthispati
ent.

Calci
um channelbl ockerscanbever yeff
ecti
vether
api
esforthosefewpati
entswithavery
robustresponsetoacut evasodilat
ortest
ing.However
,becausethi
spati
enthadanegat i
ve
responsetoavasodi latorchal
lenge,heshouldnotbetr
eatedwithacal
cium channelbl
ocker
suchasaml odi
pine.

Thesolubleguanylat
ecyclasesti
mulat
orri
ociguatwasappr ovedbytheFoodandDrug
Admini
strationi
nOctober2013t oimpr
oveexercisecapaci
tyandfuncti
onalcl
assanddelay
cl
ini
calwor seni
ngingroup1PAH, buti
scontraindi
catedwit
hphosphodiester
ase5inhi
bitor
s
suchassildenafi
lduetohypotensi
on.

Atrialseptostomycr eatesar i
ght-to-lefti
nt eratr
ialshunt
, andsever alcaseserieshaver epor
ted
hemodynami candcl i
nicalimprovement sfol l
owingthispr ocedure.Whilethecr eatedshunt
decr easessystemicar teri
aloxygensat uration,t
hegoali sani mpr ovementinsyst emicoxygen
deliverybasedont heimpr ovedcar diacout put.However ,proceduralmor t
alit
yishi ghandis
drivenbyt hesever i
tyofPAHandr ighthear tfail
ureinpat i
entsunder goingthispr ocedure.
Cur rentl
y,atr
ialseptostomyi srecommendedf orpatient
swi t
hsever ePAHandi ntractableri
ght
hear tfai
luredespit
emaxi malmedi calther apy.Thegoal soft hisprocedurearepal li
ati
onand
restorati
onandmai nt
enanceofcl inicalstabi l
it
yunt i
latransplantcanbeper f
ormed.

Quest
ion:

19. A55- year-oldwomanwi t


hahi st
oryofexer
tionaldyspneaandreducedexercise
capaci
typresentst oyourcl
ini
cforher3mont hfol
low- upeval
uati
on.Hermedicalhistor
yis
si
gnifi
cantfornicoti
nedependenceandmor bidobesity. 

Previ
ously,
anextensi
vediagnosti
cwork-upwi
thatr
anst
horaci
cechocardi
ogram (TTE),
venti
lat
ion-
per
fusionscan,chestcomput
edtomogr
aphy(CT)
,andri
ght-hear
tcatheter
izat
ion
reveal
edsever
epulmonar
yhyper
tensi
on(PH)secondaryt
ochr
onict
hromboembol
ic
pulmonar
yhypert
ensi
on(CTEPH)
.Warfar
inwasinit
iat
edandshequi
tsmoking.
 

Sheunderwentpulmonar yendar t
erectomy.Anechocardiogram perf
ormedpri
ortohospi
tal
dischar
gereveal
edimpr ovedrighthear themodynamicswithasi gni
fi
cantr
educt
ioni
nher
pulmonaryart
erypressures.Shepr esentstodayforf
oll
ow- up.Shehasnonewsympt omsand
report
sfeel
ingsi
gnifi
cantlybetter
.Vitalsarestabl
e.

Whi
choft
hef
oll
owi
ngt
est
sshoul
dbeor
der
edatt
oday'
svi
sit
?

a. D-di
mert est.
b. Lowerextremityvenousdupl
exscan.
c. 6-minut
ewal ktest.
d. ChestCT.
e. Exer
cisestresstest
.

Answer
:

19. c. 6-
minut
ewal
ktest
.

Thecorrectanswerisasi x-minutewal ktest.Surgeryisthedef i


nit
ivetherapyforCTEPH.
Pulmonaryendar t
erectomyist hesurgicalprocedureofchoi ce.Thebenef i
cialeff
ectsof
pulmonaryendar t
ectomymaybedet ectedshor t
lyaft
ersur geryandincludei mprovedexer ci
se
capacit
y,reducedpulmonar yar t
erypressure,reducedpul monaryvascul arresi
stance,and
reducedmor tal
it
y.Echocardiographymayr evealimprovedt r
icuspidregurgit
ationandr educed
ri
ghtatri
alandvent r
icularsi
ze. 

Insomepat i
ents,surgerymayhaveasubopt imaleffectonfuncti
onalcapacityand
hemodynami cs.Inthesepat i
ents,assessmentshoul dbeper for
medt odeterminewhet hera
repeatpul
monar yendar tectomyormedi calmanagementi sappropr
iate.Thegoalsofmedi cal
therapywi
thpul monar yvasodi l
atorsincludereducti
onofpul monaryvascularresi
stanceand
pulmonaryarter
ialpressur es,andimpr ovedexercisecapacityandoxygenation.Medications
i
ncludeprostanoids,solubleguanyl atecyclasesti
mul ator
s,phosphodiester
ase-5inhibit
ors,
andendotheli
nr eceptorant agonists.
 

Foll
owi ngtherapeuti
cinterventi
onf orPH, assessmentshoul dbeper for
medt odet erminethe
pati
ent '
sresponset otherapy.Ateachf ol
low- upvi sit
,a6- minut ewal ktestshoul dbeobt ai
ned.
ATTEshoul dbeobt ai
nedatl eastannual ly,andsooneri findicat ed.Thedeci sionf orar i
ght-
heartcatheter
izati
onisusual lyperformedonai ndi vi
duali
zedbasi s,usuall
year lyafterther
apy
andr epeatedifthepatientdeteri
orates.Ot herimpor t
antfoll
ow- upassessment sincludebrain
natr
iureti
cpept i
delevelandt readmi l
ltestforexer cisecapacit y.Thi spat i
entisclinical
lystabl
e.
Thereisnoi ndicati
onforD- dimer,chestCT, exercisestresst est ,orvenousdupl exscanatt his
ti
me.

Quest
ion:

20. A43- year-oldwomancomest otheemer gencyr oom complai


ningofa2- mont hhi
stor
y
ofdyspneaonexer t
ionandbi
lat
erall
owerextremityedema.Shehasahi storyofpulmonary
emboli,
fir
stdiagnosed5year
sagoandt reat
edwi th6mont hsofwar f
arintherapy.Her
echocardi
ogram atthatt
imedemonstrat
ednosi gnif
icantval
vularorotherstr
ucturalhear
t
diseaseandanormalleftventri
cul
arfuncti
on.Therightventr
icl
ewasthoughttobemildl
y
dil
atedwithmil
dlydepressedfuncti
on.Thereaf
ter
,sheagai nwasdiagnosedwit
hapulmonar
y
embol i
sm wi
thi
n2mont hsofdisconti
nuingwafar
in.Sheneverfoll
owedupwi t
hthe
recommendedhemat ol
ogi standselfdi
sconti
nuedwar f
arinuponleavi
ngthehospi
tal

Shecur r
entl
yt akesbirthcontrolpil
lsandnoot hermedicat
ions.Shehasnotseenamedi cal
professionalinmor ethan4year s.Onexami nat
ion,
herbloodpr essurei
s94/68mm Hg,heart
rate106bpm andr egular,l
ungsar eclear
,andneckveinsarei ncr
eased.Hercar
diac
exami nati
ondemonst ratesrightventri
cul
arheave,anondisplacedpointofmaxi
malimpulse,
andar egul
art achycardicrhythm withaloudsecondcomponentoft hesecondhear
tsound.
Herext remit
iesdemonst rat
e2+edemabi lat
erall
y.

Whi
choft
hef
oll
owi
ngi
smostl
ikel
ytobeseenonherel
ect
rocar
diogr
am?

a. AtallRwavei nleadVI .
b. Ventri
cularpre-exci
tati
on.
c. AlongQTi nt
erval.
d. NegativePwavesi nleadsI
IandI
IIandaVF.
e. At
rialfi
bril
lat
ion.

Answer
:

20. a. At
allRwavei
nleadVI
.

Thecor r
ectansweri sat al
lRwavei nleadV1.Thepat i
enthaspul monaryhyper tension
secondar yt
ochroni cpulmonar yembol i
.AtallRwavei sconsi
stentwithrightventricul
ar
hypertr
ophyandpul monar yhypertensi
on.Inacut epulmonaryembol i,onemi ghtexpectanS
wavei nleadI(duet ori
ghtaxisf r
om leftpost
eriorfascicul
arblock),
Qwavei nleadI II
,and
i
nvertedTwavei nleadIII(McGinn- Whitesi
gn).Itisfoundinonly10%ofpat i
entswi thanacute
pulmonar yemboli.Atri
alfi
bril
lat
iononlyhasaweakassoci at
ionwi t
hpul monar yembol i
.Long
QT,Wol f
f-Parki
nson- Whitesyndrome, oratri
alfociarenotclassical
lyseeninchr onic
pulmonar yhypert
ension.

Quest
ion:

21. A33- year


-oldwomanwi thahi st
oryofhypertensi
onpr esentswi
t hsharpchestpainat
rest,
associ at
edwi thdyspnea.Sheis10dayspost oper at
ivefr
om ahyst erectomy.Herresti
ng
oxygensat urati
onis91%.Herexami nati
onisotherwisenormal .Heronl
ymedi cati
onisbirt
h
controlpil
ls.Herelect
rocardi
ogram showssi nustachycardiaandT- wavei nversi
oninthe
anteri
orleads.TroponinIiselevatedat0.8ng/L.Shei sgivenaspiri
nintheemer gencyroom,
butcont i
nuest ocompl ai
nofdyspneaandchestpai n.

Whi
choft
hef
oll
owi
ngshoul
dbet
henextst
ep?

a. Obtaincomput edtomographypul monar


yangi
ogr
am.
b. Schedulecardiaccathet
eri
zati
on.
c. Referforur
gentcar di
accatheter
izat
ion.
d. Scheduleexercisestr
esstest.
Answer
:

21. a. Obt
aincomput
edt
omogr
aphypul
monar
yangi
ogr
am.

Thenonspeci f
icelectr
ocardi
ogram findi
ngsinayoungpat i
entwithalowpr etestprobabil
it
yfor
coronaryarter
ydiseaseshouldpr omptconsider at
ionofalt
ernat
ivediagnoses.Whi letr
oponin
el
evationisspecifi
cformyocardialinjur
y,i
tisnotspeci f
icf
ortheetiol
ogy,andpul monar y
embolism (PE)isaknowncauseoft roponinelevati
on.Inpati
entswithPE,thepr esenceof
tr
oponinelevati
oni sanadversepr ognosti
cindicatorandshouldpromptanaggr essive
diagnosti
candt herapeut
iccourse.

Quest
ion:

22. A45- year-oldwomanpr esentswith6mont hsofwor seningdyspneaonexer ti


on.Her
symptomshavewor senedtothepointthatsheisunabl etoper f
orm herdail
yacti
viti
es.She
undergoesechocardiogr
aphiceval
uation,ri
ght-heartcatheterizati
on,hi
gh-resol
utionchest
computedtomogr aphy(CT),andchestCTwi thapul monar yembol i
sm (PE)prot
ocol,which
reveal
edadiagnosisofprimarypulmonar yhypertensi
on( PH) .Bosentantherapyisini
ti
ated.

Whi
choft
hef
oll
owi
ngi
sthebesteval
uat
iont
odet
ermi
neef
fi
cacyoft
her
apy?

a. 6-minutewal kdist
ance>380met ers.
b. VO2 great
erthan18ml /min/
kg.
c. Cardiacindex≥3.5L/min/m2.
d. WorldHeal t
hOr gani
zat
ion(WHO)f uncti
onalcl
assIIorI
IIsympt
oms.
e. Brai
nnat ri
ureti
cpepti
de(BNP)levellessthan500pg/ml.

Answer
:

22. a. 6-
minut
ewal
kdi
stance>380met
ers.

Theabsol uteincreaseinthedistancewalkedf r
om baseli
newhi l
eont reat
menthasnotbeen
shownt ocorr
elatewellwithlong-ter
m outcomes.However ,anabsolutevalue,thatis,
a
thresholddist
ance, hasbeenshownt opredictsurvi
val
.Thisthreshol
dwas380met ersinone
studyand440met ersinanother.Achi
eving6- minutewal
kdi st
ancesabovet hisrangeisa
gener al
lyacceptedgoalforpatientsontherapiesforpul
monar yarter
yhypertension.Other
predictorsofeff
icacyincl
udenor malornearnor malBNP, normalornearnor malr i
ght
ventri
cularfuncti
on, WHOf uncti
onalclassIorIIsymptoms, car
diacindex=2. 5L/mi m2,
n/ or
VO2 ≥15ml /min/kg.

Quest
ion:

23. A40- year-


oldwomani sseen6weekspostdi schargeforpulmonar yhypert
ension( PH).
Sincedi
schar ge,
herdyspneahasi mproved,butshecont i
nuest ohaveexerti
onalfat
igueand
mildort
hopnea.Hermedi calhist
oryissigni
ficantf
ortobaccouse, hyper
tension,
andobesi ty.
Hercurrentmedicati
onsincludeloopdiuret
icandanangi otensi
n-converti
ngenzymei nhibitor
.
Onexami nat
ion,herbloodpressureis132/78mm Hg, hear
trate72bpm, andbodymassi ndex
33.Herjugul
arvenouspr essureis10cm H2O, andherlungsar ecl
ear. 

Car
diovascul
arexami
nat
ionr
eveal
snor
malS1 
andS2,
andr
ight
-si
deS4.Nomur
mur
s,r
ubs,
or
heavesappr eciated.Ext
remi t
ieshavet raceedema.Echocar di
ogram att
hetimeof
hospitalizationshowedsever econcent r
iclef
tventr
icul
arhypertr
ophywithnormalsystol
ic
function.Pul monar yart
erysyst oli
cpressurewasest i
mat edat88mm Hg.Sheunder wentright
-
heartcat heterizationandherr esult
sare:ri
ghtatri
alpressure15mm Hg;r i
ghtvent
ri
cular
pressur e90/ 15mm Hg;pul monar yarterypressur
e90/ 25(47)mm Hg;pulmonarycapill
ary
wedgepr essure( PCWP)22mm Hg;aor tapressur
e150/ 100mm Hg;car di
acouput3.9L/min;
2
cardiaci ndex1. 8L/min/m.

Basedonther
ight
-hear
tcat
het
eri
zat
ionhemodynami
cs,
whatf
urt
herst
udi
esshoul
dbe
consi
der
ed?

a. Venti
lati
on-per
fusionscan.
b. Repeatright
-heartcathet
eri
zat
ionwithacutevasodi
lat
orchal
lenge.
c. Transesophagealechocardi
ography(TEE)
.
d. Cardiacmagneticresonanceimaging(MRI)
.

Answer
:

23. a. Vent
il
ati
on-
per
fusi
onscan.

hecor rectansweri
savent il
ati
on-per
fusionscant oexcludethr
omboembolicdi
sease.Her
calculatedpulmonar
yvascul arr
esist
ance( PVR)is6.4(normal<2.0)
.Thi
sisder
ivedbydivi
ding
thetranspulmonarypressuregradi
ent/cardiacoutput
.Thisimpli
esther
ecouldbeasecondary
processbesi dest
heelevatedPCWPatwor kinthi
spati
ent. 

Thisclini
calscenarioisanexampl eofreacti
vepost capill
aryPH,givent heelevat
ionofthe
surrogateofleftatr
ialpressureandat ranspulmonar ypressuregradient>12mm Hg, i
mpl yi
ng
thatthereisalsoacomponentofl eftheartdiseasepr esent.Gi
venherobesi tyandsignif
icant
tobaccouse, thereareotherdiseaseprocessest hatmaybepl ayi
ngar olei
nt hepati
ent’
sPH,
suchas  sleepapneaorchr oni
cthromboembol i
cdisease.Comput edt omographyofthechest
withapul monar yembol i
sm protocoloraV/ Qscani st henextbestst epinthispati
ent,
andt he
V/Qscani snowpr eferr
ed. 

Asleepst udywoul dalsobeappropri


ate.Acutevasodi
lat
ortesti
ngwi t
hinhal
ednitr
icoxideis
rel
ati
velycontraindi
cat edi
npati
entswithmildlyel
evat
edleftheartf
il
li
ngpressur
es,especiall
y
aft
erar ecentcli
nicaldecompensati
onast hi
scanacut el
yincreasethePCWPbyi ncreasi
ng
pulmonar yfl
owandl eadtocl
ini
caldecompensat i
onduetopul monaryedema.Thereisnor eal
advantageineitheraTEEorcar di
acMRIi nthissit
uati
on.

Quest
ion:

24. A45- year


-ol
dmani sadmit
tedt
ot hehospit
alwit
hshort
nessofbreat
h.Hi
shist
oryi
s
si
gni
fi
cantformorbi
dobesi t
yandhypert
ensiondiagnosed15year
sago.Hedoesnotrecal
lhi
s
bl
oodpressuremedi
cations.Hisbl
oodpressureonadmi ssi
oni
s162/100mm Hg,hear
trat
e80
bpm,andbodymassi ndexis41. 

Anechocardiogram i
sperformedthat 
showsl ef
tventr
icul
arhypert
rophywi t
hnor mallef
t
ventr
icul
arsystoli
cfunct
ion;theri
ghtventr
icl
eisnormalinsizeandsystoli
cf unct
ion,wit
h
pulmonaryarter
ysystoli
cpressureesti
matedat59mm Hg.Ar ight
-heartcatheter
izat
ionis
perf
ormed,andt hehemodynami csare:r
ightatr
ialpr
essure10mm Hg;r ightventr
icul
ar
pressur
e62/10mm Hg;
 pul
monaryar
ter
ypr
essure(PAP)62/
21(
35)mm Hg;pulmonar
y
capil
lar
ywedgepr
essur
e(PCWP)17mm Hg;car
diacouput4.
1L/
min,andcar
diaci
ndex2.
2
2
L/min/m.

Whichofthefol
lowingi
stheappr
opri
atecl
ini
calWorl
dHeal
thOr
gani
zat
ion(
WHO)
cl
assif
icat
ionofthi
spul
monaryhyper
tensi
on(PH)?

a. Gr
oup5.
b. Gr
oup4.
c. Gr
oup1.
d. Gr
oup3.
e. Gr
oup2.

Answer
:

24. e. Gr
oup2.

Thi
sclini
calscenari
oisanexampl eofPHsecondarytoleftheartdisease,whi chisthemost
commoncl assofPH( WHOGr oup2).ThePHi saresultofreacti
vepost capil
laryPH, given
el
evat
ionofsur r
ogatelef
tatr
ialpressur
eandat ranspulmonar ypressuregradi ent>12mm Hg.
Aswit
hr eacti
vepostcapi
ll
aryPH, passi
vepostcapil
lar
yPHi sdefinedbyameanPAP  ≥25mm
HgandaPCWP>15mm Hg.However ,thet
ranspulmonarypr essuregradientis>12mm Hg.  

Group1ispulmonaryart
erialhypertensi
on(PAH),defi
nedasmeanPAP≥25mm Hgatr estand
meanPCWP<15mm Hg.Gr oup3i sdef i
nedasPHduet ochroni
clungdi
seaseandor
hypoxemia.Group4i
sdefinedasPHsecondar ytochronicthromboembol
icdi
sease.Gr
oup5
i
ncludesPHf r
om mult
if
actorialcauseswithoutaclear
,unif
yingdi
agnosi
s.
Syst
emicDi
sor
der
sAf
fect
ingt
heCi
rcul
ator
y
Syst
em
Quest
ion:

1. A44- year-oldwomani sreferredtoyoubyherpr i


mar ycar ephysi cianduet odyspnea
onexerti
on,dizziness,andvaguecompl ai
ntsforthelast3mont hs.Thepat ienthasno
signi
fi
cantmedi calhist
or y.Herf atherhadast rokewhenhewas50year sol d.Shet ell
syou
thatshehasbeenexper iencingf ever,chil
ls,
weightloss,art
hr algias,andbl uedi scolorat
ionof
herfi
ngersandt oesi nthecol d.Onexami nati
onherhear tratei s88bpm, bloodpr essure
133/78mm Hg, andpul seoxygeni s100%onr oom air.Shei sfebr i
leat101°F, herfingersare
cyanoti
c,andt hereisdigitalclubbing.Onauscul tat
ion,hercar diacexam r evealsal oudS1 and
asoftdiast
olicsoundandr umbl eattheapex.Anechocar diogram i sobtained( Figure1).

Whi
choft
hef
oll
owi
ngi
sthemostl
ikel
ycauseofhersympt
oms?

a. Nonbacteri
althrombot i
cendocar
dit
is.
b. Metast
aticrenalcellcarci
noma.
c. SubacuteInfect
iveendocardi
ti
s.
d. Fungalendocardit
is.
e. Cardi
acmyxoma.
Answer
:

1. e. Car
diacmyxoma.

Myxomasaccountf or40- 50%ofpr i


mar ycardiactumor s.Approximately90%aresol i
taryand
pedunculat
ed,and75- 85%occuri ntheleftatri
alcavit
y.Upt o25%ofcasesar efoundi nthe
ri
ghtatr
ium.Mostcasesar esporadic.Approximately10%ar efami l
ialandaretr
ansmi ttedi
nan
autosomaldominantmode.Mul t
ipletumor soccurinappr oxi
mat el
y50%off amilialcasesand
aremoref r
equent
lylocatedinthevent r
icl
e( 13%vs.2%i nsporadiccases).
 

Symptomsr angefr
om nonspeci
fi
candconst
ituti
onaltosuddencardiacdeath.Inabout20%of
cases,myxomamaybeasympt omati
canddiscover
edasani ncident
alfi
nding.Const i
tut
ional
symptomst hati
ncl
udefever
,weightl
oss,
art
hralgi
as,andRaynaudphenomenonar eobserved
i
n50%ofpat i
ent
s.Thesesymptomsmayber elat
edtooverproducti
onofinterl
eukin-6. 

Theechocar diogram clearl


yshowsawel l-definedcar diacmassobst ruct ingthemi tr
alor i
fi
ce,
suggesti
veofl eftat
rialmyxomawi thahi ghr iskofembol izati
on.Car diaccomput ed
tomographyandmagnet icresonancei magi ngcanbeper f
ormedf ortissuechar act
er i
zati
on.
Inf
ecti
veendocar di
ti
sandnonbact eri
alendocar dit
isarelesslikel
ygi vent heechofindingsand
hersympt omsr elat
edt oobstructi
onofbl oodf lowf rom theleftat
rium t ot heventri
clebyt he
mass.Fungalendocar dit
iscancausel argeveget ati
onst hatmaybecomeobst r
uctive,butare
nottypi
callyassociatedwi t
ht hesympt omsdescr i
bedher ethatarer elatedt otheIL-6secr eti
on.
Renalcellcarcinoma, alt
houghassoci atedwi thsyst emicsympt oms, t
ypi call
yspreadst hrough
theinf
eri
orvenacavat otherightheart.Sympt omsar ecausedbyobst ruct i
onatthemi tral
valveori
fi
ce.Val vedamagemayr esultinmi tralr egurgi
tati
on.Sever edi zziness/syncopei s
experi
encedbyappr oximately20%ofpat ients.Themostf requentcausei npat i
entswi t
hl ef
t
atr
ialmyxomasi sobst r
ucti
onoft hemi t
ralvalve.Sympt omsmaychangeast hepat i
ent
changesposi ti
ons.

Quest
ion:

2. A58- year-oldwomani srefer


redt
oyourclini
cf oreval
uati
onandmanagementofat r
ial
fi
bril
lat
ion(AF) .Hercomor bi
ditiesincl
udehypert
ension,metaboli
csyndrome,and
dysfuncti
onalut eri
nebleeding.Shewasf ir
stdiagnosedwi t
hparoxysmalAF6year spri
orto
presentati
on.Hersympt omsi ncludeshort
nessofbreath,dyspneaonexert
ion,pal
pit
ati
ons,
andf at
igue.Herhusbandnot est hatshesnor
es.Whi l
eshecanexper i
enceepisodesatallt
imes
oftheday, shenot icest
hesympt omsmost l
ywhensheawakens.  

Onexami nation,herbl
oodpr essureis132/78mm Hg, herheartr
ateis78bpm, andherbody
massi ndexis36.Hercar di
ovascul arexam i
sunr emar kabl
e.Medicati
onsincludemet oprol
ol
succi
nat e50mgdai l
y,l
isi
nopril20mgdai ly,andr i
varoxaban20mgdai l
y.Herwor kupshowsD
-dimerandt hyroi
d-sti
mulatinghormonewi t
hinthenor malrange.Echocardiogram
demonst rat
esanej ect
ionfr
act i
onof55%, septalwal lt
hicknessof1.6cm, andal ef
tatri
um of
4.4cm.Theel ect
rocar
diogram revealsAFatar ateof80bpm andl ef
tventri
cularhypert
rophy
(LVH).

Whi
choft
hef
oll
owi
ngi
sthenextbestst
epi
nthi
spat
ient
?

a. Ami
odarone200mgdai l
y.
b. I
ncr
easemetoprol
oldoseto100mgdai
ly.
c. Cat
heterabl
ati
on.
d. Fl
ecai
nide50mgt wicedai
ly.
e. Pol
ysomnography.

Answer
:

2. e. Pol
ysomnogr
aphy.

Thepat i
enti
sobeseandherear lymor ningsympt omsmaywel lrepresentAFpr ovokedby
unt r
eatedsleepapnea.Treatmentofherunder lyi
ngsleepapneamayi mpr oveherAFand
associ at
edsymptoms.Bef orepursui
ngphar macologi
corinvasi
vether api
es, t
her apyshouldbe
directedattheunderl
yingcause.Flecainideisnotindi
catedinpati
entswi thstructuralhear
t
diseaseandLVH.Ami odaroneisnotfir
st -l
inet
herapyinayoungpat ient.Catheterablati
onis
usual l
yreservedf
orpatientswhof ai
lmedi calt
herapy.

Quest
ion:

3. Ther
eisani
ncr
easei
ncar
diovascul
ardi
seaseassoci
atedwi
thobst
ruct
ivesl
eepapnea
(OSA)
.

Whichoft
hef
oll
owi
ngef
fect
shasbeenshownt
obeexer
tedupont
hecar
diovascul
arsyst
em
byOSA?

a. Decreasedi
nflammat orymar ker
s.
b. I
ncreasedparasympat hetict
one.
c. Decreasedendot hel
inlevel
s.
d. I
ncreasedplateletacti
vati
on.
e. Decreasedfi
brinogenlevels.

Answer
:

3. d. I
ncr
easedpl
atel
etact
ivat
ion.

Thecor r
ectansweri sincreasedpl ateletactivati
on.OSAr esult
sinani ncreaseinprothrombotic
acti
vi t
yinvivo. 
Thi sincludesani ncreasei npl atel
etactivati
onandani ncreaseinfi
brinogen
l
evel s.Pati
entswi t
hOSAal soexhi bitincreasesi nsympat hetictone,leadingt ohi
gherr est
ing
pulsesandal sopot entiall
yaggravat i
ngunder lyi
nghyper tension.Endot heli
nlevelsare
i
ncr easedinpatientswi thOSA, and, inter
estingly,decreasef ol
lowingt reatmentwith
cont i
nuouspositiveai r
waypr essur e.Elevatedendot heli
nl evel
sar ethoughtt ocontri
but et
o
hyper t
ensioninunt reatedOSApat i
ent s.Inf
lammat or
ymar kersarealsoi ncreasedinsome
patients.

Quest
ion:

4. Youar eseei
nga42- year-ol
dwomani ncli
nicforfol
low-upofstageCdi l
ated
cardi
omyopat hyandNewYor kHeartAssoci
ati
on(NYHA)classIIheartfai
lur
ewi t
hal ef
t
ventr
icularejecti
onfract
ion(LVEF)of35%.Shedeniesanydyspnea, or
thopnea,chestpain,
or
palpi
tations.Herhusband,whoaccompani esher,
expressesconcernthatsheoftenfall
sasleep
watchingTVatt heendoftheday.Shewal ks40minuteseveryotherdayforexerci
se
(unchangedovert hepast2year s).
 
Hermedi calhistor yincludespost par
tum car diomyopat hy,whichhasbecomechr onic.She
doesnotsmokeordr inkanyal cohol.Shei songui deline-di
rectlymedi caltherapyf orher
cardiomyopathy.Physi calexami nati
oni sremar kableforabodymassi ndexof34. 5,whichis
stablesi
nceherl astvisit,bloodpr essure100/ 58mm Hg, heartrate64bpm, respiratoryrat
e12
breathspermi nut e,andoxygensat urati
on99%onr oom ai r
.Neckvei nsar eflatsi
t ti
ngupr i
ght
,
andt hehepatojugul arreflexisnegat ive.Thel ungsar eclearbilater
all
ywi t
hnor ales.The
cardiacexaminat iononl yr evealsaI I
/VIhol osystol
icmur muratt heapex.Theext r emiti
es
revealnoedema.Recentl absincl udeat hyroid-sti
mulat i
nghor monel evelof1. 2andnor mal
compl et
ebloodcount .

Whi
choft
hef
oll
owi
ngi
sthenextbestt
estt
oeval
uat
eherongoi
ngf
ati
gue?

a. Pol
ysomnogr am.
b. Car
diopul
monar yexer ci
setest
ing.
c. Pul
monaryf uncti
ont est
s.
d. Car
diacmagnet i
cr esonanceimaging(
MRI
).
e. Ri
ghtheartcatheteri
zati
on.

Answer
:

4. a. Pol
ysomnogr
am.

Sleepdi sordersar ecommoni npat i


entswi t
hhear tfailure( HF) ,
wi thupt o61%ofHFpat ient s
havingei thercent ralorobst ructivesl eepapnea( OSA) . Despi tehavi ngl esssl eept i
meand
sleepef f
iciencycompar edwi tht hosewi thoutHF, pat i
ent swi thHF, includi ngthosewi th
document edsl eepdi sorders, rarel
yr epor texcessi vedayt i
mesl eepi ness.  
Thus, ahi ghdegr ee
ofsuspi cionforsl eepdi sorder sshoul dbemai ntainedf ort hesepat ients.Inamaj ortrial
,
cont i
nuousposi ti
veai rwaypr essur eforOSAwasef fectivei ndecr easingt heapnea–hypopnea
i
ndex, i
mpr ovingnoct urnaloxygenat i
on, increasingLVEF, loweringnor epinephr i
nel evels, and
i
ncr easingt hedist ancewal kedi n6mi nutes;t hesebenef it
swer esust ainedf orupt o2year s.
Smal lerstudiessuggestt hatcont inuousposi tiveai rwaypr essur ecani mpr ovecar diacf unction
andqual i
tyofl i
feinpat i
entswi t
hHFandOSA.Manypat ientswi thHFhavecent ralsleepapnea,
andt herear erecentdat athatsuggestt reatmentwi thser vo- adapt i
vevent il
ati
onmayact uall
y
resultinwor secar di
ovascul arout comes.  Alt
houghwor seni ngHFcanal socausef atigue, she
hasnoevi denceofvol umeover loadorl ow- flowst ateonexam, andshehasst ableHF
sympt omsonar egimenofneur ohor monal -
modi f
yi ngdr ugs, sothatr i
ghthear tcathet erizati
on,
cardiopulmonar yexer cisetest i
ng, orcar diacMRIwoul dnotbeusef ulatt histime.Depr ession
shoul dalsobeconsi dered,especi allyifthesl eepst udyi snor mal.

Quest
ion:

5. A72- year-oldwomanwi t
hhist oryofischemi ccar diomyopat hycomeswi thher
husbandt oyourcar di
ologyclinicforroutinefollow-up.Thepat i
entreportsstabledyspnea
whil
ecl i
mbi ngst ai
rs.Hercurrentmedi cati
onsi ncludeaspi rin81mgdai ly,at
or vastati
n40mg
dail
y,carvedilol12.5mgt wicedai l
y,lisinopri
l40mgdai ly,andf urosemide40mgdai l
y.Vi
tal
si
gnsar ehear trate76bpm, bloodpr essur e122/ 65mm Hg, 16r espi
rat
ionspermi nut
e,and
oxygensat urati
on96%whi lebr eat
hingambi entai r
.Onexam, shei seuvolemic.An
echocardiogram f r
om 6mont hsagodemonst ratedal eftvent r
icularej
ectionfract i
onof32%.An
el
ectrocardiogram obtainedint heofficeshowssi nusr hythm wi thincompl et
elef tbundle
br
anchbl
ock(
LBBB)
,QRSdur
ati
on110msec,
andevi
denceofapr
iori
nfer
iori
nfar
ct.

Herhusbandment
ionst
hatwhilehi
swif
eissleepi
ngatni
ght,hehasnoticedt
hatshewi
llst
op
br
eathi
ngforsometi
me,fol
lowedbyper
iodsofmorerapi
d,deepbreat
hing.

Whi
choft
hef
oll
owi
ngdoyour
ecommendf
ort
reat
mentofhercondi
ti
on?

a. Theophylli
ne.
b. Conti
nuecur renttherapy.
c. Cardi
acr esynchronizati
onther
apy.
d. Adapti
veser voventi
lati
on.
e. Spi
ronolactone.

Answer
:

5. e. Spi
ronol
act
one.

Cheynes- Stokesr espirati


onsareaf orm ofcent
r alsl
eepapnea, asubsetofsl eep-di
sordered
breathing,t
hatmaybeseeni npat i
entwithheartfail
ure.Thebestt r
eatmentt odat eis
optimizati
onoft her apyforcongest i
veheartfai
lure.Adapti
veser vovent
ilat
ionwasassoci ated
withhar mint heSERVE- HFt r
ialforpat
ientswit
hhear tfail
ureandcent ralsleepapneaandt hus
woul dnotbei ndicat ed.Thispati
enthasani ncompl eteLBBBf orwhichcar diac
resynchronizati
ont herapywouldnotbei ndicat
ed.Theophyl l
inehasnor oleint hetr
eatmentof
Cheynes- Stokesr espirati
ons.Giventhi
spat i
ent'sheartfai
luresympt oms, additionof
spironolact
onei st hebestopt i
on.

Quest
ion:

6.  
55-year
-oldobesemanwi t
hsympt omaticpar oxysmalatri
alf
ibr
il
lati
onpresentsto
cli
nicforchroni
cfati
gue.Hefeel
stir
edal ltheti
me, andf al
lsasleepwheneverheiswatching
TVorr eadi
ngabook.Hei sfearf
ulbecausehef ellasleepr ecent
lywhi
ledrivi
ngintr
affi
c,which
result
edinami normotorvehicl
eaccident.Hiswifesayst hathesnoresloudlyandhas
episodeswhenhest opsbreathi
ngatnight.

Forwhi
choft
hef
oll
owi
ngi
sthepat
ientati
ncr
easedr
isk,
ifl
eftunt
reat
ed?

a. Al
teredtestoster
onelevels.
b. Atr
ialf
ibri
ll
ation.
c. Symptomat icbradycar
dia.
d. Val
vularhear tdi
sease.
e. Suddencar diacdeathduringwakef
ulhour
s.

Answer
:

6. b. At
ri
alf
ibr
il
lat
ion.

Thecor r
ectansweri sincreasedriskofatr
ialf
ibr
il
lati
onrel
atedtountr
eat edobstructi
vesleep
apnea(OSA) .Ithasbeenshownt hatuntr
eatedOSAi sassociat
edwithrecurrentatri
al
fi
bri
ll
ati
on,includingaftercardi
oversi
on.However, whentr
eated,t
heincidenceofr ecurr
ent
atr
ialf
ibri
ll
ationdecreases.Increasedri
skofsuddencar di
acdeathistrueinpatientswithOSA,
buthasbeenf oundtooccurdur ingsleephours,notduringwakef
ulness.Valvularheartdi
sease
i
snotassociatedwi t
hthepr esenceofOSA.Longst andingOSAcanl eadtopul monar y
hypert
ensi
onandr esultanttri
cuspidregurgi
tat
ion,buttherei
snodirectcorrel
ation.Thereis
ei
thernochangeoradecr easeintestoster
onelevelassociat
edwiththepresenceofsl eep
apnea.Whil
eapnei cepisodescanr esulti
narrhythmiassuchasbradycardia,theseoccurwhi l
e
sl
eeping,t
herefor
ear enotconsideredsympt omat i
c.WhenOSAi str
eated,orwhi lethepati
ent
i
sawake, t
heseapnei c-associat
edar r
hythmiasresolve.

Quest
ion:

7. Youar econsultedregardinga40- year-


oldwomanwhohasknownCushi ng'ssyndr
ome
andpresentswi thatransi
entischemicattack.Herechocardiogr
am showsalargeleftat
ri
al
myxoma30mm x32mm x28mm.Shet ell
syouherf at
herhadat umorcutoutofhisheart
aroundthesameageasshei snow, butthet umorgrewbackseveralyear
slat
er.Sheneeds
surgi
calint
ervention,
butherr ealconcer
ni sherchil
dren,ages24and18years,andt hei
rri
skof
si
mi l
arhealt
hi ssues.

Whichofthefol
lowi
ngdoyour ecommendt
othepat
ientr
egar
dingt
heher
it
abi
li
tyor
t
ransmissi
bil
it
yofhercondi
ti
on?

a. Sugge stthatt
hepatientbet
estedfortheFBN1genemut at
ion.
b. Testthemyxomaf orCongoredstaini
ngfoll
owingsurgery.
c. Obtai
nechocar di
ographyonherchi l
dren.
d. Reassur ethepat
ientther
earenosuchher it
ablecondit
ionstoconsi
der
.
e. Sugge stthatt
hepatientbet
estedforEpstei
n-Barrvi
rus.

Answer
:

7. c. Obt
ainechocar
diogr
aphyonherchi
ldr
en.

Mostmyxomasar esporadic,
butt
heymaybef amili
al.Inpar t
icular,t
heCar neyCompl exor
CarneySyndromei sanautosomaldomi nantfamili
almyxomasyndr ome.Thi spati
entscenario
descri
bespr i
marypigmentednodularadrenocorti
caldi sease(PPNAD) ,whichcanpr oduce
Cushingsydrome, anendocri
nedi
sordercommonl yassoci atedwi thCar neySyndr ome.Gi ven
thefamil
ialpatt
ern,t
heansweristoexami nethechildrenf orsimilarfeaturesoft hi
ssyndr ome.
Epstei
n-Barrvir
us,FBN1genemut ati
ons( Marfansyndr ome) ,andcar diacamyl oidosis(Congo
redstai
ning)areunrel
atedtocar
diacmyxomas, thereforetestingisnotwar ranted.

Quest
ion:

8. A48-year-oldwomanwi thahi storyofhypertensionandseizur


esseeksassistance
withsmoki ngcessation.Shehassmoked1packperdayf ort
helast25years.Shetr
iedtoqui
t
severalyearsagobyusi nganicoti
nepat ch,
butwasunsuccessf ul.Medicati
onsincl
ude
hydrochlorot
hiazide25mgandl evetir
acet am 1000mgt wicedai
ly.Shetel
lsyouthather
brotherwasr ecentl
yabletostopsmoki ngaf t
erheusedbupr opi
onpharmacot her
apyandshe
woul dli
ketot r
ybupr opi
on.Shedeniesanydepr essi
vesympt oms.
Whi
choft
hef
oll
owi
ngwoul
dbet
hemostappr
opr
iat
erecommendat
ionf
ort
hispat
ient
?

a. Nicoti
nenasalspray6dosesdai l
y.
b. Bupropion150mgt wicedaily.
c. Nicoti
nei nhal
er10cart
ri
dgesdaily.
d. Nicoti
negum asneeded.
e. Varenicl
ine1mgt wicedail
y.

Answer
:

8. e. Var
eni
cli
ne1mgt
wicedai
ly.

Thehighestabst i
nenceratesarewi t
hvarenicli
netherapy,andgivenherfailedat t
empti nthe
pastwitht
heni cotinepatch,i
tisanappropriatetherapy.Bupropi
oniscont raindi
cat edwit
hher
hi
storyofseizures.Ofthethreenicoti
ne-
replacementt herapi
es,t
henicotinei nhalerist
hemost
ef
fecti
venicoti
nei nter
venti
onbecauseoft heincreaseddependencyr ateswi ththenasalspray,
butiti
ssti
lli
nferiortovareni
cli
net her
apy.Inaddi t
iontomedicalther
apy, patientsshould
al
waysbepr ovidedwi thcounseli
ngandavai lableresources,
suchast hequi tli
ne.

Quest
ion:

9. A66-
year-
oldr
ight-
handedmanpr esentswit
hdizzi
nessandri
ghtarmfati
guewhil
e
painti
nghisgar
agedoor.Hehasbil
ateralcervi
calandsupracl
avi
cul
arbrui
ts.Theri
ghtar
m
pressurei
s100/72mm Hg, andt
heleftarm pressur
eis146/80mm Hg.

Magnet
icr
esonanceangiogr
aphyshowsa90%stenosi
soft heinnomi
nateart
ery,a60%
st
enosi
softher
ighti
nter
nalcarot
idar
ter
y,anda75%stenosisofthel
efti
nter
nalcar
oti
dar
ter
y.

Whi
choft
hef
oll
owi
ngt
reat
ment
sisi
ndi
cat
edf
orhi
ssympt
oms?

a. Leftcar
otidar
teryendart
erect
omy.
b. Leftcar
otidar
terystent
.
c. Innominatear
terystent
.
d. Rightcarot
idart
eryendart
erect
omy.

Answer
:

9. c. I
nnomi
nat
ear
ter
yst
ent
.

Thispati
enthasclassicsymptomsofrightarm claudicati
onandsubcl avi
anstealresult
ing
fr
om ahigh-gradestenosisoft
heinnominateartery.Withrightarm ef
fort
,ther
eisli
mb
vasodi
lat
ionandr et
rogradefl
owdownt herightvertebralarter
ytoaccommodat et
hedi l
ated
vessel
s.Theleftandri
ghtcaroti
darter
ystenosesar enotcausi ngthesesympt oms,ther
efore
tr
eatmentsdir
ectedatthesearenotthecorrectoptions.
Quest
ion:

10. A63- year-oldmal enonsmokerunder wentcar oti


dultr
asonographyaspartofhis
evaluationforasyncopalevent .Whilediamet erreductionwasnotevi dentduetopoorimage
quality,
Doppl errecordi
ngssuggest edincreasedvel ociti
esinthelefti
nter
nalcar
oti
dartery
l
eadi ngt oanest i
mat i
onof<50%di seaseint herightinternalcar
otidarter
yand50-69%
stenosisi nt
hel efti
nternalcar
otidart
ery.Carotidultrasonographyperformed2yearsearl
ier
suggest ed<50%bi l
ater
aldisease,r
aisi
ngt heconcer naboutpr ogressivel
eft
-si
dedstenosis.

Hewasr ef
err
edforcaroti
dart
eryangiography:magnet
icresonanceangiogram (MRA)is
showninFigur
e1.Serum chemist
ri
esi ndicat
edalow-densityl
ipoprot
eincholest
eroll
evelof
63mg/dl,r
educedfrom 114mg/dlfr
om 2year spri
orwit
hi nsti
tut
ionofli
fest
ylechangesand
l
ow-dosestati
nther
apy.

Whichoft
hef
oll
owi
ngi
stheappr
opr
iat
emanagementofcar
oti
dar
ter
yat
her
oscl
erosi
sfort
his
pat
ient
?

a. Ref
erf ori
nvasiveheadandneckangi
ogr
aphy.
b. Ref
erf orl
eftcarot
idendar
ter
ect
omy.
c. I
ncreasestatindose.
d. Noc hangeinmanagement .

Answer
:

10. d. Nochangei
nmanagement
.
Thecont rast-enhancedMRApr ovidesexcel l
entvisuali
zationoft hecar oti
dar teri
es, wit
hno
evidenceofsi gni f
icantst enosis, andmi l
dl uminalirr
egulari
tyint heinternalcar oti
dar t
er i
es
consistentwithhi sor iginaldupl excarotidultr
asoundexami nation.Thus, invasiveangi ography
toconf i
rmt hesef indingsi snotnecessar y.MRAwasusef ulinthisinstancet or esolve
discordantser ialultr
asoundf indings;intheabsenceofsympt omsandsi gnif
icantst enosis,
endarterectomyi snoti ndicated.Thi spatient’
slow- densit
ylipopr ot
einachi evedwi thst ati
n
therapyandl ifestylechangesi swel lbelowt argetforapat i
entwi thevidentat herosclerosis,and
managementshoul dr emai nunchanged.

Quest
ion:

11. A56- year-ol


dwomanpr esentstotheemergencydepart
mentwi t
hsympt omsof
amaur osi
sfugax.Hersympt omsr esolvewithi
n15minutesofpresentati
on.Shehasamedical
hi
storyofhypertension,andsheisanact i
vesmoker.Herheartr
atei s85bpm andbl ood
pressurei
s142/ 84mm Hg.Physi calexami nat
ioni
snotabl
eforar ightcaroti
dbrui
t.An
el
ectrocar
diogram revealsnor
malsi nusrhythm.Headcomputedt omogr aphyisnormal
.

Whi
choft
hef
oll
owi
ngi
sthemostappr
opr
iat
enextst
ep?

a. Electr
oencephal
ogram.
b. Dischargehome.
c. Implantabl
elooprecorder
.
d. Erythr
ocytesedi
ment at
ionrat
e.
e. Carotidart
eryul
tr
asonography.

Answer
:

11. e. Car
oti
dar
ter
yul
tr
asonogr
aphy.

Thecor rectansweri nt hiscasei stoper form addi t


ionalevaluationforsuspectedsympt omat i
c
caroti
dar terydiseasewi thul t
rasonogr aphyorcomput edtomogr aphy/magneticresonance
angiography.I fsi
gnificantcarotiddiseasei sfoundi nthi
spat i
ent ,i
twouldwar rant
considerationofintervention.Itisnotappr opriatetodischarget hispati
entwit
houtf ur
ther
evaluati
on, becausei tmaydef errevascul ari
zat i
oninsympt omat i
ccaroti
dstenosi s.Transient
i
schemi cat tackisthemosti mpor tantpredi ctorofstroke,withupt o13%ofpat i
ent shavinga
strokewi t
hin90daysofi nit
ialpresentati
onandupwar dof30%wi t
hin5years.Thi spatient'
s
sympt omsofamaur osisfugaxi snotl i
kelyt obeexpl ainedbyasei zureortempor alarteri
ti
s.
Occultatrialfi
bril
lat
ioni sani mpor t
antcauseofcr yptogenicstroke,andani mpl antabl
el oop
recordermayhel ptomaket hi
sdi agnosisaf termor eurgentdi seaseshavebeenexcl uded.

Quest
ion:

12. A40- year-


oldwomanwi thnomedicalhist
orypresentstotheemer gencyroom with
progressivel
eft
-sidedfacialweaknessof2daysdur ati
on.Heriniti
alsymptomsst art
ed1week
agowi thneckpain,headache,andsomenausea.Shedeni estakinganynewmedi cati
onsand
saidshewasi nami norcaraccident1daypriortoonsetofhersympt oms.Herbloodpr essur
e
i
s142/ 82mm Hgi nbot harmswi thahear
trateof105bpm onpr esentat
ion.Examinati
onis
notableforpt
osisoft heri
ghteyeandconst r
icti
onoftherightpupil.
Whi
choft
hef
oll
owi
ngi
sthemostappr
opr
iat
edi
agnost
icst
udyf
ort
hispat
ient
?

a. Chestcomput edtomography(CT)
.
b. Caroti
dcomput edtomographyangiography(CTA)
.
c. Transthoracicecho(TTE)withami cr
ocavit
ati
onstudy
d. TranscranialDoppler
.
e. Magne ti
cr esonanceimaging(MRI)ofthebrai
n.

Answer
:

12. b. Car
oti
dcomput
edt
omogr
aphyangi
ogr
aphy(
CTA)
.

Thisyoungpat i
entwi thantecedentminornecktrauma, pain, andHorner’
ssyndr ome( ptosi
s,
anhydrosis,andmi osi
s)mostl ikel
yhasacar ot
idart
erydi ssect i
on.Ei
therCTAorMR
angiography(MRA)ar eaccept ablemeanstoevaluatefordi ssecti
on.Acuteischemi cstroke
ei
therfrom vasculardiseaseorembol idoesnotcauseHor ner’ssyndr
ome.Ther efore,aTTE
wit
hmi crocavit
ationisnothelpf ul
.APancoasttumorwoul dbeunusuali nthispatientand
wouldcauseani psil
ateralHorner’
ssyndromeresult
ingi nleft-si
dedsympt oms.Ther efore,
the
chestCTi snotindicated.MRIwoul dnoteval
uatethevascul atur
eandanMRAwi l
lbeneeded.

Quest
ion:

13. An80- year-oldwomanpr esentstotheemer gencyroom (ER)wi thacuteri


ghtarm
weaknessandsl urredspeech.Shehasahi st
oryofpoor l
ycontr
olleddiabetesmel l
it
us,
hypertensi
on, atr
ialfi
br i
ll
ation,
andcor onaryart
erydiseasewit
hpr eviousstentplacement.The
pati
entisaccompani edbyherdaught erwhonot esthathermother ’
ssympt omsst art
ed
approximately90mi nutesago.Shewasseeni ntheER2mont hsear li
erwi t
hat r
ansient
i
schemi cattackatwhi cht i
meherat r
ialfi
bri
ll
ati
onwasdi agnosed.Cur rentmedicati
onsincl
ude
i
nsulin,aspi
rin81mg, li
sinopri
l5mg, warfari
n5mg, andmetoprolol25mgbi d.
 

Onexami nation,shei safebri


le.Herheartrateis88bpm, bl
oodpr essurei s210/100mm Hg,
andr espi
r atoryrateis16br eathspermi nute.Herneur
ologicexami nati
oni sconsist
entwitha
3
l
eftmi ddl
ecer ebralarter
ystroke.Plat
eletcountis239,000mm , i
nt ernat
ionalnor mali
zedrat
io
i
s1. 8,andpar ti
althromboplastinti
mei s28seconds.Aheadcomput edt omogr aphyobtai
ned
wit
hi n30mi nutesoft hepatientarr
ivi
ngi ntheERshowsal arge, acut e,l
eft-si
dedmi ddl
e
cerebralar t
erystroke.Thepat ienthasnohi stor
yofpri
orgastroint esti
nalbleedingor
i
ntracranialhemor rhageandhashadnor ecentsurger
ies.

Whi
choft
hef
oll
owi
ngt
her
api
esi
smostappr
opr
iat
eatt
hist
ime?

a. Clopidogrel
.
b. Full
-doseaspiri
n.
c. Recombi nantti
ssueplasmi
nogenact
ivat
or.
d. Argatroban.
e. Unfracti
onatedhepari
n.
Answer
:

13. b. Ful
l-
doseaspi
ri
n.

Thepat i
entishavi nganacut ei schemi cstroke.Despiteherear lypresentati
on,shehassever al
contr
aindicati
onst olyti
cs,includi ngelevat
edbl oodpr essure,recentstrokewithi
n3mont hs,
andconcur rentuseofwar farin.Cont raindi
cationstolyticsincludesystolicbl
oodpressure>185
mm Hgordi astoli
c>110mm Hg, activeint
ernalbleeding, pl
ateletcount<100,000mm3, heparin
administrat
ionwi t
hin48hour s,cur r
entuseofant i
coagul ants(int
ernati
onalnormali
zedr ati
o
>1.7,prothrombint i
me>15sec) ,currentuseofdi rectthrombini nhi
bit
or sordir
ectf
actorXa
i
nhibi
tor swithelevatedsensitivel aborator
yt ests,
bloodgl ucoseconcent rati
on<50mg/ dl,and
comput edtomogr aphydemonst rati
ngmul til
obarinfarction.
 

Atpr esent,theusef ulnessofar gatrobanorot hert hrombi ninhi


bitor
sfort r
eatmentofpat ients
withacut eischemi cst rokeisnotwel lestablished( ClassI I
b,LevelofEvidence[LOE]B) .These
agent sshoul dbeusedi nthesettingofcl ini
calt ri
als.Urgentant i
coagulati
onwi th
unfract i
onatedhepar i
norl owmol ecul arwei ghthepar i
n,withthegoalofpr eventi
ngear ly
recurrentstroke, halti
ngneur ologicalwor seni ng,orimpr ovingoutcomesaf teracut eischemi c
strokei snotr ecommendedf ortreat mentofpat i
entswi thacuteischemicst r
oke( ClassI II
,LOE
A).Or aladmi nistr
ationofaspi ri
n( initialdose325mg)wi thin24-48hour safterstrokeonseti s
recommendedf ortr
eatmentofmostpat ient
s( Cl
assI ,LOEA) .Theusefulnessofcl opidogr elf
or
thetreat mentofacut eischemi cstrokei snotwel lestabli
shed( Cl
assIIb,LOEC) .Fur t
her
resear chtestingtheusef ul
nessoft heemer gencyadmi nist
rati
onofclopidogrelint hetreatment
ofpat ientswithacut est rokeisrequi red.

Quest
ion:

14. A65- year- ol


dmanpr esentstoar uralur gentcar ecli
nicwi thsympt omsofper i
or al
numbnessandr i
ghtar mt i
ngl i
ngthatst art
edappr oximat el
y2hour sagowhi lehewasdr ivi
ng
tothest ore.Thepat ienthasnotseenaphysi cianinyear s,deni esanymedi calhi st
ory,and
takesnomedi cations.Exami nati
onatt heur gentcar eclinicreveal sanobesemal einnoacut e
distr
ess.Hei saf ebri
le.Pulsei s128bpm andi rregular.Bloodpr essureis155/ 100mm Hg.
Respiratoryrateis18br eathspermi nute.Neur ologicalexami nat i
oni ssigni f
icantforflattened
right
-sidednasol abialfold,lossofsensat ionovert herightforear m, andl ossoft wo- point
discri
mi nati
oni nther i
ghthand.Labor atorydat a, i
ncludingabasi cchemi strypanel ,acompl et
e
bloodcount ,internati
onalnor mali
zedr ati
o( I
NR) ,prothrombi nt i
me, andpar t
ialthrombopl astin
ti
mear eal lwithinnor mallimi t
s.Electr
ocar diogram showsat rialfibril
lat
ionwi t
havent ricular
responseof130bpm.  

Duetoconcer nsforanacutei
schemicstroke,
thepat
ientistransferr
edtotheneareststr
oke
cent
er.Thepat i
entarr
ivesatt
hestrokecenter
’semergencydepar tment1.
5hour sl
ater.An
i
mmedi ateheadcomput edtomographyshowsanacut eischemi cstr
okeinthethalamuswith
nohemor rhage.Hisneurol
ogi
calexaminati
onandvit
alsignsar eunchanged.

Whi
choft
hef
oll
owi
ngi
sthemostappr
opr
iat
ether
apy?

a. St
reptoki
nase.
b. Aspi
rin.
c. Cl
opidogrel
.
d. Enoxopar
in.
e. Recombinantt
issue-
typepl
asmi
nogenact
ivat
or(
rtPA)
.

Answer
:

14. e. Recombi
nantt
issue-
typepl
asmi
nogenact
ivat
or(
rtPA)
.

Thepatientinthi
sscenariopr
esentswithacuteischemicstroke.Hispresentationtoast roke
centeri
swi t
hinawi ndowof3-4.5hoursaftersymptom onset.Hisplat
eletsarenor mal,andhis
bloodpressureis<185/110mm Hg.Accor dingtotheAmericanHear tAssociation/American
StrokeAssociati
on2013guidel
inesforthemanagementofi schemi cstroke,i
ntravenousr tPA
(0.9mg/kg,maxi mum dose90mg)i srecommendedf oradmi nist
rat
iont oeli
giblepati
entswho
canbet reat
edint het
imeperi
odof3- 4.5hoursaft
erstr
okeonset( ClassI ,
LevelofEvidence
B). 

Quest
ion:

15. A65- year-oldmanwi t


hahi storyoft obaccoabuse, hypert
ensi
on,hyper li
pidemia,
obesity,andsl eepapneapr esentst ot heemer gencydepar tmentwithlef
t-si
dedhemi paresis
andf aci
aldr oop.Thesympt omsbegan8hour spri
ortopr esent
ati
onbuthadr esolvedatthe
ti
meofeval uationintheemer gencydepar t
ment .Hiscurrentmedicati
onsincludeaspi r
in81mg
dail
y, at
enolol50mgdai l
y,l
isinopril20mgdai l
y,andatorvastat
in40mgdai ly.Revi ewof
systemsi sremar kableforoccasi onalepi sodesofleft
-sidednumbness.Car diacexami nationis
unremar kableexceptf orbil
ateralcarot i
dbr uit
s. 

Hisel
ectr
ocardi
ogr
am showsnor malsinusr
hythm andi
snormal
.Computedtomographi
c
angi
ographyshowedr
ightinternalcaroti
dst
enosi
sof75%justdi
stalt
othebi
fur
cati
onand
mini
malplaqueint
helef
tinternalcaroti
d.

Whi
choft
hef
oll
owi
ngi
sthemostappr
opr
iat
emanagementoft
hispat
ient
’scar
oti
dst
enosi
s?

a. I
ncreaseaspir
into325mg.
b. Beginaspir
in/
dipyr
idamole.
c. Caroti
dendarter
ectomy(CEA).
d. Beginclopi
dogrel
.
e. Not r
eatmentneeded.

Answer
:

15. c. Car
oti
dendar
ter
ect
omy(
CEA)
.

Thispatienthassi gni
fi
cantcar oti
ddi seaseandi sexperiencingt ransientischemi catt
acks
(TIAs)li
kelyfrom embolizati
onofcar ot
idpl aque.Hewar rantst r
eatmentt opr eventapossibl
e
devastati
ngst rokeinthefuture.Insever alrandomi zedtrialsinvolvingpatientswhohadaTI A
orstrokeassoci atedwit
hi psi
lateralcaroti
dst enosis,CEAr educedt hesubsequentr i
skof
stroke.I
nt heNASCETst udy,patientswi t
hst enosisof70%ormor er andoml yassignedtoCEA
plusmedi caltherapyhada2- yearriskofi psil
ateralst
rokeof9%compar edwi t
h26%i nthe
groupassi gnedt omedicaltherapyal one.Amet a-analysisoft hemaj ort
rialsofCEAshowed
thatthebenef i
tfrom CEAwasgr eatestwheni twasper formedwi thin2weeksoft heonsetof
TIAorst r
oke, rat
herthanlater.Thepat ientisalreadyonaspi rin,andaddi ngaddi ti
onal
ant
ipl
atel
etagent
sisnotr
ecommended.

Quest
ion:

16. I r
respecti
veofthemet hodf
orevaluat
ingcarot
idart
erydisease(
incl
udingduplex
ult
rasonography,computedtomographicangiogr
aphy,magneti
cresonanceangiography,and
cathet
erangiography)
,whichofthefol
lowingfact
orsbestcor
rel
ateswithri
skoffuturestr
oke?

a. Tortuosit
yofdistalart
eries.
b. Statusofthecont r
alat
eralcaroti
dar
ter
y.
c. Plaqueulcerati
on.
d. Percentageofcarotidstenosis.
e. Extentofplaque

Answer
:

16. d. Per
cent
ageofcar
oti
dst
enosi
s.

Themosti mpor tantinf


or mat iongai nedfrom eval uat
ionofthecar ot
idarter
ies,irr
especti
veof
i
magi ngmodal it
y,istheper cent ageofcar oti
dstenosi s.Theprognosisforpatientswithcar
otid
diseaseismostcl oselylinkedt ot hedegreeofst enosis,wit
ha2mm r esidualluminaldiameter
ora60- 70%r educti
oni ndi amet erassociatedwi t
hamar kedincreaseinther i
skofst r
oke.
Plaqueulcerati
onsar ecommon, butdonotst ronglyassociatewithsubsequenti psil
ater
al
i
schemi cstroke.Extensivepl aquemayf avorcarot i
dendar t
erect
omyr atherthancar oti
d
stenti
ng.Occludedcont ralat eralcaroti
dar ter
iesmayf avorcarot
idstenti
ngr at
hert han
endarter
ectomyf ordefiniti
vet reatment.

Quest
ion:

17. A42- year-


oldwomanpr esentstoyouroffi
cewi thcompl
aint
soffati
gue, malai
se,
shortnessofbreat
h,andpr ogressi
velowerextremityedemaoverthepast2mont hs.Shehas
nopr i
orhist
oryofcardiopulmonarydisease.Examinat i
onshowsnormaljugularvenous
pulsati
on,cl
earlungs,2+pitti
ngedemaoft hebi l
aterall
owerext
remit
ies,
wi t
hsever al
cutaneousgrowths,andpigment edlesi
onsoft heskin.

At
ranst
hor
aci
cechocar
diogr
am i
sshowni
nFi
gur
e1.

Whichoft
hef
oll
owingcl
ini
calsyndr
omei
smostl
ikel
ytobeassoci
atedwi
tht
heabnor
mal
it
y
depi
ctedi
nFi
gure1?
a. Car
neycompl ex.
b. VonRe ckl
inghausen’sdisease.
c. Loef
fl
er’sendocar di
ti
s.
d. Noonan’ssyndr ome.
e. Car
cinoidsyndr ome.

Answer
:

17. a. Car
neycompl
ex.

Thecor rectoptionisCar neycompl exwi t


har i
ghtatri
almyxomashown.Thi si
sar are
autosomaldomi nantneopl asticdisor derchar acter
izedbycar diacmyxomas, cutaneous
myxomasorpi gment edlesions, andendocr inetumor s.Carci
noidt umor sar eusuall
ylocatedin
thegast r
ointesti
nalsystem ( 90%)orl esscommonl yi npulmonar yorgeni touri
narylocations.
Cardiacinvolvementusual l
ymani festsasf ibroti
cchangest or i
ghthear tstructur
esr esulti
ngin
valvul
arr egurgit
ati
onorst enosi s.Car diacinvolvementi nLoeffl
er ’
shyper eosinophil
ic
syndromeusual lyisexhibit
edbyendocar di
tisorendomyocar di
t i
s.Themostcommoncar diac
abnormal it
yinNoonan’ ssyndr omei sdysplast i
cpul monaryvalvest enosis.Von
Recklinghausen’ sdisease(neur ofibromat osis)isnotknownt obeassoci at edwithcardiac
tumor sbutmaybeassoci atedwi t
hcongeni talheartdefect
s.
Quest
ion:

18. A60- year-oldmani sadmittedwithat r


ansi
entischemicattack(TIA).Hehasahi stor
y
ofgast roesophagealr efl
uxdisorderandost eoar
thr
it
isbutnosi gni
fi
cantcardiovascularri
sk
factors.Histemper atureis98.3°F,bloodpressurei
s134/ 72mm Hg, heartrateis82bpm, and
oxygensat ur
at i
onis99%onr oom ai r.Hi
slungsareclear
, hear
tsoundsincludeasof tsystol
ic
mur mur ,andt herearenoext raheartsounds.Theskinexami nati
onisunremar kable.Heis
referr
edf orachestwal lechocardi
ogr am,andbecauseofanabnor malit
y,at ransesophageal
echocar diogram isperformed( Fi
gure1) .

Whi
choft
hef
oll
owi
ngi
sthemostl
ikel
yet
iol
ogyoft
heabnor
mal
it
yshowni
nFi
gur
e1?

a. Fi
br oelast
oma.
b. My xoma.
c. Endoc ardi
tis.
d. Bloodcyst s.
e. Thrombus.
Answer
:

18. a. Fi
broel
ast
oma.

Thecor rectopt i
onispapi ll
aryf i
broel
astoma( PFE).Thepr esent ati
ondescribedand
abnor malit
yshownar etypicalofasympt omat i
cPFE.PFEi sthesecondmostcommonpr i
mar y
car diacmass.Appr oximat elythree-f
ourthsofPFEsor iginatef rom thevalves( aort
ic44%, mit
ral
35%, tr
icuspid15%, andpul moni c8%).Whi l
et hemaj orit
yofpat ientsareasympt omat i
c,the
mostcommonpr esent i
ngsympt omsar et r
ansientischemi cat t
ackandst r
oke, f
ollowedby
angi na, myocar di
alinfarcti
on, heartf
ail
ure, suddendeat h, andsyncope.Tr eatmentofchoi cefor
sympt omat i
cPFEi ssurgicalexcision.Cli
nicalfoll
ow- upcoul dbeconsi deredf orasympt omat ic
PFEswhent heyareofl argesi ze(>1cm)andhavehi ghmobi l
ity.Bloodcyst sarecar di
ac
massest hatarecommonl yattachedtoval vularsurfaces.Theyar echaracteri
zedby
mi crobubbl econt r
astent eri
ngt hedivert
icularcavityandof tenbecomehear tshapeddur i
ng
vent ri
cularsyst ol
e.Theyar erarelyseeninadul ts.Surger yisindi catedforbloodcyst swhen
valvedest r
uct i
onispr esent.Thel ackofcl i
nicalfi
ndingsofi nfectiveendocarditi
sandt he
locat i
onont heaor t
icsider atherthanvent r
icularsideoft hel eaf l
etmakeendocar dit
isless
li
kel y.Therei snosuggest i
ont hisisathrombus.

Quest
ion:

19. A64- year


-ol
d-manpr esentstoyouroff
iceforevaluati
onofaheartmass.Hismedi cal
hist
oryincl
udeshyperl
ipidemia,t
obaccoabuse, andchronicobstr
uct
ivepulmonarydisease.
Whileoutoftown2weeksagovi si
ti
ngfamily,
hewasadmi t
tedtoanoutsi
dehospitalf
orfever
s,
cough,anddyspnea.Hesayst hathissymptomsi mprovedt osomeextentaft
err
eceiving
anti
biot
icsforpneumonia.

Anechocardiogr
am i
sperfor
med,andacardiacmassisi
dent
if
iedi
nther
ightat
ri
um t
hatdoes
notappeartobeeit
herveget
ati
onorthrombus.
Whichofthefoll
owi
ngisthemostli
kelycar
diactumor
?

a. Li
poma.
b. Papil
laryf
ibr
oelastoma.
c. Metastati
cdisease.
d. Sarcoma.
e. Myxoma.

Answer
:

19. c. Met
ast
ati
cdi
sease.

Thecor rectopt i
onismet astati
cdisease.Excludingthr
ombiandveget ati
ons,mal i
gnant
met astasesar ethemostcommonl yfoundcar diacmasses,occurr
ing30-1000timesmor e
frequentlythanpr i
marycar diacmasses.Forexampl e,
inaNor thAmericanstudyincludi
ng
>11, 000aut opsiesperf
ormedbet ween1973and2004, 266cardi
acmasseswer efound.Of
these,264wer eduetomal ignantmet astasi
s.Commonmal i
gnanciestometastasizetothe
hear ti
ncludelung, br
east,gastroi
ntesti
nal,mel anoma,andleukemia/l
ymphoma.Among
primar
ycardi
acmasses,myxomasarethemostcommon,fol
lowedbyli
pomasandpapil
lar
y
fi
broel
ast
omas.Mal
ignantpri
marycar
diacmassesar
eexceedingl
yrar
e;however
,among
these,
sar
comasarethemostcommonlyoccurr
ingmasses.

Quest
ion:

20. A42- year-ol


dmancomest oyouroff
iceforeval
uati
onofanabnormallipi
dpanel
obtainedintheprocessofappl
yingforl
if
einsurance.Hedoesnothaveanysympt omsofchest
painordyspnea.Hewr estl
edincoll
ege,butdoesnotexerci
seregul
arl
yatpresent.Hisbl
ood
pressureis142/86mm Hg, hei
ghtis70inches,weighti
s209pounds,andwai stci
rcumfer
ence
i
s42i nches.
 

Thefasti
ngli
pidpanelshowedtotalchol
ester
ol201mg/dl
,tr
igl
yceri
des196mg/ dl
,l
ow-densi
ty
l
ipoprot
einchol
ester
ol(LDL-C)131mg/ dl
,andhigh-
densi
tyl
ipoprot
einchol
est
erol(
HDL-C)34
mg/dl.

Whichofthefol
lowingdiet
arychangeswoul
dyour ecommendforr
educi
nghi
sri
skf
orf
utur
e
car
diovascul
arevents,
inaddit
iontoaregul
arexer
cisepr
ogram?

a. Di
etwi
thsimil
arcomposi
ti
ontopresent
,butwi
thlowertotalcal
ori
es.
b. Di
etwi
thfewersi
mplecar
bohydr
atesandlowertotalcal
ori
es;addmorepr
otei
nfr
om
f
ish.
c. St
ri
ctno-
carbohydrat
ediet,butsimil
arintot
alcal
ori
es;addmoreprot
einfr
om fi
sh.
d. St
ri
ctno-
carbohydrat
edietlowerintotalcal
ori
es,
andaddagl assofr
edwinedaily.
e. St
ri
ctl
ow-fatdi
etloweri
nt otalcal
ori
es,andaddaglassofredwine3timesperweek.

Answer
:

20. b. Di
etwithfewersi
mpl
ecar
bohydr
atesandl
owert
otalcal
ori
es;addmor
e
pr
otei
nfrom fi
sh.

Thispatienthast hemet aboli


csyndrome,def i
nedast hreeormor eofthef oll
owing:blood
pressure≥130/ 85mm Hg, fast
ingbl
oodgl ucose≥100mg/ dl
,waistci
rcumf erence>40inches
i
nmenor35i nchesinwomen, HDL-C<40mg/ dli nmenor<50mg/ dlinwomen, and
tri
glycer
ides>150mg/ dl.Causesaremul t
ifactorial,butincl udedailycalori
ci nt
akeconsi stent
ly
exceedingcal ori
esburnedbyexer ci
se,sedentar yl if
estyles, i
nsul
inresistance,andgenet ic
makeup.I nit
ialtreat
mentr evol
vesaroundexer ciseanddi etmodi fi
cat
ion.Low- glycemici ndex
foodsaret houghtt oavoidthenascenti
nsul i
nr esi stance, andlowertotalcal or
icint
akeis
crucial
.Omega- 3fatt
yacidsmayal sobebenef i
ci al;thus, fewersimplecar bohydrat
es,lower
totalcal
oricintake,andmor efishi
sthebestanswerchoi ce. 

Low-orno- carbohydratediets(suchasAt ki
nsorSout hBeach)areunproven,butsimil
artot
al
cal
ori
cint
akei sunli
kelytosubst ant
ial
lyalt
ertheli
pidprof
il
e.Alt
houghmoder at
ealcoholint
ake
canrai
selevelsofHDL- C,pref
erenti
almetabolism ofet
hanoli
ntheliverwil
lresulti
nhigher
l
evel
softriglyceri
desandwi l
lnotbehel pf
ulinthi
ssyndrome.

Quest
ion:

21. A57- year-


oldwomanpresentsforaf
oll
ow-upoff
icevi
sit
.Shehasahistoryof
hyper
tensi
on,hyperl
ipi
demi
a,di
abetes,
andtobaccouseandunderwentper
cutaneous
coronar
yint
ervent
iontoherl
eftci
rcumfl
excoronar
yart
ery2year
spr
ioraf
terpr
esent
ingwi
tha
non–ST-segmentelevat
ionmyocardi
ali
nfar
cti
on(NSTEMI)

Today,shedenieschestpainandshortnessofbreath,
butrepor
tshavingtroublesl
eepi
ng,
decreasedappeti
te,l
ossoffocus,andlossofi
nteresti
nherhobbies.Shehasbeencompl i
ant
withhermedicalther
apy,whichincl
udesaspir
in81mgdai l
y,metoprol
olsuccinat
e50mgdaily,
rosuvastat
in20mgdai l
y,andvarenicl
ine1mgt wicedail
y. 

Bloodpr
essureis132/72mm Hg, pulsei
s72bpm,andrespi
rat
oryr
atei
s12br
eat
hs/
minut
e.
Theremainderofherphysi
calexaminati
oni
swit
hinnor
mallimit
s.

Whi
choft
hef
oll
owi
ngi
sthebestnextst
epi
nmanagementoft
hispat
ient
?

a. Di
sconti
nuemet oprolol
.
b. Di
sconti
nuevarenicli
ne.
c. St
artesci
tal
opram.
d. Di
sconti
nuerosuvastati
n.
e. Ref
erthepati
enttoapsychiat
ri
st

Answer
:

21. b. Di
scont
inuevar
eni
cli
ne.

Thepatienti
saf ormersmokerwi t
hmulti
pleri
skfact
orsf
orcor onaryarter
ydi seasewhohad
anNSTEMI2year spri
or.Sheisbeingtr
eatedwit
hvareni
cli
ne,apar ti
alagonistofthenicoti
nic
acet
ylcholi
nereceptor
,whichbothreducesthesymptomsofnicotinewithdrawalandr educes
theeff
ectofcigaret
tesmoking.Thepati
entexhi
bit
stypi
calsideeffect
sofvar enicl
ine,
andt hi
s
medicati
onshoul dbediscont
inued. 

Thereisnoi ndicati
onforbegi
nninganant i
depressantmedi cati
onorr eferr
ingt opsychi
at r
y
unlessthesesympt omsper si
stedafterdiscont
inuationofthevar enicli
ne.Bet a-blocker
sar e
l
essl i
kelytobet hecul
pr i
tandareindicatedinthispost -
MIpat i
ent .Ther ehasbeenno
consist
ent,rel
iableevi
dencetosuggestt hatstatintherapyi
sr el
at edtot heset ypesofcentral
nervoussystem sideeffect
s.Giventhatthepatienthasknowncor onar yarter
ydi sease,she
meet saClassIr ecommendationforlipi
d- l
oweringtherapywi t
hast ati
n.

Quest
ion:

22. A75- year-


oldwomanpr esentswithabdomi nalpai
nandshortnessofbr
eath.Physi
cal
examinati
oni sunremar
kable.Echocardi
ogram showsani l
l-def
inedmassintheri
ght
vent
ri
cularapex.Sheundergoescont r
ast-enhancedcomput edtomographyoft
hechestand
abdomen, showingtheabnormalit
ydepictedinFigure1.

Whi
choft
hef
oll
owi
ngi
sthemostl
ikel
ydi
agnosi
s?
a. Intr
acar
diacthrombus.
b. Tumorme t
astasi
s.
c. Lipomatoushypertr
ophyoft
heseptum.
d. Prominentri
ghtventri
cul
art
rabecul
ati
ons.

Answer
:

22. b. Tumormet
ast
asi
s.

Thisimageshowsal obulatedmassofmi xedatt


enuat ionintheapexoft heri
ghtventr
icle
(Fi
gure2) .Thi
si sanunusuallocati
onf ort hr
ombus.Thel argesizeandhet erogeneousdensity
ismor econsistentwit
htumor .Li
pomat oushyper t
rophyaf f
ectstheinteratr
ialsept
um, notthe
ventr
icularseptum,andwoul dbeoff atdensi t
y,nott hedensityofthelesionshowni nthis
image.Pr ominenttr
abeculati
onstypicallyhaveat ubularandmor eli
nearappear anceandar e
ofunifor
m densi t
y.Thismassr epr
esent edamet astasisfr
om ahepat oma.
Quest
ion:

23. A73- year


-oldmanpr esentstocli
nicwithnonspeci
fi
csympt omsofmalai
se,weight
l
oss,andlowbackpain.Inspiteofthi
s,heledanactiveandunrestr
ict
edli
fe.Hi
smedi cal
hist
oryi
ncludesaremotehistoryofaort
ocoronarybypassandresecti
onofalef
trenalcell
carci
noma.Hewasadmi ttedtothehospitalaf
teranabnormalf
indingonabdominalultr
asound.

Anechocar
diogram wasor
der
ed.Figur
e1i sasti
llf
rameoft
heapi
calf
our
-chambervi
ew.
Fi
gure2isastil
lfr
ameimageoftheinf
eri
orvenacava(I
VC).

Whi
choft
hef
oll
owi
ngi
sthemostl
ikel
yet
iol
ogyoft
hiscar
diacmass?

Fi
gur
e1
Fi
gur
e2

a. At
rialmyxoma.
b. Hepatocell
ularcar
cinoma.
c. Recurrentr
enalcellcar
cinoma.
d. Deepvenoust hrombosisofthel
owerext
remi
ty.

Answer
:

23. c. Recur
rentr
enalcel
lcar
cinoma.

Themostcommont umortoi nvadet heheartviat heIVCi sr enalcel


lcarci
noma.Intr
aluminal
growthoccur sviatherenalvei nsandI VC,of
teni ntother ightatri
um.Lesscommonl y,
theIVC
maybei nvadedbyhepat ocellularcarci
noma, uterineleiomyoma, andcancersoftheadrenal
gland,
includingpheochromocyt oma.Alt
houghat ri
almyxomascanar isefr
om theri
ghtatri
um,
theyareusual l
ylocatedont hef ossaovali
swi thnoext ensionf r
om theIVCandareof t
en
pedunculated.Deepvenoust hrombosi swouldbel essl i
kelyinanact i
vepersonwithno
complaintsoflegpai norswel li
ng.
Quest
ion:

24. A50- year


-ol
dmancomest oyourclini
cforanewpat i
entvi
sit
.Hehasahi st
oryof
hypert
ensionandhyperl
ipi
demi a,
forwhichheisonaml odi
pineandrosuvastati
n.Hehasbeen
smokingonepackperdayf orthelast35years.Hedoesnotdrinkanyalcohol.Hi
sexam is
unremarkabl
eandar ecentl
ipi
dpanelshowedal ow-
densit
yl i
popr
otei
n( LDL)of84mg/ dl
.Hi
s
wif
ehasbeenhoundi nghimt oquitsmoking,buthef
eels“t
hedamaget hathasbeendonei s
done.”

Whi
choft
hef
oll
owi
ngdoyout
ellhi
m,r
egar
dingt
heef
fectofsmoki
ngcessat
ion?

a. I
twil
levent
uall
yreducehisriskoflungcancertot hatofanonsmoker .
b. Ther
eisminimalcar
diovascularbenefi
tofcessationforlong-standingheavysmokers.
c. Swi
tchi
ngtochewingtobaccowi llr
educehiscar diovascularr
isktot hatofanonsmoker
.
d. I
twil
levent
uall
yreducehisriskofcoronarydiseaset othatofanonsmoker .
e. Swi
tchi
ngtoelect
roni
ccigaretteswil
lreducehiscar diovascul
arriskt othatofa
nonsmoker
.

Answer
:

24. d. I
twi
llevent
ual
lyr
educehi
sri
skofcor
onar
ydi
seaset
othatofanonsmoker
.

Thebenefi
tsofsmokingcessati
onaresignif
icanti
nallsmokers,regardlessoflengthof
smokinghist
ory.Smokingcessat
ionleadstodecreasedmor t
ali
ty,decreasedcardiovascul
ar
event
s,anddecreasedinci
denceofcancer.Alt
houghtheexacttimingvar ies,
studi
essuggest
thatt
hecoronaryheartdi
seaseri
skreturnstobasel
ine(nonsmokerl evel)wit
hin15yearsof
smokingcessati
on. 

Althoughthecancerri
skalsosubst
antial
lydecreases,
itwil
lalwaysremainel
evatedcompared
tononsmokers.TheAmer i
canHear tAssoci
ation(AHA)advisesthatsmokel
esstobacco
productsareharmfulandshouldbediscouragedbycli
nici
ans.Ther i
skofel
ectr
oniccigar
ett
es
i
suncl ear
,buta2014AHApol icyst
atementdi scour
agestheuseunt i
lmoredataareavail
abl
e.
Syst
emi
cHyper
tensi
onandHypot
ensi
on
Quest
ion:

1. A55- year-oldwomanwi t
ha40- yearhist
oryofi
nsuli
n-dependentdi abet
esmel li
tus
presentstoyourof f
iceforcareofherhypert
ension(HTN).Herserum creati
ninehasbeen
graduall
ywor seni
ngovert hepast10year sfr
om 1.3to4.2mg/ dl
.Herbl oodpressur
e( BP)has
beenincreasi
ngovert hepastyearfr
om 138/84mm Hgt o152/ 90mm Hg.Shehast r
ied
l
if
estylemodificat
ion,buthersystol
icBPhasr emained>150mm Hg.Shedeni essnoring,
dayti
mesomnol ence,cigaret
tesmoking,fr
equentheadaches,ordiaphoreti
cspell
s. 

Onphysi
calexamination,herbodymassindexis24andherBPi s154/92mm Hginbotharms.
Herl
ungsareclear.Cardiacexaminat
ionisnot
ableforanS4.Therearenoabdomi
nalbrui
ts.
Ther
eistr
acepr et
ibi
aledema.Ther ei
snor adi
al-
femoralpul
sedelay.Herpot
assi
um i
s5.2
mmol/L,
andhercr eati
nineis4.2mg/dl.

Whi
choft
hef
oll
owi
ngi
sthemostl
ikel
ycausef
orherHTN?

a. Chronickidneydisease(CKD)
.
b. Coarctat
ionoftheaor t
a.
c. Pheochromocyt oma.
d. Obstruct
ivesleepapnea.
e. Pri
mar yhyperaldoster
onism.

Answer
:

1. a. Chr
oni
cki
dneydi
sease(
CKD)
.

CKDisbothacauseandaconsequenceofHTN.Mor ethan80%ofpat ientswit


hCKDhaveHTN.
Iti
sthoughtthatthishighrat
eofHTNi npat
ient
swithCKDi smulti
fact
orial
,buti
nlargeparti
s
duetosodium retenti
on.Shedoesnothavesignsorsymptomssuggest i
ngobstruct
ivesl
eep
apnea,aor
ticcoarctati
on,orpheochr
omocytoma.Herpotassi
um of5.2mmol /Lmakes
hyperal
doster
onism lessli
kely.

Quest
ion:

2. A50-year-ol
dmanpr esentstoyouroffi
cesayinghewant stoseeacardiol
ogisttocar
e
forhishypert
ension(HTN) .Hisbloodpressure(BP)wasnot edtobeelevat
edlastyear,andhi
s
i
nternistr
ecommendedl ifestylemodifi
cati
onandl i
sinopr
il
,whichhasbeent i
tr
atedto20
mg/ day.Wit
ht hesemedicat i
ons,hisBPathomehasbeenaver aging132/80mm Hg.He
deniessnori
ng, dayt
imesomnol ence,f
requentheadaches,ordi
aphoreti
cspell
s.Bothofhis
parentsdevelopedHTNint hei
r50s. 

Onphysi calexaminati
on,
hisBPi s136/ 86mm Hgi nbot harms.Hislungsandcardiac
exami nat
ionar eunr
emarkable.Hisabdomeni ssoftwithoutbrui
ts.Therei
snoradial-
femor
al
delay.Labsshownor malelectrol
ytes.Anelect
rocardi
ogram showssi nusr
hyt
hm and
i
ncr easedvoltagewit
houtsecondar yrepol
ari
zati
onchanges.
Whi
choft
hef
oll
owi
ngdoyour
ecommend?

a. Nofurthereval
uationatt
hecurrenttime.
b. Renalmagnet i
cresonanceangiography.
c. Comput edtomographyofthethoracicaor
ta.
d. Pl
asmaal dost
eroneandreni
nlevels.
e. Twenty-fourhoururi
neforfr
acti
onatedmet anephr
inesandcat
echol
ami
nes.

Answer
:

2. a. Nof
urt
hereval
uat
ionatt
hecur
rentt
ime.

Whi l
epr i
mar yhyperaldoster
onism, renalarter
yst enosis,andpheochr omocytomaarethemost
commoncausesofsecondar yHTN, theseoccuri n<10%ofadul tswithHTN,andthevast
maj ori
tyofadultpatientshavepr imar yHTN.Test i
ngf orsecondar ycausesofHTNistypi
cal
ly
reservedforthosepat ientswit
hr esist antHTN, new-onsetHTNatayoungage, orsymptoms,
signs,orlabssuggest ingaspeci f
iccauseofsecondar yHTN, suchasheadachesand
diaphoreticspel
ls,abdomi nalbruits, orserum hypokalemi a.This50-year-
oldpati
entwit
hwell
-
cont r
oll
edBPonasi ngleanti
hyper tensi vemedicineandanunr emar kabl
eexaminati
onand
normall absdonotwar r
antworkupf orsecondar ycausesofHTN.  

Quest
ion:

3. A48- year-ol
dwomanpr esentsforevaluationofherwor seninghyper
tension(HTN) .She
hasbeent r
eatedwi thaml odi pi
ne5mgqdandhydr ochlorothi
azide25mgqdandr eport
st hat
hersyst
olicbloodpr essure( BP)pr evi
ouslyaveragedi nt he130smm Hgbuti snowaver aging
i
nt he150smm Hg.Shest atesshehadbeenpr escribedr amipri
l,butt
hiswasdiscontinued
becausehercr eat
ininehadi ncreasedfrom 1.4to2. 8mg/ dlwhent aki
ngramipri
l.Shedeni es
snori
ng,daytimesomnol ence, f
r equentheadaches, ordiaphor et
icspell
s.Shedeniescigarette
smoking,hyperli
pidemia, diabetes,orfamilyhistoryofHTN.  

Onphysi
calexami
nati
on,herBPi s162/74mm Hgi nbotharms.Herl
ungsareclear.Car
diac
exami
nat
ionisnot
ableforanS4 sound.Shehasbi
later
almidabdominalsyst
oli
cbruit
s.Ther
e
i
snoradi
al-f
emoralpul
sedelay. 

Herser
um pot
assi
um i
s4.
6mmol
/L,
andherser
um cr
eat
ini
nei
s1.
4mg/
dl.

Whi
choft
hef
oll
owi
ngdoyour
ecommend?

a. Computedtomogr aphyangiographyofherrenalart
eri
es.
b. Tri
alofangiot
ensin-r
eceptorbl
ockade.
c. Twenty-f
ourhourur i
neformetanephri
nesandcat echol
ami nes.
d. Computedtomogr aphyangiographyofthethoraci
caorta.
e. Pl
asmaal doster
oneandr eninconcent
rati
ons.

Answer
:
3. a. Comput
edt
omogr
aphyangi
ogr
aphyofherr
enalar
ter
ies.

Thispati
ent’spresent at
ioni ssuggest i
veofr enalarter
yst enosis( RAS),withworseningofher
previ
ouslycontrolledHTN, wor senedrenalfunctionaftert r
eatmentwi thanangi otensin-
convert
ingenzymei nhibit
or, andanabdomi nalbr ui
t.Whi lemostcasesofRASar eduet o
ather
osclerosi
s, approximat ely10%ar eduet ofibromuscul ardysplasi
a( FMD).FMDi smore
commoni nfemal es,about90%off emalesinsomeser ies.FMDt endstooccuri nyounger
women, alt
hought hemeanagei ntheFMDr egistrywas52year sold.Ther adi
ogr aphic
appearanceoft heRAScanhel pdisti
nguishbetweenat heroscleroti
candFMDst enosis,wi
th
ather
oscleroti
cdi seaseusual lyinvolvi
ngt heostialorpr oximalrenalarterywhil
eFMDusual ly
i
nvolvesthemi ddl eordistalsegment .

Quest
ion:

4. A70- year-ol
ddiabeti
c,hyperl
ipidemicfemalesmokerisrefer
redfort
reatmentofher
hyper
tension(HTN) .Hercurr
entmedi cati
onsincludechlor
thal
idone25mgqd, amlodipi
ne10
mgqd, candesar t
an32mgqd, andterazosin2mgqd.Shedeni essnori
ng,dayti
me
somnolence,frequentheadaches,ordiaphoret
icspell
s. 

Onphysicalexaminati
on,herbodymassi ndexis24,bloodpressureinbotharmsis190/98
mm Hg, andBPi nherlegsis194/
104mm Hg.Herl ungsar ecl
ear.Cardiacexami
nati
onis
notabl
eforanS4 andagr ade1/6systol
iceject
ionmurmur .Shehasar ightmidabdominalbr
uit
.
Therei
snor adial
-femoralpul
sedelay. 

Herpot
assi
um i
s4.
2mmol
/L,
andhercr
eat
ini
nei
s1.
1mg/
dl.

Whi
choft
hef
oll
owi
ngdoyour
ecommend?

a. Pl
asmaal dosteroneandr eninconcentrations.
b. Angi
otensin-convert
ingenzymei nhi
bit
ordi agnost
icchall
enge.
c. Magneti
cr esonanceangiogr aphyofthet horaci
caorta.
d. Uri
nar
ysodi um andpot assium level
s.
e. Computedt omographyangi ographyoft herenalart
eries.

Answer
:

4. e. Comput
edt
omogr
aphyangi
ogr
aphyoft
her
enalar
ter
ies.

Thispatientpresentswi t
hr efr
actoryhypertension.Sheishyper tensive,despit
egooddosesof
fourantihypert
ensivemedi cations.Thisshouldl eadtowor kupf orsecondar ycausesof
hypertension.Shedoesnothavedayt imesomnol enceorhistor yofsnor ingtosuggestsleep
apnea.Ther eisnodr opinbl oodpr essurebetweent hearmsandl egs, makingcoarctat
ionless
l
ikely.Thispatienthasmul tipl
er i
skfactorsforatheroscl
erot i
cr enalarterystenosi(
RAS) ,
i
ncludinghersmoki ng,diabetes,andhyper l
ipi
demi a.Asyst oli
cabdomi nallater
ali
zingbruiti
s
specifi
cbutnotsensi ti
vef orRAS, andwar rantsadditi
onaleval uation.

Quest
ion:
5. A55-year
-ol dmanisr
eferredfortreatmentofhi shyper
tensi
on( HTN) .Despi
teramipr
il
10mgqd, amlodi
pi ne10mgqd, andhydr ochlorot
hiazide25mgqd, hisbl oodpressure(BP)
hasbeen160/100mm Hg.Hi swi f
er epor
tsloudsnor i
ng.Hedeniescigarettesmokingandno
di
abetesorhyperl
ipidemi
a.Hedeni esanyot hersystemicsymptoms, suchasdi aphoresi
s,
pal
pit
ati
ons,orheadache. 

Onphysi
calexaminati
on,heweighs210poundsandhasabodymassi ndexof27.Hi
sBPis
162/
98mm Hgi nbothar
ms.Lungsandcardiovascul
arexami
nat
ionar
eunr emar
kabl
e.Ther
e
i
snoabdomi nalbr
uitorr
adial
-femor
alpul
sedelay. 

Labsr
evealnor
malel
ect
rol
ytesandr
enalf
unct
ion.

Whi
choft
hef
oll
owi
ngi
stheappr
opr
iat
enextst
epi
nhi
smanagement
?

a. Twe nt
y-fourhourur i
nef ormet anephri
nesandcat
echol
ami
nes.
b. Serum aldosteroneandr eninlevels.
c. Nof ur
therdiagnostictestsneeded.
d. Pol
ysomnogr aphy.
e. Abdomi nalcomput edangi ogr
aphy.

Answer
:

5. d. Pol
ysomnogr
aphy.

Thispati
entmostl i
kelyhasobst r
uctivesleepapnea( OSA)basedonhi shistory.Treatmentof
sleepapneaincludesweightlossandcont i
nuousposi ti
veairwaypr essure(CPAP) ,whichhelps
preventupperairwaycoll
apsedur i
ngexpi rat
ion.CPAPhasbeenshownt olowerBPi npat i
ents
withOSA.Thispat i
entdoesnothaveot herri
skf actor
sforatherosclerosisorabdomi nalbruit
s
tosuggestrenalarter
ystenosis.Heisl essli
kelytohavehyper aldosteronism or
pheochromocyt omawi t
hnormalel ectr
olytesandnopar oxysmalsyst emicsympt oms, suchas
headaches,diaphoret
icspel
ls,orcyclicuncontrolledHTN.

Quest
ion:

6. A42-year-
oldmanwithahistoryofhypert
ensionisseeninanur gentcar
eclini
cfor
blur
ri
ngofvi si
on.Hehasbeenonl i
sinopri
l40mgaday.Hehadr unoutofhismedicationf
or
thepast2weeks.Hi sbl
oodpressur
e( BP)is196/132mm Hg, heartr
ateis78bpm, and
physicalexaminat
ioni
sother
wisenor malexceptforanS4 
gall
oponcar diacauscul
tati
on.

Whi
choft
hef
oll
owi
ngi
sthenextmostappr
opr
iat
est
epi
nthemanagementoft
hispat
ient
?

a. Hospi
taladmi
ssi
onforrenalarter
ydenervation.
b. Hospi
taladmi
ssi
onforintr
avenousl abet
alol
.
c. Di
schargehomeincr
easingli
sinoprilt
o80mgdai l
y.
d. Di
schargehomeonpriormedications.
e. Hospi
taladmi
ssi
onandr esumepr iormedicati
ons.

Answer
:
6. b. Hospi
taladmi
ssi
onf
ori
ntr
avenousl
abet
alol
.

Hypertensiveurgencyi sthesit
uati
oninwhichapatient'
sdiast
oli
cbl oodpr essure(BP)i
s>120
mm Hg.I fthereisacuteorr api
dlyworseni
ngtarget
-organdamage, t
henthet er
m usedis
hypertensi
veemer gency.Inthi
scase,hisbl
urr
edvisionisconcerningforretinaledemaor
i
njury.Hypertensiveurgencycanbemanagedasanout pat
ient
,buthyper t
ensi veemergency
requir
esadmi ssiont oauni twi
thcardi
ovascul
armoni t
ori
ngfacil
iti
esforpar enteral
anti
hyper t
ensivetherapy.

Quest
ion:

7. A54- year-
oldwomanwi thahistoryofhyper t
ension(HTN)isseeni nyourof f
icef
ora
routinevisit
.Sheof f
ersnocompl ai
ns.Herphysi calexaminati
onisnor malexceptforablood
pressur e(BP)of172/ 102mm Hg, heartrat
eof54bpm, anS4 gal
loponcar diacexami nat
ion,
andmi l
dabdomi nalobesi t
y.Hercur r
entmedi cati
onsincludeextendedr el
easedil
tiazem 300
mgdai l
y,hydrochlorothi
azide50mgdai ly,
andl i
sinopri
l40mgdai ly.Shereportsthatsheis
compl iantwithhermedi cationsandnot esthatherBPsathomear ever ysimil
artothese
currentvalues.

Whi
choft
hef
oll
owi
ngi
sthenextst
epi
nhermanagement
?

a. Scheduleforr
enalar terydenervat
ion.
b. I
ncreasingdil
ti
azem t o360mgdai ly.
c. Addit
ionofatenolol50mgdai l
y.
d. Addit
ionofspironolactone25mgdai ly.
e. A1,500calori
er est
ricteddiet
.

Answer
:

7. d. Addi
ti
onofspi
ronol
act
one25mgdai
ly.

Near l
yallcaseslabeledr esi
stantHTNcanhavet hei
rBPnor mali
zedwithaggressi
vemedi cal
therapy,wit
hatl eastthreeantihyper
tensi
veagentsuptomaximallyapprovedortol
erated
doses, andinsomecaseswi t
hf ourorfi
vedr
ugsofdiff
erentcl
asses,i
ncludi
nganal doster
one
antagonistifappropri
ate.

Quest
ion:

8. A43-year-ol
dmanwi t
houtpri
ormedicalhi
storyisseeninyourof f
ice3daysaf ter
recordi
ngabloodpressure( BP)of160/110mm Hgwi t
hanaut omatedmachi neinhi
s
pharmacywhileshoppingwi thhiswif
e.Heisasympt omatic,
takesnomedi cati
ons,r
arelyuses
alcohol
,andusesnoi l
li
citdrugs.Hismotherandfatherbothhadhypertension,andhisf at
her
died2yearsagoofast roke.Hisphysi
calexaminati
oni snormalexceptforaBPof166/ 112
mm Hg.Ar epeatBPatt heendoft heexaminat
ionis164/114mm Hg.

Whi
choft
hef
oll
owi
ngi
sthemostappr
opr
iat
emedi
calr
egi
menf
ort
hispat
ient
?
a. At
enol olandhydrochlorothiazide.
b. Li
sinopr i
l.
c. Li
sinopr i
landhydrochlorothiazide.
d. At
enol ol.
e. Li
festylemodif
icati
onwi thdai l
yexer ci
se.

Answer
:

8. c. Li
sinopr
ilandhydr
ochl
orot
hiazi
de.

ForpersonswithBP>20/10mm Hgabovegoal ,i
niti
alt
reatmentshouldbewi thtwodrugs
chosenfrom 1)areni
n-angi
otensin-al
dost eronesystem i
nhibit
or(angiot
ensin-conver
ti
ng
enzymei nhi
bit
ororangi
otensi
n- r
ecept orblocker
),2)adihydropyr
idinecalci
um channel
blocker
,and3)athiazi
deorthiazi
de- l
ikediureti
c.

Quest
ion:

9. A36-year -oldwoman, 35weekspr egnant,i


sseenintheemergencydepar tmentwit
h
shor t
nessofbr eath.Herbloodpressure( BP)hasbeenlabil
eduringherpregnancy, andsheis
mai ntai
nedonhydr alazi
ne50mg3t i
mesdai ly.HerBPis180/110mm Hg, heartrateis110
bpm andr egular,andl ungswithbibasil
arrales.Hercar
diacexaminati
onisregular,wit
haflow
mur murandl owerext r
emiti
eswith2+edema.Shei sgi
venint
ravenouslabet
aloland
furosemidewi t
ht her esul
ti
ngBPof140/ 95mm Hgandhear trat
eof95bpm.Aur i
nalysi
s
reveals3+proteinuria.

Whi
choft
hef
oll
owi
ngi
sthenextmostappr
opr
iat
est
epi
nhermanagement
?

a. Oralli
sinopri
l.
b. I
ntravenousdi goxin.
c. Renalultraf
il
trat
ion.
d. I
nductionofl abor.
e. I
ntravenousmagnesi um sul
fat
e.

Answer
:

9. d. I
nduct
ionofl
abor
.

Urgentdelivery/i
nduct ionisrecommendedi nwomenwi t
hgestati
onalhyper tension(HTN)and
protei
nuriaint hesettingofot heradversecondit
ionssuchaspul monar yedema, coagulopathy,
orfetaldi
stress.Pr eeclampsi aisdefi
nedasbot hHTN( 140/90mm Hg)andpr oteinur
iaorin
faceofnewHTNwi thoutpr otei
nuri
a:platel
etcount<100,
000,serum creatini
ne>1. 1,pul
monar y
edema, li
verfunctiont estsmor ethantwot i
mesnor mal,andcerebralorvisualsympt oms.This
usuallyoccursi nthet hir
dt ri
mesterofpregnancy.Eclampsi
aisdi f
ferenti
atedfrom
preeclampsiabyt headdi t
ionofseizures.
Quest
ion:

10. A34- year-ol


dwomanwi thhist
oryofhypert
ension(HTN)andobesitypresent stoyour
off
iceforevaluat
iondur ingpregnancy.ShehashadHTNf or9years,
andwor kupsf or
secondarycausesi nt hepasthavebeennegat i
ve.Shewast r
eatedwithat
enololi nthepastbut
waschangedt olabetaloli
npreparati
onforpregnancy.Sheisnow22weekspr egnantand
doingwell.Shebr i
ngsherhomebl oodpressure(BP)log,whi
chshowsBPr angi ngfrom 135-
160systolicover65- 88diast
olic.Shehasnosympt oms.Medicati
onsincl
udepr enatal
vi
taminsandl abetalol200mgbi d.
 

Onexamination,sheiswel
lappeari
ng.Herhear
tr at
ei s90bpm, BPis155/88mm Hg, and
oxygensat
urationis98%.Herjugul
arvenouspressurei snormal
,andlungsarecl
ear
.Heart
hasregul
arrhythm andnomur murs.Abdomenisgr avid,andext
remit
ieswit
hnoedema.
Hematocri
tis34%, andcr
eati
nineis0.8mg/dl
.

Whi
choft
hef
oll
owi
ngi
stheopt
imalBPi
nthi
spr
egnantwoman?

a. 120/
80mm Hg.
b. 160/
100mm Hg.
c. 130/
85mm Hg.
d. 145/
95mm Hg.
e. 140/
90mm Hg.

Answer
:

10. e. 140/
90mm Hg.

Thispati
enthadHTNpr iortopr
egnancy,andsoherpregnancy-rel
atedHTNi stermedchr onic
HTN.Inwomenwi t
hchr oni
cHTN, t
hetreatmentgoal
sremainthesameaspr i
ortopr egnancy.
Gestat
ionalHTNiselevatedBPthatoccursinwomenafterthe20thweekofgest ati
on.Inthese
women, AmericanColl
egeofObstetr
ici
ansandGynecologist
sguideli
nesrecommendno
tr
eatmentunti
lBPis>160/ 110mm Hg.

Quest
ion:

11. A65- year-oldpat ientpresentst oyourcar di ologyclini


cf oraroutinef ol
low- upvi sit
.He
hasahi storyofcor onar yar ter
ydiseasewi thadr ug- eluti
ngst enttohisleftanteriordescendi ng
3year sago, hyperli
pidemi a,andborder li
nehyper tensi on(HTN) ,whichhehascont rolledwi t h
dietf
ort hepastf ewyear s.Her eport
st hathehasr ecent l
ybeendi agnosedwi thmet astatic
renalcellcarcinomaandi saboutt ost arttherapywi t
hbevaci zumab.Hedeni esanyr ecent
cardiovascularsympt omsorchangesi nexercisetol erance.Hei scurrentlytakingaspi ri
n81
mgdai l
y,cl
opidogr el75mgdai ly,
andsi mvast at
in40mgdai l
y.Onexami nation,hishear trate
i
s58bpm andbl oodpr essur eis138/ 88mm Hg.Hi scar diovascularexami nat i
oni sot herwi se
unremar kable.Youdi scusst hecardiovascul arri
sksoft herapywi thvascul arendot helial
growt hfact
or( VEGF)i nhibi t
orswithhi m.

I
naddi
ti
ont
othr
omboembol
ism,
whi
choft
hef
oll
owi
ngot
hercar
diovascul
arcompl
icat
ionsof
t
hebevaci
zumabt
her
apyi
smostl
ikel
ytobeseeni
nthi
spat
ient
?

a. Lef
tventri
culardysf
unct
ion.
b. Coronaryvasospasm.
c. HTN.
d. QTpr ol
ongation.
e. Atr
ialfi
bri
ll
ati
on.

Answer
:

11. c. HTN.

Mul ti
pleinvesti
gat i
veandcl inicalobser vat i
onshavedemonst r
at edHTNt obeacommoncl ass
effectresult
ingf rom tr eat mentwi thVEGFi nhibi tors,occur r
ingin10- 30%ofpat ient st reated
withbevaci zumab.Ther atesofsubst ant i
alHTNappeart odependont heant iangiogeni cagent
used, t
het umort ype, andpat ient -r
elatedf actor s,includingageandcomor bidity.Angi ogenesi s
i
nhi bit
or-rel
atedHTNi st ypical l
ymanageabl ewi thear l
yi ni
ti
ati
onofphar macol ogi ct her apyto
reachaccept edbl oodpr essur et argets.Pr eferredant i
hypertensiveagent sf orangi ogenesi s
i
nhi bit
or-associ atedHTNi ncludeangi otensin- conver tingenzymei nhibitorsand
dihydropyr i
dinecal cium channelbl ockers, alt
hought herearemi ni
maldat at osuggest
super i
orit
yofasi nglecl assofagent s.Earlyandaggr essiveini
tiationofant ihyper tensi ve
therapyappear st ohel pmai ntai ntreatmentschedul eandr educet her iskofsubst ant ial
compl icati
ons,incl udingmal ignantHTNandr ever sibleposteri
orl eukoencephal opat hy.
Bevaci zumabi snotknownt ohaveanysi gni f
icantr iskofQTpr olongat ionorar rhyt hmi as.
Ther eisasmal lincreasei ncar diacischemi aint r
ialsofbevacizumab, whi chi slikel yduet o
thromboembol iccompl icationsr athert handi rectef f
ectont hecor onar yar teriesorvasospasm.
Ther eisanappr oximat e20%i ncidenceofvenoust hromboembol i
sm andanappr oximat e5- 6%
i
nci denceofar terialthr ombosi si npat i
entst reat edwi thbevacizumab.

Quest
ion:

12. Youar easkedtogiveapr esent


ationonhyper
tensi
on(HTN)toagr
oupofmedi
cal
st
udents.Thegroupisint
erestedinoutcomes,pr
eval
ence,andt
reat
mentofHTNi
nthegener
al
popul
ati
onaswel lasseveralethni
cgroups.

Whi
choft
hef
oll
owi
ngi
sthemostappr
opr
iat
est
atement
?

a. Me naremoreli
kelytobet
reatedthanwomen.
b. Pati
ents>80yearsofageareunli
kelytobenef
itf
rom t
reat
ment.
c. Preval
enceisgreat
eramongwoment hanmen.
d. Preval
enceisgreat
estamongnon- Hispani
cbl
ackpopulat
ioncompar
edwi
thot
her
popul
ations.
e. HTNisnotamaj
orr
iskf
act
orf
ori
nci
denthear
tfai
lur
e.
Answer
:

12. d. Pr
eval
enceisgreat
estamongnon-
Hispani
cbl
ackpopul
ati
oncompar
edwi
th
ot
herpopul
ati
ons.

Basedont hemostr ecentdat


afrom theNat ionalHealthandNutri
ti
onExami nati
onSur vey,HTN
prevalenceishighestamongnon- Hispani cblackadultswhencompar edwithot herethnic
groups.Non- HispanicAsianadult
shadal owerpr eval
enceofawarenessthant heot herraces
andHi spanic-ori
gingroupsandlowert reatmentthannon- Hi
spanicwhiteandnon- Hispanic
blackadults.Menandwomenhaveasi mi l
arprevalenceandawarenessofHTN.However ,
mor ewoment hanmenwer etr
eati
ngt heirHTNandhadi tundercontr
ol.TheHYVET
(HypertensionintheVeryElderl
yTrial
)st udydemonst rat
edthebenefi
toftreatmentofHTNi n
patients>80yearsofage.HTNi sthel eadingmodi fi
ableri
skfact
orforthepreventionofhear t
fail
ure.

Quest
ion:

13. A65- year-


oldmalewithhypert
ensionanddiabet
esisseenintheoff
ice.Hi
s
medicati
onsincl
udeamlodi
pine10mgqdandmet formin500mgbi d.Hisbl
oodpressur
e(BP)
i
s145/ 92mm Hg.Cardi
acexami nat
ionreveal
snormalheartsounds.Cr
eati
ninecl
ear
anceis
nor
mal ,andt
hereisnoprot
einur
ia.

Whi
choft
hef
oll
owi
ngi
sthemostappr
opr
iat
etr
eat
mentr
ecommendat
ion?

a. Fur
osemi de20mgqd.
b. Noc hangeinmedicat
ions.
c. Li
sinopri
l10mgqd.
d. Metoprol
olsucci
nate25qd.
e. Spi
ronolact
one25mgqd.

Answer
:

13. c. Li
sinopr
il10mgqd.

Thereismoder ateevidencet osuppor ti


nit
iat
ingdr ugtreatmentwithanangi otensin-converti
ng
enzymeinhi bit
or ,
angi ot
ensin-receptorbl
ocker,calci
um channelbl ocker
,orthiazide-type
diur
eti
cint henonbl ackhypertensivepopulat
ion,includingthosewithdiabetes.Inthebl ack
hypert
ensivepopul ati
on,incl
udingt hosewithdiabetes,acal ci
um channelblockerort hiazi
de-
typedi
ureticisr ecommendedasi niti
alt
herapy.Inmanycases, twohypertensivedr ugsar e
neededforbl oodpr essurecont rol
.

Quest
ion:

14. A67- year -


oldfemal ei
sseenf orevaluati
onofpal pit
ations.Shehasahi st
oryof
hypert
ension( HTN)andhyper chol
esterolemia.Medicati
onsi ncludelisi
noprilandat orvastat
in.
Onphysicalexami nati
on,herheartrat
ei s90bpm andbl oodpr essureis182/ 74mm Hg.
Cardi
acexami nati
oni snormalexceptfora grade3/6cr escendo- decrescendomur muratt he
l
eftuppersternalbor derthatr
adiatesintothecaroti
dsandagr ade3/ 6apicalsystolicmur mur.
Anelectr
ocardi ogram showssi nusrhythm withoccasionalpremat ureventri
cul arcontracti
ons
(PVCs).A24- hourambul atorymoni t
ordemonst rat
esmul t
ipl
eepi sodesofat rialf
ibri
ll
ation(AF)
andt
woepi
sodesofanar
rowcompl
ext
achycar
dia.

Whi
choft
hef
oll
owi
ngf
indi
ngsi
nthi
spat
ienti
smostl
ikel
yther
esul
tofpoor
lyt
reat
edHTN?

a. Atr
ioventr
icul
arnodalr
e-ent
rantt
achycar
dia.
b. AF.
c. PVCs .
d. Mit
ralregurgi
tat
ion.
e. Aorti
cstenosi
s.

Answer
:

14. b. AF.

HTNi sassociat
edwi t
hmul ti
plecardiovascularcomplicati
ons.Thesei ncludelef
tventri
cular
hypert
rophy,whichisanear l
yfindi
ngi npatientswit
hHTN, heartfai
lur
e( bothwithreducedand
preser
vedejecti
onf r
acti
on),str
oke(ischemi candhemor r
hagic),i
schemi cheartdisease,
chroni
ckidneydisease,andAF.Ther eisnotacl earincreaseinotherar r
hythmias,suchas
PVCsorat r
ioventr
icul
arnodalre-entranttachycardi
a,orvalvularheartdisease,suchasmi tr
al
valveprol
apseoraor t
icstenosi
s.

Quest
ion:

15. A55- year-ol


dfemal
eisseeni
ntheoffi
ceforcar
diovascul
arr
iskassessment.Herbl
ood
pressure(
BP)is155/92mm Hg.ShechecksherBPatherlocalphar
macyweekl yandstat
es
thatiti
snormal.

I
naddi
ti
ontol
ifest
ylechanges,
whi
choft
hef
oll
owi
ngi
sthemostappr
opr
iat
e
r
ecommendat
ions?

a. Ambulat
oryBPmoni t
ori
ng.
b. Nofur
therrecommendati
ons.
c. Ret
urntooffi
cein1weekforBPcheck.
d. Ret
urntophar macyf
orr
epeatBPcheck.
e. St
artl
isi
nopril10mgdai
ly.

Answer
:

15. a. Ambul
ator
yBPmoni
tor
ing.

Ambul at oryBPmoni toringoffersmanybenef itsandaddspr ognosti


cval uet oBPr eadi ngs
obtainedi ntheof fi
ce.Itpr ovi
desagr eaterdegr eeofriskassessmentt ot hatofBPsobt ainedin
theof f
iceset ti
ng.Indeed, subclinicalorgandamagei smor eclosel
ycor relat
edwi th24- hour
meant hanwi thof f
iceBP, and24- hourmeansyst ol
icordi astoli
cBPhasast eeperr elationship
withcar diovascularmor bidorf at
alevent sthant hecor respondingofficeBPval ues.Itishel pf
ul
i
neval uat i
ngpat ientswi thsuspect edresistanthyper t
ensi on(HTN)andpr edi
ctiveoft hoseat
ri
skf orvar i
ouscompl ications,includi
ngend- stagekidneydi sease.Absenceofnoct urnal
decreasei nBP, ornoct urnaldipping,isassoci atedwithmar kersofrenaldysf uncti
on.
Repeat ingso- call
ednor malBPatherl ocalphar macyi soflitt
leval
ue.HerBPmayr emai n
elevateddur ingof f
iceeval uati
onbutnor malathome, suggest i
veof“ whitecoatHTN. ”St arti
ng
drugt
herapymaybepr ematur
eatt
hispointandassociatedwi
thpoorcompl i
anceift
hepati
ent
i
snotconvincedshehasHTN.However,tr
uewhitecoatHTNhasbeenshownt opr
edictt
he
ri
skofcardi
ovascul
arevent
satanint
ermediat
elevelbetweennormalBPandt rueHTN,and
mostHTNexper t
swouldrecommendtreat
mentoncedi agnosi
sisestabl
ished.

Quest
ion:

16. A50- year-


oldmalewi
thaf
amil
yhistor
yofhyper
tensi
on(
HTN)isaskedtocheckhi
s
bl
oodpr
essure(BP)athomet
oeval
uat
efor"whit
ecoatHTN."Hi
sBPintheoff
icei
s160/70
mm Hg,
whileBPschecked4ti
mesdai
lyathomeforaweekaver
age130/70mm Hg.

Whi
choft
hef
oll
owi
ngi
sthemostappr
opr
iat
erecommendat
ion?

a. Recommendexer ciselifest
ylechanges.
b. St
artmetopr
ololsucci nate25mgqd.
c. St
arthydr
ochlor
ot hiazide25qd.
d. St
artaspi
ri
n81mgqd.
e. St
artl
isi
nopri
l10qd.

Answer
:

16. a. Recommendexer
cisel
if
est
ylechanges.

Whi t
ecoatHTNi snotabeni gncondi ti
on.Indivi
dual
swi t
hwhi tecoatsyndr omet endt ohavea
greaterprevalenceofmet abolicri
skf actor
sandmor ecommoncoexi stencewi thor gan
damage, andt heyhaveagr eatertendencyf orBPtoriseovertimeandf orsust ainedHTNand
diabetesmel l
itustodevelop.Theyal sohaveal ong-t
erm cardi
ovascularriskthati slessthan
thatofsustainedHTN, butneverthelessgreaterthanthatofindivi
dualswithin-andout -of-
offi
ceBPnor mal i
ty.Assuch, r
ecommendat ionsforl
if
estylemodi f
icat
ionsar ereasonabl eeven
i
fphar macologi ctherapyisnotindicated.Aspi r
ini
snotindicatedforprimarypr eventioninmen
<55year sofage.

Quest
ion:

17. A28- year-oldwomencomesi nforevaluationofrecurr


entheadaches.Shehasbeen
toldathealthscr eeningsinthepastthatshehasbeenhyper t
ensive.Sheisonnomedi cati
ons
andhasnoheal thproblems.Sheeat saheal thydietandgoest othegym 5days/ week.She
deniestobaccouseordr uguseanddr inksanoccasi onalglassofwi ne.Onexami nation,her
bloodpressur eis190/ 100mm Hgont herightand188/ 98mm Hgont helef
t.Herhear tratei
s
76bpm, andbodymassi ndexis22.Fundoscopi cexami nati
onr eveal
sarteri
ovenousni cking
andnohemor rhages.Lungexami nati
oni sclear,andcardiovascularexaminati
onr eveals
normalS1/ S2,andS4 ispresent.Ext
remi t
iesarewi t
houtedemaandwi t
hequalpulseswi t
hout
delay.Compl etebloodcountandel ectrolyt
esar enormal.Creati
nineis1.0mg/ dl.
Whi
choft
hef
oll
owi
ngi
smostl
ikel
ytoi
dent
if
ythecauseofherhyper
tensi
on(
HTN)
?

a. Transt
horacicechocardiogram.
b. Abdominalcomput edtomogr aphyangi
ogr
aphy.
c. Renalart
eri
alDoppl erst
udy.
d. Dexamethasonesuppr essiontest
.
e. Urinemetanephri
nes.

Answer
:

17. c. Renalar
ter
ialDoppl
erst
udy.

Thispat i
entisayoungwomanwi thsever eHTNandevi denceofend- organdamageon
physicalexami nation.Themostl i
kelyet i
ologyisfi
bromuscul ardyspl
asia( FMD).Evaluat i
onfor
thi
sr eversibl
ecausei simportant.Initi
alscreeni
ngwi t
hr enalDopplercanbeacost -effecti
ve
screeningt ool
.Ifthereisevidenceofi ncr
easedvelocit
ies, t
henextstepwoul dbeinvasive
evaluati
on.Comput edtomographyangi ographyandabdomi nalmagnet icresonance
angiographybot hhavehi ghaccur acyt odetectFMD.Thesecanbeusef ulinanondi agnost i
c
ult
rasoundwi t
hhi ghi ndexofsuspi cion.Echocardi
ogram maybehel pfultoevaluateleft
ventri
cularhyper t
rophy, butgivenlackofmur murandequalbl oodpressure,identi
fi
cationof
coarctati
onoft heaor taislessl i
kely.Shedoesnotr epor tsymptomsoff lushing,pal
pitati
ons, or
sweat i
ngt hatwoul dsuggestpheochr omocytoma.Sheal sodoesnothavecent r
alobesi t
y,
buffalohump, ormoonf aci
est hatwoul dsuggestCushing’ ssyndrome.

Quest
ion:

18. A39- year -ol


dmani sevaluatedinyourof fi
ceforhyper
tension.Hehasbeenheal t
hyand
i
sonnomedi cations.Hei sanonsmoker ,eatsaveget ari
andiet,dr
inksoccasi
onalal
cohol,and
walks2mi l
esever yday.Hi sexaminationrevealsbloodpressure(BP)142/92mm Hgont he
ri
ght,and144/94mm Hgont hel
eft.Hishear tr
ateis58bpm andbodymassi ndexis21.His
cardi
ovascul
arexami nat
ionisnor mal.Hebr i
ngsi nrecor
dingsfrom hi
shomeBPmoni tor,
whichyoucalibrateandf i
ndaccur ate.Hi
sr eadingsrangefrom 135/88mm Hgt o150/96mm
Hg.Hiselect
rolytes,uri
nalysi
s,andcr eat
ini
near enor mal
.

I
naddit
iontocont
inui
nghi
sheal
thyl
if
est
yleandl
ower
inghi
ssal
tint
ake,
whi
choft
hef
oll
owi
ng
doyourecommend?

a. Metoprol
olsucci
nat
e.
b. Li
sinopri
l.
c. RepeatBPi n3months.
d. Amilodi
pine.
e. Losart
an.
Answer
:

18. d. Ami
lodi
pine.

Thispati
enthasconsi stentBP>140/ 90mm Hgandt hereforewouldbenef i
tfr
om
pharmacologictherapy.Heal r
eadypr act
icesahealthyl i
festyl
eandhassever alhomer eadi
ngs
thatareal
soel evated.Initi
altr
eatmentwi thdihydr
opyridinecalcium channelblockeror
thi
azidedi
ur et
icisconsi deredfir
st-
li
net herapy.Angiotensin-conver
tingenzymei nhibi
torscan
beconsideredint hosewi threnaldysfuncti
on.Beta-blockersar enowf our
th-l
inetherapy.

Quest
ion:

19. Manychangesi nl i
festyl
esuchaswei ghtl
oss,maint
enanceofnormalbodyweight ,
i
ncreasedphysi
calacti
vit
y, moderationofal
coholi
ntake,
anddietar
ymodifi
cat
iontoinclude
morefrui
ts,
vegetabl
es,andl ow-fatdair
yproduct
s,andareducti
oninsodi
um insal
t-sensiti
ve
i
ndivi
dualscanlowerbloodpr essure.

I
nwhichofthefoll
owi
nggroupshavecl
ini
calt
ri
alsshownt
hatt
hesel
if
est
ylechangescan
pr
eventhyper
tensi
on(HTN)?

a. Preecl
ampsi a.
b. Prehypertensi
on.
c. Chronickidneydisease.
d. Renalarterystenosi
s.
e. Hyperaldoster
onism

Answer
:

19. b. Pr
ehyper
tensi
on.

Eachoft hesehealt
hylif
estyl
esshouldbet hebasisfort
reatmentofallindi
viduals,especiall
y
thoseatincreasedcardi
ovascularri
sk.Inpati
entswithprehyper
tension,adopti
onoft hese
healt
hylifest
ylescanreducethedevelopmentofHTN.Pat ientswit
hpr eecl
ampsi aar eatriskof
developi
ngHTNast heyageandshoul dadoptheal t
hylif
estyl
es,al
thoughnocl inicaltri
alsare
avail
ableinthispopul
ation.Healt
hyli
festyl
esalsoshouldbei mpli
catedinthosewi thHTNf rom
anycausebutar el
essli
kelytoreducebloodpressureinthosewithsecondar yHTN.

Quest
ion:

20. A53- year-


oldmanisr
efer
redt
oyouformanagementofhyper
tensi
on(HTN)
.Hisbl
ood
pressur
eis163/94mm Hg.Hi
sheartr
atei
s82bpm, hei
ghti
s70inches,andwei
ghti
s260
pounds.

Presenceofwhichoft
hefol
lowi
ngconst
ell
ati
onsofsi
gnsandsympt
omswoul
dbemost
helpf
ulini
denti
fyi
ngapossi
blecont
ri
but
ingfact
ort
ohisHTN?

a. Morningheadachesandwaki ngwi
thchoki
ngsensat
ion.
b. Photophobi
aandvi sualaur as.
c. Epi
sodiccutaneousflushinganddiarr
hea.
d. Ast
erixi
sandt el
angiectasias.
e. Noct
uri
aandpost
-voi
ddr
ibbl
ing.

Answer
:

20. a. Mor
ningheadachesandwaki
ngwi
thchoki
ngsensat
ion.

Thispatienthasabodymassi ndexof37. 3kg/ m2,i


denti
fyinghi m wit
hCl assIIobesit
y.Obesit
y
i
sf r
equent lyassoci atedwi thobstr
uctivesleepapnea( OSA) . Epi
demi ological,animal
,and
cli
nicalstudiesal lsuggestt hatOSAcancauseorcont ri
but etoHTN.Mor ningheadaches,loud
snoring,gaspi ngorchoki ngforai
r,andexcessdayt imef atiguemaybesi gnsofOSAandi nthe
appropriatecl i
nicalcont extshoul
dpr omptconsi derat
ionf orpolysomnogr aphy( sl
eep
study).
Cirrhosis( asterixi
sandt el
angiectasias),
benignprost ati
chyper plasia(nocturi
aandpost
-voiddribbling),migr aineheadaches( photophobiaandvi sualaur as)
,andcar cinoi
dsyndrome
(epi
sodiccut aneousf lushinganddiarrhea)ar enotassociatedwi thHTN.

Quest
ion:

21. A58- year-oldmanpr esentstoyourclinicforevaluati


onofr esistanthypertension(HTN).
Hispastmedicalhistor
yisnot abl
eforcoronarydi seasewithpri
ort hree-vesselbypasssur gery
atage49years.Heexer cisesregul
arlydespit
echr onicstableangina.Her eportsoccasional
ly
wakingwithachoki ngsensationatnight
.Heavoi dsexcesssal tandt riestoeataheal thydiet
.
Hismedicati
onsincludeaspirin81mg, ator
vastat i
n80mg, hydrochlorothiazide50mg,
enal
apri
l20mg, aml odi
pine10mg, andmet oprolol100mgdai l
y.Onexami nati
on,hisheart
2
rat
eis65bpm, bloodpr essure(BP)is152/75mm Hg, andbodymassi ndexi s36kg/ m.

Whi
choft
hef
oll
owi
ngi
smostl
ikel
ytoi
mpr
ovet
hispat
ient
'sBPcont
rol
?

a. Renalsympatheticdenervati
on.
b. Continuouspositi
veair
waypr essure(
CPAP)
.
c. Implantabl
epulmonaryar t
erypressur
emoni
tor
.
d. Chelati
ontherapy.
e. Enhanc edext
ernalcounterpul
sati
on.

Answer
:

21. b. Cont
inuousposi
ti
veai
rwaypr
essur
e(CPAP)
.

Treatmentofobst ructi
vesleepapnea( OSA)wi t
hCPAPcani mproveBP, althought heef f
ect
maybemodest( adecr easeinsystolicBPofabout3- 6mm Hg) .Thispat ientmostl ikelyhas
OSA.Ther efore,CPAPi sthecorrectchoi ce.TheSYMPLI CI
TYHTN- 3(RenalDener vationin
PatientsWi t
hUncont roll
edHyper t
ensi on)tri
alf
ail
edtoshowbenef itwithr enalsympat heti
c
denervat i
on.Al thoughquest ionsregar dingadequacyofdenervati
oni nthei nterventionar m
havebeenr aised,asofOct ober2015, t her
eisnoroleforsympatheticrenaldener vationout si
de
ofthecont extofacl inicalt
ri
alfortreatmentofHTN.Enhancedext er nalcount erpulsat i
onhas
beenusedt ot r
eatangi nabutwoul dnotbet hetr
eatmentofchoiceforHTN.Ther ei snor olef
or
chelationther apyint het r
eatmentofHTN.Ther eisnor ol
eforimplantablepul monar ypr essure
moni torsint het r
eatmentofHTN.
Quest ion:

22. A57-
year
-ol
dmanpr
esent
stoyourcl
ini
cformanagementofHTN.Hi
smedi
cat
ions
i
ncludeamlodipine10mg,l
isi
nopril40mg, andchlor
thal
idone25mg.Onexami nation,hi
s
heartr
ateis78bpm, bl
oodpressure(BP)is156/93mm Hg, andbodymassindexis42kg/ m2.
Heartexaminat
ionisnot
abl
ef oranS4 andsustai
ned,nondisplacedpoi
ntofmaxi
mali mpulse.

Whi
choft
hef
oll
owi
ngt
est
smaybemostappr
opr
iat
eforeval
uat
ionoft
hispat
ient
'sHTN?

a. Holtermoni t
or.
b. Ti
lttable.
c. Exercisetol
erancetest.
d. Polysomnogr aphy.
e. 5-Hydr oxyi
ndoleacet
icaci
d(5-
HIAA)
.

Answer
:

22. d. Pol
ysomnogr
aphy.

HTNandobst ruct i
vesleepapnea( OSA)f requent l
yco- exi
st,wit
h50%ofOSApat ientshavi
ng
HTN, and30%ofhyper tensivepat i
ent shavingOSA.I naddi t
ion,hyper
tensivepatientswit
han
abnormall
yblunteddi pi nBPwi thsleep( nondi ppers)aremor eli
kelytohaveOSA.TheSevent h
Reportoft
heJoi ntNat i
onalCommi t
t eeonPr evention,Detecti
on,Eval
uation,andTr eat
mentof
HighBloodPressur erecogni zessleepapneaasoneoft heidentif
iabl
ecausesofHTN.I n
pati
entswit
hresi stantHTN( elevat
edBPdespi t eadequatedosesoft hreediff
erent
anti
hypert
ensi
veagent s,includingadi ur
et i
c),secondar ycausesofHTNshoul dbeconsi der
ed. 

Inthispat i
entwi thClassI I
Iobesity(bodymassi ndex≥40kg/ m2)
, eval
uationforsi
gnsor
sympt omsofsl eepapneawoul dbeappr opriat
e, fol
lowedbypossi blepolysomnogr am (sleep
study)i fi
ndicated.5- HIAAisthebr eakdownpr oductofser otoni
n, and24- houruri
nar y5- HIAA
excretionmaybeusedasascr eenf orcarcinoi
dsyndr ome.Ther eisnosuspi cionforcar cinoi
d
syndromebasedont hei nf
ormationpr ovided,andcar cinoidsyndr omeismor eoft
en
charact er
izedbyf lushingspell
saccompani edbyhypot ensionratherthanHTN.Ti lttabl
et ests,
exercisetolerancet ests,andHoltermoni torsar enotindicatedfort heevaluat
ionofHTN.

Quest
ion:

23. A66- year -oldmanwi thhyper tension( HTN)isrefer


redf orevaluati
onofr esi
stant HTN.
Hispastmedi calhistoryi
ssignif
icantf orcoronaryar t
erydisease,sleepapnea, anddi abetes.
Hismedi cationsi ncludelisi
nopri
l40mg, hydrochlorot
hiazi
de25mg, amlodipine10mg, and
met oprololsucci nate150mg.Hewal ks 2 milesdail
y,useshi spositi
vepr essurevent il
ati
on
deviceni ghtl
y, andf oll
owsasodi um- restri
cteddiet.Thephysi calexam revealsabl ood
pressure( BP)of172/ 90mm Hgont hel eft
,168/88mm Hgont heri
ght,andhear trate58bpm.
Cardiovascul arexam r eveal
sanS4.Hi slungsar eclear,andther earenoabdomonalbr ui
ts.
Labor at
or yval uesincludesodium 131mmol /L,potassium 4.
1mmol /
L,creati
tine1. 5mg/ dl,
andt hyroid-stimulat i
nghormone1. 5I U/ ml.

Whichofthef
oll
owi
ngismostappr
opr
iat
einhi
scar
e,i
naddi
ti
ont
orei
nfor
cingmedi
cat
ion
compli
anceandsodi
um r
est
ri
cti
on?

a. I
ncr
easemet
opr
olol
succi
nat
eto200mgdai
ly.
b. Addl
osartan50mgdai l
y.
c. Addspi
ronolact
one25mgdai l
y.
d. Addcl
onidine0.
1mgt wicedai
ly.
e. Adddoxazosin1mgatbedtime.

Answer
:

23. c. Addspi
ronol
act
one25mgdai
ly.

Resi stanthyper t ensionisdefinedasaBPl evelthatremainsabovegoaldespi t


et heconcur rent
useoft hreeant ihyper t
ensivemedi cati
ons, incl
udingat hiazidediuretic.Despit
eaheal t
hy
l
ifestyleandt akingf ourantihypertensivemedi cations,t
hisman’ sBPr emainsabovegoal .Whi l
e
evaluat i
onf orrenalar t
erystenosi smaybeappr opri
ate,t
reatmentofhi sBPatt hist imewi than
addi ti
onalagenti swar r
anted.Spi r
onol actonei sanef f
ecti
veadd- ondr uginthet r
eat mentof
resistanthyper tension.ThePat hway2st udyexami nedspironolactone, bi
soprolol,and
doxazosi nasadd- ontherapyt oat hree-drugr egimen.Oft hoser eceivingspir
onol act one,58%
achievedt het argetBP, ascompar edwi t
h42%oft hoseondoxazosi nand43%oft hoseon
bisopr olol.Givent hi
spatient’
sr estinghear trate,hismetoprololshoul dnotbet it
rat edf ur
ther.
Combi nationt her apywithanangi otensin-conver ti
ngenzymei nhibi
torandanangi otensin-
recept orbl ockeri snotrecommended.

Quest
ion:

24. A22- yearol dmani sref err


edf orevaluationofhyper tension( HTN)i dent i
fiedinapr e-
empl oymentphysi cal.Hehasnosi gnifi
cantpastmedi calhi story.Hehasaf ami lyhistoryof
HTN  inhisfat herandpat ernalgr andmot her.Hei sonnomedi cations.Her uns3mi lesdaily,
eatsavegandi et ,
isal i
felongnonsmoker ,anddr inkssoci al l
y.Physi calexam r evealsabl ood
pressur eof168/ 92mm Hgont heright,174/88mm Hgont heleftside, heartrate66bpm,
height75  i
nches, weight180l bs,andabodymassi ndexof22.Car diovascul arexam r eveals
normalS1  andanS2  withoutmur mur sappreciated.Lungswer ecl earthroughout .Pulses2+
andequali nal lextremities.Nobr ui
tswer enot ed.Labor at oryval uesi ncludesodi um 132
mmol /L,potassi um 5.4mmol / L,chlori
de97mmol /L,bicar bonat e28mmol /L,bloodur ea
nit
rogen22mg/ dl,andcr eat
inine0. 7mg/ dl.Urinalysisr eveal snopr ot ei
n.

Whi
choft
hef
oll
owi
ngi
smostl
ikel
ytor
evealt
heet
iol
ogyofhi
sHTN?

a. Serum renin/
aldosterone.
b. Transthoraci
cechocar diogr
am.
c. Renalultr
asound.
d. Thoracicmagnet i
cr esonanceangi
ogr
aphy(
MRA)
.
e. Serum catecholamines.

Answer
:

24. a. Ser
um r
eni
n/al
dost
erone.
Inyoungpat i
entswi t
hnewl ydiagnosedHTN, i
tisimportanttobeal ertforsecondar ycausesof
HTN.I nthispatientwithaf amil
yhi st
oryandel evatedser um potassi um, considerati
onof
hyperaldosteronism shouldbeent ert
ained.Ser um reni
nandal dosteronel evel
swi llconfi
rmthi
s
diagnosis.Whi l
eanechocar di
ogram mayr eveallef
tventricul
arhyper t
rophy, t
hiswoul dbea
secondar ymani f
estati
onofhi sHTN.Hedoesnothaveevi dencebyexam ofcoar ctati
onor
renalarterystenosis;t
hereforeimaginghi srenalarteri
esandt hor acicaor taareunl i
kel
ytobe
reveal
ing.Hedoesnothavesympt omsoff l
ushi ng,
headache, palpitati
ons, ordiarrheathat
wouldsuggestpheochr omocyt oma.

Quest
ion:

25. A59- year-oldmani sseenforeval uat


ionofhypertension(HTN) .Hehasnohi stor
yof
diabetes,kidneydi sease,orcoronaryarterydisease.Hewasst art
ed6mont hsagoon
hydrochlorothiazide25mgdai l
yandl i
sinopri
l20mgdai l
y.Hisbloodpr essure(BP)i
s148/ 82
mm Hgandhear trateis72bpm, whichar econsist
entwithhomer ecordi
ngs.Laboratory
valuesincludeser um pot assi
um of4.1mmol /L,serum cr
eatinineof1.0mg/ dl,
andur i
nalysi
s
withoutevidenceofpr oteinur
ia.

Whichoft
hefol
lowingi
sthemostappr
opr
iat
enextst
ep,
inaddi
ti
ont
orei
nfor
cingdi
et,
exer
cise,
andsodi
um r
estri
cti
on?

a. Addme topr ol
olsucci
nate50mgdai ly.
b. Increaselisinopri
lto40mgdai l
y.
c. Pe r
form 24- hourambulator
yBPmoni t
oring.
d. Cont i
nuecur r
entregi
men.
e. Evaluatepl asmar eni
nandaldoster
onel evel
s.

Answer
:

25. b. I
ncr
easel
isi
nopr
ilt
o40mgdai
ly.

Themostl ikelyreasonforthepoorcont r
olofhi sHTNi sinadequatether
apy.Mostpati
ents
requireatl
easttwo, ifnott
hree,drugsforadequat eBPcont rol
.Themaximum recommended
doseofl i
sinopri
lis40mgdai l
y,andthisisthebestchoi ce.Beta-bl
ocker
sarenotfi
rst-
li
ne
therapyforessentialHTN.Withanor malpot assium level,andinadequat
emedicalther
apy,
thereisnoi ndi
cationtoobtai
nr enin/
aldost
er onelevels.

Quest
ion:

26. A30- year-oldwomanwi t


habi cuspidaorti
cval
ve,mil
daor t
icst
enosis,andhistoryof
surgicall
yr epair
edcoar ct
ati
onoft heaortapresent
stocli
nic16weekspr egnant.Physical
exami nati
onr evealsapulseof90bpm, bloodpressur
eof120/80mm Hgi ntherightarm,a
systoli
cej ecti
onclick,aII
/VIsystoli
cejecti
onmur muratt
heleftupperst
ernalborder,andmild
brachialfemor aldelay.

Whi
choft
hef
oll
owi
ngi
sthi
spat
ientati
ncr
easedr
iskf
ordevel
opi
ngdur
ingpr
egnancy?

a. Hyper
tensi
ve-
rel
atedcondi
ti
ons.
b. Atr
ialarr
hythmias.
c. Spontaneousaorticdissect
iondur
ingsecondt
ri
mest
er.
d. Ventri
cul
ararrhythmias.
e. Thromboticevents.

Answer
:

26. a. Hyper
tensi
ve-
rel
atedcondi
ti
ons.

Althoughpatient
swi t
habi cuspidaor t
icvalvemaydevel opaor t
icrootdil
ation,which
predisposesthem tospontaneousdi ssect
ion,thismostof t
enoccur sinlategestati
onort he
postpartum peri
od.Neitheranincreaseinar r
hythmi corthrombot i
cevent sisassoci at
edwith
theseleft-
heartl
esions.However ,
thereisadef ini
teincreasei nhypertensi
ve- r
elat
edcondi t
ions
duringpregnancyinwomenwi thpr i
oraorti
ccoar ctati
on, despit
enosi gnif
icantresidual
gradient.

Quest
ion:

27. A30- year -oldwomanat26weeksgest ationpresentsf orevaluati


onofhyper tensi
on
(HTN).Shehasnosi gnif
icantpastmedi calhistory.Physicalexaminat i
onr eveal
sapul seof90
bpm, bloodpressur e(BP)of160/ 90mm Hgi nt her i
ghtarm, 158/90i nthel ef
tarm,andaI I
/VI
systol
icejecti
onmur muratt heleftsternalbor der.Shehassymmet ri
cpul seswithouta
brachial-f
emor aldelay.Bloodwor k,i
ncludi ngliverfuncti
ont ests,i
snor mal .A24-houruri
ne
sampl edetected0. 1gramsofpr oteinpr esent.Youf oll
owedhert hroughoutherpr egnancyand
postpartum periodandherBPnor malizedwi thin1mont hfollowingdelivery.

Whi
choft
hef
oll
owi
ngi
sthecor
rectdi
agnosi
s?

a. Aorti
ccoar ct
ation.
b. Preeclampsia.
c. HELLP( hemolysi
s,el
evat
edl
iverenzymel
evel
s,andl
owpl
atel
etl
evel
s)syndr
ome.
d. Pregnancy- i
nducedHTN.
e. Pre-existi
ngsystemicHTN.

Answer
:

27. d. Pr
egnancy-
inducedHTN.

Fi
veper centofpregnantwomenhavepr e-
existi
ngHTN, defi
nedbyel evatedBPsdocument ed
pri
orto20weeksgest ation.Morethan10%ofwomenbecomehyper tensiveinthesecondhal f
ofpregnancy.IftheBP returnstonormalvalueswithi
n6weeksofdel iver
y, t
heyareclassifi
ed
ashavingpr egnancy-i
nducedHTN, whichisthecorrectdi
agnosi sint hi
scase.Anaddi ti
onal
5%ofwomendevel opHTNi nthesecondhalfofpregnancy,associatedwi th>0.3gramsof
prot
einintheur i
neovera24- hourperi
od,whichisdefinedaspr eeclampsi a.Aort
iccoarctati
on
i
sar arecauseofHTN, andi soft
enaccompani edbyar adial
-femor aldelayonexami nation.
HELLPsyndr omeoccur si nwomenwi t
hpreeclampsia,andincludesel evatedli
verenzymes,
hemolyticanemia,andt hrombocytopenia.
Question:

28. A29-
year
-ol
dwomanwi
thani
ntr
aut
eri
nepr
egnancyat25weeksgest
ati
onpr
esent
sto
theemer gencyr oom wi thincr easedheadachesf or1 week.Herpastmedi calhistor
yincludes
type2di abetesmel litusandobesi ty.Atbaseline,shehasheadachesappr oximat el
yonceor
twiceamont h,i
ncr easingi nsever i
tyovert helastweek.Shehasmi l
dphot ophobia.Her
temper atur
eis37. 2Cel sius, bloodpr essure(BP)168/ 112mm Hg, heartrate86bpm, and
oxygensat urat
ionof99%onr oom ai r. 
Fundoscopi cexami nat i
oni snor mal .Jugularvenouspul sati
oni snor mal.Pointofmaxi mal
i
mpul seisnondisplaced.Thehear tisr egul
arwi t
hnomur mur s,rubs,orgallops.Lungsar e
clear.Abdomeni sgr avid.Ext remiti
esshow1+pi tti
ngedema.Neur ol
ogicexam i sgrosslyint
act
withnor malrefl
exes.Labor atoryfindi ngsshowahemogl obi
nof11. 2g/dl,plat
eletcountof
199,000cel l
s/mm3, creatinineof1. 0mg/ dl
,andnor maltransami nases,uricacid,andlactate
dehydr ogenaselevel s.Ur i
nalysi sreveal sspecif
icgravit
yof1. 022,negativeforleukocytes,
blood, pr
otei
n,ket ones, andgl ucose.

Whi
choft
hef
oll
owi
ngi
sthemostappr
opr
iat
enextst
epi
nthemanagementoft
hispat
ient
?

a. I
mmediatedeli
veryoft
hefetus.
b. Admini
str
ati
onoforalat
enolol.
c. Pai
nrel
iefandcloseBPmoni tor
ing.
d. Admini
str
ati
onoforall
osartan.
e. Admini
str
ati
onoforalmethyldopa.

Answer
:

28. e. Admi
nist
rat
ionofor
almet
hyl
dopa.

Methyldopai stheprefer
redagent ,duet odatademonst r
atingstableut er
oplacentalbloodflow
andfetalhemodynami csinlong- t
erm studies.Whi l
elabetalolmaybeoner easonabl e
alt
ernati
ve,otherbeta-blockerssuchasat enololhavebeenassoci atedwithintr
auterinegrowth
rest
rict
ionandar eoftenavoided, especial
lyear l
yinpregnancy.Angi ot
ensin-converti
ng
enzymei nhibit
orsandangi otensin-recept
orbl ockersarecont r
aindicatedduet ofetalgrowth
rest
rict
ion,oli
gohydramni os,renalfail
ure,
anddeat h.
 

Hal l
mar ksofpr eeclampsiaincludepr oteinur
ia(>2.0gin24hour s)
,elevatedcreatinineand
transami nases, l
owpl at
eletcount ,andper si
stentepi
gastr
icpai n,f
indingsnotpr esentinthis
case.I ndicati
onsf ordel
iveryforpreeclampsi aincl
udegest ati
on>38weeks, HELLP( hemolysi
s,
elevatedliverenzymes, lowplat el
etcount )syndrome,severefetalgrowthr est
ricti
on,
nonr eassuringfetaltest
ing,orol i
gohydr amnios.TheRepor toftheNat i
onalHighBl ood
Pr essureEducat i
onPr ogram Wor kingGr ouponHi ghBloodPr essurei nPregnancy
recommendst reatmentforBP  l
evelsabove150- 160/
100- 110mm Hg, r
atherthanobser vati
on
alone.

Quest
ion:

29. A52- year


-oldmani sseenincl
ini
cforeval
uationofhypert
ension(
HTN)diagnosed1
yearago.Hei sonnomedi cat
ions.Hi
sfather
,mother,andtwosibl
ingshaveHTN.Hehasbeen
fol
lowingtheDiet
aryApproachestoStop Hypert
snsion(DASH)dietandexer
cisi
ng4timesper
weekf ort
helastyear.Hehasbeencheckinghisbloodpressur
e(BP)athomef ort
helast6
mont hsandsaysitis“hi
gh.” 

Hi
sBP 
i
ncl
ini
cis162/
92mm Hg,
inbot
har
ms,
andhear
trat
eis72bpm.Hi
scar
diovascul
ar
exam revealsregul
arratewithanS4, butnomur murs.Pul
sesaresymmet
ri
candequal
thr
oughoutandwi t
houtedema.Hi sel ect
rocar
diogr
am (ECG)showssi
nusrhyt
hm wi
thvol
tage
cri
ter
iaforleftvent
ri
cularhypert
rophy( LVH).

Whi
choft
hef
oll
owi
ngi
sthenextmostappr
opr
iat
est
epi
ntr
eat
ment
?

a. St
artchl
orthali
done25mgdai l
y.
b. Per
form 24-hourambulat
oryBPmonit
ori
ng.
c. I
nit
iat
emet oprol
olsucci
nate50mgdail
y.
d. Per
form arenalul
tr
asound.
e. Pr
escri
beast ri
ctsodi
um rest
ri
cti
on(
<500mgperday)
.

Answer
:

29. a. St
artchl
ort
hal
idone25mgdai
ly.

Init
ialhypertensivetreat
mentshoul dincludeat hiazi
dediureti
c,calcium- channelblocker
,
angiotensin-conver t
ingenzymei nhibi
tor,orangiotensi
n-receptorblockeri nthegeneralnon-
blackpopul ati
on, whilei
nt hegeneralblackpopulat i
on,
itshouldbeat hiazidediur
eticor
calcium channelbl ocker
.IfthetargetBP isnotreachedwi thi
n1mont hafteri
nitat
ingtherapy,
thedoseoft heinit
almedi cati
onshouldbei ncreasedorasecondmedi cati
onshoul dbeadded. 

Thispat i
ent'
sphysicalexaminati
onr evealedevidenceofl ong-st
andingHTN  (
S4,LVHonECG) ,
sot heexpenseofa24- hourambulatory BP monitortoverif
yHTN  i
snotj usti
fi
ed.Thereisno
evidencet osuggestthatsuchastrictsodium restri
cti
on( <500mgperday)i sbenefici
ali
nthi
s
setting.I
fthepati
enthasanunusual l
ysever eandsuddenonsetofHTN  orevidenceof
resistantHTN af
terini
ti
atingpharmacol ogictr
eatment ,
 furt
hereval
uationf orsecondaryHTN,
i
ncl udingarenalult
rasound,wouldbeappr opri
ate.
Val
vul
arDi
sease
Quest
ion:

1. A38-year
-oldAsianwomani sref
er r
edtoyouf orthef
ir
stti
mebecauseofper ipheral
edema.Shemovedt ot
heUni tedStatesfrom China12yearsagoandunder wentami tr
alval
ve
replacementforrheumaticmitralst
enosis.Atthetime,shehadmildtr
icuspi
dst enosis(TS).
Shehasdonewel lunti
labout6mont hsago, whenshebegant onot
icesomer ecurrentbrief
palpit
ati
onsandsomebi lat
erallegedema.

Herexami nati
onrevealsevi
dencef orrightheartfail
urewit
hanelevatedjugul arvenous
pressureandapr ominentAwavenot ed.Shehasal oudgrade3/4diastoli
cr umbleatthef i
fth
i
ntercostalspaceatthelef
tsternalborderthatincreaseswithi
nspi
ration.Mechani calopening
andclosingsoundsf rom t
hemi tral
valveareevi dent.Shehasanenlargedliverabouttwo
fi
ngersbelowt hecostalmargin,butnoasci t
es.Ther eis2+edematoj ustabovet heankles.

Anecho/ Doppl errevealsevidenceforsevereTSandt ri


vialt
ri
cuspidregurgi
tat
ion.Themean
tr
icuspidgradientisest i
matedat12mm Hg, andthetri
cuspidpressurehalf
-t
imei s240msec.
Ther i
ghtatri
um i squiteenlarged.Rightventr
icul
arfunct
ionappearstobenor mal,andthe
mechani calmitralvalveisfuncti
oningwel l
.Thereismini
malaor t
icvalvethi
ckening.

Youini
ti
atemedicalt
herapybyaddi
ngtor
semide40mg/dayandspir
onolact
one25mg/dayto
hermedicalr
egi
men, whichi
ncl
udeswarf
ari
n.Youal
soobtai
na24-hourambulat
orymoni
tor
.

Sher etur
nsin4weekswi t
hnoedemabutst
il
lwi
thdyspneaonexer
ti
on.Her24-
hourmoni
tor
revealsparoxysmalat
ri
alt
achyar
rhyt
hmi
as.

Whi
choft
hef
oll
owi
ngi
sthenextbestst
epi
nhermanagement
?

a. Surgicalt
ri
cuspidval
ve(TV)replacement
.
b. Digoxin.
c. I
ncreasedi ur
eti
cdose.
d. Percutaneousball
oontr
icuspi
dval vul
opl
ast
y.
e. Surgicalt
ri
cuspidval
vul
oplasty.

Answer
:

1. a. Sur
gicalt
ri
cuspi
dval
ve(
TV)r
epl
acement
.

Thecorr
ectansweri ssur gi
caltri
cuspidvalvereplacement.Thepati
entwi t
honlymildto
moderateTVst enosismayi mprovewi t
hdi ur
esis.Furosemideispoorlyabsorbedwhenthere
i
sboweledema, andal t
er nati
vessuchast orsemideorbumet anide,
bot hofwhichhavebett
er
bowelabsorpti
on, shouldbeusedi nst
eadf ormanypat ient
s(especi
allyiff
urosemi
deistri
ed
andnotfoundef fecti
ve).Overt i
me, di
uret
icsmaynotbeabl etomaintainadequate
hemodynami csast hepr oblem i
smechani calobst r
ucti
onattheTVl evel.
 

Thepati
entpresentedher
ehassever
eTSbyal lpar
ameters.The2014AmericanHear
t
Associ
ati
on/Amer i
canColl
egeofCar
diol
ogyvalvul
arhear
tdiseasegui
del
inedefi
nessever
e
TSasdemonst rati
ngapr essurehalf
-t
ime>190msecandval vear ea<1.0cm2.Neithersurgi
cal
norpercut
aneousval vuloplast
yhasbeenpr ovenveryeffect
iveforTS, asbothprocedures
oft
enleadtowor seningt r
icuspi
dregurgi
tat
ion.Tri
cuspidvalvereplacementisrecommended
whensympt omsper sistdespiteadequat
emedi calmanagement .Digoxincanbeusedi n
sedent
arypati
entsf orratecontroli
natri
alf
ibri
ll
ati
on,buthaslit
tlerolehere.

Quest
ion:

2. A34-year-
oldisadmitt
edwi thfeverandanewmur mur.Heisaknownsubst ance
abuser,
andhecl ai
mshehasf eltpoor
lyforsever
al mont
hs.Hehashada10- l
bweightloss
andrecurr
ingnightsweats.I
nt hepastfewweeks, hehasnot
edprogressivedyspneaandis
sl
eepingon2-3pi l
lows.Hehasnoedema.Hei sonnomedi cat
ionsatthisti
me. 

Hisexaminati
onr evealsafri
ght
enedyoungmanwi t
hevidenceofi ntr
avenousdr uguseon
botharms.Histemper atur
eis38.5°C,hear
trateis103bpm, bloodpr essureis95/ 50mm Hg,
andresti
ngsaturationis96%.Hislungsrevealbibasil
arfinerales.Hisjugularvenouspr essur
e
i
snotelevated,andt herei
snotri
cuspidregurgit
ation.Hisapexi shyperdynami c,andthereisa
grade1/2aorti
csyst oli
cmurmurandgr ade2/ 4diastoli
cmur mural ongthel ef
tsternalborder
.
AnS3 i
spresent,andanS1  i
ssoft
.Hehasnoedema.  

Youobtainbloodcultur
es,anelectr
ocardiogram (ECG),chestX-ray,
andanechocardi
ogram.
Theecho/ Doppl
erreveal
sahyper dynami cleftvent
ri
clewithevidencef
orsever
eaor t
ic
regur
git
ation(AR)andvegetat
ionsconsi stentwithendocardit
isonhisaor
ti
cvalve.Oneofthe
i
magesf rom theechocardi
ogr
am i sshowni nFi gur
e1.

Basedont
hepr
ovi
dedi
nfor
mat
ionandhi
scur
rentcl
ini
calst
atus,
youbegi
nempi
ri
cant
ibi
oti
cs.

Whi
choft
hef
oll
owi
ngi
sthenextappr
opr
iat
est
epi
nhi
smanagement
?
a. St
artint
ravenousaf terl
oadreductionwi t
hanangiotensi
n-conver
ti
ngenzymei
nhi
bit
or.
b. Ur
gentsur gi
calconsul t
ati
onf oraorti
cvalverepl
acement.
c. St
artabeta-blockertoreducet achycardi
a.
d. Schedul
eat ransesophageal echocardiogram (
TEE).
e. Ur
gentcardiaccat heter
izat
ion.

Answer
:

2. b. Ur
gentsur
gicalconsul
tat
ionf
oraor
ti
cval
ver
epl
acement
.

Thepat ienthasobvi ousi nfecti


veendocar diti
sinvolvi
ngt heaor ti
cval ve.Heisincongesti
ve
heartfailure(CHF),tachycar di
c, andf ebril
e.Hisechocar diographyr evealsprecl
osureofthe
mi t
ralvalveduet other apidlyrisingleftvent r
iculardi
astolicpressureasar esul
tofhissevere
AR.Thi sal soresult
sint hesof tS1 obser ved.Asopposedt ochr onicAR, thediff
erence
betweent heaorti
candl ef tventri
cularpr essuresi ndi
astolemaybesmal li
nacuteAR, and
theremaybel i
tt
lediastoli
cmur mur .Likewiset hepulsepr essuremaynotbewi de,andther
e
maybenoneoft heclassichemodynami cfindingsofchr onicAR.Schemat ichemodynami c
traci
ngsofchr onicversusacut eARar eshowni nFigure2.  

Inf
ecti
veendocar dit
ispatient
swi t
hmi t
ralval
vepreclosureandCHFshoul dbeconsideredfor
eit
herurgentorearlysurgery.Theyareatriskforsuddentotallossofaort
icval
vecompet ence
andcatastr
ophicandl et
halheartfai
l
ure.Heneedst obeconsi deredforur
gent/
emergency
surger
y.Furtherstudiesandt her
apyshouldnotdelayhi ssurgery.Hedoesnotneeda
cathet
eri
zati
onathi sageandwi thhist
enuoushemodynami cs. 
ATEEwoul dbeast andar dpartoft hi
ssur gi
calint
ervent
ion,andwhi l
eitisindicatedin
pati
entswithaorti
cval veendocar diti
st obet t
erassesstheveget at
ionsizesandt ohelpdefine
thepresenceofanaor ticabscess, thepr ocedureshouldnotdel aysurgicali
nterventi
on.There
i
sl i
tt
lerol
eforafter
loadr educti
oni nt hisacut ephasewithhislowsyst emicbloodpr essur
e.
Histachycardi
aisappr opriateforhisser ioushemodynami cst ate,
andbet a-
blockersli
kewise
wouldnotbeappr opriate.Inadditi
on, ther educedheartrat
ef r
om abet a-bl
ockerwoul d
i
ncreasediastol
icti
meandt hedur ati
onofARperbeat .

(
Figur
e2)

Quest
ion:

3. A22-year-
oldwomani sreferr
edtoyouf oradviceregardinganti
coagul at
ion
managementi fshegetspregnant
.Shehasahi st
oryofendocar dit
isi
nvolvingbot hhermitr
al
andaorti
cvalves3yearsago.Shehadanaor ti
cvalver epairandamechani calmi t
ralval
ve
repl
acementatthatti
me.Shehasbeenonwar far
insincet hen.Shehasclinicall
ydonewel l
andisasympt omati
c.Sheisgetti
ngmar r
iedthisyearandwant stohavechi ldr
en.Her
examinat
ionisbenignexceptfortr
ivi
alaort
icregurgi
tationandt hemechani calmitr
alval
ve
sounds.

Hermedicati
onsincl
udebir
thcontr
ol pi
ll
s,aspi
ri
n81mg, andwarfar
in4mgeacheveni
ng.She
hasbeenreli
abl
yf ol
lowi
ngheri
nternati
onalnormal
izedr
ati
os,andtheval
uesr
angefr
om 2.
5
to3.
0mont hl
y.
Basedoncurr
entgui
del
ines,
whichofthef
oll
owingmedicalopt
ionswoul
dyour
ecommendt
o
heri
nregar
dtotheuseofanti
coagul
ati
onduri
ngpregnancy?

a. Becauseoftheri
sksi
nvolved,permanentster
il
izat
iontopreventapr
egnancy.
b. Weight-
basedlowmolecularweighthepar
in(LMWH)f ortheenti
repr
egnancyandt
hen
changetoUFHj ustpri
ortodeli
very.
c. We i
ght-
basedLMWHf orthefi
rsttri
mest er
,warfar
infort
hesecondandthi
rdtri
mester,
andthenUFHj ustpri
ortodeli
very.
d. We i
ght-
basedUFHdur ingtheentir
epr egnancy.
e. Warfar
intheenti
repregnancyuntilti
mef ordeli
veryandthenchanget
ounfract
ionat
ed
hepari
n(UFH).

Answer
:

3. e. Warfar
intheenti
repr
egnancyunt
ilt
imef
ordel
iver
yandt
henchanget
o
unf
racti
onatedhepar
in(
UFH).

Thecor r
ectansweri sthatshemaycont i
nuetouseherl ow-dosewarfari
nf ort
heentir
e
pregnancyandchanget oUFH( withactivat
edparti
althromboplasti
ntimeofmor ethan
twofol
dcont rol
)justpri
ortovaginaldeli
very.Thi
sisbecauset heembr yopathyfr
om war f
arin
appearstobedosedependent .I
tisnowaCl assI(LevelofEvidenceB)recommendat ionthat
thosepatient
son≤5mg/ dayofwar f
ari
nmaysaf elyremainonwar f
ari
nt heenti
repregnancy,
changingt oUFHj ustpri
ortodel
ivery. 

Theot heropt i
onsmaybeconsi deredi ft
hedailydoseofwar f
ari
nis>5mg/ day.Inthat
sit
uat ion, t
hewar far
inshouldbedi sconti
nuedf orthefi
rsttri
mesterandei therUFH( wi
th
measur edacti
vatedparti
althrombopl ast
intimeofmor et hantwofol
dcont r
ol)orLMWH( wit
h
measur edanti
-Xaof0.8-1.
2U/ ml4-6hour spostdose)maybeused.Wei ght -
baseddosi ngof
UFHandLMWHdoesnotpr ovideadequat eanti
coagulanteffecti
nmanycasesbecauseoft he
i
ncr easedvol umeofdi stri
buti
ont hatoccur sduri
ngpregnancy.War fari
nt hencanbe
reinstitutedforthesecondandt hi
rdtrimesterandstoppedj ustpr
iortodel i
veryandr eplaced
withUFHasment ionedearli
er.Ifsheisotherwisehealthy,shewouldnotneedt oconsider
steril
izat i
on.

Quest
ion:

4. Rheumatichear tdi
seaser emainsamaj orprobl
em i
nt hedevel
opingworld.Acut
e
rheumati
cfevercontinuestopl aguemuchofAf r
icaandAsia.Iti
sesti
mat edt
hat60%of
pati
entswit
hacut erheumat i
cf everdevelopcarditi
sandval
vul i
ti
s,andchroni
crheumatic
heartdi
seasemayf ollow.Mitralvalvestenosi
s( MS)isthemostcommonoft hesechronic
l
esionsandcanhaveamaj orimpactonsur vi
val.

Withouti
nter
venti
on,whichoft
hef
oll
owi
ngi
stheest
imat
ed5-
yearsur
vivalr
atef
rom t
heonset
ofsymptomsi npati
ent
swi t
hMS?

a. 10%.
b. 75%.
c. 25%.
d. 50%.
e. 95%.

Answer
:

4. d. 50%.

Earl
yst udi
esofMSsuggest eda10-yearmortal
it
yrateof33-70%wi t
ha20- yearmortali
tyrat
e
of80-87%.Inthesehistori
calst
udies,t
herewasani nci
denceofhear tf
ail
ureinabout60%and
embol i
cevent
si n20%.I na1991review,itwasnotedthati
npat i
entswhor ef
useintervent
ion,
the5-yearsur
vivalrat
ewasabout50%.I ndevel
opingcountr
ies,t
het i
mef r
om theinit
ialbout
ofrheumaticfeverandthedevelopmentofsympt omaticMSi sgeneral
lyaboutthr
eet ofour
decades. 

Quest
ion:

5. Percut
aneousbal l
oonMVcommi ssurotomy(orvalvul
oplasty)hasbeenpr oventobe
asef f
ectiveassurgi
calcommi ssurotomyi nthetreat
mentofr heumat i
cmi tr
alst
enosis.To
assesst hesuit
abil
it
yforballoonval vul
oplasty,oneofthemostcommonl yusedcrit
eri
aarethe
Massachuset tsGeneralHospital(MGH)cr iter
iathati
nvolvegradingcertainfeat
uresoftheMV
andaddi ngtheresult
stodet er
mi neanMVscor e.Fourfeatur
esar eincl
udedi nt
hisscori
ng
system, andeacharegr aded1- 4.

Whi
choft
hef
oll
owi
ngf
eat
uresofr
heumat
icMVi
sNOTi
ncl
udedi
ntheMGHscor
ingsyst
em?

a. Submitralchordalthi
ckeni
ngandscar
.
b. MVmobi li
ty.
c. MVc al
ci um.
d. MVt hi
ckeni ng.
e. Mit
ralregur gi
tat
ion(MR).

Answer
:

5. e. Mi
tr
alr
egur
git
ati
on(
MR)
.

Theonefeat
urenotincl
udedi nt
hemi t
ralscor
ingsyst
em isMR.Whi lei
tisacceptedt
hatmore
than2+MReliminat
est hepossi
bil
it
yfortheball
oonproceduretobeef f
ective,t
hedegreeof
MRwasnotpar toft
hecr i
ter
ia.Atot
alscoreof<8foreachoftheot herfourfeat
ureshasbeen
foundt
oresul
tinthehighestrat
eofsuccessfrom t
heballoonvalvulopl
astyprocedur
e. 

Cli
nicalf
actorssuchasage, NewYor kHear tAssoci
ationclass,
andthepr esenceorabsence
ofatri
alfi
bril
lat
ionalsohavebeenshownt obepredicti
veofsuccessfulout comes,ashasthe
fi
nalMVar eaandgr adientaft
erthepr ocedure.I
npat i
entswhohaveunder gonesurgical
commi ssurotomy,balloonvalvulopl
ast ycanstil
lbeeffecti
veusingt
hesameanat omic
feat
urestohel pguideappr opri
ateness. 

Quest
ion:

6. Mit
ralr
egur
git
ati
on(MR)r
esul
tswhenther
eisl
ossofintegr
it
yoft
hemitr
alappar
atus.
Broadl
y,t
hecausesoftheMRaredi
ff
erent
iat
edastowhet
hertheyarepr
imar
yorsecondaryi
n
natur
e.
Whichofthef oll
owinganat omi cf eatur
esfavor
ssecondar
ymi
tr
alval
ve(
MV)di
seaseas
opposedtochr onicprimaryMVdi sease?
a. Restr
ictedleafletmobi li
ty.
b. Abnormal mi t
ralchor dallength.
c. Calci
um bui l
d-upint heleaf l
et.
d. Excessivemi t
ralvalvulart i
ssue.
e. MVe ndocar di
tis.

Answer
:

6. a. Rest
ri
ctedl
eaf
letmobi
li
ty.

Inchr onicsecondaryMR, t
heMVi susual l
ynormal ,buttheremaybesever el ef
tvent ri
cular
(LV)syst ol
icdysfuncti
onandmal alignmentoft hepapi l
larymuscles.Thisr esult
si nabnor mal
tetheringoftheMVwi t
hr estri
ctedl eafletmobil
ity(orprolapse),
poorleafletcoapt ati
on,and
subsequentMR.Annul ardil
atati
onmayal socont r
ibute.TheCar pent
ierclassifi
cationis
somet imesusedt or ef
lectthis.TypeIi sduet oannul ardilat
ati
on.TypeI Irefl
ectsatl eastone
l
eaf l
etpr ol
apsing.TypeIIIaresultswhent herei
sr estri
ctedleafl
etmobi l
ityinbothsyst oleand
diastole.TypeIII
boccur swhent herei srest
ri
ctedleaf l
etmobi l
it
yonlyinsyst ole.

I
nprimaryMVdi sease,
ther
eisanabnormali
tyi
ntheval
veitsel
fori
nsomecomponentofthe
mit
ralapparatus.Themitr
alappar
atuscanbedefi
nedbyeachoftheval
vecomponent
sthat
f
uncti
ont opreventMR.Theseincl
udetheannul
us,thel
eaf
lets,
thechor
dae,
andthepapi
ll
ary
muscles.
 

Adescr i
pti
onofhowt hesest ructureswor ktogethertopr eventMRi sr el
evant .Itisi
mpor tant
tonotethatbothmi tr
alleafl
et sattachviachor daet obot hpapil
larymuscl es.WhenLV
contr
actionbegins,bothpapi ll
arymuscl escont ractandpul ltheleafletstowar deachot her.As
contr
actioncontinues,t
hel eafletsabutagainsteachot her,andt heLVpr essur eoneachsi de
oftheleafl
etstendtohol dthem t ogether.Thisisreferredtoast hekeyst oneef fectasitis
simil
artothemi ddlekeystoneatt hetopofanar chway.Wi thcont inuingvent ri
cular
contr
action,t
hepapi l
lar
ymuscl escont i
nuet opul landt heannuluscont r
act s.Anyabnor mal i
ty
i
nt hepapill
arymuscl econtraction,anyabnor mal chordallength, anyexcessi veMVt issue, or
anyabnor mali
tyinannul arsizeorcont ri
butionmayr esultinMR.  

Themostcommoncauseofpr i
maryordegener ati
veMRi sMVpr ol
apsefoll
owedbyot her
s,
suchasinf
ecti
veendocardi
ti
s,connect
ivetissuediseases,rheumat
icheartdi
sease,cl
eftMV,
cal
cif
ici
nfi
lt
rat
ion,
andradiat
ionheartdisease.Obviousl
y,thesurgi
calt
reatmentdif
fer
s
dependi
ngonwhethertheMRi sprimaryorsecondar yinnature.

Quest
ion:

7. Youhavebeenf oll
owinga52-year-ol
dwomanf ormitr
alprolapseformanyyear s.She
hasalwaysbeenasympt omat i
candonexami nationhashadmul ti
plemi tr
alcl
ickswit
hasof t
mitr
alinsuf
fi
ciencymur mur.Peri
odi
cecho/ Dopplerstudieshaveconf ir
medmi tr
alval
ve
prol
apse,mil
dmi tr
alregur
gitat
ion(MR),andapr eservedleftvent
ricularej
ecti
onfract
ionwith
normalchambersizes.

Whi
legar
deni
ng,
shenot
icessudden,
shar
pchestpai
nandt
hent
her
api
donsetofshor
tness
ofbreathandtachycar
dia.Shei
sfri
ghtenedandcall
s911.Sheistr
ansport
edtot
he
emergencydepartmentwheresheisinflor
idpul
monaryedema.Sheistr
eatedwi
th
i
ntravenous(I
V)diuret
icsandqui
cklystabi
li
zesbutr
emainsdyspnei
c.

Bythetimeyouseeher ,shehasahear trateof110bpm, bloodpressureof90/60mm Hg,and


room airsaturat
ionof90%.Shei stachypneicsi
tt
ingboltupright
.Thereareral
esallt
hewayup
onbothsi desofherlungsandagr ade2/6holosyst
oli
cmur murwithanassociat
edS3.She
i
mpr ovesqui ckl
ywithIVfurosemidetherapy.

Thef
ourchambervi
ewsofherecho/
Doppl
erst
udyar
eshowni
nVi
deos1and2.

Sheunder
goescar
diaccat
het
eri
zat
ionwi
tht
hef
oll
owi
ngper
ti
nentf
indi
ngs:

 Normalcoronaries.Ri
ghtatri
alpressur
emeanof10mm Hg.
 Rightventr
icul
arpressure65/35mm Hg( meanof45mm Hg)
.
 Pulmonarycapill
arywedgepr essureVwaveof40mm Hg(meanof33mm Hg)
.
 Aorti
c100/ 80mm Hg.
 Leftvent
ri
cularpressure100/29mm Hg.
 Cardiacoutput3.2L/min.
 Cardiaci
ndex1. 8L/min/m2.

Basedont
hesedat
a,whi
choft
hef
oll
owi
ngi
sthenextbestst
epi
nhermanagement
?

 
htt
p:/
/medi a.
mycrowdwisdom.com.
s3.amazonaws.
com/
acc/
quest
ions/
asset
_val
ve/
a1ea56f
4-
9ae0-436d-b015-
96ecaf
378850.mp4

ht
tp:
//media.
mycrowdwisdom.com.
s3.
amazonaws.
com/
acc/
quest
ions/
asset
_val
ve/
23f
de7df
-
eb2a-46d5-
bcb9-
cfcb0cb7762c.
mp4

a. I
Vdopami ne5mcg/ kg/mi n.
b. Urgentsurgicalmit
ralvalverepair
/repl
acement
.
c. Hepari
nbol usandinfusion.
d. Ni
troprussideinf
usion.
e. I
ntra-
aorti
cbal l
oonpumpi nsert
ion.

Answer
:

7. b. Ur
gentsur
gicalmi
tr
alval
ver
epai
r/r
epl
acement
.

Thecor rectansweri sur gentsur gicalinterventi


on.Whi lecont i
nueddi uresi
sandt heuseof
after
loadr educti
ont her apy,suchasi nt ravenousar teri
alvasodi latorsorani ntra-aorticbal loon
pump, mayhel pthehemodynami cstransi ent
ly,asmi ghtani not ropesuchasdopami ne, she
needsur gentmi t
ralval ver epairorr eplacement .Shehasal readyst abi
li
zedusi ngI Vdi uretics,
andherhemodynami csandechocar diogramsr eflectthatshehasacut emi tralchor dalr upt ure.
Asopposedt ochr onicsever eMR, herl eftatr
ium andl eftventricleareunpr epar edf ort he
suddeni ncreaseinpr eloadr esulti
ngf rom t heacuter egurgit
ation.Herpul monar yhyper t
ensi on
i
ssecondar ytoherhi ghpul monar ycapi ll
arywedgepr essure,andt hereisnosuggest iont hati t
i
sr el
atedt opulmonar yembol i,ashert ranspul monar ygradienti snor mal.Shet her eforedoes
notneedhepar i
nt herapyf orthatpossi bi l
it
y.Thefactt hatherMRi sonlygrade2/ 6r eflectst he
highleftatri
alpressur ei ntheacut eset ti
ng.Thehemodynami cdi ffer
encebet weenchr oni cMR
andacut
eMRi
sschemat
ical
lyshowni
nFi
gur
e1.

(
Figur
e1)

Quest
ion:

8. A40-year-oldwomanpr esentswithincreasingdyspneaovert hepast6mont hs.On


exami nati
on,shehasabl oodpr essur
eof130/ 84mm Hg, heartrateof78bpm wi thincreased
j
ugul arvenouspr essureto10cm, fewbibasil
arrales,ear
lysystoli
ccl ickwit
hagr ade3/ 6mi d
tolatesystoli
cmur murbesthear dattheapex, andnoper i
pheraledema.Echocar diography
showsl eftventr
iculareject
ionfracti
on(LVEF)of75%, wit
hmoder atelyenlar
gedLVwi th
mar kedprolapseoft heposteri
ormi tr
alvalveleafl
etwithseveremi tralregurgi
tat
ion( MR)
extendingintotwopul monar yveins.Herpulmonar yarter
ypeaksyst olicpr
essureis45mm Hg.

Whi
choft
hef
oll
owi
ngdoyour
ecommendi
sassoci
atedwi
thi
ncr
easedl
ongevi
ty?

a. Li
sinopr i
l.
b. Mitralvalver epai
r.
c. Diureti
ct herapy.
d. Carvedilol.
e. Bosent an.
Answer
:

8. b. Mi
tr
alval
ver
epai
r.

Mi t
ralvalvesurger y,pr
eferablywithr
epair,isaClassIrecommendationforsympt omati
c
patientswithchr onicseverepr i
maryMRandLVEF>30%.Vasodi lat
ortherapyinnormotensi
ve
patientswithchr onicpri
mar yMRhasnotbeenshownt obeofsigni
ficantbenefi
t.Whil
e
diureti
csar ehel
pf ulintreati
ngcongestivesympt omsofheartfai
lur
e, t
heyhavenotbeen
shownt oimprovel ongevity.Thispat
ient'
spulmonaryhypertensi
onissecondaryt oher
valvularheartdi
sease, andt heendotheli
nr ecept
orantagoni
stbosentanwoul dnotbe
appr opri
atetherapy.

Quest
ion:

9. A64-year-ol
dwomanwi thahi st
oryofnonischemi ccar di
omyopat hy,
completeheart
block,andabi ventr
iculari
mpl antabl
ecar di
overt
er-defi
bri
ll
atorpl aced4yearsagoisadmi t
ted
withrecurr
entf ever
sandbact eremiawi t
hmet hi
cil
lin-
sensit
iveSt aphyl
ococcusaureus.Sheis
tr
eatedwi t
hint r
avenous( I
V)naf cil
li
nandgent amicinwithclinicalimpr
ovement.
Transesophagealechocar diography( TEE)doesnotdemonst rateanyevidenceofendocardit
is.
After3days, r
epeatbl oodculturesshowper sist
entbacteremi a.

Whi
choft
hef
oll
owi
ngi
sthemostappr
opr
iat
enextst
ep?

a. Complete6-
weeksofIVant i
bioti
csfol
lowedbychr onicanti
biot
icsforsuppr
essi
on.
b. Ext
racti
onofherbi
ventri
cularimplant
ablecar
dioverter-
defi
bri
ll
atorsyst
em.
c. Complete6weeksofIVant i
bioti
csfol
lowedbyrepeatbl oodcult
ures.
d. RepeatTEE.
e. Taggedwhit
ebloodcellscan.

Answer
:

9. b. Ext
ract
ionofherbi
vent
ri
cul
ari
mpl
ant
abl
ecar
diover
ter
-def
ibr
il
lat
orsyst
em.

Thispatientispresent i
ngwithpersist
entbacteremiawithSt aphylococcus,andintheabsence
ofaclearalternativesource,t
hereforethei
mpl anteddevicemustbepr esumedt obethe
sourceofongoi ngi nf
ection,
despi t
eanegativeTEE.Wi t
hper si
stentbacter
emia,theonly
vi
ablest r
ategyfordef ini
ti
veresoluti
onisextract
ionoftheimpl anteddevice.Thi
spatienthas
compl et
ehear tblockandi spacemakerdependentandt hereforewillr
equireatemporary
pacingsolutionunt i
lthebacteremiahasclearedandanewper manentdevi cecanbe
i
mpl anted. 

RepeatTEEatt hi
spoi ntwi l
lnotl i
kel
ychangemanagementandi snotver ysensi t
iveor
specifi
cf ordiagnosingveget at
ionsondevi celeads.About30%ofpat ient
swi t
hi mpl ant
ed
devi
cel eadsmayhavemobi l
et hrombithataregenerall
ynotinfected.Taggedwhi tebloodcel
l
scansorposi tronemi ssi ontomogr aphyscanshavel owr esol
utionandwi l
lagainnotchange
managementi nthispat i
ent.Att hi
spoint,t
hebact er
emiaispersistentdespitesever aldaysof
appropriateant i
bioti
ct herapy,andsimpl yconti
nuingorchangingant ibiot
icswillbeunl i
kel
yto
achievest er
il
izati
on. 
Quest
ion:

10. A70- year-


oldmanishospit
ali
zedwithsubacutebacteri
alendocardi
ti
s.Bloodcultur
es
growSt r
eptococcusbovi
s.Echocardi
ographydemonst rat
esa3mm mobi levegetat
iononthe
anteri
orleafl
etofthemit
ralval
vewithmildmit
ralregurgit
ati
onandnoot hervalvul
arlesi
ons.
Leftvent
r i
cularej
ecti
onf
racti
onis65%.Examinati
oni sunremarkable.

I
naddi
ti
ont
oant
ibi
oti
cs,
whi
choft
hef
oll
owi
ngi
smostappr
opr
iat
eforhi
smanagement
?

a. Surgi
calconsul
t.
b. Uri
nalysi
s.
c. Col
onoscopy.
d. DentalX-r
ays.
e. Coronaryangi
ography.

Answer
:

10. c. Col
onoscopy.

Strept
ococcusbovi sendocardit
isisoft
enassoci atedwit
hmal i
gnancyofthegastr
ointest
inal
tract
,socolonoscopyt oevaluatet
hispossibil
it
yshouldbeconsi der
ed.Thesmallvegetat
ion
andmi ldmitralr
egurgit
ati
ondonotr equir
esur gi
cal i
nter
venti
on,sotherei
salsonoindicati
on
forcoronaryangiographyatthisti
me.Streptococcusbovisisnottypi
call
yassoci
atedwi t
h
dentalcari
es.Urinal
ysistolookforuri
narytractinvol
vementwoul dbeindi
cat
edfor
Staphylococcali
nfecti
onsbutnotf orStr
eptococcusbovis.

Quest
ion:

11. A54- year-


oldmanpr esentsforr out
ineevaluat
ionforknownchr onicaorti
cvalve
regur
git
ati
onduet obicuspi
daor ti
cval vemorphology.Heremai nsasympt omati
c.Physical
examinati
onreveal
sahear trat
eof84bpm andbl oodpressureof140/ 50mm Hg.Physi cal
examinati
onreveal
sagr ade2/6syst olicej
ecti
onmur mural ongt helef
tsternalborderanda
grade3/4diast
oli
cflowmur mural ongt hesamear ea.Echocardiogram reveal
sadi lat
edlef
t
ventr
icl
ewithsevereaort
icregurgit
ation.

Whichofthefol
lowi
ngechocar
diogr
aphi
cpar
amet
erswoul
dbeani
ndi
cat
ionf
orsur
gical
aor
ticval
verepl
acementatt
histi
me?

a. LVejecti
onf racti
on55%.
b. Venacont ract awidth0.7cm.
c. Aort
icregur gitantfract
ion65%.
d. Lef
tventricular( LV)end-syst
oli
cdi
mensi
on5.
3cm.
e. LVend-diast olicdimension6.0cm.
Answer
:

11. d. Lef
tvent
ri
cul
ar(
LV)end-
syst
oli
cdi
mensi
on5.
3cm.

Basedont he2014Amer i
canCollegeofCardiol
ogy/AmericanHeartAssociat
ionvalvularheart
di
seasegui del
ine,t
hepati
enthasevidenceforsevereaor
ticval
veregurgi
tati
on.Surgicalrepai
r
i
sindicat
edi nanasymptomaticpati
entwhent hereisevi
denceforLVdysfunction,
LVej ection
fr
acti
on5.0cm, LVend-
diast
oli
cdiameter>6.5cm, orwhenundergoingothercardi
acsur gery.
 

Quest
ion:

12. A32- year-oldwomanpr esentstocli


nicwithseveralmonthsofpr ogr essivedyspneaon
exert
ion,ascites,andlowerext remit
yedema.At r
ansthor
acicechocar di
ogr am demonst r
ates
anestimatedr ightventr
icular(RV)systol
icpressureof90mm Hg, RVenl argementand
dysfuncti
on, andseveret r
icuspidregurgi
tat
ionwi t
havenacont ract
aof0. 8cm.She
undergoesr i
ght -
heartcatheter
izati
onthatconfir
mst hediagnosi
sofsever epul monar yarter
ial
hypertensi
onwi t
hapul monar yvascularr
esistanceof12Wooduni ts.Leftvent r
icularend-
diast
olicpressureis10mm Hg.Hercar diacindexis1.7L/min/m2.Ther eisnor eacti
vit
yto
i
nhaledni t
ricoxide.

Whi
choft
hef
oll
owi
ngi
sthenextbestst
epi
nhermanagement
?

a. Diureti
c.
b. Pulmonar yvasodil
ator
.
c. Calcium channelbl
ocker.
d. Tri
cuspidvalvesurgery.
e. Digit
ali
s.

Answer
:

12. b. Pul
monar
yvasodi
lat
or.
Quest
ion:

13. A27- year-ol


dwomani sseeni ntheemergencydepartmentforseveraldaysof
progressi
vedyspnea, or
thopnea,paroxysmalnoctur
naldyspnea,andper i
pheraledema.She
hadasi milarpresentati
ontotheemer gencydepartment3weeksago, andwasgi vena
prescri
pti
onf or40mgoncedai lyoffurosemideandmet oprol
ol25mgt wicedail
y.Sheis25
weekspr egnant.Hervi t
alsi
gnsar eaheartrateof102bpm, bl
oodpr essure100/60mm Hg,
andoxygensat ur at
ion100%on2l it
ersnasalcannulaoxygen.
 

Physicalexami nationr evealsat achycardiaat88bpm.Ther earebilat


eralr
alest wo-thir
dsof
thewayupt hepost er
iorl ungfiel
ds.Therei sanelevatedj ugularvenouspressur etot heangle
ofthemandi bleupr ight, a1/6hol osyst
olicmur muratt heapexwi t
ha2/ 4diastolicmur mur,
andanopeni ngsnapi sal sopresent.Therei str
aceper ipheraledema.Echocar diogram reveal
s
normalleftventricularf uncti
onwi thrheumat icmit
ralstenosi sandameanmi t
ralvalve
gradientof16mm Hgwi thtri
vialmitr
alregurgit
ati
on.Theest imatedri
ghtventricularsystol
ic
pressureis50mm Hg.

Whi
choft
hef
oll
owi
ngi
sthebestnextst
epf
ormanagement
?

a. Del
iver ynowandt henmi t
ralval
verepl
acementsur gerysoonther
eaf
ter
.
b. Mit
r alvalver
eplacementsur ger
y.
c. Percutaneousmi t
ralcommi ssurot
omy(valvulopl
asty)
.
d. Cont i
nuecurrentobservat
ionalmanagement .
e. Percutaneousmi t
raledge-t
o-edgemitr
alvalverepai
r(Mitr
aCli
p).

Answer
:

13. c. Per
cut
aneousmi
tr
alcommi
ssur
otomy(
val
vul
opl
ast
y).

Percutaneousmi tr
alvalvecommi ssurotomy( val
vul
oplasty)isaCl assIIarecommendat ionin
theabsenceofsi gnif
icantmi t
ralregurgitat
ionandaf avorableWi l
kinsScorei nthe2014
AHA/ ACCval vul
ardiseasegui del
inesf orpregnantpati
entswi thClassIIIorIVheartf ail
ure
symptomsdespi t
emedi caltherapy.Del i
veryandthensur gerywoul dnotbeopt imal
managementatt hispoint
,unlessf oraspeci f
icobstetr
icindicati
on,duet oissuesrelatedto
prematurit
y.Hersympt omshavepr ogresseddespit
edi ur
eticandbet ablockade, t
huscur r
ent
medicaltherapywoul dnotbet hebestchoi ce.Percutaneousmi traledge-t
o-edgemi tralvalve
repai
r(MitraCli
p)isnotindicatedast hepat i
enthasmi tr
alstenosis,notmi t
ralregurgitat
ion. 

Quest
ion:

14. A26- year-


oldwomani sseeni nconsultati
onont heobst etr
icsinpati
entwar dfor
eval
uationofwor seni
ngdyspnea6hour safterdel i
veryofaf ull-
ter
m healthybabyboy.She
deni
eschestpai n.Herexami nationreveal
sherhear trateis105bpm, bl
oodpr essureis
100/60mm Hgandequali nbot hupperextremi ti
es,andj ugularvenouspr essureiselevated.
Sheist achycar
dicwit
har egul arrhyt
hm, agrade3/ 6latesystolicmur murwi t
hami d-syst
oli
c
cl
ickatt heapex,andral
es2/ 3oft heposteri
orl ungf i
eldsbilateral
ly.Oxygensat ur
ationis99%
on2l i
terssupplementaloxygen.Theel ectr
ocar diogram iswithoutanyi schemi cST-Twave
changes.
Whi
choft
hef
oll
owi
ngi
sthemostl
ikel
ydi
agnosi
s?

a. Mit
ralr
egurgit
ati
on.
b. Pul
monaryembol ism.
c. Aor
ticst
enosis.
d. Cor
onarydissect
ion.
e. Aor
ticdi
ssecti
on.

Answer
:

14. a. Mi
tr
alr
egur
git
ati
on.

Regur git
antval velesions, evenwhensever e,ar egener all
ywel ltolerateddur i
ngpr egnancy.
However , asi nthi
sscenar i
o, shortl
yaf t
erdeli
ver ywi t
hsi gni
ficantf lui
dshi f
tsfrom t he
placenta,asi gnifi
cantvol umel oadmayoccur ,leadingt oovertcongest i
vehear tfail
ur e.
Coronar ydi ssecti
onmayoccurdur ingpregnancyorshor tl
yaf terdel i
ver y,
butwi thoutchest
painorel ect r
ocardiographi cchanges, thisi
snott hebestanswer .Pulmonar yembol ism or
amni oti
cf luidembol ism alsomayoccurwi thpr egnancyoraf terdel i
ver y.However ,thi spati
ent
hasevi denceofr i
ght-andl eft
-sidedheartfail
ur eonexami nation, andl eft-
sidedfailur ewould
beabsentwi thapul monar yembol i
sm.Aor ti
cdi ssectioncanoccurdur ingpr egnancyor
i
mmedi atelypost partum int hecont extofMar fansyndr ome, Loeys- Di etzsyndr ome, or
coarctati
onoft heaor ta.Thescenar iopresentedi snotconsi stentwi t
haor ticdissect ion.Her
exami nationf i
ndingsar enotsuggest iveofaor ti
cstenosi s.

Quest
ion:

15. A60- year-ol


dmani shospital
izedforfurt
hereval uat
ionoffever,chi
ll
s, nightsweats,
andwei ghtl
oss.Bl oodcultur
esgrowmet hi
cil
li
n-sensit
iveStaphyl
ococcusaur eus.Hei s
startedonappr opri
ateanti
biot
iccoverage.Transesophagealechocar diogr
am r evealsnormal
l
ef tventri
cul
arf uncti
onwi thabicuspidaort
icvalvewi t
hmoder ateaor t
icvalver egurgi
tat
ion
andan8mm veget ati
on.Theotherheartvalvesarewi thoutevi
denceofveget at ionor
dysf uncti
on.Thepat ienthasanumberofquest i
onsr egardi
nghispr ognosiswi thendocardit
is,
i
ncl udinghisriskofdyi ng.

Whichofthefol
lowi
ngbestdescr
ibest
hei
n-hospi
tal
mor
tal
it
yforpat
ient
swi
thi
nfect
ive
endocar
dit
is?

a. 60%.
b. 80%.
c. 40%.
d. 20%.
e. 5%.

Answer
:

15. d. 20%.

Avarietyoff
actor
shavebeenassoci
atedwithevenhighermor t
ali
tyi
nselect
edpatients,such
asadvancedheartfai
lur
esympt
omsonpr esentati
onandnotunder goingval
vesurgery.A
mult
idisci
pli
nar
ytreat
mentappr
oachinvol
vingcardiol
ogy,i
nfecti
ondiseasespeci
ali
sts,and
car
diacsur
ger
ytoachi
eveopt
imal
out
comesi
srecommended.
 

Quest
ion:

16. A35- year


-ol
dmal eintravenousdrugabuserpr esentswit
hworseningshortnessof
breathanddyspneaonexer t
ion.Onexami nati
on,hi
sbl oodpressurei
s120/55mm Hg, heart
ratei
s95bpm, andr espi
rat
ionr at
eis20.Oxygensatur ati
onis93%onr i
ghtatri
um.Car di
ac
auscultati
ondemonst rat
esabl owingdecrescendodiastoli
cmur murhear
dovert he
precordium andalatediast
olicrumbleattheapexwithnoopeni ngsnap.S1 i
ssof t
,andS2 i
s
paradoxicall
yspl
it.

Whi
choft
hef
oll
owi
ngi
stheet
iol
ogyoft
hispat
ient
'
sshor
tnessofbr
eat
h?

a. Pulmonicregurgi
tati
on.
b. Tri
cuspidstenosis.
c. Mitr
alstenosis(MS).
d. Mitr
alregurgi
tati
on.
e. Aorti
cregurgi
tati
on(AR).

Answer
:

16. e. Aor
ti
cregur
git
ati
on(
AR)
.

Thephysi calexami nat


ionfindingsdescr i
bedar econsistentwi t
hAR;t heblowingdecr escendo
diastolicmur muri stheregurgitati
on, t
hepar adoxicalsplitS2 i
sduet odel ayedaor t
icvalve
closur efr
om t hei ncreasedleftvent r
icularvolume, andthel atediastolicr
umbl eisconsi st
ent
withanAust i
nFl intmur mur.Aust inFli
ntmur muri shear dint heset t
ingofsever eARandi s
thoughtt obeduet oear l
yclosur eofthemi t
ralvalveresulti
ngi nfunct i
onalMS.I tisfr
equent l
y
mi stakenf orvalvularMS, butthel ackofopeni ngsnapi sacl uethatt hisi
snott hecase.S1  i
s
sof tinARl ikel
yduet oear l
yclosur eofthemi t
ralvalvefrom theel evatedleftventri
cul
arend-
diastolicpressur e.Ingeneral,S1 invalvularMSi sloudunt i
lthevalvei sseverelyinvol
ved.

Quest
ion:

17. A75- year-oldmanpr esentswit


h3daysofchestpai nandwor seningdyspneaon
exert
ion.Onexami nati
on,hisbloodpressurei
s125/ 60mm Hg, heartrateis95bpm, and
j
ugularvenouspr essureis12cm.Ther eisaharshsystol
icmur murhear dt hr
oughoutthe
precordium wit
houti nspi
rator
yaugment ati
on.Attheapex,i
tisnotedt hatfoll
owinga
premat urevent
ricularcontr
acti
on,themur murbecomest r
ansientl
ylouder .An
el
ectrocardiogr
am showsQwavesi ntheinfer
iorl
eads.

Ift
hepati
entunder
wentahemodynami
ccar
diaccat
het
eri
zat
ion,
whi
choft
hef
oll
owi
ngwoul
d
befound?

a. Larger i
ghtvent
ri
cularoutf
lowt racttoPApressuregradient.
b. Increasedpulmonaryart
ery( PA)oxygensaturat
ion.
c. LargeVwaveonpul monarycapi ll
arywedgepressuretracing.
d. LargeVwaveonr ightat
ri
al pressuretr
aci
ng.
e. Largeleftvent
ri
cletoaort
agr adient.
Answer
:

17. e. Lar
gel
eftvent
ri
clet
oaor
tagr
adi
ent
.

Thepat i
enthasdegener ativeorage- rel
atedAS, themostcommonet i
ologyofAS.Onphysi cal
examinat i
on,hedemonst ratesGal l
avardinphenomenon—ahar shASmur murhear dbestatthe
apex.Thisisduet othehigh- f
requencycomponent soft heASmur murradi atingtot heleft
ventr
icularapex.I
talsocanbedi f
ferenti
atedf r
om MRbynot ingagapbet weenS1, themur mur,
andthenS2.TheMRmur murst artswi t
hS1 andcont inuest hr
oughS2 untilthemi tr
alvalve
opens.Themur murofASal soincreaseswi t
hbr adycardiaorafterapause, suchasaf t
era
prematureventri
cularcontr action;anMRmur murwoul dnotchange.Thi si snott ricuspid
regur
gitati
on,whichwoul dbehol osystoli
candbesthear datthelef
tlowerst ernalborder.Nor
i
st hi
spul monicstenosi
s, whi chwoul dtypical
lybestbehear dattheleftupperst ernalborder.
 

Bothoft heser ight-


sidedmur murswoul dbecomel ouderwithinspirati
on.Ifahemodynami c
cardiaccatheter i
zati
onwer eperformed, i
twoul ddemonst r
ateal argeleftventricl
etoaor t
a
pressuregr adient,consi
stentwithAS.Al argeVwaveonpul monar ycapi l
larywedgepr essure
i
sconsi stentwi thsignif
icantMR.Al argeVwaveont heri
ghtat r
ium isconsi stentwith
signifi
canttri
cuspi dregurgit
ati
on.IncreasedPAoxygensat urati
onsuggest sthepr esenceofa
l
ef t
-to-
ri
ghtshunt ,
suchasavent ri
cularseptaldefect.Aventri
cularsept aldefectmur muri s
typical
lycont i
nuousandhear datt hesternalborder,
nottheapex.Al arger i
ghtventricular
outflowtracttoPApr essuregradientisconsistentwithpul
moni cst enosis. 

Quest
ion:

18. A35- year-


oldwomanpr esent
sforshort
nessofbreath.Shehasahistor
yofdiabetes
andhypertension.Onphysicalexami
nati
on,herbloodpr
essureis140/85mm Hg, hear
trateis
88bpm, andbodymassi ndexis38.Herjugul
arvenouspressurei
snormal
, andshehas1- 2+
bi
lat
erall
owerext remit
yedema.

Sherecentlyunderwentanechocardi
ogram,whichwastechnical
lydif
fi
cul
t,butshowed
nor
malbi ventr
icul
arfunct
ion,mil
dconcentr
icl
eftvent
ri
cularhypert
rophy,
andmoder at
e
pul
moni cregurgit
ati
on.Ventr
icl
esarenormali
nsize.

Whi
choft
hef
oll
owi
ngi
sthenextbestst
ep?

a. Compr essi
onstockings.
b. Transesophagealechocar di
ogram.
c. Antibi
oti
cprophylaxisfordentalpr
ocedur
es.
d. Repeatechocardiogram in6mont hs.
e. Chestcomput edtomogr aphyangiogr
aphy.

Answer
:

18. a. Compr
essi
onst
ocki
ngs.

Thismor bidl
yobesepat
ienthasani nci
dentalfi
ndi
ngofmoderatepulmonaryregurgi
tation.
Thisfi
ndingiscommonandr equi
resnoaddi t
ional
workuporint
ervent
ion.Theref
ore,repeat
echocardiographyandt
ransesophagealechocardi
ogr
aphyarenotindi
cated.Pulmonar y
regurgit
ationdoesnotr equireanti
bioti
cprophyl axi
s.Othert hanshortnessofbr eat
h, t
he
patienthasnoot herknownr i
skfactorsforpulmonar yembol i
sm andisnottachycar dic,
thereforecomputedt omogr aphyangiographyi snotindicated.Herlowerextremi t
yedemai s
notsecondar ytoright-
sidedheartfail
ureint hesett
ingofanor malj
ugularvenouspr essure.
Heredemai ssecondar ytovenousi nsuff
iciencyintheset t
ingofmor bidobesity.Compr essi
on
stockingsareindicatediftheedemai sbot hersomet othepat ient

Quest
ion:

19. A35- year


-oldmanr ecentl
ymovedt otheUnit
edStatesandi sseenforshortnessof
breat
h.Aftermedi calopti
mizat
ionwithdiur
esisandbeta-
blockade,echocardi
ographyr eveal
s
thatt
hemi tr
alvalveleafl
etsandapparatusaresever
elycal
cifi
edandl eafl
etshavedecr eased
excursi
on.Themi tralval
veareais1.4cm2 wit
hameangr adientof12mm Hgandapr essure
half
-t
ime(PHT)of180msec.Thet ri
cuspidvalvei
salsonotablythi
ckenedwi t
haval vear eaof
0.9cm2,ameangr adientof7mm Hg, andaPHTof220msec.

Whi
choft
hef
oll
owi
ngi
sthenextbestst
epi
nthi
spat
ient
'smanagement
?

a. Mit
ralandt r
icuspi
dvalvesur
ger
y.
b. Mit
ralvalvesurger
y.
c. Mit
ralvalvul
oplast
y.
d. Tr
icuspidvalvesurger
y.
e. Mit
ralandt r
icuspi
dvalvul
opl
ast
y.

Answer
:

19. a. Mi
tr
alandt
ri
cuspi
dval
vesur
ger
y.

Thepat ienthassymptomat i
crheumaticvalvediseasewi thbot hseveremi t
ralandt r
icuspi
d
stenosis.Severemitr
alstenosisi
sdefinedasanabnor malvalvewi thaPHTof150msecand
aval veareaof1.5cm2.Mi t
ralval
vestenosisischar acter
izedasver ysevereifPHTi s220
msecandt hevalveareais1cm2.Sever etr
icuspidst enosisisdefinedasat hickenedvalve
2
withaPHTof190msecandaval veareaof1cm .Theonl ydefini
tivetreatmentf orrheumati
c
tr
icuspidst enosi
sissurgi
calrepl
acement.Dependi ngont heWi l
kinsscor e,t
hemi t
ralval
ve
mi ghtbemanagedwi t
hvalvulopl
astyorsurgery.

Quest
ion:

20. A66- year-


oldpati
entpresent
switht ypi
calangina,butnosympt omsofheartfai
lur
e.He
hasahist
oryofhypertension,
smoking,type2di abet
es, andhyper
li
pidemia.Healsohasa
str
ongfamil
yhi st
oryofcoronaryart
erydisease.Anucl earst
resst
estrevealsaposit
ive
el
ectr
ocardi
ogram withstr
ess,chestpai
n, andtransi
entischemicdil
atati
ononthestress
i
mage. 

Heundergoescar
diaccatheteri
zati
onandsignifi
cantthr
ee-
vesseldi
seaseisfound.His
echocar
diogr
am r
evealsanej ect
ionfr
act
ion(EF)of50%andevi denceforseveremit
ral
regur
git
ati
on(MR).Thereisanar eaofmi
ldleftventr
icul
arhypoki
nesi
alateral
ly.

Whi
choft
hef
oll
owi
ngmi
tr
alval
veechocar
diogr
aphi
cpar
amet
ersshoul
dpr
omptr
epai
roft
he
mit
ralval
vei
ntheset
ti
ngofconcomi
tantcor
onar
yar
ter
ybypassgr
aft
ing(
CABG)andanEFof
50%?

a. MRvenacontr
acta=0.4cm.
b. Mi
tr
alval
veeff
ecti
veregur
git
antor
if
i ERO)=42mm2.
ce(
c. MRregur
git
antvol
ume=38cc.
d. MRregur
git
antfr
acti
on=40%.

Answer
:

20. b. Mi
tr
alval
veef
fect
iver
egur
git
antor
if
i ERO)=42mm2.
ce(

Thecor rectanswerisalargemitralval
veEROar eaof>40mm2.Hemodynami call
ysevere
valvelesionsshouldundergorepairorrepl
acementatthet
imeofCABG, evenifpr
esumably
asympt omat i
c.Theli
keli
hoodofsympt omsr ef
erabl
etoseverel
esionswi t
hin2-3year
softhe
diagnosisishigh,andthereoperati
vecardi
acsurgerymort
ali
tyisgreateri
nthatti
mef r
ame
thanthemor t
ali
tyassoci
atedwithCABGcombi nedwit
hvalvesurgery.
 

Thedefinit
ionofsevereMRbyechocar diogr
aphiccri
teri
aincl
udesavenacontr
acta≥0.
7cm,
regurgi
tantvol
ume≥60cc, regur gi
tantfr
acti
on≥50%, orERO≥40mm2.Themi tr
alval
ve(ERO)
2
of42mm   isthustheonl
yr esponset hatdef
inessevereMRbyt hegui
del
ines,
andthisf
indi
ng
warrantsconcomitantmit
ralvalverepairwit
hCABG.

Quest
ion:

21. A74- year


-ol
dmanpr esent
stotheambulator
ycli
nicf
ortheeval
uationofdyspnea.He
i
saf ar
merwhosesymptomsbeganappr oxi
mately2monthsago.Hisdyspneaoccursonly
dur
ingmoderat
eorsever
elevelsofexer
ti
on,suchasbai
li
nghayorcarryi
ng50poundsacksof
dai
ryfeedi
nhisdai
lywork.
 

Thepat ient’
smedi calhist
or yconsistsoft r
eat edhyper t
ension,andar emot edeepvenous
thr
ombosi sthatoccur r
edi ntheset t
ingofkneesur gery.Thef amilyhistoryisunr emarkable.
Hiscur r
entmedi cat i
onsar ehydrochlorothi
azi de(25mgdai ly)andmet oprolol(
25mgt wi
ce
dail
y).Onphysi cal exami nation,hisbloodpr essurei s132/74mm Hgandhear trat
eis64bpm.
Thel ungsar eclear .Jugularvenouspul seisnor mal.Caroti
dupst rokehasmi ni
maldel ay.The
apicalimpulseisl ocal i
zedandnotenl arged.Ther eisagr ade3/ 6late-peakingsyst ol
icejecti
on
mur murhear dbestatt her i
ghtupperst ernal border.Noeject i
onclickispr esent.Theaor ti
c
componentoft hesecondhear tsoundi saudi blebuti ssli
ghtlydiminished;t hepulmoni c
componenti snor mal .Peri
pher alexaminationshowsnoedema.Ther emai nderoft he
exami nati
oni snor mal .
 

Thepat
ientunder
goesat
ranst
hor
aci
cechocar
diogr
am wi
tht
hef
oll
owi
ngr
esul
ts:
 

•Leftventr
icular(LV)ejecti
onfracti
on(calculated):68% 
•Int
erventr
icularseptalwallthi
ckness:13mm 
•Meanaor ticvalvegradient:30mm Hg 
2
•Aorti
cvalvear ea:1.3cm  
•Esti
mat edpeakr ightventri
cularsyst
oli
cpr essure:32mm Hg
Whi
choft
hef
oll
owi
ngi
sthenextbestst
ep?

a. Comput edtomogr aphicangiogram.


b. Transcatheteraorti
cvalvereplacement .
c. Dobut amineinfusi
onechocar diography.
d. Cardiaccatheteri
zati
on.
e. Surgicalaorti
cvalvereplacement .

Answer
:

21. d. Car
diaccat
het
eri
zat
ion.

Thecor rectansweriscardiaccat heteri


zat
ion.Thepatientpresentsforevaluat i
onof
sympt omsofexer ti
onaldyspnea.Thephysi calexaminationsuggestssevereaor t
icstenosi
s
(AS).However ,theechocardiogram showsonl ymoder ateAS.Insympt omat icpat i
entswitha
discrepancybetweencl i
nicalexami nati
onandt heresult
soft henoninvasivet esti
ng,invasi
ve
hemodynami ccatheter
izati
oni sindicat
edtodet er
minet hetrueseveri
tyoft heaor ti
cvalve
l
esion( ClassIrecommendat ion). 

Inthispat ient,echocardiographymayhaveunder estimat edtheASbypooral i


gnmentoft he
Doppl erpr obewi ththeaor t
icsyst olicj
et( i
.e.,l
eadingt ounder estimationofthegradient)
,and
bysampl ingt heLVout flowt r
actt i
mevel ocityint
egr altoocloset otheaor t
icvalve(i
.e.
,leading
toover estimat i
onoft het i
mevel ocityintegr almeasur ement ).I
nvasi vehemodynami c
catheterizationforaor t
icgradienti snotindi catedinpat ientswi t
hconcor dantresult
sfrom the
cli
nicalexami nati
onandechocar diogram duet other i
skoft hepr ocedure,parti
cular
lyin
regardt ost r
oker iskcrossingthest enot
icaor t
icvalve(Cl assIIIrecommendat ion)

Neithersurgerynorcat
heter
-basedaor
ti
cvalvereplacementshoul
dbeconsi
deredunti
lthe
severit
yoft heASisdeter
mined.Adobut aminestudyisnotindi
cat
edunl
essthecardi
ac
outputislow. 

Quest
ion:

22. A74- year


-oldwomanpresentstoyourcl
ini
cwit
ha1- monthhistor
yofpr ogr
essive
shor
tnessofbr eathandankl
eedema.ShehasNewYor kHeartAssociati
on( NYHA)functi
onal
cl
assIIIheartfai
lur
e.Hermedicalhi
stor
yincl
udeshypert
ensi
onandhyper l
ipi
demia,butshe
hasotherwisebeenhealt
hy.

Onexamination,
hervi t
alsi
gnsarenormal.Ther
eisanel evat
edjugularvenouspr essuretot
he
j
awwi t
hnoCVwave.Thecar oti
dsdonotappeartohaveadel ay.Therewasasi ngl
esecond
hear
tsoundandagr ade3/6late-
peaki
ngsystol
iceject
ionmur muralongt hesecondr i
ght
i
nter
costalspaceradiati
ngtothecarot
ids.Thel
eftvent
ricl
e(LV)isenlargedwithpalpableA
wave.Thereisnoedema.

Atwo-dimensionalechocardi
ogram r
eveal
samoder
atelydil
atedLVwithacal
culat
edeject
ion
fr
act
ionof36%.Ther eisglobalhypoki
nesi
sandmoderateLVhypertr
ophy.Aconti
nuouswave
Dopplerechocardi
ogram acrosstheaor
ti
cval
vei
sshowni nFigure1.
Cardi
accathet
eri
zat
ionr
eveal
ednosigni
fi
cantcoronar
yarter
ydiseaseandmi
ldpul
monar
y
hyper
tensi
onduetoanelevat
edpul
monarycapil
larywedgepressur
e.

Whi
choft
hef
oll
owi
ngwoul
dyounowr
ecommend?

a. Referfortr
anscathet
eraorticval
verepl
acement(TAVR)
.
b. Medicalmanagement .
c. Referforsurgi
calaort
icvalverepl
acement(SAVR).
d. Dobutamineechocardiogram.

Answer
:

22. c. Ref
erf
orsur
gicalaor
ti
cval
ver
epl
acement(
SAVR)
.

Thecorrectansweri
sSAVR.Theechor eveal
sapeakveloci
tyaort
icst
enosi
s(AS)j
etofat
l
east4msec, confi
rmingsevereAS.Thus,thi
spat
ienthassymptomati
cASwi t
hahighmean
gradi
entandLVdysf unct
ion.
 

Theuseofdobut amineechocar
diogr
aphyisreservedforlow-
gradi
ent,
loweject
ionf
racti
on
(EF)AS(ClassI
Ia).Iti
sdefi
nedasameangr adientof≤30mm HgandanEF<40%.
Dobutamineisi
nf usedtoamaximaldoseof20mcg/ kg/
mi nt
odisti
nguishbet
weentrueAS
and“pseudo-AS”(i
.e.,
severeLVdysfunct
ionandnotsever
eAS).I
fameangradientof>40mm
Hgcanbeachi evedwi t
ht hedobut
amineinf
usion,t
henonecanassumet
hevalveandnotthe
vent
ri
cleistheli
mitingfactori
nthecl
ini
calsyndrome.
 

Thispat
ienthasnosigni
fi
cantcomor bi
dit
iesi
nherhi
stor
y.Theref
ore,
shewouldnotbe
consi
deredacandidatef
orpercutaneousTAVR.Ther
eisnoroleformedi
calmanagementof
sever
esympt omat
icAS,excepttoreli
evesymptomspri
ortoval
verepl
acement

Quest
ion:

23. A54- year


-oldwomanpr esentstotheambul at
orycli
nicforevaluat
ionofshor t
nessof
breath.Shehasbeenf ol
lowedf ormitr
alstenosiswit
hahistoryofrheumat i
cfeverattheage
of14year s.Twomont hsago,shebeganexper ienci
ngsomemi l
ddyspneadur inglight
exercise,suchasj oggi
ngandi ndoorcycli
ng.Shewor ksasanur se,andwhileshehasbeen
abletocont inuetowork,shefeelsmor eti
redthaninyearspast.Shehasnopal pi
tati
onsand
shedeni esanyot herheartf
ail
ur esymptoms. 

Atranst horacicechocar di
ogr am showsnor malleftvent
ricularsystol
icfuncti
on( ej
ecti
on
fr
action65%) .Mitr
alstenosi sispr esentwithameanmi tralgradientof5mm Hganda
calculatedvalveareaof1. 9cm2  (atheartrate60bpm) .Theval veappear spli
ablewithno
commi ssuralcalci
fi
cation( herechoscor e=7).Sheunder goescar di
accatheteri
zati
onandthe
i
nvasi vely-
derivedmitr
algr adi entmeani s12mm Hgwi thami t
ralvalveareacalcul
atedtobe
0.9cm2.Mi l
dpul monaryhyper t
ension(40/ 20withmean29mm Hg)i spresent.Coronary
angiogr aphyshowsnoat herosclerosis.

Whi
choft
hef
oll
owi
ngi
sthenextbestst
epf
ort
hispat
ient
?

a. Cardiopul
monaryexer ci
setest
.
b. Transesophagealechocardi
ography(TEE)
.
c. Repeatresti
ngechocar di
ogram i
n6mont hs.
d. Cardiacmagneticresonanceimaging(MRI).

Answer
:

23. a. Car
diopul
monar
yexer
ciset
est
.

Thecor
rectansweri
saf
unct
ional
test(
car
diopul
monar
yst
resst
est
).

Thepat i
entpresentswi thmi l
dquest ionabl esympt oms, andherechocar diographi
cf i
ndings
suggestonl ymoder at
emi t
ralst
enosi s.However ,theinvasi vehemodynami cdatasuggest
moresever edisease.Int hi
sdiscrepancy, theinvasivedat amaybei ncorrectduetot heuseof
pulmonar ycapi l
lar
ywedgepr essur ef orcal cul
ati
onoft hemi tr
algradient.Pulmonarycapill
ary
wedgepr essuresaccur atel
yreflectmeanl eftat
rial(
LA)pr essurei
nt hevastmaj ori
tyof
pati
ent s.However ,pul
monar ycapill
ar ywedgepr essur es( PCWP)cannotal waysber el
iedupon
fortheaccur atecalculati
onoft hemi t
r algradientduet ot hecharacteri
sti
ct emporaldelayand
dampi ngoft heydescenti nthePCWPt racing.Anat r
ialt r
ans-sept
alpunct uretomeasur etrue
LApr essureandgr adi
enti soneopt iont hatcoul dresolvet hegradientdiscrepancyissue. 

Doppl
erechocar
diogr
aphyi
snotal
waysr
eli
abl
eforassessmentoft
het
ransmi
tr
algr
adi
ent
ei
ther
,andinfactt
hecur
rentguidel
inepref
erst
heuseoft
hepr
essur
ehal
f-
ti
met
odeci
de
sever
it
y(>220msecisconsi
deredsevere).
 

Inthispati
ent,f
urthereval uati
oni sindicat
edtoassessherf unct
ionalst atus.Astress
echocardiogram wouldbeoneopt i
on,andacar di
opul monarystresst estanother.Balloon
mitralval
vulopl
astywoul dbei ndicatedinthepresenceofelevatedt ransmi tr
algradient(>15
mm Hg)dur i
nganexer ci
sest udyori nthepresenceofasi gnifi
cantcar diaclimit
ationby
cardiopul
monar ystresst esti
ng.Bal l
oonmi tr
alvalvulopl
astywoul dbepr efer
redoversur gical
valverepl
acementduet othepl i
ablenat ur
eofthel esi
onandt helowechoscor e.Shewoul d
needaTEEanywaypr i
ort oaper cutaneousvalvuloplast
yprocedur e.Car diacMRIwoul dnotbe
l
ikelytoaddanyaddi t
ionalinformat i
on.

Quest
ion:

24. A22- year-


oldwomanpr esentsf
orroutinefol
low-upinyourmedi caloff
ice.Whenshe
was13yearsold,t
hepati
entsuffer
edfrom rheumati
cf everwi
thcar di
ti
s,andrecoveredful
ly
withmedi
calther
apyal
one.Shenowexer cisesregular
lyasar unnercovering4mi l
esin35
minut
esseveralt
imesperweek.Thereisnohi st
oryofatri
alar
r hyt
hmiasorpr i
orembolic
event
s.Sheisonnomedicati
onexceptbirt
hcont rol
pill
s.

Onphysicalexaminat
ion,sheis67i nchestal
landwei ghs127lbs.Herbloodpressurei s
122/72mm Hgandhear trat
eisr egul
arat76bpm.Lungsar eclear
.Herr i
ghtheartpressur
es
appearnormal.Thefi
rstheartsoundi si
ncreased.Bothcomponentsoft hesecondhear t
soundarenormal.Asof tear
lydiastol
icrumbleispresentt
hatisprecededbyacr ispopening
snap.Thereappearst
obewi deseparati
onofA2-openingsnapinterval
.Noper i
pheraledemais
present

Theechocardi
ogram showsr heumaticmi t
ralst
enosis.Themeant r
ansmi t
ralgradi
entis4mm
Hg,whichisobservedatahear tr
ateof88bpm.Byt hepr essurehalf
-ti
memet hod,
themi tral
2
val
veareaiscalculat
edtobe2. 3cm .Trivi
al mi
tralr
egurgitat
ionispresent.Themitralvalve
appearspl
iabl
e(echocardi
ographicscore=2) .Withsupinebicycleexerci
seandani ncreasein
herheartr
ateto122bpm, themeanmi t
ralgradi
entincreasest o8mm Hg.

Whi
choft
hef
oll
owi
ngi
sthemostappr
opr
iat
enextr
ecommendat
ionf
ort
hispat
ient
?

a. Vi
tami nKant agonistt
herapy.
b. Transesophagealechocardiography(TEE)
.
c. Oralpenici
ll
inV250mgt wi cedail
y.
d. Surgicalmit
ralvalverepl
acement .
e. Percutaneousballoonmitralval
votomy.

Answer
:

24. c. Or
alpeni
cil
li
nV250mgt
wicedai
ly.

Thecorrectansweriscontinuedpeni cil
li
nprophylaxis.Fi
gure1out l
inestheguidel
ineoptions
thati
ncludeoralpenici
ll
inVt wicedaily,monthlybenzathinepenici
ll
inGintramuscular
i
njecti
on,ordail
ysulfadi
azine.Cur r
entguideli
nesal sorecommendcont i
nuinganti
biot
icsfor
secondarypreventi
oninpat i
entswi t
hr heumaticfeverandcardit
is(Figure2).Fort
hose
pati
entswithresi
dualvalvulardisease,therecommendeddur at
ionistocont i
nuepenicil
li
n
prophylaxi
sfor10year sfrom t
helastepisodeofacuter
heumat i
cf everorunt
ilt
heageof40.I
f
therewasacutecar dit
is,
butnor esi
dualvalvedi
sease,t
herecommendat i
onisfor10year
sor
untilage21(whicheverislonger)
.Ift
herewasr heumati
cfeverwithoutcardi
ti
s,the
recommendat i
onisf or5yearsoruntilt
heageof21( whicheveri
sl onger)

Thepatienthasevidenceofrheumat icmitr
alstenosisonphysicalexami nationand
echocardi
ography.However ,herl
esionseveri
tycur r
entl
yismi l
d,andshei sasympt omatic
wit
hnoevi denceforpulmonar yhypertensi
on.Thus, t
hereisnoindicati
onf oreither
percut
aneousorsur gi
caltherapy.TEEwoul dbei ndi
catedtoruleoutleftatri
althrombusin
pati
entsbeingconsideredforpercut
aneousbal loonmi t
ralval
votomy.Avi taminKant agoni
st
wouldber equi
redifatr
ialarr
hythmiaswer epresentortherewasevi dencef orembolioran
atr
ialt
hrombus.

(
Figur
e1)
(
Figur
e2)

Quest
ion:

25. Youareaskedt
oseea60-year-
oldmanwhohashadamur mursincehi
steenageyear
s.
Heent
eredmi
li
tar
yser
vice,
however
,andhasnotseenacar
diol
ogi
stsi
ncethen.
 

Hefinal
lyhadarout
inephysi
cal
examinati
onwit
hhisi
nter
nist
,whoor
der
edan
echocar
diogr
am andDoppler
,wi
ththefol
lowi
ngper
ti
nentr
esult
s: 

•Leftventriculareject
ionfracti
on(LVEF):55% 
•LVwal lthickness:1.1cm 
•Esti
mat edpul monaryarterypressur
eviatri
cuspi
dregur
git
ant(
TR)jetvel
oci
ty:40mm Hg 
•Aorti
cval vet hickeni
ng/calci
fi
ed 
•Aorti
cDoppl er:Peakinstantaneousgradi
ent:3.
6msec,meangradi
ent30mm Hg, est
imat
ed
aort
icvalvear ea( AVA):1.1cm2 
•Aorti
cr ootsize:2. 8cm 
•Leftatr
ialsize:3. 8cm 

Thepat
ientcl
aimshehasnot
icedanincr
easei
nexer
ti
onaldyspneal
atel
yandisunableto
cl
imbafli
ghtofst
air
swit
houtgett
ing“wi
nded.
”Hehasnothadsyncopeorpr
esyncope,but
doeshavesomechestt i
ght
nessatt
imeswithext
remeexer
ti
on.Hedeni
esanyor
thopnea,
paroxysmalnoct
urnal
dyspnea,
edema,orpal
pit
ati
ons.
 

Onexami nati
on,hisbl
oodpr essurei s120/80mm Hgandhear tratei
s78bpm.Hi slungsare
cl
ear .Hehasnoj ugul
arvenousdi stenti
onorhepat oj
ugul
arrefl
ux.Hiscarot
idsuggestslow
volt
agewi t
hdelayandt her
ei satransmi t
tedaorticmurmur.HehasamodestLVheavei nthe
l
eftlater
alposi
tionwithapalpableAwave.Onauscul t
ati
on,t
hereisasystol
icmur murthatis
crescendo-
decrescendoinnat ur
ewi thmi dpeakingandasingleS2. 
Noeject
ioncl
ickisheard.
ThereisanS4.Noaor ti
cregurgit
ationormi tr
alregurgi
tat
ioni
saudi bl
e.Ther
eisnoedema.

Whati
sthenextst
epi
nhi
smanagement
?

a. Car
diacmagnet
icresonanceimagi
ng(MRI)
.
b. Car
diaccat
het
eri
zation.
c. Car
diaccomput
edt omography(
CT).
d. Si
ngle-
phot
onemissioncomputedtomogr
aphy(
SPECT)myocar
dialper
fusi
on.

Answer
:

25. b. Car
diaccat
het
eri
zat
ion.

Thecor r
ectanswerist oproceedwithcardiaccathet
erizat
iongivenhissympt omat i
cstatus
andthedi scr
epancybet weentheclini
calfi
ndingsandt heecho/Doppler
.Givenhi sage,the
pati
entli
kelyhasabi cuspidaorti
cvalveandal t
ernat
iveimaging,suchascardi acMRIandCT,
areclear
lyofvalueindecipheringwhetherhehasanassoci at
edaorti
caneur ysm, evenwhen
theechodoesnotsuggestanenl argedaorta.
 

CardiacMRImi ghtbeabl etoclar


ifyt heaorti
cvalvesever i
tyandpr ovi
dequanti
tat
iveLV
functi
onaldat
a, buti
sanunnecessar yst
epi fyoubelievethepat i
enthassymptomat i
caorti
c
stenosis(
AS)basedont hecli
nical exam (i
.e.,
lossofej ect
ionsound, mi
d-l
atepeakingaort
ic
mur mur,l
ossofA2 result
inginasi nglesecondhear tsound) .Cardi
acMRIcanal soprovide
evidenceofpr
iormyocar diali
njuryandmyocar dialperfusi
on.Hedoeshavesomechest
pressure,
butamyocar di
alperf
usionst udyi snothelpfulindecipheri
ngtheseveri
tyofhisAS. 

Iti
sconsideredaClassIindi
cati
on(LevelofEvi
denceC)toproceedt
oaninvasi
vest
udyi
n
symptomaticpati
entswherethereappearstobeadiscr
epancybetweent
he
echocar
diographi
cfindi
ngsandt hecl
ini
calexam andhi
stor
y. 

Quest
ion:

26. A40- year-


oldwomanpr esentsforrout
inegener almedi
calevaluation.Thr
eeyear
sago,
sheunderwentpercutaneousclosureofanostium secundum atr
ialseptaldefect
.Att
hatti
me,
ther
ightventr
icl
ewasmoder at
elydilat
edanddysfunctional
,butshewasasympt omati
c.She
hasdonewel lsi
ncetheprocedure,andreport
snosympt oms. 

Herphysi
calexaminat
ionisunremarkabl
e.Transthor
acicechocardi
ogr
aphy(TTE)
demonstr
atesnormall
eftventr
icul
ar(LV)f
unction(ej
ecti
onfracti
on65%).Theri
ghtvent
ri
clei
s
mil
dlyenl
argedwithmilddepr
essionofsystol
icfunct
ion. 
Onechocardi
ography,theatri
alseptaloccl
uderi
swellvi
sual
izedattheatr
ialsept
um wit
hno
evi
denceofdevi
ceer osion.Dopplercolorf
lowacr
osstheatr
ialsept
um showsnor esi
dual
shunt
ing.Abubblestudyi snor
mal .

Thepati
entwi
llbeunder
goi
ngarouti
necleani
ngofhert
eet
hint
henextmont
h.Shewoul
dli
ke
toknowabouttheneedf
orant
ibi
oti
cprophyl
axi
s.

Whi
choft
hef
oll
owi
ngi
sthemostappr
opr
iat
enextst
epf
ort
hispat
ient
?

a. Amox i
cil
lin2g,30-60minutespriortotheprocedure.
b. Cl
indamyci n600mg, 30-60minutespriort
ot heprocedure.
c. Peni
cil
linG250mg, 30-60minutespr i
ortotheprocedure.
d. Noantibioti
cprophyl
axisisi
ndicated.
e. Amox i
cil
lin4g,30-60minutespriortotheprocedure.

Answer
:

26. d. Noant
ibi
oti
cpr
ophyl
axi
sisi
ndi
cat
ed.

Shedoesnotneedant i
bioti
cprophyl axis.Antibioticpr ophylaxisi sindi
cat edforpat i
entswi th:
1)prost
het i
ccar diacvalvesorpr osthet i
cmat eri
al usedf orvalver epai
r,2)pr i
orinfecti
ve
endocardit
is,3)unr epair
edcyanot iccongeni t
al hear tdisease, 4)sur gi
callyorper cutaneously
repai
redcongeni talheartdefectsdur ingt hefirst6mont hsaf t
ert heprocedur e,5)repai r
ed
congenit
alhear tdiseasewi thresidual defectsatt hesi teoradj acenttothesi teofapr osthesis,
or6)cardiactranspl antreci
pientswi thval vul
ardi seaseduet oast r
ucturallyabnormalval ve.It
shouldalsobeconsi deredinpat i
entswhohaveanact ivesour ceofi nfection.Thecor rectdose
ofamoxicill
in,whenused, is2g, 20-60mi nutesbef oret heprocedur e.

Quest
ion:

27. Thedeci si
ont oint
erveneandtheappropri
atechoiceofint
erventi
oninvalvularhear
t
di
seaseshoul
dbebasedont heri
sk-benef
itanal
ysi
sthatincludestheoperat
ivemor tal
it
y,t
he
pat
ient
’sdegr
eeoffrail
ty,
thenumberofcur r
entmajororgansystemst hatar
edysf uncti
onal
,
andanyspeci
ficpr
oceduralimpedimentsthatwouldl
imittheli
keli
hoodthattheprocedure
wouldbeasuccess.
 

Inyourevaluationofan80- year-ol
dpat ientwithsever eaorti
cstenosis(AS),youdetermine
thathi
sSoci etyofThor aci
cSur geons( STS)pr edict
edr i
skofmor tali
ty(PROM)i s7.0%.Hei s
rel
ati
velyactive,li
vesalone,andcont inuest oplaygolfont heweekends.Hehasmoder ate
chronickidneydiseasewi thacreatinineof1. 9mg/ dl.Thoughapr iorsmoker ,
hismostr ecent
pulmonaryf unctiontestshavenotsuggest edsignif
icantlungdisease.Hehadapr iorbypass
procedureandabdomi nalaorti
caneur ysm r epair
.Heot herwisehasf el
twellunti
lrecentl
y,
whenhehasnot edsomedyspneaonexer ti
on.Hi saorti
cvalvemeangr adi
entisnow
esti
mat edat45mm Hgandt heaorticval vear eais0.7cm2.

Basedont
helat
estguidel
ines,
hiscombi
nedr
iskassessmentwoul
dpl
acehi
mint
owhi
chof
t
hef ol
l
owi
ngcategor
ies?

a. Hi
ghri
sk.
b. Pr
ohi
bit
iver
isk.
c. Lowrisk.
d. Int
ermediat
eri
sk.

Answer
:

27. d. I
nter
medi
ater
isk.

Thecor r
ectanswerisint
ermedi
ater
isk.Thel
atestval
vularhear
tdi
seaseguidel
inessuggest
thattheri
sk-
benefitf
orpati
ent
sneedingaval
vularint
ervent
ioncanbebestdefi
nedusingthe
foll
owingguidel
ines:
 

1. Lowr i
sk:STSPROM <4.0%AND noevidencef
orfr
ail
tyAND 
noot
hermaj
oror
gan
syst
em dysf
uncti
onAND noprocedur
alspeci
fi
cimpediment

2. Int
ermedi
ateri
sk:STSPROM 4-8%OR atl
eastonei
ndexoff
rai
lt
y(mi
ld)
OR 
oneor
gan
syst
em i
nvol
vement 
OR apossi
blepr
ocedur
alimpediment

3. Highrisk:STSPROM >8.
0% 
OR 
twoormoreindi
cesoffrail
ty(moder
ate-
sever
e) 
OR 
up
tot
womajororgansyst
emscompromi
sedOR 
probabl
eprocedurali
mpediment.
 

4. Pr
ohi
bit
iver
isk:Pr
edi
ctedsurgical
riskormaj ormor bi
dit
yat1yearof>50% 
OR 
thr
eeor
moremaj
oror
gansystemscompr omised 
OR  sever
epr ocedurali
mpedi
ment. 

Inthepat ientdescr ibed,hissur gicalriskf rom t heSTSscor epl aceshi mi nthei ntermedi ate
ri
sk.Heal sohassomemi ld-moder atecompr omi seinhi sr enalf unct ion, buthehasnoobvi ous
frai
ltydespi tehisageandt her ei snocl earpr ocedur al i
mpedi ment ,althoughcomput ed
tomogr aphy( CT)mi ghtbeusef ul t
odeci dei fhispr i
orcor onar ygr aftsar eadher i
ngt ohi s
sternum.Hewoul dthereforebeconsi der edati ntermedi ater iskbasedont hei nformat ion
provided.Lowr iskisi ncorrect ,astheSTSscor eistoohi ghandhehasanor gansyst em
i
nvol ved.Hi ghr i
skisi ncorrect ,ashi sSTSscor eisnot>8%andhehasnof railtynort womaj or
organsyst emsi nvolved.Pr ohi biti
ver i
ski si ncor rectfort hesamer easons, ashemeet snone
ofthemostsever el
yrest r
ictivecr it
eria.Themostdi fficultdeci sioni nt hissi tuationi sof t
en
definingf railt
y.Mostoft heavai l
ableef fortst oquant i
fyf rail
tyt akei ntoaccountt hepat ient
’s
i
ndependencei ndoi ngr outineact i
viti
es, suchaseat ing, bat hing,dr essi ng,transf erri
ng,
toil
eting, i
ncont inence, ambul ating(wi t
handwi thoutassi stance) ,andt hedi stanceonecan
ambul ate.Pr ocedurali mpedi ment scanal sobequi t
esubj ect i
ve, buti ncl udet hepr esenceofa
porcel ainaor ta,lackofvascul araccess, priorchestr adiation, adher entcor onar ygr aftstothe
sternum, chestmal format i
on, presenceofat racheost omy, etc. 

Quest
ion:

28. A70- year-


oldmani sr eferredformanagementofaort
icstenosi
s(AS).Heret
ir
edthis
pastyearf
rom thelocaluniversity.Heandhiswifewalkdai
l
yf or30minutesever
ymor ni
ng.
Hedeniesanyexertionall
imitations,angi
na,orsyncope.
 
Anechocar di
ogram demonst r
atesanaor ticvelocit
yof3. 5msec, val
veareaof1. 0cm2,
ejecti
onfract
ion60%, andmi l
dleftventr
icular(LV)hyper t
rophyi spresent.Hisexam isnotabl
e
forthelackofhypertensi
onorvascul ardisease.Hehasagr ade3/ 6systoli
cejecti
onmur mur
overthebaseoft heheartradi
atingtothecar oti
ds, cl
avicl
es, andapex.S2 appearspreserved.
ThereisanS4.Ther eisnoevidenceofr i
ghtorl eftheartvolumeover load.

Whi
choft
hef
oll
owi
ngdoyour
ecommend?

a. Fol
low-upexam year
lyandobtai
nechoonl
yifexam changes.
b. Repeatechoin3years.
c. Fol
low-uponlyi
fhedevelopssympt
oms.
d. Repeatechoin1year.
e. Repeatechoin6mont hs.

Answer
:

28. d. Repeatechoi
n1year
.

Thecorrectansweri
sserialexamsand( year
ly)echocardi
ography.Thi
srecommendati
oni s
predi
cat
edont hepotent
ialforsi
lentandvari
ableprogressi
onofAS, andthepot
enti
alf
or
progr
essiveLVdysfunct
ionandt hedevelopmentofotherconcomi t
antval
vul
arhear
tdisease.
 

Thef
requencyoffol
low-
upi
sbasedontheseveri
tyofdi
sease:aor
ti
cveloci
ty≥4.
0msecf
oll
ow
-
upin1year;>3.
0-3.
9msec,
every1-
2years;>2.
0-2.
9msec,every3-5year
s. 

Thispat i
enthasmoder at
eAS.Thus, thefol
low-upechoshoul dbeevery1- 2year
s.The
averageannual rat
eofprogressioninASisani ncr
easeinaor t
icveloci
tyof0.3msec, mean
gradientof5-10mm Hg, andadecr easei
naor t
icvalveareaof0. 1cm2 oncemoder ateASi s
present.Morerapidprogressi
oncanbeant i
cipatedinthepresenceofadvancedageand
extensiveval
vularcal
cifi
cati
on,butprogr
essionishighl
yvariable.Repeatechocardiographi
c
assessmenti sindi
cat
edi fsymptomsorachangei nexam develops. 

Quest
ion:

29. Youhavebeenfol
lowi
ngf
oroveradecadea71-
year-
oldmanwi t
hknownbi
cuspi
d
aor
ti
cval
vest
enosi
s.Hei
sacoll
egeprof
essorandi
sveryphysi
cal
lyacti
ve.
 

Hehadcar di
acmagnet i
cresonanceimagingthatreveal
ednoevi denceforanassociat
ed
dil
atedascendi ngaorta.Hi
sechoeshaver evealedslowwor seni
ngoft hesever
it
yofhisaorti
c
stenosis(AS),andhenowhasapeaki nst
antaneousDoppl ervel
ocityof4.
1msec, an
estimatedmeanaor t
icgradientof42mm Hg, andanaor t
icvalveareaof0.9cm2withnormal
l
ef tvent
ri
cularfuncti
on.Hecont inuest
odenyanyexer ti
onalsympt omsofchestpressure,
dyspnea,ordizziness.

Whi
choft
hef
oll
owi
ngi
stheappr
opr
iat
enextst
epi
nhi
smanagement
?

a. Surgicalaor
ticval
vereplacement(
SAVR).
b. Exercisestr
esstest
.
c. Transcathet
eraorti
cvalverepl
acement(
TAVR)
.
d. Cont
inuet
omoni
torever
y6mont
hs.

Answer
:

29. b. Exer
cisest
resst
est
.

Thecorrectanswerisafuncti
onal
test
,suchasanel
ectrocardi
ogram (
ECG)stresstest
.The
pat
ientnowhasevi denceforsi
gni
fi
cantaor
ti
cval
vestenosis.However,hedeniessymptoms.
 

Iti
sr easonabletopr oceeddirectl
ytosur geryi
nasympt omaticsevereASi fthegradi
entis
“super-severe”(i
.e.
,veloci
ty>5msecandt hepati
entisl owrisk(ClassIIaindi
cati
on,Levelof
EvidenceB) ,buthedoesnotmeett hesecrit
eri
a.Thus, proceedi
ngdi r
ectlytoSAVRi sincorr
ect
.
Hedemonst ratesnohi gh-
riskfeaturesandhehasabi cuspidaor t
icvalve,makingan
evaluationforTAVRi ncor
rectatthispointtoo.
 

ASpr ogr essessl owly,andi tissafetomoni torhisechocar di


ogr amsper iodically,t
houghever y
6mont hsmaybet oof requent.Thecor rectansweri s,therefor
e, toperform anexer ci
se
treadmi l
ltest .Thi
sshoul dal waysbedoneundert hesupervisi
onofaphysi cian, wit
hcl ose
moni t
or i
ngofbot htheECGandbl oodpr essureresponse.Thet estshoul dbeconsi dered
positi
vei fanysympt omsar eprovoked, ifthereisanabnor malhemodynami cr esponse
(hypotensi onoral ackofr ise>20mm Hgi nsystemi cbloodpr essurewi thstress, orST
abnormal i
tiesdevelop).Byusi ngthismet hod, upto29%ofpat ientsthoughtt obe
asympt omat iccanbei dentifi
edasbei ngl imitedbyt heaorticval vest
enosi s.Exer ci
sest ress
testi
ngshoul dnotbeper formedi npatientswi thsympt omsofASori nt hosewi th“super-
severe”AS( meangr adient>55mm Hg) . 

Quest
ion:

30. Youar easkedt oseean87- year-


oldwomanwhohasknownaor ti
cstenosi
s.Shel i
ves
i
nanassi stedlivi
ngenvi r
onment ,butisquiteindependentandcont i
nuestokni
tandvisitwit
h
fri
endsandf ami l
y.Shehadapr iorcerebr
ovascularaccident(
CVA)2year sago,buthas
recoveredexceptf orami l
dr i
ghtfootdrop,causinghertoambul at
ewi t
hacaneorwal ker.Her
medi calhistoryisalsopositivef
orhyper t
ension,mildtype2diabetes,mil
dobesit
y,ari
ghthip
replacement ,andapr iorcolonresecti
onf orcoloncancer(12yearsago). 

Shehasr ecentlynoti
cedthatherst
ami
nahasdecl
inedandnowshei sdyspnei
cafter
ambulati
ngj ust30-40feet
.Shehasnoort
hopneaorpar
oxysmalnoctur
naldyspnea,butdoes
not
ebilat
er alankl
eedema.Shehasnopalpit
ati
onsorchestpai
n,andhasnotfall
ensinceher
st
roke.
 

Onherexam, shehasobvi oussevereaorti


cst enosis(AS),andthisi
sconfirmedbyan
echocardi
ogram thatrevealssevereASwi thpr eser
vedleftvent
ricul
arsystol
icf
uncti
on.Her
peakgradientis4.8m/ secandmeangr adientis50mm Hg, wit
hanaor ti
cvalvear
eaof0.5
2
cm .Perti
nentlaboratorydata,however,r
eveal hercreati
ninei
s3. 2mg/dlandshehasa
severel
ycalcifi
caorta(porcelai
naorta)onthescr eeningcomput edtomographi
cangiogr
am.

Basedonthepr
ecedi
ngf
indi
ngs,
whi
choft
hef
oll
owi
ngt
her
apeut
icopt
ionsdoyou
recommend?

a. Per
cut
aneousbal
loonaor
ti
cval
vul
opl
ast
y.
b. Me di
caltherapyal
one.
c. Transcathet
eraorti
cvalverepl
acement(
TAVR)
.
d. Surgicalaor
ticval
vereplacement(
SAVR).

Answer
:

30. c. Tr
anscat
het
eraor
ti
cval
ver
epl
acement(
TAVR)
.

Thecorr
ectansweri
sTAVR.Shehassomef r
ail
tyandli
mitedact
ivi
ty,
alt
houghsheconti
nues
toenj
oyfamil
yandsedent
aryact
ivi
ty.Percut
aneousbal
loonaort
icval
vulopl
ast
yisnotcorr
ect
,
asiti
sareasonabl
eopt
iononl
yasabr idgetoSAVRorTAVR.  

Shehascr i
teri
atobeconsi deredashavi
ngpr ohi
bit
ivesur
gicalr
isk,
givenherpor
cel
ainaor
ta,
si
gnif
icantrenaldisease,
andCVAwi thr
esi
dual.Thishaslef
therrat
herfrai
landwit
ha
cal
culat
edSoci et
yofThor acicSurgeon(
STS)r i
skof>8%.
Thesurgicalopti
onseemsof fthet
ablef
orthesereasons.
 

Shecouldoptf orcontinuedmedicalther
apyknowingthati
twi l
levent
uallyf
ailgi
venthe
severi
tyofherAS, orshecoul dbeconsideredacandi
dateforTAVRi nasetti
ngwher et
herei
s
ateam ofphysiciansdedicatedtotheprocedur
e.Basedont henewestguidel
ines,TAVRis
recommendedi fpatientshaveanindicati
onforAVR,buthaveapr ohi
bit
iveri
skforSAVRand
haveapr edi
ctedpost -
TAVRsur vi
valof>12mont hs.
 

Quest
ion:

31. An84- year-


oldmanpr esentstothecli
nicwit
hmilddyspneaonexer tion.Hismedical
hist
oryincludeshypert
ension,hyperl
ipi
demia,chr
onicki
dneydisease,andr ecurr
entgout.Hi
s
examinationreveal
saheartr at
eof75bpm andabl oodpressur
eof160/ 65mm Hg.Hehasa
grade1/6ear l
ypeakingsystoli
cejecti
onmur murinthesecondrighti
nterspaceandagr ade
2/4diastoli
cmur murattheleftst
ernalborder

Theechocardiogr
am showspr eservedleftventr
icularej
ectionf
racti
on(
LVEF)andmoder at
e
LVhypert
rophy.Therewasmoder atelef
tatrial
enl ar
gementandgr ade2di
astoli
cdysf
unct i
on.
Theaort
icvalvewasscleroti
candt ril
eafl
et.Ther egurgi
tantvol
umeoftheaorti
cregur
gitati
on
(AR)i
sesti
mat edat35cc/ beatandt hewidthoft hejeti
s50%oft heLVoutf
lowt r
ackdiameter.

Basedont
hesedat
a,whi
choft
hef
oll
owi
ngdef
inest
hesever
it
yofhi
sAR?

a. Mil
d.
b. Severe.
c. Tr
ivial
.
d. Mode rat
e.

Answer
:

31. d. Moder
ate.

Asshowninthet
able(
Figure1)oft
hest
agesofchroni
cAR, f
rom t
he2014AHA/
ACCval
vul
ar
gui
del
ine,
thi
spat
ienti
swithint
hemoderat
erangeofsever
it
yofAR.
(
Figur
e1)

Quest
ion:

32. A50- year-


oldphysi
cianwitha5-yearhi
storyofmitralvalveprol
apsepresentswi
than
acutei
ncreasei nhismurmur .Si
xweeksagohehadwhathet houghtwasthe"flu"andf
elt
poorl
yforthenextcoupl eweeks.Hehasnowr ecover
edandi sbackatwor kfullti
me.Hesays
hefeel
swel landnot esnochangeinstamina,chestpain,
ordyspnea.  

Hisexam reveal
sabloodpressur
eof110/ 76mm Hg, hear
trat
eof78bpm, andoxygen
saturat
ionof98%.Hisl
ungsareclearandhiscardi
acexam reveal
sagrade4/6holosyst
oli
c
mur muroverthepr
ecordi
um andloudestattheapex.Ther
eisanassoci
atedS3.
 

Hisechocardi
ogram demonstrat
esaf lail poster
iorleafl
etfrom chordalrupt
ureandan
2
eff
ecti
veregurgi
tantori
fi
ceof0.35cm , leftventri
cular(LV)ejecti
onf r
acti
onof60%,end-
systol
icdi
mension4.5cm, andleftatr
ial dimensionof4. 5cm.Hi select
rocar
diogr
am (
ECG)
showssinusrhythm,occasionalpremat ureventricularcontract
ions,andanarrowQRSwi t
h
nonspecif
icECGchanges.
Whi
choft
hef
oll
owi
ngdoyour
ecommend?

a. Li
sinopril
.
b. Carvedil
ol.
c. Digoxin.
d. Surgicalr
efer
ral
.
e. Continuedobser
vat
ion.

Answer
:

32. d. Sur
gicalr
efer
ral
.

Thecor rectanswerissurgicalrepai
r.Al
thoughthi
spati
entclai
mstobeasympt omati
c,hehas
developedacut eseveremi t
ralregur
git
ati
on(MR)complicatedbyaflai
lmi t
ralleaf
let
.Hehas
LVsyst oli
cdysfuncti
onexhi bi
tedbyani ncr
easedend-
systoli
cdimension.Surgical
managementi sclearl
yrecommendedatt hi
sti
me( Cl
assI).Ther
eiscurrentl
ynocompel l
ing
evi
dencet osupportaft
erl
oadr educti
onorinot
ropi
cagents,suchasdigoxin,inanysituat
ion
butinacut edecompensat edMR.

Quest
ion:

33. A34- year-oldwomani sref err


edt oyouwi thshor tnessofbr eathandt henewonsetof
atr
ialf
ibril
lation.Afteryourexami nationandf oll
owi ngtheper f
ormanceoft hechestwall
echo/Doppl er,youf indshehassi gnificantmi tr
al stenosis(meangr adient15mm Hg)wi t
h
mildmitralregur git
at i
on(MR).Herest i
mat edpul monar ysyst oli
cpr essur esar eel
evatedat64
mm Hgandherl eftatri
alsi
zeis5. 0cm.Herval vularleafl
etsappearf usedont he
echocardiogr am, butthereissti
llgoodmobi l
it
ywi thmi ld-moder ateval vularthi
ckeni
ng,only
minimalcal cium, andmi ld-
moder atesubmi tr
alscarandr etracti
on.Youconcl udethatherecho
scoreforthisi sappr oximatel
y8.However , youar econcer nedr egar dingthesever i
tyofherMR,
butthetranst horacicechosuggest sonl ymi l
dMRt houghi tisclearlyaudi bleonexam.

Whi
choft
hef
oll
owi
ngi
sthenextappr
opr
iat
est
udy?

a. Cardiaccathet
eri
zation.
b. Cardiacmagneticresonanceimaging(MRI)
.
c. Cardiaccomputedt omography(CT).
d. Transesophagealechocardi
ography(TEE)
.

Answer
:

33. d. Tr
ansesophagealechocar
diogr
aphy(
TEE)
.

Thecor r
ectansweri saTEE.Pati
entswit
hr heumat i
cvalvularmi tralstenosi shavebeenshown
todowel lf
ollowingper cut
aneousball
oonmi tralval
vulopl
ast yiftheval veispl i
abl
ewith
commi ssuralfusion.Long-t
ermresul
tsmirrorthoseofsur gicalcommi ssur otomy.Thi
spati
ent
hasindicat
ionsf orinter
venti
oninhermitr
alstenosis,ashermeangr adientis>10mm Hgata
hear
trate<70bpm, therei
spulmonaryhypertensionandat r
ialfibril
lati
on, andsheis
symptomat i
c.Themit
ralvalvei
ssai
dtobeamenabl etoapercutaneousapproachwi
th
rel
ati
velyl
owechoscor e.Eachcomponentoftheechoscor
eisgr aded1-4,
wi t
h4beingthe
mostsevere.Thecomponent si
ncl
udevalvul
armobili
ty,
val
vularthi
ckeni
ng,theamountof
cal
cif
icati
on,andtheamountofsubmitr
alapparat
usscarandr et
racti
on.
 

Notincludedi nthescor i
ngsyst em, however,i
st hedegr eeofMR.Thepr ocedureisineffective
i
ftheMRi s>2+.Thenextbestt estt odecidethatist heTEE.Thi swoul dbeneededr egar dl
ess
ofwhet herabal loonvalvuloplastypr ocedur
ei sproposed, sinceanat ri
althrombusneedst o
beexcludedbef orethepr ocedurecanbeper formedsaf ely.CardiacMRIcanest imatet he
degreeofMRbynot i
ngt heregurgi tantfl
owand, ifpr operl
yobt ained,canpr ovi
desome
evidencef oranat ri
alappendaget hrombus;thus, itwoul dbeasecondchoi ceinthissituation.
Thecar diacCTwoul dnotpr ovidet heMRsever i
tyi nformation,andcar di
accat heter
ization
shouldber eser
vedf oreitherresolvingunclearinfor mationf rom thenoninvasivestudiesorf or
theactualpr ocedure. 

Quest
ion:

34. Youhavebeenaskedt oconsul tona30- year-ol


dwomanwi thsymptomati
csevere
rheumat i
cmi tr
alstenosis,whoappear stohavear elat
ivel
ylowechoscoreforsuccessusi
ng
ball
oonmi tralvalvul
oplasty.Priort
ot hepr ocedure, youobtainatr
ansesophageal
echocardiogrm( TEE)thatr evealsaleftatri
alappendaget hrombus.Shehasneverhada
cli
nicalembol i
ceventandi sinnormal sinusr hythm att heti
meoftheTEE.Hercurrent
medi cat
ionsincludeaspi r
in81mgdai l
y,furosemi de40mgdai l
y,spir
onol
actone12.5mgdail
y,
andmet oprolol25mgt wicedai l
y.Shei salsot akingabi rt
hcontr
olpil
l.

Basedont hi
sinfor
mati
on,
your
ecommendr
epeat
ingt
heTEEi
n4-
6weeksbef
oreat
tempt
ing
t
hemi t
ralval
vulopl
ast
y.

Dur
ingt
hatt
ime,
whi
choft
hef
oll
owi
ngdoyoual
sor
ecommend?

a. Di
sconti
nuet hebi
rthcont
rolpi
ll
.
b. Begi
ndabi gat
ranat150mgt wi
ceaday.
c. Begi
nwar fari
n.
d. I
ncreasetheaspir
indoseto325mgaday.

Answer
:

34. c. Begi
nwar
far
in.

Thecor rectansweri
st oini
ti
atewarfar
intherapy.Whilebeingont hebir
thcontrolpil
lmay
contri
but etoahypercoagul
ablestat
e,therearenodat athati
tsdisconti
nuati
onwoul dr esul
tin
dissol
utionoftheappendaget hrombus.Similar
ly,t
herearecurrentl
ynodat athatinapat i
ent
withvalvulardi
seaseandpar oxysmalatri
alf
ibri
llat
ionthatei
therincr
easingtheaspirindose
ortheuseofotherant
it
hrombinagent
s,suchast henovelant
icoagul
ants,
issuperi
ortothe
useofwarfar
in.I
ndeed,t
henoveloral
anticoagul
antsarecont
raindi
catedforat
ri
alfi
bri
ll
ati
on
ther
apyinpati
entswit
hmi t
ral
stenosi
s. 

Theuseofwar fari
nint hissetti
nghasaCl assIindi
cati
on( LevelofEvi
denceB) .Avit
aminK
antagonistal
soi sindicatedinmi t
ral
stenosispati
entswithparoxysmal,persi
stent
,or
permanentat r
ialfi
bril
lati
on, ori
nthosewhohavehadapr i
orembol i
cevent.Themostr ecent
guidel
inealsohasadded, asaClassIIbindi
cati
on(LevelofEvidenceC),thatvi
taminK
antagonistsbeconsi deredwhent hemi t
ralval
veareais55mm byTTE, ori
ftherei
s
spontaneousleftatri
alappendageoncont rast
.

Quest
ion:

35. A63- year-


oldmanwi thNewYorkHear tAssociationclassIheartfail
urepresentsto
yourcl i
nicwi t
hanewl ydiagnosedhear
tmur mur .Hisechocar di
ogram revealsaleftventr
icul
ar
(LV)end- syst ol
icdiameterof50mm andLVej ect i
onfraction(EF)of55%.Ther eispost er
ior
mitralleafletprolapsewi t
hcol orf
lowevi
dencef orsignif
icantmi tr
alr
egur gi
tati
on(MR) .The
2
effecti
ver egur gi
tantorif
ice(ERO)ismeasuredat45mm .Hi sbloodpressureduringthe
exami nationi s140/ 80mm Hg.

Basedont
heechocar
diogr
aphi
cfi
ndi
ngs,
whi
choft
hef
oll
owi
ngwoul
dyour
ecommend?

a. Mitr
alvalverepl
acement.
b. Repeatechocardi
ogram i
n6mont hs.
c. Mitr
alvalverepai
r.
d. Endovascularmit
ralval
verepai
r.

Answer
:

35. c. Mi
tr
alval
ver
epai
r.

Thecorrectansweri
smi tr
alvalverepai
r.Thepat
ienthassever
eMRbyquant i
tat
ive
2
echocar
diography(r
egurgit
antvolume>60ccandERO>40mm ) .Alt
houghheis
asymptomat i
c,t
heventr
icl
ei sdil
atedandtheEFisnotnormalint heset
ti
ngofsevereMR.
 

Iti
saCl assIindicati
ontoreferforsur
gicalr
epairinanasympt omat
icpat
ientwi
thsevereMR
withanEFbet ween30- 60%and/ oranend-syst
olicdiamet
er>40mm.I sol
atedposter
iorl
eaf
let
prolapseisi
dealf orval
verepairwit
ha>90%chanceofsuccess.Endovascul armit
ralval
ve
repairi
snotindicatedinthi
spat i
entwhoisnotathi ghsur
gicalri
sk.

Quest
ion:

36. A51- year-


oldwomani sreferr
edtoyouwi thmi t
ralval
vepr ol
apseandsever emi t
ral
regurgi
tat
ion( MR).ShehashadknownMRf ormanyyear sandhasbeenf ol
lowedwi t
hserial
echocardi
ogr aphy.Hermostrecentecho/Dopplerrevealssevereposter
iorleaf
letprol
apse
withavalvularaneurysmalbul
geandpossi blet
or nmi t
ralchord.Theprolapseinvol
ves
approxi
mat elyone-t
hirdoft
hepost er
iormit
rall
eaflet.
 
Theot
herf
eat
uresofherechocar
diogr
am ar
easf
oll
ows:
 

•Lef
tvent ri
cular( LV)end- diastol
icdimension:5.
8cm 
•LVend- systolicdimensi on:4. 1cm 
•LVejectionf racti
onest imat e:50% 
•Eff
ecti
ver egur gitantori
fice( ERO):0.45cm2 
•Mil
dt r
icuspidr egur gi
tation 
•Pulmonar ysyst olicpressur eesti
mat e:45mm Hg 
•Nomi t
ralgr adient  
•Noaorticval vedi sease 
•Mil
dLVhyper trophy

Basedont
hesedat
a,whi
choft
hef
oll
owi
ngwoul
dbet
henextappr
opr
iat
est
epf
ort
hispat
ient
?

a. Af
terloadreducti
on
b. Mitr
alvalverepai
r.
c. Exerciset
esti
ng.
d. Mitr
alvalverepl
acement
.

Answer
:

36. b. Mi
tr
alval
ver
epai
r.

Thecor r
ectansweri smi t
ralval
verepair.Thepat i
enthasreachedaClassIindi
cati
onforan
i
nterventi
onf orherMR.HerLVej ecti
onf r
actionisnowbel ow60%andherLVend- systol
i
c
dimensionis>4.0cm.Shehassever eMR( ERO0. 4cm2).Ther
eisnoneedt oconsi
derany
functi
onaltesti
ngint hi
ssituati
on.Inadditi
on, t
herearenodatathataft
erl
oadreducti
onalter
s
thecourseofherdi seaseatthispoint.Sheshoul dundergoaninter
vent
ion.
 

Shedoesnothaveapr ohibi
tivesurgicalr
isk.Thus, apercut aneousappr oachi snotwarrant
ed
andsheshouldhaveasur gicali
nterventi
onei therbyst ernot omyorl imit
edt horacot
omy.
Whent heprol
apseinvolves<50%oft hepost eriorleaf
let,thesur gicalapproachshould
def
ini
telybevalverepairandnotr eplacement .Indeed,fort hemostpar t,
mi t
ralrepai
rshould
al
waysbeaf i
rstchoiceinthosewi thdegener ativemitralval vedisease. 

Cur r
entguideli
nesemphasizethati
tisaClassII
I(har
m)indicati
ontoper f
orm mitr
alval
ve
replacementinapat i
entwit
h<50%oft heposter
iorl
eafl
eti
nvolved;r
ather,theyal
lshoul
dhave
mitralrepai
r.Theonlyexcept
ionwouldbeinthosesit
uati
onswher emitralrepai
rwas
attempt edandfai
led.

Quest
ion:

37. Themostcommoncauseoft
ri
cuspi
dregur
git
ati
on(
TR)i
sfunct
ionali
nnat
ure.

Whichofthefoll
owi
ngisconsi
deredaClassI(
LevelofEvi
denceC)i
ndi
cat
ionf
ort
ri
cuspi
d
val
verepairwhensever
eTRispresent
?

a. Whenapacemakerordef
ibri
ll
atorisrequi
red.
b. Att
heti
meoflef
t-si
dedvalvesurgery.
c. Whenther
eareatr
ialar
rhythmias.
d. Whenpulmonar
yhypertensionispr
esent
.

Answer
:

37. b. Att
het
imeofl
eft
-si
dedval
vesur
ger
y.

Thecorrectansweri satthetimeofl ef
t-sidedvalvesurger y.Inapproximat
ely80%oft hecases
ofTR,theanat omi cunderpinningsaref unctionali
nnat ure.Thi sr
elat
estothef actt
hatchor dal
att
achment sf orthethreeleafl
etsareof tenont heventricularwallandnotspeci f
icpapill
ary
muscles.Ifther i
ghtventricl
e(RV)di l
atesf oranyreason, itresult
sinpoorleafl
etcoapt at
ion
duetother esultanttet
heringoft hevalve.Inaddition,
thet ricuspi
dannularrevertsfr
om its
normalsaddl e-shapedelli
psoidtoapl anarci rcul
arstr
uct urewhent heRVenl arges,f
ur t
her
di
scouragingappr opri
ateleaf
letcoaptation. 

Thedeci si onast owhent ooper ateinfuncti


onal TRi snotani nfrequentcl i
nicaldilemma.I tis
aCl assIi ndicationt oper f
ormt ricuspi
dvalver epairwhenr epai
ringorr eplacingthemi tr
alor
aorticvalveandt hereisassoci atedsevereTR.Whi lepacemaker sand/ ordefibri
llatorsarea
sour ceofwor seni ngTRwheni tispresent,t
herei snoi ndicati
ont orepairfunctionalTRpr i
orto
theirimpl antati
on.I ngener al,
funct i
onalTRr espondst or educingt hepulmonar ypr essures
whenpul monar yhyper tensionispr esent,
butitisnoti ndicatedwi thouttreati
ngt heRV
afterloadpr oblem.Fi nally,whil
er educingTRmayr educet hevolumeover l
oadont her i
ght
atri
um andr educeat ri
alar r
hythmi as,i
tiscurrentlynotconsi deredapr imar yindicat i
onfor
i
nter veningont het r
icuspidvalvei nfuncti
onal TR.

Quest
ion:

38. A43- year-


oldwomanwi t
hamechani calaor
ticval vereplacementf orendocar dit
isis
admitt
edwi thpulmonar yedemaf ol
lowing2weeksofpr ogressivedyspnea.Shehashada
l
onghi st
oryofpoormedi calcompl i
anceandt headmi ssioninternati
onalnormalizedr ati
o(INR)
was1.0.Onexami nation,sheisanxiousandi nrespiratorydistressandhypot ensive.Shei s
afebr
il
e,withal oudaor t
icregurgi
tantmur mur.Shei niti
allyr
espondst ocont i
nuousposi t
ive
ai
rwaypr essure,i
ntravenous( I
V)diuresis,andinotropes.Ahepar i
ndripisstart
ed.An
emergencyechocar diogram revealssever eaort
icstenosi sandr egur
git
ation,wit
ha
questi
onablelargemassont heaorti
cval ve.Afluoroscopi cviewoft heaor t
icvalvereveals
oneleafl
etfrozenint heopenposi ti
on.

Basedont
hecl
ini
calsi
tuat
ion,
whi
choft
hef
oll
owi
ngi
sthenextbestst
ep?

a. Beginstr
eptoki
nase.
b. Beginti
ssueplasminogenacti
vat
or.
c. Referf
oremer gencysurger
y.
d. Conti
nueIVhepar i
n.

Answer
:
38. c. Ref
erf
oremer
gencysur
ger
y.

Thecor rectansweri ssurgicalinterventi


on.I napat i
entwi t
hapr obablelargethrombusbur den
andevi dencef orsevereprostheticvalvedysf unct ionwi thNewYor kHear tAssociati
on(NYHA)
classI I
IorIVsympt oms, surgical r
emoval ofthet hrombusi sindicated.Atransesophageal
echocar di
ogr am al
sowoul dhavebeenusef ultochar acteri
zetheval veandpr ovi
dean
estimateoft hethrombussi ze.However ,insomecases, iti
ssimpl yeasiertousef l
uoroscopy;
i
nt hiscase,itwasdi agnostic.Fibrinol
ytictherapyi sreasonabl eonlyinthosewi thNYHA
functionalcl
assIorI Isympt oms.Ther i
skoff ibri
nol yictherapyisembol izat
ionofthe
thrombus, andt heri
skishigheri fthet hrombusi smobi l
eorover0. 8cm.I Vheparinshouldbe
continuedunt ilt
hetimeofsur geryandasabr idget ot herapeuticINR.

Quest
ion:

39. Youar econsul t


edtoseea58- year-ol
dmanwhounder wentcar diacsurgeryyester
day
forsevere,sympt omat i
caor t
icstenosis(AS)duet oabi cuspidval ve.Duringsurgery,he
recei
vesamechani calbil
eafl
etaor t
icvalve.Thesurgeonal soper formsr ootrepai
rt otr
eat
milddil
atationoft hethoracicaorta(mid-ascendingaorticdiamet er,45mm) .Apostoperat
ive
echocardiogr am showst hevalvetobef unctioningnormally(meangr adient,9mm Hg)wi tha
l
eftventri
cul arejecti
onfracti
on( LVEF)65%.Thepat i
ent’shospitalcour sehasbeen
uncompl i
cat ed.Thereisnohi storyofpri
orst rokeoratr
ialfi
bril
lation.Thesur gi
calteam has
askedyout oi ni
ti
ateanti
thrombot i
ctherapyf ortheaort
icvalvepr osthesis.

Whi
choft
hef
oll
owi
ngi
sthemostappr
opr
iat
enextst
ep?

a. Aspi
ri
n81mgdai
lyandwar
far
inwi
thani
nter
nat
ionalnor
mal
izedr
ati
o(I
NR)goalof2.
0
-
3.0.
b. Ri
varoxaban20mgdai
ly.
c. Dabi
gatran150mgtwi
cedail
y.
d. Warf
arinwit
hanINRgoalof2.
5-3.
5.
e. Warf
arinwit
hanINRgoalof3.
5-4.
5.

Answer
:

39. a. Aspir
in81mgdail
yandwar
far
inwi
thani
nter
nat
ionalnor
mal
izedr
ati
o(I
NR)
goalof2.
0-3.
0.

War far
inisindicatedwi t
hat argetINRof2. 0-3.
0inpat i
entswi thabi leafl
etorMedt ronic-
Hal l
mechani calaor t
icvalvepr osthesiswhodonothavehi gh-riskfeatures,suchasLVdysf unction,
atri
alfibri
ll
ati
on, pri
orthromboembol ism, andhypercoagul ablecondi ti
ons.Inpati
ent swith
high-ri
skfeatures,aSt arr-
Edwar dsval ve,
oradi scvalveinaddi itontoaMedt r
onic-Hallvalve,
war f
arinisindicatedwi t
hat argetINRof2. 5-
3.5.Theaddi tionofl ow-doseaspiri
nhasbeen
shownt odecreaset heriskoft hromboembol i
sm forpatient swit
hal lprosthet
icheartvalves.
Dabigat r
anandr i
varoxabanar enotindicatedforanti
thrombot i
ct herapyofprostheticheart
valves.

Quest
ion:

40. A71-
year
-ol
dmanr
etur
nsf
orf
oll
ow-
upofhi
spr
ost
het
icaor
ti
cval
ve.Hehada23mm
biopr
ostheti
cval veplaced16year sagoforendocar di
ti
sandhasdonewel l
.Hisinit
ial
echocardi
ogramsr eveal
edmi nimalgradi
ent,butoverthel
astfewyears,therehasbeen
progr
essiveevidencef orworseningaorti
cstenosis(AS)wit
hmi l
daor t
icregurgit
ati
on(AR).He
i
sactiveanddeni esanysympt omsofexer t
ionaldyspnea,
chestpressure,ordizzi
ness.Hehas
nocardiovascularsympt oms. 

Hisexami nationisnor
malexceptf orthemedianst er notomyscarandagr ade3/6mi d-peaking
systoli
cejectionmurmurandmi lddi
astoli
cmur mur .AnS4  isalsopresent
.Anechocar diogram
providesthef ol
lowi
nginformation:l
eftvent
ri
culareject ionfracti
on(LVEF)of55%, LVend-
diastol
e4. 9cm, LVend-systol
e2.3cm, andLVwal lthickness1. 1cm.Thepeaki nstant
aneous
aorti
cgr adienti
s4.3msecandt hemeanaor ti
cgr adienti s44mm Hg.Theaor t
icvalvear eais
2
0.9cm , withmi l
dAR,andt hereisnotri
cuspidregur gitati
onjettoesti
matepulmonar y
pressures.

Basedont
hesedat
a,whi
choft
hef
oll
owi
ngshoul
dbeyournextst
epf
ort
hispat
ient
?

a. Exerci
setest
ing.
b. Dobutaminestressechocardi
ogram.
c. Referforaor
ti
cvalvereplacement(AVR)
.
d. Tr
ansesophagealechocar di
ogram (
TEE)
.

Answer
:

40. a. Exer
ciset
est
ing.

Pati
entswit
hprosthet
icvalvestenosisaregener all
yhandledsimilar
lytot hosewi t
hnat i
ve
val
vestenosi
s.Thispati
enthassever eAS, yetdeniessymptoms.Hi sval vegradientsand
aort
icval
vearea(AVA)areconsi stentwithsevereAS, butar
enotsuper -severe(peakgr adi
ent
of5msec).Thus,anexercisetestisappropriatetoassesswhet herheist rul
yasympt omatic.
Thisshoul
dbedonewi thaphysi cianpresent.Thetestshouldbeconsi deredposi t
ivei fany
symptomsareprovoked,ifther
ei sanabnor mal hemodynami cresponse( i.
e.,
hypot ensionor
l
ackofar i
se>20mm Hgi nsystemi cbloodpr essur
ewi t
hstress,orSTabnor mali
ti
es
devel
op).
 

Byusingthismet hod,upto29%ofpat ientsthoughttobeasympt omati


cwi l
ldemonst rate
funct
ionalderangement.Attimes, t
hestenosisobservedaf t
ersur
ger
yissimplyduet otoo
smallavalvebeingusedatt het i
meofi mplantati
on.Thisisref
err
edtoas“patient
-prostheti
c
mismatch.”Becauseoft hi
s,itisimpor
tanttoal waysobtai
nanechocardiogram earl
yaf tert
he
surgi
calprocedure.Severemi smatchi
sconsi deredpresentinAVRwhent heAVAi s<0. 65
cm2. 

Atthecurrentt
ime,bothsurgi
calandpercutaneousAVRmaybeconsideredifhebecomes
sympt omati
corhasaposi t
ivefuncti
onal
study.Noadditi
onali
nfor
mati
onislikel
ytobe
gleanedfr
om aTEE, givent
heresultsoft
hechestwallechointhi
spar
ti
cularpati
ent.
Dobutaminestressechoisnotindicat
edashehasanel evat
edmeangradientconsi
stentwi
th
severeASandpr eservedLVfuncti
on. 

Quest
ion:

41. Youar
easkedt
oseea23-
year
-ol
dpr
egnantwomanwhoi
snear
ingt
heendofher
secondtri
mest er.Shehashadcont i
nuouspr enatalcare,butherphysici
annotedsheseemst o
behavingmor er espiratorysymptomst hanhewoul dhaveexpect ed.Heorder
edacar diac
echocardi
ogram, whichr eveal
edanewdi agnosisofsever emitr
alstenosi
s.Sheisst
ar t
edon
hydrochol
orothiazideandmet oprol
ol.Herbloodpr essureis115/60mm Hgandhear trateis
65bpm.Despi tet his,shecontinuestodemonst ratewor seni
ngdyspnea.

Whi
choft
hef
oll
owi
ngi
sthenextbestst
ep?

a. I
ncreasebeta-blockade.
b. Therapeuti
cabor t
ion.
c. Sur
gi calmi
tralvalverepl
acement
.
d. Bal
loonmi tr
alvalvulopl
asty.

Answer
:

41. d. Bal
loonmi
tr
alval
vul
opl
ast
y.

Thecorrectanswerisballoonval vuloplast
y,ift
hevalvei ssuitabl
ef orbal
loonvalvuloplasty,
ther
eisnoat r
ialcl
ot,andt hereisnosever emi t
ralregurgit
ation.Pregnancycompl icatesthe
careofpati
entswithvalvularhear tdisease.Asagener ali
ty,regurgi
tantvalvularl
esionsare
tol
erat
edmuchbet terthanst enoticlesions,aspregnancyi ncursamar kedr educt
ion
i
n aft
erl
oadandamar kedi ncreasei ncardiacoutputrequirement. 

Duri
ngthef i
rsttwot r
imesters,muchofthecardiacout
putisaccomplishedbyi ncreasi
ngt he
st
rokevolume, whilehear
tr atei
ncreasesdominatedur
ingthethi
rdtri
mest er
.Thegr adientin
mit
ralstenosisisdependentonbot hthestrokevol
umet hr
oughthevalveandt heamountof
di
astol
ictime.Ast heheartrateincr
easesinthethir
dtr
imester
,diast
olicti
mef all
s,andt he
gr
adientwor sensfurt
her.
 

TheFoodandDr ugAdmi nist


rati
onconsider shydrocholorothiazi
deacl assBagentand
appropri
ateinpregnancy.Al t
houghal lofthebet a-
blockersareconsi deredclassCdr ugs,they
arerouti
nelygivenandendor sedbyt hegui deli
nes.Ther earemanyst udiesvali
dati
ngt heuse
ofpercutaneousmi tralvalvulopl
astyduringpr egnancy,andt hereisli
ttl
er i
sktothefetusf r
om
theprocedureort heradiationduringthesecondandt hi
rdtrimester.Surgicalmit
ralvalve
repl
acementcanbeper formed, butatani ncreasedfetalri
sk,wi t
hthet i
meper iodbetweent he
20thand28t hweekr epresentingtheleastrisk.Ift
heval vei
samenabl et ocommi ssurotomy,a
percut
aneousappr oachi spreferr
ed.

Quest
ion:

42. A20- year-


oldwomanwi t
hahi storyofmitralval
verepair3yearsprior(formitr
al
regur
gitati
on[ MR]duet omi tr
alprolapse)isadmittedwithfeverofunknowncausef orone
week.Inretrospect,shehasbeenf eeli
ngpoorlyforatleast2mont hsandnot esher
temperaturetendstoi ncreaseintheevening.Shegener all
ycangett hetemperaturereduced
withacetaminophenandhasbeenabl etoconti
nuet ogot oschool.Shehasl ostafewpounds,
buthasnoot herspecifi
ccompl ai
nt s.
 

Sher ecent
lywentt
oanur gentcar
efaci
li
ty,
wher eshewaspr
escri
bedazithromycin.Shehas
takenthisoverf
ort
helast3days.Onexami nat
ion,shei
smil
dlyf
ebril
e(37.8°C)and
tachycardi
cat105bpm.Herbl oodpr
essureis100/75mm Hg.Herlungsareclear.Cardi
ac
exam reveal
snoj ugul
arvenousdistenti
on,noCVwave, andnohepatojugularref
lux.Her
carot
idisnormal.Apexishyperdynami cwithnothri
ll
s.Onauscult
ati
on,thereisagrade3/6
MRmur murradi
at i
ngtotheaxi
llaandt othebackwi t
hanassociat
edS3.Ther emainderofher
exam i
snor mal,
wi t
hnoper i
pheralsti
gmat aforendocar
dit
is.
 

Anechocar di
ogr
am isobtained,anditrevealsmoderatet osevereMRwi t
habr i
ghttarget
observedontheanteri
orleafl
etthatisoffi
ciallyr
eadasei thercal
cium orapossibleveget
ati
on.
Leftventr
icul
ar(
LV)functionisnor malasist heLVsize,andt her
earenoot herabnormali
ti
es
observed.Youordert
woset sofbl oodculturesandadmi thert ot
hehospital.Af
ter24hours,
nei
thersetofcul
turesispositi
veforanygr owt hofbacteri
a.

Whi
choft
hef
oll
owi
ngi
sthebestnextst
ep?

a. Begi
nampi ci
ll
inandgentami
cin.
b. Begi
nceftr
iaxone.
c. Repeatbl
oodcultur
es.
d. Begi
nvancomyci nandri
fampi
n.

Answer
:

42. c. Repeatbl
oodcul
tur
es.

Thecorr
ectansweristowaitunti
lbloodcul
tur
escanbeobt ainedof
fantibi
oti
csbefor
e
star
ti
ngempiri
ctherapy.Thepati
enthasevi
denceforfeverandacli
nicalscenar
ioi
nwhich
endocar
dit
isi
scertainl
ypossi
ble.Youaregi
vennopr i
orexamst oknowi ft
heMRi snewor
worsefr
om previ
ousexams. 

Thet argetonthemi tralvalvei snotdef i


niti
veandat ransesophagealechocar diogr am would
nowbei ndicat
edt obet t
erdef inethel esion.Sher eceivedant ibioti
csf r
om theur gentcar evisi
t.
Thus, thelackofpositivit
yoft hei nit
ial bl
oodcul t
uresmaysi mpl ybeduet otheant ibi
otic
therapyshehasr ecei
ved.Sher emainsst able,soant ibioti
csshoul dnotbei nit
iat edunt i
l
cultureshavebeenobt ainedt hathavenotbeenpot ential
lyinfluencedbyt hepr iorantibi
otic
therapy.Att hi
sstage, shedoesnotmeett hemodi f
iedDukecr i
teriaforendocar dit
is,anda
secondsetofcul t
uresshoul dbeobt ai nedandnot her apyinit
iatedunt ilt
heser esultsareback.
Iftheculturesthenbecomeposi t
ive,theappr opri
ateant i
bioti
cscanbeadmi nisteredort he
patientcanbeempi ri
callytreatedf orcul t
ur e-negati
veendocar dit
is.

Quest
ion:

43. A36- year-


oldwomani sseenf orevaluati
onofanewmur mur .Shei sactive,buthas
milddyspneaonexer t
ionandmi l
df atigue.Shehasnothadanor exiaorf ever.Shei sonno
medications.Sheisafebril
e,herbloodpr essureis132/ 60mm Hg, andhear trat
ei s90bpm.
Shehas5cm j ugularvenousdistenti
onwhenupr ight.Chestexaminat i
oniscl ear,andther
eis
agrade3/ 6systoli
cejecti
onmur muratt heleftuppersternalbor
der, al
ongwi thagr ade3/4
decrescendodiastoli
cmur muratt heleftlowersternalborder.
 

Anechocar diogram reveal


sabicuspidaort
icval
vewit
hsevereaort
icinsuffi
ciency.Thelef
t
vent
ricularend-diast
olicdi
mension(LVEDD)is6.0cm andl
eftvent
ri
cularend- syst
oli
c
dimension( LVESD)is5. 0cm.Ej
ecti
onf r
acti
onis55%.Nootherval
vularlesi
onsar enoted,and
t
heaor
ti
crootsi
zenor
mal
.

Whi
choft
hef
oll
owi
ngi
sthebestnextst
epf
ort
hispat
ient
?

a. Aorti
cvalvereplacement.
b. Nif
edipi
nesust ained-
rel
easeoral
ly30mgdai ly.
c. Tr
ansesophagealechocar di
ogram (TEE).
d. Li
sinopr
iloral
ly5mgdai l
y.
e. Repeattr
ansthor aci
cechocardi
ogram (TTE)i
n6mont
hs.

Answer
:

43. a. Aor
ti
cval
ver
epl
acement
.

Thecor
rectanswerissurgicalaort
icval
verepl
acement.Thecr i
ter
iaf
orsurgicalreplacement
i
nsevereaort
ici
nsuffi
ciencyincludeanLVEDD>6. 5cm,LVESD>5. 0cm, andejectionfracti
on
<50%orsymptoms.Afterloadreducti
oninnormotensi
vepatient
shasnotbeenshownt o
i
mproveoutcomesinpat i
entswithsevereaort
icr
egurgit
ati
on.TEEwillnotprovideaddi t
ional
i
nfor
mationfr
om thatseenont heTTE.

Quest
ion:

44. Whi l
etheval vul
arhear tdiseaseguidel
inesgener al
lyfocusont hei nt
erventi
oncr i
ter
ia
forindivi
dualvalvesandindividual l
esions(ei
therstenosisorregurgit
ati
on) ,somepat i
ents
presentwithmi xedvalvul
ardisease, wherebot hst
enosisandr egurgitat
ionar epresent.The
gradientsandr esult
antventr
icularvolumesar egreatl
yaf f
ectedbybot hr egurgi
tati
onand
stenosis,makingt hetr
adit
ional t
hresholdsforint
erventi
onl i
kelyunrel
iableinsomecases.

Accor
dingl
y,t
hel
atestval
vul
arhear
tdi
seasegui
del
i
nesat
tempt
edt
ohel
presol
vet
hei
ssue.

Basedont
hecur
rentvalvul
arhear
tdi
seasegui
del
ines,
whi
chgui
del
inecr
it
eri
afori
nter
vent
ion
shoul
dbeusedi
npatientswit
hmixedval
vul
ardi
sease?

a. Cr
it
eri
afort
heregurgitantrathert
hanthestenot
icl
esi
on.
b. Cr
it
eri
afort
hestenoticratherthantheregur
git
antl
esi
on.
c. Rel
yonsymptomsal one.
d. Cr
it
eri
afort
hedomi nantval vul
arl
esion.

Answer
:

44. d. Cr
it
eri
afort
hedomi
nantval
vul
arl
esi
on.

Thecor rectansweri stouset hecrit


eriaforthedomi nantabnor mal i
tyint heval ve.For
i
nst ance,ift herei
ssever eaor t
icst
enosis(AS)andonl ymi l
d-moder ateaor ticregurgit
ati
on
(AR) ,
thent heAScr it
eriaprevail
.Conversely,ifthereismi l
d-moder ateASwi t
hsever eAR, t
hen
theARcr i
ter iawouldpr evai
l.Manypat i
entswi thmi xedvalvedi seasedevel ophemodynami c
consequencesear l
ierthantheseverityofeitherl esi
onmi ghtpredi ct.Forinst ance,pulmonary
hyper t
ensionmi ghtbeexpect edearli
erinmi xedmi t
ralstenosisandmi tralregur gi
tati
onthan
wi t
heitherl esionalone.Simi l
arl
y,areducedl eftventri
cularend- systoli
cdi mensi onorejecti
on
fracti
onmi ghtbeexpect edearli
eri
nmi xedAS/ ARdi seasethanwi theitheral one. 
Thedeci si
ontoint
erveneoftencomesdownt oacl
ini
caljudgmentcall.Exerci
setest
ingmay
hel
pi fcardi
acli
mitati
oncanbedemonst ratedint
hoseappar ent
lyasympt omati
c.Alt
hough
unstudiedatthi
spoint,ot
herparameter
s,suchasbiomarkersmayal sobeofval ueinthe
fut
uretohel pdi
sti
nguishventri
cul
ardysfuncti
oni
nmi xedvalvul
ardisease.

Quest
ion:

45. Thehemodynami ct
raci
ngshowni
nFi
gur
e1mi
ghtbeseeni
nwhi
choft
hef
oll
owi
ng
congeni
talsyndr
omes?

a. Marfansyndrome.
b. Prader-
Will
isyndrome.
c. Holt-
Oram syndrome.
d. Shonec omplex.

Answer
:

45. d. Shonecompl
ex.
Thet racingshowni nFigure1depi ctsasi gni
ficantgr adientbetweent helef tventri
cleandt he
femor alar t
ery.Thi scanbeseenwi thhyper tr
ophi ccar diomyopat hy,aorticval vestenosis,
subval vularandsupr aval
vularaor ticstenosi s,andcoar ctati
onoft heaor t
a.Shonecompl exi s
associ atedwi thl ef
theartobstructivedi sease, includingsupr avalvularmi tralstenosisand
parachut emi tralvalve,aswellassubval vularaor ti
cst enosis,valvul
araor ticst enosis,
and
coarctat i
onoft heaor ta.Holt-
Or am syndr omei scausedbyamut ati
oni nt het ranscri
pti
on
factorTBX5andi sassociatedwi that r
ial septal defectsandr adioulnardef ect s.Prader-
Wi l
liis
acongeni talsyndr omeassoci atedwi thl ossoff unctionofgenesonchr omosome15, butdoes
nothavesi gnificantcardiacmani festations.Mar f
ansyndr omei sadi seaseoft heconnect i
ve
ti
ssuet hataffect smulti
pleorgansyst ems.Themostsi gnifi
cantcar diovascul armani f
estation
ofMar fansyndr omeisaor t
opat hy.

Quest
ion:

46. A44- year


-ol
dwomanwi
thr
heumat
icmi
tr
alst
enosi
sisbei
ngeval
uat
edf
ormi
tr
al
val
vul
opl
ast
y. 

Hercardiaccat heteri
zati
ondemonst r
atesthef ol
lowinghemodynami cs:pulmonaryartery
90/45mm Hgwi thameanof60mm Hg, pulmonarycapill
arywedgepr essure30mm Hg, l
eft
ventri
cularend-diastoli
cpressur
e5mm Hg, andcar diacoutput3.0l/
min.Thecalculatedmi t
ral
2
val
ve( MV)i s1.0cm .Echocar di
ogram revealsfusedMVcommi ssur
es, mil
dthi
ckeningofthe
mitrall
eafletsandchor dalappar
atus,f
ocalcal ci
fi
cationoft
ipoft heanteri
ormit
rall
eaf l
et,
and
mildmi t
ral r
egurgitat
ion.

Whi
choft
hef
oll
owi
ngi
sthenextbestst
epi
nhercar
e?

a. I
ntr
avenousepoprostenol
.
b. Per
cutaneousball
oonmi t
ral
val
vul
opl
ast
y.
c. Bosent
antherapy.
d. Sur
gicalMVreplacement.

Answer
:

46. b. Per
cut
aneousbal
loonmi
tr
alval
vul
opl
ast
y.

Herel evatedpulmonar yarter


ypr essuresarear esul
tofherchronicall
yelevatedwedge
pressur e.Withrel
iefofhermi t
ralstenosis,herwedgepressurewillli
kelynormali
zeandher
pulmonar yart
erypressureswi l
ldeclineoverthenextseveralmonths.Thi si
snotnecessar
ily
thecasewi thotherval
vularlesi
ons.HerMVanat omyisamenabl etoper cut
aneousther
apy,
andt hisisthetreatmentofchoi ce.SurgicalMVr epl
acementisappr opri
ateformorecal
cif
ied
valveanat omyorsi gnif
icantmitralregurgi
tati
on. 

Quest
ion:

47. A70- year-


oldmanwi thhi
stor
yofcoronaryart
erydisease,di
abet
es,andhyper
tensi
on
i
sadmi tt
edwithfeverandfati
gue.Hehashadr heumatoi
dar t
hri
ti
sfor20years,
whichhas
resul
tedinchroni
cmi t
ralandaort
icr
egurgi
tat
ion. 

Onexami
nat
ion,
hehasaswol
lenr
ightkneet
hati
swar
m andt
ender
.Car
diacexami
nat
ion
showsapulseof100bpm.Hisbloodpressur
eis120/
60mm Hg,
andhehasmi
ldaor
ti
cand
mitr
alr
egur
gitat
ion.Hi
stemper
aturei
s38.1°C.
 

Hisbasel
ineelectrocardiogr
am (ECG)showssi nusr hyt
hm andf ir
st-degr
eeheartbl
ock.
ReviewofpreviousECGshowst hatthef
ir
st-degreeheartblockisold.Anechocardiogr
am is
done,whi
chshowsmi l
dtomoder ateaor
ti
cr egurgi
tati
on,withanappar entveget
ati
onattached
totheaor
ticvalve.Bloodcultur
esar eposit
iveformet hi
cil
li
n-sensit
iveStaphyl
ococcusaureus
ontwooft wosampl es.Heisstart
edonnaf cil
li
nandgent ami ci
n. 

Atransesophagealechocardi
ogram i
sperf
ormed,andt
herei
sevidenceforaveget
ati
onof0.8
cm indiameterontheaorti
cvalve.Ther
eissi
gni
fi
cantaor
ti
cregurgi
tat
ionandquesti
onabl
e
evi
denceofapar avalvul
arabscess. 

Thepat i
entisst
art
edonanti
bioti
cs.Overthenext3days,
thetemperatur
eshowssome
decrease,andheissympt
omat i
call
yimproved.Onday6,thepat
ienthasarout
ineECG(
Figur
e
1).

Whi
choft
hef
oll
owi
ngdoyour
ecommendatt
hist
ime?

a. Diagnosticel
ectrophysiologystudy.
b. Broadenant i
biot
iccover age.
c. Surgicalaort
icvalvereplacement.
d. Permanentpacemakeri mpl ant
ati
on.
e. Cont
inuet
oobser
ve.

Answer
:

47. c. Sur
gicalaor
ti
cval
ver
epl
acement
.

Thepr esenceofcomplet eheartbl


ockisl
ikel
yduet oi
nvol vementofconduct i
onsyst em wit
h
perivalvulari
nvasi
onorabscess.Thi sisaClassIindi
cationf orurgentsur gicalinterventi
on.
Onewoul dnotpl
aceaper manentpacemakerinanactivelyinfect
edpat ientatt hist ime,
thoughat emporarypacemakermi ghtbeanopt i
onpriortosur gery.Therei snoval uein
obser vati
ononcet hediagnosisofanabscesshasbeenmade.Onemustassumei nfect
ious
i
nvasi onoft heconductionsystem i
nthepresenceofcompl eteheartblockeveni ft he
transesophagealechocar diogr
am doesnotconfir
m abscess.

Quest
ion:

48. A64- year


-ol
dhealthywomanwi t
haknownhear tmurmurpresent
swi
thr
ecenthi
stor
y
ofdyspneaonexerti
on(threebl
ocks).Coronaryangi
ogr
aphyperf
ormed10monthsago
reveal
ednoevidenceofcoronar
yarterydisease.
 

Echocar di
ogram datai
ncludecal culat
edej
ect
ionfract
ion65%, moderat
econcentr
icleft
vent
ricularhyper
trophy(wallthickness15mm) ,
bicuspidaort
icvalvewi
thpeakvelocit
y5
m/sec, aort
icval
vemeangr adient58mm Hg,andaor ti
cvalvearea0.9cm2.Midascending
aort
ai s5.0cm.

Whi
choft
hef
oll
owi
ngt
reat
ment
swoul
dyour
ecommend?

a. Ball
oonaor t
icval
vulopl
asty.
b. Me di
calmanagement .
c. Surgicalaor
ti
cvalverepl
acement(
AVR)andr
epl
acementoft
heascendi
ngaor
ta.
d. Transcathet
erAVR( TAVR).
e. SurgicalAVR.

Answer
:

48. c. Sur
gicalaor
ti
cval
ver
epl
acement(
AVR)andr
epl
acementoft
heascendi
ng
aor
ta.

Thepatienthasbicuspi
daorti
cvalve,severesympt omati
caorti
cstenosis(AS),andamaxi
mal
ascendi
ngaor ti
cdiameterof>4.
5cm.Sur gicaltr
eatmentofbot
hdisordersisaCl assI
recommendat i
on.Thereisnorol
ef ormedi calmanagementinseveresympt omaticAS.
Bicuspi
daor t
icval
veandassociatedaortopat hyneedtobeaddressedsurgical
ly.
 

Currentrecommendati
onsaret
orepl
acetheascendi ngaort
aif>5.5cm orexhibi
tsagrowth
rateof>0.5cm/year.I
faor
ti
cval
vesurgeryisbei
ngunder t
aken,t
her ecommendat i
onisto
replacetheaor
taif>4.
5cm.Ball
oonaorti
cvalvul
oplastyi
ssomet i
mesr eser
vedasabr idgeto
surgeryinunst
ablepati
ent
sorasapall
iativemeasurefort
heinoperablepat
ient(Cl
assIIb)
.
Finall
y,TAVRhasnotyetbeenappr
ovedf orcl
ini
calusefortr
eatmentofbicuspidaort
icvalve.
Quest
ion:

49. A52- year-


oldwomani sref
erredf
oreval
uat
ionofahear
tmur
mur
.Ar
epr
esent
ati
ve
tr
ansesophagealechocar
diogram i
mageisshowninFigur
e1.

Whi
choft
hef
oll
owi
ngi
sthemostl
ikel
ydi
agnosi
s?

a. Parachutemit
ralval
ve.
b. Fl
ailmitr
all
eafl
et.
c. Subaorti
cmembr ane.
d. Systol
icant
eri
ormot i
onoft
hemi
tr
alval
ve.

Answer
:

49. b. Fl
ailmi
tr
all
eaf
let
.

Thei
mageshowsf
lai
loft
heA2 
scal
lopoft
hemi
tr
alval
ve.

Quest
ion:

50. A52-year-oldwomani sr
eferr
edtotheechocar di
ographylabor
ator
yforahear
tmurmur
.
Shei
sasymptomat i
c.Sheisonnomedi cat
ion.Herbloodpressureis130/
75mm Hg.Figur
e1
i
sthepar
ast
ernallongaxisvi
ew.Figure2isatransesophagealimage. 
Theechocar
diogr
am showsaf l
ail
mi trall
eafl
etispresent
.Leftventr
icul
ar(LV)end-
syst
oli
c
di
mensionis45mm, LVej ect
ionfracti
oni s55%,meangr adienti
s3mm Hg, peakvel
oci
tyis
1.
5msec, mitr
alr
egurgi
tantvolumei s100ml /beat,
effect
iveregurgi
tantor
ifi
ceareai
s0.67
cm2.Est
imatedpulmonaryart
erialpressure(r
est)i
s45mm Hg.

Whi
choft
hef
oll
owi
ngwoul
dbet
heappr
opr
iat
enextst
ep?

 
Figur
e1
Fi
gur
e2
a. Fol
lowser ial
lybyechocar diogr
aphy.
b. Surgicalmitr
alvalverepair.
c. Percutaneousmi tr
alvalverepair
.
d. Beginlisi
nopril20mg/ day.

Answer
:

50. b. Sur
gicalmi
tr
alval
ver
epai
r.

Thepatientmeetsthe2014valvularhear
tdiseaseguidel
inecri
ter
iaforseveremitr
alval
ve
regur
git
ation.Shealsomeet
sgui deli
necri
teri
aforint
erventi
onwithLVend- syst
oli
cdimensi
on
of>4.0cm andLVej ect
ionf
racti
onof55%.

Quest
ion:

51. A52- year-


oldwomani srefer
redtotheechocar
diogr
aphyl
abor
ator
yforaheartmurmur.
Sheisasymptomati
c.Herbloodpressur
eis130/75mm Hg.Figur
e1istheparast
ernall
ong-
axi
sview.Fi
gure2isatransesophageali
mage. 

Leftventr
icul
ar(
LV)end-syst
oli
cdimensi
onis45mm,LVejecti
onfracti
on(EF)is55%,
mean
gradienti
s3mm Hg, peakveloci
tyi
s1.5msec,mi
tr
alr
egur
gitantvolume(RV)is100ml/beat
,
andef f
ect
iver
egurgit
antori
fi
cearea( EROA)i
s0.67cm2.Est
imat
edpul
monar
yar
ter
ial
pressure(
rest
)is45mm Hg.
Whichofthefoll
owingisthebestrecommendati
onfort
reatment
?
 

Fi
gur
e1
Fi
gur
e2
a. Referralformit
ralvalvesurgery.
b. Continuedsurveil
lance.
c. Percutaneousballoonvalvuloplast
y.
d. Vasodi l
atort
herapy.

Answer
:

51. a. Ref
err
alf
ormi
tr
alval
vesur
ger
y.

TheMRi ssever e(RV>60cc;EROA>0. 40cm2)andsecondar ytoflailanteri


orleafl
et.TheEFis
mildlydepressed,andtheend-
systol
icdimensionis>40mm.I nthepr esenceofmi ldly
depressedEF, therei
snoadvantaget omedi
cal management .Thepeakvel oci
tyacrossthe
mitralval
veisincreasedduetosevereMRandnotst enosis,renderingvalvuloplast
y
i
nappr opri
ate.TheAHA/ ACCguideli
neonvalvularheartdiseasegivesaCl assIindicati
onfor
asympt omatic,sever
eMRandmi l
dt omoderateLVdysf unction(EF30- 60%)orLVend- syst
oli
c
dimension>40mm.

Quest
ion:

52. A29- year


-ol
dmani st
ransf
err
edwithent
erococcusfaecal
isendocar
dit
is.Hehasbeen
onappr
opr
iateanti
biot
ict
her
apyfor3weeks.Hi
sphysicalexaminat
ionshowshei saf
ebri
le.
Hi
shear tr
atei
s100bpm, bl
oodpressurei
s100/60mm Hg,andjugul
arvenouspressurei
s12
cm H2O.Hehasagrade3/6pansystol
icmurmuratapexandmil
dbibasi
larcr
ackl
es.Figur
es1,
2,
and3ar eobtai
ned.Cal
cul
atedej
ecti
onfract
ion(
EF)is36%.

Whi
choft
hef
oll
owi
ngi
stheappr
opr
iat
emanagementf
ort
hispat
ient
?

Fi
gur
e1
Fi
gur
e2
Fi
gur
e3

a. Af
terl
oadreducti
ont her
apy.
b. Bet
a-bl
ockertherapy.
c. Pr
omptmi tr
alvalverepl
acement.
d. Fi
nishthecourseofantibi
oti
csandr
e-eval
uat
e.

Answer
:

52. c. Pr
omptmi
tr
alval
ver
epl
acement
.

Thepatienthassever emi t
ralr
egurgitat
ion,signi
fi
cantlef
tventr
icul
ardysfuncti
on,
and
evi
dencef orhear
tf ai
lure.Thetachycardiaisacompensat orymechanism andshouldnotbe
tr
eated.Thereisnor oleforvasodil
atortherapyinthi
ssetti
ngashi ssyst
olicbl
oodpressur
eis
only100mm Hg, andt r
eatmentoft hemi t
ralregurgi
tat
ionshoul
dbesur gi
cal. 

Quest
ion:

53. A22- year-


oldmanwi thfamili
alMarf
ansyndromepresent
swithdyspneaonexer
ti
on.
Fouryearspri
ortopresentat
ion,heunder
wentaort
icrootr
epairwi
thaort
icval
ve(AV)
resuspensi
on(val
ve-spar
ingsurgery).
 
Physicalexami nationreveal
sahi gharchedpalat
e,ar
achnodactyl
y,andaseverepectus
excavatum.Bl oodpr essureis134/34mm Hg.Thepoi ntofmaximalimpul
seisdisplaced
i
nferolat
erall
y.Ther eisasyst ol
iccl
ick.Agrade4/6harsh,l
ate-
peaki
ngsystol
iceject
ion
murmuri sauscul tatedacrosstheprecordi
um.Thereisagrade4/4holodi
astoli
cmur murat
thelowerleftsternalborder.
 

Fi
gur
e1showsacont
inuouswaveDoppl
eracr
osst
heAV.
 

Lef
tvent
ri
cul
aroutfl
owt
ract(
LVOT)di
ameteri
s2.4,LVOTt
imevel
oci
tyi
ntegr
al(
TVI
)is26,
meanAVgradienti
s87mm Hg,andAVTVIis167.

Yourr
ecommendat
iont
othepat
ienti
swhi
choft
hef
oll
owi
ng?

a. Aorti
cvalver eplacement(AVR)
.
b. Rosspr ocedur e.
c. TranscatheterAVR( TAVR)
.
d. Continuedsur veil
lance.
Answer
:

53. a. Aor
ti
cval
ver
epl
acement(
AVR)
.

Thepat i
enthassevereaorti
cstenosi
s(andaort
icregurgi
tat
ion)andissympt omat
ic,whi
chi
s
aClassIindicat
ionforAVR.Thereisnorolef
ormedi calmanagementint hi
sscenari
o.ARoss
procedurewouldbet echni
call
yimpossi
blewi
thpreviousaorti
crootsurgery.TAVRisnot
approvedforthi
sindicati
on.

Quest
ion:

54. A22- year-


oldmanwi thfamili
alMarf
ansyndromepresent
swithdyspneaonexer
ti
on.
Fouryearspri
ortopresentat
ion,heunder
wentaort
icrootr
epairwi
thaort
icval
ve(AV)
resuspensi
on(val
ve-spar
ingsurgery).
 

Physicalexami nationreveal
sahi gharchedpalat
e,ar
achnodactyl
y,andaseverepectus
excavatum.Bl oodpr essureis134/34mm Hg.Thepoi ntofmaximalimpul
seisdisplaced
i
nferolat
erall
y.Ther eisasyst ol
iccl
ick.Agrade4/6harsh,l
ate-
peaki
ngsystol
iceject
ion
murmuri sauscul tatedacrosstheprecordi
um.Thereisagrade4/6holodi
astoli
cmur murat
thelowerleftsternalborder.
 

Fi
gur
e1showsacont
inuouswaveDoppl
eracr
osst
heAV.
 

Lef
tvent
ri
cul
aroutfl
owt
ract(
LVOT)di
ameteri
s2.4,LVOTt
imevel
oci
tyi
ntegr
al(
TVI
)is26,
meanAVgradienti
s87mm Hg,andAVTVIis167.

Whi
choft
hef
oll
owi
ngi
sthecor
rectaor
ti
cval
vear
ea(
AVA)i
nthi
spat
ient
?
a. 0.
6cm2.
b. 1cm2.
7cm2.
c. 0.
9cm2.
d. 0.

Answer
:

54. c. 7cm2.
0.

AVAiscalcul
atedbythecont
inui
tyequat
ion.(
ASjet=aor
ti
cstenosi
sjet
)AVAxTVIASjet=
LVOTareaxTVILVOT.ThusAVA=LVOTar eaxLVOTTVI/TVIASjet=0.785xD2xTVI
LVOT/TVIASjet=4.52x26/167=0.7cm2.

Quest
ion:

55. An85- year-


oldwomanwi thsevereaorticstenosisandmul t
iplesurgi
calri
skfact
orsis
dyspneicatr
est,butmental
lycompetent.Sheisr efusedcardi
acsur ger
yduet othemulti
ple
comorbidi
ti
esandi sbei
ngevaluatedforpossibletranscat
heteraort
icvalverepl
acement
(TAVR).Acoronaryart
erycomputedtomogr aphicangi ogr
am ofheraor t
aandr unof
fvessel
sis
perf
ormed. 
Findi
ngsincl
udeaort
icannul
usdimensionsof21mm x28mm.Mi ni
mum il
iof
emoral
measurementsare4.8mm ontheleftand5.2mm onther
ight
.Therei
sseverecal
cif
icat
ionof
theaort
afrom t
herenalar
ter
iest
ot heil
iacbi
fur
cat
ion.

Whichofthefol
lowingconcl
usi
onscanbedr
awnf
rom t
hesedat
ainr
egar
dtot
heuseofa
t
ranscat
heterapproacht
oAVR?

a. Sur
gicalAVRi sthebestoption.
b. Anonfemor alapproachforTAVRisthebestopti
on.
c. TAVRispr ecl
udedbyt hedegreeofaort
iccal
cif
icati
on.
d. Astandardfemor alappr
oachTAVRappr oachisfeasi
bleandt
hebestopt
ion.
e. TAVRnotf easibl
eduet otheaort
icannul
arsi
ze.

Answer
:

55. b. Anonf
emor
alappr
oachf
orTAVRi
sthebestopt
ion.

Comput edtomogr aphicangi ographyisthegoldstandardforannularmeasur ement .The


pati
ent’sannularsizewoul dnotpr ecludeTAVR, butherminimum il
iofemoraldiamet er
swoul d
precl
udet hetr
ansf emor alapproach.Thedegr eeofcalci
fi
cationoftheabdomi nalaor t
aisnot
i
ni t
selfacontraindicat
ion,butthepr esenceofextremetortuosi
tyoraneurysm woul dr equi
re
caref
ulconsideration.Shedoesnotwi shfort
hesur gi
calopti
on.Adi r
ectaorti
c,apical,or
subclavianapproachmi ghtbef easible.

Quest
ion:

56. A35- year


-oldwomanpr esent
sat24weeksgest at
ion,newt oyourpract
ice.Shehasa
hi
storyofr heumaticheartdi
sease,alt
houghshecannotr ecallwhichvalveisaf
fected.She
stoppedallmedicationsatthetimeofconception.Shewasr eferr
edduet oincr
easingf at
igue
overthepastmont h.Oneval uati
on,herhear
trate(HR)isregularat100bpm andbl ood
pressureis105/73mm Hg.Acar di
acexaminationreveal
sapr ominentS1,anopeningsnap,
andagr ade2/ 4diastol
icmur murattheapexthatdoesnotr adiat
e.HerS2 i
snor mal,andlungs
areclear.Noechocar di
ogram isavail
abl
etoday.

I
nconsi
der
ingt
het
her
apeut
icopt
ions,
whi
choft
hef
oll
owi
ngdoyouadvi
se?

a. Beginangiotensi
n- converti
ngenzymei
nhi
bit
or.
b. El
ectivet
ermi nat
ionofpr egnancy.
c. Eme r
gentbal l
oonval vuloplast
y.
d. Surgi
calmi tr
alvalver epair
.
e. Beginbeta-bl
ocker .

Answer
:

56. e. Begi
nbet
a-bl
ocker
.

Thi
spati
ent'
sexami nat
ionfi
ndingsareconsistentwithmitr
alst enosi
s,andthepreferredini
ti
al
managementstrategywoul dbeHRsl owi
ngwi thabet a-
blocker.Aspr egnancyprogresses,t
he
HRincr
easesandr esul
tsindecreaseddi
astoli
cf i
ll
ingwit
hi ncreasingleftat
ri
alpressures.
Managementincludesmai ntenanceofsi
nusr hythm andcont r
olofHRt opromotetimef or
adequatefil
li
ng.I
nvasivestr
ategi
esarereser
vedf orsi
tuat
ionswi
thseverecompr
omi
se
unresponsivetomedicalt
herapy.Angi
otensi
n-convert
ingenzymei
nhibi
torsar
e
contrai
ndicat
edinpregnancy. 

Quest
ion:

57. A55- year-


oldmanhasahi stor
yofhypertensi
on.Overthepast6months,hehas
noticeddecreasedexer
cisetol
erance.Heget sshortofbreathonwalki
ngamile.On
exami nat
ion,hi
sbloodpressur
ei s140/ 60mm Hg.Hi spulseis80bpm wi
thnosignsofhear
t
fai
lure.Hehasamur murofaort
icr egurgi
tat
ion(
AR) .
 

Hisel
ect
rocar
diogr
am showslef
tventri
cularhyper
trophy(
LVH)
.Hi
svent
il
ati
on/
per
fusi
on
scani
snormal.Pul
monaryfunct
iontestsarenormal. 

Echocar
diogr
aphyshowsmi
ldLVH,
sever
eAR,
eject
ionf
ract
ionof50%,
andLVend-
diast
oli
c
di
mensionof60mm.

Whi
choft
hef
oll
owi
ngi
sthebestopt
ion?

a. Aorti
cvalvereplacement(AVR)
.
b. Li
sinopri
l.
c. Metoprol
ol.
d. Conti
nuedsur veil
lance.
e. Nif
edipi
ne.

Answer
:

57. a. Aor
ti
cval
ver
epl
acement(
AVR)
.

ClassIindicat
ionsf orAVRar e:1)sympt omati
cpatientswi t
hsevereARi r
respect
iveofLV
systol
icfuncti
on( Level ofEvidenceB),2)asymptomat i
cpatientswi
thchronicsevereARand
LVsystolicdysfunct i
on( ej
ectionfr
acti
on<50%)atr est(LevelofEvi
denceB) ,
and3)patient
s
withchronicsevereARwhi l
eunder goi
ngcoronaryarter
ybypassgr aft
ingorsurgeryonthe
aortaorotherhear tvalves(LevelofEvidenceC).
Quest
ion:

58. Whichoft
hef
oll
owi
ngi
sthemostl
ikel
ycausef
ort
hehemodynami
cfi
ndi
ngsi
nFi
gur
es
1and2?

Fi
gur
e1
Fi
gur
e2
a. Peri
cardi
altamponade.
b. Tr
icuspidregurgi
tat
ion.
c. Pr
imarypul monaryhypert
ensi
on.
d. Tr
icuspidstenosi
s.

Answer
:

58. b. Tr
icuspi
dregur
git
ati
on.

Thesimilar
it
ybetweent
her i
ghtatri
alandrightventri
cularpressur
etraci
ngsi
ndicatesthatt
he
tr
icuspi
dvalveisal
mostfuncti
onall
yabsent ,meaningthatitisopenduri
ngbothdiastol
eand
systol
e.Thus,t
herei
sseveretr
icuspidregurgit
ati
on.

Quest
ion:

59. A52- year-


oldwomanpr esent
sintheambulat
orycli
nicf
orherannualgeneralmedi
cal
examinati
on.Sherepor
tsnosympt oms.Thepati
entt
akeshydrochlor
othi
azi
de( 12.
5mgdai l
y)
forhyper
tensi
onandatorvast
atin(10mgdaily)f
orahist
oryofhyperl
ipi
demia.Her 
brot
herhad
anaorti
cvalverepl
acementatage50year s.

Onphysi
calexami
nat
ion,
thebl
oodpr
essur
eis110/
70mm Hgandhear
trat
eis80bpm.The
headandneckexami nat
ioni snormal
.Lungsar ecleartoauscult
ati
on.Caroti
dupst r
okei s
normal.Agrade1/ 6earl
y-peakingsyst
olicejecti
onmur murisheardattheleftlowersternal
border
.Themur murdi minishessli
ghtl
ywi t
hsquat -
to-
standmaneuver.TheS2  isnor
mal .There
arenoclicksorrubs.S3 
andS4  ar
eabsent.Peripher
alpulsesareint
act.Thereisnoper i
pheral
edema.
Thepatientasksaboutherneedf orfurt
herevaluati
onandt hepotenti
alforvalvesurgeryinthe
fut
ure.
 

Whi
choft
hef
oll
owi
ngi
sthenextbestst
ep?

a. Calcium scoreassessment .
b. Nof urthert
estingrequir
ed.
c. Exercisestresstest.
d. Transthoracicechocardiogr
am.
e. Cardiacmagnet i
cresonanceimagi
ng.

Answer
:

59. d. Tr
anst
hor
aci
cechocar
diogr
am.

Thepat
ienti
sasympt
omat
ic,
andherphysi
cal
findi
ngsar
econsi
stentwi
thani
nnocent
murmur.
 

Charact eri
sticsofani nnocentmurmurareasof t(gr
ade1or2)ej ecti
onmur murt hatpeaks
earl
yi nsyst ole,withoutevidencef
orothercardiacpathol
ogy(i.
e.,normalapicalimpulseand
caroti
dupst r oke;nodi astoli
cmurmur,S3 
orS4,cli
cksorrubs,si
gnsorsympt omsofhear t
fai
lure).However ,becauseoft hef
amilyhist
oryofapr obablebi
cuspi daort
icvalveandthe
mur mur ,sheshoul dbescr eenedf
orthepresenceofabi cuspi
dval vewithechocardiogr
aphy.
Noneoft heot herst udiesareappr
opri
ate.
 

Quest
ion:

60. A55- year-


oldmani sadmi t
tedtot hehospi talf
orevaluationandmanagementof
fati
gueandl owerext r
emi t
yedema.Hef i
rstnot edhissympt oms4mont hsago.Thepatient
beli
eveshehasgai nedmor ethan25pounds( 11.4kg)sincethattime.Thereisnoassociated
chestdiscomf or
t,hist
oryofpal pi
tati
ons, f
ever ,orconsti
tuti
onalsympt oms.Thepati
entisa
non-insuli
n-dependentdi abeti
c.Hehasbeent r
eatedforhyper t
ensionfor8yearsonstable
dosesofl isi
nopr i
l(10mgdai l
y)andaml odipine(10mgdai l
y).Twoyear sago,heunderwent
pacemakerpl acementf orsympt omat i
cbr adycardia.Thepat i
entalsohasbeent r
eat
edf or
obstructi
vesl eepapnea;hi slastevaluati
on6mont hsagoshowedadequat efitandf
unct i
onof
hiscont i
nuouspost i
ti
veairwaypr essure machi ne.Hisregularphysici
anprescri
bedloop
diureti
cs2mont hsagowi thoutanybenef icialeff
ect. 

Onphysicalexami nati
on,thepatientisobese( bodymassi ndex35.2kg/ m2).Bloodpressurei
s
128/72mm Hgwi thahear trateof72bpm.Thel ungsar eclear.Thejugularvenouspulseis
l
ocatedatthel evel ofthejawwi t
hpat i
entsitt
ingupr i
ght.Ther eisaprominentV  wave.The
carot
idcontouri snor mal.Precor
di al
palpati
onr evealsasl i
ghtlif
t(1+)attheleftlowerster
nal
border
.A2/ 6bl owi ngholosystol
icmur murispr esentint hi
sregionwi t
hanear lydiast
oli
c
fi
ll
ingsoundthataccent uatesduringinspir
ati
on.Apul satil
eli
verispresent.Onper i
pheral
examinati
on,ther eissevereedema( 3+)invol
vingt hethighsandl owerlegs. 
ChestX- rayrevealscardiomegalywi t
hmi l
denlargementoftheri
ghtventr
icularcontour.The
l
ungf ieldsshownoedema.At ransthoracicechocardi
ogram (
TTE)showsnor malleft
ventri
cul arsystoli
cfuncti
on(ej
ectionfracti
on65%)andsever etr
icuspi
dregur gi
tat
ion(TR)by
colorflowDoppl er
.Mild-
moderatesyst ol
icdysfuncti
onandenlar
gementoft herightventri
cle
i
spr esent .Esti
mat edri
ghtvent
ricularsystol
icpressurei
s30mm Hg.Doppl er
echocar diographyofthehepaticveinsshowsnoexpi rat
oryr
ever
sal.

Whi
chi
fthef
oll
owi
ngi
sthemostl
ikel
ycauseoft
heTR?

a. Pul
monar yhypertension.
b. Restri
cti
vecardi
omyopat hy.
c. Cons t
ri
cti
veperi
caridit
is.
d. Pacemakerleadimpi ngement.
e. Ebstei
n'sanomaly.

Answer
:

60. d. Pacemakerl
eadi
mpi
ngement
.

Thepat i
enthasapr imaryabnor mal it
yint hetri
cuspidvalve.Itmayber el
at edt ori
ght
ventri
cularsystoli
cdysf unctionwi t
hr etr
actionofthetri
cuspi dval
vechor daeandi ncreased
annulus.Thepat i
entpresentswi thsympt omsandsi gnsofr i
ghtheartfailur e.Thedifferenti
al
diagnosisincl
udesconst r
icti
veper icardi
tis,astr
uctural
lyabnor maltri
cuspi dval ve(suchas
Ebstein'
sorpr olapse),
andsecondar yTRduet opulmonar yhypertensi
on( suchassl eepapnea
orrestri
cti
vecar diomyopat hy).Recent l
y,therehasbeeni ncreasedrecogni tionoft hepot ent
ial
forsevereTRcausedbydevi celeadimpl antati
onintotherightventri
cle. 

TTEmaynotdet ectsuchleafletimpingement ,andtransesophagealechocar diographyshould


bepur suedi nsuspectedpat i
ents.Inast udyof41pat ientswhounder wenttr
icuspidsurgery
fordeviceleafletimpingementatt heMayoCl i
nic,onlyfive(12%)ofthepat i
entshadan
accuratepr eoperati
vediagnosisonTTE.Dat afrom theDoppl erinter
rogati
onoft hehepatic
veindonotsuppor tthepresenceofconst r
ict
ivepericardit
is.Therewasnocommentabout
apicaldisplacementoft heseptal orposteri
ortri
cuspidleaflet
sthatwoul dbeseeni nEbstein'
s
anomal y.Thepul monar ypressureswer enormal byecho/ Doppler,suggesti
ngnosecondar y
causef ortheTR.

Quest
ion:

61. A30- year-


oldmani sr efer
redf orevaluationofamur mur .Her eport
snosympt oms, and
hasnosi gnifi
cantmedicalhistory.Onphysi cal exami nati
on,hishei ghtis69i nchesandwei ght
i
s185l bs.Thebl oodpressurei s135/ 70mm Hgwi thar egularhear trateof78bpm.Hi slungs
areclear.Prominentcarot
idpul sati
onsar epr esent.Thej ugularvenouspul seisatthel evelof
thesternalnotch.Theapicalimpul seisslightlyenlargedandl aterallydisplacedtotheant eri
or
axil
lar
yl i
ne.TheS1 andS2 arenor mal ,andanS3  ispresent.Therei sanear lysystol
icclickthat
doesnotchangewi thi
nspiration.Bot hasof t(grade2/ 6)crescendo- decrescendosyst oli
c
mur murandasof t(
grade2/ 4)decr escendodi astoli
cmur murarepr esentalongtheleftsternal
border.
Whi
choft
hef
oll
owi
ngi
sthemostl
ikel
yval
vul
arabnor
mal
it
y?

a. Patentductusarteri
osus.
b. Bi
cuspidaor t
icvalve(BAV)wi t
hregurgit
ati
on.
c. Pul
moni cvalvestenosiswithregurgi
tati
on.
d. Rheumat i
cmi t
ralstenosisandregurgit
ati
on.
e. Degenerati
veaorticvalvestenosi
swi t
hregurgi
tat
ion.

Answer
:

61. b. Bi
cuspi
daor
ti
cval
ve(
BAV)wi
thr
egur
git
ati
on.

Pulmoni cvalvestenosisisunl i
kelybecauset hemur murdoesnotchangewi thr espirationand
thejugularvenouspul sei snormal .Thei ntensityofthepul moni cejecti
oncl i
ckdecr easeswi t
h
i
nspirati
on.Degener ati
veaor ti
cst enosisi snotassoci atedwi t
hasyst oli
ccl i
ck,andhei sonl
y
30year sold.Rheumat icmi tr
alstenosi smaybeassoci atedwi thadi astoli
cclick,butnotone
thatoccur sduri
ngsyst ole.Patentduct usar teri
osuspr oducesacont inuousmachi ne-l
ike
mur murandi snotassoci at
edwi t
hacl ickorwi depul sepr essur e.BAVcanbeassoci atedwi t
h
suddencessat ionofval veopening, leadingt oanej ecti
onsyst oli
ccl i
ck,andof teni s
associatedwi t
hsignif
icantAVr egurgitation.Findingsofchr onicaor ti
cr egurgit
ationwi thlef
t
ventri
cularvolumeover loadincludeanenl ar
gedandl aterall
ydi splacedapi calpulse, wide
aorti
cpul sepressure,andanS3.

Quest
ion:

62. An80- year-


oldmani sadmi t
tedtothehospitalf
oranacut eexacerbati
onofhear tfail
ure.
Thepat i
enthasaknownhi st
oryofaor t
icstenosis,
buthasnotsoughtr ecentmedi calcare.For
thepast3year s,hehashadexer ti
onaldyspnea.Hissymptomshavei ncreasedsignifi
cantly
overthepastmont h,
andnowar eaccompani edbynoctur
nalsympt omsandor t
hopnea.Ther e
arenocompl aintsofchestdi scomfortorsyncope.Thepatienthasahi st
oryofpr i
orst r
oke,
diabetesmell
itus,andt r
eatedhypertension.Thereisnoknownhi st
oryofcor onaryartery
diseaseoratr
ial ar
rhythmia. 

Onphysi calexami nation,heappear sfr


ail
.Thepat ientis160cm t al
landwei ghs70. 2kg.Hi s
bloodpr essur eis104/ 72mm Hgwi thaheartrateof80bpm.Di ffusecr acklesarepr esentin
bothl ungf ields.Thej ugularvenouspul seiselevatedtot heangl eoft hej awwitht hepatient
sitt
ingupr i
ght .Thecar oti
dpul sesarediminishedanddel ayedincont our.Hisapicalimpul sei
s
displacedt ot heant er
ioraxi
ll
ar yl
ine.Asoft,mid-peakingsyst oli
cej ecti
onmur muri shear dat
ther i
ghtupperst ernalborder.Theaor t
iccomponentoft hesecondhear tsoundissl ightl
y
diminished, whilethepul moniccomponenti ssignifi
cantl
yi ncreased.At hir
dhear tsoundi s
present .Per i
pher aledema( 2+)ofbot hlowerext r
emi t
iesisnot ed. 

Transt
horacicechocardiographyshowsal eftvent
ri
cularej
ectionf r
acti
onof35%wi thadi l
ated
l
eftventr
icle(end-di
astol
icdiamet er
,58mm) .Themeanaor t
icval vegradi
entis24mm Hg
withavalvear eaof0.7cm2 derivedfrom t
hecont i
nui
tyequation.Car di
acoutputiscal cul
ated
tobe3.2 L/min.Moder at
etricuspidregur
gitati
onispresentwithanest imat
edr i
ghtvent ri
cular
systol
icpressureof62mm Hg.Ther emainderoftheechocardi ogram i
sunremar kable.
I
ntr
avenousdi
uresi
sisi
nit
iat
ed,
andsympt
omsi
mpr
ove.

Whi
choft
hef
oll
owi
ngi
sthemostappr
opr
iat
enextst
ep?

a. Dobut aminest
ressechocardiogram.
b. Cardiacmagneticresonancei magi
ng(MRI)
.
c. Rightheartcat
heteri
zati
on.
d. Transesophagealechocardiogram (
TEE)
.
e. Exercisestr
essechocardiogram.

Answer
:

62. a. Dobut
ami
nest
ressechocar
diogr
am.

Thepat ienthasevi denceofl ow- out put ,low- gradi entaor t


icst enosi s.Thi sent itycanbedi fficult
todi sti
ngui shf r
om pat i
entswhohavemi l
dormoder ateaor ticst enosiswi thl owcar di
acout put
duet oanet iologyothert hant heaor t
icval velesi on.I ti si mpor tantt onot et hatt heval vear ea
i
sf low- dependent .Thel owval vear eai nt hispat ient( 0.7cm2)maybeduet oei ther
anat omi callysever eaor ti
cstenosi s,oracar diomyopat hyi nwhi chdecr easedcont ractil
it
y
l
eadst oadecr easeinaor t
icvalveopeni ng.Themostappr opr i
atenextst epi sadobut ami ne
stressst udy, whichmaybeper formedwi thechocar diogr aphyori nt hecar di accat heter i
zat i
on
l
abor atory.Thegoaloft hedobut ami nest udyi st oexami net hesever i
tyoft heaor ti
cst enosi s
withani ncr easei ntransvalvularf lowwi thmeangr adi ent>40mm Hg.Pat ient swi thsever e,
i
ntr i
nsi caor ticstenosist ypi
cal l
ywi l
lexhi bitahi ghaor ti
cgr adientwi thaper sistentlylowval ve
areawi thdobut amineadmi nistration.Conver sel y,inpat ientswi thoutanat omi callysever e
aor t
icst enosi s,theaor t
icvalvear eawi llincreasesi gni ficant lywi thani ncreasei ncont ract i
lity
2
andcar diacout put(typicall
y>1. 2cm ) .ATEE, car diacMRI , andonl yright -
hear t
cathet erizationwoul dnothel pdi stingui shwhet hert hel owgr adientwasduet ol owout put
rel
at edt ot heaor ti
cst enosisorani nher entcar diomyopat hy.Exer cisest udi esi napat ientwi th
sever eaor ticst enosisandsympt omsar enotr ecommended. 

Quest
ion:

63. A55- year


-ol
dwomani sref
err
edtoyouf oreval
uat
ionandt reat
mentofaorti
c
regurgi
tat
ion(AR).Shehasaknownhi st
oryofbicuspi
daort
icvalve.Thepat
ientexerci
sesfour
tofivet
imesperweekonanupr i
ghtst
ati
onarybicycl
eandreport
snosympt oms.Hermedi cal
hist
oryconsistsofmedical
lyt
reat
edhyperl
ipi
demi aandahyster
ect omy.
 

Onphysicalexaminat
ion,t
hebloodpr essureis126/74mm Hgwi thaheartr
ateof84bpm.
Lungsarecleart
oauscultat
ion.Theleftventr
icul
ar(LV)api
cali
mpul sei
snormalandnot
di
splaced.Asoft,
grade2/4diastol
icdecrescendomur murispresentmaxi
mall
yatthelef
t
ster
nalborder
.Asystoli
cejecti
oncli
cki snoted.Therei
snoS3 orS4,andnoperi
pher
aledema.
 

Transt
horaci
cechocar
diogr
am demonst
rat
esanLVej
ecti
onf
ract
ionof59%wi
thanend-
systol
icdi
mensionof33mm andend-di
ast
oli
cdi
mensi
onof50mm.

Whi
choft
hef
oll
owi
ngshoul
dbepr
escr
ibed?

a. Met
opr
ololt
art
rat
e.
b. Nif
edipine.
c. Aspir
in.
d. Enal
apr i
l.
e. Nome di
cati
oni
ndi
cat
ed.

Answer
:

63. e. Nomedi
cat
ioni
ndi
cat
ed.

Forpat i
entswi thAR,long-termt herapywithvasodi l
atorsisindicat
edi npat i
ent swit
hsevere
regurgit
ationandsympt omsorLVdysf uncti
ononl ywhensur gi
calinterventioncannotbe
performedduet ocomor bidit
y.Vasodi l
atortherapyalsomaybeconsi deredi nasympt omatic
pati
entswi thsevereregur git
ationwhohaveLVdi lat
ationbutnor malsyst oli
cf unct
ion,andas
abridget oaorticval
ver eplacementi npatientswithsever eregurgi
tati
onanddecompensat ed
heartfail
ure.However ,intheabsenceofot herindicati
ons( e.
g.,hypertension),vasodil
ator
therapyisnoti ndi
catedi nasympt omati
cpat i
entswi t
hmi l
dormoder at eARandnor mal
systoli
cfunction.Ther eisnoi ndicati
onforaspirinorbet a-
blockersformodi f i
cati
onofherr i
sk
foreventualaor t
icvalvereplacement .

Quest
ion:

64. Youar easkedt oseea67- year-


oldmanwhowasadmi tted2daysagot othehospital
.
Twomont hsago, t
hepat i
entunder wentaorti
cvalvereplacementwi t
ha26mm por cine
bioprosthesi sandcor onaryarterybypassgr aft
ingwithplacementofal eftinter
nalmammar y
arter
yt othemi ddleleftant
eriordescending.Thesur ger
yandhospi talcoursewer e
uncompl icated, exceptforsternaldehiscencethatoccurredsoonafterthepr ocedure.Overthe
pastweek, thepat i
enthashadsympt omsofl et
hargy,hadpoorappet i
te,andf el
tfeveri
sh.His
physicianf oundhi mt obef ebril
e(103°F)andpr omptlyadmitt
edt hepatienttoyourhospi t
al
servi
ce.Thepat i
enthasnocompl aint
sofdyspneaorchestpai n.Therear enogeni touri
nary
sympt oms.  

Onphysi calexamination,t
hetemper atur
ei s102.
5°F.Thehear trateis101bpm withablood
pressureof110/ 70mm Hg.Thel ungsar ecleart
oauscultati
on.Jugul arvenouspul
seisnot
elevated.Carot
idcont our
sarenor mal.Agr ade2/6earl
y-peakingsystolicmurmurispresentat
therightuppersternalborder
.Ther eisnoS3 orS4.Theperipheralexami nat
ioni
sunremarkable.
 
Hi ssternalwoundl ookswellhealed,butthester
num wasunst abl
e.

AchestX- rayshowsnormalcardiacsil
houett
eandbi later
alinf
ilt
rateswit
hnopl eural
eff
usions.Bloodcult
uresaredrawnandpendi ng.Anel ect
rocardiogram r
eveal
sl ef
tvent
ri
cul
ar
hypertr
ophywi t
hnormalPRi nt
erval.Atranst
horaci
c echocardiogram wasperformed,and
no vegetat
ionsare 
evi
dent.

Whi
choft
hef
oll
owi
ngi
sthemostappr
opr
iat
enextst
ep?

a. Posit
r onemissiontomogr aphycomputedtomogr
aphy.
b. Transesophagealechocar diogr
aphy(TEE)
.
c. Chestcomput edt omography.
d. Cardiacmagnet icresonanceimaging.
e. Cardiaccatheteri
zati
on.
Answer
:

64. b. Tr
ansesophagealechocar
diogr
aphy(
TEE)
.

Inpati
entswi thapr ostheti
cval ve,TEEi spreferredtoeval uateforendocar diti
sasi tismor e
sensit
iveindet ectingveget ationsandcompl icationst hant ransthoracicimagi ng.Inpat i
ents
withsuspectedendocar di
tis,bloodculturesshoul dbedr awnusi ngt woset sfrom different
sit
es.Surgeryf orpr ostheti
cval veendocar dit
isshoul dbeconsi deredwhent hereisevi denceof
heartfai
lur
e, valvedehi scence, progressivevalvedegener at
ion,compl icat
ions( e.g.,
abscess),
persi
stentbact eremi a,orrecur r
entembol i
.Antimi cr
obi altherapyf orprostheti
cval ve
endocardit
isshoul dutil
izevancomyci nunt i
lsuscept ibili
tytooxaci ll
inhasbeendet ermi ned.I
f
theTEEi snegat ive,thenot herimagingmodal i
ti
esmi ghtbeconsi dered.Ot hersourcesof
i
nfecti
ons,suchasmedi astiniti
sorpneumoni a,shoul dbesoughti fthereisnoevi dencef or
endocardit
is.Ther ei snoindi cati
onforcor onar yangiogr aphyatt histi
me. 

Quest
ion:

65. A55year -ol


dmal efar
mercomest oseeyouint heoutpatientcl
ini
cfornewsympt oms
ofshortnessofbreath.Thepatienthasal i
fel
onghistoryofabi cuspidaor
ticvalve.Overthe
past2mont hs,hebegannot i
ngsympt omsdur i
ngmoder atelevelsofacti
vit
y(e.g.,choppi
ng
wood,balinghay).Withsever
eexer ti
on,thepati
entoccasionallyexperi
enceslightheadedness.
Thereisnohi st
oryofchestdiscomf or
t,coronar
yarterydisease,hypert
ension,orcardiac
arr
hythmi a.
 

Hisphysicalexaminat i
onisremarkableforalate-peakingsystol
icejecti
onmur murattheri
ght
uppersternalborderandasi ngl
ecomponentS2.At ranst
horacicechocardi
ogram
demonst r
atessever e,bi
cuspidaor
ticstenosi
s( valvearea0.7cm2, meangradient56mm Hg) ,
mildaorti
cregurgitati
on,andnormal l
eftvent
ricularsystol
icfuncti
on( ej
ect
ionfract
ion62%).
 

Yourecommendsurgi
calaor
ti
cval
verepl
acement.Thepati
entst
rongl
ydesi
resat
issue
prost
hesi
s,ashewant
stoavoidt
heneedforant
icoagul
ati
onwithwarf
ari
n.

Whi
choft
hef
oll
owi
ngst
atement
siscor
rectr
egar
dingt
heuseofat
issuepr
ost
het
icval
ve?

a. Autogr
aft
shavepoor
ersurvi
val
comparedwit
htissueval
ves.
b. Thehemodynamicpr
ofi
leofastent
edt
issueval
veissimi
lart
oamechani
cal
pr
osthesis.
c. Ther i
skofsurgicalr
eoper
ati
onissimi
lart
other
iskofi
nit
ialoper
ati
on.
d. Struct
uraldegenerat
ioni
sage-r
elat
ed.

Answer
:

65. d. St
ruct
uraldegener
ati
oni
sage-
rel
ated.

Ther
eisani
ncreasedriskofst
ruct
uraldegener
ati
onofatissueprost
hesi
sinyoungerpat
ient
s
compar
edwit
hol derpati
ent
s.Thecauseforthi
sincr
easedriski
snotcomplet
elyknownbut
l
ikelydue, atleasti npart,t
othehi ghval vestressconf err edbyyounger ,
relati
vel
ymor eactive
i
ndi viduals.Theuseofanaut ograft(i.
e.,Rosspr ocedur e)isnotinferi
ortoat i
ssuepr osthetic
valve.St entedt issuepr ost
heseshaveasmal leref fectiveor i
ficeareaandt ypi
callyhigher
residualgr adi entst hanmechani cal valves.Stentlesst issuepr ostheseshaveabet t
er
hemodynami cpr of i
lethanstentedval ves,butaret echni call
ymor echall
engingt osurgicall
y
i
mpl antast heyar esewnt otheinsi deoft heaorticr oot.Ther i
skofsurgicalreoperati
oni s
significantlyrelat edt opati
entmor bi di
ty,buti
st ypicallyt wof oldt
ot hr
eefol
dhi gherthant he
ri
skoft hei nit
ial surgery.

Quest
ion:

66. Youar eseeinga66- year-ol


dwomanwhounder wentcardiacsur ger
yyesterdayfor
symptomat i
cmi tr
alregurgitat
ion.Duringtheprocedur e,t
hesur geonpl aceda31mm St .Jude
mechanicalbileaf
letprosthesi
si nthemi t
ralposi
ti
on, inadditi
ont oper f
ormingtri
cuspid
annul
oplastyforsever etr
icuspidregurgit
ati
on.Post operati
veat r
ialfi
bril
lat
ionoccurredand
spont
aneousl yresolvedwi t
hin24hour s.Hermedi calhistor
yissi gnif
icantonlyfor
hyper
tension.Apr eoperati
vecor onaryangiogram haddemonst ratednor malart
eri
es.

Whichoft
hef
oll
owi
ngi
sthemostappr
opr
iat
elong-
ter
m ant
it
hrombot
ict
her
apyf
ort
his
pat
ient
?

a. Warf
ari
nINRgoalof2.5-
3.5.
b. Aspi
ri
n81mgdaily;warfar
inint
ernat
ionalnormal
izedr
ati
o(I
NR)goalof2.
5-3.
5.
c. Aspi
ri
n81mgdaily;warfar
inINRgoal of2.
0-3.
0.
d. Dabi
gatr
an150mgt wicedail
y.

Answer
:

66. b. Aspi
ri
n81mgdai
ly;war
far
ini
nter
nat
ionalnor
mal
izedr
ati
o(I
NR)goalof2.
5-
3.5.

Forpatientswithamechani calvalveprost hesi


sinthemi t
ralposition,warfari
nshoul dbe
uti
li
zedwi thanI NRgoalof2. 5-3.5.Highergoalsmaybeconsi der edinpat i
entswhohave
thr
omboembol iceventswhi l
eant icoagulatedwithi
nthisINRr ange.Randomi zedt r
ial
shave
demonst rat
edr educti
onoft hromboembol iceventswit
ht headdi t
ionofaspi ri
n( 75-
100mg
dai
ly)t
owar far
inanticoagulati
oni npatientswithmechani calvalves.Clopidogrel,i
naddi
tiont
o
warfar
in,shouldbeconsi der
edi npat i
entswhocannott akeaspi rin.Aspiri
nalonei snot
eff
ecti
vef orprophylaxisagainstval vethrombosis.Dabigatr
ani snotr ecommendedf or
anti
thr
ombot ict her
apyf ormechani calvalves.

Quest
ion:

67. A65- year-oldmancal l


syouroffi
cewithquest i
onsregardi
nghismi tr
alvalveprost
hesis.
Twoyear sago, thepat i
entunder
wentsurgerywi thpl
acementofa31mm por ci
nemi tr
alval
ve
prosthesi
sf orsympt omatic,
severer
egurgi
tati
onduet omyxomat ousdisease.Hehasdone
wellsincesurgery, wit
hnosympt omsorcompl icati
onsinfoll
ow-up.Hislastechocardi
ogram,
whichwasper formed2mont hsago,demonstratedameant ransmitr
algradientof2mm Hg
andtrivi
alregurgitati
on.

Forwhi
choft
hef
oll
owi
ngpr
ocedur
esshoul
dhet
akeant
ibi
oti
csaspr
ophyl
axi
sagai
nst
i
nfect
iveendocar
dit
is?

a. Noant i
bioti
cprophylaxi
sneeded.
b. Transesophagealechocardi
ography.
c. Surveil
lancecol
onoscopy.
d. Cystoscopyformalignancyremoval.
e. Drainageofaskinabscess.

Answer
:

67. e. Dr
ainageofaski
nabscess.

Accor dingtothemostr ecentguideli


nes,ant ibioti
cpr ophyl axisagainstendocar dit
isisl
imited
onlytopat ientsathi ghestriskofcompl i
cat ions.Thesehi gh-ri
skpatientsincludethosewi t
h
prosthet i
chear tvalves.Ant ibi
ot i
cprophylaxi sisr equiredi nthesepat i
entswhoar eundergoing
dentalpr ocedur est hatinvolvemani pul
ationoft hegi ngivaltissues,theper i
apicalregi
onoft he
teet
h, orper f
orat i
onofor almucosa.Ant ibioticpr ophyl axisalsomaybeconsi deredinhigh-ri
sk
pati
ent sforrespi ratoryt r
actprocedurest hati nvolveinci sionofthemucosa( e.
g.,
tonsi
llectomyoradenoi dectomy)andf orgeni t
our inary,skin,orgastroint
est i
nalprocedures
thati
nvol veinfect edar eas.Patientsunder goingdi agnost i
cpr oceduresint heabsenceoft hese
conditionsdonotr equireant i
bioti
cprophyl axis. 

Quest
ion:

68. A50- year-


oldmanpr esent
st oyourambul at
oryoff
icef
orevaluati
onofaorti
cstenosi
s
(AS)andanaor t
icaneurysm.Tenyear sago,t
hepat i
entwasdiagnosedwithabicuspi
daor t
ic
valve(BAV)duri
ngar outinegeneralmedicalexaminati
on.Heisanact i
veandasymptomat i
c
i
ndi vi
dualwi
thgoodexer cisetol
erance.Thepatientt
akesnomedi cat
ions.
 

Onphysicalexami nat
ion,t
hebl oodpressureis110/70mm Hgwi t
hahear tr
ateof66bpm.
Thecaroti
dcont ourisnormal.Asystoli
ccl i
ckandgr ade2/6earl
y-peaki
ngsystol
icej
ecti
on
mur murarepresentattherightuppersternalbor
der.Theaorti
ccomponentoft heS2 
is
preser
ved.Thereisnodi ast
olicmurmurandnoS3  orS4.Per
ipher
alexaminati
onshowsno
edema. 

Thepat
ientunder
goesanechocar
diogr
am,
whi
chdemonst
rat
esaBAVandt
hef
oll
owi
ngdat
a: 

Lef
tventr icularejectionfracti
on:60%. 
Lef
tend- diast ol
icdimensi on:50mm.  
Ascendingaor t
adi mension:4.6cm.  
Aort
icrootdi amet er:3.
8cm.  
Lef
tatri
alvol umei ndex:32ml /m2.
Meanaor ticvalvegr adient:10mm Hg.  
2
Aort
icval vear ea:2.0cm .

Whichoft
hef
oll
owi
ngwoul
dbet
hemostappr
opr
iat
eti
met
oper
for
m anechocar
diogr
am
agai
n?

a. Ever
y3year
s.
b. Ever
y2years.
c. Ever
yyear
.
d. Ever
y5yearsorl
ater
.

Answer
:

68. c. Ever
yyear
.

Forpatientswi t
haBAVandanascendi ngaor ti
caneurysm >4.5cm, annuali
magingofthe
aneurysm isrecommended.Forpat ient
swithmi ldASwi t
houtanascendi ngaort
icaneurysm,
seri
alechocar diographyevery3-5yearsisrecommendedi ntheabsenceofachangei n
cl
ini
calst at
usorphysi calfi
ndi
ngs.Echocardiographyi
srecommendedannual l
yfor
asympt omaticpat i
entswi t
hsevereASandever y1-2year
sint hosewithmoderateAS.
Pati
entsshoul dbeadvi sedtor
eportchangesi nsymptomspr omptly.

Quest
ion:

69. A29- year-ol


dwomani sref
erredbyherinter
nistforeval
uat
ionofanewlyident
ifi
ed
murmur.Thepat i
entrepor
tsnocardiovascul
arsymptomsandhasnosi gni
fi
cantmedical
hist
oryexceptforaknownpenicil
li
naller
gy.Heronlyprescri
bedmedici
neisanoral
contr
acepti
ve.Shedeniesrecr
eati
onal dr
uguse. 

Onphysi calexami nati


on,thereisasystol
icclickfol
lowedbyagr ade2/6lat
esyst
oli
cmur mur,
consi
stentwi thmi tr
alregurgit
ati
on.Noot herabnormalf i
ndingsarepresent
.Atr
ansthor
acic
echocardiogram demonst ratesmyxomat ousmi tr
alvalvediseasewithmoderat
eregurgi
tat
ion
andmi l
dl ef
tat r
ialdil
atati
on.Theleftventri
cul
arend- syst
oli
cdimensionis32mm, and
ej
ecti
onf racti
oni s65%.

Whi
choft
hef
oll
owi
ngt
ypesofant
ibi
oti
cpr
ophyl
axi
sisr
equi
redpr
iort
odent
alpr
ocedur
es?

a. Noantibi
oticpr
ophyl
axi
sisr
equi
red.
b. Amox i
cil
li
n2g.
c. Amox i
cil
li
n4g.
d. Cl
indamycin800mg.
e. Cl
indamycin600mg.

Answer
:

69. a. Noant
ibi
oti
cpr
ophyl
axi
sisr
equi
red.

Themostcur rentguideli
nesdonotr ecommendant i
biot
icprophylaxi
sf orendocarditi
sf or
myxomat ousmi t
ralvalvedisease.Endocar di
tisprophylaxisisonlyrecommendedf or
condi
tionsassociatedwiththehi ghestriskofadver seoutcomef rom endocar di
ti
s.These
condi
tionsare:1)prostheti
ccar diacvalveorpr osthet
icvalverepairmat eri
al,
2)priorhistor
yof
i
nfect
iveendocar di
tis,
3)car di
act ransplantr
ecipientswithvalvul
opathy, 4)completel
y
repairedcongenit
alheartdi
seasewithper
cutaneousorsurgi
calr
epairoccurr
ingwi
thi
nt he
previous6mont hs,5)repai
redcongeni
tal
heartdiseasewi
thresi
dualshuntsordef
ectsthat
i
mpai rendot
heli
ali
zat
ionofprosthet
icmater
ial
,and6)unrepai
redcyanoti
ccongeni
talheart
disease. 

Forpatientswiththeseconditi
ons, ant
ibi
oti
cpr ophylaxi si
sr ecommendedf ordental
proceduresthatinvolvemanipulati
onoft hegingivaltissues, t
heper i
api
calregionoftheteeth,
orperforati
onofor almucosa.Ant i
biot
icprophylaxisal somaybeconsi deredforprocedures
thati
nvolveincisi
onoft herespirat
orymucosa( e.
g.,tonsil
lectomyoradenoi dectomy)andf or
genit
our i
nary,
skin,orgastroi
ntesti
nalprocedurest hatinvolveinfectedareas.Pati
ents
undergoingdiagnost i
cproceduresintheabsenceoft hesecondi tionsdonotr equi
reanti
biotic
prophylaxi
s. 

Quest
ion:

70. A51- year-


oldmani srefer
redtoyouforevaluati
onofachr oni
cmur mur.At20year sof
age,heunderwentsurgicalr
epairofbi
cuspi
daor t
icvalve( BAV)st
enosi
s.Thepat i
enthasbeen
fol
lowedint
ermitt
entlyandhasnoot hersi
gnif
icantmedi calhi
stor
y.Heisphysical
lyacti
veand
asymptomatic,
regularl
yrunning5mi l
eswit
houtlimitat
ion. 

Onphysi calexami nati


on, thepat i
entis184cm t allandwei ghs84kg.Thehear trateisregul
ar
at60bpm wi thabl oodpr essur eof135/ 55mm Hg.Thel ungsar ecleartoauscul tati
on.The
l
ef tventricular(LV)apicalimpul seisenl argedandl ateral
lydisplacedt otheanterioraxil
lar
y
l
ine.Agr ade2/ 6ear l
y-peaki ngsyst oli
cmur murandadecr escendogr ade2/ 4diastoli
c
mur murbot har epresental ongt hel eftsternal border.TheS2Per ipheralexamination
demonst ratesapul sati
leuvul aandacol lapsingwat erhammerr adialpulse.
 
Thepat ientunder goesat ransthor acicechocar diogram thatdemonst ratesBAV.LVend-
diastoli
cdi mensionis7. 1cm, LVend- syst oli
cdi mensi onis4.2cm, andLVej ectionfracti
on
(calcul
ated)i s57%.I ntervent r
icularsept alwal lthicknessis1. 2cm, LVpost eri
orwal lthi
ckness
i
s1. 2cm, aort
icrootdiamet eri s4.2cm, AVr egur gi
tantvolumei s70ml ,AVregur gi
tant
fracti
oni s60%, andcar di
acout putis6. 8L/ min.

Whi
choft
hef
oll
owi
ngi
sthenextbestst
ep?

a. Surgicalaorti
cvalvereplacement(SAVR).
b. Seri
alechocar di
ography.
c. Transcatheteraorti
cvalvereplacement(
TAVR)
.
d. Sympt om-li
mi t
edexercisestresstest
.

Answer
:

70. b. Ser
ialechocar
diogr
aphy.

Foranasympt omaticpati
entwithsever
eAR, SAVRi srecommendedwhent herei
ssignif
icant
LVdil
atat
ion( end-
diast
oli
cdimension>6.5cm (Cl
assI I
b)orend-syst
oli
cdimension>5.0cm
(Cl
assII
ar ecommendat i
on).SAVRalsomaybeconsi deredatlowerl
evelsofLVdilat
ati
on
whenthereisevidenceofprogressi
vecavit
ydil
atati
on,declini
ngexer
cisetoler
ance,or
abnor
malhemodynami cresponsestoexerci
se(ClassIIbrecommendation).
 

Accor
dingt
orecentAmer
icanHear
tAssoci
ati
on/
Amer
icanCol
legeofCar
diol
ogygui
del
ines,
themostappropriat
ei ni
ti
alr
ecommendat i
onistorepeattheechocardiogr
am in6-
12mont hs
todetermi
neiftheLVdi mensionsarestableorenl
arging.Asympt om-li
mitedexer
cisest
ress
testisr
easonabletoclari
fyf
unctionalcapaci
tyi
npat i
entswithequivocalsymptoms.TAVR
current
lyi
snotappr ovedaft
errepair
edvalveswithAR.

Quest
ion:

71. Youar easkedtoeval uateandmanagea79- year-


oldwomanwi thaorti
cstenosis(
AS).
Forthepast3mont hs,shehasnot edprogressi
veshor t
nessofbreathwithdail
yhousehold
chores.Hermedicalhi
storyissigni
fi
cantforhypert
ension,atr
ansienti
schemi catt
ack(2year
s
ago),andchr
onicrenaldisease.Shecurrentl
ytakeslosart
an(50mgdai ly)andaspiri
n(81mg
dai
ly).
 

Onphysi calexaminati
on,shei s158cm t al
l andweighs65kg.Thebl oodpr essureis124/72
mm Hgwi thahear tr
ateof84bpm.Thej ugularvei
nsar enotdistended.Car oti
dsare2+tardus.
Herlungsar ecleartoauscultati
on.Theapi cali
mpulsei ssustai
nedanddi splaced2cm tothe
l
eftofthemi d-cl
avicul
arli
ne.Agr ade3/6har shlat
e-peaki
ngsyst oli
cejectionmur murisheard
thr
oughoutt heprecordium,loudestatther i
ghtuppersternalborder,andradi at
estothe
carot
ids.Thereisnodi astol
icmur mur.Ther estofherphysicalexami nat
ioni snormal.
 

Laboratoryst
udiesincl
udeahemogl obinof12. 1g/ dlandacr eati
nineof1.6mg/ dl
.An
echocardiogr
am demonst r
atesconcent r
icleftventricul
arhyper t
rophywithanej ecti
onfract
ion
of65%.Cal ci
fi
cASi spresentwithameangr adi entof42mm Hgandacal cul
atedvalvearea
of0.7cm2.Thecor onaryangiographyrevealsan80%st enosisintheproxi
mal  
leftanter
ior
descending(LAD)arter
yandat otal
 r
ightcoronar yarter
yst enosi
s.Herestimatedoper ati
ve
mortali
tyforcardi
acsurgeryiscalculat
edas5. 6%.

Whi
choft
hef
oll
owi
ngi
sthenextbestst
epi
nthi
spat
ient
’smanagement
?

a. Surgicalaort
icvalvereplacement(AVR)withcoronaryar
ter
ybypassgr af
ti
ng.
b. Ball
oonaor ti
cvalvuloplastyandpercut
aneouscoronaryi
ntervent
ion.
c. Drug-elut
ingstentplacementi nt
heLAD.
d. Percutaneouscoronar yinter
vent
ionandtranscat
heterAVR.

Answer
:

71. a. Sur
gicalaor
ti
cval
ver
epl
acement(
AVR)wi
thcor
onar
yar
ter
ybypassgr
aft
ing.

Thepatienthassymptomat i
c,severeASandsi gni
fi
cantcoronaryarterydiseaseinvol
vingher
LADartery.Hersurgi
calri
skisintermediate.Themostappr opr
iatetherapyissurgicalAVRand
cor
onaryar ter
ybypassgraft
ing.TranscatheterAVRpr esent
lyi
sr estr
ictedtohigh-r
iskor
i
noperablepat i
ent
swithseveresympt omat i
cAS.Balloonaorti
cvalvuloplastywouldnotbe
i
ndicat
ed, excepti
ncri
ti
call
yillpati
entsasabr idgetoAVRorasapal l
iati
veprocedure.

Quest
ion:

72. A56-
year
-ol
dwomani
sref
err
edt
oyourof
fi
cef
ordyspnea.Hersympt
omsbegan
sever
alyearsago,andhaveprogressedsteadil
yoverthepast12months.Pr
esent
ly,
she
cannotwalkmorethan100yar dswithoutpreci
pit
ati
nghersymptoms.Shetakesmetopr
olol
50mgdai l
yforhypert
ension.
 

Thejugularvenouspressureiselevat
edt o9cm H20wi thpr ominentVwaves.Asl i
ghtl
eft
par
asternalli
ft(1+)i
spresent.Thefir
stheartsoundi snormalinintensit
y.Thepul moni
c
componentoft hesecondhear tsoundisincreased.Bothasof tgrade2/4diastoli
crumbleand
a3/6holosystolicmurmurarehear dattheapex.Att heleftst
ernalborder,
agrade2/ 6systol
ic
ej
ecti
onmur murispresentandincreaseswithinspir
ation.Apulsatil
eli
verisevidentwi
thmi l
d
l
owerext r
emi t
yedema( 1+).
 

At ransthoraci
cechocardiogram demonst r
atesnor malleftventri
cul
arfuncti
on(eject
ion
fracti
on65%)andr heumat i
cmi t
ralstenosi
s( meangr adientis14mm Hg;val veareais1.0
cm2) .Themi tr
alval
veispliablewit
hnocommi ssuralcalci
ficat
ion.Moderatetoseveremit
ral
regur gi
tat
ionandseveret r
icuspidregurgi
tati
onar epresent.Ther i
ghtventr
icl
eismi l
dlydi
lat
ed
wi t
hgener ali
zedhypokinesis.Thepulmonar yarter
ysyst oli
cpressureisesti
matedat60mm
Hg.Cor onar yangi
ographydemonst r
atesmildatherosclerosis.

Whi
choft
hef
oll
owi
ngi
sthenextbestst
ep?

a. Opt
imizedmedi calt
herapy.
b. Bal
loonmitralval
vulopl
asty.
c. Car
diacsurgery.
d. Car
diopul
monar ystresstest
.

Answer
:

72. c. Car
diacsur
ger
y.

Thepat i
enthassever emi tralstenosisandr egur git
ationwit
hseveresympt omsofdypsnea
(NewYor kHear tAssociationf uncti
onal cl
assI II)
.Althoughhermitralval
vei spl
iable,t
he
presenceofmoder atetosever emi tr
alregurgitati
oni sacontrai
ndicati
ont oball
oon
valvuloplast
y.Car diacsurger y,wit
hmi t
r alvalvereplacementandt ri
cuspidvalverepair,i
s
i
ndi catedduet othesever enat ureofhersympt oms, thepr
esenceofpul monaryhyper t
ension,
andheri nel
igi
bili
tyforvalvuloplasty.Acar diopulmonar ystr
esstestwoul dbeindicatedonlyif
thereisadi screpancybet weent hecli
nical f
indingsandt heresul
tsoft heechocardiogram. 

Quest
ion:

73. A78- year-ol


dwomani sr eferr
edtoyouforevaluati
onoffat
igueandshortnessof
breat
h.Hersympt omsbegansever almont
hsago, occurr
ingatmoderat
elevel
sofexer t
ion.
Shehasnoot hercompl ai
nts,
includingnochestpainorsyncope.Thepati
entrecei
vesat enol
ol
100mgdai l
y,li
sinopri
l20mgdai l
y,andhydrochl
orothi
azide25mgdai l
yforl
ong-standi
ng
hypert
ensi
on. 

Onphysi
calexami
nat
ion,
thebl
oodpr
essur
eis150/
90mm Hgwi
thahear
trat
eof72bpm.
Theexaminati
onisremarkabl
eforclearl
ungs,anda2/6mid-
peakingsyst
oli
cej
ect
ionmur
mur
thati
sheardbestatther
ightupperster
nalbor
der.Atr
anst
horaci
cechocardi
ogr
am
demonstr
atesthefol
lowi
ngdata: 

Lef
tvent r
icularejectionfraction(LVEF):55% 
Ventri
cularsept alwallthickness:13mm 
Posteri
orwal lthickness:13mm 
Meanaor ticvalvegr adient:30mm Hg 
Peakaor t
icveloci t
y:3.4msec 
Aort
icvalvear ea:0.7cm2 
Esti
mat edrightvent r
icularpressure:35mm Hg 
2
Str
okevol umei ndex:30ml /

Ahemodynami
ccar
diaccat
het
eri
zat
ionconf
ir
mst
heechocar
diogr
aphi
cfi
ndi
ngs.

Whatisthi
spat
ient’
sexpectedmort
ali
tyaf
teraor
ti
cval
verepl
acement(AVR)compar
edwi
th
pat
ient
swithhi
gh-gradi
entaort
icst
enosi
sandnormalLVEFwhoundergoAVR?

a. Unknown.
b. Simil
ar.
c. Higher.
d. Lowe r
.

Answer
:

73. d. Lower
.

Paradoxicallow-gr
adient,sever eaorti
cstenosisandpr eservedLVEFisanewl yr
ecognized
cli
nicalenti
tythatposesanumberofdi agnosti
candt herapeuti
cchall
enges.Whi
letheaor t
ic
2
valveareaiswi t
hinthesever er ange(<1.0cm ) ,
theaorticgradi
entisl
owduet oarelat
ively
2
l
owst r
okevol ume(<35ml /m )despi tepreservati
onoft heLVEF.Thel owstr
okevolume
occursduet oavar i
etyofet iologies,
suchasanel evat
edsyst emicaft
erl
oadorahypertrophied
ventri
cleandsmal lcavit
ysi ze.  

Presentl
y,ther
eisnocl earconsensusont her ol
eofaort
icval
vereplacement(AVR)inthi
s
enti
ty.Someoft hese“par adoxicallow-
gradientaor
ticst
enosi
s”pati
entsmaybenef i
tfr
om
AVR, butcohortst
udieshavef oundoutcomest obeworseaft
ersurgeryinlow-gr
adi
entnormal
LVEFpat i
entscompar edwi thout comesafterAVRi npati
ent
swithhigh-gradi
ent,
nor
malLVEF
andsevereaor t
icstenosis.

Quest
ion:

74. A28- year-ol


dwomanpr esentstotheemer gencyroom 26weekspr egnantwith
progr
essi
vedyspneaandpal pi
tati
onsint hepast1-2weeks.Shehasahi storyofbeingver
yill
asachil
d.Examinati
onfi
ndsani r
regularheartr
ateof140bpm wi t
habl oodpressureof85/60
mm Hg.Jugularvenouspressureis8cm abovet hesternalangl
e.Grade2/ 4diast
oli
crumble
i
sheardatapex.Therearebibasil
arcrackles.
 

Shei
sgivenat
enolol.Herhear
tratecomesdownt
o90bpm,andshefeel
ssomewhatbett
er.
2
Themeanmitr
alvalve(MV)gradienti
s12mm Hg.TheMVar
eais0.
85cm .Ther
eismi l
d
mi
tr
alregur
git
ati
on.Ej
ect
ionf
ract
ioni
spr
eser
ved.Thepeakpul
monar
yar
ter
ypr
essur
eis50
mm Hg.

Whi
choft
hef
oll
owi
ngwoul
dyoudonext
?

a. Ur
gentper cut
aneousball
oonvalvulopl
ast
y.
b. MVreplacement.
c. Ur
gentdirectcur
rentcar
dioversi
on.
d. Ti
tr
atebeta-bl
ockersandstartant
icoagul
ati
on.

Answer
:

74. d. Ti
tr
atebet
a-bl
ocker
sandst
artant
icoagul
ati
on.

Thei nit
ialtreat
mentwillbemedi calt
herapywiththepati
enttreat
edtohaveasl owerheartr
ate,
startanticoagulant
s,andtreatf
orcongest i
veheartfai
lur
e.Thereisnorol
eforcardiover
sionat
thistime, assheisnowr easonablyratecontr
oll
edwi t
himprovedsymptoms, shehasnotbeen
previouslyant i
coagul
ated,andthedurationofatr
ialfi
bri
ll
ati
onisunknown. 

Ifsympt omsf ai
ltoimprovewi thadditionalmedi caltherapy,fut
ureoptionsi
nclude
transesophagealechocar diogram/dir
ectcur rentcardioversi
on.Indicat
ionsfori
ntervent
ion
(eit
herpercutaneousorsur gi
cal)aresympt omsr efr
actorytomedi calmanagement .I
fmedical
managementf ail
s,ball
oonval vulopl
ast y(
wi thpelvicshiel
ding)woul dbetheprocedureof
choiceifthevalvemor phologyi sappropriate.Sur gi
calMVr eplacementduringpregnancy
carri
esa30%r i
skoffetallossandi sthereforeal astresort.

Quest
ion:

75. A31- year-


oldmanpr esent
stoyourcli
nicf
orannualf oll
ow-upforhisbicuspidval
ve
andsevereaor ti
cregurgit
ation.Hehasnoothersigni
ficantmedicalhist
oryandi sa
nonsmoker .Hehasbeenhavi ngyearl
yechocardi
ograms, butconti
nuestof eelwellandhave
nosympt oms.Hewor ksfullti
meandr uns3 t
imesperweekf orabout30mi nutes.Hetel
lsyou
thathehasnotnot edanychangei nhisexer
ciset
oleranceoverthepastyear .Hehasno
palpi
tat
ions,chestpain,orsyncope. 

Onexami nati
on,heiswel lappear i
ng.Hishei
ghti
s72i nchesandweightis180lbs.Hisheart
rateis62bpm, bloodpressur eis145/50mm Hg, andoxygensatur
ati
onis98%.Hi slungsare
clear
.Hisjugularvenouspr essureisnormal,
andhehasanor malS1 
andS2.Thereisagrade
1/6systoli
cmur muratthel eftsternal
border
,fol
lowedbyagr ade3/4decrescendodiast
oli
c
mur mur.Thereisnolowerext remityedema. 

Hisechocardiogr
aphyt odaydemonstr
atesnormal l
eftventr
icul
ar(LV)funct
ionandnoLV
hypertr
ophy.However ,hi
sLVend-syst
olicdimensi
oni s5.2cm, i
ncreasedfr
om 4.7cm t
he
yearbeforeand4.4cm f r
om 2yearsbefore.HisLVejecti
onfract
ionis50%.Youtellhi
mt hati
t
i
sr easonabl
et oproceedtoaort
icval
vereplacementinthenearfuture.

Youarr
angeanappoi
ntmentwi
tht
hecar
diacsur
geonandor
derwhi
choft
hef
oll
owi
ng
addi
ti
onalt
est
ing?

a. Cor
onar
yangi
ogr
aphy.
b. Car
diopul
monarystresst
est
.
c. Computedtomographyangi
ographyofthechest
.
d. Magneti
cresonanceangi
ographyofthebrai
nvascul
atur
e.

Answer
:

75. c. Comput
edt
omogr
aphyangi
ogr
aphyoft
hechest
.

Patientswi t
hbi cuspidvalveshaveahi ghi ncidenceofascendi ngaor t
icaneurysm, thought
secondar ytoi ntr
insi
cabnor mali
ti
esint heaor ticwallconnectivet i
ssue.Af ewalsohave
evidencef orcoar ct
ati
on.Thi scanoccuri ntheabsenceofsi gnifi
cantval vedisease.Itis
i
mpor tantt oevaluatethesi zeoftheascendi ngaor t
apr i
ortosur gerysincei tmayneedt obe
repairedatt hesamet i
meast hevalve.Ther efore,comput edtomogr aphyangiographyoft he
chesti sthemostappr opri
at eanswer.Magnet i
cresonanceangi ogr aphyoft hebrai
ncoul dbe
consider edi nLoeys-Dietzsyndr omeort oconsi derrevi
ewf orcer ebralaneurysms, butthisi
s
notindi catedi nthissit
uation.Hedoesnotneedcor onaryangi ography.

Quest
ion:

76. Youar easkedt oseeayoungwomanwhoi si nthe28thweekofherf i


rstpr
egnancy.
Shehashadver ylit
tl
eprenatalcare,buttell
syout hatsher emember sbeingtoldshehada
mur murasachi ld.Shehadbeendoi ngfineupunt ilt
hepastf ewweekswhenshedevel oped
dyspneaonexer tionandmi l
dor t
hopnea.Onexami nati
on,herheartrateis112bpm and
regular
,bloodpressureis125/72mm Hg, andjugularvenouspr essureismildlyel
evat
ed.She
hasmi nimalbibasil
arral
esandagr ade2/ 4diastol
icmur murhear dattheapex.Shehasno
l
owerext remit
yedema.Anel ectr
ocar di
ogram revealssinustachycardia.Anechocardi
ogram
demonst rat
essignifi
cantrheumaticmi t
ralstenosiswi t
hameangr adientof8mm Hg.

Whi
choft
hef
oll
owi
ngi
sthebestmedi
calt
her
apyt
ohel
pimpr
ovehersympt
oms?

a. Amiodarone.
b. Di
goxin.
c. Li
sinopri
l.
d. Metoprol
ol.

Answer
:

76. d. Met
opr
olol
.

Thisyoungwomanhasnewl ydiscoveredmi t
ral st
enosisandisnottolerati
ngtheincreased
hear trateandi ncreasedplasmavol umeofpr egnancy.Shei sneari
ngtheendofhersecond
tri
mest erwhenhervol umest atusandcar diacout putreachthei
rpeak.Thebestopt ionisto
i
ncr easeherdi ast
olicfi
ll
ingtimebydecr easingherhear tratewit
hbeta-blockade.Metoprol
ol
i
swel ltoler
ateddur ingpregnancyandmi ghtallevi
atehersympt omsenought ogether
throught her estofherpr egnancywi t
houtapr ocedur e.Bal
loonmitr
alvalvuloplast
yissafe
afterthefi
rsttrimester
,butnotwi t
houtri
sk.Bothangi
otensi
n-conver
ti
ngenzymei nhi
bit
ors
andangi otensin-r
eceptorblocker
sareterat
ogensandshouldnotbeusedi nwomenwhoar e
pregnantorbr eastfeeding.Sheisinsi
nusr hyt
hm,andthereisnoindicat
ionfordi
goxi
n.
Ami odaroneshoul dnotbeusedi npregnancy. 

Quest
ion:

77. A60- year-


oldwomancomesf oratr
ansesophagealechocar diogram (TEE)pri
ortoa
pl
annedmi tralvalvul
oplastyformitralst
enosis.Althoughther
eisnopr i
orhistor
yofat r
ial
fi
bri
ll
ati
on( AF),atthetimeofpr esentati
onfortheTEE, sheisnotedtobei nAF.Shei snoton
anti
coagulati
on.Herl aborat
orydat aarenormal exceptforacreati
nineof2.0mg/ dlwit
h
gl
omer ul
arf i
lt
rati
onrateof30ml /min.TheTEEi sshowni nFigure1.

Whi
choft
hef
oll
owi
ngi
sthebestnextcour
seofact
ion,
basedont
heTEE?

a. Aspi
rinandclopidogr
el.
b. Anoveloralant
icoagulantagent
.
c. Unfr
acti
onatedheparinandwar f
ari
nther
apy.
d. Lowmol
ecul
arwei
ghthepar
in(
LMWH)andwar
far
int
her
apy.

Answer
:

77. c. Unf
ract
ionat
edhepar
inandwar
far
int
her
apy.

Thereisclearl
yal eftatr
ialappendaget hr
ombus.Thepat ienthasnew- onsetAFwi thalarge
appendaget hrombust hatrequir
esi mmediat
eant i
coagulation.Thecorrectansweristoadmi t
herforheparinandwar f
arintherapy,anddel
ayconsiderationofvalvulopl
astyuntilf
urt
her
i
magi ngisperformedi n6weeks.Thenovel anti
coagulantshavenotbeenappr ovedforAF
rel
atedtomi tr
alvalvularstenosi
s.LMWHshoul dnotbeusedi napatientwiththi
sdegr eeof
chronicki
dneydi sease.Clopidogrelandaspi
rinhavebeenshownt obei nferi
ortowar f
ari
nfor
AF. 

Quest
ion:

78. A49- year


-ol
dmani seval
uatedf
orprogressivedyspneaandedema.Forthepast
severalmonths,hehasexperi
encedworseni
ngdyspneawi t
hmoderateact
ivi
ti
esand
worseninglowerextr
emit
yedemaandabdomi nalfull
nessdespi
teescal
ati
ngdosesof
diur
etics.
 

Onphysicalexaminat i
on, t
emperaturei snormal,
bloodpr essureis118/65mm Hg, pulserate
i
s88bpm, andrespirationratei
s18br eaths/mi
nute.Lungsar eclearbil
aterall
y.Thereis
j
ugularvenousdistentiontothemandi blewithapromi nentCVwave.Hear trhyt
hm isregular
.
Thereisagrade2/ 6low- pi
tched,holosystoli
cmur mural ongt helef
tsternalborderthat
i
ncreasesduringinspirati
on.Thel i
verspani senlarged,andt hereisbil
aterallowerextr
emity
edema. 

Elect
rocardiogr am showssi nusrhythm at80bpm andi ncompl eteri
ghtbundlebranchblock.
Transthoracicechocar diogr aphyshowsnor mal lef
tventr
iculardimensionsandsystol
ic
functi
on,butt her i
ghtvent ri
cleisdil
atedwi t
hreducedsyst oli
ccont r
acti
on.Thereispoor
coaptati
onoft hetri
cuspi dval ve(
TV)leaflet
swi thmobil
e, t
hickenedleafl
etsandsevere
tri
cuspidregur git
ati
on( TR) .Therightventri
cularsyst
oli
cpr essureisesti
matedat30mm Hg.
Ther i
ghtatrium isalsodi lated.Uri
nedr ugscreenandbloodcul turesarenegati
ve.

Whi
choft
hef
oll
owi
ngi
sthemostappr
opr
iat
emanagementf
ort
hispat
ient
?

a. Angiot
ensin-
convert
ingenzymei
nhi
bit
or.
b. TVrepairorr
eplacement.
c. Si
ldenafi
l.
d. Di
goxin.

Answer
:
78. b. TVr
epai
rorr
epl
acement
.

ThispatientwithsevereTRshoul dunder goTVr eplacementsur geryorrepair


.Thenormal
esti
mat edrightventri
cul
arsystoli
cpr essur
ebyDoppl erechocar diographysuggest
s
regur
gitati
oncausedbyl eafletormyocar di
al dysfuncti
on,rathert hanfunct
ionalTRdueto
pulmonar yhypertensi
on.Hehasevi denceofr i
ghtventri
cularvol umeover l
oad,i
ncl
uding
dil
ati
onandr educedsystol
icfuncti
on.Theonl ydefini
ti
vether apyf orsever
e,symptomati
cTR
i
ssur gery,eit
hertri
cuspi
dannul oplastyorreplacement. 

Thet i
mingofTVsur geryischallengi
ng, butshoul dbeconsi deredinpatientswi thsevere
regurgit
ationandeithersympt omsorevi denceofpr ogressi
veri
ghtventricularenlargementor
dysfunctiondespit
eadequat ediureti
ct herapy.Di goxinisincor
rect.Al
thought hereisevi
dence
ofrightventri
cul
ardysf unct
ion,nost udiesconcl usivelydemonstrateabenef itfordigoxi
nor
otherinotropi
ctherapyf orr
ight-
sidedhear tfailur
e.Ther eisnoindicat
ionf orsildenafi
lwit
h
normalpul monaryar t
erypressuresandnocl earadvant ageforangiot
ensi n-
conver t
ingenzyme
i
nhibiti
oni nthesett
ingofnor mal l
eftvent r
icularfunction.

Quest
ion:

79. Youar easkedtoseeayoungwomanwhoi sinthe28thweekofherf ir


stpregnancy.
Shehadbeendoingfineupunt i
lthepastfewweeks, whenshedevelopeddyspneaonexer t
ion
andmi l
dort
hopnea.Onexami nati
on,herheartrat
eis112bpm andr egularandbloodpr essure
i
s125/ 72mm Hg.Herj ugularvenouspressureismildl
yelevat
ed,shehasmi nimalbi basi
lar
ral
es,andagrade2/4di ast
olicmurmuri sheardattheapex.Youhavenomedi calrecords
becauseshegrewupi nSout hAmer i
ca,butshedoesr ememberapr olongedfebri
leill
nessas
achil
d.

Herechocar
diogr
am reveal
sevi
dencef
orr
heumat
icmi
tr
alst
enosi
s.Theel
ect
rocar
diogr
am
r
evealssi
nustachycardi
a.

Whi
choft
hef
oll
owi
ngi
sthebestmedi
calt
her
apyt
ohel
pimpr
ovehersympt
oms?

a. Di
goxin0.25mgqd.
b. Li
sinopri
l5mgqd.
c. Hydral
azine25mgt
id.
d. Metoprol
ol12.
5mgbid.

Answer
:

79. d. Met
opr
olol12.
5mgbi
d.

Thisyoungwomanhasnewl ydiscoveredmi t
ralst
enosis.Whileshemayneedf urosemide,
the
bestopti
oni st
oincreaseherdiastoli
cfill
ingt
imebydecr easingherheartrat
ewi thbeta-
bl
ockade.Noneoft heotheroptionswoul dimproveherhemodynami csinthi
ssituati
on.
Pregnancyresult
sinmar kedaft
er l
oadr educt
ionandani ncreaseincardi
acoutputandpr el
oad.
Oftenapatient
'scli
nical
classwor sensbyoneNewYor kHear tAssociati
onclassdur i
ng
pregnancy.Shehasnowdevel opedmi ldcongest i
veheartfail
ure.
 

You might also like